0% found this document useful (0 votes)
2K views900 pages

Gate Ece 2003-2019 Solved

Uploaded by

Abdul Sattar
Copyright
© © All Rights Reserved
We take content rights seriously. If you suspect this is your content, claim it here.
Available Formats
Download as PDF, TXT or read online on Scribd
0% found this document useful (0 votes)
2K views900 pages

Gate Ece 2003-2019 Solved

Uploaded by

Abdul Sattar
Copyright
© © All Rights Reserved
We take content rights seriously. If you suspect this is your content, claim it here.
Available Formats
Download as PDF, TXT or read online on Scribd
You are on page 1/ 900

GATE ESE PSU’s 2019-20

ECE ENGINEERING
GATE ECE 2003-2019 SOLVED

GATE ECE 2003-2019 SOLVED Detail Solution

CONTENT COVERED:
1.Theory Notes
2.Explanation
3.Derivation
4.Example
5.Shortcut & Formula Summary
6.Previous year Paper Q. Sol.
Noted-: Single Source Follow, Revise
Multiple Time Best key of Success
1
Page

http://www.orbitmentor.com [email protected]
A-PDF Merger DEMO : Purchase from www.A-PDF.com to remove the watermark

GATE EC
2003

Q. 1-30 Carry One Mark Each

MCQ 1.1 The minimum number of equations required to analyze the circuit shown in the
figure is

(A) 3 (B) 4
(C) 6 (D) 7
SOL 1.1 Hence (B) is correct option.
Number of loops = b − n + 1
= minimum number of equation
Number of branches = b = 8
Number of nodes = n = 5
Minimum number of equation
= 8−5+1 = 4
MCQ 1.2 A source of angular frequency 1 rad/sec has a source impedance consisting of 1 Ω
resistance in series with 1 H inductance. The load that will obtain the maximum
power transfer is
(A) 1 Ω resistance
(B) 1 Ω resistance in parallel with 1 H inductance
(C) 1 Ω resistance in series with 1 F capacitor
(D) 1 Ω resistance in parallel with 1 F capacitor
SOL 1.2 For maximum power transfer
ZL = ZS* = Rs − jXs

Brought to you by: Nodia and Company Visit us at: www.nodia.co.in


PUBLISHING FOR GATE
Page 2 GATE EC 2003 www.gatehelp.com

Thus ZL = 1 − 1j
Hence (C) is correct option.
MCQ 1.3 A series RLC circuit has a resonance frequency of 1 kHz and a quality factor
Q = 100 . If each of R, L and C is doubled from its original value, the new Q of the
circuit is
(A) 25 (B) 50
(C) 100 (D) 200
SOL 1.3 Hence (B) is correct option.
Q = 1 L
R C
When R, L and C are doubled,
Q' = 1 2L = 1 L =Q
2R 2C 2R C 2
Thus Q' = 100 = 50
2

MCQ 1.4 The Laplace transform of i (t) is given by


I (s) = 2
s (1 + s)
At t " 3, The value of i (t) tends to
(A) 0 (B) 1
(C) 2 (D) 3
SOL 1.4 From the Final value theorem we have
lim i (t) = lim sI (s) = lim s 2 = lim 2 =2
t"3 s"0 s " 0 s (1 + s) s " 0 (1 + s)

Hence (C) is correct answer


MCQ 1.5 The differential equation for the current i (t) in the circuit of the figure is

2 2
(A) 2 d 2i + 2 di + i (t) = sin t (B) d 2i + 2 di + 2i (t) = cos t
dt dt dt dt
2 2
(C) 2 d 2i + 2 di + i (t) = cos t (D) d 2i + 2 di + 2i (t) = sin t
dt dt dt dt
SOL 1.5 Applying KVL we get,
di (t) 1
sin t = Ri (t) + L
dt
+
C
# i (t) dt

Brought to you by: Nodia and Company Visit us at: www.nodia.co.in


PUBLISHING FOR GATE
Page 3 GATE EC 2003 www.gatehelp.com

di (t)
or sin t = 2i (t) + 2
dt
+ # i (t) dt
Differentiating with respect to t , we get
2di (t) 2d2 i (t)
cos t = + + i (t)
dt dt2
Hence (C) is correct option.
MCQ 1.6 n -type silicon is obtained by doping silicon with
(A) Germanium (B) Aluminium
(C) Boron (D) Phosphorus
SOL 1.6 Pentavalent make n −type semiconductor and phosphorous is pentavalent.
Hence option (D) is correct.
MCQ 1.7 The Bandgap of silicon at 300 K is
(A) 1.36 eV (B) 1.10 eV
(C) 0.80 eV (D) 0.67 eV
SOL 1.7 Hence option (B) is correct.
For silicon at 0 K Eg0 = 1.21 eV
At any temperature
EgT = Eg0 − 3.6 # 10 - 4 T
At T = 300 K,
Eg300 = 1.21 − 3.6 # 10 - 4 # 300 = 1.1 eV
This is standard value, that must be remembered.
MCQ 1.8 The intrinsic carrier concentration of silicon sample at 300 K is 1.5 # 1016 /m 3 . If
after doping, the number of majority carriers is 5 # 1020 /m 3 , the minority carrier
density is
(A) 4.50 # 1011/m 3 (B) 3.333 # 10 4 /m 3
(C) 5.00 # 1020 /m 3 (D) 3.00 # 10 - 5 /m 3
SOL 1.8 By Mass action law
np = ni2
2 16
p = ni = 1.5 # 10 # 120 .5 # 1016 = 4.5 # 1011
n 5 # 10
Hence option (A) is correct.
MCQ 1.9 Choose proper substitutes for X and Y to make the following statement correct
Tunnel diode and Avalanche photo diode are operated in X bias ad Y bias
respectively
(A) X: reverse, Y: reverse (B) X: reverse, Y: forward
(C) X: forward, Y: reverse (D) X: forward, Y: forward
SOL 1.9 Tunnel diode shows the negative characteristics in forward bias. It is used in forward
Brought to you by: Nodia and Company Visit us at: www.nodia.co.in
PUBLISHING FOR GATE
Page 4 GATE EC 2003 www.gatehelp.com

bias.
Avalanche photo diode is used in reverse bias.
Hence option (C) is correct.
MCQ 1.10 For an n − channel enhancement type MOSFET, if the source is connected at a
higher potential than that of the bulk (i.e. VSB > 0 ), the threshold voltage VT of the
MOSFET will
(A) remain unchanged (B) decrease
(C) change polarity (D) increase
SOL 1.10 Hence option (D) is correct.
MCQ 1.11 Choose the correct match for input resistance of various amplifier configurations
shown below :
Configuration Input resistance
CB : Common Base LO : Low
CC : Common Collector MO : Moderate
CE : Common Emitter HI : High
(A) CB − LO, CC − MO, CE − HI
(B) CB − LO, CC − HI, CE − MO
(C) CB − MO, CC − HI, CE − LO
(D) CB − HI, CC − LO, CE − MO
SOL 1.11 For the different combinations the table is as follows

CE CE CC CB
Ai High High Unity
Av High Unity High
Ri Medium High Low
Ro Medium Low High
Hence (B) is correct option.
MCQ 1.12 The circuit shown in the figure is best described as a

(A) bridge rectifier (B) ring modulator


(C) frequency discriminator (D) voltage double
SOL 1.12 This circuit having two diode and capacitor pair in parallel, works as voltage

Brought to you by: Nodia and Company Visit us at: www.nodia.co.in


PUBLISHING FOR GATE
Page 5 GATE EC 2003 www.gatehelp.com

doubler.
Hence (D) is correct option.
MCQ 1.13 If the input to the ideal comparators shown in the figure is a sinusoidal signal of 8
V (peak to peak) without any DC component, then the output of the comparators
has a duty cycle of

(A) 1/2 (B) 1/3


(C) 1/6 (D) 1/2
SOL 1.13 If the input is sinusoidal signal of 8 V (peak to peak) then
Vi = 4 sin ωt
The output of comparator will be high when input is higher than Vref = 2 V and
will be low when input is lower than Vref = 2 V. Thus the waveform for input is
shown below

From fig, first crossover is at ωt1 and second crossover is at ωt2 where
4 sin ωt1 = 2V
Thus ωt1 = sin - 1 1 = π
2 6
ωt2 = π − π = 5π
6 6
5π π
−6
Duty Cycle = 6 =1
2π 3
Thus the output of comparators has a duty cycle of 1 .
3
Hence (B) is correct option.
MCQ 1.14 If the differential voltage gain and the common mode voltage gain of a differential
amplifier are 48 dB and 2 dB respectively, then common mode rejection ratio is
(A) 23 dB (B) 25 dB

Brought to you by: Nodia and Company Visit us at: www.nodia.co.in


PUBLISHING FOR GATE
Page 6 GATE EC 2003 www.gatehelp.com

(C) 46 dB (D) 50 dB
SOL 1.14 Hence (C) is correct option.
CMMR = Ad
Ac
or 20 log CMMR = 20 log Ad − 20 log Ac
= 48 − 2 = 46 dB
Where Ad "Differential Voltage Gain
and AC " Common Mode Voltage Gain
MCQ 1.15 Generally, the gain of a transistor amplifier falls at high frequencies due to the
(A) internal capacitances of the device
(B) coupling capacitor at the input
(C) skin effect
(D) coupling capacitor at the output
SOL 1.15 The gain of amplifier is
− gm
Ai =
gb + jωC
Thus the gain of a transistor amplifier falls at high frequencies due to the internal
capacitance that are diffusion capacitance and transition capacitance.
Hence (B) is correct option.
MCQ 1.16 The number of distinct Boolean expressions of 4 variables is
(A) 16 (B) 256
(C) 1023 (D) 65536
SOL 1.16 The number of distinct boolean expression of n variable is 22n . Thus
22 = 216 = 65536
4

Hence (D) is correct answer.


MCQ 1.17 The minimum number of comparators required to build an 8-bits flash ADC is
(A) 8 (B) 63
(C) 255 (D) 256
SOL 1.17 In the flash analog to digital converter, the no. of comparators is equal to 2n - 1,
where n is no. of bit.s
So, 2n - 1 = 28 − 1 = 255
Hence (C) is correct answer.
MCQ 1.18 The output of the 74 series of GATE of TTL gates is taken from a BJT in
(A) totem pole and common collector configuration
(B) either totem pole or open collector configuration
(C) common base configuration

Brought to you by: Nodia and Company Visit us at: www.nodia.co.in


PUBLISHING FOR GATE
Page 7 GATE EC 2003 www.gatehelp.com

(D) common collector configuration


SOL 1.18 When output of the 74 series gate of TTL gates is taken from BJT then the
configuration is either totem pole or open collector configuration .
Hence (B) is correct answer.
MCQ 1.19 Without any additional circuitry, an 8:1 MUX can be used to obtain
(A) some but not all Boolean functions of 3 variables
(B) all functions of 3 variables but non of 4 variables
(C) all functions of 3 variables and some but not all of 4 variables
(D) all functions of 4 variables
SOL 1.19 A 2n: 1 MUX can implement all logic functions of (n + 1) variable without andy
additional circuitry. Here n = 3 . Thus a 8 : 1 MUX can implement all logic functions
of 4 variable.
Here (D) is correct answer.
MCQ 1.20 A 0 to 6 counter consists of 3 flip flops and a combination circuit of 2 input gate
(s). The common circuit consists of
(A) one AND gate (B) one OR gate
(C) one AND gate and one OR gate (D) two AND gates
SOL 1.20 Counter must be reset when it count 111. This can be implemented by following
circuitry

Hence (D) is correct answer.


MCQ 1.21 The Fourier series expansion of a real periodic signal with fundamental frequency
f0 is given by gp (t) =/ cn e j2πf t . It is given that c3 = 3 + j5 . Then c−3 is
0

(A) 5 + j3 n =- 3
(B) − 3 − j5
(C) − 5 + j3 (D) 3 − j5
SOL 1.21 Hence (D) is correct answer.
HereC3 = 3 + j5
For real periodic signal
C−k = Ck*
Thus C−3 = Ck = 3 − j5
MCQ 1.22 Let x (t) be the input to a linear, time-invariant system. The required output is
4π (t − 2). The transfer function of the system should be
(A) 4e j4πf (B) 2e−j8πf
(C) 4e−j4πf (D) 2e j8πf

Brought to you by: Nodia and Company Visit us at: www.nodia.co.in


PUBLISHING FOR GATE
Page 8 GATE EC 2003 www.gatehelp.com

SOL 1.22 Hence (C) is correct answer.


y (t) = 4x (t − 2)
Taking Fourier transform we get
Y (e j2πf ) = 4e−j2πf2 X (e j2πf ) Time Shifting property

Y (e j2πf )
or j 2π f
= 4e−4jπf
X (e )

Thus H (e j2πf ) = 4e−4jπf


MCQ 1.23 A sequence x (n) with the z −transform X (z) = z 4 + z2 − 2z + 2 − 3z−4 is applied as an
input to a linear, time-invariant system with the impulse response h (n) = 2δ (n − 3)
where
1, n = 0
δ (n) = )
0, otherwise
The output at n = 4 is
(A) − 6 (B) zero
(C) 2 (D) − 4
SOL 1.23 Hence (B) is correct answer.
We have h (n) = 3δ (n − 3)
or H (z) = 2z−3 Taking z transform
−4
X (z) = z + z − 2z + 2 − 3z
4 2

Now Y (z) = H (z) X (z)


= 2z−3 (z 4 + z2 − 2z + 2 − 3z−4)
= 2 (z + z−1 − 2z−2 + 2z−3 − 3z−7)
Taking inverse z transform we have
y (n) = 2[ δ (n + 1) + δ (n − 1) − 2δ (n − 2)+ 2δ (n − 3) − 3δ (n − 7)]
At n = 4 ,y (4) = 0
MCQ 1.24 Fig. shows the Nyquist plot of the open-loop transfer function G (s) H (s) of a system.
If G (s) H (s) has one right-hand pole, the closed-loop system is

(A) always stable


(B) unstable with one closed-loop right hand pole
(C) unstable with two closed-loop right hand poles
(D) unstable with three closed-loop right hand poles

Brought to you by: Nodia and Company Visit us at: www.nodia.co.in


PUBLISHING FOR GATE
Page 9 GATE EC 2003 www.gatehelp.com

SOL 1.24 Hence (A) is correct option.


Z = P−N
N " Net encirclement of (− 1 + j0) by Nyquist plot,
P " Number of open loop poles in right hand side of s − plane
Z " Number of closed loop poles in right hand side of s − plane
Here N = 1 and P = 1
Thus Z =0
Hence there are no roots on RH of s −plane and system is always stable.
MCQ 1.25 A PD controller is used to compensate a system. Compared to the uncompensated
system, the compensated system has
(A) a higher type number (B) reduced damping
(C) higher noise amplification (D) larger transient overshoot
SOL 1.25 PD Controller may accentuate noise at higher frequency. It does not effect the type
of system and it increases the damping. It also reduce the maximum overshoot.
Hence (C) is correct option.
MCQ 1.26 The input to a coherent detector is DSB-SC signal plus noise. The noise at the
detector output is
(A) the in-phase component (B) the quadrature - component
(C) zero (D) the envelope
SOL 1.26 The input is a coherent detector is DSB - SC signal plus noise. The noise at the
detector output is the in-phase component as the quadrature component nq (t) of
the noise n (t) is completely rejected by the detector.
Hence (A) is correct option.
MCQ 1.27 The noise at the input to an ideal frequency detector is white. The detector is
operating above threshold. The power spectral density of the noise at the output is
(A) raised - cosine (B) flat
(C) parabolic (D) Gaussian
SOL 1.27 The noise at the input to an ideal frequency detector is white. The PSD of noise at
the output is parabolic
Hence (C) is correct option.
MCQ 1.28 At a given probability of error, binary coherent FSK is inferior to binary coherent
PSK by.
(A) 6 dB (B) 3 dB
(C) 2 dB (D) 0 dB
SOL 1.28 Hence (B) is correct option.
We have Pe = 1 erfc c Ed m
2 2η

Brought to you by: Nodia and Company Visit us at: www.nodia.co.in


PUBLISHING FOR GATE
Page 10 GATE EC 2003 www.gatehelp.com

Since Pe of Binary FSK is 3 dB inferior to binary PSK


MCQ 1.29 The unit of 4# H is
(A) Ampere (B) Ampere/meter
(C) Ampere/meter 2 (D) Ampere-meter
SOL 1.29 By Maxwells equations
4# H = 2D + J
2t
Thus 4# H has unit of current density J that is A/m2
Hence (C) is correct option
MCQ 1.30 The depth of penetration of electromagnetic wave in a medium having conductivity
σ at a frequency of 1 MHz is 25 cm. The depth of penetration at a frequency of 4
MHz will be
(A) 6.25 dm (B) 12.50 cm
(C) 50.00 cm (D) 100.00 cm
SOL 1.30 Hence (B) is correct option.
We know that δ \ 1
f
Thus δ2 = f1
δ1 f2
δ2 = 1
25 4
or δ2 = 1 # 25 = 12.5 cm
4

Q.31-90 Carry Two Marks Each

MCQ 1.31 Twelve 1 Ω resistance are used as edges to form a cube. The resistance between two
diagonally opposite corners of the cube is
(A) 5 Ω (B) 1 Ω
6

(C) 6 Ω (D) 3 Ω
5 2
SOL 1.31 For current i there is 3 similar path. So current will be divide in three path

Brought to you by: Nodia and Company Visit us at: www.nodia.co.in


PUBLISHING FOR GATE
Page 11 GATE EC 2003 www.gatehelp.com

so, we get
Vab − b i # 1l − b i # 1l − b 1 # 1l = 0
3 6 3
Vab = R = 1 + 1 + 1 = 5 Ω
eq
i 3 6 3 6
Hence (A) is correct option.
MCQ 1.32 The current flowing through the resistance R in the circuit in the figure has the
form P cos 4t where P is

(A) (0.18 + j0.72) (B) (0.46 + j1.90)


(C) − (0.18 + j1.90) (D) − (0.192 + j0.144)
SOL 1.32 Data are missing in question as L1 &L2 are not given

The circuit for Q. 33 & 34 is given below.


Assume that the switch S is in position 1 for a long time and thrown to position
2 at t = 0 .

MCQ 1.33 At t = 0+ , the current i1 is


(A) − V (B) − V
2R R
Brought to you by: Nodia and Company Visit us at: www.nodia.co.in
PUBLISHING FOR GATE
Page 12 GATE EC 2003 www.gatehelp.com

(C) − V (D) zero


4R
SOL 1.33 Data are missing in question as L1 &L2 are not given
MCQ 1.34 I1 (s) and I2 (s) are the Laplace transforms of i1 (t) and i2 (t) respectively. The
equations for the loop currents I1 (s) and I2 (s) for the circuit shown in the figure,
after the switch is brought from position 1 to position 2 at t = 0 , are
R + Ls + Cs1 − Ls I1 (s) V
(A) > H= G = G
s
=
− Ls R + Cs I2 (s)
1
0
R + Ls + Cs1 − Ls I1 (s) − Vs
(B) > 1 H= G == G
− Ls R + Cs I2 (s) 0
R + Ls + Cs1 − Ls − Vs
(C) >
R + Ls + Cs1 H=I2 (s)G = 0 G
I1 (s)
=
− Ls
R + Ls + Cs1 − Cs V
(D) >
R + Ls + Cs1 H=I2 (s) = 0 G
I1 (s)
G
s
=
− Ls
SOL 1.34 At t = 0 - circuit is in steady state. So inductor act as short circuit and capacitor
act as open circuit.

At t = 0 - , i1 (0 -) = i2 (0 -) = 0
vc (0 -) = V
At t = 0+ the circuit is as shown in fig. The voltage across capacitor and current in
inductor can’t be changed instantaneously. Thus

At t = 0+ , i1 = i2 =− V
2R
Hence (A) is correct option.

Brought to you by: Nodia and Company Visit us at: www.nodia.co.in


PUBLISHING FOR GATE
Page 13 GATE EC 2003 www.gatehelp.com

MCQ 1.35 An input voltage v (t) = 10 2 cos (t + 10c) + 10 5 cos (2t + 10c) V is applied to a
series combination of resistance R = 1 Ω and an inductance L = 1 H. The resulting
steady-state current i (t) in ampere is
(A) 10 cos (t + 55c) + 10 cos (2t + 10c + tan−1 2)
(B) 10 cos (t + 55c) + 10 3
2 cos (2t + 55c)
(C) 10 cos (t − 35c) + 10 cos (2t + 10c − tan−1 2)
(D) 10 cos (t − 35c) + 3
2 cos (2t − 35c)
SOL 1.35 Hence (C) is correct option
v (t) = 10 2 cos (t + 10c) + 10 5 cos (2t + 10c)
1 4444 2 4444 3 1 4444 4 2 4444 43
v1 v2

Thus we get ω1 = 1 and ω2 = 2


Now Z1 = R + jω1 L = 1 + j1
Z2 = R + jω2 L = 1 + j2
v (t) v (t)
i (t) = 1 + 2
Z1 Z2
10 2 cos (t + 10c) 10 5 cos (2t + 10c)
= +
1+j 1 + j2
10 2 cos (t + 10c) 10 5 cos (2t + 10c)
= +
12 + 22 + tan−1 1 12 + 22 tan−1 2
10 2 cos (t + 10c) 10 5 cos (2t + 10c)
= +
2 + tan−1 45c 5 tan−1 2
i (t) = 10 cos (t − 35c) + 10 cos (2t + 10c − tan−1 2)
MCQ 1.36 The driving point impedance Z (s) of a network has the pole-zero locations as
shown in the figure. If Z (0) = 3 , then Z (s) is

3 (s + 3) 2 (s + 3)
(A) (B)
s + 2s + 3
2
s + 2s + 2
2

3 (s + 3) 2 (s − 3)
(C) 2 (D) 2
s + 2s + 2 s − 2s − 3

SOL 1.36 Hence (B) is correct option.


Zeros =− 3
Pole1 =− 1 + j
Pole 2 =− 1 − j

Brought to you by: Nodia and Company Visit us at: www.nodia.co.in


PUBLISHING FOR GATE
Page 14 GATE EC 2003 www.gatehelp.com

K (s + 3)
Z (s) =
(s + 1 + j)( s + 1 − j)
K (s + 3) K (s + 3)
= =
(s + 1) − j
2 2
(s + 1) 2 + 1
From problem statement Z (0) ω = 0 = 3
Thus 3K = 3 and we get K = 2
2
2 (s + 3)
Z (s) = 2
s + 2s + 2
MCQ 1.37 The impedance parameters z11 and z12 of the two-port network in the figure are

(A) z11 = 2.75 Ω and z12 = 0.25 Ω (B) z11 = 3 Ω and z12 = 0.5 Ω
(C) z11 = 3 Ω and z12 = 0.25 Ω (D) z11 = 2.25 Ω and z12 = 0.5 Ω
SOL 1.37 Using 3− Y conversion

R1 =2 # 1 = 2 = 0. 5
2+1+1 4
R2 = 1 # 1 = 1 = 0.25
2+1+1 4
R3 = 2 # 1 = 0.5
2+1+1
Now the circuit is as shown in figure below.

Now z11 = V1 = 2 + 0.5 + 0.25 = 2.75


I1 I2 = 0

z12 = R3 = 0.25

Brought to you by: Nodia and Company Visit us at: www.nodia.co.in


PUBLISHING FOR GATE
Page 15 GATE EC 2003 www.gatehelp.com

Hence (A) is correct option.


MCQ 1.38 An n −type silicon bar 0.1 cm long and 100 μm2 i cross-sectional area has a majority
carrier concentration of 5 # 1020 /m 2 and the carrier mobility is 0.13 m2 /V-s at 300
K. If the charge of an electron is 1.5 # 10 - 19 coulomb, then the resistance of the bar
is
(A) 106 Ohm (B) 10 4 Ohm
(C) 10 - 1 Ohm (D) 10 - 4 Ohm
SOL 1.38 Hence option (A) is correct.
ρl
We that R = , ρ = 1 and α = nqun
A σ
From above relation we have
R = 1
nqμn A
= 0.1 # 10 - 2 = 106 Ω
20 - 19 - 12
5 # 10 # 1.6 # 10 # 0.13 # 100 # 10
MCQ 1.39 The electron concentration in a sample of uniformly doped n -type silicon at 300
K varies linearly from 1017 /cm 3 at x = 0 to 6 # 1016 /cm 3 at x = 2μm . Assume a
situation that electrons are supplied to keep this concentration gradient constant
with time. If electronic charge is 1.6 # 10 - 19 coulomb and the diffusion constant
Dn = 35 cm 2 /s, the current density in the silicon, if no electric field is present, is
(A) zero (B) -112 A/cm 2
(C) +1120 A/cm 2 (D) -1120 A/cm 2
SOL 1.39 Hence option (D) is correct.
dn = 6 # 1016 − 1017
dx 2 # 10 - 4 − 0
=− 2 # 1020
Now Jn = nqμe E + Dn q dn
dx
Since no electric field is present, E = 0 and we get
So, Jn = qDn dn
dx
= 1.6 # 10 - 19 # 35 # (− 2 # 1020) =− 1120 A/cm 2
MCQ 1.40 Match items in Group 1 with items in Group 2, most suitably.
Group 1 Group 2
P. LED 1. Heavy doping
Q. Avalanche photo diode 2. Coherent radiation
R. Tunnel diode 3. Spontaneous emission
S. LASER 4. Current gain

Brought to you by: Nodia and Company Visit us at: www.nodia.co.in


PUBLISHING FOR GATE
Page 16 GATE EC 2003 www.gatehelp.com

(A) P - 1, Q - 2, R - 4, S - 3
(B) P - 2, Q - 3, R - 1, S - 4
(C) P - 3 Q - 4, R - 1, S - 2
(D) P - 2, Q - 1, R - 4, S - 3
SOL 1.40 LED works on the principal of spontaneous emission.
In the avalanche photo diode due to the avalanche effect there is large current gain.
Tunnel diode has very large doping.
LASER diode are used for coherent radiation.
Hence option (C) is correct.
MCQ 1.41 At 300 K, for a diode current of 1 mA, a certain germanium diode requires a
forward bias of 0.1435 V, whereas a certain silicon diode requires a forward bias of
0.718 V. Under the conditions state above, the closest approximation of the ratio
of reverse saturation current in germanium diode to that in silicon diode is
(A) 1 (B) 5
(C) 4 # 103 (D) 8 # 103
SOL 1.41 Hence option (C) is correct.
I = Io `e η V − 1j
V D1
We know that si T

where η = 1 for germanium and η = 2 silicon. As per question


Io `e e − 1j = Io `e ηV − 1j
V
Dsi V DGe
hVT
n Ge T

VDsi 0.718
= eVηV − 1 = e 2 #026.1435
# 10 − 1
Io
= 4 # 103
-3
or si T

Io e 26 # 10 − 1
e ηV − 1
DGe
si -3
T

MCQ 1.42 A particular green LED emits light of wavelength 5490 Ac. The energy bandgap of
the semiconductor material used there is
(Plank’s constant = 6.626 # 10 - 34 J − s )
(A) 2.26 eV (B) 1.98 eV
(C) 1.17 eV (D) 0.74 eV
SOL 1.42 Hence option (A) is correct
−34 8
Eg = hc = 6.626 # 10 # −310# 10 = 3.62 J
λ 54900 # 10
E (J) −19
In eV Eg (eV) = g = 3.62 # 10−19 = 2.26 eV
e 1.6 # 10
Alternatively
Eg = 1.24 eV = 1.24 = 2.26 eV
λ (μm) 5490 # 10−4 μm

MCQ 1.43 When the gate-to-source voltage (VGs) of a MOSFET with threshold voltage of 400
mV, working in saturation is 900 mV, the drain current is observed to be 1 mA.

Brought to you by: Nodia and Company Visit us at: www.nodia.co.in


PUBLISHING FOR GATE
Page 17 GATE EC 2003 www.gatehelp.com

Neglecting the channel width modulation effect and assuming that the MOSFET is
operating at saturation, the drain current for an applied VGS of 1400 mV is
(A) 0.5 mA (B) 2.0 mA
(C) 3.5 mA (D) 4.0 mA
SOL 1.43 We know that
ID = K (VGS − VT ) 2
I (VGS2 − VT ) 2
Thus D2
=
ID1 (VGS1 − VT ) 2
Substituting the values we have
ID2 = (1.4 − 0.4) = 4
2

ID1 (0.9 − 0.4) 2


or ID2 = 4IDI = 4 mA
Hence option (D) is correct.
MCQ 1.44 If P is Passivation, Q is n −well implant, R is metallization and S is source/drain
diffusion, then the order in which they are carried out in a standard n −well CMOS
fabrication process, is
(A) P − Q − R − S (B) Q − S − R − P
(C) R − P − S − Q (D) S − R − Q − P
SOL 1.44 In n −well CMOS fabrication following are the steps :
(i) n − well implant (Q)
(ii) Source drain diffusion (S)
(iii) Metalization (R)
(iv) Passivation (P)
Hence option (B) is correct.
MCQ 1.45 An amplifier without feedback has a voltage gain of 50, input resistance of 1 k Ω
and output resistance of 2.5 kΩ. The input resistance of the current-shunt negative
feedback amplifier using the above amplifier with a feedback factor of 0.2, is
(A) 1 kΩ (B) 1 kΩ
11 5
(C) 5 kΩ (D) 11 kΩ
SOL 1.45 Hence (A) is correct option.
We have Ri = 1kΩ, β = 0.2, A = 50

Ri
Thus, Rif = = 1 kΩ
(1 + Aβ) 11

MCQ 1.46 In the amplifier circuit shown in the figure, the values of R1 and R2 are such that
the transistor is operating at VCE = 3 V and IC = 1.5 mA when its β is 150. For a
transistor with β of 200, the operating point (VCE , IC ) is

Brought to you by: Nodia and Company Visit us at: www.nodia.co.in


PUBLISHING FOR GATE
Page 18 GATE EC 2003 www.gatehelp.com

(A) (2 V, 2 mA) (B) (3 V, 2 mA)


(C) (4 V, 2 mA) (D) (4 V, 1 mA)
SOL 1.46 The DC equivalent circuit is shown as below. This is fixed bias circuit operating in
active region.

In first case
VCC − IC1 R2 − VCE1 = 0
or 6 − 1.5mR2 − 3 = 0
or R2 = 2kΩ
IB1 = IC1 = 1.5m = 0.01 mA
β1 150
In second case IB2 will we equal to IB1 as there is no in R1.
Thus IC2 = β2 IB2 = 200 # 0.01 = 2 mA
VCE2 = VCC − IC2 R2 = 6 − 2m # 2 kΩ = 2 V
Hence (A) is correct option.
MCQ 1.47 The oscillator circuit shown in the figure has an ideal inverting amplifier. Its
frequency of oscillation (in Hz) is

(A) 1 (B) 1
(2π 6 RC) (2πRC)

(C) 1 (D) 6
( 6 RC) (2πRC)

Brought to you by: Nodia and Company Visit us at: www.nodia.co.in


PUBLISHING FOR GATE
Page 19 GATE EC 2003 www.gatehelp.com

SOL 1.47 The given circuit is a R − C phase shift oscillator and frequency of its oscillation is
f = 1
2π 6 RC
Hence (A) is correct option.
MCQ 1.48 The output voltage of the regulated power supply shown in the figure is

(A) 3 V (B) 6 V
(C) 9 V (D) 12 V
SOL 1.48 If we see th figure we find that the voltage at non-inverting terminal is 3 V by the
zener diode and voltage at inverting terminal will be 3 V. Thus Vo can be get by
applying voltage division rule, i.e.
20 V = 3
20 + 40 o
or V0 = 9 V
Hence (C) is correct option.
MCQ 1.49 The action of JFET in its equivalent circuit can best be represented as a
(A) Current controlled current source
(B) Current controlled voltage source
(C) Voltage controlled voltage source
(D) Voltage controlled current source
SOL 1.49 For a JFET in active region we have
IDS = IDSS c1 − VGS m
2

VP
From above equation it is clear that the action of a JFET is voltage controlled
current source.
Hence option (D) is correct.
MCQ 1.50 If the op-amp in the figure is ideal, the output voltage Vout will be equal to

Brought to you by: Nodia and Company Visit us at: www.nodia.co.in


PUBLISHING FOR GATE
Page 20 GATE EC 2003 www.gatehelp.com

(A) 1 V (B) 6 V
(C) 14 V (D) 17 V
SOL 1.50 The circuit is as shown below

V+ = 8 (3) = 8 kΩ
1+8 3
V+ = V- = 8 V
3
Now applying KCL at inverting terminal we get
V- − 2 + V- − Vo = 0
1 5
or Vo = 6V- − 10
= 6 # 8 − 10 = 6 V
3
Hence (B) is correct option.
MCQ 1.51 Three identical amplifiers with each one having a voltage gain of 50, input resistance
of 1 kΩ and output resistance of 250 Ω are cascaded. The opened circuit voltages
gain of the combined amplifier is
(A) 49 dB (B) 51 dB
(C) 98 dB (D) 102 dB
SOL 1.51 The equivalent circuit of 3 cascade stage is as shown in fig.

Brought to you by: Nodia and Company Visit us at: www.nodia.co.in


PUBLISHING FOR GATE
Page 21 GATE EC 2003 www.gatehelp.com

V2 = 1k 50V1 = 40V1
1k + 0.25k
Similarly V3 = 1k 50V2 = 40V2
1k + 0.25k
or V3 = 40 # 40V1
Vo = 50V3 = 50 # 40 # 40V1
or AV = Vo = 50 # 40 # 40 = 8000
V1
or 20 log AV = 20 log 8000 = 98 dB
Hence (C) is correct option.
MCQ 1.52 An ideal sawtooth voltages waveform of frequency of 500 Hz and amplitude 3 V
is generated by charging a capacitor of 2 μF in every cycle. The charging requires
(A) Constant voltage source of 3 V for 1 ms
(B) Constant voltage source of 3 V for 2 ms
(C) Constant voltage source of 1 mA for 1 ms
(D) Constant voltage source of 3 mA for 2 ms
SOL 1.52 If a constant current is made to flow in a capacitor, the output voltage is integration
of input current and that is sawtooth waveform as below :
#
t
VC = 1 idt
C 0
The time period of wave form is
T = 1 = 1 = 2 m sec
f 500
-3

#
20 # 10
Thus 3= 1 idt
2 # 106 0
or i (2 # 10 - 3 − 0) = 6 # 10 - 6
or i = 3 mA
Thus the charging require 3 mA current source for 2 msec.
Hence (D) is correct option
MCQ 1.53 The circuit in the figure has 4 boxes each described by inputs P, Q, R and outputs
Y, Z with Y = P 5 Q 5 R and Z = RQ + PR + QP
The circuit acts as a

Brought to you by: Nodia and Company Visit us at: www.nodia.co.in


PUBLISHING FOR GATE
Page 22 GATE EC 2003 www.gatehelp.com

(A) 4 bit adder giving P + Q


(B) 4 bit subtractor giving P − Q
(C) 4 bit subtractor giving Q-P
(D) 4 bit adder giving P + Q + R
SOL 1.53 Hence (B) is correct answer.
We have Y = P 5 Q 5 R
Z = RQ + PR + QP
Here every block is a full subtractor giving P − Q − R where R is borrow. Thus
circuit acts as a 4 bit subtractor giving P − Q .
MCQ 1.54 If the function W, X, Y and Z are as follows
W = R + PQ + RS X = PQRS + PQRS + PQRS
Y = RS + PR + PQ + P .Q Z = R + S + PQ + P .Q .R + PQ .S
Then,
(A) W = Z, X = Z (B) W = Z, X = Y
(C) W = Y (D) W = Y = Z
SOL 1.54 Hence (A) is correct answer.
W = R + PQ + RS
X = PQRS + PQRS + PQRS
Y = RS + PR + PQ + PQ
= RS + PR $ PQ $ PQ
= RS + (P + R )( P + Q)( P + Q)
= RS + (P + PQ + PR + QR )( P + Q)
= RS + PQ + QR (P + P ) + QR
= RS + PQ + QR
Z = R + S + PQ + PQR + PQS
= R + S + PQ $ PQR $ PQS
= R + S + (P + Q )( P + Q + R)( P + Q + S)
= R + S + PQ + PQ + PQS + PR + PQR
+ PRS + PQ + PQS + PQR + QRS
= R + S + PQ + PQS + PR + PQR + PRS

Brought to you by: Nodia and Company Visit us at: www.nodia.co.in


PUBLISHING FOR GATE
Page 23 GATE EC 2003 www.gatehelp.com

+ PQS + PQR + QRS


= R + S + PQ (1 + S) + PR (1 + P ) + PRS
+ PQS + PQR + QRS
= R + S + PQ + PR + PRS + PQS
+ PQR + QRS
= R + S + PQ + PR (1 + Q ) + PQS + QRS
= R + S + PQ + PR + PQS + QRS
Thus W = Z and X = Z
MCQ 1.55 A 4 bit ripple counter and a bit synchronous counter are made using flip flops
having a propagation delay of 10 ns each. If the worst case delay in the ripple
counter and the synchronous counter be R and S respectively, then
(A) R = 10 ns, S = 40 ns (B) R = 40 ns, S = 10 ns
(C) R = 10 ns S = 30 ns (D) R = 30 ns, S = 10 ns
SOL 1.55 Propagation delay of flip flop is
tpd = 10 nsec
Propagation delay of 4 bit ripple counter
R = 4tpd = 40 ns
and in synchronous counter all flip-flop are given clock simultaneously, so
S = tpd = 10 ns
Hence (B) is correct answer.
MCQ 1.56 The DTL, TTL, ECL and CMOS famil GATE of digital ICs are compared in the
following 4 columns

(P) (Q) (R) (S)


Fanout is minimum DTL DTL TTL CMOS
Power consumption is TTL CMOS ECL DTL
minimum
Propagation delay is CMOS ECL TTL TTL
minimum
The correct column is
(A) P (B) Q
(C) R (D) S
SOL 1.56 The DTL has minimum fan out and CMOS has minimum power consumption.
Propagation delay is minimum in ECL.
Hence (B) is correct answer.
MCQ 1.57 The circuit shown in the figure is a 4 bit DAC

Brought to you by: Nodia and Company Visit us at: www.nodia.co.in


PUBLISHING FOR GATE
Page 24 GATE EC 2003 www.gatehelp.com

The input bits 0 and 1 are represented by 0 and 5 V respectively. The OP AMP
is ideal, but all the resistance and the 5 v inputs have a tolerance of ! 10%. The
specification (rounded to nearest multiple of 5%) for the tolerance of the DAC is
(A) ! 35% (B) ! 20%
(C) ! 10% (D) ! 5%
SOL 1.57 Hence (A) is correct answer.
Vo =− V1 :R bo + R b1 + R b2 + R b 3D
R 2R 4R 4R
Exact value when V1 = 5 , for maximum output
VoExact =− 5 :1 + 1 + 1 + 1 D =− 9.375
2 4 8
Maximum Vout due to tolerance
Vo max =− 5.5 :110 + 110 + 110 + 110 D
90 2 # 90 4 # 90 8 # 90
=− 12.604
Tolerance = 34.44% = 35%
MCQ 1.58 The circuit shown in figure converts

(A) BCD to binary code (B) Binary to excess - 3 code


(C) Excess -3 to gray code (D) Gray to Binary code
SOL 1.58 Hence (D) is correct answer.
Let input be 1010; output will be 1101
Let input be 0110; output will be 0100

Brought to you by: Nodia and Company Visit us at: www.nodia.co.in


PUBLISHING FOR GATE
Page 25 GATE EC 2003 www.gatehelp.com

Thus it convert gray to Binary code.


MCQ 1.59 In the circuit shown in the figure, A is parallel-in, parallel-out 4 bit register, which
loads at the rising edge of the clock C . The input lines are connected to a 4 bit bus,
W . Its output acts at input to a 16 # 4 ROM whose output is floating when the
input to a partial table of the contents of the ROM is as follows

Data 0011 1111 0100 1010 1011 1000 0010 1000

Address 0 2 4 6 8 10 11 14

The clock to the register is shown, and the data on the W bus at time t1 is 0110.
The data on the bus at time t2 is

(A) 1111 (B) 1011


(C) 1000 (D) 0010
SOL 1.59 After t = t1, at first rising edge of clock, the output of shift register is 0110, which
in input to address line of ROM. At 0110 is applied to register. So at this time data
stroed in ROM at 1010 (10), 1000 will be on bus.
When W has the data 0110 and it is 6 in decimal, and it’s data value at that add
is 1010

Brought to you by: Nodia and Company Visit us at: www.nodia.co.in


PUBLISHING FOR GATE
Page 26 GATE EC 2003 www.gatehelp.com

then 1010 i.e. 10 is acting as odd, at time t2 and data at that movement is 1000.
Hence (C) is correct answer.
MCQ 1.60 In an 8085 microprocessor, the instruction CMP B has been executed while the
content of the accumulator is less than that of register B . As a result
(A) Carry flag will be set but Zero flag will be reset
(B) Carry flag will be rest but Zero flag will be set
(C) Both Carry flag and Zero flag will be rest
(D) Both Carry flag and Zero flag will be set
SOL 1.60 CMP B & Compare the accumulator content with context of Register B
If A < R CY is set and zero flag will be reset.
Hence (A) is correct answer.
MCQ 1.61 Let X and Y be two statistically independent random variables uniformly
distributed in the ranges (− 1, 1) and (− 2, 1) respectively. Let Z = X + Y . Then
the probability that (z #− 1) is
(A) zero (B) 1
6

(C) 1 (D) 1
3 12
SOL 1.61 The pdf of Z will be convolution of pdf of X and pdf of Y as shown below.
#- 3 fZ (z) dz
z
Now p [Z # z] =

#- 3fZ (z) dz
-2
p [Z #− 2] =

= Area [z #− 2]
= 1 # 1 #1 = 1
2 6 12

Brought to you by: Nodia and Company Visit us at: www.nodia.co.in


PUBLISHING FOR GATE
Page 27 GATE EC 2003 www.gatehelp.com

Hence (D) is correct option.


MCQ 1.62 Let P be linearity, Q be time-invariance, R be causality and S be stability. A
discrete time system has the input-output relationship,
x (n) n$1
y (n) = *0, n=0
x (n + 1) n # − 1
where x (n) is the input and y (n) is the output. The above system has the properties
(A) P, S but not Q, R (B) P, Q, S but not R
(C) P, Q, R, S (D) Q, R, S but not P
SOL 1.62 System is non causal because output depends on future value
For n # 1 y (− 1) = x (− 1 + 1) = x (0)
y (n − n0) = x (n − n0 + 1) Time varying
y (n) = x (n + 1) Depends on Future
i.e. y (1) = x (2) None causal
For bounded input, system has bounded output. So it is stable.
y (n) = x (n) for n $ 1
= 0 for n = 0
= x (x + 1) for n #− 1
So system is linear.
Hence (A) is correct answer.

Common data for Q 63 & 64 :

Brought to you by: Nodia and Company Visit us at: www.nodia.co.in


PUBLISHING FOR GATE
Page 28 GATE EC 2003 www.gatehelp.com

The system under consideration is an RC low-pass filter (RC-LPF) with


R = 1 k Ω and C = 1.0 μ F.

MCQ 1.63 Let H (f) denote the frequency response of the RC-LPF. Let f1 be the highest
H (f1)
frequency such that 0 # f # f1 $ 0.95 . Then f1 (in Hz) is
H (0)
(A) 324.8 (B) 163.9
(C) 52.2 (D) 104.4
SOL 1.63 The frequency response of RC-LPF is
H (f) = 1
1 + j2πfRC
Now H (0) = 1
H (f1) 1
= $ 0.95
H (0) 1 + 4π2 f12 R2 C2
or 1 + 4π2 f12 R2 C2 # 1.108
or 4π2 f12 R2 C2 # 0.108
or 2πf1 RC # 0.329
or f1 # 0.329
2πRC
or f1 # 0.329
2πRC
or f1 # 0.329
2π1k # 1μ
or f1 # 52.2 Hz
Thus f1 max = 52.2 Hz
Hence (C) is correct answer.
MCQ 1.64 Let tg (f) be the group delay function of the given RC-LPF and f2 = 100 Hz. Then
tg (f2) in ms, is
(A) 0.717 (B) 7.17
(C) 71.7 (D) 4.505
SOL 1.64 Hence (A) is correct answer
H (ω) = 1
1 + jωRC
θ (ω) =− tan−1 ωRC
dθ (ω) RC
tg =− =
dω 1 + ω2 R2 C2
= 10−3 = 0.717 ms
1 + 4π # 10 4 # 10−6
2

Common Data for Questions 65 & 66 :

Brought to you by: Nodia and Company Visit us at: www.nodia.co.in


PUBLISHING FOR GATE
Page 29 GATE EC 2003 www.gatehelp.com

X (t) is a random process with a constant mean value of 2 and the auto
correlation function Rxx (τ) = 4 (e - 0.2 τ + 1).

MCQ 1.65 Let X be the Gaussian random variable obtained by sampling the process at t = ti
and let
#
3 x2

Q (α) = − 1 e dy 2

α 2π
The probability that 6x # 1@ is
(A) 1 − Q (0.5) (B) Q (0.5)

(C) Q c 1 m (D) 1 − Q c 1 m
2 2 2 2
SOL 1.65 Hence (D) is correct option.
We have RXX (τ) = 4 (e - 0.2 τ + 1)
RXX (0) = 4 (e - 0.2 0 + 1) = 8 = σ2
or σ = 2 2 Given
mean μ =0
Now P (x # 1) = Fx (1)
X−μ
= 1 − Qc
σ m
at x = 1

= 1 − Qc 1 − 0 m = 1 − Qc 1 m
2 2 2 2

MCQ 1.66 Let Y and Z be the random variable obtained by sampling X (t) at t = 2 and t = 4
respectively. Let W = Y − Z . The variance of W is
(A) 13.36 (B) 9.36
(C) 2.64 (D) 8.00
SOL 1.66 Hence (C) is correct option.
W = Y−Z
E [W2] = E [Y − Z] 2
= E [Y2] + E [Z2] − 2E [YZ]
= σw2
We haveE [X2 (t)] = Rx (10)
= 4 [e - 0.2 0 + 1] = 4 [1 + 1] = 8
E [Y2] = E [X2 (2)] = 8
E [Z2] = E [X2 (4)] = 8
E [YZ] = RXX (2) = 4 [e−0.2 (4 − 2) + 1] = 6.68
E [W2] = σw2 = 8 + 8 − 2 # 6.68 = 2.64
MCQ 1.67 Let x (t) = 2 cos (800π) + cos (1400πt). x (t) is sampled with the rectangular pulse
train shown in the figure. The only spectral components (in kHz) present in the
sampled signal in the frequency range 2.5 kHz to 3.5 kHz are

Brought to you by: Nodia and Company Visit us at: www.nodia.co.in


PUBLISHING FOR GATE
Page 30 GATE EC 2003 www.gatehelp.com

(A) 2.7, 3.4 (B) 3.3, 3.6


(C) 2.6, 2.7, 3.3, 3.4, 3.6 (D) 2.7, 3.3
SOL 1.67 Hence (D) is correct option.
The frequency of pulse train is
f 1- 3 = 1 k Hz
10
The Fourier Series coefficient of given pulse train is
−T /2
Cn = 1 #
o

Ae−jnω t dt o

To −T /2 o

−To /6
= 1
To #
−To /6
Ae−jηω t dt
o

= A [e−jω t] −−TT //66


o o

To (− jηωo) o

= A (e−jω t − e jηω T /6)


o o o

(− j2πn)
= A (e jηπ/3 − e−jηπ/3)
j2πn
or Cn = A sin ` nπ j
πn 3
From Cn it may be easily seen that 1, 2, 4, 5, 7, harmonics are present and 0, 3, 6, 9,..
are absent. Thus p (t) has 1 kHz, 2 kHz, 4 kHz, 5 kHz, 7 kHz,... frequency component
and 3 kHz, 6 kHz.. are absent.
The signal x (t) has the frequency components 0.4 kHz and 0.7 kHz. The sampled
signal of x (t) i.e. x (t)* p (t) will have
1 ! 0.4 and 1 ! 0.7 kHz
2 ! 0.4 and 2 ! 0.7 kHz
4 ! 0.4 and 4 ! 0.7 kHz
Thus in range of 2.5 kHz to 3.5 kHz the frequency present is
2 + 0.7 = 2.7 kHz
4 − 0.7 = 3.3 kHz
MCQ 1.68 The signal flow graph of a system is shown in Fig. below. The transfer function
C (s)/ R (s) of the system is

Brought to you by: Nodia and Company Visit us at: www.nodia.co.in


PUBLISHING FOR GATE
Page 31 GATE EC 2003 www.gatehelp.com

(A) 6 (B) 6s
s + 29s + 6
2
s + 29s + 6
2

s (s + 2) s (s + 27)
(C) 2 (D) 2
s + 29s + 6 s + 29s + 6

SOL 1.68 Mason Gain Formula


Σpk 3 k
T (s) =
3
In given SFG there is only forward path and 3 possible loop.
p1 = 1
31 = 1 + 3 + 24 = s + 27
s s s
L1 = − 2 , L2 = − 24 and L3 = − 3
s s s
where L1 and L3 are non-touching
C (s) p1 3 1
This =
R (s) 1 − (loop gain) + pair of non − touching loops
^ s +s27 h ^ s h
s + 27

= =
1 − ^ −s3 − 24s − s2 h + −s2 . −s3 1 + 29s + s62

s (s + 27)
= 2
s + 29s + 6
Hence (D) is correct option.

MCQ 1.69 The root locus of system G (s) H (s) = K has the break-away point
s (s + 2)( s + 3)
located at
(A) (− 0.5, 0) (B) (− 2.548, 0)
(C) (− 4, 0) (D) (− 0.784, 0)
SOL 1.69 We have
1 + G (s) H (s) = 0
or 1+ K =0
s (s + 2)( s + 3)
or K =− s (s2 + 5s2 + 6s)
dK =− (3s2 + 10s + 6) = 0
ds
which gives s = − 10 ! 100 − 72 =− 0.784, − 2.548
6
The location of poles on s − plane is

Brought to you by: Nodia and Company Visit us at: www.nodia.co.in


PUBLISHING FOR GATE
Page 32 GATE EC 2003 www.gatehelp.com

Since breakpoint must lie on root locus so s =− 0.748 is possible.


Hence (D) is correct option.
MCQ 1.70 The approximate Bode magnitude plot of a minimum phase system is shown in Fig.
below. The transfer function of the system is

(s + 0.1) 3 (s + 0.1) 3
(A) 108 (B) 107
(s + 10) 2 (s + 100) (s + 10)( s + 100)
(s + 0.1) 2 (s + 0.1) 3
(C) (D)
(s + 10) 2 (s + 100) (s + 10)( s + 100) 2

SOL 1.70 The given bode plot is shown below

At ω = 0.1 change in slope is + 60 dB " 3 zeroes at ω = 0.1


At ω = 10 change in slope is − 40 dB " 2 poles at ω = 10
At ω = 100 change in slope is − 20 dB " 1 poles at ω = 100
K ( 0s.1 + 1) 3
Thus T (s) = s
( 10 + 1) 2 ( 100
s
+ 1)
Now 20 log10 K = 20
or K = 10
10 ( 0s.1 + 1) 3 108 (s + 0.1) 3
Thus T (s) = s =
( 10 + 1) 2 ( 100
s
+ 1) (s + 10) 2 (s + 100)
Hence (A) is correct option.

Brought to you by: Nodia and Company Visit us at: www.nodia.co.in


PUBLISHING FOR GATE
Page 33 GATE EC 2003 www.gatehelp.com

MCQ 1.71 A second-order system has the transfer function


C (s)
= 2 4
R (s) s + 4s + 4
With r (t) as the unit-step function, the response c (t) of the system is represented
by

SOL 1.71 The characteristics equation is


s2 + 4s + 4 = 0
Comparing with
s2 + 2ξωn + ωn2 = 0
we get 2ξωn = 4 and ωn2 = 4
Thus ξ =1 Critically damped
ts = 4 = 4 = 2
ξωn 1#2
Hence (B) is correct option.
MCQ 1.72 The gain margin and the phase margin of feedback system with
G (s) H (s) = 8 are
(s + 100) 3
(A) dB, 0c (B) 3, 3
(C) 3, 0c (D) 88.5 dB, 3
SOL 1.72 Hence (B) is correct option.
MCQ 1.73 The zero-input response of a system given by the state-space equation
xo1 1 0 x1 x1 (0) 1
=xo G = =1 1G=x G and =x (0)G = = 0 G is
2 2 2

Brought to you by: Nodia and Company Visit us at: www.nodia.co.in


PUBLISHING FOR GATE
Page 34 GATE EC 2003 www.gatehelp.com

tet et
(A) = G (B) = G
t t
et t
(C) = t G (D) = t G
te te
SOL 1.73 We have
xo1 1 0 x1 x1 (0) 1
=xo G = =1 G=
1 x2 G and =
x2 (0) G == G
0
2
1 0
A ==
1 1G
s 0 1 0 s−1 0
(sI − A) = = G −= G = =
0 s 1 1 − 1 s − 1G
(s − 1) 0 1
0
1 > H = > +1 H
−1 s−1
(sI − A) =
(s − 1) 2 + 1 (s − 1)
1
(s − 1) 2 s−1
t
L−1 [(sI − A) −1] = eAt = = t t G
e 0
te e
et 0 1 et
x (t) = e # [x (t0)] = = t t G= G = = t G
At
te e 0 te
Hence (C) is correct option.
MCQ 1.74 A DSB-SC signal is to be generated with a carrier frequency fc = 1 MHz using a
non-linear device with the input-output characteristic V0 = a0 vi + a1 vi3 where a0
and a1 are constants. The output of the non-linear device can be filtered by an
appropriate band-pass filter.
Let Vi = Aci cos (2πfi ct) + m (t) is the message signal. Then the value of fci (in MHz)
is
(A) 1.0 (B) 0.333
(B) 0.5 (D) 3.0
SOL 1.74 Hence (C) is correct option.
vi = Ac1 cos (2πfc t) + m (t)
v0 = ao vi + avi3
v0 = a0 [Ac' cos (2πfc' t) + m (t)] + a1 [Ac' cos (2πfc' t) + m (t)] 3
= a0 Ac' cos (2πfc' t) + a0 m (t) + a1 [(Ac' cos 2πfc' t) 3
+ (Ac' cos (2πfc') t) 2 m (t) + 3Ac' cos (2πfc' t) m2 (t) + m3 (t)]
1 + cos (4πfc' t)
= a0 Ac' cos (2πfc' t) + a0 m (t) + a1 (Ac' cos 2fc' t) 3 + 3a1 Ac'2 ; E m (t)
2
= 3a1 Ac' cos (2πfc' t) m2 (t) + m3 (t)

The term 3a1 Ac' ( cos 42πf t ) m (t) is a DSB-SC signal having carrier frequency 1. MHz.
'
c

Thus 2fc' = 1 MHz or fc' = 0.5 MHz

Brought to you by: Nodia and Company Visit us at: www.nodia.co.in


PUBLISHING FOR GATE
Page 35 GATE EC 2003 www.gatehelp.com

Common Data for Question 75 & 76 :


Let m (t) = cos [(4π # 103) t] be the message signal &
c (t) = 5 cos [(2π # 106 t)] be the carrier.

MCQ 1.75 c (t) and m (t) are used to generate an AM signal. The modulation index of the
Total sideband power
generated AM signal is 0.5. Then the quantity is
Carrier power
(A) 1 (B) 1
2 4

(C) 1 (D) 1
3 8
SOL 1.75 Hence (D) is correct option.
PT = Pc c1 + α m
2

2
P (0.5) 2
Psb = Pc α = c
2

2 2
or Psb = 1
Pc 8

MCQ 1.76 c (t) and m (t) are used to generated an FM signal. If the peak frequency deviation
of the generated FM signal is three times the transmission bandwidth of the AM
signal, then the coefficient of the term cos [2π (1008 # 103 t)] in the FM signal (in
terms of the Bessel coefficients) is
(A) 5J4 (3) (B) 5 J8 (3)
2

(C) 5 J8 (4) (D) 5J4 (6)


2
SOL 1.76 Hence (D) is correct option.
AM Band width = 2fm
Peak frequency deviation = 3 (2fm) = 6fm
6f
Modulation index β = m = 6
fm
The FM signal is represented in terms of Bessel function as
3
xFM (t) = Ac /Jn (β) cos (ωc − nωn) t
n =- 3
ωc + nωm = 2π (1008 # 103)
2π106 + n4π # 103 = 2π (1008 # 103), n = 4
Thus coefficient = 5J4 (6)
MCQ 1.77 Choose the correct one from among the alternative A, B, C, D after matching an
item in Group 1 with most appropriate item in Group 2.
Group 1 Group 2
P. Ring modulator 1. Clock recovery

Brought to you by: Nodia and Company Visit us at: www.nodia.co.in


PUBLISHING FOR GATE
Page 36 GATE EC 2003 www.gatehelp.com

Q. VCO 2. Demodulation of FM
R. Foster-Seely discriminator 3. Frequency conversion
S. Mixer 4. Summing the two inputs
5. Generation of FM
6. Generation of DSB-Sc
(A) P − 1; Q − 3; R − 2; S − 4 (B) P − 6; Q = 5; R − 2; S − 3
(C) P − 6; Q − 1; R − 3; S − 2 (D) P − 5; Q − 6; R − 1; S − 3
SOL 1.77 Hence (B) is correct option.
Ring modulation $ Generation of DSB - SC
VCO $ Generation of FM
Foster seely discriminator $ Demodulation of fm
mixer $ frequency conversion
MCQ 1.78 A superheterodyne receiver is to operate in the frequency range 550 kHz - 1650
kHz, with the intermediate frequency of 450 kHz. Let R = Cmax /Cmin denote the
required capacitance ratio of the local oscillator and I denote the image frequency
(in kHz) of the incoming signal. If the receiver is tuned to 700 kHz, then
(A) R = 4.41, I = 1600 (B) R = 2.10, I − 1150
(C) R = 3.0, I = 600 (D) R = 9.0, I = 1150
SOL 1.78 Hence (A) is correct option.
fmax = 1650 + 450 = 2100 kHz
fmin = 550 + 450 = 1000 kHz
or f = 1
2π LC
frequency is minimum, capacitance will be maximum
f2
R = Cmax = max 2
= (2.1) 2
Cmin fmin
or R = 4.41
fi = fc + 2fIF = 700 + 2 (455) = 1600 kHz
MCQ 1.79 A sinusoidal signal with peak-to-peak amplitude of 1.536 V is quantized into 128
levels using a mid-rise uniform quantizer. The quantization-noise power is
(A) 0.768 V (B) 48 # 10 - 6 V2
(B) 12 # 10 - 6 V2 (D) 3.072 V
SOL 1.79 Hence (C) is correct option.
2mp
Step size δ = = 1.536 = 0.012 V
L 128
(0.012) 2
Quantization Noise power = δ =
2

12 12
= 12 # 10−6 V2

Brought to you by: Nodia and Company Visit us at: www.nodia.co.in


PUBLISHING FOR GATE
Page 37 GATE EC 2003 www.gatehelp.com

MCQ 1.80 If Eb , the energy per bit of a binary digital signal, is 10 - 5 watt-sec and the one-sided
power spectral density of the white noise, N0 = 10 - 6 W/Hz, then the output SNR
of the matched filter is
(A) 26 dB (B) 10 dB
(C) 20 dB (D) 13 dB
SOL 1.80 Hence (D) is correct option.
Eb = 10 - 6 watt-sec
No = 10 - 5 W/Hz
6
(SNR) matched filler = EN
o
= 10 - 5 = .05
2
o
2 # 10
(SNR)dB = 10 log 10 (0.05) = 13 dB
MCQ 1.81 The input to a linear delta modulator having a step-size 3= 0.628 is a sine wave
with frequency fm and peak amplitude Em . If the sampling frequency fx = 40 kHz,
the combination of the sine-wave frequency and the peak amplitude, where slope
overload will take place is
Em fm
(A) 0.3 V 8 kHz
(B) 1.5 V 4 kHz
(C) 1.5 V 2 kHz
(D) 3.0 V 1 kHz
SOL 1.81 Hence (B) is correct option.
3 fs
For slopeoverload to take place Em $
2πfm
This is satisfied with Em = 1.5 V and fm = 4 kHz
MCQ 1.82 If S represents the carrier synchronization at the receiver and ρ represents the
bandwidth efficiency, then the correct statement for the coherent binary PSK is
(A) ρ = 0.5, S is required (B) ρ = 1.0, S is required
(C) ρ = 0.5, S is not required (D) ρ = 1.0, S is not required
SOL 1.82 Hence (A) is correct option.
If s " carrier synchronization at receiver
ρ " represents bandwidth efficiency
then for coherent binary PSK ρ = 0.5 and s is required.
MCQ 1.83 A signal is sampled at 8 kHz and is quantized using 8 - bit uniform quantizer.
Assuming SNRq for a sinusoidal signal, the correct statement for PCM signal with
a bit rate of R is
(A) R = 32 kbps, SNRq = 25.8 dB (B) R = 64 kbps, SNRq = 49.8 dB
(C) R = 64 kbps, SNRq = 55.8 dB (D) R = 32 kbps, SNRq = 49.8 dB
SOL 1.83 Hence (B) is correct option.

Brought to you by: Nodia and Company Visit us at: www.nodia.co.in


PUBLISHING FOR GATE
Page 38 GATE EC 2003 www.gatehelp.com

Bit Rate = 8k # 8 = 64 kbps


(SNR)q = 1.76 + 6.02n dB
= 1.76 + 6.02 # 8 = 49.8 dB
MCQ 1.84 Medium 1 has the electrical permittivity ε1 = 1.5ε0 farad/m and occupies the
region to the left of x = 0 plane. Medium 2 has the electrical permittivity ε2 = 2.5ε0
farad/m and occupies the region to the right of x = 0 plane. If E1 in medium 1 is
E1 = (2ux − 3uy + 1uz ) volt/m, then E2 in medium 2 is
(A) (2.0ux − 7.5uy + 2.5uz ) volt/m (B) (2.0ux − 2.0uy + 0.6uz ) volt/m
(C) (2.0ux − 3.0uy + 1.0uz ) volt/m (D) (2.0ux − 2.0uy + 0.6uz ) volt/m
SOL 1.84 Hence (C) is correct option.
We have E1 = 2ux − 3uy + 1uz
E1t = − 3uy + uy and E1n = 2ux
Since for dielectric material at the boundary, tangential component of electric field
are equal
E1t =− 3uy + uy = E2t (x = 0 plane)
E1n = 2ux
At the boundary the for normal component of electric field are
D1n = D2n
or ε1 E1n = ε2 E2n
or 1.5εo 2ux = 2.5εo E2n
or E2n = 3 ux = 1.2ux
2.5
Thus E2 = E2t + E2n =− 3uy + uz + 1.2ux
MCQ 1.85 If the electric field intensity is given by E = (xux + yuy + zuz ) volt/m, the potential
difference between X (2, 0, 0) and Y (1, 2, 3) is
(A) + 1 volt (B) − 1 volt
(C) + 5 volt (D) + 6 volt
SOL 1.85 Hence (C) is correct option.
We have E = xux + yuy + zuz
dl = utx dx + uty dy + utz dz
#XE.dl
Y
VXY =−

#1 #2 ydyutz + #3 zdzuzt
2 0 0
= xdxutx +
0
2 2 y2 2 0
=−= x + +z G
2 1 2 2 2 3
=− 1 [22 − 12 + 02 − 22 + 02 − 32] = 5
2

MCQ 1.86 A uniform plane wave traveling in air is incident on the plane boundary between

Brought to you by: Nodia and Company Visit us at: www.nodia.co.in


PUBLISHING FOR GATE
Page 39 GATE EC 2003 www.gatehelp.com

air and another dielectric medium with εr = 4 . The reflection coefficient for the
normal incidence, is
(A) zero (B) 0.5+180c
(B) 0.333+0c (D) 0.333+180c
SOL 1.86 Hence (D) is correct option.
μ
η =
ε
Reflection coefficient
η − η1
τ = 2
η2 + η1
Substituting values for η1 and η2 we have
μ
− με
= 1 − εr = 1 −
o 0

τ = εμε o r o 4 since εr = 4
μ
εε +
o
o r ε
o
o
1 + εr 1+ 4
= − 1 = 0.333+180c
3

MCQ 1.87 If the electric field intensity associated with a uniform plane electromagnetic wave
traveling in a perfect dielectric medium is given by E (z, t) = 10 cos (2π107 t − 0.1πz)
V/m, then the velocity of the traveling wave is
(A) 3.00 # 108 m/sec (B) 2.00 # 108 m/sec
(C) 6.28 # 107 m/sec (D) 2.00 # 107 m/sec
SOL 1.87 Hence (B) is correct option.
We have E (z, t) = 10 cos (2π # 107 t − 0.1πz)
where ω = 2π # 107 t
β = 0.1π
u = ω = 2π # 10 = 2 # 108 m/s
7
Phase Velocity
β 0.1π

MCQ 1.88 A short - circuited stub is shunt connected to a transmission line as shown in
fig. If Z0 = 50 ohm, the admittance Y seen at the junction of the stub and the
transmission line is

Brought to you by: Nodia and Company Visit us at: www.nodia.co.in


PUBLISHING FOR GATE
Page 40 GATE EC 2003 www.gatehelp.com

(A) (0.01 − j0.02) mho (B) (0.02 − j0.01) mho


(C) (0.04 − j0.02) mho (D) (0.02 + j0) mho
SOL 1.88 The fig of transmission line is as shown below .
[Z + jZo tan βl]
We know that Zin = Zo L
[Zo + jZL tan βl]
For line 1, l = λ and β = 2π , ZL1 = 100Ω
2 λ
[Z + jZo tan π]
Thus Zin1 = Zo L = ZL = 100Ω
[Zo + jZL tan π]
For line 2, l = λ and β = 2π , ZL2 = 0 (short circuit)
8 λ
[0 + jZo tan π4 ]
Thus Zin2 = Zo = jZo = j50Ω
[Zo + 0]
Y = 1 + 1 = 1 + 1 = 0.01 − j0.02
Zin1 Zin2 100 j50

Hence (A) is correct option.

Brought to you by: Nodia and Company Visit us at: www.nodia.co.in


PUBLISHING FOR GATE
Page 41 GATE EC 2003 www.gatehelp.com

MCQ 1.89 A rectangular metal wave guide filled with a dielectric material of relative permittivity
εr = 4 has the inside dimensions 3.0 cm # 1.2 cm. The cut-off frequency for the
dominant mode is
(A) 2.5 GHz (B) 5.0 GHz
(C) 10.0 GHz (D) 12.5 GHz
SOL 1.89 Hence (A) is correct option.
8
u = c = 3 # 10 = 1.5 # 108
ε0 2
In rectangular waveguide the dominant mode is TE10 and
fC = v ` m j2 + ` n j2
2 a b
8
= 1.5 # 10 1 2 + 0 2 = 1.5 # 108 = 2.5 GHz
` 0.03 j ` b j
2 0.06

MCQ 1.90 Two identical antennas are placed in the θ = π/2 plane as shown in Fig. The
elements have equal amplitude excitation with 180c polarity difference, operating
at wavelength λ. The correct value of the magnitude of the far-zone resultant
electric field strength normalized with that of a single element, both computed for
φ = 0 , is

(A) 2 cos b 2πs l (B) 2 sin b 2πs l


λ λ

(C) 2 cos a πs k (D) 2 sin a πs k


λ λ

SOL 1.90 Hence (D) is correct option.


ψ
Normalized array factor = 2 cos
2
ψ = βd sin θ cos φ + δ
θ = 90c,
d = 2 s,
φ = 45c,
δ = 180c
ψ βd sin θ cos φ + δ
Now 2 cos = 2 cos ; E
2 2
= 2 cos 8 2π 2 s cos 45c + 180 B
λ. 2 2

Brought to you by: Nodia and Company Visit us at: www.nodia.co.in


PUBLISHING FOR GATE
Page 42 GATE EC 2003 www.gatehelp.com

= 2 cos 8 πs + 90cB = 2 sin ` πs j


λ λ

Answer Sheet
1. (B) 19. (D) 37. (A) 55. (B) 73. (C)
2. (C) 20. (D) 38. (A) 56. (B) 74. (C)
3. (B) 21. (D) 39. (D) 57. (A) 75. (D)
4. (C) 22. (C) 40. (C) 58. (D) 76. (D)
5. (C) 23. (B) 41. (C) 59. (C) 77. (B)
6. (D) 24. (A) 42. (A) 60. (A) 78. (A)
7. (B) 25. (C) 43. (D) 61. (D) 79. (C)
8. (A) 26. (A) 44. (B) 62. (A) 80. (D)
9. (C) 27. (C) 45. (A) 63. (C) 81. (B)
10. (D) 28. (B) 46. (A) 64. (A) 82. (A)
11. (B) 29. (C) 47. (A) 65. (D) 83. (B)
12. (D) 30. (B) 48. (C) 66. (C) 84. (C)
13. (B) 31. (A) 49. (B) 67. (D) 85. (C)
14. (C) 32. (*) 50. (B) 68. (D) 86. (D)
15. (B) 33. (*) 51. (C) 69. (D) 87. (B)
16. (D) 34. (A) 52. (D) 70 (A) 88. (A)
17. (C) 35. (C) 53. (B) 71 (B) 89. (A)
18. (B) 36. (B) 54. (A) 72 (B) 90. (D)

Brought to you by: Nodia and Company Visit us at: www.nodia.co.in


PUBLISHING FOR GATE
GATE EC
2004

Q.1 - 30 Carry One Mark Each

MCQ 1.1 Consider the network graph shown in the figure. Which one of the following is NOT
a ‘tree’ of this graph ?

(A) a (B) b
(C) c (D) d
SOL 1.1 For a tree there must not be any loop. So a, c, and d don’t have any loop. Only b
has loop.
Hence (B) is correct option.
MCQ 1.2 The equivalent inductance measured between the terminals 1 and 2 for the circuit
shown in the figure is

Brought to you by: Nodia and Company Visit us at: www.nodia.co.in


PUBLISHING FOR GATE
Page 2 GATE EC 2004 www.gatehelp.com

(A) L1 + L2 + M (B) L1 + L2 − M
(C) L1 + L2 + 2M (D)L1 + L2 − 2M
SOL 1.2 The sign of M is as per sign of L If current enters or exit the dotted terminals of
both coil. The sign of M is opposite of L If current enters in dotted terminal of a
coil and exit from the dotted terminal of other coil.
Thus Leq = L1 + L2 − 2M
Hence (D) is correct option.
MCQ 1.3 The circuit shown in the figure, with R = 13 Ω, L = 14 H and C = 3 F has input
voltage v (t) = sin 2t . The resulting current i (t) is

(A) 5 sin (2t + 53.1c) (B) 5 sin (2t − 53.1c)


(C) 25 sin (2t + 53.1c) (D) 25 sin (2t − 53.1c)
SOL 1.3 Here ω = 2 and V = 1+0c
Y = 1 + jω C + 1
R jω L
= 3 + j2 # 3 + 1 1 = 3 + j4
j2 # 4
= 5+ tan - 1 4 = 5+53.11c
3
I = V * Y = (1+0c)( 5+53.1c) = 5+53.1c
Thus i (t) = 5 sin (2t + 53.1c)
Hence (A) is correct option.
MCQ 1.4 For the circuit shown in the figure, the time constant RC = 1 ms. The input voltage
is vi (t) = 2 sin 103 t . The output voltage vo (t) is equal to

Brought to you by: Nodia and Company Visit us at: www.nodia.co.in


PUBLISHING FOR GATE
Page 3 GATE EC 2004 www.gatehelp.com

(A) sin (103 t − 45c) (B) sin (103 t + 45c)


(C) sin (103 t − 53c) (D) sin (103 t + 53c)
SOL 1.4 Hence (A) is correct option.
vi (t) = 2 sin 103 t
Here ω = 103 rad and Vi = 2 +0c
1
jω C 1
Now V0 = .Vt = V
R+ 1 1 + j ωCR i
jω C
= 1 2 +0c
1 + j # 103 # 10 - 3
= 1 − 45c
v0 (t) = sin (103 t − 45c)
MCQ 1.5 For the R − L circuit shown in the figure, the input voltage vi (t) = u (t). The
current i (t) is

SOL 1.5 Hence (C) is correct option.


Input voltage vi (t) = u (t)
Taking laplace transform Vi (s) = 1
s
Brought to you by: Nodia and Company Visit us at: www.nodia.co.in
PUBLISHING FOR GATE
Page 4 GATE EC 2004 www.gatehelp.com

Impedance Z (s) = s + 2
V (s) 1
I (s) = i =
s + 2 s (s + 2)
or I (s) = 1 ; 1 − 1 E
2 s s+2
Taking inverse laplace transform
i (t) = 1 (1 − e−2t) u (t)
2
At t = 0 , i (t) = 0
At t = 12 , i (t) = 0.31
At t = 3 , i (t) = 0.5
Graph (C) satisfies all these conditions.
MCQ 1.6 The impurity commonly used for realizing the base region of a silicon n − p − n
transistor is
(A) Gallium (B) Indium
(C) Boron (D) Phosphorus
SOL 1.6 Trivalent impurities are used for making p type semiconductor. Boron is trivalent.
Hence option (C) is correct
MCQ 1.7 If for a silicon npn transistor, the base-to-emitter voltage (VBE ) is 0.7 V and the
collector-to-base voltage (VCB) is 0.2 V, then the transistor is operating in the
(A) normal active mode (B) saturation mode
(C) inverse active mode (D) cutoff mode
SOL 1.7 Here emitter base junction is forward biased and base collector junction is reversed
biased. Thus transistor is operating in normal active region.
Hence option (A) is correct.
MCQ 1.8 Consider the following statements S1 and S2.
S1 : The β of a bipolar transistor reduces if the base width is increased.
S2 : The β of a bipolar transistor increases if the dopoing concentration in the base
is increased.
Which remarks of the following is correct ?
(A) S1 is FALSE and S2 is TRUE
(B) Both S1 and S2 are TRUE
(C) Both S1 and S2 are FALSE
(D) S1 is TRUE and S2 is FALSE
SOL 1.8 Hence option (D) is correct.
We have β = α
1−α

Brought to you by: Nodia and Company Visit us at: www.nodia.co.in


PUBLISHING FOR GATE
Page 5 GATE EC 2004 www.gatehelp.com

Thus α -" β -
α ." β .
If the base width increases, recombination of carrier in base region increases and
α decreases & hence β decreases. If doping in base region increases, recombination
of carrier in base increases and α decreases thereby decreasing β . Thus S1 is true
and S2 is false.
MCQ 1.9 An ideal op-amp is an ideal
(A) voltage controlled current source (B) voltage controlled voltage source
(C) current controlled current source (D) current controlled voltage source
SOL 1.9 An ideal OPAMP is an ideal voltage controlled voltage source.
Hence (B) is correct option.
MCQ 1.10 Voltage series feedback (also called series-shunt feedback) results in
(A) increase in both input and output impedances
(B) decrease in both input and output impedances
(C) increase in input impedance and decrease in output impedance
(D) decrease in input impedance and increase in output impedance
SOL 1.10 In voltage series feed back amplifier, input impedance increases by factor (1 + Aβ)
and output impedance decreases by the factor (1 + Aβ).
Rif = Ri (1 + Aβ)
Rof = Ro
(1 + Aβ)
Hence (C) is correct option.
MCQ 1.11 The circuit in the figure is a

(A) low-pass filter (B) high-pass filter


(C) band-pass filter (D) band-reject filter
SOL 1.11 This is a Low pass filter, because
At ω = 3 V0 = 0
Vin
and at ω = 0 V0 = 1
Vin
Hence (A) is correct option.

Brought to you by: Nodia and Company Visit us at: www.nodia.co.in


PUBLISHING FOR GATE
Page 6 GATE EC 2004 www.gatehelp.com

MCQ 1.12 Assuming VCEsat = 0.2 V and β = 50 , the minimum base current (IB) required to
drive the transistor in the figure to saturation is

(A) 56 μA (B) 140 mA


(C) 60 mA (D) 3 mA
SOL 1.12 Applying KVL we get
VCC − IC RC − VCE = 0
or IC = VCC − VCE = 3 − 0.2 = 2.8 mA
RC 1k

Now IB = IC = 2.8m = 56 μA
β 50
Hence option (A) is correct.
MCQ 1.13 A master - slave flip flop has the characteristic that
(A) change in the output immediately reflected in the output
(B) change in the output occurs when the state of the master is affected
(C) change in the output occurs when the state of the slave is affected
(D) both the master and the slave states are affected at the same time
SOL 1.13 A master slave D-flip flop is shown in the figure.

In the circuit we can see that output of flip-flop call be triggered only by transition
of clock from 1 to 0 or when state of slave latch is affected.
Hence (C) is correct answer.
MCQ 1.14 The range of signed decimal numbers that can be represented by 6-bits 1’s
complement number is
(A) -31 to +31 (B) -63 to +63
(C) -64 to +63 (D) -32 to +31

Brought to you by: Nodia and Company Visit us at: www.nodia.co.in


PUBLISHING FOR GATE
Page 7 GATE EC 2004 www.gatehelp.com

SOL 1.14 The range of signed decimal numbers that can be represented by n − bits 1’s
complement number is − (2n - 1 − 1) to + (2n - 1 − 1).
Thus for n = 6 we have
Range =− (26 - 1 − 1) to + (26 - 1 − 1)
=− 31 to + 31
Hence (A) is correct answer.
MCQ 1.15 A digital system is required to amplify a binary-encoded audio signal. The user
should be able to control the gain of the amplifier from minimum to a maximum
in 100 increments. The minimum number of bits required to encode, in straight
binary, is
(A) 8 (B) 6
(C) 5 (D) 7
SOL 1.15 The minimum number of bit require to encode 100 increment is
2n $ 100
or n $7
Hence (D) is correct answer.
MCQ 1.16 Choose the correct one from among the alternatives A, B, C, D after matching an
item from Group 1 most appropriate item in Group 2.
Group 1 Group 2
P. Shift register 1. Frequency division
Q. Counter 2. Addressing in memory chips
R. Decoder 3. Serial to parallel data conversion
(A) P − 3, Q − 2, R − 1 (B) P − 3, Q − 1, R − 2
(C) P − 2, Q − 1, R − 3 (D) P − 1, Q − 2, R − 2
SOL 1.16 Shift Register " Serial to parallel data conversion
Counter " Frequency division
Decoder " Addressing in memory chips.
Hence (B) is correct answer.
MCQ 1.17 The figure the internal schematic of a TTL AND-OR-OR-Invert (AOI) gate. For
the inputs shown in the figure, the output Y is

(A) 0 (B) 1
(C) AB (D) AB
SOL 1.17 For the TTL family if terminal is floating, then it is at logic 1.
Thus Y = (AB + 1) = AB .0 = 0

Brought to you by: Nodia and Company Visit us at: www.nodia.co.in


PUBLISHING FOR GATE
Page 8 GATE EC 2004 www.gatehelp.com

Hence (A) is correct answer.


MCQ 1.18 Given figure is the voltage transfer characteristic of

(A) an NOMS inverter with enhancement mode transistor as load


(B) an NMOS inverter with depletion mode transistor as load
(C) a CMOS inverter
(D) a BJT inverter
SOL 1.18 Hence option (C) is correct
MCQ 1.19 The impulse response h [n] of a linear time-invariant system is given by
h [n] = u [n + 3] + u [n − 2) − 2n [n − 7] where u [n] is the unit step sequence. The
above system is
(A) stable but not causal (B) stable and causal
(C) causal but unstable (D) unstable and not causal
3
SOL 1.19 A system is stable if / h (n) < 3. The plot of given h (n) is
n =− 3

3 6
Thus / h (n) = /
n =− 3
h (n)
n =− 3

= 1+1+1+1+2+2+2+2+2
= 15 < 3
Hence system is stable but h (n) ! 0 for n < 0 . Thus it is not causal.
Hence (A) is correct answer.
MCQ 1.20 The distribution function Fx (x) of a random variable x is shown in the figure. The
probability that X = 1 is

Brought to you by: Nodia and Company Visit us at: www.nodia.co.in


PUBLISHING FOR GATE
Page 9 GATE EC 2004 www.gatehelp.com

(A) zero (B) 0.25


(C) 0.55 (D) 0.30
SOL 1.20 Hence (D) is correct option.
F (x1 # X < x2) = p (X = x2) − P (X = x1)
or P (X = 1) = P (X = 1+) − P (X = 1 -)
= 0.55 − 0.25 = 0.30
MCQ 1.21 The z -transform of a system is H (z) = z −z0.2 . If the ROC is z < 0.2 , then the
impulse response of the system is
(A) (0.2) n u [n] (B) (0.2) n u [− n − 1]
(C) − (0.2) n u [n] (D) − (0.2) n u [− n − 1]
SOL 1.21 Hence (D) is correct answer.
H (z) = z z < 0.2
z − 0.2
We know that
− an u [− n − 1] * 1 z <a
1 − az−1
Thus h [n] =− (0.2) n u [− n − 1]
MCQ 1.22 The Fourier transform of a conjugate symmetric function is always
(A) imaginary (B) conjugate anti-symmetric
(C) real (D) conjugate symmetric
SOL 1.22 The Fourier transform of a conjugate symmetrical function is always real.
Hence (C) is correct answer.
MCQ 1.23 The gain margin for the system with open-loop transfer function
2 (1 + s)
G (s) H (s) = , is
s2
(A) 3 (B) 0
(C) 1 (D) − 3
SOL 1.23 The open loop transfer function is
2 (1 + s)
G (s) H (s) =
s2
Substituting s = jω we have
2 (1 + jω)
G (jω) H (jω) = ...(1)
− ω2

Brought to you by: Nodia and Company Visit us at: www.nodia.co.in


PUBLISHING FOR GATE
Page 10 GATE EC 2004 www.gatehelp.com

+G (jω) H (jω) =− 180c + tan−1 ω


The frequency at which phase becomes − 180c, is called phase crossover frequency.
Thus − 180 =− 180c + tan−1 ωφ
or tan−1 ωφ = 0
or ωφ = 0
The gain at ωφ = 0 is
G (jω) H (jω) = 2 1 + ω2 = 3
ω2
Thus gain margin is = 1 = 0 and in dB this is − 3 .
3
Hence (D) is correct option

MCQ 1.24 Given G (s) H (s) = K .The point of intersection of the asymptotes of
s (s + 1)( s + 3)
the root loci with the real axis is
(A) − 4 (B) 1.33
(C) − 1.33 (D) 4
SOL 1.24 Centroid is the point where all asymptotes intersects.
ΣReal of Open Loop Pole − ΣReal Part of Open Loop Pole
σ =
ΣNo.of Open Loop Pole − ΣNo.of Open Loop zero
= − 1 − 3 =− 1.33
3
Hence (C) is correct option.
MCQ 1.25 In a PCM system, if the code word length is increased from 6 to 8 bits, the signal
to quantization noise ratio improves by the factor
(A) 8 (B) 12
6
(C) 16 (D) 8
SOL 1.25 When word length is 6
` N jN = 6 = 2 = 2
S 2#6 12

When word length is 8


` N jN = 8 = 2 = 2
S 2#8 16

^ N hN = 8
S 16
Now = 212 = 2 4 = 16
^ N hN = 6
S
2
Thus it improves by a factor of 16.
Hence (C) is correct option.
MCQ 1.26 An AM signal is detected using an envelop detector. The carrier frequency and
modulating signal frequency are 1 MHz and 2 kHz respectively. An appropriate
value for the time constant of the envelop detector is
(A) 500μ sec (B) 20μ sec

Brought to you by: Nodia and Company Visit us at: www.nodia.co.in


PUBLISHING FOR GATE
Page 11 GATE EC 2004 www.gatehelp.com

(C) 0.2μ sec (D) 1μ sec


SOL 1.26 Hence (B) is correct option.
Carrier frequency fc = 1 # 106 Hz
Modulating frequency
fm = 2 # 103 Hz
For an envelope detector
2πfc > 1 > 2πfm
Rc
1 < RC < 1
2πfc 2πfm
1 < RC < 1
2πfc 2πfm
1 < RC < 1
2π10 6
2 # 103
1.59 # 10 - 7 < RC < 7.96 # 10 - 5
so, 20 μsec sec best lies in this interval.
MCQ 1.27 An AM signal and a narrow-band FM signal with identical carriers, modulating
signals and modulation indices of 0.1 are added together. The resultant signal can
be closely approximated by
(A) broadband FM (B) SSB with carrier
(C) DSB-SC (D) SSB without carrier
SOL 1.27 Hence (B) is correct option.
SAM (t) = Ac [1 + 0.1 cos ωm t] cos ωm t
sNBFM (t) = Ac cos [ωc t + 0.1 sin ωm t]
s (t) = SAM (t) + SNB fm (t)
= Ac [1 + 0.1 cos ωm t] cos ωc t + Ac cos (ωc t + 0.1 sin ωm t)
= Ac cos ωc t + Ac 0.1 cos ωm t cos ωc t
+ Ac cos ωc t cos (0.1 sin ωm t) − Ac sin ωc t. sin (0.1 sin ωm t)
As 0.1 sin ωm t ,+ 0.1 to − 0.1
so cos (0.1 sin ωm t) . 1
As when θ is small cos θ . 1 and sin θ , θ, thus
sin (0.1 sin ωm t) = 0.1 sincos ωc t cos ωm t + Ac cos ωc t − Ac 0.1 sin ωm t sin ωc t
= 2Ac cos ωc t + 0.1Ac cos (ωc + ωm) t
1 44 2 44 3 1 4444 4 2 4444 43
cosec USB
Thus it is SSB with carrier.
MCQ 1.28 In the output of a DM speech encoder, the consecutive pulses are of opposite
polarity during time interval t1 # t # t2 . This indicates that during this interval
(A) the input to the modulator is essentially constant
(B) the modulator is going through slope overload

Brought to you by: Nodia and Company Visit us at: www.nodia.co.in


PUBLISHING FOR GATE
Page 12 GATE EC 2004 www.gatehelp.com

(C) the accumulator is in saturation


(D) the speech signal is being sampled at the Nyquist rate
SOL 1.28 Consecutive pulses are of same polarity when modulator is in slope overload.
Consecutive pulses are of opposite polarity when the input is constant.
Hence (A) is correct option.
MCQ 1.29 The phase velocity of an electromagnetic wave propagating in a hollow metallic
rectangular waveguide in the TE10 mode is
(A) equal to its group velocity
(B) less than the velocity of light in free space
(C) equal to the velocity of light in free space
(D) greater than the velocity of light in free space
SOL 1.29 We know that vp > c > vg .
Hence (D) is correct option.
MCQ 1.30 Consider a lossless antenna with a directive gain of + 6 dB. If 1 mW of power is fed
to it the total power radiated by the antenna will be
(A) 4 mW (B) 1 mW
(C) 7 mW (D) 1/4 mW
SOL 1.30 Hence (A) is correct option.
4πU (θ, φ)
We have GD (θ, φ) =
Prad
For lossless antenna
Prad = Pin
Here we have Prad = Pin = 1 mW
and 10 log GD (θ, φ) = 6 dB
or GD (θ, φ) = 3.98
Thus the total power radiated by antenna is
4πU (θ, φ) = Prad GD (θ, φ) = 1 m # 3.98 = 3.98 mW

Q.31 - 90 Carry Two Marks Each

MCQ 1.31 For the lattice shown in the figure, Za = j2 Ω and Zb = 2 Ω . The values of the open
z11 z12
circuit impedance parameters 6 z @ = =
z21 z22 G
are

Brought to you by: Nodia and Company Visit us at: www.nodia.co.in


PUBLISHING FOR GATE
Page 13 GATE EC 2004 www.gatehelp.com

1−j 1+j 1−j 1+j


(A) =
1 + j 1 + jG
(B) =
−1 + j 1 − j G
1+j 1+j 1 + j −1 + j
(C) =
1 − j 1 − jG
(D) =
−1 + j 1 + j G
SOL 1.31 We know that
V1 = z11 I1 + z12 I2
V2 = z11 I1 + z22 I2
where z11 = V1
I1 I = 0
2

z21 = V2
I1 I = 0
1

Consider the given lattice network, when I2 = 0 . There is two similar path in the
circuit for the current I1. So I = 1 I1
2

For z11 applying KVL at input port we get


V1 = I (Za + Zb)
Thus V1 = 1 I1 (Za + Zb)
2
z11 = 1 (Za + Zb)
2
For Z21 applying KVL at output port we get
V2 = Za I1 − Zb I1
2 2
Thus V2 = 1 I1 (Za − Zb)
2
z21 = 1 (Za − Zb)
2
For this circuit z11 = z22 and z12 = z21. Thus
R V
S Za + Zb Za − Zb W
z11 z12
=z z G = SS Za − Zb Za + Zb WW
2 2
21 22
S 2 2 W
T X
Here Za = 2j and Zb = 2Ω
z11 z12 1+j j−1
Thus = G ==
z21 z22 j − 1 1 + jG
Hence (D) is correct option.

Brought to you by: Nodia and Company Visit us at: www.nodia.co.in


PUBLISHING FOR GATE
Page 14 GATE EC 2004 www.gatehelp.com

MCQ 1.32 The circuit shown in the figure has initial current iL (0−) = 1 A through the inductor
and an initial voltage vC (0−) =− 1 V across the capacitor. For input v (t) = u (t),
the Laplace transform of the current i (t) for t $ 0 is

(A) s (B) s+2


s +s+1
2
s +s+1 2

(C) 2 s − 2 (D) 2 1
s +s+1 s +s+1
SOL 1.32 Applying KVL,
Ldi (t) 1
#0
3
v (t) = Ri (t) + + i (t) dt
dt C
Taking L.T. on both sides,
I (s) vc (0+)
V (s) = RI (s) + LsI (s) − Li (0+) + +
sC sC
v (t) = u (t) thus V (s) = 1
s
Hence 1 = I (s) + sI (s) − 1 + I (s) − 1
s s s
2 + 1 = I (s) 6s2 + s + 1@
s s
or I (s) = 2 s + 2
s +s+1
Hence (B) is correct option.
MCQ 1.33 Consider the Bode magnitude plot shown in the fig. The transfer function H (s) is

(s + 10) 10 (s + 1)
(A) (B)
(s + 1)( s + 100) (s + 10)( s + 100)
102 (s + 1) 103 (s + 100)
(C) (D)
(s + 10)( s + 100) (s + 1)( s + 10)

Brought to you by: Nodia and Company Visit us at: www.nodia.co.in


PUBLISHING FOR GATE
Page 15 GATE EC 2004 www.gatehelp.com

SOL 1.33 The given bode plot is shown below

At ω = 1 change in slope is +20 dB " 1 zero at ω = 1


At ω = 10 change in slope is − 20 dB " 1 poles at ω = 10
At ω = 100 change in slope is − 20 dB " 1 poles at ω = 100
K (s + 1)
Thus T (s) = s
( 10 + 1)( 100
s
+ 1)
Now 20 log10 K =− 20 " K = 0.1
0.1 (s + 1) 100 (s + 1)
Thus T (s) = s =
( 10 + 1)( 100 + 1) (s + 10)( s + 100)
s

Hence (C) is correct option.


Vo (s)
MCQ 1.34 The transfer function H (s) = of an RLC circuit is given by
Vi (s)
H (s) = 106
s + 20s + 106
2

The Quality factor (Q-factor) of this circuit is


(A) 25 (B) 50
(C) 100 (D) 5000
SOL 1.34 Characteristics equation is
s2 + 20s + 106 = 0
Comparing with s2 + 2ξωn s + ωn2 = 0 we have
ωn = 106 = 103
2ξω = 20
Thus 2ξ = 203 = 0.02
10
Now Q = 1 = 1 = 50
2ξ 0.02
Hence (B) is correct option.
MCQ 1.35 For the circuit shown in the figure, the initial conditions are zero. Its transfer
V (s)
function H (s) = c is
Vi (s)

Brought to you by: Nodia and Company Visit us at: www.nodia.co.in


PUBLISHING FOR GATE
Page 16 GATE EC 2004 www.gatehelp.com

(A) 1 (B) 106


s + 106 s + 106
2
s2 + 103 s + 106

(C) 103 (D) 106


s2 + 103 s + 106 s2 + 106 s + 106

SOL 1.35 Hence (D) is correct option.


V (s)
H (s) = 0
Vi (s)
1
= sC = 2 1
R + sL + 1 s LC + sCR + 1
sC
= 2 −2 1
s (10 # 10 ) + s (10−4 # 10 4) + 1
−4

= −6 2 1 = 2 106
10 s + s + 1 s + 106 s + 106

d 2y dy
MCQ 1.36 A system described by the following differential equation 2 + 3 + 2y = x (t) is
dt dt
initially at rest. For input x (t) = 2u (t), the output y (t) is
(A) (1 − 2e−t + e−2t) u (t) (B) (1 + 2e−t − 2e−2t) u (t)
(C) (0.5 + e−t + 1.5e−2t) u (t) (D) (0.5 + 2e−t + 2e−2t) u (t)
SOL 1.36 Hence Correct Option is (A)

2 + 3 dt + 2y = x ^ t h
d2y dy
Given,
dt
Taking Laplace Transformation both sides, we have
6s + 3s + 2@Y ^s h = X ^s h = s
2 2

Y ^s h = 2 =1− 2 + 1
s ^s + 1h^s + 2h s s + 1 s + 2
or

Increasing Laplace transformation gives,


y ^ t h = ^1 − 2e−t + e−2t h u ^ t h
MCQ 1.37 Consider the following statements S1 and S2
S1 : At the resonant frequency the impedance of a series RLC circuit is zero.
S2 : In a parallel GLC circuit, increasing the conductance G results in increase in
its Q factor.

Brought to you by: Nodia and Company Visit us at: www.nodia.co.in


PUBLISHING FOR GATE
Page 17 GATE EC 2004 www.gatehelp.com

Which one of the following is correct?


(A) S1 is FALSE and S2 is TRUE
(B) Both S1 and S2 are TRUE
(C) S1 is TRUE and S2 is FALSE
(D) Both S1 and S2 are FALSE
SOL 1.37 Impedance of series RLC circuit at resonant frequency is minimum, not zero.
Actually imaginary part is zero.
Z = R + j ` ωL − 1 j
ωC
At resonance ωL − 1 = 0 and Z = R that is purely resistive. Thus S1 is false
ωC
Now quality factor Q =R C
L
Since G = 1 , Q = 1 C
R G L
If G - then Q . provided C and L are constant. Thus S2 is also false.
Hence (D) is correct option.
MCQ 1.38 In an abrupt p − n junction, the doping concentrations on the p −side and n -side
are NA = 9 # 1016 /cm 3 respectively. The p − n junction is reverse biased and the
total depletion width is 3 μm. The depletion width on the p −side is
(A) 2.7 μm (B) 0.3 μm
(C) 2.25 μm (D) 0.75 μm
SOL 1.38 We know that
Wp NA = Wn ND
3 μ # 1016
or Wp = Wn # ND = = 0.3 μ m
NA 9 # 1016
Hence option (B) is correct.
MCQ 1.39 The resistivity of a uniformly doped n −type silicon sample is 0.5Ω - mc. If the
electron mobility (μn) is 1250 cm 2 /V-sec and the charge of an electron is 1.6 # 10 - 19
Coulomb, the donor impurity concentration (ND) in the sample is
(A) 2 # 1016 /cm 3 (B) 1 # 1016 /cm 3
(C) 2.5 # 1015 /cm 3 (D) 5 # 1015 /cm 3
SOL 1.39 Hence option (B) is correct.
Conductivity σ = nqun
or resistivity ρ = 1 = 1
σ nqμn
Thus n = 1 = 1 = 1016 /cm 3
qρμn - 19
1.6 # 10 # 0.5 # 1250
For n type semiconductor n = ND

Brought to you by: Nodia and Company Visit us at: www.nodia.co.in


PUBLISHING FOR GATE
Page 18 GATE EC 2004 www.gatehelp.com

MCQ 1.40 Consider an abrupt p − n junction. Let Vbi be the built-in potential of this junction
and VR be the applied reverse bias. If the junction capacitance (Cj ) is 1 pF for
Vbi + VR = 1 V, then for Vbi + VR = 4 V, Cj will be
(A) 4 pF (B) 2 pF
(C) 0.25 pF (D) 0.5 pF
SOL 1.40 We know that
eεS NA ND
1

Cj = ;
2 (Vbi + VR)( NA + ND) E
2

Thus Cj \ 1
(Vbi + VR)
C j2 (Vbi + VR) 1 1 =1
Now = =
C j1 (Vbi + VR) 2 4 2
Cj1
or Cj2 = = 1 = 0.5 pF
2 2
Hence option (D) is correct.
MCQ 1.41 Consider the following statements Sq and S2.
S1 : The threshold voltage (VT ) of MOS capacitor decreases with increase in gate
oxide thickness.
S2 : The threshold voltage (VT ) of a MOS capacitor decreases with increase in
substrate doping concentration.
Which Marks of the following is correct ?
(A) S1 is FALSE and S2 is TRUE
(B) Both S1 and S2 are TRUE
(C) Both S1 and S2 are FALSE
(D) S1 is TRUE and S2 is FALSE
SOL 1.41 Increase in gate oxide thickness makes difficult to induce charges in channel. Thus
VT increases if we increases gate oxide thickness. Hence S1 is false.
Increase in substrate doping concentration require more gate voltage because
initially induce charges will get combine in substrate. Thus VT increases if we
increase substrate doping concentration. Hence S2 is false.
Hence option (C) is correct.
MCQ 1.42 The drain of an n-channel MOSFET is shorted to the gate so that VGS = VDS . The
threshold voltage (VT ) of the MOSFET is 1 V. If the drain current (ID) is 1 mA for
VGS = 2 V, then for VGS = 3 V, ID is
(A) 2 mA (B) 3 mA
(C) 9 mA (D) 4 mA

Brought to you by: Nodia and Company Visit us at: www.nodia.co.in


PUBLISHING FOR GATE
Page 19 GATE EC 2004 www.gatehelp.com

SOL 1.42 We know that


ID = K (VGS − VT ) 2

IDS = (VGS2 − VT )
2
Thus
IDI (VGS1 − VT ) 2
Substituting the values we have
ID2 = (3 − 1) = 4
2

ID1 (2 − 1) 2
or ID2 = 4IDI = 4 mA
Hence option (D) is correct.
MCQ 1.43 The longest wavelength that can be absorbed by silicon, which has the bandgap
of 1.12 eV, is 1.1 μm. If the longest wavelength that can be absorbed by another
material is 0.87 μm, then bandgap of this material is
(A) 1.416 A/cm 2 (B) 0.886 eV
(C) 0.854 eV (D) 0.706 eV
SOL 1.43 Hence option (A) is correct.
Eg \ 1
λ
= λ1 = 1.1
Eg2
Thus
Eg1 λ2 0.87
or Eg2 = 1.1 # 1.12 = 1.416 eV
0.87

MCQ 1.44 The neutral base width of a bipolar transistor, biased in the active region, is 0.5 μ
m. The maximum electron concentration and the diffusion constant in the base are
1014 / cm 3 and Dn = 25 cm 2 /sec respectively. Assuming negligible recombination in
the base, the collector current density is (the electron charge is 1.6 # 10 - 19 Coulomb)
(A) 800 A/cm 2 (B) 8 A/cm 2
(C) 200 A/cm 2 (D) 2 A/cm 2
SOL 1.44 Concentration gradient
dn = 1014 = 2 # 1018
dx -4
0.5 # 10
q = 1.6 # 10 - 19 C
Dn = 25
dn = 1014
dx 0.5 # 10 - 4
JC = qDn dn
dx
= 1.6 # 10 - 19 # 25 # 2 # 1018 = 8 A/cm 2

Hence option (B) is correct.

Brought to you by: Nodia and Company Visit us at: www.nodia.co.in


PUBLISHING FOR GATE
Page 20 GATE EC 2004 www.gatehelp.com

MCQ 1.45 Assume that the β of transistor is extremely large and VBE = 0.7V, IC and VCE in
the circuit shown in the figure

(A) IC = 1 mA, VCE = 4.7 V (B) IC = 0.5 mA, VCE = 3.75 V


(C) IC = 1 mA, VCE = 2.5 V (D) IC = 0.5 mA, VCE = 3.9 V
SOL 1.45 The thevenin equivalent is shown below

VT = R1 V = 1
#5 = 1 V
R1 + R2 C 4+1
Since β is large is large, IC . IE , IB . 0 and
IE = VT − VBE = 1 − 0.7 = 3 mA
RE 300
Now VCE = 5 − 2.2kIC − 300IE
= 5 − 2.2k # 1m − 300 # 1m
= 2.5 V
Hence (C) is correct option
MCQ 1.46 A bipolar transistor is operating in the active region with a collector current of 1
mA. Assuming that the β of the transistor is 100 and the thermal voltage (VT ) is
25 mV, the transconductance (gm) and the input resistance (rπ) of the transistor in
the common emitter configuration, are
(A) gm = 25 mA/V and rπ = 15.625 kΩ
(B) gm = 40 mA/V and rπ = 4.0 kΩ
(C) gm = 25 mA/V and rπ = 2.5 k Ω
(D) gm = 40 mA/V and rπ = 2.5 kΩ

Brought to you by: Nodia and Company Visit us at: www.nodia.co.in


PUBLISHING FOR GATE
Page 21 GATE EC 2004 www.gatehelp.com

SOL 1.46 When IC >> ICO


IC
gm = = 1mA = 0.04 = 40 mA/V
VT 25mV
β
rπ = = 100 - 3 = 2.5 kΩ
gm 40 # 10
Hence (D) is correct option.
MCQ 1.47 The value of C required for sinusoidal oscillations of frequency 1 kHz in the circuit
of the figure is

(A) 1 μF (B) 2π μF

(C) 1 μF (D) 2π 6 μF]


2π 6
SOL 1.47 The given circuit is wein bridge oscillator. The frequency of oscillation is
2πf = 1
RC
or C = 1 = 1 = 1 μ
2πRf 2π # 10 # 10
3 3 2π
Hence (A) is correct option.
MCQ 1.48 In the op-amp circuit given in the figure, the load current iL is

(A) − Vs (B) Vs
R2 R2

Brought to you by: Nodia and Company Visit us at: www.nodia.co.in


PUBLISHING FOR GATE
Page 22 GATE EC 2004 www.gatehelp.com

(C) − Vs (D) Vs
RL R1
SOL 1.48 The circuit is as shown below

We know that for ideal OPAMP


V- = V+
Applying KCL at inverting terminal
V- − Vs + V- − V0 = 0
R1 R1
or 2V- − Vo = Vs ...(1)
Applying KCL at non-inverting terminal
V+ V − Vo
+ IL + + =0
R2 R2
or 2V+ − Vo + IL R2 = 0 ...(2)
Since V- = V+ , from (1) and (2) we have
Vs + IL R2 = 0
or IL =− Vs
R2
Hence (A) is correct option.
MCQ 1.49 In the voltage regulator shown in the figure, the load current can vary from 100
mA to 500 mA. Assuming that the Zener diode is ideal (i.e., the Zener knee current
is negligibly small and Zener resistance is zero in the breakdown region), the value
of R is

(A) 7 Ω (B) 70 Ω
(C) 70 Ω (D) 14 Ω
3
SOL 1.49 If IZ is negligible the load current is

Brought to you by: Nodia and Company Visit us at: www.nodia.co.in


PUBLISHING FOR GATE
Page 23 GATE EC 2004 www.gatehelp.com

12 − Vz = I
L
R
as per given condition
100 mA # 12 − VZ # 500 mA
R
At IL = 100 mA 12 − 5 = 100 mA VZ = 5 V
R
or R = 70Ω
At IL = 500 mA 12 − 5 = 500 mA VZ = 5 V
R
or R = 14 Ω
Thus taking minimum we get
R = 14 Ω
Hence (D) is correct option.
MCQ 1.50 In a full-wave rectifier using two ideal diodes, Vdc and Vm are the dc and peak values
of the voltage respectively across a resistive load. If PIV is the peak inverse voltage
of the diode, then the appropriate relationships for this rectifier are
(A) Vdc = Vm , PIV = 2Vm (B) Idc = 2 Vm , PIV = 2Vm
π π

(C) Vdc = 2 Vm , PIV = Vm (D) Vdc Vm , PIV = Vm


π π
SOL 1.50 Hence (B) is correct option.
MCQ 1.51 The minimum number of 2- to -1 multiplexers required to realize a 4- to -1
multiplexers is
(A) 1 (B) 2
(C) 3 (D) 4

SOL 1.51 Number of MUX is 4 = 2 and 2 = 1. Thus the total number 3 multiplexers is
3 2
required.
Hence (C) is correct answer.
MCQ 1.52 The Boolean expression AC + BC is equivalent to
(A) AC + BC + AC (B) BC + AC + BC + ACB
(C) AC + BC + BC + ABC (D) ABC + ABC + ABC + ABC
SOL 1.52 Hence (D) is correct answer.
AC + BC = AC1 + BC 1
= AC (B + B ) + BC (A + A)
= ACB + ACB + BC A + BC A
MCQ 1.53 11001, 1001, 111001 correspond to the 2’s complement representation of which one
of the following sets of number
(A) 25,9, and 57 respectively (B) -6, -6, and -6 respectively
Brought to you by: Nodia and Company Visit us at: www.nodia.co.in
PUBLISHING FOR GATE
Page 24 GATE EC 2004 www.gatehelp.com

(C) -7, -7 and -7 respectively (D) -25, -9 and -57 respectively


SOL 1.53 Hence (C) is correct answer.
11001 1001 111001
00110 0110 000110
+1 +1 +1
00111 0111 000111
7 7 7
Thus 2’s complement of 11001, 1001 and 111001 is 7. So the number given in the
question are 2’s complement correspond to -7.

MCQ 1.54 The 8255 Programmable Peripheral Interface is used as described below.
(i) An A/D converter is interface to a microprocessor through an 8255.
The conversion is initiated by a signal from the 8255 on Port C. A signal on Port
C causes data to be stobed into Port A.
(ii) Two computers exchange data using a pair of 8255s. Port A works as a
bidirectional data port supported by appropriate handshaking signals.
The appropriate modes of operation of the 8255 for (i) and (ii) would be
(A) Mode 0 for (i) and Mode 1 for (ii)
(B) Mode 1 for (i) and Mode 2 for (ii)
(C) Mode for (i) and Mode 0 for (ii)
(D) Mode 2 for (i) and Mode 1 for (ii)
SOL 1.54 For 8255, various modes are described as following.
Mode 1 : Input or output with hand shake
In this mode following actions are executed
1. Two port (A & B) function as 8 - bit input output ports.
2. Each port uses three lines from C as a hand shake signal
3. Input & output data are latched.
Form (ii) the mode is 1.
Mode 2 : Bi-directional data transfer
This mode is used to transfer data between two computer. In this mode port A can
be configured as bidirectional port. Port A uses five signal from port C as hand
shake signal.
For (1), mode is 2
Hence (D) is correct answer.
MCQ 1.55 The number of memory cycles required to execute the following 8085 instructions
(i) LDA 3000 H
(ii) LXI D, FOF1H
would be

Brought to you by: Nodia and Company Visit us at: www.nodia.co.in


PUBLISHING FOR GATE
Page 25 GATE EC 2004 www.gatehelp.com

(A) 2 for (i) and 2 for (ii) (B) 4 for (i) and 3 for (ii)
(C) 3 for (i) and 3 for (ii) (D) 3 for (i) and 4 for (ii)
SOL 1.55 LDA 16 bit & Load accumulator directly this instruction copies data byte from
memory location (specified within the instruction) the accumulator.
It takes 4 memory cycle-as following.
1. in instruction fetch
2. in reading 16 bit address
1. in copying data from memory to accumulator
LXI D, (F0F1) 4 & It copies 16 bit data into register pair D and E.
It takes 3 memory cycles.
Hence (B) is correct answer.
MCQ 1.56 In the modulo-6 ripple counter shown in figure, the output of the 2- input gate is
used to clear the J-K flip-flop
The 2-input gate is

(A) a NAND gate (B) a NOR gate


(C) an OR gate (D) a AND gare
SOL 1.56 In the modulo - 6 ripple counter at the end of sixth pulse (i.e. after 101 or at 110)
all states must be cleared. Thus when CB is 11 the all states must be cleared. The
input to 2-input gate is C and B and the desired output should be low since the
CLEAR is active low
Thus when C and B are 0, 0, then output must be 0. In all other case the output
must be 1. OR gate can implement this functions.
Hence (C) is correct answer.
MCQ 1.57 Consider the sequence of 8085 instructions given below
LXI H, 9258
MOV A, M
CMA
MOV M, A
Which one of the following is performed by this sequence ?
(A) Contents of location 9258 are moved to the accumulator

Brought to you by: Nodia and Company Visit us at: www.nodia.co.in


PUBLISHING FOR GATE
Page 26 GATE EC 2004 www.gatehelp.com

(B) Contents of location 9258 are compared with the contents of the accumulator
(C) Contents of location 8529 are complemented and stored in location 8529
(D) Contents of location 5892 are complemented and stored in location 5892
SOL 1.57 Hence (A) is correct answer.
LXI H, 9258H ; 9258H " HL
MOV A, M ; (9258H) " A
CMa ; A"A
MOV M, A ; A"M
This program complement the data of memory location 9258H.
MCQ 1.58 A Boolean function f of two variables x and y is defined as follows :
f (0, 0) = f (0, 1) = f (1, 1) = 1; f (1, 0) = 0
Assuming complements of x and y are not available, a minimum cost solution for
realizing f using only 2-input NOR gates and 2- input OR gates (each having unit
cost) would have a total cost of
(A) 1 unit (B) 4 unit
(C) 3 unit (D) 2 unit
SOL 1.58 Hence (D) is correct answer.
We have f (x, y) = xy + xy + xy = x (y + y) + xy = x + xy
or f (x, y) = x + y
Here compliments are not available, so to get x we use NOR gate. Thus desired
circuit require 1 unit OR and 1 unit NOR gate giving total cost 2 unit.
MCQ 1.59 It is desired to multiply the numbers 0AH by 0BH and store the result in the
accumulator. The numbers are available in registers B and C respectively. A part
of the 8085 program for this purpose is given below :
MVI A, 00H
LOOP ------
------
-----
HLT
END
The sequence of instructions to complete the program would be
(A) JNX LOOP, ADD B, DCR C
(B) ADD B, JNZ LOOP, DCR C
(C) DCR C, JNZ LOOP, ADD B
(D) ADD B, DCR C, JNZ LOOP
SOL 1.59 Hence (D) is correct answer.
MVI A, 00H ; Clear accumulator
LOOP ADD B ; Add the contents of B to A

Brought to you by: Nodia and Company Visit us at: www.nodia.co.in


PUBLISHING FOR GATE
Page 27 GATE EC 2004 www.gatehelp.com

DCR C ; Decrement C
JNZ LOOP ; If C is not zero jump to loop
HLT
END
This instruction set add the contents of B to accumulator to contents of C times.
Hence (D) is correct answer.
MCQ 1.60 A 1 kHz sinusoidal signal is ideally sampled at 1500 samples/sec and the sampled
signal is passed through an ideal low-pass filter with cut-off frequency 800 Hz. The
output signal has the frequency.
(A) zero Hz (B) 0.75 kHz
(C) 0.5 kHz (D) 0.25 kHz
SOL 1.60 Hence Correct Option is (C)
Here fs = 1500 samples/sec, fm = kHz
The sampled frequency are 2.5 kHz, 0.5 kHz, Since LPF has cut-off frequency
800 Hz, then only output signal of frequency 0.5 kHz would pass through it
MCQ 1.61 A rectangular pulse train s (t) as shown in the figure is convolved with the signal
cos2 (4p # 103 t). The convolved signal will be a

(A) DC (B) 12 kHz sinusoid


(C) 8 kHz sinusoid (D) 14 kHz sinusoid
SOL 1.61 Hence Correct Option is (D)
S ^ t h = 1 61 + 2 cos ωs t + 2 cos 2ωs t + .....................@
Ts
^1 + cos 8π # 103 t h
cos 4π # 10 t =
2 3
2
ωs = 2π = 2π # 10 # 103
0.1 # 10−3
3

S^t h * x^t h = # S^τ h # ^τ − t hdτ


−3
3

= # 10 # 10 61 + 2 cos ω t + 2 cos 2ω t + ........@dt


3
s s
−3

61 + cos 8π # 103 t@
# 2
Brought to you by: Nodia and Company Visit us at: www.nodia.co.in
PUBLISHING FOR GATE
Page 28 GATE EC 2004 www.gatehelp.com

So, frequencies present will be fs ! fm, 2fs ! 3fs ! fm; fs = 10 kHz

fm = 8π # 10 = 4 kHz
3


Hence 14 kHz sinusoidal signal will be present
MCQ 1.62 Consider the sequence x [n] = [− 4 − j51 + j25]. The conjugate anti-symmetric part
-
of the sequence is
(A) [− 4 − j2.5, j2, 4 − j2.5] (B) [− j2.5, 1, j2.5]
(C) [− j2.5, j2, 0] (D) [− 4, 1, 4]
SOL 1.62 Hence (A) is correct answer.
We have x (n) = [− 4 − j5, 1 + 2j, 4]
-

x *( n) = [− 4 + j5, 1 − 2j, 4]
-

x *( − n) = [4, 1 − 2j, − 4 + j5]


-

x (n) − x* (− n)
xcas (n) =
2
= [− 4 − j 25 , 2j 4 − j 25 ]
-

MCQ 1.63 A causal LTI system is described by the difference equation


2y [n] = αy [n − 2] − 2x [n] + βx [n − 1]
The system is stable only if
(A) α = 2 , β < 2 (B) α > 2, β > 2
(C) α < 2 , any value of β (D) β < 2 , any value of α
SOL 1.63 Hence (C) is correct answer.
We have 2y (n) = αy (n − 2) − 2x (n) + βx (n − 1)
Taking z transform we get
2Y (z) = αY (z) z−2 − 2X (z) + βX (z) z−1
βz−1 − 2
=c m
Y (z)
or ...(i)
X (z) 2 − αz−2
z ( β2 − z)
or H (z) = 2 α
(z − 2 )
It has poles at ! α/2 and zero at 0 and β/2 . For a stable system poles must lie
inside the unit circle of z plane. Thus
α <1
2
or α <2
But zero can lie anywhere in plane. Thus, β can be of any value.

Brought to you by: Nodia and Company Visit us at: www.nodia.co.in


PUBLISHING FOR GATE
Page 29 GATE EC 2004 www.gatehelp.com

MCQ 1.64 A causal system having the transfer function H (s) = 1/ (s + 2) is excited with
10u (t). The time at which the output reaches 99% of its steady state value is
(A) 2.7 sec (B) 2.5 sec
(C) 2.3 sec (D) 2.1 sec
SOL 1.64 Hence (C) is correct option.
We have r (t) = 10u (t)
or R (s) = 10
s
Now H (s) = 1
s+2
C (s) = H (s) $ R (s) = 1 $ 10 10
s + 2 s s (s + 2)
or C (s) = 5 − 5
s s+2
c (t) = 5 [1 − e−2t]
The steady state value of c (t) is 5. It will reach 99% of steady state value reaches
at t , where
5 [1 − e−2t] = 0.99 # 5
or 1 − e−2t = 0.99
e−2t = 0.1
or − 2t = ln 0.1
or t = 2.3 sec
MCQ 1.65 The impulse response h [n] of a linear time invariant system is given as
− 2 2 n = 1, − 1
h [ n] = * 4 2 n = 2, − 2
0 otherwise
If the input to the above system is the sequence e jπn/4 , then the output is
(A) 4 2 e jπn/4 (B) 4 2 e−jπn/4
(C) 4e jπn/4 (D) − 4e jπn/4
SOL 1.65 Hence (D) is correct answer.
We have x (n) = e jπn/4
and h (n) = 4 2 δ (n + 2) − 2 2 δ (n + 1) − 2 2 δ (n − 1)
+ 4 2 δ (n − 2)
Now y (n) = x (n)* h (n)
3 2
= / x (n − k) h (k) = / x (n − k) h (k)
k =− 3 k =− 2

or y (n) = x (n + 2) h (− 2) + x (n + 1) h (− 1)
+ x (n − 1) h (1) + x (n − 2) h (2)
π π π π
j (n + 2) j (n + 1) j (n − 1)
= 4 2e 4
−2 2e 4
−2 2e 4
+ 4 2 e j (n − 2)
4

Brought to you by: Nodia and Company Visit us at: www.nodia.co.in


PUBLISHING FOR GATE
Page 30 GATE EC 2004 www.gatehelp.com

= 4 2 6e j (n + 2) + e j (n − 2)@ − 2 2 6e j (n + 1) + e j @
π π π π
4 4 4 4
(n − 1)

= 4 2 e j n 6e j + e−j @ − 2 2 e j n 6e j + e−j @
π π π π π π
4 2 2 2 4 4

π π
= 4 2 e j n [0] − 2 2 e j n [2 cos π4 ]
4 4

y (n) =− 4e j n
r

or 4

MCQ 1.66 Let x (t) and y (t) with Fourier transforms F (f) and Y (f) respectively be related as
shown in Fig. Then Y (f) is

(A) − 1 X (f/2) e−jπf (B) − 1 X (f/2) e j2πf


2 2
(C) − X (f/2) e j2πf (D) − X (f/2) e−j2πf
SOL 1.66 From given graph the relation in x (t) and y (t) is
y (t) =− x [2 (t + 1)]
F
x (t) X (f)
Using scaling we have
1 X f
a ca m
F
x (at)
1X f
2 c2m
F
Thus x (2t)
Using shifting property we ge
x (t − t0) = e−j2πft X (f) 0

j2πf
e−j2πf (− 1) 1 X b l = e X b l
F f f
Thus x [2 (t + 1)]
2 2 2 2
j 2π f
f
−e Xc m
F
− x [2 (t + 1)]
2 2
Hence (B) is correct answer.
MCQ 1.67 A system has poles at 0.1 Hz, 1 Hz and 80 Hz; zeros at 5 Hz, 100 Hz and 200 Hz.
The approximate phase of the system response at 20 Hz is
(A) − 90c (B) 0c
(C) 90c (D) − 180c
SOL 1.67 Approximate (comparable to 90c) phase shift are
Due to pole at 0.01 Hz " − 90c
Due to pole at 80 Hz " − 90c
Due to pole at 80 Hz " 0
Due to zero at 5 Hz " 90c

Brought to you by: Nodia and Company Visit us at: www.nodia.co.in


PUBLISHING FOR GATE
Page 31 GATE EC 2004 www.gatehelp.com

Due to zero at 100 Hz " 0


Due to zero at 200 Hz " 0
Thus approximate total − 90c phase shift is provided.
Hence (A) is correct option.

MCQ 1.68 Consider the signal flow graph shown in Fig. The gain x5 is
x1

1 − (be + cf + dg) bedg


(A) (B)
abcd 1 − (be + cf + dg)
abcd 1 − (be + cf + dg) + bedg
(C) (D)
1 − (be + cf + dg) + bedg abcd

SOL 1.68 Mason Gain Formula


Σpk 3 k
T (s) =
3
In given SFG there is only one forward path and 3 possible loop.
p1 = abcd
31 = 1
3= 1 − (sum of indivudual loops) - (Sum of two non touching loops)
= 1 − (L1 + L2 + L3) + (L1 L3)
Non touching loop are L1 and L3 where
L1 L2 = bedg

C (s) p1 3 1
Thus =
R (s) 1 − (be + cf + dg) + bedg
= abcd
1 − (be + cf + dg) + bedg
Hence (C) is correct option
−2 2
If A = =
1 − 3G
MCQ 1.69 , then sin At is

sin (− 4t) + 2 sin (− t) − 2 sin (− 4t) + 2 sin (− t)


(A) 1 = G
3 − sin (− 4t) + sin (− t) 2 sin (− 4t) + sin (− t)
sin (− 2t) sin (2t)
(B) =
sin (t) sin (− 3t)G
sin (4t) + 2 sin (t) 2 sin (− 4t) − 2 sin (− t)
(C) 1 =
3 − sin (− 4t) + sin (t) 2 sin (4t) + sin (t) G
cos (− t) + 2 cos (t) 2 cos (− 4t) + 2 cos (− t)
(D) 1 = G
3 − cos (− 4t) + cos (− t) − 2 cos (− 4t) + cos (t)

Brought to you by: Nodia and Company Visit us at: www.nodia.co.in


PUBLISHING FOR GATE
Page 32 GATE EC 2004 www.gatehelp.com

SOL 1.69 Hence (A) is correct option


−2 2
A ==
1 − 3G
We have

Characteristic equation is
[λI − A] = 0
λ + 2 −2
or =0
−1 λ + 3
or (λ + 2)( λ + 3) − 2 = 0
or λ2 + 5λ + 4 = 0
Thus λ1 =− 4 and λ2 =− 1
Eigen values are − 4 and − 1.
Eigen vectors for λ1 =− 4
(λ1 I − A) X1 = 0
λ1 + 2 − 2 x11
or = 1 λ + 3G=x G = 0
1 21

− 2 − 2 x11
=− 1 − 1G=x G = 0
21

or − 2x11 − 2x21 = 0
or x11 + x21 = 0
We have only one independent equation x11 =− x21.
Let x21 = K , then x11 =− K , the Eigen vector will be
x11 −K −1
=x G = = K G = K = 1 G
21

Now Eigen vector for λ2 =− 1


(λ2 I − A) X2 = 0
λ2 + 2 − 2 x12
or = − 1 λ + 3G=x G = 0
2 22

1 − 2 x12
or =− 1 2 G=x G = 0
22

We have only one independent equation x12 = 2x22


Let x22 = K , then x12 = 2K . Thus Eigen vector will be
x12 2K 2
=x G = = K G = K = 1 G
22

Digonalizing matrix
x11 x12 −1 2
M == G ==
x21 x22 1 1G
1 −2
M−1 = ` − 1 j=
1 − 1G
Now
3 −
Brought to you by: Nodia and Company Visit us at: www.nodia.co.in
PUBLISHING FOR GATE
Page 33 GATE EC 2004 www.gatehelp.com

Now Diagonal matrix of sin At is D where


sin (λ1 t) 0 sin (− 4t) 0
D == G ==
0 sin (λ2 t) 0 sin (λ2 t)G
Now matrixB = sin At = MDM−1
− 1 2 sin (− 4t) 0 1 −2
=−` 1 j= G=
3 1 1 0 sin (− t) − 1 − 1G
G=

− sin (− 4t) − 2 sin (− t) 2 sin (− 4t) − 2 sin (− t)


=−` 1 j=
3 sin (− 4t) + 2 sin (t) − 2 sin (− 4t) − sin (− t)G
− sin (− 4t) − 2 sin (− t) 2 sin (− 4t) − 2 sin (− t)
=−` 1 j=
3 sin (− 4t) − sin (− t) − 2 sin (− 4t) + 2 sin (− t)G
sin (− 4t) + 2 sin (− t) − 2 sin (− 4t) + 2 sin (− t)
= ` 1 j= Gs
3 − sin (− 4t + sin (− t) 2 sin (− 4t) + sin (− t)

MCQ 1.70 The open-loop transfer function of a unity feedback system is


G (s) = K
s (s + s + 2)( s + 3)
2

The range of K for which the system is stable is


(A) 21 > K > 0 (B) 13 > K > 0
4

(C) 21 < K < 3 (D) − 6 < K < 3


4
SOL 1.70 For ufb system the characteristic equation is
1 + G (s) = 0

1+ K1 + G (s) =0
s (s + 2s + 2)( s + 3)
2

s 4 + 4s3 + 5s2 + 6s + K = 0
The routh table is shown below. For system to be stable,
(21 − 4K)
0 < K and 0 <
2/7
This gives 0 < K < 21
4

s4 1 5 K
s3 4 6 0
s2 7
2 K
21 − 4K
s1 7/2
0
s0 K

Brought to you by: Nodia and Company Visit us at: www.nodia.co.in


PUBLISHING FOR GATE
Page 34 GATE EC 2004 www.gatehelp.com

Hence (A) is correct option


MCQ 1.71 For the polynomial P (s) = s2 + s 4 + 2s3 + 2s2 + 3s + 15 the number of roots which
lie in the right half of the s −plane is
(A) 4 (B) 2
(C) 3 (D) 1
SOL 1.71 Hence (B) is correct option.
We have P (s) = s5 + s 4 + 2s3 + 3s + 15
The routh table is shown below.
If ε " 0+ then 2ε +ε 12 is positive and −15ε2−ε +2412ε − 144 is negative. Thus there are two sign
2

change in first column. Hence system has 2 root on RHS of plane.

s5 1 2 3
s4 1 2 15
s3 ε − 12 0
2ε + 12
s2 ε 15 0
−15ε − 24ε − 144
2
s1 2ε + 12

s0 0

MCQ 1.72 The state variable equations of a system are : xo1 =− 3x1 − x2 = u, xo2 = 2x1 and
y = x1 + u . The system is
(A) controllable but not observable
(B) observable but not controllable
(C) neither controllable nor observable
(D) controllable and observable
SOL 1.72 Hence (D) is correct option.
x1 − 3 − 1 x1 1
We have = G = = G= G = 0G
+ u
x2 2 0 x2
x1 1
and Y = [1 0]= G + = Gu
x2 2
−3 −1 1
Here A == G , B = = G and C = [1 0]
2 0 0
The controllability matrix is
QC = [B AB ]
1 −3
==
0 2G
det QC ! 0 Thus controllable
The observability matrix is

Brought to you by: Nodia and Company Visit us at: www.nodia.co.in


PUBLISHING FOR GATE
Page 35 GATE EC 2004 www.gatehelp.com

Q0 = [CT AT CT ]
1 −3
==
0 − 1G
!0

det Q0 ! 0 Thus observable

1 0
Given A = =
0 1G
MCQ 1.73 , the state transition matrix eAt is given by

0 e−t et 0
(A) > −t H (B) = G
e 0 0 et
e−t 0 0 et
(C) > H (D) = G
0 e−t et 0
SOL 1.73 Hence (B) is correct option.
s 0 1 0 s−1 0
(sI − A) = = G −= G ==
0 s 0 1 0 s − 1G
(s − 1) 0 1
0
1 = 0 (s − 1)G > 0 H
s−1
(sI − A) −1 = = 1
(s − 1) 2 s−1

eAt = L−1 [(sI − A)] −1

et 0
== G
0 et

MCQ 1.74 Consider the signal x (t) shown in Fig. Let h (t) denote the impulse response of the
filter matched to x (t), with h (t) being non-zero only in the interval 0 to 4 sec. The
slope of h (t) in the interval 3 < t < 4 sec is

(A) 1 sec - 1 (B) − 1 sec - 1


2

(C) − 1 sec - 1 (D) 1 sec - 1


2
SOL 1.74 The impulse response of matched filter is
h (t) = x (T − t)
Since here T = 4 , thus
h (t) = x (4 − t)
The graph of h (t) is as shown below.

Brought to you by: Nodia and Company Visit us at: www.nodia.co.in


PUBLISHING FOR GATE
Page 36 GATE EC 2004 www.gatehelp.com

From graph it may be easily seen that slope between 3 < t < 4 is − 1.
Hence (B) is correct option.
MCQ 1.75 A 1 mW video signal having a bandwidth of 100 MHz is transmitted to a receiver
through cable that has 40 dB loss. If the effective one-side noise spectral density at
the receiver is 10 - 20 Watt/Hz, then the signal-to-noise ratio at the receiver is
(A) 50 dB (B) 30 dB
(C) 40 dB (D) 60 dB
SOL 1.75 The SNR at transmitter is
SNRtr = Ptr
NB
10 - 3 = 109
10 - 20 # 100 # 106
In dB SNRtr = 10 log 109 = 90 dB
Cable Loss = 40 db
At receiver after cable loss we have
SNRRc = 90 − 40 = 50 dB
Hence (A) is correct option.
MCQ 1.76 A 100 MHz carrier of 1 V amplitude and a 1 MHz modulating signal of 1 V
amplitude are fed to a balanced modulator. The ourput of the modulator is passed
through an ideal high-pass filter with cut-off frequency of 100 MHz. The output
of the filter is added with 100 MHz signal of 1 V amplitude and 90c phase shift as
shown in the figure. The envelope of the resultant signal is

(A) constant (B) 1 + sin (2π # 106 t)

(C) 5 − sin (2π − 106 t) (D) 5 + cos (2π # 106 t)


4 4
SOL 1.76 Hence (C) is correct option.
We have fc = 100 MHz = 100 # 106 and fm = 1 MHz
= 1 # 106

Brought to you by: Nodia and Company Visit us at: www.nodia.co.in


PUBLISHING FOR GATE
Page 37 GATE EC 2004 www.gatehelp.com

The output of balanced modulator is


VBM (t) = [cos ωc t][ cos ωc t]
= 1 [cos (ωc + ωm) t + cos (ωc − ωm) t]
2
If VBM (t) is passed through HPF of cut off frequency fH = 100 # 106 , then only
(ωc + ωm) passes and output of HPF is
VHP (t) = 1 cos (ωc + ωm) t
2
Now V0 (t) = VHP (t) + sin (2π # 100 # 106) t

= 1 cos [2π100 # 106 + 2π # 1 # 106 t] + sin (2π # 100 # 106) t


2
= 1 cos [2π108 + 2π106 t] + sin (2π108) t
2
= 1 [cos (2π108 t) t cos (2π106 t)] − sin [2π108 t sin (2π106 t) + sin 2π108 t]
2
= 1 cos (2π106 t) cos 2π108 t + `1 − 1 sin 2π106 t j sin 2π108 t
2 2
This signal is in form
= A cos 2π108 t + B sin 2π108 t
The envelope of this signal is
= A2 + B2
2 2
= ` 1 cos (2π106 t)j + `1 − 1 sin (2π106 t j
2 2
= 1 cos2 (2π106 t) + 1 + 1 sin2 (2π106 t) − sin (2π106 t)
4 4
= 1 + 1 − sin (2π106 t)
4
= 5 − sin (2π106 t)
4

MCQ 1.77 Two sinusoidal signals of same amplitude and frequencies 10 kHz and 10.1 kHz are
added together. The combined signal is given to an ideal frequency detector. The
output of the detector is
(A) 0.1 kHz sinusoid (B) 20.1 kHz sinusoid
(C) a linear function of time (D) a constant
SOL 1.77 Hence (A) is correct option.
s (t) = A cos [2π10 # 103 t] + A cos [2π10.1 # 103 t]
Here T1 = 1 = 100μ sec
10 # 103
and T2 = 1 = 99μ sec
10.1 # 103
Period of added signal will be LCM [T1, T2]

Brought to you by: Nodia and Company Visit us at: www.nodia.co.in


PUBLISHING FOR GATE
Page 38 GATE EC 2004 www.gatehelp.com

Thus T = LCM [100, 99] = 9900μ sec


Thus frequency f = 1 = 0.1 kHz
9900μ

MCQ 1.78 Consider a binary digital communication system with equally likely 0’s and 1’s.
When binary 0 is transmitted the detector input can lie between the levels − 0.25
V and + 0.25 V with equl probability : when binary 1 is transmitted, the voltage
at the detector can have any value between 0 and 1 V with equal probability. If the
detector has a threshold of 0.2 V (i.e., if the received signal is greater than 0.2 V,
the bit is taken as 1), the average bit error probability is
(A) 0.15 (B) 0.2
(C) 0.05 (D) 0.5
SOL 1.78 The pdf of transmission of 0 and 1 will be as shown below :

Probability of error of 1
P (0 # X # 0.2) = 0.2
Probability of error of 0 :
P (0.2 # X # 0.25) = 0.05 # 2 = 0.1

P (0 # X # 0.2) + P (0.2 # X # 0.25)


Average error =
2
= 0.2 + 0.1 = 0.15
0
Hence (A) is correct option.
MCQ 1.79 A random variable X with uniform density in the interval 0 to 1 is quantized as
follows :
If 0 # X # 0.3 , xq = 0
If 0.3 < X # 1, xq = 0.7
where xq is the quantized value of X.
The root-mean square value of the quantization noise is
(A) 0.573 (B) 0.198
(C) 2.205 (D) 0.266
SOL 1.79 Hence (B) is correct option.
The square mean value is
#- 3 (x − xq) 2 f (x) dx
3
σ2 =

Brought to you by: Nodia and Company Visit us at: www.nodia.co.in


PUBLISHING FOR GATE
Page 39 GATE EC 2004 www.gatehelp.com

#0 (x − xq) 2 f (x) dx
1
=

#0 #0.3 (x − 0.7) 2 f (x) dx


0. 3 0. 1
= (x − 0) 2 f (x) dx +
3 0. 3 3 2 1
= ; x E + ; x + 0.49x − 14 x E
3 0 3 2 0. 3
or σ2 = 0.039
RMS = σ2 = 0.039 = 0.198
MCQ 1.80 Choose the current one from among the alternative A, B, C, D after matching an
item from Group 1 with the most appropriate item in Group 2.
Group 1 Group 2
1. FM P. Slope overload
2. DM Q. μ-law
3. PSK R. Envelope detector
4. PCM S. Hilbert transform
T. Hilbert transform
U. Matched filter
(A) 1 - T, 2 - P, 3 - U, 4 - S (B) 1 - S, 2 - U, 3 - P, 4 - T
(C) 1 - S, 2 - P, 3 - U, 4 - Q (D) 1 - U, 2 - R, 3 - S, 4 - Q
SOL 1.80 Hence (C) is correct option.
FM $ Capture effect
DM $ Slope over load
PSK $ Matched filter
PCM $ μ − law
MCQ 1.81 Three analog signals, having bandwidths 1200 Hz, 600 Hz and 600 Hz, are sampled
at their respective Nyquist rates, encoded with 12 bit words, and time division
multiplexed. The bit rate for the multiplexed. The bit rate for the multiplexed
signal is
(A) 115.2 kbps (B) 28.8 kbps
(C) 57.6 kbps (D) 38.4 kbps
SOL 1.81 Since fs = 2fm , the signal frequency and sampling frequency are as follows
fm1 = 1200 Hz $ 2400 samples per sec
fm2 = 600 Hz $ 1200 samples per sec
fm3 = 600 Hz $ 1200 samples per sec
Thus by time division multiplexing total 4800 samples per second will be sent.
Since each sample require 12 bit, total 4800 # 12 bits per second will be sent
Thus bit rate Rb = 4800 # 12 = 57.6 kbps
Hence (C) is correct option.
MCQ 1.82 Consider a system shown in the figure. Let X (f) and Y (f) and denote the Fourier

Brought to you by: Nodia and Company Visit us at: www.nodia.co.in


PUBLISHING FOR GATE
Page 40 GATE EC 2004 www.gatehelp.com

transforms of x (t) and y (t) respectively. The ideal HPF has the cutoff frequency
10 kHz.

The positive frequencies where Y (f) has spectral peaks are


(A) 1 kHz and 24 kHz (B) 2 kHz and 244 kHz
(C) 1 kHz and 14 kHz (D) 2 kHz and 14 kHz
SOL 1.82 The input signal X (f) has the peak at 1 kHz and − 1 kHz. After balanced modulator
the output will have peak at fc ! 1 kHz i.e. :
10 ! 1 $ 11 and 9 kHz
10 ! (− 1) $ 9 and 11 kHz
9 kHz will be filtered out by HPF of 10 kHz. Thus 11 kHz will remain. After
passing through 13 kHz balanced modulator signal will have 13 ! 11 kHz signal i.e.
2 and 24 kHz.
Thus peak of Y (f) are at 2 kHz and 24 kHz.
Hence (B) is correct option.
MCQ 1.83 A parallel plate air-filled capacitor has plate area of 10 - 4 m 2 and plate separation
of 10 - 3 m. It is connect - ed to a 0.5 V, 3.6 GHz source. The magnitude of the
displacement current is ( ε = 361π 10 - 9 F/m)
(A) 10 mA (B) 100 mA
(C) 10 A (D) 1.59 mA
SOL 1.83 The capacitance is
C = εo A = 8.85 # 10 - 3 # 10 = 8.85 # 10 - 13
- 12 -4

d 10
The charge on capacitor is
Q = CV = 8.85 # 10 - 13 = 4.427 # 10 - 13
Displacement current in one cycle
Q
I = = fQ = 4.427 # 10 - 13 # 3.6 # 109 = 1.59 mA
T
Hence (D) is correct option.

Brought to you by: Nodia and Company Visit us at: www.nodia.co.in


PUBLISHING FOR GATE
Page 41 GATE EC 2004 www.gatehelp.com

MCQ 1.84 A source produces binary data at the rate of 10 kbps. The binary symbols are
represented as shown in the figure given below.

The source output is transmitted using two modulation schemes, namely Binary
PSK (BPSK) and Quadrature PSK (QPSK). Let B1 and B2 be the bandwidth
requirements of BPSK and QPSK respectively. Assume that the bandwidth of he
above rectangular pulses is 10 kHz, B1 and B2 are
(A) B1 = 20 kHz, B2 = 20 kHz (B) B1 = 10 kHz, B2 = 20 kHz
(C) B1 = 20 kHz, B2 = 10 kHz (D) B1 = 20 kHz, B2 = 10 kHz
SOL 1.84 The required bandwidth of M array PSK is
BW = 2Rb
n
where 2n = M and Rb is bit rate
For BPSK, M = 2 = 2n $ n = 1

Thus B1 = 2Rb = 2 # 10 = 20 kHz


1
For QPSK, M = 4 = 2n $ n = 2

Thus B2 = 2Rb = 10 kHz


2
Hence (C) is correct option.
MCQ 1.85 Consider a 300 Ω, quarter - wave long (at 1 GHz) transmission line as shown in
Fig. It is connected to a 10 V, 50 Ω source at one end and is left open circuited at
the other end. The magnitude of the voltage at the open circuit end of the line is

(A) 10 V (B) 5 V
(C) 60 V (D) 60/7 V
SOL 1.85 Hence (C) is correct option.
VL = ZO
Vin Zin
Brought to you by: Nodia and Company Visit us at: www.nodia.co.in
PUBLISHING FOR GATE
Page 42 GATE EC 2004 www.gatehelp.com

or VL = ZO Vin = 10 # 300 = 60 V
Zin 50

MCQ 1.86 In a microwave test bench, why is the microwave signal amplitude modulated at 1
kHz
(A) To increase the sensitivity of measurement
(B) To transmit the signal to a far-off place
(C) To study amplitude modulations
(D) Because crystal detector fails at microwave frequencies
SOL 1.86 Hence (D) is correct option.
MCQ 1.87 If E = (atx + jaty) e jkz - kωt and H = (k/ωμ) (aty + katx ) e jkz - jωt , the time-averaged Poynting
vector is
(A) null vector (B) (k/ωμ) atz
(C) (2k/ωμ) atz (D) (k/2ωμ) atz
SOL 1.87 Hence (A) is correct option.
Ravg = 1 Re [E # H*]
2
E # H* = (atx + jaty) e jkz − jωt # k (− jatx + aty) e−jkz + jωt
ωμ
= atz ; k − (− j) (j) k E = 0
ωμ ωμ
Thus Ravg = 1 Re [E # H*] = 0
2

MCQ 1.88 Consider an impedance Z = R + jX marked with point P in an impedance Smith


chart as shown in Fig. The movement from point P along a constant resistance
circle in the clockwise direction by an angle 45c is equivalent to

(A) adding an inductance in series with Z


(B) adding a capacitance in series with Z
(C) adding an inductance in shunt across Z
(D) adding a capacitance in shunt across Z

Brought to you by: Nodia and Company Visit us at: www.nodia.co.in


PUBLISHING FOR GATE
Page 43 GATE EC 2004 www.gatehelp.com

SOL 1.88 Suppose at point P impedance is


Z = r + j (− 1)
If we move in constant resistance circle from point P in clockwise direction by an
angle 45c, the reactance magnitude increase. Let us consider a point Q at 45c from
point P in clockwise direction. It’s impedance is
Z1 = r − 0.5j
or Z1 = Z + 0.5j
Thus movement on constant r - circle by an +45c in CW direction is the addition
of inductance in series with Z .
Hence (A) is correct option.
MCQ 1.89 A plane electromagnetic wave propagating in free space is incident normally on a
large slab of loss-less, non-magnetic, dielectric material with ε > ε0 . Maxima and
minima are observed when the electric field is measured in front of the slab. The
maximum electric field is found to be 5 times the minimum field. The intrinsic
impedance of the medium should be
(A) 120π Ω (B) 60π Ω
(C) 600π Ω (D) 24π Ω
SOL 1.89 Hence (D) is correct option.
1− Γ
We have VSWR = Emax = 5 =
Emin 1+ Γ
or Γ =2
3
Thus Γ =− 2
3
η − η1
Now Γ= 2
η2 + η1
η − 120π
or −2 = 2
3 η2 + 120π
or η2 = 24π
MCQ 1.90 A lossless transmission line is terminated in a load which reflects a part of the
incident power. The measured VSWR is 2. The percentage of the power that is
reflected back is
(A) 57.73 (B) 33.33
(C) 0.11 (D) 11.11
SOL 1.90 Hence (D) is correct option.
1− Γ
The VSWR 2=
1+ Γ
or Γ =1
3

Brought to you by: Nodia and Company Visit us at: www.nodia.co.in


PUBLISHING FOR GATE
Page 44 GATE EC 2004 www.gatehelp.com

Pref
Thus = Γ2= 1
Pinc 9
or Pref = Pinc
9
i.e. 11.11% of incident power is reflected.

Answer Sheet
1. (B) 19. (A) 37. (D) 55. (B) 73. (B)
2. (D) 20. (D) 38. (B) 56. (C) 74. (B)
3. (A) 21. (D) 39. (B) 57. (A) 75. (A)
4. (A) 22. (C) 40. (D) 58. (D) 76. (C)
5. (C) 23. (D) 41. (C) 59. (D) 77. (A)
6. (C) 24. (C) 42. (D) 60. (C) 78. (A)
7. (A) 25. (C) 43. (A) 61. (D) 79. (B)
8. (D) 26. (B) 44. (B) 62. (A) 80. (C)
9. (B) 27. (B) 45. (C) 63. (C) 81. (C)
10. (C) 28. (A) 46. (D) 64. (C) 82. (B)
11. (A) 29. (D) 47. (A) 65. (D) 83. (D)
12. (A) 30. (A) 48. (A) 66. (B) 84. (C)
13. (C) 31. (D) 49. (D) 67. (A) 85. (C)
14. (A) 32. (B) 50. (B) 68. (C) 86. (D)
15. (D) 33. (C) 51. (C) 69. (A) 87. (A)
16. (B) 34. (B) 52. (D) 70 (A) 88. (A)
17. (A) 35. (D) 53. (C) 71 (B) 89. (D)
18. (C) 36. (A) 54. (D) 72 (D) 90. (D)

Brought to you by: Nodia and Company Visit us at: www.nodia.co.in


PUBLISHING FOR GATE
GATE EC
2005

Question 1 - 30 Carry One Mark Each

MCQ 1.1 The following differential equation has


d2 y dy 3
3 c 2 m + 4 c m + y2 + 2 = x
dt dt
(A) degree = 2 , order = 1 (B) degree = 1, order = 2
(C) degree = 4 , order = 3 (D) degree = 2 , order = 3
SOL 1.1 Order is the highest derivative term present in the equation and degree is the power
of highest derivative term.
Order = 2 , degree = 1
Hence (B) is correct answer.
MCQ 1.2 Choose the function f (t); − 3 < t < 3 for which a Fourier series cannot be defined.
(A) 3 sin (25t) (B) 4 cos (20t + 3) + 2 sin (710t)
(C) exp (− t ) sin (25t) (D) 1
SOL 1.2 Fourier series is defined for periodic function and constant.
3 sin (25t) is a periodic function.
4 cos (20t + 3) + 2 sin (710t) is sum of two periodic function and also a periodic
function.
e− t sin (25t) is not a periodic function, so FS can’t be defined for it.
1 is constant
Hence (C) is correct option.
MCQ 1.3 A fair dice is rolled twice. The probability that an odd number will follow an even
number is
(A) 1/2 (B) 1/6
(C) 1/3 (D) 1/4
SOL 1.3 Probability of coming odd number is 12 and the probability of coming even number
is 12 . Both the events are independent to each other, thus probability of coming odd
number after an even number is 12 # 12 = 14 .
Hence (D) is correct answer.

Brought to you by: Nodia and Company Visit us at: www.nodia.co.in


PUBLISHING FOR GATE
Page 2 GATE EC 2005 www.gatehelp.com

d2 y dy
MCQ 1.4 A solution of the following differential equation is given by 2
−5 + 6y = 0
dx dx
(A) y = e2x + e−3x (B) y = e2x + e3x
(C) y = e−2x + 33x (D) y = e−2x + e−3x
SOL 1.4 Hence (B) is correct answer.
d2 y dy
We have 2
−5 + 6y = 0
dx dx
The A.E. is m2 − 5m + 6 = 0
m = 3, 2
The CF is yc = C1 e3x + C2 e2x
Since Q = 0 , thus y = C1 e3x + C2 e2x
Thus only (B) may be correct.
MCQ 1.5 The function x (t) is shown in the figure. Even and odd parts of a unit step function
u (t) are respectively,

(A) 1 , 1 x (t) (B) − 1 , 1 x (t)


2 2 2 2

(C) 1 , − 1 x (t) (D) − 1 , − 1 x (t)


2 2 2 2
SOL 1.5 Hence (A) is correct answer.
g (t) + g (− t)
Ev{g (t)} =
2
g (t) − g (− t)
odd{g (t)} =
2
Here g (t) = u (t)
u (t) + u (− t) 1
Thus ue (t) = =
2 2
u (t) − u (− t) x (t)
uo (t) = =
2 2

MCQ 1.6 The region of convergence of z − transform of the sequence


b 6 l u (n) − b 5 l u (− n − 1) must be
5 n 6 n

(A) z < 5 (B) z > 5


6 6

Brought to you by: Nodia and Company Visit us at: www.nodia.co.in


PUBLISHING FOR GATE
Page 3 GATE EC 2005 www.gatehelp.com

(C) 5 < z < 6 (D) 6 < z < 3


6 5 5
SOL 1.6 Hence (C) is correct answer.
Here x1 (n) = ` 5 jn u (n)
6
X1 (z) = 1 ROC : R1 " z > 5
1 − ^ 65 z−1h 6
x2 (n) =−` 6 jn u (− n − 1)
5
X1 (z) = 1 − 1 ROC : R2 " z < 6
1 − ^ 65 z−1h 5
Thus ROC of x1 (n) + x2 (n) is R1 + R2 which is 5 < z < 6
6 5

MCQ 1.7 The condition on R, L and C such that the step response y (t) in the figure has no
oscillations, is

(A) R $ 1 L (B) R $ L
2 C C

(C) R $ 2 L (D) R = 1
C LC
SOL 1.7 Transfer function is
1 1
Y (s) sC 1 LC
= = 2 =
U (s) R + sL + 1 s LC + scR + 1 2 R
s + s+ 1
sC L LC
Comparing with s + 2ξωn s + ωn = 0 we have
2 2

Here 2ξωn = R ,
L
and ωn = 1
LC
Thus ξ = R LC = R C
2L 2 L
For no oscillations, ξ $ 1
Thus R C $ 1
2 L
or R $2 L
C
Hence (C) is correct option.

Brought to you by: Nodia and Company Visit us at: www.nodia.co.in


PUBLISHING FOR GATE
Page 4 GATE EC 2005 www.gatehelp.com

MCQ 1.8 The ABCD parameters of an ideal n: 1 transformer shown in the figure are
n 0
>0 x H

The value of x will be


(A) n (B) 1
n

(C) n2 (D) 12
n
SOL 1.8 For given transformer
I2 = V1 = n
I1 V2 1
or I1 = I2 and V1 = nV2
n
Comparing with standard equation
V1 = AV2 + BI2
I1 = CV2 + DI2
A B n 0
=C D G = = 0 1 G
n

Thus x = 1
n
Hence (B) is correct option.

MCQ 1.9 In a series RLC circuit, R = 2 kΩ , L = 1 H, and C = 1 μF The resonant frequency


400
is
(A) 2 # 10 4 Hz (B) 1 # 10 4 Hz
π
(C) 10 4 Hz (D) 2π # 10 4 Hz
SOL 1.9 Hence (B) is correct option.
We have L = 1H and C = 1 # 10−6
400
Resonant frequency
f0 = 1 == 1
2π LC 2π 1 # 1 # 10 - 6
400
3 4
= 10 # 20 = 10 Hz
2π π

Brought to you by: Nodia and Company Visit us at: www.nodia.co.in


PUBLISHING FOR GATE
Page 5 GATE EC 2005 www.gatehelp.com

MCQ 1.10 The maximum power that can be transferred to the load resistor RL from the
voltage source in the figure is

(A) 1 W (B) 10 W
(C) 0.25 W (D) 0.5 W
SOL 1.10 Maximum power will be transferred when RL = Rs = 100Ω
In this case voltage across RL is 5 V, therefore
2
Pmax = V = 5 # 5 = 0.25 W
R 100
Hence (C) is correct option.
MCQ 1.11 The bandgap of Silicon at room temperature is
(A) 1.3 eV (B) 0.7 eV
(C) 1.1 eV (D) 1.4 eV
SOL 1.11 For silicon at 0 K,
Eg0 = 1.21 eV
At any temperature
EgT = Eg0 − 3.6 # 10 - 4 T
At T = 300 K,
Eg300 = 1.21 − 3.6 # 10 - 4 # 300 = 1.1 eV
This is standard value, that must be remembered.
Hence option (C) is correct.
MCQ 1.12 A Silicon PN junction at a temperature of 20c C has a reverse saturation current
of 10 pico - Ameres (pA). The reserve saturation current at 40cC for the same bias
is approximately
(A) 30 pA (B) 40 pA
(C) 50 pA (D) 60 pA
SOL 1.12 The reverse saturation current doubles for every 10cC rise in temperature as follows
I0 (T) = I 01 # 2(T − T )/10
1

Thus at 40c C, I0 = 40 pA
Hence option (B) is correct.
MCQ 1.13 The primary reason for the widespread use of Silicon in semiconductor device
technology is

Brought to you by: Nodia and Company Visit us at: www.nodia.co.in


PUBLISHING FOR GATE
Page 6 GATE EC 2005 www.gatehelp.com

(A) abundance of Silicon on the surface of the Earth.


(B) larger bandgap of Silicon in comparison to Germanium.
(C) favorable properties of Silicon - dioxide (SiO2)
(D) lower melting point
SOL 1.13 Silicon is abundant on the surface of earth in the from of SiO2 .
Hence option (A) is correct.
MCQ 1.14 The effect of current shunt feedback in an amplifier is to
(A) increase the input resistance and decrease the output resistance
(B) increases both input and output resistance
(C) decrease both input and output resistance
(D) decrease the input resistance and increase the output resistance
SOL 1.14 The effect of current shunt feedback in an amplifier is to decrease the input
resistance and increase the output resistance as :
Rif = Ri
1 + Aβ
Rof = R0 (1 + Aβ)
where Ri " Input resistance without feedback
Rif " Input resistance with feedback.
Hence (D) is correct option.
MCQ 1.15 The input resistance Ri of the amplifier shown in the figure is

(A) 30 kΩ (B) 10 kΩ
4
(C) 40 kΩ (D) infinite
SOL 1.15 Since the inverting terminal is at virtual ground, the current flowing through the
voltage source is
Is = Vs
10k
or Vs = 10 kΩ = R
in
Is
Hence (B) is correct option.

Brought to you by: Nodia and Company Visit us at: www.nodia.co.in


PUBLISHING FOR GATE
Page 7 GATE EC 2005 www.gatehelp.com

MCQ 1.16 The first and the last critical frequency of an RC -driving point impedance function
must respectively be
(A) a zero and a pole (B) a zero and a zero
(C) a pole and a pole (D) a pole and a zero
SOL 1.16 For stability poles and zero interlace on real axis. In RC series network the driving
point impedance is
Zins = R + 1 = 1 + sRC
Cs sC
Here pole is at origin and zero is at s =− 1/RC , therefore first critical frequency is
a pole and last critical frequency is a zero.
For RC parallel network the driving point impedance is
R 1
Zinp = Cs = R
R+ 1 1 + sRC
Cs
Here pole is s =− 1/RC and zero is at 3, therefore first critical frequency is a pole
and last critical frequency is a zero.
Hence (C) is correct option.
MCQ 1.17 The cascade amplifier is a multistage configuration of
(A) CC − CB (B) CE − CB
(C) CB − CC (D) CE − CC
SOL 1.17 The CE configuration has high voltage gain as well as high current gain. It performs
basic function of amplifications. The CB configuration has lowest Ri and highest Ro
. It is used as last step to match a very low impedance source and to drain a high
impedance load
Thus cascade amplifier is a multistage configuration of CE-CB
Hence (B) is correct option
MCQ 1.18 Decimal 43 in Hexadecimal and BCD number system is respectively
(A) B2, 0100 011 (B) 2B, 0100 0011
(C) 2B, 0011 0100 (D) B2, 0100 0100
SOL 1.18 Dividing 43 by 16 we get

g
2
16 43
32
11
11 in decimal is equivalent is B in hexamal.
Thus 4310 * 2B16
Now 410 * 01002
310 * 00112
Thus 4310 * 01000011BCD

Brought to you by: Nodia and Company Visit us at: www.nodia.co.in


PUBLISHING FOR GATE
Page 8 GATE EC 2005 www.gatehelp.com

Hence (B) is correct answer.


MCQ 1.19 The Boolean function f implemented in the figure using two input multiplexes is

(A) ABC + ABC (B) ABC + ABC


(C) ABC + ABC (D) ABC + ABC
SOL 1.19 The diagram is as shown in fig

f' = BC + BC
f = f' A + f ' 0
= f'A = ABC + ABC
Hence (A) is correct answer.
MCQ 1.20 Which of the following can be impulse response of a causal system ?

SOL 1.20 For causal system h (t) = 0 for t # 0 . Only (D) satisfy this condition.
Hence (D) is correct answer.

Brought to you by: Nodia and Company Visit us at: www.nodia.co.in


PUBLISHING FOR GATE
GATE ESE PSU’s 2019-20
ECE ENGINEERING
GATE ECE 2003-2019 SOLVED

GATE ECE 2003-2019 SOLVED Detail Solution

CONTENT COVERED:
1.Theory Notes
2.Explanation
3.Derivation
4.Example
5.Shortcut & Formula Summary
6.Previous year Paper Q. Sol.
Noted-: Single Source Follow, Revise
Multiple Time Best key of Success
1
Page

http://www.orbitmentor.com [email protected]
Page 9 GATE EC 2005 www.gatehelp.com

MCQ 1.21 Let x (n) = ( 12 ) n u (n), y (n) = x2 (n) and Y (e jω) be the Fourier transform of y (n) then
Y (e j0)
(A) 1 (B) 2
4

(C) 4 (D) 4
3
SOL 1.21 Hence (D) is correct answer.
x (n) = b 1 l u (n)
n

2
y (n) = x2 (n) = b 1 l u2 (n)
2n

2
2 n
y (n) = ;b 1 l E u (n) = b 1 l u (n)
n
or ...(1)
2 4
n=3 n=3
/ b 14 l e−jωn
n
Y (e jω) = / y (n) e−jωn =
n =− 3 n=0
n=3 n
/ ` 14 j = 1 +b1l +b1l+b1l +b1l
1 3 4
or Y (e j0) =
n=0
4 4 4 4
or Y (e j0) = 1 =4
1− 1
4
3
Alternative :
Taking z transform of (1) we get
Y (z) = 1
1 − 14 z−1
Substituting z = e jω we have
Y (e jω) = 1
1 − 14 e−jω
Y (e j0) = 1 1 = 4
1− 4 3

MCQ 1.22 Find the correct match between group 1 and group 2.
Group 1 Group 2
P. {1 + km (t) A sin (ωc t)} W. Phase modulation
Q. km (t) A sin (ωc t) X. Frequency modulation
R. A sin {ωc t + km (t)} Y. Amplitude modulation
#- 3
t
S. A sin ; ωc t + k m (t) dt E Z. DSB-SC modulation

(A) P − Z, Q − Y, R − X, S − W
(B) P − W, Q − X, R − Y, S − Z
(C) P − X, Q − W, R − Z, S − Y
(D) P − Y, Q − Z, R − W, S − X
SOL 1.22 Hence (D) is correct option.
{1 + km (t)} A sin (ωc t) $ Amplitude modulation

Brought to you by: Nodia and Company Visit us at: www.nodia.co.in


PUBLISHING FOR GATE
Page 10 GATE EC 2005 www.gatehelp.com

dm (t) Asin (ωc t) $ DSB-SC modulation


A sin {cos t + km (t)} $ Phase Modulation
A sin [ωct + k] t- 3 m (t) dt $ Frequency Modulation
MCQ 1.23 The power in the signal s (t) = 8 cos (20π − π2 ) + 4 sin (15πt) is
(A) 40 (B) 41
(C) 42 (D) 82
SOL 1.23 Hence (A) is correct answer.
s (t) = 8 cos ` π − 20πt j + 4 sin 15πt
2
= 8 sin 20πt + 4 sin 15πt
Here A1 = 8 and A2 = 4 . Thus power is
2 2 2 2
P = A1 + A2 = 8 + 4 = 40
2 2 2 2

MCQ 1.24 Which of the following analog modulation scheme requires the minimum transmitted
power and minimum channel bandwidth ?
(A) VSB (B) DSB-SC
(C) SSB (D) AM
SOL 1.24 Hence (C) is correct option.
VSB $ fm + fc
DSB - SC $ 2fm
SSB $ fm
AM $ 2fm
Thus SSB has minimum bandwidth and it require minimum power.
MCQ 1.25 A linear system is equivalently represented by two sets of state equations :
Xo = AX + BU and Wo = CW + DU
The eigenvalues of the representations are also computed as [λ] and [μ]. Which one
of the following statements is true ?
(A) [λ] = [μ] and X = W (B) [λ] = [μ] and X ! W
(C) [λ] ! [μ] and X = W (D) [λ] = [μ] and X ! W
SOL 1.25 Hence (C) is correct option
We have Xo = AX + BU where λ is set of Eigen values
and o
W = CW + DU where μ is set of Eigen values
If a liner system is equivalently represented by two sets of state equations, then for
both sets, states will be same but their sets of Eigne values will not be same i.e.
X = W but λ ! μ
MCQ 1.26 Which one of the following polar diagrams corresponds to a lag network ?

Brought to you by: Nodia and Company Visit us at: www.nodia.co.in


PUBLISHING FOR GATE
Page 11 GATE EC 2005 www.gatehelp.com

SOL 1.26 The transfer function of a lag network is


T (s) = 1 + sT β > 1; T > 0
1 + sβT

T (jω) = 1 + ω2 T2
1 + ω2 β2 T2
and +T (jω) = tan−1 (ωT) − tan−1 (ωβT)
At ω = 0 , T (jω) =1
At ω = 0 ,+T (jω) =− tan−1 0 = 0
At ω = 3 , T (jω) = 1
β
At ω = 3 ,+T (jω) =0
Hence (D) is correct option.
MCQ 1.27 Despite the presence of negative feedback, control systems still have problems of
instability because the
(A) Components used have non- linearities
(B) Dynamic equations of the subsystem are not known exactly.
(C) Mathematical analysis involves approximations.
(D) System has large negative phase angle at high frequencies.
SOL 1.27 Despite the presence of negative feedback, control systems still have problems
of instability because components used have nonlinearity. There are always some
variation as compared to ideal characteristics.
Hence (A) is correct option.
MCQ 1.28 The magnetic field intensity vector of a plane wave is given by
H (x, y, z, t) = 10 sin (50000t + 0.004x + 30) aty
t
where ay , denotes the unit vector in y direction. The wave is propagating with a
phase velocity.
(A) 5 # 10 4 m/s (B) − 3 # 108 m/s
(C) − 1.25 # 107 m/s (D) 3 # 108 m/s]

Brought to you by: Nodia and Company Visit us at: www.nodia.co.in


PUBLISHING FOR GATE
Page 12 GATE EC 2005 www.gatehelp.com

SOL 1.28 Hence (C) is correct option.


ω = 50, 000 and β =− 0.004
Phase Velocity is vP = ω = 5 # 10 - 3 = 1.25 # 107 m/s
4

β − 4 # 10
MCQ 1.29 Many circles are drawn in a Smith Chart used for transmission line calculations.
The circles shown in the figure represent

(A) Unit circles


(B) Constant resistance circles
(C) Constant reactance circles
(D) Constant reflection coefficient circles.
SOL 1.29 The given figure represent constant reactance circle.
Hence (C) is correct option.
MCQ 1.30 Refractive index of glass is 1.5. Find the wavelength of a beam of light with frequency
of 1014 Hz in glass. Assume velocity of light is 3 # 108 m/s in vacuum
(A) 3 μm (B) 3 mm
(C) 2 μm (D) 1 mm
SOL 1.30 Refractive index of glass μ = 1.5
Frequency f = 1014 Hz
c = 3 # 108 m/sec
8
λ = c = 3 # 10
14
= 3 # 10 - 6
f 10
wavelength in glass is
λg = α = 3 # 10 = 2 # 10 - 6 m
-6

μ 1.5
Hence (C) is correct option.

Question 31 - 80 Carry Two Marks Each

MCQ 1.31 In what range should Re (s) remain so that the Laplace transform of the function
e(a + 2) t + 5 exits.
(A) Re (s) > a + 2 (B) Re (s) > a + 7

Brought to you by: Nodia and Company Visit us at: www.nodia.co.in


PUBLISHING FOR GATE
Page 13 GATE EC 2005 www.gatehelp.com

(C) Re (s) < 2 (D) Re (s) > a + 5


SOL 1.31 Hence (A) is correct answer.
We have f (t) = e(a + 2) t + 5 = e5 .e(a + 2) t
Taking laplace transform we get
F (s) = e5 ;
s − (a + 2) E
1 Thus Re (s) > (a + 2)

−4 2
Given the matrix =
4 3G
MCQ 1.32 , the eigenvector is
3 4
(A) = G (B) = G
2 3
2 −1
(C) = G (D) = G
−1 2
SOL 1.32 Hence (C) is correct answer.
−4 2
A ==
4 3G
We have

Characteristic equation is
A − λI = 0
4−λ 2
or =0
4 3−λ
or (− 4 − λ)(3 − λ) − 8 =0
or − 12 + λ + λ2 − 8 =0
or λ2 + λ − 20 =0
or λ =− 5, 4 Eigen values
Eigen vector for λ =− 5
(A − λI) Xi =0
1 − (− 5) 2 x1 0
= 4 8 − 4G=x2 G
== G
0
1 2 x1 0
=0 0G=x G == G
0
R2 − 4R1
2

x1 + 2x2 = 0
Let − x1 = 2 & x2 =− 1,
2
X ==
−1G
Thus Eigen vector

2 − 0.1 1
a
MCQ 1.33 Let, A = = G and A - 1 = = 2 G. Then (a + b) =
0 3 0 b
(A) 7/20 (B) 3/20
(C) 19/60 (D) 11/20

Brought to you by: Nodia and Company Visit us at: www.nodia.co.in


PUBLISHING FOR GATE
Page 14 GATE EC 2005 www.gatehelp.com

SOL 1.33 We have


2 − 0.1 1
a
A == G and A - 1 = = 2 G
0 3 0 b
Now AA - 1 = I
2 − 0.1 12 a 1 0
or =0 3 G= G = =0 1 G
0 b
1 2a − 0.1b 1 0
or =0 3b G ==
0 1G
or 2a − 0.1 = 0 and 3b = 1
Thus solving above we have b = 1 and a = 1
3 60
Therefore a+b = 1 + 1 = 7
3 60 20
Hence (A) is correct option.
2
#0 exp c− x m dx is
1 3
MCQ 1.34 The value of the integral I =
2π 8
(A) 1 (B) π
(C) 2 (D) 2π
SOL 1.34 Gaussian PDF is
#
1 3 − (x − μ)2
f (x) = e 2σ2 dx for − 3 # x # 3
2π σ −3

# f (x) dx
3
and =1
−3

Substituting μ = 0 and σ = 2 in above we get


#
1 3 − x2
e dx = 18

2π 2 − 3
or
2π 2 0
#
1 2 3e− dx = 1 x2
8

#
1 3 − x2
or e dx = 18

2π 0
Hence (A) is correct option.
MCQ 1.35 The derivative of the symmetric function drawn in given figure will look like

Brought to you by: Nodia and Company Visit us at: www.nodia.co.in


PUBLISHING FOR GATE
Page 15 GATE EC 2005 www.gatehelp.com

SOL 1.35 For x > 0 the slope of given curve is negative. Only (C) satisfy this condition.
Hence (C) is correct answer.
MCQ 1.36 Match the following and choose the correct combination:
Group I Group 2
E. Newton-Raphson method 1. Solving nonlinear equations
F. Runge-kutta method 2. Solving linear simultaneous
equations
G. Simpson’s Rule 3. Solving ordinary differential
equations
H. Gauss elimination 4. Numerical integration
5. Interpolation
6. Calculation of Eigenvalues
(A) E − 6, F − 1, G − 5, H − 3 (B) E − 1, F − 6, G − 4, H − 3
(C) E − 1, F − 3, G − 4, H − 2 (D) E − 5, F − 3, G − 4, H − 1
SOL 1.36 Hence (C) is correct option.
Newton - Raphson " Method-Solving nonlinear eq.
Runge - kutta Method " Solving ordinary differential eq.
Simpson’s Rule " Numerical Integration
Gauss elimination " Solving linear simultaneous eq.
MCQ 1.37 Given an orthogonal matrix
R1 1 1 1 V
S W
S1 1 − 1 − 1W
A =S
1 − 1 0 0W
S W
S0 0 1 1 W
6AA @- 1 is T X
T

R1 V R1 V
S 4 0 0 0W S 2 0 0 0W
S 0 14 0 0 W S 0 12 0 0 W
(A) S 1 W (B) S 1 W
S0 0 2 0W S0 0 2 0 W
S 0 0 0 12 W S 0 0 0 12 W
Brought to you by: Nodia
T and Company
X T Visit
X us at: www.nodia.co.in
PUBLISHING FOR GATE
Page 16 GATE EC 2005 www.gatehelp.com

R1 R1 V
S 0 0 0 VW S 4 0 0 0W
S0 1 0 0W S0 1 0 0 W
(C) S (D) S 4 1 W
0 0 1 0W S0 0 4 0 W
S W
S0 0 0 1W S 0 0 0 14 W
T X T X

SOL 1.37 Hence (C) is correct option.


From orthogonal matrix
[AAT ] = I
Since the inverse of I is I , thus
[AAT ] −1 = I−1 = I
MCQ 1.38 For the circuit shown in the figure, the instantaneous current i1 (t) is

(A) 10 3 90c A (B) 10 3 − 90c A


2 2
(C) 5 60c A (D) 5 − 60c A
SOL 1.38 Applying KCL we get
i1 (t) + 5+0c = 10+60c
or i1 (t) = 10+60c − 5+0c = 5 + 5 3j − 5
or i1 (t) = 5 3 +90c = 10 3 +90c
2
Hence (A) is correct option.
MCQ 1.39 Impedance Z as shown in the given figure is

(A) j29 Ω (B) j9 Ω


(C) j19 Ω (D) j39 Ω

Brought to you by: Nodia and Company Visit us at: www.nodia.co.in


PUBLISHING FOR GATE
Page 17 GATE EC 2005 www.gatehelp.com

SOL 1.39 If L1 = j5Ω and L3 = j2Ω the mutual induction is subtractive because current enters
from dotted terminal of j2Ω coil and exit from dotted terminal of j5Ω. If L2 = j2Ω
and L3 = j2Ω the mutual induction is additive because current enters from dotted
terminal of both coil.
Thus Z = L1 − M13 + L2 + M23 + L3 − M31 + M32
= j5 + j10 + j2 + j10 + j2 − j10 + j10 = j9
Hence (B) is correct option.
MCQ 1.40 For the circuit shown in the figure, Thevenin’s voltage and Thevenin’s equivalent
resistance at terminals a − b is

(A) 5 V and 2 Ω (B) 7.5 V and 2.5 Ω


(C) 4 V and 2 Ω (D) 3 V and 2.5 Ω
SOL 1.40 Open circuit at terminal ab is shown below

Applying KCL at node we get


Vab + Vab − 10 = 1
5 5
or Vab = 7.5 = Vth
Short circuit at terminal ab is shown below

Short circuit current from terminal ab is


Isc = 1 + 10 = 3 A
5
Thus Rth = Vth = 7.5 = 2.5 Ω
Isc 3

Brought to you by: Nodia and Company Visit us at: www.nodia.co.in


PUBLISHING FOR GATE
Page 18 GATE EC 2005 www.gatehelp.com

Here current source being in series with dependent voltage source make it ineffective.
Hence (B) is correct option.
MCQ 1.41 If R1 = R2 = R4 = R and R3 = 1.1R in the bridge circuit shown in the figure, then
the reading in the ideal voltmeter connected between a and b is

(A) 0.238 V (B) 0.138 V


(C) − 0.238 V (D) 1 V
SOL 1.41 Here Va = 5 V because R1 = R2 and total voltage drop is 10 V.
Now Vb = R3 # 10 = 1.1 # 10 = 5.238 V
R3 + R4 2.1
V = Va − Vb = 5 − 5.238 =− 0.238 V
Hence (C) is correct option.
MCQ 1.42 The h parameters of the circuit shown in the figure are

0. 1 0. 1 10 − 1
(A) =
− 0. 1 0. 3 G
(B) =
1 0.05G
30 20 10 1
(C) = G (D) =
20 20 − 1 0.05G
SOL 1.42 For h parameters we have to write V1 and I2 in terms of I1 and V2 .
V1 = h11 I1 + h12 V2
I2 = h21 I1 + h22 V2
Applying KVL at input port
V1 = 10I1 + V2
Applying KCL at output port
V2 = I + I
1 2
20
or I2 =− I1 + V2
20

Brought to you by: Nodia and Company Visit us at: www.nodia.co.in


PUBLISHING FOR GATE
Page 19 GATE EC 2005 www.gatehelp.com

Thus from above equation we get


h11 h12 10 1
=h h G = =− 1 0.05G
12 22

Hence (D) is correct option.


MCQ 1.43 A square pulse of 3 volts amplitude is applied to C − R circuit shown in the figure.
The capacitor is initially uncharged. The output voltage V2 at time t = 2 sec is

(A) 3 V (B) − 3 V
(C) 4 V (D) − 4 V
SOL 1.43 Hence (B) is correct option.
Time constant RC = 0.1 # 10 - 6 # 103 = 10 - 4 sec
Since time constant RC is very small, so steady state will be reached in 2 sec. At
t = 2 sec the circuit is as shown in fig.

Vc = 3 V
V2 =− Vc =− 3 V
MCQ 1.44 A Silicon sample A is doped with 1018 atoms/cm 3 of boron. Another sample B
of identical dimension is doped with 1018 atoms/cm 3 phosphorus. The ratio of
electron to hole mobility is 3. The ratio of conductivity of the sample A to B is
(A) 3 (B) 1
3

(C) 2 (D) 3
3 2
SOL 1.44 Hence option (B) is correct.
σn = nqμn
σp = pqμp (n = p)
σp μ
= p =1
σn μn 3

Brought to you by: Nodia and Company Visit us at: www.nodia.co.in


PUBLISHING FOR GATE
Page 20 GATE EC 2005 www.gatehelp.com

MCQ 1.45 A Silicon PN junction diode under reverse bias has depletion region of width 10 μ
m. The relative permittivity of Silicon, εr = 11.7 and the permittivity of free space
ε0 = 8.85 # 10 - 12 F/m. The depletion capacitance of the diode per square meter is
(A) 100 μF (B) 10 μF
(C) 1 μF (D) 20 μF
SOL 1.45 Hence option (B) is correct.
C = ε0 εr A
d
or C = ε0 εr = 8.85 # 10−12 # 11.7 = 10.35 μ F
A d 10 # 10−6

MCQ 1.46 For an npn transistor connected as shown in figure VBE = 0.7 volts. Given that
reverse saturation current of the junction at room temperature 300 K is 10 - 13 A,
the emitter current is

(A) 30 mA (B) 39 mA
(C) 49 mA (D) 20 mA
SOL 1.46 Hence (C) is correct option.
IE = Is `e nV − 1j = 10 - 13 c
V BE

T
0.7 − 1m = 49 mA
-3
e1 # 26 # 10
MCQ 1.47 The voltage e0 is indicated in the figure has been measured by an ideal voltmeter.
Which of the following can be calculated ?

(A) Bias current of the inverting input only


(B) Bias current of the inverting and non-inverting inputs only
(C) Input offset current only
(D) Both the bias currents and the input offset current

Brought to you by: Nodia and Company Visit us at: www.nodia.co.in


PUBLISHING FOR GATE
Page 21 GATE EC 2005 www.gatehelp.com

SOL 1.47 The circuit is as shown below

Writing equation for I− have


e 0 − V− = I
-
1M
or e0 = I− (1M) + V− ...(1)
Writing equation for I+ we have
0 − V+
= I+
1M
or V+ = − I+ (1M) ...(2)
Since for ideal OPAMP V+ = V- , from (1) and (2) we have
e0 = I− (1M) − I + (1M)
= (I− − I+) (1M) = IOS (1M)
Thus if e0 has been measured, we can calculate input offset current IOS only.
Hence (C) is correct option.
MCQ 1.48 The Op-amp circuit shown in the figure is filter. The type of filter and its cut. Off
frequency are respectively

(A) high pass, 1000 rad/sec. (B) Low pass, 1000 rad/sec
(C) high pass, 1000 rad/sec (D) low pass, 10000 rad/sec
SOL 1.48 At low frequency capacitor is open circuit and voltage acr s non-inverting terminal
is zero. At high frequency capacitor act as short circuit and all input voltage appear
at non-inverting terminal. Thus, this is high pass circuit.
The frequency is given by
ω = 1 = 1 = 1000 rad/sec
RC 1 # 103 # 1 # 10 - 6
Hence (C) is correct option.

Brought to you by: Nodia and Company Visit us at: www.nodia.co.in


PUBLISHING FOR GATE
Page 22 GATE EC 2005 www.gatehelp.com

MCQ 1.49 In an ideal differential amplifier shown in the figure, a large value of (RE ).
(A) increase both the differential and common - mode gains.
(B) increases the common mode gain only.
(C) decreases the differential mode gain only.
(D) decreases the common mode gain only.
SOL 1.49 Common mode gain
ACM =− RC
2RE
And differential mode gain
ADM =− gm RC
Thus only common mode gain depends on RE and for large value of RE it decreases.
Hence (D) is correct option.
MCQ 1.50 For an n -channel MOSFET and its transfer curve shown in the figure, the threshold
voltage is

(A) 1 V and the device is in active region


(B) − 1 V and the device is in saturation region
(C) 1 V and the device is in saturation region
(D) − 1 V and the device is an active region
SOL 1.50 From the graph it can be easily seen that Vth = 1 V
Now VGS = 3 − 1 = 2 V
and VDS = 5 − 1 = 4 V
Since VDS > VGS $ VDS > VGS − Vth
Thus MOSFET is in saturation region.
Hence option (C) is correct.
MCQ 1.51 The circuit using a BJT with β = 50 and VBE = 0.7V is shown in the figure. The
base current IB and collector voltage by VC and respectively

Brought to you by: Nodia and Company Visit us at: www.nodia.co.in


PUBLISHING FOR GATE
Page 23 GATE EC 2005 www.gatehelp.com

(A) 43 μA and 11.4 Volts (B) 40 μA and 16 Volts


(C) 45 μA and 11 Volts (D) 50 μA and 10 Volts
SOL 1.51 The circuit under DC condition is shown in fig below

Applying KVL we have


VCC − RB IB − VBE − RE IE = 0
orVCC − RB IB − VBE − RE (β + 1) IB = 0 Since IE = IB + βIB
or IB = VCC − VBE
RB + (β + 1) RE
= 20 − 0.7 = 40μ A
430k + (50 + 1)1 k
Now IC = βIB = 50 # 40μ = 2 mA
VC = VCC − RC IC = 20 − 2m # 2k = 16 V
Hence (B) is correct option.
MCQ 1.52 The Zener diode in the regulator circuit shown in the figure has a Zener voltage of
5.8 volts and a zener knee current of 0.5 mA. The maximum load current drawn
from this current ensuring proper functioning over the input voltage range between
20 and 30 volts, is

Brought to you by: Nodia and Company Visit us at: www.nodia.co.in


PUBLISHING FOR GATE
Page 24 GATE EC 2005 www.gatehelp.com

(A) 23.7 mA (B) 14.2 mA


(C) 13.7 mA (D) 24.2 mA
SOL 1.52 The maximum load current will be at maximum input voltage i.e.
Vmax = 30 V i.e.
Vmax − VZ = I + I
L Z
1k
or 30 − 5.8 = IL = 0.5 m
1k
or IL = 24.2 − 0.5 = 23.7 mA
Hence (A) is correct option.
MCQ 1.53 The transistors used in a portion of the TTL gate show in the figure have β = 100 .
The base emitter voltage of is 0.7 V for a transistor in active region and 0.75 V for
a transistor in saturation. If the sink current I = 1 A and the output is at logic 0,
then the current IR will be equal to

(A) 0.65 mA (B) 0.70 mA


(C) 0.75 mA (D) 1.00 mA
SOL 1.53 The circuit is as shown below

If output is at logic 0, the we have V0 = 0 which signifies BJT Q3 is in saturation


and applying KVL we have
VBE3 = IR # 1k
or 0.75 = IR # 1k
or IR = 0.75 mA

Brought to you by: Nodia and Company Visit us at: www.nodia.co.in


PUBLISHING FOR GATE
Page 25 GATE EC 2005 www.gatehelp.com

Hence (C) is correct answer.


MCQ 1.54 The Boolean expression for the truth table shown is

(A) B (A + C)( A + C ) (B) B (A + C )( A + C)


(C) B (A + C )( A + C) (D) B (A + C)( A + C )
SOL 1.54 Hence (A) is correct answer.
We have f = ABC + ABC
= B (AC + AC ) = B (A + C)( A + C )
MCQ 1.55 Both transistors T1 and T2 show in the figure, have a β = 100 , threshold voltage of
1 Volts. The device parameters K1 and K2 of T1 and T2 are, respectively, 36 μA/V2
and 9 μA/V 2 . The output voltage Vo i s

(A) 1 V (B) 2 V
(C) 3 V (D) 4 V
SOL 1.55 Hence (D) is correct option.
MCQ 1.56 The present output Qn of an edge triggered JK flip-flop is logic 0. If J = 1, then
Qn + 1
(A) Cannot be determined (B) Will be logic 0
(C) will be logic 1 (D) will rave around
SOL 1.56 Characteristic equation for a jk flip-flop is written as
Qn + 1 = JQ n + K Qn
Where Qn is the present output
Qn + 1 is next output
So, Qn + 1 = 10 + K : 0 Qn = 0
Brought to you by: Nodia and Company Visit us at: www.nodia.co.in
PUBLISHING FOR GATE
Page 26 GATE EC 2005 www.gatehelp.com

Qn + 1 = 1
Hence (C) is correct answer.
MCQ 1.57 The given figure shows a ripple counter using positive edge triggered flip-flops. If
the present state of the counter is Q2 Q1 Q0 = 001 then is next state Q2 Q1 Q will be

(A) 010 (B) 111


(C) 100 (D) 101
SOL 1.57 Since T2 T1 T0 is at 111, at every clock Q2 Q1 Q0 will be changes. Ir present state is
011, the next state will be 100.
Hence (C) is correct answer.
MCQ 1.58 What memory address range is NOT represents by chip # 1 and chip # 2 in the
figure A0 to A15 in this figure are the address lines and CS means chip select.

(A) 0100 - 02FF (B) 1500 - 16FF


(C) F900 - FAFF (D) F800 - F9FF
SOL 1.58 Hence (D) is correct answer.
MCQ 1.59 The output y (t) of a linear time invariant system is related to its input x (t) by the
following equations
y (t)= 0.5x (t − td + T) + x (t − td ) + 0.5x (t − td + T)

Brought to you by: Nodia and Company Visit us at: www.nodia.co.in


PUBLISHING FOR GATE
Page 27 GATE EC 2005 www.gatehelp.com

The filter transfer function H (ω) of such a system is given by


(A) (1 + cos ωT) e−jωt d
(B) (1 + 0.5 cos ωT) e−jωt d

(C) (1 − cos ωT) e−jωt d


(D) (1 − 0.5 cos ωT) e−jωt
d

SOL 1.59 Hence (A) is correct answer.


y (t) = 0.5x (t − td + T) + x (t − td ) + 0.5x (t − td − T)
Taking Fourier transform we have
Y (ω) = 0.5e−jω (− t + T) X (ω) + e−jωt X (ω) + 0.5e−jω (− t − T) X (ω)
d d d

Y (ω)
or = e−jωt [0.5e jωT + 1 + 0.5e−jωT ]
d

X (ω)
= e−jωt [0.5 (e jωT + e−jωT ) + 1] = e−jωt [cos ωT + 1]
d d

Y (ω)
or H (ω) = = e−jωt (cos ωT + 1)
d

X (ω)

MCQ 1.60 Match the following and choose the correct combination.
Group 1
E. Continuous and aperiodic signal
F. Continuous and periodic signal
G. Discrete and aperiodic signal
H. Discrete and periodic signal
Group 2
1. Fourier representation is continuous and aperiodic
2. Fourier representation is discrete and aperiodic
3. Fourier representation is continuous and periodic
4. Fourier representation is discrete and periodic

(A) E − 3, F − 2, G − 4, H − 1
(B) E − 1, F − 3, G − 2, H − 4
(C) E − 1, F − 2, G − 3, H − 4
(D) E − 2, F − 1, G − 4, H − 3
SOL 1.60 For continuous and aperiodic signal Fourier representation is continuous and
aperiodic.
For continuous and periodic signal Fourier representation is discrete and aperiodic.
For discrete and aperiodic signal Fourier representation is continuous and periodic.
For discrete and periodic signal Fourier representation is discrete and periodic.
Hence (C) is correct answer.
MCQ 1.61 A signal x (n) = sin (ω0 n + φ) is the input to a linear time- invariant system having
a frequency response H (e jω). If the output of the system Ax (n − n0) then the most
general form of +H (e jω) will be

Brought to you by: Nodia and Company Visit us at: www.nodia.co.in


PUBLISHING FOR GATE
Page 28 GATE EC 2005 www.gatehelp.com

(A) − n0 ω0 + β for any arbitrary real


(B) − n0 ω0 + 2πk for any arbitrary integer k
(C) n0 ω0 + 2πk for any arbitrary integer k
(D) − n0 ω0 φ
SOL 1.61 Hence (B) is correct answer.
y (n) = Ax (n − no)
Taking Fourier transform
Y (e jω) = Ae−jω n X (e jω)
o o

Y (e jω) −jω n
or H (e jω) = jω = Ae
o o

X (e )

Thus +H (e ) =− ωo no
For LTI discrete time system phase and frequency of H (e jω) are periodic with
period 2π. So in general form
θ (ω) =− no ωo + 2πk
MCQ 1.62 For a signal x (t) the Fourier transform is X (f). Then the inverse Fourier transform
of X (3f + 2) is given by
j4πt
(A) 1 x` t j e j3πt (B) 1 x` t j e - 3
2 2 3 3
(C) 3x (3t) e−j4πt (D) x (3t + 2)
SOL 1.62 Hence (B) is correct answer.
F
x (t) X (f)
Using scaling we have
1 X f
a ca m
F
x (at)

Thus x b 1 f l
F
3X (3f)
3
Using shifting property we get
e−j2πf t x (t) = X (f + f0)
0

Thus 1 e−j 3 πt x b 1 t l
4 F
X (3f + 2)
3 3
e−j2π t x b 1 t l
F
3X (3 (f + 23 ))
2
3

b3 l
1 e−jπ t x 1 t
4
3
F
X [3 (f + 23 )]
3

MCQ 1.63 The polar diagram of a conditionally stable system for open loop gain K = 1 is
shown in the figure. The open loop transfer function of the system is known to be
stable. The closed loop system is stable for

Brought to you by: Nodia and Company Visit us at: www.nodia.co.in


PUBLISHING FOR GATE
Page 29 GATE EC 2005 www.gatehelp.com

(A) K < 5 and 1 < K < 1 (B) K < 1 and 1 < K < 5
2 8 8 2

(C) K < 1 and 5 < K (D) K > 1 and 5 > K


8 8
SOL 1.63 Hence (B) is correct option
MCQ 1.64 In the derivation of expression for peak percent overshoot
− πξ
Mp = exp e o # 100%
1 − ξ2
Which one of the following conditions is NOT required ?
(A) System is linear and time invariant
(B) The system transfer function has a pair of complex conjugate poles and no
zeroes.
(C) There is no transportation delay in the system.
(D) The system has zero initial conditions.
SOL 1.64 The peak percent overshoot is determined for LTI second order closed loop system
with zero initial condition. It’s transfer function is
T (s) = 2 ωn2
s + 2ξωn s + ωn2
Transfer function has a pair of complex conjugate poles and zeroes.
Hence (C) is correct option.
MCQ 1.65 Given the ideal operational amplifier circuit shown in the figure indicate the correct
transfer characteristics assuming ideal diodes with zero cut-in voltage.

Brought to you by: Nodia and Company Visit us at: www.nodia.co.in


PUBLISHING FOR GATE
Page 30 GATE EC 2005 www.gatehelp.com

SOL 1.65 Only one diode will be in ON conditions


When lower diode is in ON condition, then
Vu = 2k Vsat = 2 10 = 8 V
2.5k 2.5
when upper diode is in ON condition
Vu = 2k Vsat = 2 (− 10) =− 5 V
2.5k 4
Hence (B) is correct option.
MCQ 1.66 A ramp input applied to an unity feedback system results in 5% steady state error.
The type number and zero frequency gain of the system are respectively
(A) 1 and 20 (B) 0 and 20
(C) 0 and 1 (D) 1 and 1
20 20
SOL 1.66 For ramp input we have R (s) = 21
s
Now ess = lim sE (s)
s"0

Brought to you by: Nodia and Company Visit us at: www.nodia.co.in


PUBLISHING FOR GATE
Page 31 GATE EC 2005 www.gatehelp.com

R (s) 1
= lim s = lim
s"0 1 + G (s) s " 0 s + sG (s)
or ess = lim 1 = 5% = 1 Finite
s " 0 sG (s) 20
But kv = 1 = lim sG (s) = 20
ess s"0

kv is finite for type 1 system having ramp input.


Hence (A) is correct option.
MCQ 1.67 A double integrator plant G (s) = K/s2, H (s) = 1 is to be compensated to achieve
the damping ratio ζ = 0.5 and an undamped natural frequency, ωn = 5 rad/sec
which one of the following compensator Ge (s) will be suitable ?
(A) s + 3 (B) s + 99
s + 99 s+3

(C) s−6 (D) s − 6


s + 8.33 s
SOL 1.67 Hence (A) is correct option.
K (1 − s)
MCQ 1.68 An unity feedback system is given as G (s) = .
s (s + 3)
Indicate the correct root locus diagram.

SOL 1.68 Hence (A) is correct option.


MCQ 1.69 A MOS capacitor made using p type substrate is in the accumulation mode. The
dominant charge in the channel is due to the presence of
(A) holes (B) electrons
(C) positively charged icons (D) negatively charged ions
SOL 1.69 In accumulation mode for NMOS having p −substrate, when positive voltage is
applied at the gate, this will induce negative charge near p − type surface beneath
the gate. When VGS is made sufficiently large, an inversion of electrons is formed
and this in effect forms and n − channel.
Hence option (B) is correct.

Brought to you by: Nodia and Company Visit us at: www.nodia.co.in


PUBLISHING FOR GATE
Page 32 GATE EC 2005 www.gatehelp.com

MCQ 1.70 A device with input X (t) and output y (t) is characterized by: Y (t) = x2 (t). An FM
signal with frequency deviation of 90 kHz and modulating signal bandwidth of 5
kHz is applied to this device. The bandwidth of the output signal is
(A) 370 kHz (B) 190 kHz
(C) 380 kHz (D) 95 kHz
SOL 1.70 Let x (t) be the input signal where
x (t) = cos (cos t + β1 cos ωm t)
cos (2ωc t + 2β1 cos ωm t)
y (t) = x2 (t) = 1 +
2 2
3f
Here β = 2β1 and β1 = = 90 = 18
fm 5
BW = 2 (β + 1) fm = 2 (2 # 18 + 1) # 5 = 370 kHz
Hence (A) is correct option.
MCQ 1.71 A signal as shown in the figure is applied to a matched filter. Which of the following
does represent the output of this matched filter ?

SOL 1.71 The transfer function of matched filter is


h (t) = x (t − t) = x (2 − t)
The output of matched filter is the convolution of x (t) and h (t) as shown below

Brought to you by: Nodia and Company Visit us at: www.nodia.co.in


PUBLISHING FOR GATE
Page 33 GATE EC 2005 www.gatehelp.com

Hence (C) is correct option.


MCQ 1.72 Noise with uniform power spectral density of N0 W/Hz is passed though a filter
H (ω) = 2 exp (− jωtd ) followed by an ideal pass filter of bandwidth B Hz. The
output noise power in Watts is
(A) 2N0 B (B) 4N0 B
(C) 8N0 B (D) 16N0 B
SOL 1.72 Hence (B) is correct option.
We have H (f) = 2e - jωt
d

H (f) = 2
G0 (f) = H (f) 2 Gi (f)
= 4No W/Hz
The noise power is = 4No # B

MCQ 1.73 A carrier is phase modulated (PM) with frequency deviation of 10 kHz by a single
tone frequency of 1 kHz. If the single tone frequency is increased to 2 kHz, assuming
that phase deviation remains unchanged, the bandwidth of the PM signal is
(A) 21 kHz (B) 22 kHz
(C) 42 kHz (D) 44 kHz
SOL 1.73 The phase deviation is
3f
β = = 10 = 10
fm 1
If phase deviation remain same and modulating frequency is changed
BW = 2 (β + 1) fm' = 2 (10 + 1) 2 = 44 kHz
Hence (D) is correct option.
MCQ 1.74 An output of a communication channel is a random variable v with the probability
density function as shown in the figure. The mean square value of v is

Brought to you by: Nodia and Company Visit us at: www.nodia.co.in


PUBLISHING FOR GATE
Page 34 GATE EC 2005 www.gatehelp.com

(A) 4 (B) 6
(C) 8 (D) 9
SOL 1.74 As the area under pdf curve must be unity
1 (4 # k) = 1 $ k = 1
2 2
Now mean square value is
#- 3
+3
σv2 = v2 p (v) dv

#0
4
= v2 ` v j dv as p (v) = 1 v
8 8
v3
#0
4
= c 8 m dv = 8

Hence (C) is correct option.


MCQ 1.75 Which one of the following does represent the electric field lines for the mode in the
cross-section of a hollow rectangular metallic waveguide ?

SOL 1.75 Hence (D) is correct option.


MCQ 1.76 Characteristic impedance of a transmission line is 50 Ω. Input impedance of the
open-circuited line when the transmission line a short circuited, then value of the
input impedance will be.
(A) 50 Ω (B) 100 + j150Ω
(C) 7.69 + j11.54Ω (D) 7.69 − j11.54Ω
SOL 1.76 Hence (D) is correct option
Zo2 = ZOC .ZSC
2
ZZC = Zo = 50 # 50 = 50
ZOC 100 + j150 2 + 3j

Brought to you by: Nodia and Company Visit us at: www.nodia.co.in


PUBLISHING FOR GATE
Page 35 GATE EC 2005 www.gatehelp.com

50 (2 − 3j)
= = 7.69 − 11.54j
13
Hence (D) is correct option

MCQ 1.77 Two identical and parallel dipole antennas are kept apart by a distance of λ in the
4
H - plane. They are fed with equal currents but the right most antenna has a phase
shift of + 90c. The radiation pattern is given as.

SOL 1.77 The array factor is


βd sin θ + α
A = cos b l
2
Here β = 2π , d = λ and α = 90c
λ 4
2π λ
sin θ + π2 π π
Thus A = cos c λ 4 m = cos ` sin θ + j
2 4 2
The option (A) satisfy this equation.

Common Data Questions 78, 79 and 80 :


Given, rd = 20kΩ , IDSS = 10 mA, Vp =− 8 V

Brought to you by: Nodia and Company Visit us at: www.nodia.co.in


PUBLISHING FOR GATE
Page 36 GATE EC 2005 www.gatehelp.com

MCQ 1.78 Zi and Z0 of the circuit are respectively


(A) 2 MΩ and 2 kΩ (B) 2 MΩ and 20 kΩ
11

(C) infinity and 2 MΩ (D) infinity and 20 kΩ


11
SOL 1.78 The small signal model is as shown below

From the figure we have


Zin = 2 MΩ
and Z0 = rd RD = 20k 2k = 20 kΩ
11
Hence (B) is correct option.
MCQ 1.79 ID and VDS under DC conditions are respectively
(A) 5.625 mA and 8.75 V (B) 1.875 mA and 5.00 V
(C) 4.500 mA and 11.00 V (D) 6.250 mA and 7.50 V
SOL 1.79 The circuit in DC condition is shown below

Since the FET has high input resistance, gate current can be neglect and we get
VGS =− 2 V
Since VP < VGS < 0 , FET is operating in active region
(− 2) 2
ID = IDSS c1 − VGS m = 10 c1 −
(− 8) m
2
Now = 5.625 mA
VP
Now VDS = VDD − ID RD = 20 − 5.625 m # 2 k = 8.75 V
Hence (A) is correct option.
MCQ 1.80 Transconductance in milli-Siemens (mS) and voltage gain of the amplifier are

Brought to you by: Nodia and Company Visit us at: www.nodia.co.in


PUBLISHING FOR GATE
Page 37 GATE EC 2005 www.gatehelp.com

respectively
(A) 1.875 mS and 3.41 (B) 1.875 ms and -3.41
(C) 3.3 mS and -6 (D) 3.3 mS and 6
SOL 1.80 The transconductance is
gm = 2
VP ID IDSS
or, = 2 5.625mA # 10mA = 1.875 mS
8
The gain is A =− gm (rd RD)
So, = 1.875ms # 20 K =− 3.41
11
Hence (B) is correct option.

Linked Answer Questions : Q. 81 to 90 Carry Two Marks Each


Consider an 8085 microprocessor system.
MCQ 1.81 The following program starts at location 0100H.
LXI SP, OOFF
LXI H, 0701
MVI A, 20H
SUB M
The content of accumulator when the program counter reaches 0109 H is
(A) 20 H (B) 02 H
(C) 00 H (D) FF H
SOL 1.81 0100H LXI SP, 00FF ; Load SP with 00FFG
0103H LXI H, 0701 ; Load HL with 0107H
0106H MVI A, 20H ; Move A with 20 H
0108 H SUB M ; Subtract the contents of memory
; location whose address is stored in HL
; from the A and store in A
0109H ORI 40H ; 40H OR [A] and store in A
010BH ADD M ; Add the contents of memeory location
; whose address is stored in HL to A
; and store in A
HL contains 0107H and contents of 0107H is 20H
Thus after execution of SUB the data of A is 20H - 20H = 00
Hence (C) is correct answer.
MCQ 1.82 If in addition following code exists from 019H onwards,
ORI 40 H
ADD M

Brought to you by: Nodia and Company Visit us at: www.nodia.co.in


PUBLISHING FOR GATE
Page 38 GATE EC 2005 www.gatehelp.com

What will be the result in the accumulator after the last instruction is executed ?
(A) 40 H (B) 20 H
(C) 60 H (D) 42 H
SOL 1.82 Before ORI instruction the contents of A is 00H. On execution the ORI 40H the
contents of A will be 40H
00H = 00000000
40H = 01000000
ORI 01000000
After ADD instruction the contents of memory location whose address is stored in
HL will be added to and will be stored in A
40H + 20 H = 60 H
Hence (C) is correct answer.

Statement for Linked Answer Question 83 and 84 :


The open loop transfer function of a unity feedback system is given by
−2s
G (s) = 3e
s (s + 2)
MCQ 1.83 The gain and phase crossover frequencies in rad/sec are, respectively
(A) 0.632 and 1.26 (B) 0.632 and 0.485
(C) 0.485 and 0.632 (D) 1.26 and 0.632
SOL 1.83 Hence (D) is correct option
−2s
G (s) = 3e
s (s + 2)
or G (jω) = 3e−2jω
jω (jω + 2)
G (jω) = 3
ω ω2 + 4
Let at frequency ωg the gain is 1. Thus
3 =1
ωg (ωg2 + 4)
or ωg4 + 4ωg2 − 9 = 0
or ωg2 = 1.606
or ωg = 1.26 rad/sec
Now +G (jω) =− 2ω − π − tan−1 ω
2 2
Let at frequency ωφ we have +GH =− 180c
ω
− π =− 2ωφ − π − tan−1 φ
2 2
ω
or 2ωφ + tan−1 φ = π
2 2

Brought to you by: Nodia and Company Visit us at: www.nodia.co.in


PUBLISHING FOR GATE
Page 39 GATE EC 2005 www.gatehelp.com

ωφ 1 ωφ 3
or 2ωφ + c − ` jm = π
2 3 2 2
5ωφ ωφ3
or − =π
2 24 2

5ωφ
2 2
or ωφ = 0.63 rad/sec
MCQ 1.84 Based on the above results, the gain and phase margins of the system will be
(A) −7.09 dB and 87.5c (B) 7.09 dB and 87.5c
(C) 7.09 dB and − 87.5c (D) − 7.09 and − 87.5c
SOL 1.84 The gain at phase crossover frequency ωφ is
G (jωg) = 3 = 3
1

ωφ (ωφ + 4)
2
0.63 (0.632 + 4) 2

or G (jωg) = 2.27
G.M. =− 20 log G (jωg)
− 20 log 2.26 =− 7.08 dB
Since G.M. is negative system is unstable.
The phase at gain corss over frequency is
ω
+G (jωg) =− 2ωg − π − tan−1 g
2 2
=− 2 # 1.26 − π − tan−1 1.26
2 2
or =− 4.65 rad or − 266.5c
PM = 180c + +G (jωg) = 180c − 266.5c =− 86.5c
Hence (D) is correct option.

Common Data for Question 85 and 86 :


Asymmetric three-level midtread quantizer is to be designed assuming equiprobable
occurrence of all quantization levels.

MCQ 1.85 If the probability density function is divide into three regions as shown in the
figure, the value of a in the figure is
(A) 1 (B) 2
3 3
Brought to you by: Nodia and Company Visit us at: www.nodia.co.in
PUBLISHING FOR GATE
Page 40 GATE EC 2005 www.gatehelp.com

(C) 1 (D) 1
2 4
SOL 1.85 As the area under pdf curve must be unity and all three region are equivaprobable.
Thus are under each region must be 13 .
2a # 1 = 1 $ a = 2
4 3 3
Hence (B) is correct option.
MCQ 1.86 The quantization noise power for the quantization region between − a and + a in
the figure is
(A) 4 (B) 1
81 9

(C) 5 (D) 2
81 81
SOL 1.86 Hence (A) is correct option.
3 a 3
#- a #0
+a
x $ 1 dx = 1 ; x E = a
a 2
Nq = x2 p (x) dx = 2
4 2 3 0 6
Substituting a = 2 we have
3
Nq = 4
81

Statement of Linked Answer Questions 87 & 88 :


Voltage standing wave pattern in a lossless transmission line with characteristic
impedance 50 and a resistive load is shown in the figure.

MCQ 1.87 The value of the load resistance is


(A) 50 Ω (B) 200 Ω
(C) 12.5 Ω (D) 0
SOL 1.87 From the diagram, VSWR is
s = Vmax = 4 = 4
Vmin 1
When minima is at load ZO = s.ZL
or ZL = Zo = 50 = 12.5Ω
s 4

Brought to you by: Nodia and Company Visit us at: www.nodia.co.in


PUBLISHING FOR GATE
Page 41 GATE EC 2005 www.gatehelp.com

Hence (C) is correct option.


MCQ 1.88 reflection coefficient is given by
(A)− 0.6 (B) − 1
(C) 0.6 (D) 0
SOL 1.88 The reflection coefficient is
Γ = ZL − ZO = 12.5 − 50 =− 0.6
ZL + ZO 125. + 50
Hence (A) is correct option.

Statement of linked answer question 89 and89:


A sequence x (n) has non-zero values as shown in the figure.

x ( n2 − 1), For n even


MCQ 1.89 The sequence y (n) = * will be
0, For n odd

Brought to you by: Nodia and Company Visit us at: www.nodia.co.in


PUBLISHING FOR GATE
Page 42 GATE EC 2005 www.gatehelp.com

SOL 1.89 Thus (A) is correct option.


From x (n) = [ 12 , 1, 2, 1, 1, 12 ]
y (n) = x ^ n2 − 1h, n even
= 0 , for n odd
n =− 2 , y (− 2) = x ( −22 − 1) = x (− 2) = 12
n =− 1, y (− 1) = 0
n = 0, y (0) = x ( 20 − 1) = x (− 1) = 1
n = 1, y (1) = 0
n=2 y (2) = x ( 22 − 1) = x (0) = 2
n = 3, y (3) = 0
n=4 y (4) = x ( 24 − 1) = x (1) = 1
n = 5, y (5) = 0
n=6 y (6) = x ( 26 − 1) = x (2) = 12
Hence y (n) = 1 δ (n + 2) + δ (n) + 2δ (n − 2) + δ (n − 4)+ 1 δ (n − 6)
2 2
MCQ 1.90 The Fourier transform of y (2n) will be
(A) e−j2ω [cos 4ω + 2 cos 2ω + 2] (B) cos 2ω + 2 cos ω + 2

Brought to you by: Nodia and Company Visit us at: www.nodia.co.in


PUBLISHING FOR GATE
Page 43 GATE EC 2005 www.gatehelp.com

(C) e−jω [cos 2ω + 2 cos ω + 2] (D) e−j2ω [cos 2ω + 2 cos + 2]


SOL 1.90 Here y (n) is scaled and shifted version of x (n) and again y (2n) is scaled version of
y (n) giving
z (n) = y (2n) = x (n − 1)
= 1 δ (n + 1) + δ (n) + 2δ (n − 1) + δ (n − 2) + 1 δ (n − 3)
2 2
Taking Fourier transform.
Z (e jω) = 1 e jω + 1 + 2e−jω + e−2jω + 1 e−3jω
2 2
= e−jω b 1 e2jω + e jω + 2 + e−jω + 1 e−2jω l
2 2
2jω
= e−jω b e + e−2jω + e jω + 2 + e−jω
2 l

or Z (e jω) = e−jω [cos 2ω + 2 cos ω + 2]


Hence (C) is correct answer

Answer Sheet
1. (B) 19. (A) 37. (C) 55. (D) 73. (D)
2. (C) 20. (D) 38. (A) 56. (C) 74. (C)
3. (D) 21. (D) 39. (B) 57. (C) 75. (D)
4. (B) 22. (D) 40. (B) 58. (D) 76. (D)
5. (A) 23. (A) 41. (C) 59. (A) 77. (A)
6. (C) 24. (C) 42. (D) 60. (C) 78. (B)
7. (C) 25. (C) 43. (B) 61. (B) 79. (A)
8. (B) 26. (D) 44. (B) 62. (B) 80. (B)
9. (B) 27. (A) 45. (B) 63. (B) 81. (C)
10. (C) 28. (C) 46. (C) 64. (C) 82. (C)
11. (C) 29. (C) 47. (C) 65. (B) 83. (D)
12. (B) 30. (C) 48. (C) 66. (A) 84. (D)
13. (A) 31. (A) 49. (D) 67. (A) 85. (B)
14. (D) 32. (C) 50. (C) 68. (A) 86. (A)
15. (B) 33. (A) 51. (B) 69. (B) 87. (C)
16. (C) 34. (A) 52. (A) 70 (A) 88. (A)
17. (B) 35. (C) 53. (C) 71 (C) 89. (A)
18. (B) 36. (C) 54. (A) 72 (B) 90. (C)

Brought to you by: Nodia and Company Visit us at: www.nodia.co.in


PUBLISHING FOR GATE
GATE EC
2007

Q.1 to Q.20 carry one mark each

MCQ 1.1 If E denotes expectation, the variance of a random variable X is given by


(A) E [X2] − E2 [X] (B) E [X2] + E2 [X]
(C) E [X2] (D) E2 [X]
SOL 1.1 The variance of a random variable x is given by
E [X2] − E2 [X]
Hence (A) is correct option.
MCQ 1.2 The following plot shows a function which varies linearly with x . The value of the
#1 ydx
2
integral I = is

(A) 1.0 (B) 2.5


(C) 4.0 (D) 5.0
SOL 1.2 The given plot is straight line whose equation is
x +y =1
−1 1
or y = x+1
#1 ydx #1 (x + 1) dx
2 2
Now I = =
(x + 1) 2 2
=; E = − = 2.5
9 4
2 2 2
Hence (B) is correct answer.
MCQ 1.3 For x << 1, coth (x) can be approximated as

Brought to you by: Nodia and Company Visit us at: www.nodia.co.in


PUBLISHING FOR GATE
Page 2 GATE EC 2007 www.gatehelp.com

(A) x (B) x2
(C) 1 (D) 12
x x
SOL 1.3 Hence (C) is correct answer.
coth x = cosh x
sinh x
as x << 1, cosh x . 1 and sinh x . x
Thus coth x . 1
x

sin b θ l
2
MCQ 1.4 lim is
θ"0 θ
(A) 0.5 (B) 1
(C) 2 (D) not defined
SOL 1.4 Hence (A) is correct answer.
sin ^ θ2 h sin ^ θ2 h 1 lim sin ^ 2 h = 1 = 0.5
θ
lim = lim =
θ"0 θ θ " 0 2^ θ h 2 θ " 0 ^ θ2 h 2
2

MCQ 1.5 Which one of following functions is strictly bounded?


(A) 1/x2 (B) ex
2

(C) x2 (D) e - x
SOL 1.5 Hence (D) is correct answer.
We have, lim 12 = 3
x"0 x

lim x2 = 3
x"3

lim e - x = 3
x"3

lim e - x = 0
2

x"3

lim e - x = 1
2 2

Thus e - x is strictly bounded.


x"0

MCQ 1.6 For the function e - x , the linear approximation around x = 2 is


(A) (3 − x) e - 2 (B) 1 − x
(C) 63 + 3 2 − (1 − 2 ) x @e - 2 (D) e - 2
SOL 1.6 Hence (A) is correct answer.
We have f (x) = e - x = e - (x - 2) - 2 = e - (x - 2) e - 2
(x − 2) 2
= ;1 − (x − 2) + ...E e - 2
2!
= 61 − (x − 2)@ e - 2 Neglecting higher powers

Brought to you by: Nodia and Company Visit us at: www.nodia.co.in


PUBLISHING FOR GATE
Page 3 GATE EC 2007 www.gatehelp.com

= (3 − x) e - 2
MCQ 1.7 An independent voltage source in series with an impedance Zs = Rs + jXs delivers
a maximum average power to a load impedance ZL when
(A) ZL = Rs + jXs (B) ZL = Rs
(C) ZL = jXs (D) ZL = Rs − jXs
SOL 1.7 According to maximum Power Transform Theorem
ZL = Zs* = (Rs − jXs)
Hence (D) is correct option.
MCQ 1.8 The RC circuit shown in the figure is

(A) a low-pass filter (B) a high-pass filter


(C) a band-pass filter (D) a band-reject filter
SOL 1.8 At ω " 3 , capacitor acts as short circuited and circuit acts as shown in fig below

Here we get V0 = 0
Vi

At ω " 0 , capacitor acts as open circuited and circuit look like as shown in fig
below

Here we get also V0 = 0


Vi
So frequency response of the circuit is as shown in fig and circuit is a Band pass
filter.

Brought to you by: Nodia and Company Visit us at: www.nodia.co.in


PUBLISHING FOR GATE
Page 4 GATE EC 2007 www.gatehelp.com

Hence (C) is correct option.


MCQ 1.9 The electron and hole concentrations in an intrinsic semiconductor are ni per cm3
at 300 K. Now, if acceptor impurities are introduced with a concentration of NA per
cm3 (where NA >> ni , the electron concentration per cm3 at 300 K will be)
(A) ni (B) ni + NA
2
(C) NA − ni (D) ni
NA
SOL 1.9 As per mass action law
np = ni2
If acceptor impurities are introduces
p = NA
Thus nNA = ni2
2
or n = ni
NA
Hence option (D) is correct.
MCQ 1.10 In a p+ n junction diode under reverse biased the magnitude of electric field is
maximum at
(A) the edge of the depletion region on the p-side
(B) the edge of the depletion region on the n -side
(C) the p+ n junction
(D) the centre of the depletion region on the n -side
SOL 1.10 The electric field has the maximum value at the junction of p+ n .
Hence option (C) is correct.
MCQ 1.11 The correct full wave rectifier circuit is

Brought to you by: Nodia and Company Visit us at: www.nodia.co.in


PUBLISHING FOR GATE
Page 5 GATE EC 2007 www.gatehelp.com

SOL 1.11 The circuit shown in (C) is correct full wave rectifier circuit.

Hence (C) is correct option.


MCQ 1.12 In a transconductance amplifier, it is desirable to have
(A) a large input resistance and a large output resistance
(B) a large input resistance and a small output resistance
(C) a small input resistance and a large output resistance
(D) a small input resistance and a small output resistance
SOL 1.12 In the transconductance amplifier it is desirable to have large input resistance and
large output resistance.
Hence (A) is correct option.
MCQ 1.13 X = 01110 and Y = 11001 are two 5-bit binary numbers represented in two’s
complement format. The sum of X and Y represented in two’s complement format
using 6 bits is
(A) 100111 (B) 0010000
(C) 000111 (D) 101001
SOL 1.13 MSB of Y is 1, thus it is negative number and X is positive number
Now we have X = 01110 = (14) 10
and Y = 11001 = (− 7) 10
X + Y = (14) + (− 7) = 7
In signed two’s complements from 7 is
(7) 10 = 000111
Brought to you by: Nodia and Company Visit us at: www.nodia.co.in
PUBLISHING FOR GATE
Page 6 GATE EC 2007 www.gatehelp.com

Hence (C) is correct answer.


MCQ 1.14 The Boolean function Y = AB + CD is to be realized using only 2 - input NAND
gates. The minimum number of gates required is
(A) 2 (B) 3
(C) 4 (D) 5
SOL 1.14 Hence (B) is correct answer.
Y = AB + CD = AB .CD
This is SOP form and we require only 3 NAND gate

MCQ 1.15 If the closed-loop transfer function of a control system is given as T (s) s−5
(s + 2)( s + 3)
, then It is
(A) an unstable system (B) an uncontrollable system
(C) a minimum phase system (D) a non-minimum phase system
SOL 1.15 In a minimum phase system, all the poles as well as zeros are on the left half of the
s −plane. In given system as there is right half zero (s = 5), the system is a non-
minimum phase system.
Hence (D) is correct option.

MCQ 1.16 If the Laplace transform of a signal Y (s) = 1 , then its final value is
s (s − 1)
(A) − 1 (B) 0
(C) 1 (D) Unbounded
SOL 1.16 Hence (D) is correct answer.
Y (s) = 1
s (s − 1)
Final value theorem is applicable only when all poles of system lies in left half of S
-plane. Here s = 1 is right s −plane pole. Thus it is unbounded.
MCQ 1.17 If R (τ) is the auto correlation function of a real, wide-sense stationary random
process, then which of the following is NOT true
(A) R (τ) = R (− τ)
(B) R (τ) # R (0)
(C) R (τ) =− R (− τ)
(D) The mean square value of the process is R (0)
SOL 1.17 Autocorrelation is even function.
Hence (C) is correct option
MCQ 1.18 If S (f) is the power spectral density of a real, wide-sense stationary random process,
then which of the following is ALWAYS true?
(A) S (0) # S (f) (B) S (f) $ 0

Brought to you by: Nodia and Company Visit us at: www.nodia.co.in


PUBLISHING FOR GATE
Page 7 GATE EC 2007 www.gatehelp.com

#- 3 S (f) df = 0
3
(C) S (− f) =− S (f) (D)
SOL 1.18 Power spectral density is non negative. Thus it is always zero or greater than zero.
Hence (B) is correct option.
MCQ 1.19 A plane wave of wavelength λ is traveling in a direction making an angle 30c with
positive x − axis and 90c with positive y − axis. The E field of the plane wave can
be represented as (E0 is constant)
3 π x− π z π 3πz
t 0 e j c ωt −
(A) E = yE λ λ m t 0 e jc ωt − λ x −
(B) E = yE λ m

3 π x+ π z π 3πz
t 0 e jc ωt +
(C) E = yE λ λ m t 0 e jc ωt − λ x +
(D) E = yE λ m

SOL 1.19 Hence (A) is correct option.


γ = β cos 30cx ! β sin 30cy
= 2π 3 x ! 2π 1 y
λ 2 λ 2
= π 3 x! πy
λ λ
π 3 x! π y
E = ay E0 e j (ωt - γ) = ay E0 e j;ωt - c λ λ mE

MCQ 1.20 If C is code curve enclosing a surface S , then magnetic field intensity H , the
current density j and the electric flux density D are related by
(A) ##
S
H $ ds =
c
##
c j + 2t m $ d t
2D (B) H $ d l =
S S
#
c j + 2t m $ dS
2D
##
(C) ##S H $ dS = #C c j + 22Dt m $ d t (D) #C H $ d l # = ##S c j + 22Dt m $ ds
c

SOL 1.20 Hence (D) is correct option.


4# H = J + 2D Maxwell Equations
2t
## 4# H $ ds = ##
`J + 2t j .ds
2D Integral form
s s
#
H $ dl = ##
`J + 2t j .ds
2D Stokes Theorem
s

Q.21 to Q.75 carry two marks each.

MCQ 1.21 It is given that X1, X2 ...XM at M non-zero, orthogonal vectors. The dimension of
the vector space spanned by the 2M vectors X1, X2,... XM , − X1, − X2,... − XM is
(A) 2M (B) M + 1
(C) M
(D) dependent on the choice of X1, X2,... XM
SOL 1.21 For two orthogonal vectors, we require two dimensions to define them and similarly

Brought to you by: Nodia and Company Visit us at: www.nodia.co.in


PUBLISHING FOR GATE
Page 8 GATE EC 2007 www.gatehelp.com

for three orthogonal vector we require three dimensions to define them. 2M vectors
are basically M orthogonal vector and we require M dimensions to define them.
Hence (C) is correct answer.
MCQ 1.22 Consider the function f (x) = x2 − x − 2 . The maximum value of f (x) in the closed
interval [− 4, 4] is
(A) 18 (B) 10
(C) − 225 (D) indeterminate
SOL 1.22 We have
f (x) = x2 − x + 2
f'( x) = 2x − 1 = 0 " x = 1
2
f"( x) = 2
Since f"( x) = 2 > 0 , thus x = 1 is minimum point. The maximum value in closed
2
interval 6− 4, 4@ will be at x =− 4 or x = 4
Now maximum value
= max [f (− 4), f (4)]
= max (18, 10)
= 18
Hence (A) is correct answer.
MCQ 1.23 An examination consists of two papers, Paper 1 and Paper 2. The probability of
failing in Paper 1 is 0.3 and that in Paper 2 is 0.2. Given that a student has failed
in Paper 2, the probability of failing in Paper 1 is 0.6. The probability of a student
failing in both the papers is
(A) 0.5 (B) 0.18
(C) 0.12 (D) 0.06
SOL 1.23 Hence (C) is correct answer.
Probability of failing in paper 1 is P (A) = 0.3
Possibility of failing in Paper 2 is P (B) = 0.2
Probability of failing in paper 1, when
student has failed in paper 2 is P ^ BA h = 0.6
We know that
(P + B)
Pb A l =
B P (B)
or P (A + B) = P (B) P b A l = 0.6 # 0.2 = 0.12
B

d2 y
MCQ 1.24 The solution of the differential equation k2 2 = y − y2 under the boundary
dx
conditions
(i) y = y1 at x = 0 and

Brought to you by: Nodia and Company Visit us at: www.nodia.co.in


PUBLISHING FOR GATE
Page 9 GATE EC 2007 www.gatehelp.com

(ii) y = y2 at x = 3 , where k, y1 and y2 are constants, is

(A) y = (y1 − y2) exp a− x2 k + y2 (B) y = (y2 − y1) exp a− x k + y1


k k

(C) y = ^y1 − y2h sinh a x k + y1 (D) y = ^y1 − y2h exp a− x k + y2


k k
SOL 1.24 Hence (D) is correct answer.
d2 y
We have k2 2 = y − y2
dx
2
d y y y
or 2
− 2 =− 22
dx k k
A.E. D − 2 =0
2 1
k
or D =! 1
k
x x

C.F. = C1 e - + C2 e
k k

− y22
c 2 m = y2
P.I. = 1
D2 − 12 k
Thus solution is k
x x

y = C1 e - + C2 e + y2
k k

From y (0) = y1 we get


C1 + C2 = y1 − y2
From y (3) = y2 we get that C1 must be zero.
Thus C2 = y1 − y2
x

y = (y1 − y2) e - + y2
k

MCQ 1.25 The equation x3 − x2 + 4x − 4 = 0 is to be solved using the Newton - Raphson


method. If x = 2 is taken as the initial approximation of the solution, then next
approximation using this method will be
(A) 2/3 (B) 4/3
(C) 1 (D) 3/2
SOL 1.25 We have
f (x) = x3 − x2 + 4x − 4
f'( x) = 3x2 − 2x + 4
Taking x0 = 2 in Newton-Raphosn method
f (x0) 23 − 22 + 4 (2) − 4
x1 = x0 − = 2− =4
f'( x0) 3 (2) − 2 (2) + 4
2 3
Hence (B) is correct answer.
MCQ 1.26 Three functions f1 (t), f2 (t) and f3 (t) which are zero outside the interval [0, T] are
shown in the figure. Which of the following statements is correct?

Brought to you by: Nodia and Company Visit us at: www.nodia.co.in


PUBLISHING FOR GATE
Page 10 GATE EC 2007 www.gatehelp.com

(A) f1 (t) and f2 (t) are orthogonal (B) f1 (t) and f3 (t) are orthogonal
(C) f2 (t) and f3 (t) are orthogonal (D) f1 (t) and f2 (t) are orthonormal
SOL 1.26 For two orthogonal signal f (x) and g (x)
#- 3
+3
f (x) g (x) dx =0

i.e. common area between f (x) and g (x) is zero.


Hence (C) is correct options.
MCQ 1.27 If the semi-circular control D of radius 2 is as shown in the figure, then the value
of the integral 1
(s − 1)
2 #
ds is
D

(A) jπ (B) − jπ
(C) − π (D) π
SOL 1.27 We know that

s
#
2
1 ds = 2πj
− 1
[sum of residues]
D
Singular points are at s = ! 1 but only s =+ 1 lies inside the given contour, Thus
Residue at s =+ 1 is

Brought to you by: Nodia and Company Visit us at: www.nodia.co.in


PUBLISHING FOR GATE
Page 11 GATE EC 2007 www.gatehelp.com

lim (s − 1) f (s) = lim (s − 1) 1 =1


s"1 s"1 s2 − 1 2
# s2 −1 1 ds = 2πj` 12 j = πj
D
Hence (A) is correct answer.
MCQ 1.28 Two series resonant filters are as shown in the figure. Let the 3-dB bandwidth of
Filter 1 be B1 and that of Filter 2 be B2 . the value B1 is
B2

(A) 4 (B) 1
(C) 1/2 (D) 1/4

SOL 1.28 We know that bandwidth of series RLC circuit is R . Therefore


L
Bandwidth of filter 1 is B1 = R
L1
Bandwidth of filter 2 is B2 = R = R = 4R
L2 L1 /4 L1
Dividing above equation B1 = 1
B2 4
Hence (D) is correct option.
MCQ 1.29 For the circuit shown in the figure, the Thevenin voltage and resistance looking
into X − Y are

(A) 4
3 V, 2 Ω (B) 4 V, 23 Ω
(C) 4
3 V, 23 Ω (D) 4 V, 2 Ω
SOL 1.29 Here Vth is voltage across node also. Applying nodal analysis we get

Brought to you by: Nodia and Company Visit us at: www.nodia.co.in


PUBLISHING FOR GATE
Page 12 GATE EC 2007 www.gatehelp.com

Vth + Vth + Vth − 2i = 2


2 1 1
But from circuit i = Vth = Vth
1
Therefore
Vth + Vth + Vth − 2Vth = 2
2 1 1
or Vth = 4 volt
From the figure shown below it may be easily seen that the short circuit current at
terminal XY is isc = 2 A because i = 0 due to short circuit of 1 Ω resistor and all
current will pass through short circuit.

Therefore Rth = Vth = 4 = 2 Ω


isc 2
Hence (D) is correct option.
MCQ 1.30 In the circuit shown, vC is 0 volts at t = 0 sec. For t > 0 , the capacitor current iC (t)
, where t is in seconds is given by

(A) 0.50 exp (− 25t) mA (B) 0.25 exp (− 25t) mA


(C) 0.50 exp (− 12.5t) mA (D) 0.25 exp (− 6.25t) mA
SOL 1.30 The voltage across capacitor is
At t = 0+ , Vc (0+) = 0
At t = 3 , VC (3) = 5 V
The equivalent resistance seen by capacitor as shown in fig is
Req = 20 20 = 10kΩ

Brought to you by: Nodia and Company Visit us at: www.nodia.co.in


PUBLISHING FOR GATE
Page 13 GATE EC 2007 www.gatehelp.com

Time constant of the circuit is


τ = Req C = 10k # 4μ = 0.04 s
Using direct formula
Vc (t) = VC (3) − [Vc (3) − Vc (0)] e−t/τ
= VC (3) (1 − e−t/τ) + VC (0) e−t/τ = 5 (1 − e−t/0.04)
or Vc (t) = 5 (1 − e−25t)
dV (t)
Now IC (t) = C C
dt
= 4 # 10−6 # (− 5 # 25e−25t) = 0.5e−25t mA
Hence (A) is correct option.
MCQ 1.31 In the ac network shown in the figure, the phasor voltage VAB (in Volts) is

(A) 0 (B) 5+30c


(C) 12.5+30c (D) 17+30c
SOL 1.31 Hence (D) is correct option.
(5 − 3j) # (5 + 3j)
Impedance = (5 − 3j) (5 + 3j) =
5 − 3j + 5 + 3j
(5) 2 − (3j) 2
= = 25 + 9 = 3.4
10 10
VAB = Current # Impedance = 5+30c # 34 = 17+30c
MCQ 1.32 A p+ n junction has a built-in potential of 0.8 V. The depletion layer width a reverse
bias of 1.2 V is 2 μm. For a reverse bias of 7.2 V, the depletion layer width will be
(A) 4 μm (B) 4.9 μm
(C) 8 μm (D) 12 μm
SOL 1.32 Hence option (A) is correct.
W = K V + VR
Now 2μ = K 0.8 + 1.2
From above two equation we get

Brought to you by: Nodia and Company Visit us at: www.nodia.co.in


PUBLISHING FOR GATE
Page 14 GATE EC 2007 www.gatehelp.com

W = 0.8 + 7.2 = 8 =2
2μ 0.8 + 1.2 2
or W2 = 4 μ m
MCQ 1.33 Group I lists four types of p − n junction diodes. Match each device in Group I
with one of the option in Group II to indicate the bias condition of the device in
its normal mode of operation.
Group - I Group-II
(P) Zener Diode (1) Forward bias
(Q) Solar cell (2) Reverse bias
(R) LASER diode
(S) Avalanche Photodiode

(A) P - 1, Q - 2, R - 1, S - 2 (B) P - 2, Q - 1, R - 1, S - 2
(C) P - 2, Q - 2, R - 1, S- -2 (D) P - 2, Q - 1, R - 2, S - 2
SOL 1.33 Zener diode and Avalanche diode works in the reverse bias and laser diode works
in forward bias.
In solar cell diode works in forward bias but photo current is in reverse direction.
Thus
Zener diode : Reverse Bias
Solar Cell : Forward Bias
Laser Diode : Forward Bias
Avalanche Photo diode : Reverse Bias
Hence option (B) is correct.
MCQ 1.34 The DC current gain (β) of a BJT is 50. Assuming that the emitter injection
efficiency is 0.995, the base transport factor is
(A) 0.980 (B) 0.985
(C) 0.990 (D) 0.995
SOL 1.34 Hence option (B) is correct.
β
α= = 50 = 50
β + 1 50 + 1 51
Current Gain = Base Transport Factor # Emitter injection Efficiency
α = β1 # β2
or β1 = α = 50 = 0.985
β2 51 # 0.995

MCQ 1.35 Group I lists four different semiconductor devices. match each device in Group I
with its charactecteristic property in Group II
Group-I Group-II
(P) BJT (1) Population iniversion
(Q) MOS capacitor (2) Pinch-off voltage

Brought to you by: Nodia and Company Visit us at: www.nodia.co.in


PUBLISHING FOR GATE
Page 15 GATE EC 2007 www.gatehelp.com

(R) LASER diode (3) Early effect


(S) JFET (4) Flat-band voltage

(A) P - 3, Q - 1, R - 4, S - 2 (B) P - 1, Q - 4, R - 3, S - 2
(C) P - 3, Q - 4, R - 1, S - 2 (D) P - 3, Q - 2, R - 1, S - 4
SOL 1.35 In BJT as the B-C reverse bias voltage increases, the B-C space charge region width
increases which xB (i.e. neutral base width) > A change in neutral base width will
change the collector current. A reduction in base width will causes the gradient in
minority carrier concentration to increase, which in turn causes an increased in the
diffusion current. This effect si known as base modulation as early effect.
In JFET the gate to source voltage that must be applied to achieve pinch off
voltage is described as pinch off voltage and is also called as turn voltage or
threshold voltage.
In LASER population inversion occurs on the condition when concentration of
electrons in one energy state is greater than that in lower energy state, i.e. a non
equilibrium condition.
In MOS capacitor, flat band voltage is the gate voltage that must be applied to
create flat ban condition in which there is no space charge region in semiconductor
under oxide.
Therefore
BJT : Early effect
MOS capacitor: Flat-band voltage
LASER diode : Population inversion
JFET : Pinch-off voltage
Hence option (C) is correct.
MCQ 1.36 For the Op-Amp circuit shown in the figure, V0 is

(A) -2 V (B) -1 V
(C) -0.5 V (D) 0.5 V
SOL 1.36 We redraw the circuit as shown in fig.

Brought to you by: Nodia and Company Visit us at: www.nodia.co.in


PUBLISHING FOR GATE
Page 16 GATE EC 2007 www.gatehelp.com

Applying voltage division rule


v+ = 0.5 V
We know that v+ = v-
Thus v- = 0.5 V
Now i = 1 − 0.5 = 0.5 mA
1k
and i = 0.5 − v0 = 0.5 mA
2k
or v0 = 0.5 − 1 =− 0.5 V
Hence (C) is correct option.
MCQ 1.37 For the BJT circuit shown, assume that the β of the transistor is very large and
VBE = 0.7 V. The mode of operation of the BJT is

(A) cut-off (B) saturation


(C) normal active (D) reverse active
SOL 1.37 If we assume β very large, then IB = 0 and IE = IC ; VBE = 0.7 V. We assume that
BJT is in active, so applying KVL in Base-emitter loop
IE = 2 − VBE = 2 − 0.7 = 1.3 mA
RE 1k
Since β is very large, we have IE = IC , thus
IC = 1.3 mA
Now applying KVL in collector-emitter loop
10 − 10IC − VCE − IC =0
or VCE =− 4.3 V
Now VBC = VBE − VCE
= 0.7 − (− 4.3) = 5 V
Since VBC > 0.7 V, thus transistor in saturation.
Hence (B) is correct option

Brought to you by: Nodia and Company Visit us at: www.nodia.co.in


PUBLISHING FOR GATE
Page 17 GATE EC 2007 www.gatehelp.com

MCQ 1.38 In the Op-Amp circuit shown, assume that the diode current follows the equation
I = Is exp (V/VT ). For Vi = 2V, V0 = V01, and for Vi = 4V, V0 = V02 .
The relationship between V01 and V02 is

(A) V02 = 2 Vo1 (B) Vo2 = e2 Vo1


(C) Vo2 = Vo1 1n2 (D) Vo1 − Vo2 = VT 1n2
SOL 1.38 Here the inverting terminal is at virtual ground and the current in resistor and
diode current is equal i.e.
IR = ID
or Vi = I eV /V
D T
s
R
or VD = VT 1n Vi
Is R
For the first condition
VD = 0 − Vo1 = VT 1n 2
Is R
For the first condition
VD = 0 − Vo1 = VT 1n 4
Is R
Subtracting above equation
Vo1 − Vo2 = VT 1n 4 − VT 1n 2
Is R Is R
or Vo1 − Vo2 = VT 1n 4 = VT 1n2
2
Hence (D) is correct option.
MCQ 1.39 In the CMOS inverter circuit shown, if the trans conductance parameters of the
NMOS and PMOS transistors are
W
kn = kp = μn Cox Wn = μCox p = 40μA/V2
Ln Lp
and their threshold voltages ae VTHn = VTHp = 1 V the current I is

Brought to you by: Nodia and Company Visit us at: www.nodia.co.in


PUBLISHING FOR GATE
Page 18 GATE EC 2007 www.gatehelp.com

(A) 0 A (B) 25 μA
(C) 45 μA (D) 90 μA
SOL 1.39 Hence (D) is correct option
We have Vthp = Vthp = 1 V
W W
and P
= N = 40μA/V2
LP LN
From figure it may be easily seen that Vas for each NMOS and PMOS is 2.5 V
μA
Thus ID = K (Vas − VT ) 2 = 40 2 (2.5 − 1) 2 = 90 μ A
V
MCQ 1.40 For the Zener diode shown in the figure, the Zener voltage at knee is 7 V, the knee
current is negligible and the Zener dynamic resistance is 10 Ω. If the input voltage
(Vi) range is from 10 to 16 V, the output voltage (V0) ranges from

(A) 7.00 to 7.29 V (B) 7.14 to 7.29 V


(C) 7.14 to 7.43 V (D) 7.29 to 7.43 V
SOL 1.40 We have VZ = 7 volt, VK = 0, RZ = 10Ω
Circuit can be modeled as shown in fig below

Since Vi is lies between 10 to 16 V, the range of voltage across 200 kΩ


V200 = Vi − VZ = 3 to 9 volt
The range of current through 200 kΩ is
3 = 15 mA to 9 = 45 mA
200k 200k
The range of variation in output voltage
15m # RZ = 0.15 V to 45m # RZ = 0.45
Thus the range of output voltage is 7.15 Volt to 7.45 Volt
Hence (C) is correct option.
MCQ 1.41 The Boolean expression Y = ABC D + ABCD + ABC D + ABC D can be minimized
to

Brought to you by: Nodia and Company Visit us at: www.nodia.co.in


PUBLISHING FOR GATE
Page 19 GATE EC 2007 www.gatehelp.com

(A) Y = ABC D + ABC + AC D (B) Y = ABC D + BCD + ABC D


(C) Y = ABCD + BC D + ABC D (D) Y = ABCD + BC D + ABC D
SOL 1.41 Hence (D) is correct answer.
Y = ABCD + ABCD + ABC D + ABC D
= ABCD + ABC D + ABC D + ABC D
= ABCD + ABC D + BC D (A + A)
= ABCD + ABC D + BC D A+A = 1
MCQ 1.42 The circuit diagram of a standard TTL NOT gate is shown in the figure. Vi = 25
V, the modes of operation of the transistors will be

(A) Q1: revere active; Q2: normal active; Q3: saturation; Q4: cut-off
(B) Q1: revere active; Q2: saturation; Q3: saturation; Q4: cut-off
(C) Q1: normal active; Q2: cut-off; Q3: cut-off; Q4: saturation
(D) Q1: saturation; Q2: saturation; Q3: saturation; Q4: normal active
SOL 1.42 In given TTL NOT gate when Vi = 2.5 (HIGH), then
Q1 " Reverse active
Q2 " Saturation
Q3 " Saturation
Q4 " cut - off region
Hence (B) is correct answer.
MCQ 1.43 In the following circuit, X is given by

Brought to you by: Nodia and Company Visit us at: www.nodia.co.in


PUBLISHING FOR GATE
Page 20 GATE EC 2007 www.gatehelp.com

(A) X = ABC + ABC + ABC + ABC


(B) X = ABC + ABC + ABC + ABC
(C) X = AB + BC + AC
(D) X = AB + BC + AC
SOL 1.43 The circuit is as shown below

Y = AB + AB
and X = YC + YC
= (AB + AB ) C + (AB + AB ) C
= (AB + AB) C + (AB + AB ) C
= ABC + ABC + ABC + ABC
Hence (A) is correct answer.
MCQ 1.44 The following binary values were applied to the X and Y inputs of NAND latch
shown in the figure in the sequence indicated below :
X = 0,Y = 1; X = 0, Y = 0; X = 1; Y = 1
The corresponding stable P, Q output will be.

(A) P = 1, Q = 0; P = 1, Q = 0; P = 1, Q = 0 or P = 0, Q = 1
(B) P = 1, Q = 0; P = 0, Q = 1; or P = 0, Q = 1; P = 0, Q = 1
(C) P = 1, Q = 0; P = 1, Q = 1; P = 1, Q = 0 or P = 0, Q = 1
(D) P = 1, Q = 0; P = 1, Q = 1; P = 1, Q = 1
SOL 1.44 Hence (C) is correct answer.
For X = 0, Y = 1 P = 1, Q = 0
For X = 0, Y = 0 P = 1, Q = 1
For X = 1, Y = 1 P = 1, Q = 0 or P = 0, Q = 1

MCQ 1.45 For the circuit shown, the counter state (Q1 Q0) follows the sequence

Brought to you by: Nodia and Company Visit us at: www.nodia.co.in


PUBLISHING FOR GATE
Page 21 GATE EC 2007 www.gatehelp.com

(A) 00, 01, 10, 11, 00 (B) 00, 01, 10, 00, 01
(C) 00, 01, 11, 00, 01 (D) 00, 10, 11, 00, 10
SOL 1.45 For this circuit the counter state (Q1, Q0) follows the sequence 00, 01, 10, 00 ... as
shown below

Clock D1 D0 Q1 Q0 Q1 NOR Q0
00 1
1st 01 10 0
2nd 10 01 0
3rd 00 00 0

Hence (A) is correct answer.


MCQ 1.46 An 8255 chip is interfaced to an 8085 microprocessor system as an I/O mapped I/O
as show in the figure. The address lines A0 and A1 of the 8085 are used by the 8255
chip to decode internally its thee ports and the Control register. The address lines
A3 to A7 as well as the IO/M signal are used for address decoding. The range of
addresses for which the 8255 chip would get selected is

(A) F8H - FBH (B) F8GH - FCH


(C) F8H - FFH (D) F0H - F7H
SOL 1.46 Chip 8255 will be selected if bits A3 to A7 are 1. Bit A0 to A2 can be 0 or.
1. Thus address range is

Brought to you by: Nodia and Company Visit us at: www.nodia.co.in


PUBLISHING FOR GATE
Page 22 GATE EC 2007 www.gatehelp.com

11111000 F8H
11111111 FFH
Hence (C) is correct answer.
MCQ 1.47 The 3-dB bandwidth of the low-pass signal e−t u (t), where u (t) is the unit step
function, is given by
(A) 1 Hz (B) 1 2 − 1 Hz
2π 2π
(C) 3 (D) 1 Hz
SOL 1.47 Hence (A) is correct answer.
x (t) = e−t u (t)
Taking Fourier transform
X (jω) = 1
1 + jω
X (jω) = 1 2
1+ω
Magnitude at 3dB frequency is 1
2
Thus 1 = 1
2 1 + ω2
or ω = 1 rad
or f = 1 Hz

MCQ 1.48 A Hilbert transformer is a


(A) non-linear system (B) non-causal system
(C) time-varying system (D) low-pass system
SOL 1.48 A Hilbert transformer is a non-linear system.
Hence (A) is correct answer.
MCQ 1.49 The frequency response of a linear, time-invariant system is given by
H (f) = 1 + j510πf . The step response of the system is
(B) 5 61 − e− 5@ u (t)
t
(A) 5 (1 − e−5t) u (t)
(C) 1 (1 − e−5t) u (t) (D) 1 ^1 − e− 5 h u (t)
t

2 5
SOL 1.49 Hence (B) is correct answer.
H (f) = 5
1 + j10πf
H (s) = 5 = 5 = 1
1 + 5s 5^s + 15 h s + 15
Step response Y (s) = 1 a 1
s ^s + 5 h
or Y (s) = 1 1 1 = 5 − 5 1
s ^s + 5 h s s+ 5
Brought to you by: Nodia and Company Visit us at: www.nodia.co.in
PUBLISHING FOR GATE
Page 23 GATE EC 2007 www.gatehelp.com

or y (t) = 5 (1 − e−t/5) u (t)

MCQ 1.50 A 5-point sequence x [n] is given as x [− 3] = 1, x [− 2] = 1, x [− 1] = 0, x [0] = 5 and


x [1] = 1. Let X (eiω) denoted the discrete-time Fourier transform of x [n]. The value
π
of #
−π
X (e jω) dω is

(A) 5 (B) 10π


(C) 16π (D) 5 + j10π
SOL 1.50 For discrete time Fourier transform (DTFT) when N " 3
π
x [n] = 1
2π − π #
X (e jω) e jωn dω

Putting n = 0 we get
π π
x [0] = 1
2π #
−π
X (e jω) e jω0 dω = 1
2π #
−π
X (e jω) dω
π
or #
−π
X (e jω) dω = 2πx [0] = 2π # 5 = 10π

Hence (B) is correct answer.


MCQ 1.51 The z −transform X (z) of a sequence x [n] is given by X [z] = 1 −0.25z . It is given that −1

the region of convergence of X (z) includes the unit circle. The value of x [0] is
(A) − 0.5 (B) 0
(C) 0.25 (D) 05
SOL 1.51 Hence (B) is correct answer.
X (z) = 0.5 −1
1 − 2z
Since ROC includes unit circle, it is left handed system
x (n) =− (0.5) (2) −n u (− n − 1)
x (0) = 0
If we apply initial value theorem
x (0) = lim X (z) = lim 0.5 −1 = 0.5
z"3 z " 31 − 2z

That is wrong because here initial value theorem is not applicable because signal
x (n) is defined for n < 0 .
MCQ 1.52 A control system with PD controller is shown in the figure. If the velocity error
constant KV = 1000 and the damping ratio ζ = 0.5 , then the value of KP and KD
are

Brought to you by: Nodia and Company Visit us at: www.nodia.co.in


PUBLISHING FOR GATE
Page 24 GATE EC 2007 www.gatehelp.com

(A) KP = 100, KD = 0.09 (B) KP = 100, KD = 0.9


(C) KP = 10, KD = 0.09 (D) KP = 10, KD = 0.9
SOL 1.52 Hence (B) is correct option
We have Kv = lim sG (s) H (s)
s"0

(Kp + KD s) 100
or 1000 = lim s = Kp
s"0 s (s + 100)
Now characteristics equations is
1 + G (s) H (s) = 0
(K + KD s) 100
1000 = lims " 0 s p = Kp
s (s + 100)
Now characteristics equation is
1 + G (s) H (s) = 0

(100 + KD s) 100
or 1+ =0 Kp = 100
s (s + 10)
or s2 + (10 + 100KD) s + 10 4 = 0

Comparing with s2 + 2ξωn + ωn2 = 0 we get


2ξωn = 10 + 100KD
or KD = 0.9
MCQ 1.53 The transfer function of a plant is
T (s) = 5
(s + 5)( s2 + s + 1)
The second-order approximation of T (s) using dominant pole concept is
(A) 1 (B) 5
(s + 5)( s + 1) (s + 5)( s + 1)

(C) 5 (D) 1
s2 + s + 1 s2 + s + 1
SOL 1.53 Hence (D) is correct option.
We have T (s) = 5
(s + 5)( s2 + s + 1)
= 5 = 2 1
5`1 + j (s + s + 1)
s 2
s +s+1
5
In given transfer function denominator is (s + 5)[( s + 0.5) 2 + 43 ]. We can see easily
that pole at s =− 0.5 ! j 23 is dominant then pole at s =− 5 . Thus we have
approximated it.
MCQ 1.54 The open-loop transfer function of a plant is given as G (s) = s 1- 1 . If the plant is
2

operated in a unity feedback configuration, then the lead compensator that an


stabilize this control system is

Brought to you by: Nodia and Company Visit us at: www.nodia.co.in


PUBLISHING FOR GATE
Page 25 GATE EC 2007 www.gatehelp.com

10 (s − 1) 10 (s + 4)
(A) (B)
s+2 s+2
10 (s + 2) 2 (s + 2)
(C) (D)
s + 10 s + 10
SOL 1.54 Hence (A) is correct option.
G (s) = 2 1 = 1
s −1 (s + 1)( s − 1)
The lead compensator C (s) should first stabilize the plant i.e. remove 1 term.
(s − 1)
From only options (A), C (s) can remove this term

1 10 (s − 1)
ThusG (s) C (s) = #
(s + 1)( s − 1) (s + 2)
= 10 Only option (A) satisfies.
(s + 1)( s + 2)

MCQ 1.55 A unity feedback control system has an open-loop transfer function
G (s) = K
s (s + 7s + 12)
2

The gain K for which s = 1 + j1 will lie on the root locus of this system is
(A) 4 (B) 5.5
(C) 6.5 (D) 10
SOL 1.55 For ufb system the characteristics equation is
1 + G (s) = 0
or 1+ K =0
s (s2 + 7s + 12)
or s (s2 + 7s + 12) + K = 0
Point s =− 1 + j lie on root locus if it satisfy above equation i.e
(− 1 + j)[( − 1 + j) 2 + 7 (− 1 + j) + 12) + K] = 0
or K =+ 10
Hence (D) is correct option.
MCQ 1.56 The asymptotic Bode plot of a transfer function is as shown in the figure. The
transfer function G (s) corresponding to this Bode plot is

(A) 1 (B) 1
(s + 1)( s + 20) s (s + 1)( s + 20)
Brought to you by: Nodia and Company Visit us at: www.nodia.co.in
PUBLISHING FOR GATE
Page 26 GATE EC 2007 www.gatehelp.com

(C) 100 (D) 100


s (s + 1)( s + 20) s (s + 1)( 1 + 0.05s)
SOL 1.56 At every corner frequency there is change of -20 db/decade in slope which indicate
pole at every corner frequency. Thus
G (s) = K
s (1 + s)`1 + s j
20
Bode plot is in (1 + sT) form
20 log K = 60 dB = 1000
ω ω = 0. 1
Thus K =5
Hence G (s) = 100
s (s + 1)( 1 + .05s)
Hence (D) is correct option.
MCQ 1.57 The state space representation of a separately excited DC servo motor dynamics
is given as

−1 1 ω
> di H = =− 1 − 10G=ia G + =10Gu
dt 0
o
dt

where ω is the speed of the motor, ia is the armature current and u is the armature
ω (s)
voltage. The transfer function of the motor is
U (s)
(A) 2 10 (B) 2 1
s + 11s + 11 s + 11s + 11
(C) 2 10s + 10 (D) 2 1
s + 11s + 11 s + s + 11

SOL 1.57 Hence (A) is correct option.



−1 1 ω
We have > didt H = =
0
a G=
− 1 − 10 in G + =10G
u
dt

or dω =− ω + i ...(1)
n
dt
and dia =− ω − 10i + 10u ...(2)
a
dt
Taking laplace transform (i) we get
sω (s) =− ω (s) = Ia (s)
or (s + 1) ω (s) = Ia (s) ...(3)
Taking laplace transform (ii) we get
sIa (s) =− ω (s) − 10Ia (s) + 10U (s)
or ω (s) = (− 10 − s) Ia (s) + 10U (s)
= (− 10 − s)( s + 1) ω (s) + 10U (s) From (3)
or ω (s) =− [s2 + 11s + 10] ω (s) + 10U (s)

Brought to you by: Nodia and Company Visit us at: www.nodia.co.in


PUBLISHING FOR GATE
Page 27 GATE EC 2007 www.gatehelp.com

or(s2 + 11s + 11) ω (s) = 10U (s)


ω (s)
or = 2 10
U (s) (s + 11s + 11)
MCQ 1.58 In delta modulation, the slope overload distortion can be reduced by
(A) decreasing the step size (B) decreasing the granular noise
(C) decreasing the sampling rate (D) increasing the step size
SOL 1.58 Slope overload distortion can be reduced by increasing the step size
3 $ slope of x (t)
Ts
Hence (D) is correct option.
MCQ 1.59 The raised cosine pulse p (t) is used for zero ISI in digital communications. The
expression for p (t) with unity roll-off factor is given by
p (t) = sin 4πWt
4πWt (1 − 16W2 t2)
The value of p (t) at t = 1 is
4W
(A) − 0.5 (B) 0
(C) 0.5 (D) 3
SOL 1.59 Hence (C) is correct option.
sin (4πWt)
We have p (t) =
4πWt (1 − 16W2 t2)
at t = 1 it is 0 form. Thus applying L' Hospital rule
4W 0
4πW cos (4πWt)
p( ) =
1
4W

4πW [1 − 48W2 t2]


cos (4πWt)
= = cos π = 0.5
1 − 48W t
2 2 1−3

MCQ 1.60 In the following scheme, if the spectrum M (f) of m (t) is as shown, then the spectrum
Y (f) of y (t) will be

Brought to you by: Nodia and Company Visit us at: www.nodia.co.in


PUBLISHING FOR GATE
Page 28 GATE EC 2007 www.gatehelp.com

SOL 1.60 The block diagram is as shown below

Here M1 (f) = Mt (f)


j 2π B
Y1 (f) = M (f) c e + e -j2πB
2 m
j 2π B
Y2 (f) = M1 (f) c e − e -j2πB
2 m

Y (f) = Y1 (f) + Y2 (f)


All waveform is shown below

Brought to you by: Nodia and Company Visit us at: www.nodia.co.in


PUBLISHING FOR GATE
Page 29 GATE EC 2007 www.gatehelp.com

Hence (B) is correct option.


MCQ 1.61 During transmission over a certain binary communication channel, bit errors occur
independently with probability p. The probability of AT MOST one bit in error
in a block of n bits is given by
(A) pn (B) 1 − pn
(C) np (1 − p) n - 1 + (1 + p) n (D) 1 − (1 − p) n
SOL 1.61 By Binomial distribution the probability of error is
pe = n Cr pr (1 − p) n - r
Probability of at most one error
= Probability of no error + Probability of one error
= n C0 p0 (1 − p) n - 0 + n C1 p1 (1 − p) n - 1

= (1 − p) n + np (1 − p) n - 1
Hence (C) is correct option.
MCQ 1.62 In a GSM system, 8 channels can co-exist in 200 kHz bandwidth using TDMA.
A GSM based cellular operator is allocated 5 MHz bandwidth. Assuming a
frequency reuse factor of 1 , i.e. a five-cell repeat pattern, the maximum number of
5
simultaneous channels that can exist in one cell is
(A) 200 (B) 40
(C) 25 (D) 5
SOL 1.62 Bandwidth allocated for 1 Channel = 5 M Hz
Average bandwidth for 1 Channel 5 = 1 MHz
5
Total Number of Simultaneously Channel = 1M # 8 = 40 Channel
200k
Hence (B) is correct option.
MCQ 1.63 In a Direct Sequence CDMA system the chip rate is 1.2288 # 106 chips per second.
If the processing gain is desired to be AT LEAST 100, the data rate
(A) must be less than or equal to 12.288 # 103 bits per sec
(B) must be greater than 12.288 # 103 bits per sec
(C) must be exactly equal to 12.288 # 103 bits per sec
(D) can take any value less than 122.88 # 103 bits per sec
SOL 1.63 Hence (A) is correct option.
Chip Rate RC = 1.2288 # 106 chips/sec

Brought to you by: Nodia and Company Visit us at: www.nodia.co.in


PUBLISHING FOR GATE
Page 30 GATE EC 2007 www.gatehelp.com

Data Rate Rb = RC
G
Since the processing gain G must be at least 100, thus for Gmin we get
6
Rb max = RC = 1.2288 # 10 = 12.288 # 103 bps
Gmin 100

MCQ 1.64 An air-filled rectangular waveguide has inner dimensions of 3 cm # 2 cm. The wave
impedance of the TE20 mode of propagation in the waveguide at a frequency of 30
GHz is (free space impedance η0 = 377 Ω )
(A) 308 Ω (B) 355 Ω
(C) 400 Ω (D) 461 Ω
SOL 1.64 The cut-off frequency is
fc = c ` m j2 + ` n j2
2 a b
Since the mode is TE20, m = 2 and n = 0
8
fc = c m = 3 # 10 # 2 = 10 GHz
2 2 2 # 0.03
ηo 377
η' = fc 2
= 10 = 400Ω
1−c m
2
1 − c 10 10 m
f 3 # 10
Hence (C) is correct option.
MCQ 1.65 The H field (in A/m) of a plane wave propagating in free space is given by
H = xt 5 3 cos (ωt − βz) + yt`ωt − βz + π j .
η0 2
The time average power flow density in Watts is
η
(A) 0 (B) 100
100 η0

(C) 50η20 (D) 50


η0
SOL 1.65 Hence (D) is correct option.
2
= Hx2 + Hy2 = c 5 3 m + c 5 m = c 10 m
2 2
We have H 2
ηo ηo ηo
E 2 ηo H 2
η 2
For free space P = = = o c 10 m = 50 watts
2ηo 2 2 ηo ηo

MCQ 1.66 The E field in a rectangular waveguide of inner dimension a # b is given by


ωμ
E = 2 ` λ j H0 sin ` 2πx j sin (ωt − βz) yt
2

h 2 a
Where H0 is a constant, and a and b are the dimensions along the x − axis and the
y − axis respectively. The mode of propagation in the waveguide is
(A) TE20 (B) TM11
(C) TM20 (D) TE10

Brought to you by: Nodia and Company Visit us at: www.nodia.co.in


PUBLISHING FOR GATE
Page 31 GATE EC 2007 www.gatehelp.com

SOL 1.66 Hence (A) is correct option.


ωμ
E = 2 ` π j H0 sin ` 2πx j sin (ωt − βz) yt
2

h 2 a
This is TE mode and we know that
mπy
Ey \ sin ` mπx j cos `
a b j
Thus m = 2 and n = 0 and mode is TE20
MCQ 1.67 A load of 50 Ω is connected in shunt in a 2-wire transmission line of Z0 = 50Ω as
shown in the figure. The 2-port scattering parameter matrix (s-matrix) of the shunt
element is

−1 1
(A) > 12 1H
0 1
(B) =
1 0G
2
2 − 2

− 13 2 1
− 43
(C) > 3
H (D) > 4
1H
2
3 − 1
3 −3
4 4
SOL 1.67 The 2-port scattering parameter matrix is
S11 S12
S ==
S21 S22 G
(Z Z ) − Zo (50 50) − 50
S11 = L 0 = =− 1
(ZL Z0) + Zo (50 50) + 50 3
2 (ZL Zo) 2 (50 50)
S12 = S21 = = =2
(ZL Zo) + Zo (50 50) + 50 3
(Z Z ) − Zo (50 50) − 50
S22 = L o = =− 1
(ZL Zo) + Zo (50 50) + 50 3
Hence (C) is correct option.
MCQ 1.68 The parallel branches of a 2-wire transmission line re terminated in 100Ω and
200Ω resistors as shown in the figure. The characteristic impedance of the line is
Z0 = 50Ω and each section has a length of λ . The voltage reflection coefficient Γ
4
at the input is

Brought to you by: Nodia and Company Visit us at: www.nodia.co.in


PUBLISHING FOR GATE
Page 32 GATE EC 2007 www.gatehelp.com

(A) − j 7 (B) − 5
5 7

(C) j 5 (D) 5
7 7
SOL 1.68 The input impedance is
if l = λ
2
Zin = Zo ;
ZL 4
2 2
Zin1 = Zo1 = 50 = 25
ZL1 100
2 2
Zin2 = Zo2 = 50 = 12.5
ZL2 200
Now ZL = Zin1 Zin2
25 12.5 = 25
3
(50) 2
Zs = = 300
25/3
Γ = ZS − Zo = 300 − 50 = 5
ZS + Zo 300 + 50 7
Hence (D) is correct option.

MCQ 1.69 A λ dipole is kept horizontally at a height of λ0 above a perfectly conducting


2 2
infinite ground plane. The radiation pattern in the lane of the dipole (E plane)
looks approximately as

Brought to you by: Nodia and Company Visit us at: www.nodia.co.in


PUBLISHING FOR GATE
Page 33 GATE EC 2007 www.gatehelp.com

SOL 1.69 Using the method of images, the configuration is as shown below

Here d = λ, α = π, thus βd = 2π

Array factor is
βd cos ψ + α
= cos ; E
2
2π cos ψ + π
= cos ; E = sin (π cos ψ)
2
Hence (B) is correct option.
MCQ 1.70 A right circularly polarized (RCP) plane wave is incident at an angle 60c to the
normal, on an air-dielectric interface. If the reflected wave is linearly polarized, the
relative dielectric constant ξr2 is.

(A) 2 (B) 3
(C) 2 (D) 3
SOL 1.70 The Brewster angle is
tan θn = εr2
εr1
tan 60c = εr2
1
or εr2 = 3

Brought to you by: Nodia and Company Visit us at: www.nodia.co.in


PUBLISHING FOR GATE
Page 34 GATE EC 2007 www.gatehelp.com

Hence (D) is correct option.

Common Data Questions

Common Data for Questions 71, 72, 73 :


The figure shows the high-frequency capacitance - voltage characteristics of Metal/
Sio 2 /silicon (MOS) capacitor having an area of 1 # 10 - 4 cm 2 . Assume that the
permittivities (ε0 εr ) of silicon and Sio2 are 1 # 10 - 12 F/cm and 3.5 # 10 - 13 F/cm
respectively.

MCQ 1.71 The gate oxide thickness in the MOS capacitor is


(A) 50 nm (B) 143 nm
(C) 350 nm (D) 1 μm
SOL 1.71 At low voltage when there is no depletion region and capacitance is decide by SiO2
thickness only,
C = ε0 εr1 A
D
−13
or D = ε0 εr1 A = 3.5 # 10 −# 10−4 = 50 nm
C 7 # 10 12
Hence option (A) is correct
MCQ 1.72 The maximum depletion layer width in silicon is
(A) 0.143 μm (B) 0.857 μm
(C) 1 μm (D) 1.143 μm
SOL 1.72 The construction of given capacitor is shown in fig below

Brought to you by: Nodia and Company Visit us at: www.nodia.co.in


PUBLISHING FOR GATE
Page 35 GATE EC 2007 www.gatehelp.com

When applied voltage is 0 volts, there will be no depletion region and we get
C1 = 7 pF
When applied voltage is V , a depletion region will be formed as shown in fig an
total capacitance is 1 pF. Thus
CT = 1 pF
or CT = C1 C2 = 1 pF
C1 + C2
or 1 = 1 + 1
CT C1 C2
Substituting values of CT and C1 we get
C2 = 7 pF
6
D2 = ε0 εr2 A = 1 # 710 #- 1210 = 6 # 10 - 4 cm
- 12 -4
Now
C2 6 # 10
7
= 0.857 μm
Hence option (B) is correct.
MCQ 1.73 Consider the following statements about the C − V characteristics plot :
S1 : The MOS capacitor has as n -type substrate
S2 : If positive charges are introduced in the oxide, the C − V polt will shift to the
left.
Then which of the following is true?
(A) Both S1 and S2 are true
(B) S1 is true and S2 is false
(C) S1 is false and S2 is true
(D) Both S1 and S2 are false
SOL 1.73 Depletion region will not be formed if the MOS capacitor has n type substrate but
from C-V characteristics, C reduces if V is increased. Thus depletion region must
be formed. Hence S1 is false
If positive charges is introduced in the oxide layer, then to equalize the effect the
applied voltage V must be reduced. Thus the C − V plot moves to the left. Hence
S2 is true.
Hence option (C) is correct.

Brought to you by: Nodia and Company Visit us at: www.nodia.co.in


PUBLISHING FOR GATE
Page 36 GATE EC 2007 www.gatehelp.com

Common Data for Questions 74 & 75 :


Two 4-array signal constellations are shown. It is given that φ1 and φ2 constitute
an orthonormal basis for the two constellation. Assume that the four symbols in
both the constellations are equiprobable. Let N0 denote the power spectral density
2
of white Gaussian noise.

MCQ 1.74 The if ratio or the average energy of Constellation 1 to the average energy of
Constellation 2 is
(A) 4a2 (B) 4
(C) 2 (D) 8
SOL 1.74 Energy of constellation 1 is
Eg1 = (0) 2 + (− 2 a) 2 + (− 2 a) 2 + ( 2 a) 2 + (− 2 2 a) 2

= 2a2 + 2a2 + 2a2 + 8a2 = 16a2


Energy of constellation 2 is
Eg2 = a2 + a2 + a2 + a2 = 4a2

Eg1 16a2
Ratio = = =4
Eg2 4a2
Hence (B) is correct option.
MCQ 1.75 If these constellations are used for digital communications over an AWGN channel,
then which of the following statements is true ?
(A) Probability of symbol error for Constellation 1 is lower
(B) Probability of symbol error for Constellation 1 is higher
(C) Probability of symbol error is equal for both the constellations
(D) The value of N0 will determine which of the constellations has a lower probability
of symbol error

SOL 1.75 Noise Power is same for both which is N0 .


2
Thus probability of error will be lower for the constellation 1 as it has higher signal
energy.
Hence (A) is correct option.

Brought to you by: Nodia and Company Visit us at: www.nodia.co.in


PUBLISHING FOR GATE
Page 37 GATE EC 2007 www.gatehelp.com

Linked Answer Questions : Q.76 to Q.85 carry two marks each.

Statement for Linked Answer Questions 76 & 77:


Consider the Op-Amp circuit shown in the figure.

MCQ 1.76 The transfer function V0 (s)/ Vi (s) is


(A) 1 − sRC (B) 1 + sRC
1 + sRC 1 − sRC

(C) 1 (D) 1
1 − sRC 1 + sRC
SOL 1.76 The voltage at non-inverting terminal is
1
V+ = sC 1 Vi = 1 V
R + sC 1 + sCR i
Now V- = V+ = 1 V
1 + sCR i
Applying voltage division rule
(V + Vi)
V+ = R1 (V0 + Vi) = o
R1 + R1 2
1 (V + Vi)
or Vi = o
1 + sCR 2
or Vo =− 1 + 2
Vi 1 + sRC
V0 = 1 − sRC
Vi 1 + sRC
Hence (A) is correct option.

MCQ 1.77 If Vi = V1 sin (ωt) and V0 = V2 sin (ωt + φ), then the minimum and maximum values
of φ (in radians) are respectively
(A) − π and π (B) 0 and π
2 2 2

(C) − π and 0 (D) − π and 0


2
SOL 1.77 Hence (C) is correct option.
V0 = H (s) = 1 − sRC
Vi 1 + sRC
Brought to you by: Nodia and Company Visit us at: www.nodia.co.in
PUBLISHING FOR GATE
Page 38 GATE EC 2007 www.gatehelp.com

1 − jωRC
H (jω) =
1 + jωRC
+H (jω) = φ =− tan - 1 ωRC − tan - 1 ωRC
=− 2 tan - 2 ωRC
Minimum value, φmin = − π (at ω " 3)
Maximum value, φmax = 0( at ω = 0)

Statement for Linked Answer Questions 78 & 79 :


An 8085 assembly language program is given below.
Line 1: MVI A, B5H
2: MVI B, OEH
3: XRI 69H
4: ADD B
5: ANI 9BH
6: CPI 9FH
7: STA 3010H
8: HLT
MCQ 1.78 The contents of the accumulator just execution of the ADD instruction in line 4
will be
(A) C3H (B) EAH
(C) DCH (D) 69H
SOL 1.78 Line 1 : MVI A, B5H ; Move B5H to A
2 : MVI B, 0EH ; Move 0EH to B
3 : XRI 69H ; [A] XOR 69H and store in A
; Contents of A is CDH
4 : ADDB ; Add the contents of A to contents of B and
; store in A, contents of A is EAH
5 : ANI 9BH ; [a] AND 9BH, and store in A,
; Contents of A is 8 AH
6 : CPI 9FH ; Compare 9FH with the contents of A
; Since 8 AH < 9BH, CY = 1
7 : STA 3010 H ; Store the contents of A to location 3010 H
8 : HLT ; Stop
Thus the contents of accumulator after execution of ADD instruction is EAH.
Hence (B) is correct answer.
MCQ 1.79 After execution of line 7 of the program, the status of the CY and Z flags will be
(A) CY = 0, Z = 0 (B) CY = 0, Z = 1

Brought to you by: Nodia and Company Visit us at: www.nodia.co.in


PUBLISHING FOR GATE
Page 39 GATE EC 2007 www.gatehelp.com

(C) CY = 1, Z = 0 (D) CY = 1, Z = 1
SOL 1.79 The CY = 1 and Z = 0
Hence (C) is correct answer.

Statement for linked Answer Question 80 & 81 :


Consider a linear system whose state space representation is x (t) = Ax (t). If
1
the initial state vector of the system is x (0) = = G, then the system response is
−2
e−2x 1
−2tH . If the itial state vector of the system changes to x (0) = =
− 2G
x (t) = > , then
− 2e −t
e
the system response becomes x (t) = > −tH
−e
MCQ 1.80 The eigenvalue and eigenvector pairs (λi vi) for the system are
1 1 1 1
(A) e− 1 = Go and e− 2 = Go (B) e− 1, = Go and e2, = Go
−1 −2 −1 −2
1 1 1 1
(C) e− 1, = Go and e− 2, = Go (D) e− 2 = Go and e1, = Go
−1 −2 −1 −2

SOL 1.80 Hence (A) is correct option.


We have xo (t) = Ax (t)
p q
A ==
r sG
Let

1 e−2t
For initial state vector x (0) = = G the system response is x (t) = > H
−2 − 2e−2t
d −2t
e p q 1
==
r s G=− 2G
Thus > d dt −2t H
dt (− 2e )
t=0
−2 (0)
− 2e p q 1
or > 4e−2 (0) H = =r s G=− 2G
−2 p − 2q
= 4 G = = r − 2s G

We get p − 2q =− 2 and r − 2s = 4 ...(i)


1 e−t
For initial state vector x (0) = = G the system response is x (t) = > −tH
−1 −e
d
e−t p q 1
==
> d (− e−t)H r s G=− 1G
dt
Thus
dt
t=0
− (0)
−e p q 1
> e− (0) H = =r s G=− 1G
−1 p−q
= 1G = = r − s G

Brought to you by: Nodia and Company Visit us at: www.nodia.co.in


PUBLISHING FOR GATE
Page 40 GATE EC 2007 www.gatehelp.com

We get p − q =− 1 and r − s = 1 ...(2)


Solving (1) and (2) set of equations we get
p q 0 1
=r s G = =− 2 − 3G

The characteristic equation


λI − A = 0
λ −1
=0
2 λ+3
or λ (λ + 3) + 2 = 0
or λ =− 1, − 2
Thus Eigen values are − 1 and − 2
Eigen vectors for λ1 =− 1
(λ1 I − A) X1 = 0
λ1 − 1 x11
or = 2 λ + 3G=x G = 0
1 21

− 1 − 1 x11
= 2 2 G=x G = 0
21

or − x11 − x21 = 0
or x11 + x21 = 0
We have only one independent equation x11 =− x21.Let x11 = K , then x21 =− K , the
Eigen vector will be
x11 K 1
=x G = =− K G = K =− 1G
21

Now Eigen vector for λ2 =− 2


(λ2 I − A) X2 = 0
λ2 − 1 x12
or = 2 λ + 3G=x G = 0
2 22

− 2 − 1 x11
or = 2 1 G=x G = 0
21

or − x11 − x21 = 0
or x11 + x21 = 0
We have only one independent equation x11 =− x21.
Let x11 = K, then x21 =− K , the Eigen vector will be
x12 K 1
=x G = =− 2K G = K =− 2G
22

MCQ 1.81 The system matrix A is


0 1 1 1
(A) =
− 1 1G
(B) =
− 1 − 2G

Brought to you by: Nodia and Company Visit us at: www.nodia.co.in


PUBLISHING FOR GATE
Page 41 GATE EC 2007 www.gatehelp.com

2 1 0 1
(C) = G (D) =
−1 −1 − 2 − 3G
SOL 1.81 As shown in previous solution the system matrix is
0 1
A ==
− 2 − 3G
Hence (D) is correct option.

Statement for Linked Answer Question 82 & 83 :


An input to a 6-level quantizer has the probability density function f (x) as shown
in the figure. Decision boundaries of the quantizer are chosen so as to maximize the
entropy of the quantizer output. It is given that 3 consecutive decision boundaries
are’ − 1'.'0' and '1' .

MCQ 1.82 The values of a and b are


(A) a = 1 and b = 1 (B) a = 1 and b = 3
6 12 5 40

(C) a = 1 and b = 1 (D) a = 1 and b = 1


4 16 3 24
SOL 1.82 Area under the pdf curve must be unity
Thus 2a + 4a + 4b =1
2a + 8b = 1 ...(1)
For maximum entropy three region must by equivaprobable thus
2a = 4b = 4b ...(2)
From (1) and (2) we get
b = 1 and a = 1
12 6
Hence (A) is correct option.
MCQ 1.83 Assuming that the reconstruction levels of the quantizer are the mid-points of the
decision boundaries, the ratio of signal power to quantization noise power is
(A) 152 (B) 64
9 3

(C) 76 (D) 28
3
SOL 1.83 Hence correct option is ( )

Brought to you by: Nodia and Company Visit us at: www.nodia.co.in


PUBLISHING FOR GATE
Page 42 GATE EC 2007 www.gatehelp.com

Statement for Linked Answer Question 84 and 85 :


In the Digital-to-Analog converter circuit shown in the figure below,
VR = 10V and R = 10kΩ

MCQ 1.84 The current i is


(A) 31.25μA (B) 62.5μA
(C) 125μA (D) 250μA
SOL 1.84 Since the inverting terminal is at virtual ground the resistor network can be reduced
as follows

The current from voltage source is


I = VR = 10 = 1 mA
R 10k
This current will be divide as shown below

Brought to you by: Nodia and Company Visit us at: www.nodia.co.in


PUBLISHING FOR GATE
Page 43 GATE EC 2007 www.gatehelp.com

-3
Now i = I = 1 # 10 = 62.5 μ A
16 16
Hence (B) is correct answer.
MCQ 1.85 The voltage V0 is
(A) − 0.781 V (B) − 1.562 V
(C) − 3.125 V (D) − 6.250 V
SOL 1.85 The net current in inverting terminal of OP - amp is
I- = 1 + 1 = 5I
4 16 16
So that V0 =− R # 5I =− 3.125
16
Hence (C) is correct answer.

Answer Sheet
1. (A) 19. (A) 37. (B) 55. (D) 73. (C)
2. (B) 20. (D) 38. (D) 56. (D) 74. (B)
3. (C) 21. (C) 39. (D) 57. (A) 75. (A)
4. (A) 22. (A) 40. (C) 58. (D) 76. (A)
5. (D) 23. (C) 41. (D) 59. (C) 77. (C)
6. (A) 24. (D) 42. (B) 60. (B) 78. (B)
7. (D) 25. (B) 43. (A) 61. (C) 79. (C)
8. (C) 26. (C) 44. (C) 62. (B) 80. (A)
9. (D) 27. (A) 45. (A) 63. (A) 81. (D)
10. (C) 28. (D) 46. (C) 64. (C) 82. (A)
11. (C) 29. (D) 47. (A) 65. (D) 83. (*)
12. (A) 30. (A) 48. (A) 66. (A) 84. (B)
13. (C) 31. (D) 49. (B) 67. (C) 85. (C)

Brought to you by: Nodia and Company Visit us at: www.nodia.co.in


PUBLISHING FOR GATE
Page 44 GATE EC 2007 www.gatehelp.com

14. (B) 32. (A) 50. (B) 68. (D)


15. (D) 33. (B) 51. (B) 69. (B)
16. (D) 34. (B) 52. (B) 70 (D)
17. (C) 35. (C) 53. (D) 71 (A)
18. (B) 36. (C) 54. (A) 72 (B)
**********

Brought to you by: Nodia and Company Visit us at: www.nodia.co.in


PUBLISHING FOR GATE
GATE EC
2007

Q.1 to Q.20 carry one mark each

MCQ 1.1 If E denotes expectation, the variance of a random variable X is given by


(A) E [X2] − E2 [X] (B) E [X2] + E2 [X]
(C) E [X2] (D) E2 [X]
SOL 1.1 The variance of a random variable x is given by
E [X2] − E2 [X]
Hence (A) is correct option.
MCQ 1.2 The following plot shows a function which varies linearly with x . The value of the
#1 ydx
2
integral I = is

(A) 1.0 (B) 2.5


(C) 4.0 (D) 5.0
SOL 1.2 The given plot is straight line whose equation is
x +y =1
−1 1
or y = x+1
#1 ydx #1 (x + 1) dx
2 2
Now I = =
(x + 1) 2 2
=; E = − = 2.5
9 4
2 2 2
Hence (B) is correct answer.
MCQ 1.3 For x << 1, coth (x) can be approximated as

Brought to you by: Nodia and Company Visit us at: www.nodia.co.in


PUBLISHING FOR GATE
Page 2 GATE EC 2007 www.gatehelp.com

(A) x (B) x2
(C) 1 (D) 12
x x
SOL 1.3 Hence (C) is correct answer.
coth x = cosh x
sinh x
as x << 1, cosh x . 1 and sinh x . x
Thus coth x . 1
x

sin b θ l
2
MCQ 1.4 lim is
θ"0 θ
(A) 0.5 (B) 1
(C) 2 (D) not defined
SOL 1.4 Hence (A) is correct answer.
sin ^ θ2 h sin ^ θ2 h 1 lim sin ^ 2 h = 1 = 0.5
θ
lim = lim =
θ"0 θ θ " 0 2^ θ h 2 θ " 0 ^ θ2 h 2
2

MCQ 1.5 Which one of following functions is strictly bounded?


(A) 1/x2 (B) ex
2

(C) x2 (D) e - x
SOL 1.5 Hence (D) is correct answer.
We have, lim 12 = 3
x"0 x

lim x2 = 3
x"3

lim e - x = 3
x"3

lim e - x = 0
2

x"3

lim e - x = 1
2 2

Thus e - x is strictly bounded.


x"0

MCQ 1.6 For the function e - x , the linear approximation around x = 2 is


(A) (3 − x) e - 2 (B) 1 − x
(C) 63 + 3 2 − (1 − 2 ) x @e - 2 (D) e - 2
SOL 1.6 Hence (A) is correct answer.
We have f (x) = e - x = e - (x - 2) - 2 = e - (x - 2) e - 2
(x − 2) 2
= ;1 − (x − 2) + ...E e - 2
2!
= 61 − (x − 2)@ e - 2 Neglecting higher powers

Brought to you by: Nodia and Company Visit us at: www.nodia.co.in


PUBLISHING FOR GATE
Page 3 GATE EC 2007 www.gatehelp.com

= (3 − x) e - 2
MCQ 1.7 An independent voltage source in series with an impedance Zs = Rs + jXs delivers
a maximum average power to a load impedance ZL when
(A) ZL = Rs + jXs (B) ZL = Rs
(C) ZL = jXs (D) ZL = Rs − jXs
SOL 1.7 According to maximum Power Transform Theorem
ZL = Zs* = (Rs − jXs)
Hence (D) is correct option.
MCQ 1.8 The RC circuit shown in the figure is

(A) a low-pass filter (B) a high-pass filter


(C) a band-pass filter (D) a band-reject filter
SOL 1.8 At ω " 3 , capacitor acts as short circuited and circuit acts as shown in fig below

Here we get V0 = 0
Vi

At ω " 0 , capacitor acts as open circuited and circuit look like as shown in fig
below

Here we get also V0 = 0


Vi
So frequency response of the circuit is as shown in fig and circuit is a Band pass
filter.

Brought to you by: Nodia and Company Visit us at: www.nodia.co.in


PUBLISHING FOR GATE
Page 4 GATE EC 2007 www.gatehelp.com

Hence (C) is correct option.


MCQ 1.9 The electron and hole concentrations in an intrinsic semiconductor are ni per cm3
at 300 K. Now, if acceptor impurities are introduced with a concentration of NA per
cm3 (where NA >> ni , the electron concentration per cm3 at 300 K will be)
(A) ni (B) ni + NA
2
(C) NA − ni (D) ni
NA
SOL 1.9 As per mass action law
np = ni2
If acceptor impurities are introduces
p = NA
Thus nNA = ni2
2
or n = ni
NA
Hence option (D) is correct.
MCQ 1.10 In a p+ n junction diode under reverse biased the magnitude of electric field is
maximum at
(A) the edge of the depletion region on the p-side
(B) the edge of the depletion region on the n -side
(C) the p+ n junction
(D) the centre of the depletion region on the n -side
SOL 1.10 The electric field has the maximum value at the junction of p+ n .
Hence option (C) is correct.
MCQ 1.11 The correct full wave rectifier circuit is

Brought to you by: Nodia and Company Visit us at: www.nodia.co.in


PUBLISHING FOR GATE
Page 5 GATE EC 2007 www.gatehelp.com

SOL 1.11 The circuit shown in (C) is correct full wave rectifier circuit.

Hence (C) is correct option.


MCQ 1.12 In a transconductance amplifier, it is desirable to have
(A) a large input resistance and a large output resistance
(B) a large input resistance and a small output resistance
(C) a small input resistance and a large output resistance
(D) a small input resistance and a small output resistance
SOL 1.12 In the transconductance amplifier it is desirable to have large input resistance and
large output resistance.
Hence (A) is correct option.
MCQ 1.13 X = 01110 and Y = 11001 are two 5-bit binary numbers represented in two’s
complement format. The sum of X and Y represented in two’s complement format
using 6 bits is
(A) 100111 (B) 0010000
(C) 000111 (D) 101001
SOL 1.13 MSB of Y is 1, thus it is negative number and X is positive number
Now we have X = 01110 = (14) 10
and Y = 11001 = (− 7) 10
X + Y = (14) + (− 7) = 7
In signed two’s complements from 7 is
(7) 10 = 000111
Brought to you by: Nodia and Company Visit us at: www.nodia.co.in
PUBLISHING FOR GATE
Page 6 GATE EC 2007 www.gatehelp.com

Hence (C) is correct answer.


MCQ 1.14 The Boolean function Y = AB + CD is to be realized using only 2 - input NAND
gates. The minimum number of gates required is
(A) 2 (B) 3
(C) 4 (D) 5
SOL 1.14 Hence (B) is correct answer.
Y = AB + CD = AB .CD
This is SOP form and we require only 3 NAND gate

MCQ 1.15 If the closed-loop transfer function of a control system is given as T (s) s−5
(s + 2)( s + 3)
, then It is
(A) an unstable system (B) an uncontrollable system
(C) a minimum phase system (D) a non-minimum phase system
SOL 1.15 In a minimum phase system, all the poles as well as zeros are on the left half of the
s −plane. In given system as there is right half zero (s = 5), the system is a non-
minimum phase system.
Hence (D) is correct option.

MCQ 1.16 If the Laplace transform of a signal Y (s) = 1 , then its final value is
s (s − 1)
(A) − 1 (B) 0
(C) 1 (D) Unbounded
SOL 1.16 Hence (D) is correct answer.
Y (s) = 1
s (s − 1)
Final value theorem is applicable only when all poles of system lies in left half of S
-plane. Here s = 1 is right s −plane pole. Thus it is unbounded.
MCQ 1.17 If R (τ) is the auto correlation function of a real, wide-sense stationary random
process, then which of the following is NOT true
(A) R (τ) = R (− τ)
(B) R (τ) # R (0)
(C) R (τ) =− R (− τ)
(D) The mean square value of the process is R (0)
SOL 1.17 Autocorrelation is even function.
Hence (C) is correct option
MCQ 1.18 If S (f) is the power spectral density of a real, wide-sense stationary random process,
then which of the following is ALWAYS true?
(A) S (0) # S (f) (B) S (f) $ 0

Brought to you by: Nodia and Company Visit us at: www.nodia.co.in


PUBLISHING FOR GATE
Page 7 GATE EC 2007 www.gatehelp.com

#- 3 S (f) df = 0
3
(C) S (− f) =− S (f) (D)
SOL 1.18 Power spectral density is non negative. Thus it is always zero or greater than zero.
Hence (B) is correct option.
MCQ 1.19 A plane wave of wavelength λ is traveling in a direction making an angle 30c with
positive x − axis and 90c with positive y − axis. The E field of the plane wave can
be represented as (E0 is constant)
3 π x− π z π 3πz
t 0 e j c ωt −
(A) E = yE λ λ m t 0 e jc ωt − λ x −
(B) E = yE λ m

3 π x+ π z π 3πz
t 0 e jc ωt +
(C) E = yE λ λ m t 0 e jc ωt − λ x +
(D) E = yE λ m

SOL 1.19 Hence (A) is correct option.


γ = β cos 30cx ! β sin 30cy
= 2π 3 x ! 2π 1 y
λ 2 λ 2
= π 3 x! πy
λ λ
π 3 x! π y
E = ay E0 e j (ωt - γ) = ay E0 e j;ωt - c λ λ mE

MCQ 1.20 If C is code curve enclosing a surface S , then magnetic field intensity H , the
current density j and the electric flux density D are related by
(A) ##
S
H $ ds =
c
##
c j + 2t m $ d t
2D (B) H $ d l =
S S
#
c j + 2t m $ dS
2D
##
(C) ##S H $ dS = #C c j + 22Dt m $ d t (D) #C H $ d l # = ##S c j + 22Dt m $ ds
c

SOL 1.20 Hence (D) is correct option.


4# H = J + 2D Maxwell Equations
2t
## 4# H $ ds = ##
`J + 2t j .ds
2D Integral form
s s
#
H $ dl = ##
`J + 2t j .ds
2D Stokes Theorem
s

Q.21 to Q.75 carry two marks each.

MCQ 1.21 It is given that X1, X2 ...XM at M non-zero, orthogonal vectors. The dimension of
the vector space spanned by the 2M vectors X1, X2,... XM , − X1, − X2,... − XM is
(A) 2M (B) M + 1
(C) M
(D) dependent on the choice of X1, X2,... XM
SOL 1.21 For two orthogonal vectors, we require two dimensions to define them and similarly

Brought to you by: Nodia and Company Visit us at: www.nodia.co.in


PUBLISHING FOR GATE
Page 8 GATE EC 2007 www.gatehelp.com

for three orthogonal vector we require three dimensions to define them. 2M vectors
are basically M orthogonal vector and we require M dimensions to define them.
Hence (C) is correct answer.
MCQ 1.22 Consider the function f (x) = x2 − x − 2 . The maximum value of f (x) in the closed
interval [− 4, 4] is
(A) 18 (B) 10
(C) − 225 (D) indeterminate
SOL 1.22 We have
f (x) = x2 − x + 2
f'( x) = 2x − 1 = 0 " x = 1
2
f"( x) = 2
Since f"( x) = 2 > 0 , thus x = 1 is minimum point. The maximum value in closed
2
interval 6− 4, 4@ will be at x =− 4 or x = 4
Now maximum value
= max [f (− 4), f (4)]
= max (18, 10)
= 18
Hence (A) is correct answer.
MCQ 1.23 An examination consists of two papers, Paper 1 and Paper 2. The probability of
failing in Paper 1 is 0.3 and that in Paper 2 is 0.2. Given that a student has failed
in Paper 2, the probability of failing in Paper 1 is 0.6. The probability of a student
failing in both the papers is
(A) 0.5 (B) 0.18
(C) 0.12 (D) 0.06
SOL 1.23 Hence (C) is correct answer.
Probability of failing in paper 1 is P (A) = 0.3
Possibility of failing in Paper 2 is P (B) = 0.2
Probability of failing in paper 1, when
student has failed in paper 2 is P ^ BA h = 0.6
We know that
(P + B)
Pb A l =
B P (B)
or P (A + B) = P (B) P b A l = 0.6 # 0.2 = 0.12
B

d2 y
MCQ 1.24 The solution of the differential equation k2 2 = y − y2 under the boundary
dx
conditions
(i) y = y1 at x = 0 and

Brought to you by: Nodia and Company Visit us at: www.nodia.co.in


PUBLISHING FOR GATE
Page 9 GATE EC 2007 www.gatehelp.com

(ii) y = y2 at x = 3 , where k, y1 and y2 are constants, is

(A) y = (y1 − y2) exp a− x2 k + y2 (B) y = (y2 − y1) exp a− x k + y1


k k

(C) y = ^y1 − y2h sinh a x k + y1 (D) y = ^y1 − y2h exp a− x k + y2


k k
SOL 1.24 Hence (D) is correct answer.
d2 y
We have k2 2 = y − y2
dx
2
d y y y
or 2
− 2 =− 22
dx k k
A.E. D − 2 =0
2 1
k
or D =! 1
k
x x

C.F. = C1 e - + C2 e
k k

− y22
c 2 m = y2
P.I. = 1
D2 − 12 k
Thus solution is k
x x

y = C1 e - + C2 e + y2
k k

From y (0) = y1 we get


C1 + C2 = y1 − y2
From y (3) = y2 we get that C1 must be zero.
Thus C2 = y1 − y2
x

y = (y1 − y2) e - + y2
k

MCQ 1.25 The equation x3 − x2 + 4x − 4 = 0 is to be solved using the Newton - Raphson


method. If x = 2 is taken as the initial approximation of the solution, then next
approximation using this method will be
(A) 2/3 (B) 4/3
(C) 1 (D) 3/2
SOL 1.25 We have
f (x) = x3 − x2 + 4x − 4
f'( x) = 3x2 − 2x + 4
Taking x0 = 2 in Newton-Raphosn method
f (x0) 23 − 22 + 4 (2) − 4
x1 = x0 − = 2− =4
f'( x0) 3 (2) − 2 (2) + 4
2 3
Hence (B) is correct answer.
MCQ 1.26 Three functions f1 (t), f2 (t) and f3 (t) which are zero outside the interval [0, T] are
shown in the figure. Which of the following statements is correct?

Brought to you by: Nodia and Company Visit us at: www.nodia.co.in


PUBLISHING FOR GATE
Page 10 GATE EC 2007 www.gatehelp.com

(A) f1 (t) and f2 (t) are orthogonal (B) f1 (t) and f3 (t) are orthogonal
(C) f2 (t) and f3 (t) are orthogonal (D) f1 (t) and f2 (t) are orthonormal
SOL 1.26 For two orthogonal signal f (x) and g (x)
#- 3
+3
f (x) g (x) dx =0

i.e. common area between f (x) and g (x) is zero.


Hence (C) is correct options.
MCQ 1.27 If the semi-circular control D of radius 2 is as shown in the figure, then the value
of the integral 1
(s − 1)
2 #
ds is
D

(A) jπ (B) − jπ
(C) − π (D) π
SOL 1.27 We know that

s
#
2
1 ds = 2πj
− 1
[sum of residues]
D
Singular points are at s = ! 1 but only s =+ 1 lies inside the given contour, Thus
Residue at s =+ 1 is

Brought to you by: Nodia and Company Visit us at: www.nodia.co.in


PUBLISHING FOR GATE
Page 11 GATE EC 2007 www.gatehelp.com

lim (s − 1) f (s) = lim (s − 1) 1 =1


s"1 s"1 s2 − 1 2
# s2 −1 1 ds = 2πj` 12 j = πj
D
Hence (A) is correct answer.
MCQ 1.28 Two series resonant filters are as shown in the figure. Let the 3-dB bandwidth of
Filter 1 be B1 and that of Filter 2 be B2 . the value B1 is
B2

(A) 4 (B) 1
(C) 1/2 (D) 1/4

SOL 1.28 We know that bandwidth of series RLC circuit is R . Therefore


L
Bandwidth of filter 1 is B1 = R
L1
Bandwidth of filter 2 is B2 = R = R = 4R
L2 L1 /4 L1
Dividing above equation B1 = 1
B2 4
Hence (D) is correct option.
MCQ 1.29 For the circuit shown in the figure, the Thevenin voltage and resistance looking
into X − Y are

(A) 4
3 V, 2 Ω (B) 4 V, 23 Ω
(C) 4
3 V, 23 Ω (D) 4 V, 2 Ω
SOL 1.29 Here Vth is voltage across node also. Applying nodal analysis we get

Brought to you by: Nodia and Company Visit us at: www.nodia.co.in


PUBLISHING FOR GATE
Page 12 GATE EC 2007 www.gatehelp.com

Vth + Vth + Vth − 2i = 2


2 1 1
But from circuit i = Vth = Vth
1
Therefore
Vth + Vth + Vth − 2Vth = 2
2 1 1
or Vth = 4 volt
From the figure shown below it may be easily seen that the short circuit current at
terminal XY is isc = 2 A because i = 0 due to short circuit of 1 Ω resistor and all
current will pass through short circuit.

Therefore Rth = Vth = 4 = 2 Ω


isc 2
Hence (D) is correct option.
MCQ 1.30 In the circuit shown, vC is 0 volts at t = 0 sec. For t > 0 , the capacitor current iC (t)
, where t is in seconds is given by

(A) 0.50 exp (− 25t) mA (B) 0.25 exp (− 25t) mA


(C) 0.50 exp (− 12.5t) mA (D) 0.25 exp (− 6.25t) mA
SOL 1.30 The voltage across capacitor is
At t = 0+ , Vc (0+) = 0
At t = 3 , VC (3) = 5 V
The equivalent resistance seen by capacitor as shown in fig is
Req = 20 20 = 10kΩ

Brought to you by: Nodia and Company Visit us at: www.nodia.co.in


PUBLISHING FOR GATE
Page 13 GATE EC 2007 www.gatehelp.com

Time constant of the circuit is


τ = Req C = 10k # 4μ = 0.04 s
Using direct formula
Vc (t) = VC (3) − [Vc (3) − Vc (0)] e−t/τ
= VC (3) (1 − e−t/τ) + VC (0) e−t/τ = 5 (1 − e−t/0.04)
or Vc (t) = 5 (1 − e−25t)
dV (t)
Now IC (t) = C C
dt
= 4 # 10−6 # (− 5 # 25e−25t) = 0.5e−25t mA
Hence (A) is correct option.
MCQ 1.31 In the ac network shown in the figure, the phasor voltage VAB (in Volts) is

(A) 0 (B) 5+30c


(C) 12.5+30c (D) 17+30c
SOL 1.31 Hence (D) is correct option.
(5 − 3j) # (5 + 3j)
Impedance = (5 − 3j) (5 + 3j) =
5 − 3j + 5 + 3j
(5) 2 − (3j) 2
= = 25 + 9 = 3.4
10 10
VAB = Current # Impedance = 5+30c # 34 = 17+30c
MCQ 1.32 A p+ n junction has a built-in potential of 0.8 V. The depletion layer width a reverse
bias of 1.2 V is 2 μm. For a reverse bias of 7.2 V, the depletion layer width will be
(A) 4 μm (B) 4.9 μm
(C) 8 μm (D) 12 μm
SOL 1.32 Hence option (A) is correct.
W = K V + VR
Now 2μ = K 0.8 + 1.2
From above two equation we get

Brought to you by: Nodia and Company Visit us at: www.nodia.co.in


PUBLISHING FOR GATE
Page 14 GATE EC 2007 www.gatehelp.com

W = 0.8 + 7.2 = 8 =2
2μ 0.8 + 1.2 2
or W2 = 4 μ m
MCQ 1.33 Group I lists four types of p − n junction diodes. Match each device in Group I
with one of the option in Group II to indicate the bias condition of the device in
its normal mode of operation.
Group - I Group-II
(P) Zener Diode (1) Forward bias
(Q) Solar cell (2) Reverse bias
(R) LASER diode
(S) Avalanche Photodiode

(A) P - 1, Q - 2, R - 1, S - 2 (B) P - 2, Q - 1, R - 1, S - 2
(C) P - 2, Q - 2, R - 1, S- -2 (D) P - 2, Q - 1, R - 2, S - 2
SOL 1.33 Zener diode and Avalanche diode works in the reverse bias and laser diode works
in forward bias.
In solar cell diode works in forward bias but photo current is in reverse direction.
Thus
Zener diode : Reverse Bias
Solar Cell : Forward Bias
Laser Diode : Forward Bias
Avalanche Photo diode : Reverse Bias
Hence option (B) is correct.
MCQ 1.34 The DC current gain (β) of a BJT is 50. Assuming that the emitter injection
efficiency is 0.995, the base transport factor is
(A) 0.980 (B) 0.985
(C) 0.990 (D) 0.995
SOL 1.34 Hence option (B) is correct.
β
α= = 50 = 50
β + 1 50 + 1 51
Current Gain = Base Transport Factor # Emitter injection Efficiency
α = β1 # β2
or β1 = α = 50 = 0.985
β2 51 # 0.995

MCQ 1.35 Group I lists four different semiconductor devices. match each device in Group I
with its charactecteristic property in Group II
Group-I Group-II
(P) BJT (1) Population iniversion
(Q) MOS capacitor (2) Pinch-off voltage

Brought to you by: Nodia and Company Visit us at: www.nodia.co.in


PUBLISHING FOR GATE
Page 15 GATE EC 2007 www.gatehelp.com

(R) LASER diode (3) Early effect


(S) JFET (4) Flat-band voltage

(A) P - 3, Q - 1, R - 4, S - 2 (B) P - 1, Q - 4, R - 3, S - 2
(C) P - 3, Q - 4, R - 1, S - 2 (D) P - 3, Q - 2, R - 1, S - 4
SOL 1.35 In BJT as the B-C reverse bias voltage increases, the B-C space charge region width
increases which xB (i.e. neutral base width) > A change in neutral base width will
change the collector current. A reduction in base width will causes the gradient in
minority carrier concentration to increase, which in turn causes an increased in the
diffusion current. This effect si known as base modulation as early effect.
In JFET the gate to source voltage that must be applied to achieve pinch off
voltage is described as pinch off voltage and is also called as turn voltage or
threshold voltage.
In LASER population inversion occurs on the condition when concentration of
electrons in one energy state is greater than that in lower energy state, i.e. a non
equilibrium condition.
In MOS capacitor, flat band voltage is the gate voltage that must be applied to
create flat ban condition in which there is no space charge region in semiconductor
under oxide.
Therefore
BJT : Early effect
MOS capacitor: Flat-band voltage
LASER diode : Population inversion
JFET : Pinch-off voltage
Hence option (C) is correct.
MCQ 1.36 For the Op-Amp circuit shown in the figure, V0 is

(A) -2 V (B) -1 V
(C) -0.5 V (D) 0.5 V
SOL 1.36 We redraw the circuit as shown in fig.

Brought to you by: Nodia and Company Visit us at: www.nodia.co.in


PUBLISHING FOR GATE
Page 16 GATE EC 2007 www.gatehelp.com

Applying voltage division rule


v+ = 0.5 V
We know that v+ = v-
Thus v- = 0.5 V
Now i = 1 − 0.5 = 0.5 mA
1k
and i = 0.5 − v0 = 0.5 mA
2k
or v0 = 0.5 − 1 =− 0.5 V
Hence (C) is correct option.
MCQ 1.37 For the BJT circuit shown, assume that the β of the transistor is very large and
VBE = 0.7 V. The mode of operation of the BJT is

(A) cut-off (B) saturation


(C) normal active (D) reverse active
SOL 1.37 If we assume β very large, then IB = 0 and IE = IC ; VBE = 0.7 V. We assume that
BJT is in active, so applying KVL in Base-emitter loop
IE = 2 − VBE = 2 − 0.7 = 1.3 mA
RE 1k
Since β is very large, we have IE = IC , thus
IC = 1.3 mA
Now applying KVL in collector-emitter loop
10 − 10IC − VCE − IC =0
or VCE =− 4.3 V
Now VBC = VBE − VCE
= 0.7 − (− 4.3) = 5 V
Since VBC > 0.7 V, thus transistor in saturation.
Hence (B) is correct option

Brought to you by: Nodia and Company Visit us at: www.nodia.co.in


PUBLISHING FOR GATE
GATE ESE PSU’s 2019-20
ECE ENGINEERING
GATE ECE 2003-2019 SOLVED

GATE ECE 2003-2019 SOLVED Detail Solution

CONTENT COVERED:
1.Theory Notes
2.Explanation
3.Derivation
4.Example
5.Shortcut & Formula Summary
6.Previous year Paper Q. Sol.
Noted-: Single Source Follow, Revise
Multiple Time Best key of Success
1
Page

http://www.orbitmentor.com [email protected]
Page 17 GATE EC 2007 www.gatehelp.com

MCQ 1.38 In the Op-Amp circuit shown, assume that the diode current follows the equation
I = Is exp (V/VT ). For Vi = 2V, V0 = V01, and for Vi = 4V, V0 = V02 .
The relationship between V01 and V02 is

(A) V02 = 2 Vo1 (B) Vo2 = e2 Vo1


(C) Vo2 = Vo1 1n2 (D) Vo1 − Vo2 = VT 1n2
SOL 1.38 Here the inverting terminal is at virtual ground and the current in resistor and
diode current is equal i.e.
IR = ID
or Vi = I eV /V
D T
s
R
or VD = VT 1n Vi
Is R
For the first condition
VD = 0 − Vo1 = VT 1n 2
Is R
For the first condition
VD = 0 − Vo1 = VT 1n 4
Is R
Subtracting above equation
Vo1 − Vo2 = VT 1n 4 − VT 1n 2
Is R Is R
or Vo1 − Vo2 = VT 1n 4 = VT 1n2
2
Hence (D) is correct option.
MCQ 1.39 In the CMOS inverter circuit shown, if the trans conductance parameters of the
NMOS and PMOS transistors are
W
kn = kp = μn Cox Wn = μCox p = 40μA/V2
Ln Lp
and their threshold voltages ae VTHn = VTHp = 1 V the current I is

Brought to you by: Nodia and Company Visit us at: www.nodia.co.in


PUBLISHING FOR GATE
Page 18 GATE EC 2007 www.gatehelp.com

(A) 0 A (B) 25 μA
(C) 45 μA (D) 90 μA
SOL 1.39 Hence (D) is correct option
We have Vthp = Vthp = 1 V
W W
and P
= N = 40μA/V2
LP LN
From figure it may be easily seen that Vas for each NMOS and PMOS is 2.5 V
μA
Thus ID = K (Vas − VT ) 2 = 40 2 (2.5 − 1) 2 = 90 μ A
V
MCQ 1.40 For the Zener diode shown in the figure, the Zener voltage at knee is 7 V, the knee
current is negligible and the Zener dynamic resistance is 10 Ω. If the input voltage
(Vi) range is from 10 to 16 V, the output voltage (V0) ranges from

(A) 7.00 to 7.29 V (B) 7.14 to 7.29 V


(C) 7.14 to 7.43 V (D) 7.29 to 7.43 V
SOL 1.40 We have VZ = 7 volt, VK = 0, RZ = 10Ω
Circuit can be modeled as shown in fig below

Since Vi is lies between 10 to 16 V, the range of voltage across 200 kΩ


V200 = Vi − VZ = 3 to 9 volt
The range of current through 200 kΩ is
3 = 15 mA to 9 = 45 mA
200k 200k
The range of variation in output voltage
15m # RZ = 0.15 V to 45m # RZ = 0.45
Thus the range of output voltage is 7.15 Volt to 7.45 Volt
Hence (C) is correct option.
MCQ 1.41 The Boolean expression Y = ABC D + ABCD + ABC D + ABC D can be minimized
to

Brought to you by: Nodia and Company Visit us at: www.nodia.co.in


PUBLISHING FOR GATE
Page 19 GATE EC 2007 www.gatehelp.com

(A) Y = ABC D + ABC + AC D (B) Y = ABC D + BCD + ABC D


(C) Y = ABCD + BC D + ABC D (D) Y = ABCD + BC D + ABC D
SOL 1.41 Hence (D) is correct answer.
Y = ABCD + ABCD + ABC D + ABC D
= ABCD + ABC D + ABC D + ABC D
= ABCD + ABC D + BC D (A + A)
= ABCD + ABC D + BC D A+A = 1
MCQ 1.42 The circuit diagram of a standard TTL NOT gate is shown in the figure. Vi = 25
V, the modes of operation of the transistors will be

(A) Q1: revere active; Q2: normal active; Q3: saturation; Q4: cut-off
(B) Q1: revere active; Q2: saturation; Q3: saturation; Q4: cut-off
(C) Q1: normal active; Q2: cut-off; Q3: cut-off; Q4: saturation
(D) Q1: saturation; Q2: saturation; Q3: saturation; Q4: normal active
SOL 1.42 In given TTL NOT gate when Vi = 2.5 (HIGH), then
Q1 " Reverse active
Q2 " Saturation
Q3 " Saturation
Q4 " cut - off region
Hence (B) is correct answer.
MCQ 1.43 In the following circuit, X is given by

Brought to you by: Nodia and Company Visit us at: www.nodia.co.in


PUBLISHING FOR GATE
Page 20 GATE EC 2007 www.gatehelp.com

(A) X = ABC + ABC + ABC + ABC


(B) X = ABC + ABC + ABC + ABC
(C) X = AB + BC + AC
(D) X = AB + BC + AC
SOL 1.43 The circuit is as shown below

Y = AB + AB
and X = YC + YC
= (AB + AB ) C + (AB + AB ) C
= (AB + AB) C + (AB + AB ) C
= ABC + ABC + ABC + ABC
Hence (A) is correct answer.
MCQ 1.44 The following binary values were applied to the X and Y inputs of NAND latch
shown in the figure in the sequence indicated below :
X = 0,Y = 1; X = 0, Y = 0; X = 1; Y = 1
The corresponding stable P, Q output will be.

(A) P = 1, Q = 0; P = 1, Q = 0; P = 1, Q = 0 or P = 0, Q = 1
(B) P = 1, Q = 0; P = 0, Q = 1; or P = 0, Q = 1; P = 0, Q = 1
(C) P = 1, Q = 0; P = 1, Q = 1; P = 1, Q = 0 or P = 0, Q = 1
(D) P = 1, Q = 0; P = 1, Q = 1; P = 1, Q = 1
SOL 1.44 Hence (C) is correct answer.
For X = 0, Y = 1 P = 1, Q = 0
For X = 0, Y = 0 P = 1, Q = 1
For X = 1, Y = 1 P = 1, Q = 0 or P = 0, Q = 1

MCQ 1.45 For the circuit shown, the counter state (Q1 Q0) follows the sequence

Brought to you by: Nodia and Company Visit us at: www.nodia.co.in


PUBLISHING FOR GATE
Page 21 GATE EC 2007 www.gatehelp.com

(A) 00, 01, 10, 11, 00 (B) 00, 01, 10, 00, 01
(C) 00, 01, 11, 00, 01 (D) 00, 10, 11, 00, 10
SOL 1.45 For this circuit the counter state (Q1, Q0) follows the sequence 00, 01, 10, 00 ... as
shown below

Clock D1 D0 Q1 Q0 Q1 NOR Q0
00 1
1st 01 10 0
2nd 10 01 0
3rd 00 00 0

Hence (A) is correct answer.


MCQ 1.46 An 8255 chip is interfaced to an 8085 microprocessor system as an I/O mapped I/O
as show in the figure. The address lines A0 and A1 of the 8085 are used by the 8255
chip to decode internally its thee ports and the Control register. The address lines
A3 to A7 as well as the IO/M signal are used for address decoding. The range of
addresses for which the 8255 chip would get selected is

(A) F8H - FBH (B) F8GH - FCH


(C) F8H - FFH (D) F0H - F7H
SOL 1.46 Chip 8255 will be selected if bits A3 to A7 are 1. Bit A0 to A2 can be 0 or.
1. Thus address range is

Brought to you by: Nodia and Company Visit us at: www.nodia.co.in


PUBLISHING FOR GATE
Page 22 GATE EC 2007 www.gatehelp.com

11111000 F8H
11111111 FFH
Hence (C) is correct answer.
MCQ 1.47 The 3-dB bandwidth of the low-pass signal e−t u (t), where u (t) is the unit step
function, is given by
(A) 1 Hz (B) 1 2 − 1 Hz
2π 2π
(C) 3 (D) 1 Hz
SOL 1.47 Hence (A) is correct answer.
x (t) = e−t u (t)
Taking Fourier transform
X (jω) = 1
1 + jω
X (jω) = 1 2
1+ω
Magnitude at 3dB frequency is 1
2
Thus 1 = 1
2 1 + ω2
or ω = 1 rad
or f = 1 Hz

MCQ 1.48 A Hilbert transformer is a


(A) non-linear system (B) non-causal system
(C) time-varying system (D) low-pass system
SOL 1.48 A Hilbert transformer is a non-linear system.
Hence (A) is correct answer.
MCQ 1.49 The frequency response of a linear, time-invariant system is given by
H (f) = 1 + j510πf . The step response of the system is
(B) 5 61 − e− 5@ u (t)
t
(A) 5 (1 − e−5t) u (t)
(C) 1 (1 − e−5t) u (t) (D) 1 ^1 − e− 5 h u (t)
t

2 5
SOL 1.49 Hence (B) is correct answer.
H (f) = 5
1 + j10πf
H (s) = 5 = 5 = 1
1 + 5s 5^s + 15 h s + 15
Step response Y (s) = 1 a 1
s ^s + 5 h
or Y (s) = 1 1 1 = 5 − 5 1
s ^s + 5 h s s+ 5
Brought to you by: Nodia and Company Visit us at: www.nodia.co.in
PUBLISHING FOR GATE
Page 23 GATE EC 2007 www.gatehelp.com

or y (t) = 5 (1 − e−t/5) u (t)

MCQ 1.50 A 5-point sequence x [n] is given as x [− 3] = 1, x [− 2] = 1, x [− 1] = 0, x [0] = 5 and


x [1] = 1. Let X (eiω) denoted the discrete-time Fourier transform of x [n]. The value
π
of #
−π
X (e jω) dω is

(A) 5 (B) 10π


(C) 16π (D) 5 + j10π
SOL 1.50 For discrete time Fourier transform (DTFT) when N " 3
π
x [n] = 1
2π − π #
X (e jω) e jωn dω

Putting n = 0 we get
π π
x [0] = 1
2π #
−π
X (e jω) e jω0 dω = 1
2π #
−π
X (e jω) dω
π
or #
−π
X (e jω) dω = 2πx [0] = 2π # 5 = 10π

Hence (B) is correct answer.


MCQ 1.51 The z −transform X (z) of a sequence x [n] is given by X [z] = 1 −0.25z . It is given that −1

the region of convergence of X (z) includes the unit circle. The value of x [0] is
(A) − 0.5 (B) 0
(C) 0.25 (D) 05
SOL 1.51 Hence (B) is correct answer.
X (z) = 0.5 −1
1 − 2z
Since ROC includes unit circle, it is left handed system
x (n) =− (0.5) (2) −n u (− n − 1)
x (0) = 0
If we apply initial value theorem
x (0) = lim X (z) = lim 0.5 −1 = 0.5
z"3 z " 31 − 2z

That is wrong because here initial value theorem is not applicable because signal
x (n) is defined for n < 0 .
MCQ 1.52 A control system with PD controller is shown in the figure. If the velocity error
constant KV = 1000 and the damping ratio ζ = 0.5 , then the value of KP and KD
are

Brought to you by: Nodia and Company Visit us at: www.nodia.co.in


PUBLISHING FOR GATE
Page 24 GATE EC 2007 www.gatehelp.com

(A) KP = 100, KD = 0.09 (B) KP = 100, KD = 0.9


(C) KP = 10, KD = 0.09 (D) KP = 10, KD = 0.9
SOL 1.52 Hence (B) is correct option
We have Kv = lim sG (s) H (s)
s"0

(Kp + KD s) 100
or 1000 = lim s = Kp
s"0 s (s + 100)
Now characteristics equations is
1 + G (s) H (s) = 0
(K + KD s) 100
1000 = lims " 0 s p = Kp
s (s + 100)
Now characteristics equation is
1 + G (s) H (s) = 0

(100 + KD s) 100
or 1+ =0 Kp = 100
s (s + 10)
or s2 + (10 + 100KD) s + 10 4 = 0

Comparing with s2 + 2ξωn + ωn2 = 0 we get


2ξωn = 10 + 100KD
or KD = 0.9
MCQ 1.53 The transfer function of a plant is
T (s) = 5
(s + 5)( s2 + s + 1)
The second-order approximation of T (s) using dominant pole concept is
(A) 1 (B) 5
(s + 5)( s + 1) (s + 5)( s + 1)

(C) 5 (D) 1
s2 + s + 1 s2 + s + 1
SOL 1.53 Hence (D) is correct option.
We have T (s) = 5
(s + 5)( s2 + s + 1)
= 5 = 2 1
5`1 + j (s + s + 1)
s 2
s +s+1
5
In given transfer function denominator is (s + 5)[( s + 0.5) 2 + 43 ]. We can see easily
that pole at s =− 0.5 ! j 23 is dominant then pole at s =− 5 . Thus we have
approximated it.
MCQ 1.54 The open-loop transfer function of a plant is given as G (s) = s 1- 1 . If the plant is
2

operated in a unity feedback configuration, then the lead compensator that an


stabilize this control system is

Brought to you by: Nodia and Company Visit us at: www.nodia.co.in


PUBLISHING FOR GATE
Page 25 GATE EC 2007 www.gatehelp.com

10 (s − 1) 10 (s + 4)
(A) (B)
s+2 s+2
10 (s + 2) 2 (s + 2)
(C) (D)
s + 10 s + 10
SOL 1.54 Hence (A) is correct option.
G (s) = 2 1 = 1
s −1 (s + 1)( s − 1)
The lead compensator C (s) should first stabilize the plant i.e. remove 1 term.
(s − 1)
From only options (A), C (s) can remove this term

1 10 (s − 1)
ThusG (s) C (s) = #
(s + 1)( s − 1) (s + 2)
= 10 Only option (A) satisfies.
(s + 1)( s + 2)

MCQ 1.55 A unity feedback control system has an open-loop transfer function
G (s) = K
s (s + 7s + 12)
2

The gain K for which s = 1 + j1 will lie on the root locus of this system is
(A) 4 (B) 5.5
(C) 6.5 (D) 10
SOL 1.55 For ufb system the characteristics equation is
1 + G (s) = 0
or 1+ K =0
s (s2 + 7s + 12)
or s (s2 + 7s + 12) + K = 0
Point s =− 1 + j lie on root locus if it satisfy above equation i.e
(− 1 + j)[( − 1 + j) 2 + 7 (− 1 + j) + 12) + K] = 0
or K =+ 10
Hence (D) is correct option.
MCQ 1.56 The asymptotic Bode plot of a transfer function is as shown in the figure. The
transfer function G (s) corresponding to this Bode plot is

(A) 1 (B) 1
(s + 1)( s + 20) s (s + 1)( s + 20)
Brought to you by: Nodia and Company Visit us at: www.nodia.co.in
PUBLISHING FOR GATE
Page 26 GATE EC 2007 www.gatehelp.com

(C) 100 (D) 100


s (s + 1)( s + 20) s (s + 1)( 1 + 0.05s)
SOL 1.56 At every corner frequency there is change of -20 db/decade in slope which indicate
pole at every corner frequency. Thus
G (s) = K
s (1 + s)`1 + s j
20
Bode plot is in (1 + sT) form
20 log K = 60 dB = 1000
ω ω = 0. 1
Thus K =5
Hence G (s) = 100
s (s + 1)( 1 + .05s)
Hence (D) is correct option.
MCQ 1.57 The state space representation of a separately excited DC servo motor dynamics
is given as

−1 1 ω
> di H = =− 1 − 10G=ia G + =10Gu
dt 0
o
dt

where ω is the speed of the motor, ia is the armature current and u is the armature
ω (s)
voltage. The transfer function of the motor is
U (s)
(A) 2 10 (B) 2 1
s + 11s + 11 s + 11s + 11
(C) 2 10s + 10 (D) 2 1
s + 11s + 11 s + s + 11

SOL 1.57 Hence (A) is correct option.



−1 1 ω
We have > didt H = =
0
a G=
− 1 − 10 in G + =10G
u
dt

or dω =− ω + i ...(1)
n
dt
and dia =− ω − 10i + 10u ...(2)
a
dt
Taking laplace transform (i) we get
sω (s) =− ω (s) = Ia (s)
or (s + 1) ω (s) = Ia (s) ...(3)
Taking laplace transform (ii) we get
sIa (s) =− ω (s) − 10Ia (s) + 10U (s)
or ω (s) = (− 10 − s) Ia (s) + 10U (s)
= (− 10 − s)( s + 1) ω (s) + 10U (s) From (3)
or ω (s) =− [s2 + 11s + 10] ω (s) + 10U (s)

Brought to you by: Nodia and Company Visit us at: www.nodia.co.in


PUBLISHING FOR GATE
Page 27 GATE EC 2007 www.gatehelp.com

or(s2 + 11s + 11) ω (s) = 10U (s)


ω (s)
or = 2 10
U (s) (s + 11s + 11)
MCQ 1.58 In delta modulation, the slope overload distortion can be reduced by
(A) decreasing the step size (B) decreasing the granular noise
(C) decreasing the sampling rate (D) increasing the step size
SOL 1.58 Slope overload distortion can be reduced by increasing the step size
3 $ slope of x (t)
Ts
Hence (D) is correct option.
MCQ 1.59 The raised cosine pulse p (t) is used for zero ISI in digital communications. The
expression for p (t) with unity roll-off factor is given by
p (t) = sin 4πWt
4πWt (1 − 16W2 t2)
The value of p (t) at t = 1 is
4W
(A) − 0.5 (B) 0
(C) 0.5 (D) 3
SOL 1.59 Hence (C) is correct option.
sin (4πWt)
We have p (t) =
4πWt (1 − 16W2 t2)
at t = 1 it is 0 form. Thus applying L' Hospital rule
4W 0
4πW cos (4πWt)
p( ) =
1
4W

4πW [1 − 48W2 t2]


cos (4πWt)
= = cos π = 0.5
1 − 48W t
2 2 1−3

MCQ 1.60 In the following scheme, if the spectrum M (f) of m (t) is as shown, then the spectrum
Y (f) of y (t) will be

Brought to you by: Nodia and Company Visit us at: www.nodia.co.in


PUBLISHING FOR GATE
Page 28 GATE EC 2007 www.gatehelp.com

SOL 1.60 The block diagram is as shown below

Here M1 (f) = Mt (f)


j 2π B
Y1 (f) = M (f) c e + e -j2πB
2 m
j 2π B
Y2 (f) = M1 (f) c e − e -j2πB
2 m

Y (f) = Y1 (f) + Y2 (f)


All waveform is shown below

Brought to you by: Nodia and Company Visit us at: www.nodia.co.in


PUBLISHING FOR GATE
Page 29 GATE EC 2007 www.gatehelp.com

Hence (B) is correct option.


MCQ 1.61 During transmission over a certain binary communication channel, bit errors occur
independently with probability p. The probability of AT MOST one bit in error
in a block of n bits is given by
(A) pn (B) 1 − pn
(C) np (1 − p) n - 1 + (1 + p) n (D) 1 − (1 − p) n
SOL 1.61 By Binomial distribution the probability of error is
pe = n Cr pr (1 − p) n - r
Probability of at most one error
= Probability of no error + Probability of one error
= n C0 p0 (1 − p) n - 0 + n C1 p1 (1 − p) n - 1

= (1 − p) n + np (1 − p) n - 1
Hence (C) is correct option.
MCQ 1.62 In a GSM system, 8 channels can co-exist in 200 kHz bandwidth using TDMA.
A GSM based cellular operator is allocated 5 MHz bandwidth. Assuming a
frequency reuse factor of 1 , i.e. a five-cell repeat pattern, the maximum number of
5
simultaneous channels that can exist in one cell is
(A) 200 (B) 40
(C) 25 (D) 5
SOL 1.62 Bandwidth allocated for 1 Channel = 5 M Hz
Average bandwidth for 1 Channel 5 = 1 MHz
5
Total Number of Simultaneously Channel = 1M # 8 = 40 Channel
200k
Hence (B) is correct option.
MCQ 1.63 In a Direct Sequence CDMA system the chip rate is 1.2288 # 106 chips per second.
If the processing gain is desired to be AT LEAST 100, the data rate
(A) must be less than or equal to 12.288 # 103 bits per sec
(B) must be greater than 12.288 # 103 bits per sec
(C) must be exactly equal to 12.288 # 103 bits per sec
(D) can take any value less than 122.88 # 103 bits per sec
SOL 1.63 Hence (A) is correct option.
Chip Rate RC = 1.2288 # 106 chips/sec

Brought to you by: Nodia and Company Visit us at: www.nodia.co.in


PUBLISHING FOR GATE
Page 30 GATE EC 2007 www.gatehelp.com

Data Rate Rb = RC
G
Since the processing gain G must be at least 100, thus for Gmin we get
6
Rb max = RC = 1.2288 # 10 = 12.288 # 103 bps
Gmin 100

MCQ 1.64 An air-filled rectangular waveguide has inner dimensions of 3 cm # 2 cm. The wave
impedance of the TE20 mode of propagation in the waveguide at a frequency of 30
GHz is (free space impedance η0 = 377 Ω )
(A) 308 Ω (B) 355 Ω
(C) 400 Ω (D) 461 Ω
SOL 1.64 The cut-off frequency is
fc = c ` m j2 + ` n j2
2 a b
Since the mode is TE20, m = 2 and n = 0
8
fc = c m = 3 # 10 # 2 = 10 GHz
2 2 2 # 0.03
ηo 377
η' = fc 2
= 10 = 400Ω
1−c m
2
1 − c 10 10 m
f 3 # 10
Hence (C) is correct option.
MCQ 1.65 The H field (in A/m) of a plane wave propagating in free space is given by
H = xt 5 3 cos (ωt − βz) + yt`ωt − βz + π j .
η0 2
The time average power flow density in Watts is
η
(A) 0 (B) 100
100 η0

(C) 50η20 (D) 50


η0
SOL 1.65 Hence (D) is correct option.
2
= Hx2 + Hy2 = c 5 3 m + c 5 m = c 10 m
2 2
We have H 2
ηo ηo ηo
E 2 ηo H 2
η 2
For free space P = = = o c 10 m = 50 watts
2ηo 2 2 ηo ηo

MCQ 1.66 The E field in a rectangular waveguide of inner dimension a # b is given by


ωμ
E = 2 ` λ j H0 sin ` 2πx j sin (ωt − βz) yt
2

h 2 a
Where H0 is a constant, and a and b are the dimensions along the x − axis and the
y − axis respectively. The mode of propagation in the waveguide is
(A) TE20 (B) TM11
(C) TM20 (D) TE10

Brought to you by: Nodia and Company Visit us at: www.nodia.co.in


PUBLISHING FOR GATE
Page 31 GATE EC 2007 www.gatehelp.com

SOL 1.66 Hence (A) is correct option.


ωμ
E = 2 ` π j H0 sin ` 2πx j sin (ωt − βz) yt
2

h 2 a
This is TE mode and we know that
mπy
Ey \ sin ` mπx j cos `
a b j
Thus m = 2 and n = 0 and mode is TE20
MCQ 1.67 A load of 50 Ω is connected in shunt in a 2-wire transmission line of Z0 = 50Ω as
shown in the figure. The 2-port scattering parameter matrix (s-matrix) of the shunt
element is

−1 1
(A) > 12 1H
0 1
(B) =
1 0G
2
2 − 2

− 13 2 1
− 43
(C) > 3
H (D) > 4
1H
2
3 − 1
3 −3
4 4
SOL 1.67 The 2-port scattering parameter matrix is
S11 S12
S ==
S21 S22 G
(Z Z ) − Zo (50 50) − 50
S11 = L 0 = =− 1
(ZL Z0) + Zo (50 50) + 50 3
2 (ZL Zo) 2 (50 50)
S12 = S21 = = =2
(ZL Zo) + Zo (50 50) + 50 3
(Z Z ) − Zo (50 50) − 50
S22 = L o = =− 1
(ZL Zo) + Zo (50 50) + 50 3
Hence (C) is correct option.
MCQ 1.68 The parallel branches of a 2-wire transmission line re terminated in 100Ω and
200Ω resistors as shown in the figure. The characteristic impedance of the line is
Z0 = 50Ω and each section has a length of λ . The voltage reflection coefficient Γ
4
at the input is

Brought to you by: Nodia and Company Visit us at: www.nodia.co.in


PUBLISHING FOR GATE
Page 32 GATE EC 2007 www.gatehelp.com

(A) − j 7 (B) − 5
5 7

(C) j 5 (D) 5
7 7
SOL 1.68 The input impedance is
if l = λ
2
Zin = Zo ;
ZL 4
2 2
Zin1 = Zo1 = 50 = 25
ZL1 100
2 2
Zin2 = Zo2 = 50 = 12.5
ZL2 200
Now ZL = Zin1 Zin2
25 12.5 = 25
3
(50) 2
Zs = = 300
25/3
Γ = ZS − Zo = 300 − 50 = 5
ZS + Zo 300 + 50 7
Hence (D) is correct option.

MCQ 1.69 A λ dipole is kept horizontally at a height of λ0 above a perfectly conducting


2 2
infinite ground plane. The radiation pattern in the lane of the dipole (E plane)
looks approximately as

Brought to you by: Nodia and Company Visit us at: www.nodia.co.in


PUBLISHING FOR GATE
Page 33 GATE EC 2007 www.gatehelp.com

SOL 1.69 Using the method of images, the configuration is as shown below

Here d = λ, α = π, thus βd = 2π

Array factor is
βd cos ψ + α
= cos ; E
2
2π cos ψ + π
= cos ; E = sin (π cos ψ)
2
Hence (B) is correct option.
MCQ 1.70 A right circularly polarized (RCP) plane wave is incident at an angle 60c to the
normal, on an air-dielectric interface. If the reflected wave is linearly polarized, the
relative dielectric constant ξr2 is.

(A) 2 (B) 3
(C) 2 (D) 3
SOL 1.70 The Brewster angle is
tan θn = εr2
εr1
tan 60c = εr2
1
or εr2 = 3

Brought to you by: Nodia and Company Visit us at: www.nodia.co.in


PUBLISHING FOR GATE
Page 34 GATE EC 2007 www.gatehelp.com

Hence (D) is correct option.

Common Data Questions

Common Data for Questions 71, 72, 73 :


The figure shows the high-frequency capacitance - voltage characteristics of Metal/
Sio 2 /silicon (MOS) capacitor having an area of 1 # 10 - 4 cm 2 . Assume that the
permittivities (ε0 εr ) of silicon and Sio2 are 1 # 10 - 12 F/cm and 3.5 # 10 - 13 F/cm
respectively.

MCQ 1.71 The gate oxide thickness in the MOS capacitor is


(A) 50 nm (B) 143 nm
(C) 350 nm (D) 1 μm
SOL 1.71 At low voltage when there is no depletion region and capacitance is decide by SiO2
thickness only,
C = ε0 εr1 A
D
−13
or D = ε0 εr1 A = 3.5 # 10 −# 10−4 = 50 nm
C 7 # 10 12
Hence option (A) is correct
MCQ 1.72 The maximum depletion layer width in silicon is
(A) 0.143 μm (B) 0.857 μm
(C) 1 μm (D) 1.143 μm
SOL 1.72 The construction of given capacitor is shown in fig below

Brought to you by: Nodia and Company Visit us at: www.nodia.co.in


PUBLISHING FOR GATE
Page 35 GATE EC 2007 www.gatehelp.com

When applied voltage is 0 volts, there will be no depletion region and we get
C1 = 7 pF
When applied voltage is V , a depletion region will be formed as shown in fig an
total capacitance is 1 pF. Thus
CT = 1 pF
or CT = C1 C2 = 1 pF
C1 + C2
or 1 = 1 + 1
CT C1 C2
Substituting values of CT and C1 we get
C2 = 7 pF
6
D2 = ε0 εr2 A = 1 # 710 #- 1210 = 6 # 10 - 4 cm
- 12 -4
Now
C2 6 # 10
7
= 0.857 μm
Hence option (B) is correct.
MCQ 1.73 Consider the following statements about the C − V characteristics plot :
S1 : The MOS capacitor has as n -type substrate
S2 : If positive charges are introduced in the oxide, the C − V polt will shift to the
left.
Then which of the following is true?
(A) Both S1 and S2 are true
(B) S1 is true and S2 is false
(C) S1 is false and S2 is true
(D) Both S1 and S2 are false
SOL 1.73 Depletion region will not be formed if the MOS capacitor has n type substrate but
from C-V characteristics, C reduces if V is increased. Thus depletion region must
be formed. Hence S1 is false
If positive charges is introduced in the oxide layer, then to equalize the effect the
applied voltage V must be reduced. Thus the C − V plot moves to the left. Hence
S2 is true.
Hence option (C) is correct.

Brought to you by: Nodia and Company Visit us at: www.nodia.co.in


PUBLISHING FOR GATE
Page 36 GATE EC 2007 www.gatehelp.com

Common Data for Questions 74 & 75 :


Two 4-array signal constellations are shown. It is given that φ1 and φ2 constitute
an orthonormal basis for the two constellation. Assume that the four symbols in
both the constellations are equiprobable. Let N0 denote the power spectral density
2
of white Gaussian noise.

MCQ 1.74 The if ratio or the average energy of Constellation 1 to the average energy of
Constellation 2 is
(A) 4a2 (B) 4
(C) 2 (D) 8
SOL 1.74 Energy of constellation 1 is
Eg1 = (0) 2 + (− 2 a) 2 + (− 2 a) 2 + ( 2 a) 2 + (− 2 2 a) 2

= 2a2 + 2a2 + 2a2 + 8a2 = 16a2


Energy of constellation 2 is
Eg2 = a2 + a2 + a2 + a2 = 4a2

Eg1 16a2
Ratio = = =4
Eg2 4a2
Hence (B) is correct option.
MCQ 1.75 If these constellations are used for digital communications over an AWGN channel,
then which of the following statements is true ?
(A) Probability of symbol error for Constellation 1 is lower
(B) Probability of symbol error for Constellation 1 is higher
(C) Probability of symbol error is equal for both the constellations
(D) The value of N0 will determine which of the constellations has a lower probability
of symbol error

SOL 1.75 Noise Power is same for both which is N0 .


2
Thus probability of error will be lower for the constellation 1 as it has higher signal
energy.
Hence (A) is correct option.

Brought to you by: Nodia and Company Visit us at: www.nodia.co.in


PUBLISHING FOR GATE
Page 37 GATE EC 2007 www.gatehelp.com

Linked Answer Questions : Q.76 to Q.85 carry two marks each.

Statement for Linked Answer Questions 76 & 77:


Consider the Op-Amp circuit shown in the figure.

MCQ 1.76 The transfer function V0 (s)/ Vi (s) is


(A) 1 − sRC (B) 1 + sRC
1 + sRC 1 − sRC

(C) 1 (D) 1
1 − sRC 1 + sRC
SOL 1.76 The voltage at non-inverting terminal is
1
V+ = sC 1 Vi = 1 V
R + sC 1 + sCR i
Now V- = V+ = 1 V
1 + sCR i
Applying voltage division rule
(V + Vi)
V+ = R1 (V0 + Vi) = o
R1 + R1 2
1 (V + Vi)
or Vi = o
1 + sCR 2
or Vo =− 1 + 2
Vi 1 + sRC
V0 = 1 − sRC
Vi 1 + sRC
Hence (A) is correct option.

MCQ 1.77 If Vi = V1 sin (ωt) and V0 = V2 sin (ωt + φ), then the minimum and maximum values
of φ (in radians) are respectively
(A) − π and π (B) 0 and π
2 2 2

(C) − π and 0 (D) − π and 0


2
SOL 1.77 Hence (C) is correct option.
V0 = H (s) = 1 − sRC
Vi 1 + sRC
Brought to you by: Nodia and Company Visit us at: www.nodia.co.in
PUBLISHING FOR GATE
Page 38 GATE EC 2007 www.gatehelp.com

1 − jωRC
H (jω) =
1 + jωRC
+H (jω) = φ =− tan - 1 ωRC − tan - 1 ωRC
=− 2 tan - 2 ωRC
Minimum value, φmin = − π (at ω " 3)
Maximum value, φmax = 0( at ω = 0)

Statement for Linked Answer Questions 78 & 79 :


An 8085 assembly language program is given below.
Line 1: MVI A, B5H
2: MVI B, OEH
3: XRI 69H
4: ADD B
5: ANI 9BH
6: CPI 9FH
7: STA 3010H
8: HLT
MCQ 1.78 The contents of the accumulator just execution of the ADD instruction in line 4
will be
(A) C3H (B) EAH
(C) DCH (D) 69H
SOL 1.78 Line 1 : MVI A, B5H ; Move B5H to A
2 : MVI B, 0EH ; Move 0EH to B
3 : XRI 69H ; [A] XOR 69H and store in A
; Contents of A is CDH
4 : ADDB ; Add the contents of A to contents of B and
; store in A, contents of A is EAH
5 : ANI 9BH ; [a] AND 9BH, and store in A,
; Contents of A is 8 AH
6 : CPI 9FH ; Compare 9FH with the contents of A
; Since 8 AH < 9BH, CY = 1
7 : STA 3010 H ; Store the contents of A to location 3010 H
8 : HLT ; Stop
Thus the contents of accumulator after execution of ADD instruction is EAH.
Hence (B) is correct answer.
MCQ 1.79 After execution of line 7 of the program, the status of the CY and Z flags will be
(A) CY = 0, Z = 0 (B) CY = 0, Z = 1

Brought to you by: Nodia and Company Visit us at: www.nodia.co.in


PUBLISHING FOR GATE
Page 39 GATE EC 2007 www.gatehelp.com

(C) CY = 1, Z = 0 (D) CY = 1, Z = 1
SOL 1.79 The CY = 1 and Z = 0
Hence (C) is correct answer.

Statement for linked Answer Question 80 & 81 :


Consider a linear system whose state space representation is x (t) = Ax (t). If
1
the initial state vector of the system is x (0) = = G, then the system response is
−2
e−2x 1
−2tH . If the itial state vector of the system changes to x (0) = =
− 2G
x (t) = > , then
− 2e −t
e
the system response becomes x (t) = > −tH
−e
MCQ 1.80 The eigenvalue and eigenvector pairs (λi vi) for the system are
1 1 1 1
(A) e− 1 = Go and e− 2 = Go (B) e− 1, = Go and e2, = Go
−1 −2 −1 −2
1 1 1 1
(C) e− 1, = Go and e− 2, = Go (D) e− 2 = Go and e1, = Go
−1 −2 −1 −2

SOL 1.80 Hence (A) is correct option.


We have xo (t) = Ax (t)
p q
A ==
r sG
Let

1 e−2t
For initial state vector x (0) = = G the system response is x (t) = > H
−2 − 2e−2t
d −2t
e p q 1
==
r s G=− 2G
Thus > d dt −2t H
dt (− 2e )
t=0
−2 (0)
− 2e p q 1
or > 4e−2 (0) H = =r s G=− 2G
−2 p − 2q
= 4 G = = r − 2s G

We get p − 2q =− 2 and r − 2s = 4 ...(i)


1 e−t
For initial state vector x (0) = = G the system response is x (t) = > −tH
−1 −e
d
e−t p q 1
==
> d (− e−t)H r s G=− 1G
dt
Thus
dt
t=0
− (0)
−e p q 1
> e− (0) H = =r s G=− 1G
−1 p−q
= 1G = = r − s G

Brought to you by: Nodia and Company Visit us at: www.nodia.co.in


PUBLISHING FOR GATE
Page 40 GATE EC 2007 www.gatehelp.com

We get p − q =− 1 and r − s = 1 ...(2)


Solving (1) and (2) set of equations we get
p q 0 1
=r s G = =− 2 − 3G

The characteristic equation


λI − A = 0
λ −1
=0
2 λ+3
or λ (λ + 3) + 2 = 0
or λ =− 1, − 2
Thus Eigen values are − 1 and − 2
Eigen vectors for λ1 =− 1
(λ1 I − A) X1 = 0
λ1 − 1 x11
or = 2 λ + 3G=x G = 0
1 21

− 1 − 1 x11
= 2 2 G=x G = 0
21

or − x11 − x21 = 0
or x11 + x21 = 0
We have only one independent equation x11 =− x21.Let x11 = K , then x21 =− K , the
Eigen vector will be
x11 K 1
=x G = =− K G = K =− 1G
21

Now Eigen vector for λ2 =− 2


(λ2 I − A) X2 = 0
λ2 − 1 x12
or = 2 λ + 3G=x G = 0
2 22

− 2 − 1 x11
or = 2 1 G=x G = 0
21

or − x11 − x21 = 0
or x11 + x21 = 0
We have only one independent equation x11 =− x21.
Let x11 = K, then x21 =− K , the Eigen vector will be
x12 K 1
=x G = =− 2K G = K =− 2G
22

MCQ 1.81 The system matrix A is


0 1 1 1
(A) =
− 1 1G
(B) =
− 1 − 2G

Brought to you by: Nodia and Company Visit us at: www.nodia.co.in


PUBLISHING FOR GATE
Page 41 GATE EC 2007 www.gatehelp.com

2 1 0 1
(C) = G (D) =
−1 −1 − 2 − 3G
SOL 1.81 As shown in previous solution the system matrix is
0 1
A ==
− 2 − 3G
Hence (D) is correct option.

Statement for Linked Answer Question 82 & 83 :


An input to a 6-level quantizer has the probability density function f (x) as shown
in the figure. Decision boundaries of the quantizer are chosen so as to maximize the
entropy of the quantizer output. It is given that 3 consecutive decision boundaries
are’ − 1'.'0' and '1' .

MCQ 1.82 The values of a and b are


(A) a = 1 and b = 1 (B) a = 1 and b = 3
6 12 5 40

(C) a = 1 and b = 1 (D) a = 1 and b = 1


4 16 3 24
SOL 1.82 Area under the pdf curve must be unity
Thus 2a + 4a + 4b =1
2a + 8b = 1 ...(1)
For maximum entropy three region must by equivaprobable thus
2a = 4b = 4b ...(2)
From (1) and (2) we get
b = 1 and a = 1
12 6
Hence (A) is correct option.
MCQ 1.83 Assuming that the reconstruction levels of the quantizer are the mid-points of the
decision boundaries, the ratio of signal power to quantization noise power is
(A) 152 (B) 64
9 3

(C) 76 (D) 28
3
SOL 1.83 Hence correct option is ( )

Brought to you by: Nodia and Company Visit us at: www.nodia.co.in


PUBLISHING FOR GATE
Page 42 GATE EC 2007 www.gatehelp.com

Statement for Linked Answer Question 84 and 85 :


In the Digital-to-Analog converter circuit shown in the figure below,
VR = 10V and R = 10kΩ

MCQ 1.84 The current i is


(A) 31.25μA (B) 62.5μA
(C) 125μA (D) 250μA
SOL 1.84 Since the inverting terminal is at virtual ground the resistor network can be reduced
as follows

The current from voltage source is


I = VR = 10 = 1 mA
R 10k
This current will be divide as shown below

Brought to you by: Nodia and Company Visit us at: www.nodia.co.in


PUBLISHING FOR GATE
Page 43 GATE EC 2007 www.gatehelp.com

-3
Now i = I = 1 # 10 = 62.5 μ A
16 16
Hence (B) is correct answer.
MCQ 1.85 The voltage V0 is
(A) − 0.781 V (B) − 1.562 V
(C) − 3.125 V (D) − 6.250 V
SOL 1.85 The net current in inverting terminal of OP - amp is
I- = 1 + 1 = 5I
4 16 16
So that V0 =− R # 5I =− 3.125
16
Hence (C) is correct answer.

Answer Sheet
1. (A) 19. (A) 37. (B) 55. (D) 73. (C)
2. (B) 20. (D) 38. (D) 56. (D) 74. (B)
3. (C) 21. (C) 39. (D) 57. (A) 75. (A)
4. (A) 22. (A) 40. (C) 58. (D) 76. (A)
5. (D) 23. (C) 41. (D) 59. (C) 77. (C)
6. (A) 24. (D) 42. (B) 60. (B) 78. (B)
7. (D) 25. (B) 43. (A) 61. (C) 79. (C)
8. (C) 26. (C) 44. (C) 62. (B) 80. (A)
9. (D) 27. (A) 45. (A) 63. (A) 81. (D)
10. (C) 28. (D) 46. (C) 64. (C) 82. (A)
11. (C) 29. (D) 47. (A) 65. (D) 83. (*)
12. (A) 30. (A) 48. (A) 66. (A) 84. (B)
13. (C) 31. (D) 49. (B) 67. (C) 85. (C)

Brought to you by: Nodia and Company Visit us at: www.nodia.co.in


PUBLISHING FOR GATE
Page 44 GATE EC 2007 www.gatehelp.com

14. (B) 32. (A) 50. (B) 68. (D)


15. (D) 33. (B) 51. (B) 69. (B)
16. (D) 34. (B) 52. (B) 70 (D)
17. (C) 35. (C) 53. (D) 71 (A)
18. (B) 36. (C) 54. (A) 72 (B)
**********

Brought to you by: Nodia and Company Visit us at: www.nodia.co.in


PUBLISHING FOR GATE
GATE EC
2008

Q.1 - Q.20 carry one mark each.


p11 p12
MCQ 1.1 All the four entries of the 2 # 2 matrix = = p21 p22 G are nonzero, and one of its
eigenvalue is zero. Which of the following statements is true?
(A) p11 p12 − p12 p21 = 1 (B) p11 p22 − p12 p21 =− 1
(C) p11 p22 − p12 p21 = 0 (D) p11 p22 + p12 p21 = 0
SOL 1.1 The product of Eigen value is equal to the determinant of the matrix. Since one of
the Eigen value is zero, the product of Eigen value is zero, thus determinant of the
matrix is zero.
Thus p11 p22 − p12 p21 = 0
Hence (C) is correct answer.
MCQ 1.2 The system of linear equations
4x + 2y = 7
2x + y = 6 has
(A) a unique solution (B) no solution
(C) an infinite number of solutions (D) exactly two distinct solutions
SOL 1.2 The given system is
4 2 x 7
=2 1G=y G = = 6 G
4 2
A ==
2 1G
We have
4 2
and A = =0 Rank of matrix ρ (A) < 2
2 1
4 2 7
Now C == G Rank of matrix ρ (C) = 2
2 1 6
Since ρ (A) ! ρ (C) there is no solution.
Hence (B) is correct answer.
MCQ 1.3 The equation sin (z) = 10 has

Brought to you by: Nodia and Company Visit us at: www.nodia.co.in


PUBLISHING FOR GATE
Page 2 GATE EC 2008 www.gatehelp.com

(A) no real or complex solution


(B) exactly two distinct complex solutions
(C) a unique solution
(D) an infinite number of complex solutions
SOL 1.3 sin z can have value between − 1 to + 1. Thus no solution.
Hence (A) is correct solution.
MCQ 1.4 For real values of x , the minimum value of the function
f (x) = exp (x) + exp (− x) is
(A) 2 (B) 1
(C) 0.5 (D) 0
SOL 1.4 Hence (A) is correct answer.
We have f (x) = ex + e−x
For x > 0 , ex > 1 and 0 < e−x < 1
For x < 0 , 0 < ex < 1 and e−x > 1
Thus f (x) have minimum values at x = 0 and that is e0 + e−0 = 2 .
MCQ 1.5 Which of the following functions would have only odd powers of x in its Taylor
series expansion about the point x = 0 ?
(A) sin (x3) (B) sin (x2)
(C) cos (x3) (D) cos (x2)
SOL 1.5 Hence (A) is correct answer.
3 5
sin x = x + x + x + ...
3! 5!
2 4
cos x = 1 + x + x + ...
2! 4!
Thus only sin (x3) will have odd power of x .
dx (t)
MCQ 1.6 Which of the following is a solution to the differential equation + 3x (t) = 0 ?
dt
(A) x (t) = 3e - t (B) x (t) = 2e - 3t
(C) x (t) =− 23 t2 (D) x (t) = 3t2
SOL 1.6 Hence (B) is correct answer.
dx (t)
We have + 3x (t) = 0
dt
or (D + 3) x (t) = 0
Since m =− 3 , x (t) = Ce - 3t Thus only (B) may be solution.
MCQ 1.7 In the following graph, the number of trees (P) and the number of cut-set (Q) are

Brought to you by: Nodia and Company Visit us at: www.nodia.co.in


PUBLISHING FOR GATE
Page 3 GATE EC 2008 www.gatehelp.com

(A) P = 2, Q = 2 (B) P = 2, Q = 6
(C) P = 4, Q = 6 (D) P = 4, Q = 10
SOL 1.7 The given graph is

There can be four possible tree of this graph which are as follows:

There can be 6 different possible cut-set.

Hence (C) is correct option.


MCQ 1.8 In the following circuit, the switch S is closed at t = 0 . The rate of change of
current di (0+) is given by
dt

(A) 0 (B) Rs Is
L
(R + Rs) Is
(C) (D) 3
L
SOL 1.8 Initially i (0−) = 0 therefore due to inductor i (0+) = 0 . Thus all current Is will flow
in resistor R and voltage across resistor will be Is Rs . The voltage across inductor
will be equal to voltage across Rs as no current flow through R.

Brought to you by: Nodia and Company Visit us at: www.nodia.co.in


PUBLISHING FOR GATE
Page 4 GATE EC 2008 www.gatehelp.com

Thus vL (0+) = Is Rs
di (0+)
but vL (0+) = L
dt
di (0+) v (0+) Is Rs
Thus = L =
dt L L
Hence (B) is correct option.
MCQ 1.9 The input and output of a continuous time system are respectively denoted by
x (t) and y (t). Which of the following descriptions corresponds to a causal system ?
(A) y (t) = x (t − 2) + x (t + 4) (B) y (t) = (t − 4) x (t + 1)
(C) y (t) = (t + 4) x (t − 1) (D) y (t) = (t + 5) x (t + 5)
SOL 1.9 The output of causal system depends only on present and past states only.
In option (A) y (0) depends on x (− 2) and x (4).
In option (B) y (0) depends on x (1).
In option (C) y (0) depends on x (− 1).
In option (D) y (0) depends on x (5).
Thus only in option (C) the value of y (t) at t = 0 depends on x (− 1) past value. In
all other option present value depends on future value.
Hence (C) is correct answer
MCQ 1.10 The impulse response h (t) of a linear time invariant continuous time system is
described by h (t) = exp (αt) u (t) + exp (βt) u (− t) where u (− t) denotes the unit
step function, and α and β are real constants. This system is stable if
(A) α is positive and β is positive
(B) α is negative and β is negative
(C) α is negative and β is negative
(D) α is negative and β is positive
SOL 1.10 Hence (D) is correct answer.
We have h (t) = eαt u (t) + e βt u (− t)
This system is stable only when bounded input has bounded output For stability
αt < 0 for t > 0 that implies α < 0 and βt > 0 for t > 0 that implies β > 0 . Thus,
α is negative and β is positive.
MCQ 1.11 The pole-zero given below correspond to a

Brought to you by: Nodia and Company Visit us at: www.nodia.co.in


PUBLISHING FOR GATE
Page 5 GATE EC 2008 www.gatehelp.com

(A) Law pass filter (B) High pass filter


(C) Band filter (D) Notch filter
SOL 1.11 Percent overshoot depends only on damping ratio, ξ .
Mp = e− ξπ 1 − ξ
2

If Mp is same then ξ is also same and we get


ξ = cos θ
Thus θ = constant
The option (C) only have same angle.
Hence (C) is correct option.
MCQ 1.12 Step responses of a set of three second-order underdamped systems all have the
same percentage overshoot. Which of the following diagrams represents the poles
of the three systems ?

SOL 1.12 Transfer function for the given pole zero plot is:
(s + Z1)( s + Z2)
(s + P1)( s + P2)
From the plot Re (P1 and P2 )>(Z1 and Z2 )
So, these are two lead compensator.
Hence both high pass filters and the system is high pass filter.

Brought to you by: Nodia and Company Visit us at: www.nodia.co.in


PUBLISHING FOR GATE
Page 6 GATE EC 2008 www.gatehelp.com

Hence (C) is correct option.


MCQ 1.13 Which of the following is NOT associated with a p − n junction ?
(A) Junction Capacitance (B) Charge Storage Capacitance
(C) Depletion Capacitance (D) Channel Length Modulations
SOL 1.13 Channel length modulation is not associated with a p − n junction. It is being
associated with MOSFET in which effective channel length decreases, producing
the phenomenon called channel length modulation.
Hence option (D) is correct.
MCQ 1.14 Which of the following is true?
(A) A silicon wafer heavily doped with boron is a p+ substrate
(B) A silicon wafer lightly doped with boron is a p+ substrate
(C) A silicon wafer heavily doped with arsenic is a p+ substrate
(D) A silicon wafer lightly doped with arsenic is a p+ substrate
SOL 1.14 Trivalent impurities are used for making p − type semiconductors. So, Silicon wafer
heavily doped with boron is a p+ substrate.
Hence option (A) is correct
MCQ 1.15 For a Hertz dipole antenna, the half power beam width (HPBW) in the E -plane is
(A) 360c (B) 180c
(C) 90c (D) 45c
SOL 1.15 The beam-width of Hertizian dipole is 180c and its half power beam-width is 90c.
Hence (C) is correct option
MCQ 1.16 For static electric and magnetic fields in an inhomogeneous source-free medium,
which of the following represents the correct form of Maxwell’s equations ?
(A) 4$ E = 0 , 4# B = 0 (B) 4$ E = 0 , 4$ B = 0
(C) 4# E = 0 , 4# B = 0 (D) 4# E = 0 , 4$ B = 0
SOL 1.16 Maxwell equations
4− B = 0
4$ E = ρ/E
4# E =− B
4# Ht = D + J
For static electric magnetic fields
4$ B = 0
4$ E = ρ/E
4# E = 0
S
4# H = J
Hence (D) is correct option

Brought to you by: Nodia and Company Visit us at: www.nodia.co.in


PUBLISHING FOR GATE
Page 7 GATE EC 2008 www.gatehelp.com

MCQ 1.17 In the following limiter circuit, an input voltage Vi = 10 sin 100πt is applied. Assume
that the diode drop is 0.7 V when it is forward biased. When it is forward biased.
The zener breakdown voltage is 6.8 V
The maximum and minimum values of the output voltage respectively are

(A) 6.1 V, − 0.7 V (B) 0.7 V, − 7.5 V


(C) 7.5 V, − 0.7 V (D) 7.5 V, − 7.5 V
SOL 1.17 For the positive half of Vi , the diode D1 is forward bias, D2 is reverse bias and the
zener diode is in breakdown state because Vi > 6.8 .
Thus output voltage is
V0 = 0.7 + 6.8 = 7.5 V
For the negative half of Vi, D2 is forward bias thus
Then V0 =− 0.7 V
Hence (C) is correct option
MCQ 1.18 A silicon wafer has 100 nm of oxide on it and is furnace at a temperature above
1000c C for further oxidation in dry oxygen. The oxidation rate
(A) is independent of current oxide thickness and temperature
(B) is independent of current oxide thickness but depends on temperature
(C) slows down as the oxide grows
(D) is zero as the existing oxide prevents further oxidation
SOL 1.18 Oxidation rate is zero because the existing oxide prevent the further oxidation.
Hence option (D) is correct.
MCQ 1.19 The drain current of MOSFET in saturation is given by ID = K (VGS − VT ) 2 where
K is a constant.
The magnitude of the transconductance gm is

K (VGS − VT ) 2
(A) (B) 2K (VGS − VT )
VDS
Id K (VGS − VT ) 2
(C) (D)
VGS − VDS VGS
SOL 1.19 Hence option (B) is correct.
gm = 2ID = 2 K (VGS − VT ) 2 = 2K (VGS − VT )
2VGS 2VGS

Brought to you by: Nodia and Company Visit us at: www.nodia.co.in


PUBLISHING FOR GATE
Page 8 GATE EC 2008 www.gatehelp.com

MCQ 1.20 Consider the amplitude modulated (AM) signalAc cos ωc t + 2 cos ωm t cos ωc t . For
demodulating the signal using envelope detector, the minimum value of Ac should
be
(A) 2 (B) 1
(C) 0.5 (D) 0
SOL 1.20 Hence (A) is correct option
We have xAM (t) = Ac cos ωc + 2 cos ωm t cos ωc t

= AC c1 + 2 cos ωm t m cos ωc t
Ac
For demodulation by envelope demodulator modulation index must be less than or
equal to 1.
Thus 2 #1
Ac
Ac $ 2
Hence minimum value of Ac = 2

Q.21 to Q.75 carry two marks each

MCQ 1.21 The Thevenin equivalent impedance Zth between the nodes P and Q in the following
circuit is

(A) 1 (B) 1 + s + 1
s

(D) s2 + s + 1
2
(C) 2 + s + 1
s s + 2s + 1
SOL 1.21 Killing all current source and voltage sources we have,

Zth = (1 + s) ( s1 + 1)
(1 + s)( s1 + 1) [ s1 + 1 + 1 + s]
= =
(1 + s) + ( s1 + 1) s + s1 + 1 + 1

Brought to you by: Nodia and Company Visit us at: www.nodia.co.in


PUBLISHING FOR GATE
Page 9 GATE EC 2008 www.gatehelp.com

or Zth = 1
Alternative :
Here at DC source capacitor act as open circuit and inductor act as short circuit.
Thus we can directly calculate thevenin Impedance as 1 Ω
Hence (A) is correct option.
MCQ 1.22 The driving point impedance of the following network is given by
Z (s) = 2 0.2s
s + 0.1s + 2

The component values are


(A) L = 5 H, R = 0.5 Ω, C = 0.1 F (B) L = 0.1 H, R = 0.5 Ω, C = 5 F
(C) L = 5 H, R = 2 Ω, C = 0.1 F (D) L = 0.1 H, R = 2 Ω, C = 5 F
SOL 1.22 Hence (D) is correct option.
s
Z (s) = R 1 sL = 2 C
sC s + s
RC + 1
LC
We have been given
Z (s) = 2 0.2s
s + 0.1s + 2
Comparing with given we get
1 = 0.2 or C = 5 F
C
1 = 0.1 or R = 2 Ω
RC
1 = 2 or L = 0.1 H
LC

MCQ 1.23 The circuit shown in the figure is used to charge the capacitor C alternately from
two current sources as indicated. The switches S1 and S2 are mechanically coupled
and connected as follows:
For 2nT # t # (2n + 1) T , (n = 0, 1, 2,..) S1 to P1 and S2 to P2
For (2n + 1) T # t # (2n + 2) T, (n = 0, 1, 2,...) S1 to Q1 and S2 to Q2

Brought to you by: Nodia and Company Visit us at: www.nodia.co.in


PUBLISHING FOR GATE
Page 10 GATE EC 2008 www.gatehelp.com

Assume that the capacitor has zero initial charge. Given that u (t) is a unit step
function , the voltage vc (t) across the capacitor is given by
3
(A) / (− 1) n tu (t − nT)
n=1
3
(B) u (t) + 2 / (− 1) n u (t − nT)
n=1
3
(C) tu (t) + 2 / (− 1) n u (t − nT) (t − nT)
n=1

(D) / 60.5 − e− (t − 2nT) + 0.5e− (t − 2nT) − T @


3

n=1
SOL 1.23 Voltage across capacitor is
#
t
Vc = 1 idt
C 0
Here C = 1 F and i = 1 A. Therefore
#0 dt
t
Vc =

For 0 < t < T , capacitor will be charged from 0 V


#0 dt = t
t
Vc =

At t = T, Vc = T Volts
For T < t < 2T , capacitor will be discharged from T volts as
#T dt = 2T − t
t
Vc = T −

At t = 2T, Vc = 0 volts
For 2T < t < 3T , capacitor will be charged from 0 V
#2Tdt = t − 2T
t
Vc =

At t = 3T, Vc = T Volts
For 3T < t < 4T , capacitor will be discharged from T Volts
#3Tdt = 4T − t
t
Vc = T −

At t = 4T, Vc = 0 Volts
For 4T < t < 5T , capacitor will be charged from 0 V
#4Tdt = t − 4T
t
Vc =

Brought to you by: Nodia and Company Visit us at: www.nodia.co.in


PUBLISHING FOR GATE
Page 11 GATE EC 2008 www.gatehelp.com

At t = 5T, Vc = T Volts
Thus the output waveform is

Only option C satisfy this waveform.


Hence (C) is correct option.
MCQ 1.24 The probability density function (pdf) of random variable is as shown below

The corresponding commutative distribution function CDF has the form

SOL 1.24 CDF is the integration of PDF. Plot in option (A) is the integration of plot given
in question.
Hence (A) is correct option.
MCQ 1.25 The recursion relation to solve x = e - x using Newton - Raphson method is
(A) xn + 1 = e−x
n
(B) xn + 1 = xn − e−x n

−x x n2 − e−x (1 − xn) − 1
n

(C) xn + 1 = (1 + xn) e −x
n

(D) xn + 1 =
1+e n
xn − e−x
n

SOL 1.25 Hence (C) is correct answer.


We have x = e-x
or f (x) = x − e - x
f'( x) = 1 + e - x
Brought to you by: Nodia and Company Visit us at: www.nodia.co.in
PUBLISHING FOR GATE
Page 12 GATE EC 2008 www.gatehelp.com

The Newton-Raphson iterative formula is


f (xn)
xn + 1 = xn −
f'( xn)
Now f (xn) = xn − e - x n

f'( xn) = 1 + e - x n

(1 + xn) e - x
xn + 1 = xn − xn − e- x =
-x n n

Thus
1+e n
1 + e-x n

MCQ 1.26 The residue of the function f (z) = 1 at z = 2 is


(z + 2) (z − 2) 2
2

(A) − 1 (B) − 1
32 16

(C) 1 (D) 1
16 32
SOL 1.26 Hence (A) is correct answer.
Res f (z) z = a = 1 dn - 1 6(z − a) n f (z)@
(n − 1)! dzn - 1 z=a

Here we have n = 2 and a = 2

1 d (z − 2) 2 1
(2 − 1)! dz ; (z − 2) 2 (z + 2) 2 Ez = a
Thus Res f (z) z = 2 =

= d ; 1 −2
dz (z + 2) 2 Ez = a ; (z + 2) 3 Ez = a
=

=− 2 =− 1
64 32

0 1
Consider the matrix P = =
− 2 − 3G
MCQ 1.27 . The value of e p is

2e−2 − 3e−1 e−1 − e−2 e−1 + e−1 2e−2 − e−1


(A) > −2 H (B) > −1 H
2e − 2e−1 5e−2 − e−1 2e − 4e2 3e−1 + 2e−2
5e−2 − e−1 3e−1 − e−2 2e−1 − e−2 e−1 − e−2
(C) > −2 H (D) > H
2e − 6e−1 4e−2 + 6−1 − 2e−1 + 2e−2 − e−1 + 2e−2

SOL 1.27 Hence (D) is correct answer.


eP = L- 1 6(sI − A) - 1@
s 0 0 1 -1
= L e=
0 s G =− 2 − 3Go
-1

s − 1 -1
= L- 1 e= o
2 s + 3G
s+3 1
= L f> Hp
-1 (s + 1)( s + 2) (s + 1)( s + 2)
-2 s
(s + 1)( s + 2) (s + 1)( s + 2)
2e − e-1
e −e-2 -1 -2
== G
− 2e + 2e − e - 1 + 2e - 2
-1 -2

Brought to you by: Nodia and Company Visit us at: www.nodia.co.in


PUBLISHING FOR GATE
Page 13 GATE EC 2008 www.gatehelp.com

MCQ 1.28 In the Taylor series expansion of exp (x) + sin (x) about the point x = π , the
coefficient of (x − π) 2 is
(A) exp (π) (B) 0.5 exp (π)
(C) exp (π) + 1 (D) exp (π) − 1
SOL 1.28 Taylor series is given as
(x − a) 2
f (x) = f (a) + x − a f'( a) + f"( a) + ...
1! 2!
For x = π we have
(x − π) 2
Thus f (x) = f (π) + x − π f'( π) + f"( x)...
1! 2!
Now = ex + sin x
f (x)
= ex + cos x
f'( x)
= ex − sin x
f"( x)
= e π − sin π = e π
f"( π)
f"( π)
Thus the coefficient of (x − π) 2 is
2!
Hence (B) is correct answer.
MCQ 1.29 Px (x) = M exp _− 2 x i − N exp _− 3 x i is the probability density function for the
real random variable X , over the entire x axis, M and N are both positive real
numbers. The equation relating M and N is
(A) M − 2 N = 1 (B) 2M + 1 N = 1
3 3
(C) M + N = 1 (D) M + N = 3
SOL 1.29 Correct Option is ( )
MCQ 1.30 The value of the integral of the function g (x, y) = 4x3 + 10y 4 along the straight line
segment from the point (0, 0) to the point (1, 2) in the x − y plane is
(A) 33 (B) 35
(C) 40 (D) 56
SOL 1.30 The equation of straight line from (0, 0) to (1, 2) is y = 2x .
Now g (x, y) = 4x3 + 10y 4
or, g (x, 2x) = 4x3 + 160x 4
#0 g (x, 2x) = #0 (4x3 + 160x4) dx
1 1
Now

= [x 4 + 32x5] 10 = 33
Hence (A) is correct answer.
MCQ 1.31 A linear, time - invariant, causal continuous time system has a rational transfer
function with simple poles at s =− 2 and s =− 4 and one simple zero at s =− 1.

Brought to you by: Nodia and Company Visit us at: www.nodia.co.in


PUBLISHING FOR GATE
Page 14 GATE EC 2008 www.gatehelp.com

A unit step u (t) is applied at the input of the system. At steady state, the output
has constant value of 1. The impulse response of this system is
(A) [exp (− 2t) + exp (− 4t)] u (t)
(B) [− 4 exp (− 2t) − 12 exp (− 4t) − exp (− t)] u (t)
(C) [− 4 exp (− 2t) + 12 exp (− 4t)] u (t)
(D) [− 0.5 exp (− 2t) + 1.5 exp (− 4t)] u (t)
SOL 1.31 Hence (C) is correct answer.
K (s + 1)
G (s) = , and R (s) = 1
(s + 2)( s + 4) s
K (s + 1)
C (s) = G (s) R (s) =
s (s + 2)( s + 4)
= K + K − 3K
8s 4 (s + 2) 8 (s + 4)
Thus c (t) = K :1 + 1 e−2t − 3 e−4tD u (t)
8 4 8
At steady-state , c (3) = 1
Thus K = 1 or K = 8
8
8 (s + 1)
Then, G (s) = = 12 − 4
(s + 2)( s + 4) (s + 4) (s + 2)
h (t) = L−1 G (s) = (− 4e−2t + 12e−4t) u (t)
MCQ 1.32 The signal x (t) is described by
1 for − 1 # t # + 1
x (t) = )
0 otherwise
Two of the angular frequencies at which its Fourier transform becomes zero are
(A) π, 2π (B) 0.5π, 1.5π
(C) 0, π (D) 2π, 2.5π
SOL 1.32 Hence (A) is correct answer.
1 for − 1 # t # + 1
We have x (t) = )
0 otherwise
Fourier transform is
1
#− 33e−jωt x (t) dt = #−1 e−jωt 1dt
= 1 [e−jωt]−11
− jω
= 1 (e−jω − e jω) = 1 (− 2j sin ω)
− jω − jω
= 2 sin ω
ω
This is zero at ω = π and ω = 2π

Brought to you by: Nodia and Company Visit us at: www.nodia.co.in


PUBLISHING FOR GATE
Page 15 GATE EC 2008 www.gatehelp.com

MCQ 1.33 A discrete time linear shift - invariant system has an impulse response h [n] with
h [0] = 1, h [1] =− 1, h [2] = 2, and zero otherwise The system is given an input
sequence x [n] with x [0] = x [2] = 1, and zero otherwise. The number of nonzero
samples in the output sequence y [n], and the value of y [2] are respectively
(A) 5, 2 (B) 6, 2
(C) 6, 1 (D) 5, 3
SOL 1.33 Hence (D) is correct answer.
Given h (n) = [1, − 1, 2]
x (n) = [1, 0, 1]
y (n) = x (n)* h (n)
The length of y [n] is
= L1 + L2 − 1 = 3 + 3 − 1 = 5
3
y (n) = x (n) * h (n) = / x (k) h (n − k)
k =− 3
3
y (2) = / x (k) h (2 − k)
k =− 3

= x (0) h (2 − 0) + x (1) h (2 − 1) + x (2) h (2 − 2)


= h (2) + 0 + h (0) = 1 + 2 = 3
There are 5 non zero sample in output sequence and the value of y [2] is 3.
MCQ 1.34 Consider points P and Q in the x − y plane, with P = (1, 0) and Q = (0, 1). The
Q
line integral 2 #P (xdx + ydy) along the semicircle with the line segment PQ as its
diameter
(A) is − 1
(B) is 0
(C) is 1
(D) depends on the direction (clockwise or anit-clockwise) of the semicircle
SOL 1.34 Hence (B) is correct answer.
Q
I =2 #P (xdx + ydy)
Q Q
=2 #P xdx + 2 #P ydy
#1 #0 ydy = 0
0 1
=2 xdx + 2

MCQ 1.35 Let x (t) be the input and y (t) be the output of a continuous time system. Match
the system properties P1, P2 and P3 with system relations R1, R2, R3, R4
Properties Relations
P1 : Linear but NOT time - invariant R1 : y (t) = t2 x (t)
P2 : Time - invariant but NOT linear R2 : y (t) = t x (t)

Brought to you by: Nodia and Company Visit us at: www.nodia.co.in


PUBLISHING FOR GATE
Page 16 GATE EC 2008 www.gatehelp.com

P3 : Linear and time - invariant R3 : y (t) = x (t)


R4 : y (t) = x (t − 5)
(A) (P1, R1), (P2, R3), (P3, R4)
(B) (P1, R2), (P2, R3), (P3, R4)
(C) (P1, R3), (P2, R1), (P3, R2)
(D) (P1, R1), (P2, R2), (P3, R3)
SOL 1.35 Mode function are not linear. Thus y (t) = x (t) is not linear but this functions is
time invariant. Option (A) and (B) may be correct.
The y (t) = t x (t) is not linear, thus option (B) is wrong and (a) is correct. We can
see that
R1: y (t) = t2 x (t) Linear and time variant.
R2: y (t) = t x (t) Non linear and time variant.
R3: y (t) = x (t) Non linear and time invariant
R4: y (t) = x (t − 5) Linear and time invariant
Hence (B) is correct answer.
MCQ 1.36 A memory less source emits n symbols each with a probability p. The entropy of
the source as a function of n
(A) increases as log n (B) decreases as log ( n1 )
(C) increases as n (D) increases as n log n
SOL 1.36 The entropy is
m
H = / pi log2 p1i bits
i=1

Since p1 = p2 = ... = pn = 1
n
n
H = / n1 log n = log n
i=1
Hence (A) is correct option.
MCQ 1.37 {x (n)} is a real - valued periodic sequence with a period N . x (n) and X (k) form
N-point Discrete Fourier Transform (DFT) pairs. The DFT Y (k) of the sequence
N−1
y (n) = 1 / x (r) x (n + r) is
N r=0
N−1
(A) X (k) 2 (B) 1 / X (r) X (k + r)
N r=0
N−1
(C) 1 / X (r) X (k + r) (D) 0
N r=0

SOL 1.37 Hence (A) is correct answer.


N-1
Given : y (n) = 1
N r=0
/
x (r) x (n + r)

Brought to you by: Nodia and Company Visit us at: www.nodia.co.in


PUBLISHING FOR GATE
Page 17 GATE EC 2008 www.gatehelp.com

It is Auto correlation.
DFT
Hence y (n) = rxx (n) X (k) 2

MCQ 1.38 Group I lists a set of four transfer functions. Group II gives a list of possible step
response y (t). Match the step responses with the corresponding transfer functions.

(A) P − 3, Q − 1, R − 4, S − 2 (B) P − 3, Q − 2, R − 4, S − 1
(C) P − 2, Q − 1, R − 4, S − 2 (D) P − 3, Q − 4, R − 1, S − 2
SOL 1.38 Hence (D) is correct option.
P = 2 25 2ξωn = 0, ξ = 0 " Undamped Graph 3
s + 25

Q= 62 2ξωn = 20, ξ > 1 " Overdamped Graph 4


s + 20s + 62
2

R= 62 2ξωn = 12, ξ = 1 " Critically Graph 1


s + 12s + 62
2

S= 72 2ξωn = 7, ξ < 1 " underdamped Graph 2


s + 7s + 72
2

MCQ 1.39 A certain system has transfer function


G (s) = 2 s + 8
s + αs − 4
Brought to you by: Nodia and Company Visit us at: www.nodia.co.in
PUBLISHING FOR GATE
Page 18 GATE EC 2008 www.gatehelp.com

where α is a parameter. Consider the standard negative unity feedback configuration


as shown below

Which of the following statements is true?


(A) The closed loop systems is never stable for any value of α
(B) For some positive value of α, the closed loop system is stable, but not for all
positive values.
(C) For all positive values of α, the closed loop system is stable.
(D) The closed loop system stable for all values of α, both positive and negative.
SOL 1.39 Hence (C) is correct option.
The characteristic equation of closed lop transfer function is
1 + G (s) H (s) = 0
1+ 2 s+8 =0
s + αs − 4
or s 2 + αs − 4 + s + 8 = 0
or s2 + (α + 1) s + 4 = 0
This will be stable if (α + 1) > 0 " α > − 1. Thus system is stable for all positive
value of α.
MCQ 1.40 A signal flow graph of a system is given below

The set of equalities that corresponds to this signal flow graph is


Jx1N R β − γ 0 VJx1N R0 0 V
K O S WK O S W u1
(A) d K x2O = S γ α 0 WK x2O+ S0 1 We o
dt K O S u2
x3 S− α β 0 WWK x3O SS1 0 WW
L P L P
Jx1N RT0 α γ XVJx1N TR1 0 XV
K O S WK O S W u1
(B) d K x2O = S0 − α − γ WK x2O+ S0 1 We o
dt K O S u2
x3 S0 β − β WWK x3O SS0 0 WW
L P TR L P
Jx1N − α β 0 VXJx1N RT1 0 VX
K O S WK O S W u1
(C) d K x2O = S− β − γ 0 WK x2O+ S0 1 We o
dt K O S u2
x3 S α γ 0 WWK x3O SS0 0 WW
L P T XL P T X
Brought to you by: Nodia and Company Visit us at: www.nodia.co.in
PUBLISHING FOR GATE
Page 19 GATE EC 2008 www.gatehelp.com

Jx1N R− α 0 β VJx1N R1 0 V
K O S WK O S W u1
(D) d K x2O = S γ 0 α WK x2O+ S0 1 We o
dt K O S u2
x3 S− β 0 − α WWK x3O SS0 0 WW
L P T XL P T X
SOL 1.40 We labeled the given SFG as below :

From this SFG we have


xo1 =− γx1 + βx3 + μ1
xo2 = γx1 + αx3
xo3 =− βx1 − αx3 + u2
R V R VR V R V
Sx1 W S− γ 0 β WSx1 W S0 1 W u1
Thus Sx2 W = S γ 0 α WSx2 W+ S0 0 We o
SSx WW SS− β 0 − α WWSSx WW SS1 0 WW u2
3 3
T X T XT X T X
Hence (C) is correct option.
MCQ 1.41 The number of open right half plane of
G (s) = 5 10 is
s + 2s + 3s + 6s2 + 5s + 3
4 3

(A) 0 (B) 1
(C) 2 (D) 3
SOL 1.41 The characteristic equation is
1 + G (s) = 0
or s + 2s + 3s + 6s + 5s + 3 = 0
5 4 3 2

Substituting s = z1 we have
3z5 + 5z 4 + 6z3 + 3z2 + 2z + 1 = 0
The routh table is shown below. As there are tow sign change in first column, there
are two RHS poles.

z5 3 6 2
z4 5 3 1
z3 21
5
7
5

z2 4
3 3
z1 − 74
z0 1

Brought to you by: Nodia and Company Visit us at: www.nodia.co.in


PUBLISHING FOR GATE
Page 20 GATE EC 2008 www.gatehelp.com

Hence (C) is correct option.


MCQ 1.42 The magnitude of frequency responses of an underdamped second order system
is 5 at 0 rad/sec and peaks to 10 at 5 2 rad/sec. The transfer function of the
3
system is
(A) 2 500 (B) 2 375
s + 10s + 100 s + 5s + 75
(C) 2 720 (D) 2 1125
s + 12s + 144 s + 25s + 225

SOL 1.42 For underdamped second order system the transfer function is
T (s) = 2 Kωn2
s + 2ξωn s + ωn2
It peaks at resonant frequency. Therefore
Resonant frequency ωr = ωn 1 − 2ξ2
and peak at this frequency
μr = 5
2ξ 1 − ξ2
We have ωr = 5 2 , and μr = 10 . Only options (A) satisfy these values.
3
ωn = 10, ξ = 1
2
where ωr = 10 1 − 2` 1 j = 5 2
4
and μr = 1 5 1 = 10 Hence satisfied
22 1− 4 3
Hence (C) is correct option.
MCQ 1.43 Group I gives two possible choices for the impedance Z in the diagram. The
circuit elements in Z satisfy the conditions R2 C2 > R1 C1. The transfer functions V0
Vi
represents a kind of controller.

Match the impedances in Group I with the type of controllers in Group II

Brought to you by: Nodia and Company Visit us at: www.nodia.co.in


PUBLISHING FOR GATE
Page 21 GATE EC 2008 www.gatehelp.com

(A) Q − 1, R − 2 (B) Q − 1, R − 3
(C) Q − 2, R − 3 (D) Q − 3, R − 2
SOL 1.43 The given circuit is a inverting amplifier and transfer function is
Vo = − Z = − Z (sC1 R1 + 1)
Vi R 1
R1
sC R + 1
1 1

(sC2 R2 + 1)
For Q , Z =
sC2
Vo =− (sC2 R2 + 1) (sC1 R1 + 1) PID Controller
#
Vi sC2 R1
For R, Z = R2
(sC2 R2 + 1)
Vo =− R2 (sC1 R1 + 1)
#
Vi (sC2 R2 + 1) R1
Since R2 C2 > R1 C1, it is lag compensator.
Hence (B) is correct option.
MCQ 1.44 For the circuit shown in the following figure, transistor M1 and M2 are identical
NMOS transistors. Assume the M2 is in saturation and the output is unloaded.

The current Ix is related to Ibias as


(A) Ix = Ibias + Is (B) Ix = Ibias
(C) Ix = Ibias − cVDD − Vout m (D) Ix = Ibias − Is
RE
SOL 1.44 By Current mirror,
^ L h2
W
Ix = W Ibias
^ L h1
Brought to you by: Nodia and Company Visit us at: www.nodia.co.in
PUBLISHING FOR GATE
Page 22 GATE EC 2008 www.gatehelp.com

Since MOSFETs are identical,


Thus bW l = bW l
L 2 L 2
Hence Ix = Ibias
Hence (B) is correct option.
MCQ 1.45 The measured trans conductance gm of an NMOS transistor operating in the linear
region is plotted against the gate voltage VG at a constant drain voltage VD . Which
of the following figures represents the expected dependence of gm on VG ?

SOL 1.45 Hence option (C) is correct.


As VD = constant
Thus gm \ (VGS − VT ) Which is straight line.

MCQ 1.46 Consider the following circuit using an ideal OPAMP. The I-V characteristic of the
diode is described by the relation I = I 0 _eV − 1i where VT = 25 mV, I0 = 1μ A and
V
t

V is the voltage across the diode (taken as positive for forward bias). For an input
voltage Vi =− 1 V , the output voltage V0 is

(A) 0 V (B) 0.1 V


(C) 0.7 V (D) 1.1 V
SOL 1.46 The circuit is using ideal OPAMP. The non inverting terminal of OPAMP is at
ground, thus inverting terminal is also at virtual ground.

Brought to you by: Nodia and Company Visit us at: www.nodia.co.in


PUBLISHING FOR GATE
Page 23 GATE EC 2008 www.gatehelp.com

Thus current will flow from -ive terminal (0 Volt) to -1 Volt source. Thus the
current I is
0 − (− 1)
I = = 1
100k 100k
The current through diode is
I = I 0 _eV − 1i
V
t

Now VT = 25 mV and I0 = 1 μA
I = 10−6 8e 25 # 10 − 1B = 1 5
V
Thus −3

10
or V = 0.06 V
Now V0 = I # 4k + V = 1 # 4k + 0.06 = 0.1 V
100k
Hence (B) is correct option.
MCQ 1.47 The OPAMP circuit shown above represents a

(A) high pass filter (B) low pass filter


(C) band pass filter (D) band reject filter
SOL 1.47 The circuit is using ideal OPAMP. The non inverting terminal of OPAMP is at
ground, thus inverting terminal is also at virtual ground.

Brought to you by: Nodia and Company Visit us at: www.nodia.co.in


PUBLISHING FOR GATE
Page 24 GATE EC 2008 www.gatehelp.com

Thus we can write


vi = −Rv
R1 + sL sR C + 1
2

2 2

or v0
=− R2
vi (R1 + sL)( sR2 C2 + 1)
and from this equation it may be easily seen that this is the standard form of T.F.
of low pass filter
H (s) = K
(R1 + sL)( sR2 C2 + 1)
and form this equation it may be easily seen that this is the standard form of T.F.
of low pass filter
H (s) = 2 K
as + bs + b
Hence (B) is correct option.
MCQ 1.48 Two identical NMOS transistors M1 and M2 are connected as shown below. Vbias is
chosen so that both transistors are in saturation. The equivalent gm of the pair is
defied to be 2Iout at constant Vout
2Vi
The equivalent gm of the pair is

(A) the sum of individual gm ' s of the transistors


(B) the product of individual gm ’s of the transistors
(C) nearly equal to the gm of M1
g
(D) nearly equal to m of M2
g0
SOL 1.48 The current in both transistor are equal. Thus gm is decide by M1.
Hence (C) is correct option.
MCQ 1.49 An 8085 executes the following instructions
2710 LXI H, 30A0 H
2713 DAD H
2414 PCHL
All address and constants are in Hex. Let PC be the contents of the program
counter and HL be the contents of the HL register pair just after executing PCHL.
Which of the following statements is correct ?
PC = 2715H PC = 30A0H
(A) (B)
HL = 30A0H HL = 2715H
Brought to you by: Nodia and Company Visit us at: www.nodia.co.in
PUBLISHING FOR GATE
Page 25 GATE EC 2008 www.gatehelp.com

PC = 6140H PC = 6140H
(C) (D)
HL = 6140H HL = 2715H

SOL 1.49 2710H LXI H, 30A0H ; Load 16 bit data 30A0 in HL pair
2713H DAD H ; 6140H " HL
2714H PCHL ; Copy the contents 6140H of HL in PC
Thus after execution above instruction contests of PC and HL are same and that
is 6140H
Hence (C) is correct answer.
MCQ 1.50 An astable multivibrator circuit using IC 555 timer is shown below. Assume that
the circuit is oscillating steadily.

The voltage Vc across the capacitor varies between


(A) 3 V to 5 V (B) 3 V to 6 V
(C) 3.6 V to 6 V (D) 3.6 V to 5 V
SOL 1.50 Correct Option is ( )
MCQ 1.51 Silicon is doped with boron to a concentration of 4 # 1017 atoms cm3 . Assume the
intrinsic carrier concentration of silicon to be 1.5 # 1010 / cm 3 and the value of kT/q
to be 25 mV at 300 K. Compared to undopped silicon, the fermi level of doped
silicon
(A) goes down by 0.31 eV (B) goes up by 0.13 eV
(C) goes down by 0.427 eV (D) goes up by 0.427 eV
SOL 1.51 Hence option (C) is correct.
E2 − E1 = kT ln NA
ni
NA = 4 # 1017

Brought to you by: Nodia and Company Visit us at: www.nodia.co.in


PUBLISHING FOR GATE
Page 26 GATE EC 2008 www.gatehelp.com

ni = 1.5 # 1010
17
E2 − E1 = 25 # 10−3 e ln 4 # 10 10 = 0.427 eV
1.5 # 10
Hence fermi level goes down by 0.427 eV as silicon is doped with boron.
MCQ 1.52 The cross section of a JFET is shown in the following figure. Let Vc be − 2 V
and let VP be the initial pinch -off voltage. If the width W is doubled (with other
geometrical parameters and doping levels remaining the same), then the ratio
between the mutual trans conductances of the initial and the modified JFET is

1 − 2/Vp
(B) 1 e
2 1 − 1/2Vp o
(A) 4

1 − 2/Vp 1 − (2 − Vp )
(C) e o (D)
1 − 1/2Vp 1 − [1 (2 Vp )]
SOL 1.52 Hence option (C) is correct
2
Pinch off voltage VP = eW ND
εs
Let VP = VP1
Now VP1 = W12 = W2
VP2 W22 (2W) 2
or 4VP1 = VP2
Initial transconductance
gm = Kn ;1 − Vbi − VGS
Vp E
0 − (− 2)
gm1 = Kn =1 −
VP1 G
= Kn ;1 −
VP1 E
For first condition 2

For second condition


0 − (− 2)
gm2 = Kn =1 −
VP2 G
= K2 ;1 −
4VP1 E
2

1 − 2/VP1
=f p
gm1
Dividing
gm2 1 − 1/ (2VP1)
Hence VP = VP1
MCQ 1.53 Consider the Schmidt trigger circuit shown below
A triangular wave which goes from -12 to 12 V is applied to the inverting input of

Brought to you by: Nodia and Company Visit us at: www.nodia.co.in


PUBLISHING FOR GATE
Page 27 GATE EC 2008 www.gatehelp.com

OPMAP. Assume that the output of the OPAMP swings from +15 V to -15 V. The
voltage at the non-inverting input switches between

(A) − 12V to +12 V (B) -7.5 V to 7.5 V


(C) -5 V to +5 V (D) 0 V and 5 V
SOL 1.53 Let the voltage at non inverting terminal be V1, then after applying KCL at non
inverting terminal side we have
15 − V1 + V0 − V1 = V1 − (− 15)
10 10 10
or V1 = V0
3
If V0 swings from -15 to +15 V then V1 swings between -5 V to +5 V.
Hence (C) is correct option.
MCQ 1.54 The logic function implemented by the following circuit at the terminal OUT is

(A) P NOR Q (B) P NAND Q


(C) P OR Q (D) P AND Q
SOL 1.54 From the figure shown below it may be easily seen upper MOSFET are shorted and
connected to Vdd thus OUT is 1 only when the node S is 0,

Since the lower MOSFETs are shorted to ground, node S is 0 only when input P

Brought to you by: Nodia and Company Visit us at: www.nodia.co.in


PUBLISHING FOR GATE
Page 28 GATE EC 2008 www.gatehelp.com

and Q are 1. This is the function of AND gate.


Hence (D) is correct answer.
MCQ 1.55 Consider the following assertions.
S1 : For Zener effect to occur, a very abrupt junction is required.
S2 : For quantum tunneling to occur, a very narrow energy barrier is required.
Which of the following is correct ?
(A) Only S2 is true
(B) S1 and S2 are both true but S2 is not a reason for S1
(C) S1 and S2 and are both true but S2 is not a reason for S1
(D) Both S1 and S2 are false
SOL 1.55 Hence option (A) is correct.
MCQ 1.56 The two numbers represented in signed 2’s complement form are P + 11101101
and Q = 11100110 . If Q is subtracted from P , the value obtained in signed 2’s
complement is
(A) 1000001111 (B) 00000111
(C) 11111001 (D) 111111001
SOL 1.56 MSB of both number are 1, thus both are negative number. Now we get
11101101 = (− 19) 10
and 11100110 = (− 26) 10
P − Q = (− 19) − (− 26) = 7
Thus 7 signed two’s complements form is
(7) 10 = 00000111
Hence (B) is correct answer.
MCQ 1.57 Which of the following Boolean Expressions correctly represents the relation
between P, Q, R and M1

(A) M1 = (P OR Q) XOR R (B) M1 = (P AND Q) X OR R


(C) M1 = (P NOR Q) X OR R (D) M1 = (P XOR Q) XOR R
SOL 1.57 The circuit is as shown below

Brought to you by: Nodia and Company Visit us at: www.nodia.co.in


PUBLISHING FOR GATE
Page 29 GATE EC 2008 www.gatehelp.com

X = PQ
Y = (P + Q)
So Z = PQ (P + Q)
= (P + Q )( P + Q) = PQ + PQ = P 5 Q
and M1 = Z 5 R = (P 5 Q) 5 R
Hence (D) is correct answer
MCQ 1.58 For the circuit shown in the following, I0 − I3 are inputs to the 4:1 multiplexers, R
(MSB) and S are control bits.
The output Z can be represented by

(A) PQ + PQS + QRS


(B) PQ + PQR + PQS
(C) PQR + PQR + PARS + QRS
(D) PQR + PQRS + PQRS + QRS
SOL 1.58 Hence (A) is correct answer.
Z = I0 RS + I1 RS + I2 RS + I3 RS
= (P + Q ) RS + PRS + PQRS + PRS
= PRS + QRS + PRS + PQRS + PRS
The k − Map is as shown below

Brought to you by: Nodia and Company Visit us at: www.nodia.co.in


PUBLISHING FOR GATE
Page 30 GATE EC 2008 www.gatehelp.com

Z = PQ + PQS + QRS
MCQ 1.59 For each of the positive edge-triggered J − K flip flop used in the following figure,
the propagation delay is 3 t .

Which of the following wave forms correctly represents the output at Q1 ?

SOL 1.59 Since the input to both JK flip-flop is 11, the output will change every time with
clock pulse. The input to clock is

The output Q0 of first FF occurs after time 3 T and it is as shown below

The output Q1 of second FF occurs after time 3 T when it gets input (i.e. after 3 T
from t1) and it is as shown below

Brought to you by: Nodia and Company Visit us at: www.nodia.co.in


PUBLISHING FOR GATE
Page 31 GATE EC 2008 www.gatehelp.com

Hence (B) is correct answer.


MCQ 1.60 For the circuit shown in the figure, D has a transition from 0 to 1 after CLK
changes from 1 to 0. Assume gate delays to be negligible
Which of the following statements is true

(A) Q goes to 1 at the CLK transition and stays at 1


(B) Q goes to 0 at the CLK transition and stays 0
(C) Q goes to 1 at the CLK tradition and goes to 0 when D goes to 1
(D) Q goes to 0 at the CLK transition and goes to 1 when D goes to 1
SOL 1.60 The circuit is as shown below

The truth table is shown below. When CLK make transition Q goes to 1 and when
D goes to 1, Q goes to 0
Hence (A) is correct answer.
MCQ 1.61 A rectangular waveguide of internal dimensions (a = 4 cm and b = 3 cm) is to be
operated in TE11 mode. The minimum operating frequency is
(A) 6.25 GHz (B) 6.0 GHz
(C) 5.0 GHz (D) 3.75 GHz
SOL 1.61 Cut-off Frequency is
fc = c ` m j2 + ` n j2
2 a b

Brought to you by: Nodia and Company Visit us at: www.nodia.co.in


PUBLISHING FOR GATE
Page 32 GATE EC 2008 www.gatehelp.com

For TE11 mode,


3 # 1010
fc = ` 4 j + ` 3 j = 6.25 GHz
1 2 1 2
2
Hence (A) is correct option.
MCQ 1.62 One end of a loss-less transmission line having the characteristic impedance of 75Ω
and length of 1 cm is short-circuited. At 3 GHz, the input impedance at the other
end of transmission line is
(A) 0 (B) Resistive
(C) Capacitive (D) Inductive
SOL 1.62 Hence (D) is correct option.
Z + iZo tan (βl)
Zin = Zo L
Zo + iZL tan (βl)
For ZL = 0 ,Zin = iZo tan (βl)
The wavelength is
8
λ = c = 3 # 109 = 0.1 m or 10 cm
f 3 # 10
βl = 2π l = 2π # 1 = π
λ 10 5
Thus Zin = iZo tan π
5
Thus Zin is inductive because Zo tan π is positive
5
MCQ 1.63 A uniform plane wave in the free space is normally incident on an infinitely thick
dielectric slab (dielectric constant ε = 9 ). The magnitude of the reflection coefficient
is
(A) 0 (B) 0.3
(C) 0.5 (D) 0.8
SOL 1.63 Hence (C) is correct option.
μ
We have η =
ε
Reflection coefficient
η − η1
Γ= 2
η2 + η1
Substituting values for η1 and η2 we have
μ
− με
= 1 − εr = 1 −
o o

τ = εμε o r o 9 since εr = 9
μ
εε +
o
o r ε
o
o
1 + εr 1+ 9
=− 0.5
MCQ 1.64 In the design of a single mode step index optical fibre close to upper cut-off, the
single-mode operation is not preserved if
(A) radius as well as operating wavelength are halved

Brought to you by: Nodia and Company Visit us at: www.nodia.co.in


PUBLISHING FOR GATE
Page 33 GATE EC 2008 www.gatehelp.com

(B) radius as well as operating wavelength are doubled


(C) radius is halved and operating wavelength is doubled
(D) radius is doubled and operating wavelength is halved
SOL 1.64 In single mode optical fibre, the frequency of limiting mode increases as radius
decreases
Hence r \ 1
f
So. if radius is doubled, the frequency of propagating mode gets halved, while in
option (D) it is increased by two times.
Hence (C) is correct option.
MCQ 1.65 At 20 GHz, the gain of a parabolic dish antenna of 1 meter and 70% efficiency is
(A) 15 dB (B) 25 dB
(C) 35 dB (D) 45 dB
SOL 1.65 Hence (D) is correct option.
8
λ = c = 3 # 10 9 = 3
f 20 # 10 200
2
Gp = ηπ2 ` D j = 0.7 # π2 c 13 m = 30705.4
2
Gain
λ 100
= 44.87 dB
MCQ 1.66 Noise with double-sided power spectral density on K over all frequencies is passed
through a RC low pass filter with 3 dB cut-off frequency of fc . The noise power at
the filter output is
(A) K (B) Kfc
(C) kπfc (D) 3
SOL 1.66 Hence (C) is correct option.
PSD of noise is N0 = K ...(1)
2
The 3-dB cut off frequency is
fc = 1 ...(2)
2πRC
Output noise power is
= N0 = c N0 m 1 = Kπfc
4RC 2 2RC

MCQ 1.67 Consider a Binary Symmetric Channel (BSC) with probability of error being p. To
transmit a bit, say 1, we transmit a sequence of three 1s. The receiver will interpret
the received sequence to represent 1 if at least two bits are 1. The probability that
the transmitted bit will be received in error is
(A) p3 + 3p2 (1 − p) (B) p3

Brought to you by: Nodia and Company Visit us at: www.nodia.co.in


PUBLISHING FOR GATE
Page 34 GATE EC 2008 www.gatehelp.com

(C) (1 − p3) (D) p3 + p2 (1 − p)


SOL 1.67 At receiving end if we get two zero or three zero then its error.
Let p be the probability of 1 bit error, the probability that transmitted bit error is
= Three zero + two zero and single one
= 3 C3 p3 + 3C2 p2 (1 − p)
= p3 + p2 (1 − p)
Hence (D) is correct option.
MCQ 1.68 Four messages band limited to W, W, 2W and 3W respectively are to be multiplexed
using Time Division Multiplexing (TDM). The minimum bandwidth required for
transmission of this TDM signal is
(A) W (B) 3W
(C) 6W (D) 7W
SOL 1.68 Bandwidth of TDM is
= 1 (sum of Nyquist Rate)
2
= 1 [2W + 2W + 4W + 6W] = 7W
2
Hence (D) is correct option.
MCQ 1.69 Consider the frequency modulated signal
10 cos [2π # 105 t + 5 sin (2π # 1500t) + 7.5 sin (2π # 1000t)]
with carrier frequency of 105 Hz. The modulation index is
(A) 12.5 (B) 10
(C) 7.5 (D) 5
SOL 1.69 Hence (B) is correct option.
We have θi = 2π105 t + 5 sin (2π1500t) + 7.5 sin (2π1000t)
ωi = dθi = 2π105 + 10π1500 cos (2π1500t) + 15π1000 cos (2π1000t)
dt
Maximum frequency deviation is
3ωmax = 2π (5 # 1500 + 7.5 # 1000)
3 fmax = 15000
3f
Modulation index is = max = 15000 = 10
fm 1500

MCQ 1.70 The signal cos ωc t + 0.5 cos ωm t sin ωc t is


(A) FM only (B) AM only
(C) both AM and FM (D) neither AM nor FM
SOL 1.70 Hence (C) is correct option.

Brought to you by: Nodia and Company Visit us at: www.nodia.co.in


PUBLISHING FOR GATE
Page 35 GATE EC 2008 www.gatehelp.com

Common Data for Questions 71, 72 and 73 :


A speed signal, band limited to 4 kHz and peak voltage varying between +5 V and
− 5 V, is sampled at the Nyquist rate. Each sample is quantized and represented
by 8 bits.
MCQ 1.71 If the bits 0 and 1 are transmitted using bipolar pulses, the minimum bandwidth
required for distortion free transmission is
(A) 64 kHz (B) 32 kHz
(C) 8 kHz (D) 4 kHz
SOL 1.71 Hence (B) is correct option.
fm = 4 KHz
fs = 2fm = 8 kHz
Bit Rate Rb = nfs = 8 # 8 = 64 kbps
The minimum transmission bandwidth is
BW = Rb = 32 kHz
2

MCQ 1.72 Assuming the signal to be uniformly distributed between its peak to peak value,
the signal to noise ratio at the quantizer output is
(A) 16 dB (B) 32 dB
(C) 48 dB (D) 4 kHz
SOL 1.72 Hence (C) is correct option.
S0
c N m = 1.76 + 6n dB
0

= 1.76 + 6 # 8 = 49.76 dB We have n = 8


MCQ 1.73 The number of quantization levels required to reduce the quantization noise by a
factor of 4 wo
(A) 1024 (B) 512
(C) 256 (D) 64
SOL 1.73 Hence (B) is correct option.
As Noise \ 12
L
To reduce quantization noise by factor 4, quantization level must be two times i.e.
2L .
Now L = 2n = 28 = 256
Thus 2L = 512

Common data for questions 74 & 75 :


The following series RLC circuit with zero conditions is excited by a unit impulse

Brought to you by: Nodia and Company Visit us at: www.nodia.co.in


PUBLISHING FOR GATE
Page 36 GATE EC 2008 www.gatehelp.com

functions δ (t).

MCQ 1.74 For t > 0 , the output voltage vC ^ t h is

(A) 2 ^e − e th
−1
(B) 2 te 2 t
−1 3
2
t 2

3 3

(C) 2 e 2 t cos c 3 t m (D) 2 e 2 t sin c 3 t m


−1 −1

3 2 3 2
SOL 1.74 Writing in transform domain we have
Vc (s) 1
= 1 s = 2 1
Vs (s) ^s + s + 1h (s + s + 1)
Since Vs (t) = δ (t) " Vs (s) = 1 and
Vc (s) = 2 1
(s + s + 1)
3
or Vc (s) = 2 = 2
G
3 (s + 12 ) 2 + 43
Taking inverse laplace transform we have
Vt = 2 e− sin c 3 t m
t
2

3 2
Hence (D) is correct option.
MCQ 1.75 For t > 0 , the voltage across the resistor is
(A) 1 _e 2 t − e− 2 t i
3 1

=cos c 2 t m − c 2 mG
−1 t 3 1 sin 3 t
(B) e 2
3

(C) 2 e 2 t sin c 3 t m
−1

3 2

(D) 2 e 2 t cos c 3 t m
−1

3 2
SOL 1.75 Let voltage across resistor be vR
VR (s)
= 1 1 = 2 s
VS (s) ( s + s + 1) (s + s + 1)
Since vs = δ (t) " Vs (s) = 1 we get
VR (s) = 2 s = s
(s + s + 1) (s + 2 ) + 43
1 2

Brought to you by: Nodia and Company Visit us at: www.nodia.co.in


PUBLISHING FOR GATE
Page 37 GATE EC 2008 www.gatehelp.com

(s + 12 ) 1
= − 2
(s + 12 ) 2 + 43 (s + 12 ) 2 + 43
vR (t) = e− cos 3 t − 1 # 2 e− sin 3 t
1 1
or 2 2

2 2 3 2

= e− 2 =cos 3 t − 1 sin 3 tG
t

2 3 2
Hence (B) is correct option.

Linked Answer Questions : Q. 76 to Q.85 carry two marks each.

Statement for linked Answers Questions 76 & 77:


A two-port network shown below is excited by external DC source. The voltage and
the current are measured with voltmeters V1, V2 and ammeters. A1, A2 (all assumed
to be ideal), as indicated

Under following conditions, the readings obtained are:


(1) S1 -open, S2 - closed A1 = 0,V1 = 4.5 V,V2 = 1.5 V, A2 = 1 A
(2) S1 -open, S2 - closed A1 = 4 A,V1 = 6 V,V2 = 6 V, A2 = 0

MCQ 1.76 The z -parameter matrix for this network is


1. 5 1. 5 1.5 4.5
(A) =
4. 5 1. 5 G
(B) =
1.5 4.5G
1.5 4.5 4.5 1.5
(C) =
1.5 1.5 G
(D) =
1.5 4.5G
SOL 1.76 From the problem statement we have
z11 = v1 = 6 = 1. 5Ω
i1 i = 0 4
2

z12 = v1 = 4.5 = 4.5Ω


i2 i = 0
1
1
z21 = v2 = 6 = 1.5Ω
i1 i = 0 4
2

z22 = v2 = 1.5 = 1.5Ω


i2 i = 0
2
1
Thus z -parameter matrix is

Brought to you by: Nodia and Company Visit us at: www.nodia.co.in


PUBLISHING FOR GATE
Page 38 GATE EC 2008 www.gatehelp.com

z11 z12 1.5 4.5


=z z G = =1.5 1.5 G
21 22

Hence (C) is correct option.


MCQ 1.77 The h -parameter matrix for this network is
−3 3 −3 −1
(A) = G (B) =
− 1 0.67 3 0.67 G
3 3 3 1
(C) =
1 0.67 G
(D) =
− 3 − 0.67 G
SOL 1.77 From the problem statement we have
h12 = v1 = 4.5 = 3
v2 i = 0 1.5
1

h22 = i2 = 1 = 0.67
v2 i = 0 1.5
1

From z matrix, we have


v1 = z11 i1 + z12 i2
v2 = z21 i1 + z22 i2
If v2 = 0
Then i2 = − z21 = − 1.5 =− 1 = h
21
i1 z22 1.5
or i2 =− i1
Putting in equation for v1, we get
v1 = (z11 − z12) i1
v1 = h11 = z11 − z12 = 1.5 − 4.5 =− 3
i1 v = 0
2

Hence h −parameter will be


h11 h12 −3 3
=h h G = =− 1 0.67 G
21 22

Hence (A) is correct option.

Statement for Linked Answer Question 78 and 79 :


In the following network, the switch is closed at t = 0− and the sampling starts
from t = 0 . The sampling frequency is 10 Hz.

Brought to you by: Nodia and Company Visit us at: www.nodia.co.in


PUBLISHING FOR GATE
Page 39 GATE EC 2008 www.gatehelp.com

MCQ 1.78 The samples x (n), n = (0, 1, 2, ...) are given by


(A) 5 (1 − e−0.05n) (B) 5e−0.05n
(C) 5 (1 − e−5n) (D) 5e−5n
SOL 1.78 Current through resistor (i.e. capacitor) is
I = I (0+) e−t/RC
Here, I (0+) = V = 5 = 25μA
R 200k
RC = 200k # 10μ = 2 sec
I = 25e− μ A
t
2

= VR # R = 5e− V
t
2

Here the voltages across the resistor is input to sampler at frequency of 10 Hz. Thus
−n
x (n) = 5e = 5e−0.05n For t > 0
2 # 10

Hence (B) is correct answer.


MCQ 1.79 The expression and the region of convergence of the z −transform of the sampled
signal are
(A) 5z 5 , z < e−5 (B) 5z , z < e−0.05
z−e z − e−0.05

(C) 5z , z > e−0.05 (D) 5z −5 , z > e−5


z − e−0.05 z−e
SOL 1.79 Hence (C) is correct answer.
Since x (n) = 5e−0.05n u (n) is a causal signal
Its z transform is
X (z) = 5 : 1 5z
−0.05 −1 D =
1−e z z − e−0.05
Its ROC is e−0.05 z−1 > 1 " z > e−0.05

Statement for Linked Answer Questions 80 and 81:


In the following transistor circuit, VBE = 0.7 V, r3 = 25 mV/IE , and β and all the
capacitances are very large

MCQ 1.80 The value of DC current IE is

Brought to you by: Nodia and Company Visit us at: www.nodia.co.in


PUBLISHING FOR GATE
Page 40 GATE EC 2008 www.gatehelp.com

(A) 1 mA (B) 2 mA
(C) 5 mA (D) 10 mA
SOL 1.80 For the given DC values the Thevenin equivalent circuit is as follows

The thevenin resistance and voltage are


VTH = 10 # 9 = 3 V
10 + 20
and total RTH = 10k # 20k = 6.67 kΩ
10k + 20k
Since β is very large, therefore IB is small and can be ignored
Thus IE = VTH − VBE = 3 − 0.7 = 1 mA
RE 2.3k
Hence (A) is correct option.
MCQ 1.81 The mid-band voltage gain of the amplifier is approximately
(A) -180 (B) -120
(C) -90 (D) -60
SOL 1.81 The small signal model is shown in fig below

IC
gm = = 1m = 1 A/V IC . IE
VT 25m 25
Vo =− gm Vπ # (3k 3k )
=− 1 Vin (1.5k) Vπ = Vin
25
=− 60Vin
or Am = Vo =− 60
Vin

Brought to you by: Nodia and Company Visit us at: www.nodia.co.in


PUBLISHING FOR GATE
Page 41 GATE EC 2008 www.gatehelp.com

Hence (D) is correct option.

Statement For Linked Answer Question 82 & 83 :


In the following circuit, the comparators output is logic “1” if V1 > V2 and is logic
3
"0" otherwise. The D/A conversion is done as per the relation VDAC = 2n - 1 bn /
n=0
Volts, where b3

(MSB), b1, b2 and b0 (LSB) are the counter outputs. The counter starts from the
clear state.

MCQ 1.82 The stable reading of the LED displays is


(A) 06 (B) 07
(C) 12 (D) 13
SOL 1.82 Hence (D) is correct answer.
We have
3
VDAC = / 2n - 1bn = 2- 1b0 + 20 b1 + 21b2 + 22 b3
n=0
or VDAC = 0.5b0 + b1 + 2b2 + 4b3
The counter outputs will increase by 1 from 0000 till Vth > VDAC . The output of
counter and VDAC is as shown below

Clock b3 b3 b2 b0 VDAC
1 0001 0
2 0010 0.5
3 0011 1
4 0100 1.5
5 0101 2

Brought to you by: Nodia and Company Visit us at: www.nodia.co.in


PUBLISHING FOR GATE
Page 42 GATE EC 2008 www.gatehelp.com

6 0110 2.5
7 0111 3
8 1000 3.5
9 1001 4
10 1010 4.5
11 1011 5
12 1100 5.5
13 1101 6
14 1110 6.5
and when VADC = 6.5 V (at 1101), the output of AND is zero and the counter stops.
The stable output of LED display is 13.
MCQ 1.83 The magnitude of the error between VDAC and Vin at steady state in volts is
(A) 0.2 (B) 0.3
(C) 0.5 (D) 1.0
SOL 1.83 Hence (B) is correct answer.
The VADC − Vin at steady state is
= 6.5 − 6.2 = 0.3V

Statement for Linked Answer Question 84 & 85 :


The impulse response h (t) of linear time - invariant continuous time system is given
by h (t) = exp (− 2t) u (t), where u (t) denotes the unit step function.
MCQ 1.84 The frequency response H (ω) of this system in terms of angular frequency ω, is
given by H (ω)
(A) 1 (B) sin ω
1 + j2ω ω
1 jω
(C) (D)
2 + jω 2 + jω
SOL 1.84 Hence (C) is correct answer.
h (t) = e−2t u (t)
H (jω) = # h (t) e−jωt dt
3

−3

= #0 3e−2t e−jωt dt = #0 3e−(2 + jω)t dt = 1


(2 + jω)

MCQ 1.85 The output of this system, to the sinusoidal input x (t) = 2 cos 2t for all time t , is
(A) 0 (B) 2−0.25 cos (2t − 0.125π)
(C) 2−0.5 cos (2t − 0.125π) (D) 2−0.5 cos (2t − 0.25π)

Brought to you by: Nodia and Company Visit us at: www.nodia.co.in


PUBLISHING FOR GATE
Page 43 GATE EC 2008 www.gatehelp.com

SOL 1.85 Hence (D) is correct answer.


H (jω) = 1
(2 + jω)
The phase response at ω = 2 rad/sec is
+H (jω) =− tan−1 ω =− tan−1 2 =− π =− 0.25π
2 2 4
Magnitude respone at ω = 2 rad/sec is

H (jω) = 1 = 1
2 +w
2 2
2 2

Input is x (t) = 2 cos (2t)

Output i = 1 # 2 cos (2t − 0.25π)


2 2
= 1 cos [2t − 0.25π]
2

Answer Sheet
1. (C) 19. (B) 37. (A) 55. (A) 73. (B)
2. (B) 20. (A) 38. (D) 56. (B) 74. (D)
3. (A) 21. (A) 39. (C) 57. (D) 75. (B)
4. (A) 22. (D) 40. (C) 58. (A) 76. (C)
5. (A) 23. (C) 41. (C) 59. (B) 77. (A)
6. (B) 24. (A) 42. (C) 60. (A) 78. (B)
7. (C) 25. (C) 43. (B) 61. (A) 79. (C)
8. (B) 26. (A) 44. (B) 62. (D) 80. (A)
9. (C) 27. (D) 45. (C) 63. (C) 81. (D)
10. (D) 28. (B) 46. (B) 64. (C) 82. (D)
11. (C) 29. (*) 47. (B) 65. (D) 83. (B)
12. (C) 30. (A) 48. (C) 66. (C) 84. (C)
13. (D) 31. (C) 49. (C) 67. (D) 85. (D)
14. (A) 32. (A) 50. (*) 68. (D)
15. (C) 33. (D) 51. (C) 69. (B)
16. (D) 34. (B) 52. (C) 70 (C)
17. (C) 35. (B) 53. (C) 71 (B)
18. (D) 36. (A) 54. (D) 72 (C)

Brought to you by: Nodia and Company Visit us at: www.nodia.co.in


PUBLISHING FOR GATE
GATE EC
2009

Q.1 - Q.20 carry one mark each.

MCQ 1.1 The order of the differential equation


d2y dy 3
2 + c dt m + y = e
4 −t
dt
is
(A) 1 (B) 2
(C) 3 (D) 4
SOL 1.1 The highest derivative terms present in DE is of 2nd order.
Hence (B) is correct answer.
MCQ 1.2 The Fourier series of a real periodic function has only
(P) cosine terms if it is even
(Q) sine terms if it is even
(R) cosine terms if it is odd
(S) sine terms if it is odd
Which of the above statements are correct ?
(A) P and S (B) P and R
(C) Q and S (D) Q and R
SOL 1.2 The Fourier series of a real periodic function has only cosine terms if it is even and
sine terms if it is odd.
Hence (A) is correct answer.
MCQ 1.3 A function is given by f (t) = sin2 t + cos 2t . Which of the following is true ?
(A) f has frequency components at 0 and 1 Hz

(B) f has frequency components at 0 and 1 Hz


π

(C) f has frequency components at 1 and 1 Hz


2π π

(D) f has frequency components at 0.1 and 1 Hz


2π π
Brought to you by: Nodia and Company Visit us at: www.nodia.co.in
PUBLISHING FOR GATE
Page 2 GATE EC 2009 www.gatehelp.com

SOL 1.3 Given function is


f (t) = sin2 t + cos 2t = 1 − cos 2t + cos 2t = 1 + 1 cos 2t
2 2 2
The function has a DC term and a cosine function. The frequency of cosine terms is
ω = 2 = 2πf " f = 1 Hz
π
The given function has frequency component at 0 and 1 Hz.
π
Hence (B) is correct answer.
MCQ 1.4 A fully charged mobile phone with a 12 V battery is good for a 10 minute talk-time.
Assume that, during the talk-time the battery delivers a constant current of 2 A
and its voltage drops linearly from 12 V to 10 V as shown in the figure. How much
energy does the battery deliver during this talk-time?

(A) 220 J (B) 12 kJ


(C) 13.2 kJ (D) 14.4 J
SOL 1.4 The energy delivered in 10 minutes is
#0 VIdt = I #0Vdt
t t
E = = I # Area

= 2 # 1 (10 + 12) # 600 = 13.2 kJ


2
Hence (C) is correct option.
MCQ 1.5 In an n-type silicon crystal at room temperature, which of the following can have
a concentration of 4 # 1019 cm - 3 ?
(A) Silicon atoms (B) Holes
(C) Dopant atoms (D) Valence electrons
SOL 1.5 Only dopant atoms can have concentration of 4 # 1019 cm - 3 in n −type silicon at
room temperature.
Hence option (C) is correct.
MCQ 1.6 The full form of the abbreviations TTL and CMOS in reference to logic families are
(A) Triple Transistor Logic and Chip Metal Oxide Semiconductor
(B) Tristate Transistor Logic and Chip Metal Oxide Semiconductor
(C) Transistor Transistor Logic and Complementary Metal Oxide Semiconductor
(D) Tristate Transistor Logic and Complementary Metal Oxide Silicon

Brought to you by: Nodia and Company Visit us at: www.nodia.co.in


PUBLISHING FOR GATE
Page 3 GATE EC 2009 www.gatehelp.com

SOL 1.6 TTL " Transistor - Transistor logic


CMOS " Complementary Metal Oxide Semi-conductor
Hence (C) is correct answer.
MCQ 1.7 The ROC of z -transform of the discrete time sequence
x (n) = b 1 l u (n) − b 1 l u (− n − 1) is
n n

3 2
(A) 1 < z < 1 (B) z > 1
3 2 2

(C) z < 1 (D) 2 < z < 3


3
SOL 1.7 Hence (A) is correct answer
x [n] = b 1 l u (n) − b 1 l u (− n − 1)
n n

3 2
Taking z transform we have
n=3 n n =− 1
X (z) = / b 1 l z−n − / b 1 l z−n
n

n=0
3 n =− 3
2
n=3 n =− 1
/ b 13 z−1l b2z l
n
1 −1 n
= − /
n=0 n =− 3

First term gives 1 z−1 < 1 " 1 < z


3 3
Second term gives 1 z−1 > 1 " 1 > z
2 2
Thus its ROC is the common ROC of both terms. that is
1< z <1
3 2

MCQ 1.8 The magnitude plot of a rational transfer function G (s) with real coefficients is
shown below. Which of the following compensators has such a magnitude plot ?

(A) Lead compensator (B) Lag compensator


(C) PID compensator (D) Lead-lag compensator
SOL 1.8 This compensator is roughly equivalent to combining lead and lad compensators in
the same design and it is referred also as PID compensator.
Hence (C) is correct option
MCQ 1.9 A white noise process X (t) with two-sided power spectral density 1 # 10−10 W/Hz

Brought to you by: Nodia and Company Visit us at: www.nodia.co.in


PUBLISHING FOR GATE
Page 4 GATE EC 2009 www.gatehelp.com

is input to a filter whose magnitude squared response is shown below.

The power of the output process Y (t) is given by


(A) 5 # 10−7 W (B) 1 # 10−6 W
(C) 2 # 10−6 W (D) 1 # 10−5 W
SOL 1.9 Correct Option is ( )
MCQ 1.10 Which of the following statements is true regarding the fundamental mode of the
metallic waveguides shown ?

(A) Only P has no cutoff-frequency


(B) Only Q has no cutoff-frequency
(C) Only R has no cutoff-frequency
(D) All three have cutoff-frequencies
SOL 1.10 Rectangular and cylindrical waveguide doesn’t support TEM modes and have cut
off frequency.
Coaxial cable support TEM wave and doesn’t have cut off frequency.
Hence (A) is correct option.
MCQ 1.11 A fair coin is tossed 10 times. What is the probability that only the first two tosses
will yield heads?
(A) c 1 m (B) 10C2 b 1 l
2 2

2 2

(C) c 1 m (D) 10C2 b 1 l


10 10

2 2
10
SOL 1.11 Number of elements in sample space is 2 . Only one element
"H, H, T, T, T, T, T, T, T, T , is event. Thus probability is 10
1
2
Hence (C) is correct answer.
MCQ 1.12 If the power spectral density of stationary random process is a sine-squared function
of frequency, the shape of its autocorrelation is

Brought to you by: Nodia and Company Visit us at: www.nodia.co.in


PUBLISHING FOR GATE
Page 5 GATE EC 2009 www.gatehelp.com

SOL 1.12 Correct Option is ( )

MCQ 1.13 If f (z) = c 0 + c1 z−1 , then # 1 +zf (z) dz is given by


unit circle
(A) 2πc1 (B) 2π (1 + c0)
(C) 2πjc1 (D) 2π (1 + c0)
SOL 1.13 Hence (C) is correct answer
We have
f (z) = c0 + c1 z - 1
1 + f (z) 1 + c0 + c1 z - 1 z (1 + c0) + c1
f1 (z) = = =
z z z2
Since f1 (z) has double pole at z = 0 , the residue at z = 0 is
z (1 + c0) + c1
Res f1 (z) z = 0 = lim z2 .f1 (z) = lim z2 . c m = c1
z"0 z"0 z2
Hence
[1 + f (z)]
#
f1 (z) dz = # z
dz = 2πj [Residue at z = 0 ]
unit circle unit circle

= 2πjc1
MCQ 1.14 In the interconnection of ideal sources shown in the figure, it is known that the 60
V source is absorbing power.

Brought to you by: Nodia and Company Visit us at: www.nodia.co.in


PUBLISHING FOR GATE
Page 6 GATE EC 2009 www.gatehelp.com

Which of the following can be the value of the current source I ?


(A) 10 A (B) 13 A
(C) 15 A (D) 18 A
SOL 1.14 Circuit is as shown below

Since 60 V source is absorbing power. So, in 60 V source current flows from + to


- ve direction
So, I + I1 = 12
I = 12 − I1
I is always less then 12 A So, only option (A) satisfies this conditions.
Hence (A) is correct option.
MCQ 1.15 The ratio of the mobility to the diffusion coefficient in a semiconductor has the
units
(A) V - 1 (B) cm.V1
(C) V.cm - 1 (D) V.s
SOL 1.15 Hence option (A) is correct.
2
Unit of mobility μn is = cm
V. sec
2
Unit of diffusion current Dn is = cm
sec
μn 2 2
Thus unit of is = cm / cm = 1 = V−1
Dn V $ sec sec V

MCQ 1.16 In a microprocessor, the service routine for a certain interrupt starts from a fixed
location of memory which cannot be externally set, but the interrupt can be delayed
or rejected Such an interrupt is
(A) non-maskable and non-vectored
(B) maskable and non-vectored
(C) non-maskable and vectored
(D) maskable and vectored
SOL 1.16 Vectored interrupts : Vectored interrupts are those interrupts in which program
control transferred to a fixed memory location.
Maskable interrupts : Maskable interrupts are those interrupts which can be rejected

Brought to you by: Nodia and Company Visit us at: www.nodia.co.in


PUBLISHING FOR GATE
Page 7 GATE EC 2009 www.gatehelp.com

or delayed by microprocessor if it is performing some critical task.


Hence (D) is correct answer.
MCQ 1.17 If the transfer function of the following network is
Vo (s) 1
=
Vi (s) 2 + sCR

The value of the load resistance RL is


(A) R (B) R
4 2
(C) R (D) 2R
SOL 1.17 For given network we have
(RL XC ) Vi
V0 =
R + (RL XC )
RL
V0 (s)
= 1 + sRL C = RL
Vi (s) R+ R L R + RRL sC + RL
1 + sRL C

= RL = 1
R + RRL sC + RL 1+ R + RsC
RL
But we have been given
V (s) 1
T .F. = 0 =
Vi (s) 2 + sCR
Comparing, we get
1 + R = 2 & RL = R
RL
Hence (C) is correct option.
MCQ 1.18 Consider the system
dx = Ax + Bu with A = =1 0G and B = = p G
dt 0 1 q
where p and q are arbitrary real numbers. Which of the following statements about
the controllability of the system is true ?
(A) The system is completely state controllable for any nonzero values of p and q
(B) Only p = 0 and q = 0 result in controllability
(C) The system is uncontrollable for all values of p and q

Brought to you by: Nodia and Company Visit us at: www.nodia.co.in


PUBLISHING FOR GATE
Page 8 GATE EC 2009 www.gatehelp.com

(D) We cannot conclude about controllability from the given data


SOL 1.18 Hence (C) is correct option.
1 0 p
Here A == G and B = = G
0 1 q
1 0 p p
AB = = G= G == G
0 1 q q
p q
S = 8B AB B = =
q pG
S = pq − pq = 0
Since S is singular, system is completely uncontrollable for all values of p and q .
MCQ 1.19 For a message signal m (t) = cos (2πfm t) and carrier of frequency fc , which of the
following represents a single side-band (SSB) signal ?
(A) cos (2πfm t) cos (2πfc t) (B) cos (2πfc t)
(C) cos [2π (fc + fm) t] (D) [1 + cos (2πfm t) cos (2πfc t)
SOL 1.19 Hence (C) is correct option.
cos (2πfm t) cos (2πfc t) $ DSB suppressed carrier
cos (2πfc t) $ Carrier Only
cos [2π (fc + fm) t] $ USB Only
[1 + cos (2πfm t) cos (2πfc t)] $ USB with carrier
MCQ 1.20 Two infinitely long wires carrying current are as shown in the figure below. One
wire is in the y − z plane and parallel to the y − axis. The other wire is in the x − y
plane and parallel to the x − axis. Which components of the resulting magnetic
field are non-zero at the origin ?

(A) x, y, z components (B) x, y components


(C) y, z components (D) x, z components
SOL 1.20 Due to 1 A current wire in x − y plane, magnetic field be at origin will be in x
direction.
Due to 1 A current wire in y − z plane, magnetic field be at origin will be in z
Brought to you by: Nodia and Company Visit us at: www.nodia.co.in
PUBLISHING FOR GATE
Page 9 GATE EC 2009 www.gatehelp.com

direction.
Thus x and z component is non-zero at origin.
Hence (D) is correct option.

Q.21 to Q.60 carry two marks each.

MCQ 1.21 Consider two independent random variables X and Y with identical distributions.
The variables X and Y take values 0, 1 and 2 with probabilities 12 , 14 and 1
4
respectively. What is the conditional probability P (X + Y = 2 X − Y = 0) ?
(A) 0 (B) 1/16
(C) 1/6 (D) 1
SOL 1.21 Hence (C) is correct option.
We have
p (X = 0) = p (Y = 0) = 1
2
p (X = 1) = p (Y = 1) = 1
4
p (X = 2) = p (Y = 2) = 1
4
Let X+Y = 2 $ A
and X−Y = 0 $ B
Now
P (A + B)
P (X + Y = 2 X − Y = 0) =
P (B)
Event P (A + B) happen when X + Y = 2 and X − Y = 0 . It is only the case when
X = 1 and Y = 1.
Thus P (A + B) = 1 # 1 = 1
4 4 16
Now event P (B) happen when
X − Y = 0 It occurs when X = Y , i.e.
X = 0 and Y = 0 or
X = 1 and Y = 1 or
X = 2 and Y = 2
Thus P (B) = 1 # 1 + 1 # 1 + 1 # 1 = 6
2 2 4 4 4 4 16
P (A + B) 1/16
Now = =1
P (B) 6/16 6

MCQ 1.22 The Taylor series expansion of sin x at x = π is given by


x−π
(x − π) 2 (x − π) 2
(A) 1 + + ... (B) − 1 − + ...
3! 3!

Brought to you by: Nodia and Company Visit us at: www.nodia.co.in


PUBLISHING FOR GATE
Page 10 GATE EC 2009 www.gatehelp.com

(x − π) 2 (x − π) 2
(C) 1 − + ... (D) − 1 + + ...
3! 3!
SOL 1.22 Hence (D) is correct answer.
We have f (x) = sin x
x−π
Substituting x − π = y ,we get
sin (y + π)
= − 1 (sin y)
sin y
f (y + π) = =−
y y y
3 5
= − 1 cy − + − ...m
y y
y 3! 5!
y2 y 4
or f (y + π) =− 1 + − + ...
3! 5!
Substituting x − π = y we get
(x − π) 2 (x − π) 4
f (x) =− 1 + − + ...
3! 5!

MCQ 1.23 If a vector field V is related to another vector field A through V = 4# A , which of
the following is true? (Note : C and SC refer to any closed contour and any surface
whose boundary is C . )
(A) #CV $ dl = #S #C A $ d S (B) #C A $ dl = #S #CV $ d S
(C) #C Δ # V $ dl = #S #C Δ # A $ d S (D) #C Δ # V $ dl = #S #CV $ d S
SOL 1.23 Hence (B) is correct option.
We have V = 4# A ...(1)
By Stokes theorem
= # A $ dl ## (4 # A) $ ds ...(2)
From (1) and (2) we get
# A $ dl = ##V $ ds
MCQ 1.24 Given that F (s) is the one-side Laplace transform of f (t), the Laplace transform of
#0 f (τ) dτ is
t

(A) sF (s) − f (0) (B) 1 F (s)


s

#0
s
(C) F (τ) dτ (D) 1 [F (s) − f (0)]
s
SOL 1.24 By property of unilateral laplace transform

F (s) 1 0
# #
t L
f (τ) dτ + f (τ) dτ
−3 s s −3
Here function is defined for 0 < τ < t , Thus
F (s)
#
t L
f (τ)
0 s
Brought to you by: Nodia and Company Visit us at: www.nodia.co.in
PUBLISHING FOR GATE
Page 11 GATE EC 2009 www.gatehelp.com

Hence (B) is correct answer.


MCQ 1.25 Match each differential equation in Group I to its family of solution curves from
Group II
Group I Group II
dy y
A. = 1. Circles
dx x
dy y
B. =− 2. Straight lines
dx x
dy
C. =x 3. Hyperbolas
dx y
dy
D. =− x
dx y
(A) A − 2, B − 3, C − 3, D − 1
(B) A − 1, B − 3, C − 2, D − 1
(C) A − 2, B − 1, C − 3, D − 3
(D) A − 3, B − 2, C − 1, D − 2
SOL 1.25 Hence (A) is correct answer
dy y
(A) =
dx x

or # dyy = # dxx
or log y = log x + log c
or y = cx Straight Line
Thus option (A) and (C) may be correct.
dy y
(B) =−
dx x

or # dyy =− # dxx
or log y =− log x + log c
or log y = log 1 + log c
x

or y = c Hyperbola
x

MCQ 1.26 The Eigen values of following matrix are


R V
S− 1 3 5 W
S− 3 − 1 6 W
SS 0 0 3 WW
(A) 3, 3 + 5j, T6 − j X (B) − 6 + 5j, 3 + j, 3 − j

Brought to you by: Nodia and Company Visit us at: www.nodia.co.in


PUBLISHING FOR GATE
Page 12 GATE EC 2009 www.gatehelp.com

(C) 3 + j, 3 − j, 5 + j (D) 3, − 1 + 3j, − 1 − 3j


SOL 1.26 Sum of the principal diagonal element of matrix is equal to the sum of Eigen values.
Sum of the diagonal element is − 1 − 1 + 3 = 1.In only option (D), the sum of Eigen
values is 1.
Hence (D) is correct answer.
MCQ 1.27 An AC source of RMS voltage 20 V with internal impedance Zs = (1 + 2j) Ω feeds a
load of impedance ZL = (7 + 4j) Ω in the figure below. The reactive power consumed
by the load is

(A) 8 VAR (B) 16 VAR


(C) 28 VAR (D) 32 VAR
SOL 1.27 From given circuit the load current is
IL = V = 20+0c = 20+0c
Zs + ZL (1 + 2j) + (7 + 4j) 8 + 6j
= 1 (8 − 6j) = 20+0c = 2+ − φ where φ = tan - 1 3
5 10+φ 4
The voltage across load is
VL = IL ZL
The reactive power consumed by load is
Pr = VL IL* = IL ZL # IL* = ZL IL 2
2
= (7 # 4j) 20+0c = (7 + 4j) = 28 + 16j
8 + 6j
Thus average power is 28 and reactive power is 16.
Hence (B) is correct option.
MCQ 1.28 The switch in the circuit shown was on position a for a long time, and is move to
position b at time t = 0 . The current i (t) for t > 0 is given by

Brought to you by: Nodia and Company Visit us at: www.nodia.co.in


PUBLISHING FOR GATE
Page 13 GATE EC 2009 www.gatehelp.com

(A) 0.2e−125t u (t) mA (B) 20e−1250t u (t) mA


(C) 0.2e−1250t u (t) mA (D) 20e−1000t u (t) mA
SOL 1.28 At t = 0− , the circuit is as shown in fig below :

V (0−) = 100 V
Thus V (0+) = 100 V
At t = 0+ , the circuit is as shown below

I (0+) = 100 = 20 mA
5k
At steady state i.e. at t = 3 is I (3)= 0
i (t) = I (0+) e− u (t)
t
Now RCeq

(0.5μ + 0.3μ) 0.2μ


Ceq = = 0.16 μ F
0.5μ + 0.3μ + 0.2μ
1 = 1 = 1250
RCeq 5 # 103 # 0.16 # 10−6
i (t) = 20e−1250t u (t) mA
Hence (B) is correct option.
MCQ 1.29 In the circuit shown, what value of RL maximizes the power delivered to RL ?

(A) 2.4 Ω (B) 8 Ω


3
Brought to you by: Nodia and Company Visit us at: www.nodia.co.in
PUBLISHING FOR GATE
Page 14 GATE EC 2009 www.gatehelp.com

(C) 4 Ω (D) 6 Ω
SOL 1.29 For Pmax the load resistance RL must be equal to thevenin resistance Req i.e. RL
= Req . The open circuit and short circuit is as shown below

The open circuit voltage is


Voc = 100 V

From fig I1 = 100 = 12.5 A


8
Vx =− 4 # 12.5 =− 50 V

I2 = 100 + Vx = 100 − 50 = 12.5 A


4 4
Isc = I1 + I2 = 25 A

Rth = Voc = 100 = 4 Ω


Isc 25
Thus for maximum power transfer RL = Req = 4 Ω
Hence (C) is correct option.
MCQ 1.30 The time domain behavior of an RL circuit is represented by
L di + Ri = V0 (1 + Be−Rt/L sin t) u (t).
dt
For an initial current of i (0) = V0 , the steady state value of the current is given by
R
(A) i (t) " V0 (B) i (t) " 2V0
R R

(C) i (t) " V0 (1 + B) (D) i (t) " 2V0 (1 + B)


R R
SOL 1.30 Steady state all transient effect die out and inductor act as short circuits and forced
response acts only. It doesn’t depend on initial current state. From the given time
domain behavior we get that circuit has only R and L in series with V0 . Thus at
steady state
i (t) " i (3) = V0
R
Hence (A) is correct option.

Brought to you by: Nodia and Company Visit us at: www.nodia.co.in


PUBLISHING FOR GATE
Page 15 GATE EC 2009 www.gatehelp.com

MCQ 1.31 In the circuit below, the diode is ideal. The voltage V is given by

(A) min (Vi, 1) (B) max (Vi, 1)


(C) min (− Vi, 1) (D) max (− Vi, 1)
SOL 1.31 Let diode be OFF. In this case 1 A current will flow in resistor and voltage across
resistor will be V = 1.V
Diode is off, it must be in reverse biased, therefore
Vi − 1 > 0 " Vi > 1
Thus for Vi > 1 diode is off and V = 1V
Option (B) and (C) doesn’t satisfy this condition.
Let Vi < 1. In this case diode will be on and voltage across diode will be zero and
V = Vi
Thus V = min (Vi, 1)
Hence (A) is correct option.
MCQ 1.32 Consider the following two statements about the internal conditions in a n −channel
MOSFET operating in the active region.
S1 : The inversion charge decreases from source to drain
S2 : The channel potential increases from source to drain.
Which of the following is correct?
(A) Only S2 is true
(B) Both S1 and S2 are false
(C) Both S1 and S2 are true, but S2 is not a reason for S1
(D) Both S1 and S2 are true, and S2 is a reason for S1
SOL 1.32 Both S1 and S2 are true and S2 is a reason for S1.
Hence option (D) is correct.
MCQ 1.33 In the following a stable multivibrator circuit, which properties of v0 (t) depend on
R2 ?

Brought to you by: Nodia and Company Visit us at: www.nodia.co.in


PUBLISHING FOR GATE
Page 16 GATE EC 2009 www.gatehelp.com

(A) Only the frequency


(B) Only the amplitude
(C) Both the amplitude and the frequency
(D) Neither the amplitude nor the frequency
SOL 1.33 The R2 decide only the frequency.
Hence (A) is correct option
MCQ 1.34 In the circuit shown below, the op-amp is ideal, the transistor has VBE = 0.6 V and
β = 150 . Decide whether the feedback in the circuit is positive or negative and
determine the voltage V at the output of the op-amp.

(A) Positive feedback, V = 10 V. (B) Positive feedback, V = 0 V


(C) Negative feedback, V = 5 V (D) Negative feedback, V = 2 V
SOL 1.34 The circuit is shown in fig below

Brought to you by: Nodia and Company Visit us at: www.nodia.co.in


PUBLISHING FOR GATE
Page 17 GATE EC 2009 www.gatehelp.com

The voltage at non inverting terminal is 5 V because OP AMP is ideal and inverting
terminal is at 5 V.
Thus IC = 10 − 5 = 1 mA
5k
VE = IE RE = 1m # 1.4k = 1.4V IE = IC
= 0.6 + 1.4 = 2V
Thus the feedback is negative and output voltage is V = 2V .
Hence (D) is correct option.
MCQ 1.35 A small signal source Vi (t) = A cos 20t + B sin 106 t is applied to a transistor amplifier
as shown below. The transistor has β = 150 and hie = 3Ω . Which expression best
approximate V0 (t)

(A) V0 (t) =− 1500 (A cos 20t + B sin 106 t)


(B) V0 (t) = − 1500( A cos 20t + B sin 106 t)
(C) V0 (t) =− 1500B sin 106 t
(D) V0 (t) =− 150B sin 106 t
SOL 1.35 The output voltage is
hfe RC
V0 = Ar Vi .− Vi
hie
Here RC = 3 Ω and hie = 3 kΩ
Thus V0 . − 150 # 3k Vi
3k
.− 150 (A cos 20t + B sin 106 t)
Since coupling capacitor is large so low frequency signal will be filtered out, and
best approximation is
V0 .− 150B sin 106 t
Hence (D) is correct option.
MCQ 1.36 If X = 1 in logic equation 6X + Z {Y + (Z + XY )}@ {X + X (X + Y)} = 1, then
(A) Y = Z (B) Y = Z
(C) Z = 1 (D) Z = 0

Brought to you by: Nodia and Company Visit us at: www.nodia.co.in


PUBLISHING FOR GATE
Page 18 GATE EC 2009 www.gatehelp.com

SOL 1.36 We have 6X + Z {Y + (Z + XY )}@ [X + Z (X + Y)] = 1


Substituting X = 1 and X = 0 we get
[1 + Z {Y + (Z + 1Y )}][ 0 + Z (1 + Y)] = 1
or [1][ Z (1)] = 1 1 + A = 1 and 0 + A = A
or Z =1)Z=0
Hence (D) is correct answer
MCQ 1.37 What are the minimum number of 2- to -1 multiplexers required to generate a 2-
input AND gate and a 2- input Ex-OR gate
(A) 1 and 2 (B) 1 and 3
(C) 1 and 1 (D) 2 and 2
SOL 1.37 The AND gate implementation by 2:1 mux is as follows

Y = AI 0 + AI1 = AB

The EX − OR gate implementation by 2:1 mux is as follows

Y = BI0 + BI1 = AB + BA
Hence (A) is correct answer.
MCQ 1.38 Refer to the NAND and NOR latches shown in the figure. The inputs (P1, P2) for
both latches are first made (0, 1) and then, after a few seconds, made (1, 1). The
corresponding stable outputs (Q1, Q2) are

(A) NAND: first (0, 1) then (0, 1) NOR: first (1, 0) then (0, 0)
(B) NAND : first (1, 0) then (1, 0) NOR : first (1, 0) then (1, 0)
(C) NAND : first (1, 0) then (1, 0) NOR : first (1, 0) then (0, 0)
(D) NAND : first (1, 0) then (1, 1) NOR : first (0, 1) then (0, 1)

Brought to you by: Nodia and Company Visit us at: www.nodia.co.in


PUBLISHING FOR GATE
Page 19 GATE EC 2009 www.gatehelp.com

SOL 1.38 For the NAND latche the stable states are as follows

For the NOR latche the stable states are as follows

Hence (C) is correct answer.

MCQ 1.39 What are the counting states (Q1, Q2) for the counter shown in the figure below

(A) 11, 10, 00, 11, 10,... (B) 01, 10, 11, 00, 01...
(C) 00, 11, 01, 10, 00... (D) 01, 10, 00, 01, 10...
SOL 1.39 The given circuit is as follows.

The truth table is as shown below. Sequence is 00, 11, 10, 00 ...

CLK J1 K1 Q1 J2 K2 Q2
1 1 1 0 1 1 0
2 1 1 1 1 1 1
3 0 0 1 0 1 0
4 1 1 0 1 1 0

Brought to you by: Nodia and Company Visit us at: www.nodia.co.in


PUBLISHING FOR GATE
Page 20 GATE EC 2009 www.gatehelp.com

Hence (A) is correct answer.


MCQ 1.40 A system with transfer function H (z) has impulse response h (.) defined as
h (2) = 1, h (3) =− 1 and h (k) = 0 otherwise. Consider the following statements.
S1 : H (z) is a low-pass filter.
S2 : H (z) is an FIR filter.
Which of the following is correct?
(A) Only S2 is true
(B) Both S1 and S2 are false
(C) Both S1 and S2 are true, and S2 is a reason for S1
(D) Both S1 and S2 are true, but S2 is not a reason for S1
SOL 1.40 We have h (2) = 1, h (3) =− 1 otherwise h (k) = 0 . The diagram of response is as
follows :

It has the finite magnitude values. So it is a finite impulse response filter. Thus S2
is true but it is not a low pass filter. So S1 is false.
Hence (A) is correct answer.
MCQ 1.41 Consider a system whose input x and output y are related by the equation
# x (t − τ) g (2τ) dτ where h (t) is shown in the graph.
3
y (t) =
−3

Which of the following four properties are possessed by the system ?


BIBO : Bounded input gives a bounded output.
Causal : The system is causal,
LP : The system is low pass.
LTI : The system is linear and time-invariant.
(A) Causal, LP (B) BIBO, LTI
(C) BIBO, Causal, LTI (D) LP, LTI
SOL 1.41 Here h (t) ! 0 for t < 0 . Thus system is non causal. Again any bounded input x (t)
gives bounded output y (t). Thus it is BIBO stable.

Brought to you by: Nodia and Company Visit us at: www.nodia.co.in


PUBLISHING FOR GATE
Page 21 GATE EC 2009 www.gatehelp.com

Here we can conclude that option (B) is correct.


Hence (B) is correct answer.
MCQ 1.42 The 4-point Discrete Fourier Transform (DFT) of a discrete time sequence {1,0,2,3}
is
(A) [0, − 2 + 2j , 2, − 2 − 2j ] (B) [2, 2 + 2j , 6, 2 − 2j ]
(C) [6, 1 − 3j , 2, 1 + 3j ] (D) [6, − 1 + 3j , 0, − 1 − 3j ]
SOL 1.42 Hence (D) is correct answer
We have x [n] = {1, 0, 2, 3) and N = 4
N−1
X [k ] = / x [ n] e −j2πnk/N
k = 0, 1...N − 1
n=0
3
For N = 4 , X [k ] = / x [ n] e −j2πnk/4
k = 0, 1,... 3
n=0
3
Now X [ 0] = / x [n]
n=0
= x [0] + x [1] + x [2] + x [3] = 1 + 0 + 2 + 3 = 6
3
x [1] = / x [ n] e −jπn/2

n=0

= x [0] + x [1] e−jπ/2 + x [2] e−jπ + x [3] e−jπ3/2


= 1 + 0 − 2 + j3 =− 1 + j3
3
X [ 2] = / x [ n] e −jπn

n=0

= x [0] + x [1] e−jπ + x [2] e−j2π + x [3] e−jπ3


= 1+0+2−3 = 0
3
X [ 3] = / x [ n] e −j3πn/2

n=0

= x [0] + x [1] e−j3π/2 + x [2] e−j3π + x [3] e−j9π/2


= 1 + 0 − 2 − j3 =− 1 − j3
Thus [6, − 1 + j3, 0, − 1 − j3]
MCQ 1.43 The feedback configuration and the pole-zero locations of
G (s) = s2 − 2s + 2
2

s + 2s + 2
are shown below. The root locus for negative values of k , i.e. for − 3 < k < 0 , has
breakaway/break-in points and angle of departure at pole P (with respect to the
positive real axis) equal to

Brought to you by: Nodia and Company Visit us at: www.nodia.co.in


PUBLISHING FOR GATE
Page 22 GATE EC 2009 www.gatehelp.com

(A) ! 2 and 0c (B) ! 2 and 45c


(C) ! 3 and 0c (D) ! 3 and 45c
SOL 1.43 The characteristic equation is
1 + G (s) H (s) = 0
K (s2 − 2s + 2)
or 1+ =0
s2 + 2s + 2
or s2 + 2s + 2 + K (s2 − 2s + 2) = 0
K =− s2 + 2s + 2
2
or
s − 2s + 2
For break away & break in point differentiating above w.r.t. s we have
dK =− (s − 2s + 2) (2s + 2) − (s + 2s + 2) (2s − 2) = 0
2 2

ds (s2 − 2s + 2) 2
Thus (s2 − 2s + 2)( 2s + 2) − (s2 + 2s + 2)( 2s − 2) = 0
or s =! 2
Let θd be the angle of departure at pole P , then

− θd − θp1 + θz1 + θz2 = 180c


− θd = 180c − (− θp1 + θz1 + θ2)
= 180c − (90c + 180 − 45c) =− 45c
Hence (B) is correct option.
An LTI system having transfer function s +s 2+s 1+ 1 and input x (t) = sin (t + 1) is in
2
MCQ 1.44 2

steady state. The output is sampled at a rate ωs rad/s to obtain the final output
{x (k)}. Which of the following is true ?
(A) y (.) is zero for all sampling frequencies ωs
(B) y (.) is nonzero for all sampling frequencies ωs
(C) y (.) is nonzero for ωs > 2 , but zero for ωs < 2
(D) y (.) is zero for ωs > 2 , but nonzero for ω2 < 2

Brought to you by: Nodia and Company Visit us at: www.nodia.co.in


PUBLISHING FOR GATE
Page 23 GATE EC 2009 www.gatehelp.com

SOL 1.44 Hence (A) is correct answer.


MCQ 1.45 The unit step response of an under-damped second order system has steady state
value of -2. Which one of the following transfer functions has theses properties ?
(A) 2 − 2.24 (B) 2 − 3.82
s + 2.59s + 1.12 s + 1.91s + 1.91
(C) 2 − 2.24 (D) 2 − 382
s − 2.59s + 1.12 s − 1.91s + 1.91
SOL 1.45 For under-damped second order response
T (s) = 2 kωn2 where ξ < 1
s + 2ξωn s + ωn2
Thus (A) or (B) may be correct
For option (A) ωn = 1.12 and 2ξωn = 2.59 " ξ = 1.12
For option (B) ωn = 1.91 and 2ξωn = 1.51 " ξ = 0.69
Hence (B) is correct option.
MCQ 1.46 A discrete random variable X takes values from 1 to 5 with probabilities as shown
in the table. A student calculates the mean X as 3.5 and her teacher calculates the
variance of X as 1.5. Which of the following statements is true ?

k 1 2 3 4 5
P (X = k) 0.1 0.2 0.3 0.4 0.5
(A) Both the student and the teacher are right
(B) Both the student and the teacher are wrong
(C) The student is wrong but the teacher is right
(D) The student is right but the teacher is wrong
SOL 1.46 Hence (B) is correct option.
The mean is
X = Σxi pi (x)
= 1 # 0.1 + 2 # 0.2 + 3 # 0.4 + 4 # 0.2 + 5 # 0.1
= 0.1 + 0.4 + 1.2 + 0.8 + 0.5 = 3.0
X = Σxi2 pi (x)
2

= 1 # 0.1 + 4 # 0.2 + 9 # 0.4 + 16 # 0.2 + 25 # 0.1


= 0.1 + 0.8 + 3.6 + 3.2 + 2.5 = 10.2
Variance σx = X2 − ^X h2
2

= 10.2 − (3) 2 = 1.2


MCQ 1.47 A message signal given by m (t) = ( 12 ) cos ω1 t − ( 12 ) sin ω2 t amplitude - modulated
with a carrier of frequency ωC to generator s (t)[ 1 + m (t)] cos ωc t . What is the
power efficiency achieved by this modulation scheme ?
(A) 8.33% (B) 11.11%
(C) 20% (D) 25%

Brought to you by: Nodia and Company Visit us at: www.nodia.co.in


PUBLISHING FOR GATE
Page 24 GATE EC 2009 www.gatehelp.com

SOL 1.47 Hence (C) is correct option.


m (t) = 1 cos ω1 t − 1 sin ω2 t
2 2
sAM (t) = [1 + m (t)] cos ωc t
Modulation index
m (t) max
=
Vc
1 2 1 2
`2j +`2j =
m = 1
2
` 2j
1 2
2
η = m # 100 % = # 100% = 20%
m2 + 2 ` 2j +2
1 2

MCQ 1.48 A communication channel with AWGN operating at a signal to noise ration
SNR >> 1 and bandwidth B has capacity C1. If the SNR is doubled keeping
constant, the resulting capacity C2 is given by
(A) C2 . 2C1 (B) C2 . C1 + B
(C) C2 . C1 + 2B (D) C2 . C1 + 0.3B
SOL 1.48 Hence (B) is correct option.
We have C1 = B log2 `1 + S j
N
. B log2 ` S j As S >> 1
N N
If we double the S ratio then
N
C2 . B log2 ` 2S j
N
. B log2 2 + B log2 S
N
. B + C1
MCQ 1.49 A magnetic field in air is measured to be
y
B = B0 c 2 x 2 yt − 2 xt m
x +y x + y2
What current distribution leads to this field ?
[Hint : The algebra is trivial in cylindrical coordinates.]
t t
(A) J = B0 z c 2 1 2 m, r ! 0 (B) J =− B0 z c 2 2 2 m, r ! 0
μ0 x + y μ0 x + y
t
(C) J = 0, r ! 0 (D) J = B0 z c 2 1 2 m, r ! 0
μ0 x + y
SOL 1.49 Hence (C) is correct option.
y
We have Bv = B0 c 2 x 2 ay − 2 ax m ...(1)
x +y x + y2
To convert in cylindrical substituting
Brought to you by: Nodia and Company Visit us at: www.nodia.co.in
PUBLISHING FOR GATE
GATE ESE PSU’s 2019-20
ECE ENGINEERING
GATE ECE 2003-2019 SOLVED

GATE ECE 2003-2019 SOLVED Detail Solution

CONTENT COVERED:
1.Theory Notes
2.Explanation
3.Derivation
4.Example
5.Shortcut & Formula Summary
6.Previous year Paper Q. Sol.
Noted-: Single Source Follow, Revise
Multiple Time Best key of Success
1
Page

http://www.orbitmentor.com [email protected]
Page 25 GATE EC 2009 www.gatehelp.com

x = r cos φ and y = r sin φ


ax = cos φar − sin φaφ
and ay = sin φar + cos φaφ
In (1) we have
Bv = Bv0 aφ
v Bv a
Now Hv = B = 0 φ constant
μ0 μ0
Jv = 4# Hv = 0 since H is constant

MCQ 1.50 A transmission line terminates in two branches, each of length λ , as shown.
4
The branches are terminated by 50Ω loads. The lines are lossless and have the
characteristic impedances shown. Determine the impedance Zi as seen by the source.

(A) 200Ω (B) 100Ω


(C) 50Ω (D) 25Ω

SOL 1.50 The transmission line are as shown below. Length of all line is λ
4

2 2
Zi1 = Z01 = 100 = 200Ω
ZL1 50
2 2
Zi2 = Z02 = 100 = 200Ω
ZL2 50
ZL3 = Zi1 Zi2 = 200Ω 200Ω = 100Ω
2 2
Zi = Z0 = 50 = 25Ω
ZL3 100
Hence (D) is correct option.

Common Date for Question 51 and 52 :

Brought to you by: Nodia and Company Visit us at: www.nodia.co.in


PUBLISHING FOR GATE
Page 26 GATE EC 2009 www.gatehelp.com

Consider a silicon p − n junction at room temperature having the following


parameters:
Doping on the n -side = 1 # 1017 cm - 3
Depletion width on the n -side = 0.1μm
Depletion width on the p −side = 1.0μm
Intrinsic carrier concentration = 1.4 # 1010 cm - 3
Thermal voltage = 26 mV
Permittivity of free space = 8.85 # 10 - 14 F.cm - 1
Dielectric constant of silicon = 12

MCQ 1.51 The built-in potential of the junction


(A) is 0.70 V
(B) is 0.76 V
(C) is 0.82 V
(D) Cannot be estimated from the data given
SOL 1.51 Hence option (B) is correct.
We know that
NA WP = ND WN
17 −6
or NA = ND WN = 1 # 10 # 0.1−6# 10 = 1 # 1016
WP 1 # 10
The built-in potential is
Vbi = VT 1nc NA N
n i2 m
D

17 16
= 26 # 10−3 ln e 1 # 10 # 1 10 o
# 10 = 0.760
2
(1.4 # 10 )

MCQ 1.52 The peak electric field in the device is


(A) 0.15 MV . cm - 1, directed from p −region to n −region
(B) 0.15 MV . cm - 1, directed from n −region to p −region
(C) 1.80 MV . cm - 1, directed from p-retion to n −region
(D) 1.80 MV . cm - 1, directed from n −region to p −region
SOL 1.52 The peak electric field in device is directed from p to n and is
E =− eND xn from p to n
εs
= eND xn from n to p
εs
−19 17 −5
= 1.6 # 10 # 1 #−10 14
# 1 # 10 = 0.15 MV/cm
8.85 # 10 # 12
Hence option (B) is correct.

Brought to you by: Nodia and Company Visit us at: www.nodia.co.in


PUBLISHING FOR GATE
Page 27 GATE EC 2009 www.gatehelp.com

Common Data for Questions 53 and 54 :


The Nyquist plot of a stable transfer function G (s) is shown in the figure are
interested in the stability of the closed loop system in the feedback configuration
shown.

MCQ 1.53 Which of the following statements is true ?


(A) G (s) is an all-pass filter
(B) G (s) has a zero in the right-half plane
(C) G (s) is the impedance of a passive network
(D) G (s) is marginally stable
SOL 1.53 The plot has one encirclement of origin in clockwise direction. Thus G (s) has a zero
is in RHP.
Hence (B) is correct option.
MCQ 1.54 The gain and phase margins of G (s) for closed loop stability are
(A) 6 dB and 180c (B) 3 dB and 180c
(C) 6 dB and 90c (D) 3 dB and 90c
SOL 1.54 The Nyzuist plot intersect the real axis ate - 0.5. Thus
G. M. =− 20 log x =− 20 log 0.5 = 6.020 dB
And its phase margin is 90c.
Hence (C) is correct option.

Common data for Questions 55 & 56 :


The amplitude of a random signal is uniformly distributed between -5 V and 5 V.
MCQ 1.55 If the signal to quantization noise ratio required in uniformly quantizing the signal
is 43.5 dB, the step of the quantization is approximately
(A) 0.033 V (B) 0.05 V
(C) 0.0667 V (D) 0.10 V
SOL 1.55 Hence (C) is correct option.
We have SNR = 1.76 + 6n

Brought to you by: Nodia and Company Visit us at: www.nodia.co.in


PUBLISHING FOR GATE
Page 28 GATE EC 2009 www.gatehelp.com

or 43.5 = 1.76 + 6n
6n = 43.5 + 1.76
6n = 41.74 $ n . 7
No. of quantization level is
27 = 128
Step size required is
5 − (− 5)
= VH − VL = = 10
128 128 128
= .078125
. .0667
MCQ 1.56 If the positive values of the signal are uniformly quantized with a step size of 0.05
V, and the negative values are uniformly quantized with a step size of 0.1 V, the
resulting signal to quantization noise ration is approximately
(A) 46 dB (B) 43.8 dB
(C) 42 dB (D) 40 dB
SOL 1.56 For positive values step size
s+ = 0.05 V
For negative value step size
s- = 0.1 V
No. of quantization in + ive is
= 5 = 5 = 100
s+ 0.05
Thus 2n + = 100 $ n+ = 7
No. of quantization in − ve
Q1 = 5 = 5 = 50
s- 0.1
2n = 50 $ n - = 6
-
Thus
` N j+ = 1.76 + 6n = 1.76 + 42 = 43.76 dB
S +

` N j- = 1.76 + 6n = 1.76 + 36 = 37.76 dB


S -

` N j0 = 43.76 dB
Best S

Hence (B) is correct option.

Statement for Linked Answer Question 57 and 58 :


Consider for CMOS circuit shown, where the gate voltage v0 of the n-MOSFET is
increased from zero, while the gate voltage of the p −MOSFET is kept constant at
3 V. Assume, that, for both transistors, the magnitude of the threshold voltage is
1 V and the product of the trans-conductance parameter is 1mA. V - 2

Brought to you by: Nodia and Company Visit us at: www.nodia.co.in


PUBLISHING FOR GATE
Page 29 GATE EC 2009 www.gatehelp.com

MCQ 1.57 For small increase in VG beyond 1V, which of the following gives the correct
description of the region of operation of each MOSFET
(A) Both the MOSFETs are in saturation region
(B) Both the MOSFETs are in triode region
(C) n-MOSFETs is in triode and p −MOSFET is in saturation region
(D) n- MOSFET is in saturation and p −MOSFET is in triode region
SOL 1.57 For small increase in VG beyond 1 V the n − channel MOSFET goes into saturation
as VGS "+ ive and p − MOSFET is always in active region or triode region.
Hence (D) is correct option.
MCQ 1.58 Estimate the output voltage V0 for VG = 1.5 V. [Hints : Use the appropriate current-
voltage equation for each MOSFET, based on the answer to Q.57]
(A) 4 − 1 (B) 4 + 1
2 2

(C) 4 − 3 (D) 4 + 3
2 2
SOL 1.58 Hence (C) is correct option.

Statement for Linked Answer Question 59 & 60 :


Two products are sold from a vending machine, which has two push buttons P1
and P2 .
When a buttons is pressed, the price of the corresponding product is displayed in
a 7 - segment display. If no buttons are pressed, '0' is displayed signifying ‘Rs 0’.
If only P1 is pressed, ‘2’ is displayed, signifying ‘Rs. 2’
If only P2 is pressed ‘5’ is displayed, signifying ‘Rs. 5’
If both P1 and P2 are pressed, 'E' is displayed, signifying ‘Error’
The names of the segments in the 7 - segment display, and the glow of the display
for ‘0’, ‘2’, ‘5’ and ‘E’ are shown below.

Brought to you by: Nodia and Company Visit us at: www.nodia.co.in


PUBLISHING FOR GATE
Page 30 GATE EC 2009 www.gatehelp.com

Consider
(1) push buttons pressed/not pressed in equivalent to logic 1/0 respectively.
(2) a segment glowing/not glowing in the display is equivalent to logic 1/0
respectively.

MCQ 1.59 If segments a to g are considered as functions of P1 and P2 , then which of the
following is correct
(A) g = P 1 + P2, d = c + e (B) g = P1 + P2, d = c + e
(C) g = P1 + P2, e = b + c (D) g = P1 + P2, e = b + c
SOL 1.59 The given situation is as follows

The truth table is as shown below

P1 P2 a b c d e f g
0 0 1 1 1 1 1 1 0
0 1 1 0 1 1 0 1 1
1 0 1 1 0 1 1 0 1
1 1 1 0 0 1 1 1 1
From truth table we can write
a =1
b = P 1 P 2 + P1 P 2 = P 2 1 NOT Gate
c = P1 P2 + P1 P2 = P1 1 NOT Gate
d = 1 = c+e
and c = P1 P2 = P1 + P2 1 OR GATE
f = P1 P2 = P1 + P2 1 OR GATE
g = P1 P2 = P1 + P2 1 OR GATE
Thus we have g = P1 + P2 and d = 1 = c + e . It may be observed easily from figure
that
Led g does not glow only when both P1 and P2 are 0. Thus

Brought to you by: Nodia and Company Visit us at: www.nodia.co.in


PUBLISHING FOR GATE
Page 31 GATE EC 2009 www.gatehelp.com

g = P1 + P2
LED d is 1 all condition and also it depends on
d = c+e
Hence (B) is correct answer.
MCQ 1.60 What are the minimum numbers of NOT gates and 2 - input OR gates required to
design the logic of the driver for this 7 - Segment display
(A) 3 NOT and 4 OR (B) 2 NOT and 4 OR
(C) 1 NOT and 3 OR (D) 2 NOT and 3 OR
SOL 1.60 As shown in previous solution 2 NOT gates and 3-OR gates are required.
Hence (D) is correct answer.

Answer Sheet
1. (B) 13. (C) 25. (A) 37. (A) 49. (C)
2. (A) 14. (A) 26. (D) 38. (C) 50. (D)
3. (B) 15. (A) 27. (B) 39. (A) 51. (B)
4. (C) 16. (D) 28. (B) 40. (A) 52. (B)
5. (C) 17. (C) 29. (C) 41. (B) 53. (B)
6. (C) 18. (C) 30. (A) 42. (D) 54. (C)
7. (A) 19. (C) 31. (A) 43. (B) 55. (C)
8. (C) 20. (D) 32. (D) 44. (A) 56. (B)
9. (*) 21. (C) 33. (A) 45. (B) 57. (D)
10. (A) 22. (D) 34. (D) 46. (B) 58. (C)
11. (C) 23. (B) 35. (D) 47. (C) 59. (B)
12. (*) 24. (B) 36. (D) 48. (B) 60. (D)

Brought to you by: Nodia and Company Visit us at: www.nodia.co.in


PUBLISHING FOR GATE
Page 34 GATE EC 2009 www.gatehelp.com

Brought to you by: Nodia and Company Visit us at: www.nodia.co.in


PUBLISHING FOR GATE
GATE EC
2010

Q. No. 1 - 25 Carry One Mark Each

MCQ 1.1 The eigen values of a skew-symmetric matrix are


(A) always zero (B) always pure imaginary
(C) either zero or pure imaginary (D) always real
SOL 1.1 Eigen value of a Skew-symmetric matrix are either zero or pure imaginary in
conjugate pairs.
Hence (C) is correct option.

MCQ 1.2 The trigonometric Fourier series for the waveform f (t) shown below contains

(A) only cosine terms and zero values for the dc components
(B) only cosine terms and a positive value for the dc components
(C) only cosine terms and a negative value for the dc components
(D) only sine terms and a negative value for the dc components
SOL 1.2 For a function x (t) trigonometric fourier series is
3
x (t) = Ao + / [An cos nωt + Bn sin nωt]
n=1

Where, Ao = 1 # x (t) dt T0 "fundamental period


T0 T
0

An = 2 # x (t) cos nωt dt


T0 T
0

Brought to you by: Nodia and Company Visit us at: www.nodia.co.in


PUBLISHING FOR GATE
Page 2 GATE EC 2010 www.gatehelp.com

Bn = 2 # x (t) sin nωt dt


T0 T
0

For an even function x (t), Bn = 0


Since given function is even function so coefficient Bn = 0 , only cosine and constant
terms are present in its fourier series representation.
Constant term :
A0 = 1 #
3T/4
x (t) dt
T −T/4
= 1 : # Adt + # − 2AdtD
T/4 3T/4

T −T/4 T/4

= 1 :TA − 2AT D =− A
T 2 2 2
Constant term is negative.
Hence (C) is correct option.
d 2 n (x) n (x)
MCQ 1.3 A function n (x) satisfied the differential equation − 2 =0
dx 2 L
where L is a constant. The boundary conditions are : n (0) = K and n (3) = 0 . The
solution to this equation is
(A) n (x) = K exp (x/L) (B) n (x) = K exp (− x/ L )
(C) n (x) = K 2 exp (− x/L) (D) n (x) = K exp (− x/L)
SOL 1.3 Given differential equation
d 2 n (x) n (x)
− 2 =0
dx 2 L
Let n (x) = Aeλx
λx
So, Aλ2 eλx − Ae2 = 0
L
λ2 − 12 = 0 & λ = ! 1
L L
Boundary condition, n (3) = 0 so take λ =− 1
L
x
n (x) = Ae− L
n (0) = Ae0 = K & A = K
So, n (x) = Ke− (x/L)
Hence (D) is correct option.
MCQ 1.4 For the two-port network shown below, the short-circuit admittance parameter
matrix is

Brought to you by: Nodia and Company Visit us at: www.nodia.co.in


PUBLISHING FOR GATE
Page 3 GATE EC 2010 www.gatehelp.com

4 −2 1 − 0.5
(A) >
−2 4 H
(B) >
− 0.5 1 H
S S

1 0.5 4 2
(C) >
0.5 1 H
S (D) > H S
2 4
SOL 1.4 Given circuit is as shown below

By writing node equation at input port


I1 = V1 + V1 − V2 = 4V1 − 2V2 ...(1)
0.5 0.5
By writing node equation at output port
I2 = V2 + V2 − V1 =− 2V1 + 4V2 ...(2)
0.5 0.5
From (1) and (2), we have admittance matrix
4 −2
Y =>
− 2 4H
Hence (A) is correct option.
MCQ 1.5 For parallel RLC circuit, which one of the following statements is NOT correct ?
(A) The bandwidth of the circuit decreases if R is increased
(B) The bandwidth of the circuit remains same if L is increased
(C) At resonance, input impedance is a real quantity
(D) At resonance, the magnitude of input impedance attains its minimum values.
SOL 1.5 A parallel RLC circuit is shown below :

Brought to you by: Nodia and Company Visit us at: www.nodia.co.in


PUBLISHING FOR GATE
Page 4 GATE EC 2010 www.gatehelp.com

Input impedance Z in = 1
1 + 1 + jω C
R jω L
At resonance 1 = ωC
ωL
So, Z in = 1 = R (maximum at resonance)
1/R
Thus (D) is not true.
Furthermore bandwidth is ωB i.e ωB \ 1 and is independent of L,
R
Hence statements A, B, C, are true.
Hence (D) is correct option.
MCQ 1.6 At room temperature, a possible value for the mobility of electrons in the inversion
layer of a silicon n -channel MOSFET is
(A) 450 cm2 / V-s (B) 1350 cm2 / V-s
(C) 1800 cm2 / V-s (D) 3600 cm2 / V-s
SOL 1.6 At room temperature mobility of electrons for Si sample is given μn = 1350 cm2 /Vs.
For an n -channel MOSFET to create an inversion layer of electrons, a large positive
gate voltage is to be applied. Therefore, induced electric field increases and mobility
decreases.
So, Mobility μn < 1350 cm2 /Vs for n -channel MOSFET
Hence (A) is correct option.
MCQ 1.7 Thin gate oxide in a CMOS process in preferably grown using
(A) wet oxidation (B) dry oxidation
(C) epitaxial oxidation (D) ion implantation
SOL 1.7 Dry oxidation is used to achieve high quality oxide growth.
Hence (B) is correct option.
MCQ 1.8 In the silicon BJT circuit shown below, assume that the emitter area of transistor
Q1 is half that of transistor Q2

The value of current Io is approximately


(A) 0.5 mA (B) 2 mA

Brought to you by: Nodia and Company Visit us at: www.nodia.co.in


PUBLISHING FOR GATE
Page 5 GATE EC 2010 www.gatehelp.com

(C) 9.3 mA (D) 15 mA


SOL 1.8 Since, emitter area of transistor Q1 is half of transistor Q2 , so current
IE = 1 IE and IB = 1 IB
1
2 2 1
2 2

The circuit is as shown below :

VB =− 10 − (− 0.7) =− 9.3 V
Collector current
0 − (− 9.3)
I1 = = 1 mA
(9.3 kΩ)
β 1 = 700 (high), So IC . IE 1

Applying KCL at base we have


1 − IE = IB + IB
1 2

1 − (β 1 + 1) IB = IB + IB
1 1 2

I
1 = (700 + 1 + 1) B + IB
2

2 2

IB . 2
2
702
I 0 = IC = β 2 : IB = 715 # 2 . 2 mA
2 2
702
Hence (B) is correct option.
MCQ 1.9 The amplifier circuit shown below uses a silicon transistor. The capacitors CC and
CE can be assumed to be short at signal frequency and effect of output resistance
r0 can be ignored. If CE is disconnected from the circuit, which one of the following
statements is true

Brought to you by: Nodia and Company Visit us at: www.nodia.co.in


PUBLISHING FOR GATE
Page 6 GATE EC 2010 www.gatehelp.com

(A) The input resistance Ri increases and magnitude of voltage gainAV decreases
(B) The input resistance Ri decreases and magnitude of voltage gain AV increases
(C) Both input resistance Ri and magnitude of voltage gain AV decreases
(D) Both input resistance Ri and the magnitude of voltage gain AV increases
SOL 1.9 The equivalent circuit of given amplifier circuit (when CE is connected, RE is short-
circuited)

Input impedance Ri = RB || r π
Voltage gain AV = gm RC
Now, if CE is disconnected, resistance RE appears in the circuit

Input impedance R in = RB || [rπ + (β + 1)] RE


Input impedance increases
gm RC
Voltage gain AV = Voltage gain decreases.
1 + gm R E
Hence (A) is correct option.
MCQ 1.10 Assuming the OP-AMP to be ideal, the voltage gain of the amplifier shown below

Brought to you by: Nodia and Company Visit us at: www.nodia.co.in


PUBLISHING FOR GATE
Page 7 GATE EC 2010 www.gatehelp.com

is

(A) − R2 (B) − R 3
R1 R1

(D) −b R2 + R 3 l
R2 || R 3
(C) −
R1 R1
SOL 1.10 The circuit is as shown below :

So, 0 − Vi + 0 − Vo = 0
R1 R2
or Vo =− R2
Vi R1
Hence (A) is correct option.
MCQ 1.11 Match the logic gates in Column A with their equivalents in Column B

Brought to you by: Nodia and Company Visit us at: www.nodia.co.in


PUBLISHING FOR GATE
Page 8 GATE EC 2010 www.gatehelp.com

(A) P-2, Q-4, R-1, S-3 (B) P-4, Q-2, R-1, S-3
(C) P-2, Q-4, R-3, S-1 (D) P-4, Q-2, R-3, S-1
SOL 1.11 Hence Correct Option is (D)

MCQ 1.12 For the output F to be 1 in the logic circuit shown, the input combination should
be

(A) A = 1, B = 1, C = 0 (B) A = 1, B = 0, C = 0
(C) A = 0, B = 1, C = 0 (D) A = 0, B = 0, C = 1
SOL 1.12 In the circuit F = (A 5 B) 9 (A 9 B) 9 C
For two variables A 5 B = A9B
So,(A 5 B) 9 (A 9 B) = 0 (always)
F = 09C = 0$C+1$C = C
So, F = 1 when C = 1 or C = 0
Hence (A) (B) (C) are correct options.
MCQ 1.13 In the circuit shown, the device connected Y5 can have address in the range

Brought to you by: Nodia and Company Visit us at: www.nodia.co.in


PUBLISHING FOR GATE
Page 9 GATE EC 2010 www.gatehelp.com

(A) 2000-20FF (B) 2D00-2DFF


(C) 2E00-2EFF (D) FD00-FDFF
SOL 1.13 Since G2 is active low input, output of NAND gate must be 0
G2 = A15 : A14 A13 A12 A11 = 0
So, A15 A14 A13 A12 A11 = 00101
To select Y5 Decoder input
ABC = A 8 A 9 A10 = 101
Address range
A15 A14 A13 A12 A11 A10 A 9 A 8 ...............A 0

S S
0011101........A 0
2 D

^2D00 − 2DFF h
Hence (B) is correct option.
MCQ 1.14 Consider the z -transform x (z) = 5z2 + 4z−1 + 3; 0 < z < 3. The inverse z -
transform x [n] is
(A) 5δ [n + 2] + 3δ [n] + 4δ [n − 1] (B) 5δ [n − 2] + 3δ [n] + 4δ [n + 1]

(C) 5u [n + 2] + 3u [n] + 4u [n − 1] (D) 5u [n − 2] + 3u [n] + 4u [n + 1]


SOL 1.14 Hence (A) is correct option. Hence (A) is correct option.
Inverse Z − transform
We know that αZ ! a αδ [n ! a]
−1
Given that X (z) = 5z + 4z + 3
2

Inverse z-transform x [n] = 5δ [n + 2] + 4δ [n − 1] + 3δ [n]


MCQ 1.15 Two discrete time system with impulse response h1 [n] = δ [n − 1] and h2 [n] = δ [n − 2]
are connected in cascade. The overall impulse response of the cascaded system is
(A) δ [n − 1] + δ [n − 2] (B) δ [n − 4]

(C) δ [n − 3] (D) δ [n − 1] δ [n − 2]

Brought to you by: Nodia and Company Visit us at: www.nodia.co.in


PUBLISHING FOR GATE
Page 10 GATE EC 2010 www.gatehelp.com

SOL 1.15 Hence (C) is correct option


We have h1 [n] = δ [n − 1] or H1 [Z ] = Z − 1
and h 2 [n] = δ [n − 2] or H2 (Z ) = Z − 2
Response of cascaded system
H (z ) = H1 (z ) : H2 (z ) = z−1 : z−2 = z−3
or, h [n] = δ [n − 3]
MCQ 1.16 For a N -point FET algorithm N = 2m which one of the following statements is
TRUE ?
(A) It is not possible to construct a signal flow graph with both input and output
in normal order
(B) The number of butterflies in the m th stage in N/m
(C) In-place computation requires storage of only 2N data
(D) Computation of a butterfly requires only one complex multiplication.
SOL 1.16 For an N-point FET algorithm butterfly operates on one pair of samples and
involves two complex addition and one complex multiplication.
Hence (D) is correct option.
MCQ 1.17 The transfer function Y (s) /R (s) of the system shown is

(A) 0 (B) 1
s+1

(C) 2 (D) 2
s+1 s+3
SOL 1.17 From the given block diagram

H (s) = Y (s) − E (s) $ 1


s+1
E (s) = R (s) − H (s)

Brought to you by: Nodia and Company Visit us at: www.nodia.co.in


PUBLISHING FOR GATE
Page 11 GATE EC 2010 www.gatehelp.com

E (s)
= R (s) − Y (s) +
(s + 1)
E (s) :1 − 1
s + 1D
= R (s) − Y (s)

sE (s)
= R (s) − Y (s) ...(1)
(s + 1)
E (s)
Y (s) = ...(2)
s+1
From (1) and (2) sY (s) = R (s) − Y (s)
(s + 1) Y (s) = R (s)
Transfer function
Y (s)
= 1
R (s) s + 1
Hence (B) is correct option.
A system with transfer function X^^s hh = s +s p has an output y (t) = cos ^2t − π3 h for the
Ys
MCQ 1.18
input signal x (t) = p cos ^2t − π2 h. Then, the system parameter p is
(A) 3 (B) 2
3

(C) 1 (D) 3
2
SOL 1.18 Transfer function is given as
Y (s)
H (s) = = s
X (s) s + p

H (jω) =
jω + p
Amplitude Response
H (jω) = ω
ω +p2
2

Phase Response θh (ω) = 90c − tan−1 a ω k


p

Input x (t) = p cos a2t − π k


2
Output y (t) = H (jω) x (t − θh) = cos a2t − π k
3
H (jω) = p = ω
ω +p2
2

1 = 2 , (ω = 2 rad/ sec)
p 4+p2
or 4p 2 = 4 + p 2 & 3p 2 = 4

or p = 2/ 3
Alternative :
Brought to you by: Nodia and Company Visit us at: www.nodia.co.in
PUBLISHING FOR GATE
Page 12 GATE EC 2010 www.gatehelp.com

θh = 9− π − a− π kC = π
3 2 6
π = π − tan−1 ω
So,
6 2 apk

tan−1 a ω k = π − π = π
p 2 6 3
ω = tan π = 3
p a3k
2 = 3 , (ω = 2 rad/ sec)
p
or p = 2/ 3
Hence (B) is correct option
MCQ 1.19 For the asymptotic Bode magnitude plot shown below, the system transfer function
can be

(A) 10s + 1 (B) 100s + 1


0.1s + 1 0.1s + 1

(C) 100s (D) 0.1s + 1


10s + 1 10s + 1
SOL 1.19 Initial slope is zero, so K = 1
At corner frequency ω 1 = 0.5 rad/ sec , slope increases by + 20 dB/decade, so there
is a zero in the transfer function at ω 1
At corner frequency ω 2 = 10 rad/ sec , slope decreases by − 20 dB/decade and
becomes zero, so there is a pole in transfer function at ω 2
K a1 + s k
ω1
Transfer function H (s) =
a1 + ω 2 k
s

1 a1 + s k (1 + 10s)
0. 1
= =
(1 + 0.1s)
a1 + 0.1 k
s

Hence (A) is correct option


MCQ 1.20 Suppose that the modulating signal is m (t) = 2 cos (2πfm t) and the carrier signal
is xC (t) = AC cos (2πfC t), which one of the following is a conventional AM signal

Brought to you by: Nodia and Company Visit us at: www.nodia.co.in


PUBLISHING FOR GATE
Page 13 GATE EC 2010 www.gatehelp.com

without over-modulation
(A) x (t) = AC m (t) cos (2πfC t)
(B) x (t) = AC [1 + m (t)] cos (2πfC t)
(C) x (t) = AC cos (2πfC t) + AC m (t) cos (2πfC t)
4
(D) x (t) = AC cos (2πfm t) cos (2πfC t) + AC sin (2πfm t) sin (2πfC t)
SOL 1.20 Conventional AM signal is given by
x (t) = AC [1 + μm (t)] cos (2πfC t)
Where μ < 1, for no over modulation.
In option (C)
x (t) = AC :1 + 1 m (t)D cos (2πfC t)
4
Thus μ = 1 < 1 and this is a conventional AM-signal without over-modulation
4
Hence (C) is correct option.
MCQ 1.21 Consider an angle modulated signal
x (t) = 6 cos [2π # 106 t + 2 sin (800πt)] + 4 cos (800πt)
The average power of x (t) is
(A) 10 W (B) 18 W
(C) 20 W (D) 28 W
SOL 1.21 Hence (B) is correct option.
(6) 2
Power P = = 18 W
2

MCQ 1.22 If the scattering matrix [S ] of a two port network is

0.2 0c 0.9 90c


[S ] = > H, then the network is
0.9 90c 0.1 90c

(A) lossless and reciprocal (B) lossless but not reciprocal


(C) not lossless but reciprocal (D) neither lossless nor reciprocal
SOL 1.22 For a lossless network
S11 2 + S21 2 = 1
For the given scattering matrix
S11 = 0.2 0c , S12 = 0.9 90c
S21 = 0.9 90c , S22 = 0.1 90c
Here, (0.2) 2 + (0.9) 2 ! 1 (not lossless)
Reciprocity :
S12 = S21 = 0.9 90c (Reciprocal)
Hence (C) is correct option.

Brought to you by: Nodia and Company Visit us at: www.nodia.co.in


PUBLISHING FOR GATE
Page 14 GATE EC 2010 www.gatehelp.com

MCQ 1.23 A transmission line has a characteristic impedance of 50 Ω and a resistance of


0.1 Ω/m . If the line is distortion less, the attenuation constant(in Np/m) is
(A) 500 (B) 5
(C) 0.014 (D) 0.002
SOL 1.23 For distortion less transmission line characteristics impedance
Z0 = R
G
Attenuation constant
α = RG

So, α = R = 0.1 = 0.002


Z0 50

Hence (D) is correct option.


MCQ 1.24 Consider the pulse shape s (t) as shown. The impulse response h (t) of the filter
matched to this pulse is

SOL 1.24 Impulse response of the matched filter is given by


h (t) = S (T − t)

Brought to you by: Nodia and Company Visit us at: www.nodia.co.in


PUBLISHING FOR GATE
Page 15 GATE EC 2010 www.gatehelp.com

Hence (C) is correct option.


MCQ 1.25 The electric field component of a time harmonic plane EM wave traveling in a
nonmagnetic lossless dielectric medium has an amplitude of 1 V/m. If the relative
permittivity of the medium is 4, the magnitude of the time-average power density
vector (in W/m2 ) is
(A) 1 (B) 1
30π 60π

(C) 1 (D) 1
120π 240π
SOL 1.25 Intrinsic impedance of EM wave
μ μ0
η = = = 120π = 60π
ε 4ε0 2
Time average power density
2
Pav = 1 EH = 1 E = 1 = 1
2 2 η 2 # 60π 120π
Hence (C) is correct option.

Q. No. 26-51 carry two marks each :

MCQ 1.26 If ey = x1/x , then y has a


(A) maximum at x = e (B) minimum at x = e
−1
(C) maximum at x = e (D) minimum at x = e−1
SOL 1.26 Hence (A) is correct option.
1
Given that ey = x x
1
or ln ey = ln x x
or y = 1 ln x
x

Brought to you by: Nodia and Company Visit us at: www.nodia.co.in


PUBLISHING FOR GATE
Page 16 GATE EC 2010 www.gatehelp.com

= 1 1 + ln x ^− x− x h = 12 − ln2
dy 1
Now 2

dx xx x x
For maxima and minima :
dy
= 12 (1 − ln x) = 0
dx x
ln x = 1 " x = e 1
d 2y
Now =− 23 − ln x b− 23 l − 12 b 1 l
dx 2 x x x x
=− 22 + 2 ln3 x − 13
x x x
= −22 + 23 − 13 < 0
2
d x
dy 2 at x = e
1 e e e
So, y has a maximum at x = e1
MCQ 1.27 A fair coin is tossed independently four times. The probability of the event “the
number of time heads shown up is more than the number of times tail shown up”
(A) 1 (B) 1
16 8

(C) 1 (D) 5
4 16
SOL 1.27 According to given condition head should comes 3 times or 4 times
P (Heads comes 3 times or 4 times) = 4C 4 b 1 l + 4C 3 b 1 l b 1 l
4 3

2 2 2
= 1: 1 +4:1 :1 = 5
16 8 2 16
Hence (D) is correct option.
MCQ 1.28 v = xyatx + x 2 aty , then
If A # Av .dlv over the path shown in the figure is
C

(A) 0 (B) 2
3

(C) 1 (D) 2 3
SOL 1.28 Hence (C) is correct option
Av = xyatx + x 2 aty

Brought to you by: Nodia and Company Visit us at: www.nodia.co.in


PUBLISHING FOR GATE
Page 17 GATE EC 2010 www.gatehelp.com

v = dxatx + dyaty
dl
# Av : dl
v = # (xyatx + x 2 aty) : (dxatx + dyaty)
C C

= # (xydx + x 2 dy)
C

= #1/
2/ 3
xdx +
1/ 3
#2/ 3xdx + #1
3 4 dy + #3
1 1 dy
3 3 3 3
= 1 : 4 − 1 D + 3 :1 − 4 D + 4 [3 − 1] + 1 [1 − 3]
2 3 3 2 3 3 3 3
=1

MCQ 1.29 The residues of a complex function x (z) = 1 − 2z at its poles are
z (z − 1) (z − 2)
(A) 1 , − 1 and 1 (B) 1 , − 1 and − 1
2 2 2 2

(C) 1 , − 1 and − 3 (D) 1 , − 1 and 3


2 2 2 2
SOL 1.29 Hence (C) is correct option.
Given function
X (z ) = 1 − 2z
z (z − 1) (z − 2)
Poles are located at z = 0, z = 1, and z = 2
At Z = 0 residues is
R 0 = z : X (z) Z = 0 = 1 − 2 # 0 = 1
(0 − 1) (0 − 2) 2
at z = 1, R1 = (Z − 1) : X (Z ) Z = 1

= 1−2#1 = 1
1 (1 − 2)
At z = 2 , R2 = (z − 2) : X (z) z = 2

= 1 − 2 # 2 =− 3
2 (2 − 1) 2

Consider differential equation dx^ h − y (x) = x , with the initial condition y (0) = 0 .
dy x
MCQ 1.30
Using Euler’s first order method with a step size of 0.1, the value of y (0.3) is
(A) 0.01 (B) 0.031
(C) 0.0631 (D) 0.1
SOL 1.30 Hence (B) is correct option.
Taking step size h = 0.1, y (0) = 0
x y dy dy
= x+y yi + 1 = yi + h
dx dx
0 0 0 y1 = 0 + 0.1 (0) = 0

Brought to you by: Nodia and Company Visit us at: www.nodia.co.in


PUBLISHING FOR GATE
Page 18 GATE EC 2010 www.gatehelp.com

x y dy dy
= x+y yi + 1 = yi + h
dx dx
0.1 0 0.1 y2 = 0 + 0.1 (0.1) = 0.01
0.2 0.01 0.21 y 3 = 0.01 + 0.21 # 0.1 = 0.031
0.3 0.031
From table, at x = 0.3, y (x = 0.3) = 0.031

Given f (t) = L−1 ; 3 3s + 1


s + 4s2 + (k − 3) s E
MCQ 1.31 . If lim f (t) = 1, then the value of k is
t"3

(A) 1 (B) 2
(C) 3 (D) 4
SOL 1.31 Hence (D) is correct option.
f (t) = L − 1 ; 3 3s + 1
s + 4s 2 + (k − 3) s E
We have

and lim f (t) = 1


t"3

By final value theorem


lim f (t) = lim sF (s) = 1
t"3 s"0

s. (3s + 1)
or lim =1
s"0 s + 4s2 + (k − 3) s
3

s (3s + 1)
or lim 2 =1
s " 0 s [s + 4s + (k − 3)]

1 =1
k−3
or k =4
MCQ 1.32 In the circuit shown, the switch S is open for a long time and is closed at t = 0 .
The current i (t) for t $ 0+ is

(A) i (t) = 0.5 − 0.125e−1000t A (B) i (t) = 1.5 − 0.125e−1000t A

(C) i (t) = 0.5 − 0.5e−1000t A (D) i (t) = 0.375e−1000t A


SOL 1.32 Hence (A) is correct option.
Let the current i (t) = A + Be−t/τ τ " Time constant
Brought to you by: Nodia and Company Visit us at: www.nodia.co.in
PUBLISHING FOR GATE
Page 19 GATE EC 2010 www.gatehelp.com

When the switch S is open for a long time before t < 0 , the circuit is

At t = 0 , inductor current does not change simultaneously, So the circuit is

Current is resistor (AB)


i (0) = 0.75 = 0.375 A
2
Similarly for steady state the circuit is as shown below

i (3) = 15 = 0.5 A
3
−3
τ = L = 15 # 10 = 10−3 sec
Req 10 + (10 || 10)
t
i (t) = A + Be− 1 # 10 = A + Be−100t
−3

Now i (0) = A + B = 0.375


and i (3) = A = 0.5
So, B = 0.375 − 0.5 =− 0.125
Hence i (t) = 0.5 − 0.125e−1000 t A
MCQ 1.33 The current I in the circuit shown is

(A) − j1 A (B) j1 A
(C) 0 A (D) 20 A

Brought to you by: Nodia and Company Visit us at: www.nodia.co.in


PUBLISHING FOR GATE
Page 20 GATE EC 2010 www.gatehelp.com

SOL 1.33 Circuit is redrawn as shown below

Where, Z1 = jωL = j # 103 # 20 # 10−3 = 20j


Z2 = R || XC
XC = 1 = 1 =− 20j
jωC j # 103 # 50 # 10−6
1 (− 20j)
Z2 = R = 1Ω
1 − 20j
Voltage across Z2
− 20j
c 1 − 20j m
VZ = Z2 : 20 0 = : 20
Z1 + Z 2
c 20j − 1 − 20j m
2
20j

(− 20j)
=c
20j + 400 − 20j m
: 20 =− j

Current in resistor R is
V j
I = Z =− =− j A
2

R 1
Hence (A) is correct option.
MCQ 1.34 In the circuit shown, the power supplied by the voltage source is

(A) 0 W (B) 5 W
(C) 10 W (D) 100 W
SOL 1.34 The circuit can be redrawn as

Brought to you by: Nodia and Company Visit us at: www.nodia.co.in


PUBLISHING FOR GATE
Page 21 GATE EC 2010 www.gatehelp.com

Applying nodal analysis


VA − 10 + 1 + VA − 0 = 0
2 2
2VA − 10 + 2 = 0 = V4 = 4 V
Current, I1 = 10 − 4 = 3 A
2
Current from voltage source is
I 2 = I1 − 3 = 0
Since current through voltage source is zero, therefore power delivered is zero.
Hence (A) is correct option.
MCQ 1.35 In a uniformly doped BJT, assume that NE , NB and NC are the emitter, base and
collector doping in atoms/cm3 , respectively. If the emitter injection efficiency of the
BJT is close unity, which one of the following condition is TRUE
(A) NE = NB = NC (B) NE >> NB and NB > NC

(C) NE = NB and NB < NC (D) NE < NB < NC


SOL 1.35 Emitter injection efficiency is given as
γ = 1
1 + NB
NE
To achieve γ = 1, NE >> NB
Hence (B) is correct option.
MCQ 1.36 Compared to a p-n junction with NA = ND = 1014 /cm3 , which one of the following
statements is TRUE for a p-n junction with NA = ND = 1020 /cm3 ?
(A) Reverse breakdown voltage is lower and depletion capacitance is lower
(B) Reverse breakdown voltage is higher and depletion capacitance is lower
(C) Reverse breakdown voltage is lower and depletion capacitance is higher
(D) Reverse breakdown voltage is higher and depletion capacitance is higher
SOL 1.36 Reverse bias breakdown or Zener effect occurs in highly doped PN junction through
tunneling mechanism. In a highly doped PN junction, the conduction and valence

Brought to you by: Nodia and Company Visit us at: www.nodia.co.in


PUBLISHING FOR GATE
Page 22 GATE EC 2010 www.gatehelp.com

bands on opposite sides of the junction are sufficiently close during reverse bias
that electron may tunnel directly from the valence band on the p-side into the
conduction band on n -side.
Breakdown voltage VB \ 1
NA ND
So, breakdown voltage decreases as concentration increases
Depletion capacitance
C =' eεs NA ND
1
1/2

2 (Vbi + VR) (NA + ND)


Thus C \ NA ND
Depletion capacitance increases as concentration increases
Hence (C) is correct option.
MCQ 1.37 Assuming that the flip-flop are in reset condition initially, the count sequence
observed at QA , in the circuit shown is

(A) 0010111... (B) 0001011...


(C) 0101111... (D) 0110100....
SOL 1.37 Let QA (n), QB (n), QC (n) are present states and QA (n + 1), QB (n + 1), QC (n + 1) are
next states of flop-flops.
In the circuit
QA (n + 1) = QB (n) 9 QC (n)
QB (n + 1) = QA (n)
QC (n + 1) = QB (n)
Initially all flip-flops are reset
1st clock pulse
QA = 0 9 0 = 1
QB = 0
QC = 0
2 nd clock pulse
QA = 0 9 0 = 1
QB = 1
QC = 0
rd
3 clock pulse

Brought to you by: Nodia and Company Visit us at: www.nodia.co.in


PUBLISHING FOR GATE
Page 23 GATE EC 2010 www.gatehelp.com

QA = 1 9 0 = 0
QB = 1
QC = 1
4 th clock pulse
QA = 1 9 1 = 1
QB = 0
QC = 1
So, sequence QA = 01101.......
Hence (D) is correct option.
MCQ 1.38 The transfer characteristic for the precision rectifier circuit shown below is (assume
ideal OP-AMP and practical diodes)

SOL 1.38 The circuit is as shown below

Brought to you by: Nodia and Company Visit us at: www.nodia.co.in


PUBLISHING FOR GATE
Page 24 GATE EC 2010 www.gatehelp.com

Current I = 20 − 0 + Vi − 0 = 5 + Vi
4R R R
If I > 0, diode D2 conducts
So, for 5 + VI > 0 & VI > − 5, D2 conducts
2
Equivalent circuit is shown below

Output is Vo = 0 . If I < 0 , diode D2 will be off


5 + VI < 0 & V < − 5, D is off
I 2
R
The circuit is shown below

0 − Vi + 0 − 20 + 0 − Vo = 0
R 4R R

or Vo =− Vi − 5

At Vi =− 5 V, Vo = 0
At Vi =− 10 V, Vo = 5 V
Hence (B) is correct option.
MCQ 1.39 The Boolean function realized by the logic circuit shown is

Brought to you by: Nodia and Company Visit us at: www.nodia.co.in


PUBLISHING FOR GATE
Page 25 GATE EC 2010 www.gatehelp.com

(A) F = Σm (0, 1, 3, 5, 9, 10, 14) (B) F = Σm (2, 3, 5, 7, 8, 12, 13)

(C) F = Σm (1, 2, 4, 5, 11, 14, 15) (D) F = Σm (2, 3, 5, 7, 8, 9, 12)


SOL 1.39 Output of the MUX can be written as
F = I 0 S 0 S1 + I1 S 0 S1 + I 2 S 0 S1 + I 3 S 0 S1
Here, I 0 = C, I1 = D, I2 = C , I 3 = CD
and S 0 = A, S1 = B
So, F = C A B + D A B + C A B + C DA B
Writing all SOP terms
F = A B C D + A B C D + A BCD + A B C D
1 44 2
m
44 3 1 44 2 m
44 3 S m7
1 44 2
m
4
43
3 2 5

+A B C D + A B C D + ABC D
1 44 2
m
4
4 3 1 44 2
m
44 3 S
m 12
9 8

F = / m (2, 3, 5, 7, 8, 9, 12)

Hence (D) is correct option.


MCQ 1.40 For the 8085 assembly language program given below, the content of the accumulator
after the execution of the program is

(A) 00H (B) 45H


(C) 67H (D) E7H
SOL 1.40 By executing instruction one by one
MVI A, 45 H & MOV 45 H into accumulator, A = 45 H
STC & Set carry, C = 1
CMC & Complement carry flag, C = 0
RAR & Rotate accumulator right through carry

Brought to you by: Nodia and Company Visit us at: www.nodia.co.in


PUBLISHING FOR GATE
Page 26 GATE EC 2010 www.gatehelp.com

A = 00100010
XRA B& XOR A and B
A = A 5 B = 00100010 5 01000101 = 01100111 = 674
Hence (C) is correct option.
MCQ 1.41 A continuous time LTI system is described by

d 2 y (t) dy (t) dx (t)


2 +4 + 3y (t) = 2 + 4x (t)
dt dt dt
Assuming zero initial conditions, the response y (t) of the above system for the
input x (t) = e−2t u (t) is given by

(A) (et − e3t) u (t) (B) (e−t − e−3t) u (t)

(C) (e−t + e−3t) u (t) (D) (et + e3t) u (t)


SOL 1.41 System is described as
d 2 y (t) dt (t) dx (t)
+4 + 3y (t) = 2 + 4x (t)
dt 2 dt dt
Taking laplace transform on both side of given equation
s 2 Y (s) + 4sY (s) + 3Y (s) = 2sX (s) + 4X (s)
(s 2 + 4s + 3) Y (s) = 2 (s + 2) X (s) s
Transfer function of the system
Y (s) 2 (s + 2) 2 (s + 2)
H (s) = = 2 =
X (s) s + 4s + 3 (s + 3) (s + 1)
Input x (t) = e−2t u (t)
or, X (s) = 1
(s + 2)
Output Y (s) = H (s) : X (s)
2 (s + 2)
Y (s) = : 1
(s + 3) (s + 1) (s + 2)
By Partial fraction
Y (s) = 1 − 1
s+1 s+3
Taking inverse laplace transform
y (t) = (e−t − e−3t) u (t)
Hence (B) is correct option.
MCQ 1.42 The transfer function of a discrete time LTI system is given by

Brought to you by: Nodia and Company Visit us at: www.nodia.co.in


PUBLISHING FOR GATE
Page 27 GATE EC 2010 www.gatehelp.com

2 − 3 z−1
H (z) = 4
1 − z + 1 z−2
3 −1
4 8
Consider the following statements:
S1: The system is stable and causal for ROC: z > 1/2
S2: The system is stable but not causal for ROC: z < 1/4
S3: The system is neither stable nor causal for ROC: 1/4 < z < 1/2
Which one of the following statements is valid ?
(A) Both S1 and S2 are true (B) Both S2 and S3 are true
(C) Both S1 and S3 are true (D) S1, S2 and S3 are all true
SOL 1.42 Hence (C) is correct option.
We have
2 − 34 z − 1
H (z) =
1 − 34 z − 1 + 18 z − 2
By partial fraction H (z ) can be written as
H (z ) = 1 1
1 −1 +
^1 − 2 z h ^ 1 − 1 −1
4z h
For ROC : z > 1/2

h [n] = b 1 l u [n] + b 1 l u [n], n > 0


n n
1 = an u [n], z > a
2 4 1 − z −1
Thus system is causal. Since ROC of H (z ) includes unit circle, so it is stable also.
Hence S1 is True
For ROC : z < 1
4
h [n] =−b 1 l u [− n − 1] + b 1 l u (n), z > 1 , z < 1
n n

2 4 4 2
System is not causal. ROC of H (z ) does not include unity circle, so it is not stable
and S 3 is True
MCQ 1.43 The Nyquist sampling rate for the signal
sin (500πt) sin (700) πt
s (t) = # is given by
πt πt
(A) 400 Hz (B) 600 Hz
(C) 1200 Hz (D) 1400 Hz
SOL 1.43 Hence(C) is correct option.
S (t) = sin c (500t) sin c (700t)
S (f ) is convolution of two signals whose spectrum covers f 1 = 250 Hz and f 2 = 350 Hz
. So convolution extends
f = 25 + 350 = 600 Hz
Nyquist sampling rate

Brought to you by: Nodia and Company Visit us at: www.nodia.co.in


PUBLISHING FOR GATE
Page 28 GATE EC 2010 www.gatehelp.com

N = 2f = 2 # 600 = 1200 Hz

MCQ 1.44 A unity negative feedback closed loop system has a plant with the transfer function
G (s) = s + 21s + 2 and a controller Gc (s) in the
2

feed forward path. For a unit set input, the transfer function of the controller that
gives minimum steady state error is
(A) Gc (s) = s + 1 (B) Gc (s) = s + 2
s+2 s+1
(s + 1) (s + 4)
(C) Gc (s) = (D) Gc (s) = 1 + 2 + 3s
(s + 2) (s + 3) s
SOL 1.44 Steady state error is given as
sR (s)
eSS = lim
s " 0 1 + G (s) GC (s)

R (s) = 1 (unit step unit)


s
eSS = lim 1
s " 0 1 + G (s) GC (s)

= lim 1
s"0 GC (s)
1+ 2
s + 2s + 2
eSS will be minimum if lim GC (s) is maximum
s"0
In option (D)
lim GC (s) = lim 1 + 2 + 3s = 3
s"0 s"0 s
So, eSS = lim 1 = 0 (minimum)
s"0 3

Hence (D) is correct option.


MCQ 1.45 X (t) is a stationary process with the power spectral density Sx (f ) > 0 , for all f .
The process is passed through a system shown below

Let Sy (f ) be the power spectral density of Y (t). Which one of the following
statements is correct
(A) Sy (f ) > 0 for all f
(B) Sy (f ) = 0 for f > 1 kHz
(C) Sy (f ) = 0 for f = nf0, f0 = 2 kHz kHz, n any integer

Brought to you by: Nodia and Company Visit us at: www.nodia.co.in


PUBLISHING FOR GATE
Page 29 GATE EC 2010 www.gatehelp.com

(D) Sy (f ) = 0 for f = (2n + 1) f0 = 1 kHz , n any integer


SOL 1.45 For the given system, output is written as
y (t) = d [x (t) + x (t − 0.5)]
dt
dx (t) dx (t − 0.5)
y (t) = +
dt dt
Taking laplace on both sides of above equation
Y (s) = sX (s) + se−0.5s X (s)
Y (s)
H (s) = = s (1 + e−0.5s)
X (s)
H (f ) = jf (1 + e−0.5 # 2πf ) = jf (1 + e− πf )
Power spectral density of output
SY (f ) = H (f ) 2 SX (f ) = f 2 (1 + e− πf ) 2 SX (f )

For SY (f ) = 0 , 1 + e− πf = 0
f = (2n + 1) f0
or f0 = 1 KHz
Hence (D) is correct option.
MCQ 1.46 A plane wave having the electric field components Evi = 24 cos ^3 # 108 − βy h atx
V/m and traveling in free space is incident normally on a lossless medium with
μ = μ0 and ε = 9ε0 which occupies the region y $ 0 . The reflected magnetic field
component is given by
(A) 1 cos (3 # 108 t + y) atx A/m
10π

(B) 1 cos (3 108 t + y) at A/m


20π # x

(C) − 1 cos (3 # 108 t + y) atx A/m


20π

(D) − 1 cos (3 # 108 t + y) atx A/m


10π
SOL 1.46 In the given problem

Reflection coefficient
η2 − η 1
τ = = 400π − 120π =− 1
η2 + η 1 40π + 120π 2

Brought to you by: Nodia and Company Visit us at: www.nodia.co.in


PUBLISHING FOR GATE
Page 30 GATE EC 2010 www.gatehelp.com

τ is negative So magnetic field component does not change its direction Direction
of incident magnetic field
atE # atH = atK
atZ # atH = aty
atH = atx ( + x direction)
So, reflection magnetic field component
Hr = τ # 24 cos (3 # 108 + βy) atx , y $ 0
η
= 1 # 24 cos (3 # 108 + βy) atx , y $ 0
2 # 120π

β = ω = 3 # 108 = 1
8

vC 3 # 10
So, Hr = 1 cos (3 # 108 + y) atx , y $ 0
10π
Hence (A) is correct option.
MCQ 1.47 In the circuit shown, all the transmission line sections are lossless. The Voltage
Standing Wave Ration(VSWR) on the 60 Ω line is

(A) 1.00 (B) 1.64


(C) 2.50 (D) 3.00
SOL 1.47 For length of λ/4 transmission line
Z + jZo tan βl
Z in = Zo ; L
Zo + jZL tan βl E
ZL = 30 Ω , Zo = 30 Ω, β = 2π , l = λ
λ 4
So, tan βl = tan b 2π : λ l = 3
λ 4
R ZL V
S tan βl + jZo W 2
Z in = Zo S W = Z 0 = 60 Ω
S Zo + jZL W ZL
S tan βl W
T X
For length of λ/8 transmission line
Z + jZo tan βl
Z in = Zo ; L
Zo + jZL tan βl E

Brought to you by: Nodia and Company Visit us at: www.nodia.co.in


PUBLISHING FOR GATE
Page 31 GATE EC 2010 www.gatehelp.com

Zo = 30 Ω, ZL = 0 (short)
tan βl = tan b 2π : λ l = 1
λ 8
Z in = jZo tan βl = 30j
Circuit is shown below.

Reflection coefficient
60 + 3j − 60
τ = ZL − Zo = = 1
ZL + Zo 60 + 3j + 60 17
1+ τ
VSWR = = 1 + 17 = 1.64
1− τ 1 − 17
Hence (B) is correct option.

Common Data Questions: 48 & 49 :


Consider the common emitter amplifier shown below with the following circuit
parameters:
β = 100, gm = 0.3861 A/V, r0 = 259 Ω, RS = 1 kΩ, RB = 93 kΩ,
RC = 250 kΩ, RL = 1 kΩ, C1 = 3 and C2 = 4.7 μF

MCQ 1.48 The resistance seen by the source vS is


(A) 258 Ω (B) 1258 Ω
(C) 93 kΩ (D) 3
SOL 1.48 By small signal equivalent circuit analysis

Brought to you by: Nodia and Company Visit us at: www.nodia.co.in


PUBLISHING FOR GATE
Page 32 GATE EC 2010 www.gatehelp.com

Input resistance seen by source vs


R in = vs = Rs + Rs || rs
is
= (1000 Ω) + (93 kΩ || 259 Ω) = 1258 Ω
Hence (B) is correct option.
MCQ 1.49 The lower cut-off frequency due to C2 is
(A) 33.9 Hz (B) 27.1 Hz
(C) 13.6 Hz (D) 16.9 Hz
SOL 1.49 Cut-off frequency due to C2
fo = 1
2π (RC + RL) C2
fo = 1 = 271 Hz
2 # 3.14 # 1250 # 4.7 # 10−6
Lower cut-off frequency
f
fL . o = 271 = 27.1 Hz
10 10
Hence (B) is correct option.

Common Data Question : 50 & 51 :


The signal flow graph of a system is shown below:

MCQ 1.50 The state variable representation of the system can be


1 1 0 −1 1 0
xo = > H x +> Hu
(A) x = >− 1 0H x + >2H u
o
(B) −1 0 2
yo = [0 0.5] x yo = 80 0.5B x

1 1 0 −1 1 0
xo = > H x +> Hu xo = > H x +> Hu
(C) −1 0 2 (D) −1 0 2
yo = 80.5 0.5B x yo = 80.5 0.5B x

Brought to you by: Nodia and Company Visit us at: www.nodia.co.in


PUBLISHING FOR GATE
Page 33 GATE EC 2010 www.gatehelp.com

SOL 1.50 Assign output of each integrator by a state variable

xo1 =− x1 + x2
xo2 =− x1 + 2u
y = 0.5x1 + 0.5x2
State variable representation
−1 1 0
xo = > H x + > Hu
−1 0 2
yo = [0.5 0.5] x
Hence (D) is correct option.
MCQ 1.51 The transfer function of the system is
(A) s2+ 1 (B) s2− 1
s +1 s +1

(C) s+1 (D) s−1


s +s+1
2
s +s+1
2

SOL 1.51 By masson’s gain formula

Transfer function
H (s) =
Y (s)
=
/ PK ΔK
U (s) Δ
Forward path given
P1 (abcdef ) = 2 # 1 # 1 # 0.5 = 12
s s s
P2 (abcdef ) = 2 # 1 # 1 # 0.5
3
Loop gain L1 (cdc) =− 1
s
L2 (bcdb) = 1 # 1 # − 1 = −21
s s s
Δ = 1 − [L1 + L2] = 1 − :− 1 − 12 D = 1 + 1 + 12
s s s s
Δ1 = 1, Δ2 = 2

Brought to you by: Nodia and Company Visit us at: www.nodia.co.in


PUBLISHING FOR GATE
Page 34 GATE EC 2010 www.gatehelp.com

= P1 Δ 1 + P2 Δ 2
Y (s)
So, H (s) =
U (s) Δ
1 :1+1:1
2 s (1 + s)
=s = 2
1
1+ + 2 1 (s + s + 1)
s s
Hence (C) is correct option.

Linked Answer Questions: Q. 52 to Q. 55

Statements for Linked Answer Question : 52 & 53 :


The silicon sample with unit cross-sectional area shown below is in thermal
equilibrium. The following information is given: T = 300 K electronic charge
= 1.6 # 10−19 C , thermal voltage = 26 mV and electron mobility = 1350 cm2 / V-s

MCQ 1.52 The magnitude of the electric field at x = 0.5 μm is


(A) 1 kV/cm (B) 5 kV/cm
(C) 10 kV/cm (D) 26 kV/cm
SOL 1.52 Sample is in thermal equilibrium so, electric field

E = 1 = 10 kV/cm
1 μm

Hence (C) is correct option.


MCQ 1.53 The magnitude of the electron of the electron drift current density at x = 0.5 μm is
(A) 2.16 # 10 4 A/cm2 (B) 1.08 # 10 4 A/m2
(C) 4.32 # 103 A/cm2 (D) 6.48 # 102 A/cm2
SOL 1.53 Electron drift current density
Jd = ND μn eE
= 1016 # 1350 # 1.6 # 10−19 # 10 # 1013
= 2.16 # 10 4 A/cm2
Hence (A) is correct option.

Brought to you by: Nodia and Company Visit us at: www.nodia.co.in


PUBLISHING FOR GATE
Page 35 GATE EC 2010 www.gatehelp.com

Statement for linked Answer Question : 54 & 55 :


Consider a baseband binary PAM receiver shown below. The additive channel noise
n (t) is with power spectral density Sn (f ) = N 0 /2 = 10−20 W/Hz . The low-pass filter
is ideal with unity gain and cut-off frequency 1 MHz. Let Yk represent the random
variable y (tk ).
Yk = Nk , if transmitted bit bk = 0
Yk = a + Nk if transmitted bit bk = 1
Where Nk represents the noise sample value. The noise sample has a probability
density function, PNk (n) = 0.5αe− α n (This has mean zero and variance 2/α 2 ).
Assume transmitted bits to be equiprobable and threshold z is set to a/2 = 10−6 V .

MCQ 1.54 The value of the parameter α (in V − 1 ) is


(A) 1010 (B) 107
(C) 1.414 # 10−10 (D) 2 # 10−20
SOL 1.54 Let response of LPF filters
1, f < 1 MHz
H (f ) = *
0, elsewhere
Noise variance (power) is given as
2
H (f ) No df = 22 (given)
fo
P = σ2 = #0
α
2 # 10−20 df = 22
1 # 106
#0
α
2 # 10−20 # 106 = 22
α
α = 1014
2

or α = 107
Hence (B) is correct option.
MCQ 1.55 The probability of bit error is
(A) 0.5 # e−3.5 (B) 0.5 # e−5
(C) 0.5 # e−7 (D) 0.5 # e−10
SOL 1.55 Probability of error is given by

Brought to you by: Nodia and Company Visit us at: www.nodia.co.in


PUBLISHING FOR GATE
Page 36 GATE EC 2010 www.gatehelp.com

Pe = 1 [P (0/1) + P (1/0)]
2
α/2
P (0/1) = #− 3 0.5e− α n − a dn = 0.5e−10
where a = 2 # 10−6 V and α = 107 V − 1

P (1/0) = #a/32 0.5e− α n dn = 0.5e−10

Pe = 0.5e−10
Hence (D) is correct option.

Q. No. 56 - 60 Carry One Mark Each :

MCQ 1.56 Which of the following options is closest in meaning to the world below:
(A) Cyclic (B) Indirect
(C) Confusing (D) Crooked
SOL 1.56 Circuitous means round about or not direct. Indirect is closest in meaning to this
circuitous
(A) Cyclic : Recurring in nature
(B) Indirect : Not direct
(C) Confusing : lacking clarity of meaning
(D) Crooked : set at an angle; not straight
Hence (B) is correct option.
MCQ 1.57 The question below consists of a pair of related words followed by four pairs of
words. Select the pair that best expresses the relation in the original pair.
Unemployed: Worker
(A) fallow : land (B) unaware: sleeper
(C) wit : jester (D) renovated : house
SOL 1.57 A worker may by unemployed. Like in same relation a sleeper may be unaware.
Hence (B) is correct option.
MCQ 1.58 Choose the most appropriate word from the options given below to complete the
following sentence;
If we manage to ____ our natural resources, we would leave a better planet for
our children.
(A) uphold (B) restrain
(C) Cherish (D) conserve
SOL 1.58 Here conserve is most appropriate word.
Hence (D) is correct option.

Brought to you by: Nodia and Company Visit us at: www.nodia.co.in


PUBLISHING FOR GATE
Page 37 GATE EC 2010 www.gatehelp.com

MCQ 1.59 Choose the most appropriate word from the options given below to complete the
following sentence:
His rather casual remarks on politics ___ his lack of seriousness about the subject
(A) masked (B) belled
(C) betrayed (D) suppressed
SOL 1.59 Betrayed means reveal unintentionally that is most appropriate.
Hence (C) is correct option.
MCQ 1.60 25 persons are in a room, 15 of them play hockey, 17 of them football and 10 of
them play both hockey and football. Then the number of persons playing neither
hockey nor football is ;
(A) 2 (B) 17
(C) 13 (D) 3
SOL 1.60 Hence (D) is correct option.
Number of people who play hockey n (A) = 15
Number of people who play football n (B) = 17
Persons who play both hockey and football n (A + B) = 10
Persons who play either hockey or football or both :
n (A , B) = n (A) + n (B) − n (A + B)
= 15 + 17 − 10 = 22
Thus people who play neither hockey nor football = 25 − 22 = 3

Q. No. 61-65 Carry Two Marks Each

MCQ 1.61 Modern warfare has changed from large scale clashes of armies to suppression of
civilian populations. Chemical agents that do their work silently appear to be
suited to such warfare; and regretfully, there exist people in military establishments
who think that chemical agents are useful tools for their cause.
Which of the following statements best sums up the meaning of the above passage :
(A) Modern warfare has resulted in civil strife.
(B) Chemical agents are useful in modern warfare.
(C) Use of chemical agents in warfare would be undesirable
(D) People in military establishment like to use agents in war
SOL 1.61 Hence (D) is correct option.
MCQ 1.62 If 137 + 276 = 435 how much is 731 + 672 ?
(A) 534 (B) 1403
(C) 1623 (D) 1513
SOL 1.62 Since 7 + 6 = 13 but unit digit is 5 so base may be 8 as 5 is the remainder when 13

Brought to you by: Nodia and Company Visit us at: www.nodia.co.in


PUBLISHING FOR GATE
Page 38 GATE EC 2010 www.gatehelp.com

is divided by 8. Let us check.


137 8 731 8
276 8 672 8
435 Thus here base is 8. Now 1623
Hence (C) is correct option.
MCQ 1.63 5 skilled workers can build a wall in 20 days; 8 semi-killed worker can build a wall
in 25 days; 10 unskilled workers can build a wall in 30 days. If a team has 2 killed,
6 semi-killed and 5 unskilled workers, how long will it take to build the wall
(A) 20 days (B) 18 days
(C) 16 days (D) 15 days
SOL 1.63 Hence (D) is correct option.
Let W be the total work.
Per day work of 5 skilled workers =W
20
Per day work of one skill worker = W =W
5 # 20 100
Similarly per day work of 1 semi-skilled workers = W = W
8 # 25 200
Similarly per day work of one semi-skill worker = W = W
10 # 30 300
Thus total per day work of 2 skilled, 6 semi-skilled and 5 unskilled workers is
= 2W + 6W + 5W = 12W + 18W + 10W = W
100 200 300 600 15
Therefore time to complete the work is 15 days.
MCQ 1.64 Given digits 2, 2, 3, 3, 4, 4, 4 how many distinct 4 digit numbers greater than 3000
can be formed
(A) 50 (B) 51
(C) 52 (D) 54
SOL 1.64 As the number must be greater than 3000, it must be start with 3 or 4. Thus we
have two case:
Case (1) If left most digit is 3 an other three digits are any of 2, 2, 3, 3, 4, 4, 4, 4.
(1) Using 2, 2, 3 we have 3223, 3232, 3322 i.e. 3! = 3 no.
2!
(2) Using 2, 2, 4 we have 3224, 3242, 3422 i.e. 3! = 3 no.
2!
(3) Using 2, 3, 3 we have 3233, 3323, 3332 i.e. 3! = 3 no.
2!
(4) Using 2, 3, 4 we have 3! = 6 no.
(5) Using 2, 4, 4 we have 3244, 3424, 3442 i.e. 3! = 3 no.
2!

Brought to you by: Nodia and Company Visit us at: www.nodia.co.in


PUBLISHING FOR GATE
Page 39 GATE EC 2010 www.gatehelp.com

(6) Using 3, 3, 4 we have 3334, 3343, 3433 i.e. 3! = 3 no.


2!

(7) Using 3, 4, 4 we have 3344, 3434, 3443 i.e. 3! = 3 no.


2!

(8) Using 4, 4, 4 we have 3444 i.e. 3! = 1 no.


3!
Total 4 digit numbers in this case is
1 + 3 + 3 + 3 + 6 + 3 + 3 + 3 + 1 = 25
Case 2 : If left most is 4 and other three digits are any of 2, 2, 3, 3, 3, 4, 4, 4.
(1) Using 2, 2, 3 we have 4223, 4232, 4322 i.e. . 3! = 3 no
2!

(2) Using 2, 2, 4 we have 4224, 4242, 4422 i.e. . 3! = 3 no


2!

(3) Using 2, 3, 3 we have 4233, 4323, 4332 i.e. . 3! = 3 no


2!
(4) Using 2, 3, 4 we have i.e. . 3! = 6 no
(5) Using 2, 4, 4 we have 4244, 4424, 4442 i.e. . 3! = 3 no
2!

(6) Using 3, 3, 3 we have 4333 i.e 3! = 1. no.


3!

(7) Using 3, 3, 4 we have 4334, 4343, 4433 i.e. . 3! = 3 no


2!

(8) Using 3, 4, 4 we have 4344, 4434, 4443 i.e. . 3! = 3 no


2!

(9) Using 4, 4, 4 we have 4444 i.e. 3! = 1. no


3!
Total 4 digit numbers in 2nd case = 3 + 3 + 3 + 6 + 3 + 3 + 1 + 3 + 1 = 26
Thus total 4 digit numbers using case (1) and case (2) is = 25 + 26 = 51
Hence (B) is correct option.
MCQ 1.65 Hari(H), Gita(G), Irfan(I) and Saira(S) are sibilings (i.e. brothers and sisters). All
were born on Ist January. The age difference between any two successive siblings
(that is born one after another) is less than 3 years. Given the following facts:
(i) Hari’s age + Gita’s age > Irfan’s age + Saira’s age
(ii) The age difference between Gita and Saira is 1 year. However, Gita is not the
oldest and Saira is not the youngest
(iii) There are not twins.
In what order were they born (oldest first)
(A) HSIG (B) SGHI

Brought to you by: Nodia and Company Visit us at: www.nodia.co.in


PUBLISHING FOR GATE
Page 40 GATE EC 2010 www.gatehelp.com

(C) IGSH (D) IHSG


SOL 1.65 Let H , G , S and I be ages of Hari, Gita, Saira and Irfan respectively.
Now from statement (1) we have H + G > I + S
Form statement (2) we get that G − S = 1 or S − G = 1
As G can’t be oldest and S can’t be youngest thus either GS or SG possible.
From statement (3) we get that there are no twins
(A) HSIG : There is I between S and G which is not possible
(B) SGHI : SG order is also here and S > G > H > I and G + H > S + I which is
possible.
(C) IGSH : This gives I > G and S > H and adding these both inequalities we
have I + S > H + G which is not possible.
(D) IHSG : This gives I > H and S > G and adding these both inequalities we
have I + S > H + G which is not possible.
Hence (B) is correct option.

Answer Sheet
1. (C) 13. (B) 25. (C) 37. (D) 49. (B) 61. (D)
2. (C) 14. (A) 26. (A) 38. (B) 50. (D) 62. (C)
3. (D) 15. (C) 27. (D) 39. (D) 51. (C) 63. (D)
4. (A) 16. (D) 28. (C) 40. (C) 52. (C) 64. (B)
5. (D) 17. (B) 29. (C) 41. (B) 53. (A) 65. (B)
6. (A) 18. (B) 30. (B) 42. (C) 54. (B)
7. (B) 19. (A) 31. (D) 43. (C) 55. (D)
8. (B) 20. (C) 32. (A) 44. (D) 56. (B)
9. (A) 21. (B) 33. (A) 45. (D) 57. (B)
10. (A) 22. (C) 34. (A) 46. (A) 58. (D)
11. (D) 23. (D) 35. (B) 47. (B) 59. (C)
12. (*) 24. (C) 36. (C) 48. (B) 60. (D)

Brought to you by: Nodia and Company Visit us at: www.nodia.co.in


PUBLISHING FOR GATE
GATE EC
2011

Q. No. 1 – 25 Carry One Mark Each

MCQ 1.1 Consider the following statements regarding the complex Poynting vector Pv for the
power radiated by a point source in an infinite homogeneous and lossless medium.
Re(Pv ) denotes the real part of Pv.S denotes a spherical surface whose centre is
at the point source, and nt denotes the unit surface normal on S. Which of the
following statements is TRUE?
(A) Re(Pv ) remains constant at any radial distance from the source
(B) Re(Pv ) increases with increasing radial distance from the source
(C) ## Re (Pv) .nt dS remains constant at any radial distance from the source
S

(D) ## Re (Pv) .nt dS decreases with increasing radial distance form the source
S
SOL 1.1 Power radiated from any source is constant.
Hence (C) is correct option..
MCQ 1.2 A transmission line of characteristic impedance 50 Ω is terminated by a 50 Ω
load. When excited by a sinusoidal voltage source at 10 GHz, the phase difference
between two points spaced 2 mm apart on the line is found to be π radians. The
4
phase velocity of the wave along the line is
(A) 0.8 # 108 m/s (B) 1.2 # 108 m/s

(C) 1.6 # 108 m/s (D) 3 # 108 m/s


SOL 1.2 We have d = 2 mm and f = 10 GHz
Phase difference = 2π d = π ;
λ 4
or = λ = 8d = 8 # 2 mm = 16 mm
v = fλ = 10 # 109 # 16 # 10−3
= 1.6 # 108 m/ sec
Hence (C) is correct option.
MCQ 1.3 An analog signal is band-limited to 4 kHz, sampled at the Nyquist rate and

Brought to you by: Nodia and Company Visit us at: www.nodia.co.in


PUBLISHING FOR GATE
Page 2 GATE EC 2011 www.gatehelp.com

the samples are quantized into 4 levels. The quantized levels are assumed to be
independent and equally probable. If we transmit two quantized samples per second,
the information rate is ________ bits / second.
(A) 1 (B) 2
(C) 3 (D) 4
SOL 1.3 Quantized 4 level require 2 bit representation i.e. for one sample 2 bit are required.
Since 2 sample per second are transmitted we require 4 bit to be transmitted per
second.
Hence (D) is correct option.
MCQ 1.4 The root locus plot for a system is given below. The open loop transfer function
corresponding to this plot is given by

s (s + 1)
(A) G(s) H(s) = k
(s + 2) (s + 3)
(s + 1)
(B) G(s) H(s) = k
s (s + 2) (s + 3) 2

(C) G(s) H(s) = k 1


s (s − 1) (s + 2) (s + 3)
(s + 1)
(D) G(s) H(s) = k
s (s + 2) (s + 3)
SOL 1.4 For given plot root locus exists from − 3 to 3, So there must be odd number of
poles and zeros. There is a double pole at s =− 3
Now poles = 0, − 2, − 3, − 3
zeros =− 1
k (s + 1)
Thus transfer function G (s) H (s) =
s (s + 2) (s + 3) 2
Hence (B) is correct option.
MCQ 1.5 A system is defined by its impulse response h (n) = 2n u (n − 2). The system is
(A) stable and causal (B) causal but not stable
(C) stable but not causal (D) unstable and non-causal

Brought to you by: Nodia and Company Visit us at: www.nodia.co.in


PUBLISHING FOR GATE
Page 3 GATE EC 2011 www.gatehelp.com

SOL 1.5 Function h (n) = an u (n) stable if a < 1 and Unstable if a H 1


We have h (n) = 2n u (n − 2);
Here a = 2 therefore h (n) is unstable and since h (n) = 0 for n < 0
Therefore h (n) will be causal. So h (n) is causal and not stable.
Hence (B) is correct option.
MCQ 1.6 If the unit step response of a network is (1 − e− αt), then its unit impulse response is
(A) αe− αt (B) α−1 e− αt
(C) (1 − α−1) eαt (D) (1 − α) e− αt
SOL 1.6 Hence (A) is correct option.
Impulse response = d (step response)
dt
= d (1 − e− αt)
dt
= 0 + αe− αt = αe− αt

MCQ 1.7 The output Y in the circuit below is always ‘1’ when

(A) two or more of the inputs P, Q, R are ‘0’


(B) two or more of the inputs P, Q, R are ‘1’
(C) any odd number of the inputs P, Q, R is ‘0’
(D) any odd number of the inputs P, Q, R is ‘1’
SOL 1.7 The given circuit is shown below:

(PQ QR ) PR = (PQ + QR PR )
= PQ + QR + PR
= PQ + QR + PR

Brought to you by: Nodia and Company Visit us at: www.nodia.co.in


PUBLISHING FOR GATE
Page 4 GATE EC 2011 www.gatehelp.com

If any two or more inputs are ‘1’ then output y will be 1.


Hence (B) is correct option.
MCQ 1.8 In the circuit shown below, capacitors C1 and C2 are very large and are shorts at
the input frequency. vi is a small signal input. The gain magnitude vo at 10 M
vi
rad/s is

(A) maximum (B) minimum


(C) unity (D) zero
SOL 1.8 For the parallel RLC circuit resonance frequency is,
ωr = 1 = 1 = 10 M rad/s
LC 10 # 10 # 1 # 10−9
−6

Thus given frequency is resonance frequency and parallel RLC circuit has maximum
impedance at resonance frequency
Gain of the amplifier is gm # (ZC RL) where ZC is impedance of parallel RLC
circuit.
At ω = ωr , ZC = R = 2 kΩ = ZC max .
Hence at this frequency (ωr ), gain is
Gain ω = ω = gm (ZC RL) = gm (2k 2k) = gm # 103 which is maximum. Therefore
r

gain is maximum at ωr = 10 M rad/ sec .


Hence (A) is correct option.
MCQ 1.9 Drift current in the semiconductors depends upon
(A) only the electric field
(B) only the carrier concentration gradient
(C) both the electric field and the carrier concentration
(D) both the electric field and the carrier concentration gradient
SOL 1.9 Hence (C) is correct option.
Drift current Id = qnμn E
It depends upon Electric field E and carrier concentration n

Brought to you by: Nodia and Company Visit us at: www.nodia.co.in


PUBLISHING FOR GATE
Page 5 GATE EC 2011 www.gatehelp.com

MCQ 1.10 A Zener diode, when used in voltage stabilization circuits, is biased in
(A) reverse bias region below the breakdown voltage
(B) reverse breakdown region
(C) forward bias region
(D) forward bias constant current mode
SOL 1.10 Zener diode operates in reverse breakdown region.

Hence (B) is correct option.


MCQ 1.11 The circuit shown below is driven by a sinusoidal input vi = v p cos (t/RC ) . The
steady state output vo

(A) (v p /3) cos (t/RC ) (B) (v p /3) sin (t/RC )


(C) (v p /2) cos (t/RC ) (D) (v p /2) sin (t/RC )
SOL 1.11 The given circuit is shown below

For parallel combination of R and C equivalent impedance is


R$ 1
jω C R
Zp = =
R+ 1 1 + j ωRC
jω C
Transfer function can be written as

Brought to you by: Nodia and Company Visit us at: www.nodia.co.in


PUBLISHING FOR GATE
Page 6 GATE EC 2011 www.gatehelp.com

R
Vout = Z p 1 + jωRC
=
Vin Zs + Zp R+ 1 + R
jωC 1 + jωRC
jωRC
=
jωRC + (1 + jωRC) 2
j
= Here ω = 1
j + (1 + j) 2 RC
Vout = j
=1
Vin (1 + j) + j
2 3

v out = b p l cos (t/RC)


V
Thus
3
Hence (A) is correct option.
MCQ 1.12 Consider a closed surface S surrounding volume V. If rv is the position vector of a
point inside S, with nt the unit normal on S, the value of the integral ## 5rt.nv dS is
(A) 3V (B) 5V
(C) 10V (D) 15V
SOL 1.12 From Divergence theorem, we have
v = #A
v Adv
### 4$ v $ nt ds
s
The position vector
rv = ^utx x + uty y + utz z h
v
Here, A = 5rv, thus
c 2x 2z m ^ h
4$ A v = utx 2 + uty 2 + utz 2 : utx x + uty y + utz z
2y
= c dx +
dx dy dz m
dy dz
+ 5 = 3 # 5 = 15

So, ##s 5rv $ nt ds = ### 15 dv = 15V


Hence (D) is correct option
TE
MCQ 1.13 The modes in a rectangular waveguide are denoted by TM where mn
mn

m and n are the eigen numbers along the larger and smaller dimensions of the
waveguide respectively. Which one of the following statements is TRUE?
(A) The TM 10 mode of the wave does not exist
(B) The TE 10 mode of the wave does not exist
(C) The TM 10 and TE 10 the modes both exist and have the same cut-off frequencies
(D) The TM 10 and TM 01 modes both exist and have the same cut-off frequencies
SOL 1.13 TM11 is the lowest order mode of all the TMmn modes.
Hence (A) is correct option.
dy
MCQ 1.14 The solution of the differential equation = ky, y (0) = c is
dx
Brought to you by: Nodia and Company Visit us at: www.nodia.co.in
PUBLISHING FOR GATE
Page 7 GATE EC 2011 www.gatehelp.com

(A) x = ce−ky (B) x = kecy

(C) y = cekx (D) y = ce−kx


SOL 1.14 Hence (C) is correct answer
dy
We have = ky
dx

Integrating # dy
y
= # k dx + A

or ln y = kx + A
Since y (0) = c thus ln c = A
So, we get, ln y = kx + ln c
or ln y = ln ekx + ln c
or y = cekx
MCQ 1.15 The Column-I lists the attributes and the Column-II lists the modulation systems.
Match the attribute to the modulation system that best meets it
Column-I
P. Power efficient transmission of signals
Q. Most bandwidth efficient transmission of voice signals
R. Simplest receiver structure
S. Bandwidth efficient transmission of signals with Significant dc component
Column-II
1. Conventional AM
2. FM
3. VSB
4. SSB-SC
(A) P-4;Q-2;R-1;S-3 (B) P-2;Q-4;R-1;S-3
(C) P-3;Q-2;R-1;S-4 (D) P-2;Q-4;R-3;S-1
SOL 1.15 In FM the amplitude is constant and power is efficient transmitted. No variation
in power.
There is most bandwidth efficient transmission in SSB- SC. because we transmit
only one side band.
Simple Diode in Non linear region ( Square law ) is used in conventional AM that
is simplest receiver structure.
In VSB dc. component exists.
Hence (B) is correct option.
d 2y dy
MCQ 1.16 The differential equation 100 − 20 + y = x (t) describes a system with an in
dt 2 dt

Brought to you by: Nodia and Company Visit us at: www.nodia.co.in


PUBLISHING FOR GATE
Page 8 GATE EC 2011 www.gatehelp.com

put x (t) and an output y (t). The system, which is initially relaxed, is excited by a
unit step input. The output y(t) can be represented by the waveform

SOL 1.16 Hence (A) is correct option.


d2 y dy
We have 100 2 − 20 + y = x (t)
dt dt

Applying Laplace transform we get


100s2 Y (s) − 20sY (s) + Y (s) = X (s)

Y (s) 1
or H (s) = =
X (s) 100s2 − 20s + 1
1/100 A
= 2 =
s − (1/5) s + 1/100 s2 + 2ξωn s + ω2
Here ωn = 1/10 and 2ξωn =− 1/5 giving ξ =− 1
Roots are s = 1/10, 1/10 which lie on Right side of s plane thus unstable.
MCQ 1.17 For the transfer function G (jω) = 5 + jω , the corresponding Nyquist plot for
positive frequency has the form

Brought to you by: Nodia and Company Visit us at: www.nodia.co.in


PUBLISHING FOR GATE
Page 9 GATE EC 2011 www.gatehelp.com

SOL 1.17 We have G (jω) = 5 + jω


Here σ = 5 . Thus G (jω) is a straight line parallel to jω axis.
Hence (A) is correct option.
MCQ 1.18 The trigonometric Fourier series of an even function does not have the
(A) dc term (B) cosine terms
(C) sine terms (D) odd harmonic terms
SOL 1.18 For an even function Fourier series contains dc term and cosine term (even and odd
harmonics).
Hence (C) is correct option.
MCQ 1.19 When the output Y in the circuit below is ‘1’, it implies that data has

(A) changed from 0 to 1 (B) changed from 1 to 0


(C) changed in either direction (D) not changed
SOL 1.19 For the output to be high, both inputs to AND gate should be high.
The D-Flip Flop output is the same, after a delay.
Let initial input be 0; (Consider Option A)
then Q = 1 (For 1 D-Flip Flop). This is given as input to 2 FF.
st nd

Let the second input be 1. Now, considering after 1 time interval; The output of 1st
Flip Flop is 1 and 2nd FF is also 1. Thus Output = 1.
Hence (A) is correct option.
MCQ 1.20 The logic function implemented by the circuit below is (ground implies logic 0)

Brought to you by: Nodia and Company Visit us at: www.nodia.co.in


PUBLISHING FOR GATE
Page 10 GATE EC 2011 www.gatehelp.com

(A) F = AND ^P, Q h (B) F = OR ^P, Q h


(C) F = X NOR ^P, Q h (D) F = X OR ^P, Q h
SOL 1.20 Hence (D) is correct option.
F = S1 S 0 I 0 + S1 S 0 I1 + S1 S 0 I 2 + S1 S 0 I 3
I0 = I3 = 0
F = PQ + PQ = XOR (P, Q) ( S1 = P, S 0 = Q )
MCQ 1.21 The circuit below implements a filter between the input current ii and the output
voltage vo . Assume that the opamp is ideal. The filter implemented is a

(A) low pass filter (B) band pass filter


(C) band stop filter (D) high pass filter
SOL 1.21 The given circuit is shown below :

From diagram we can write


Ii = Vo + Vo
R1 sL1

Brought to you by: Nodia and Company Visit us at: www.nodia.co.in


PUBLISHING FOR GATE
Page 11 GATE EC 2011 www.gatehelp.com

Transfer function
H (s) = Vo = sR1 L1
I1 R1 + sL1
jω R 1 L 1
or H (jω) =
R 1 + jω L 1
At ω = 0
H (jω) = 0
At ω = 3
H (jω) = R1 = constant . Hence HPF.
Hence (D) is correct option.
MCQ 1.22 A silicon PN junction is forward biased with a constant current at room temperature.
When the temperature is increased by 10ºC, the forward bias voltage across the
PN junction
(A) increases by 60 mV (B) decreases by 60 mV
(C) increases by 25 mV (D) decreases by 25 mV
SOL 1.22 For every 1c C increase in temperature, forward bias voltage across diode decreases
by 2.5 mV. Thus for 10c C increase, there us 25 mV decreases.
Hence (D) is correct option.
MCQ 1.23 In the circuit shown below, the Norton equivalent current in amperes with respect
to the terminals P and Q is

(A) 6.4 − j 4.8 (B) 6.56 − j 7.87


(C) 10 + j 0 (D) 16 + j 0
SOL 1.23 Replacing P − Q by short circuit as shown below we have

Using current divider rule the current Isc is

Brought to you by: Nodia and Company Visit us at: www.nodia.co.in


PUBLISHING FOR GATE
Page 12 GATE EC 2011 www.gatehelp.com

ISC = 25 (16 0 ) = (6.4 − j4.8) A


25 + 15 + j30
Hence (A) is correct option.
MCQ 1.24 In the circuit shown below, the value of RL such that the power transferred to RL
is maximum is

(A) 5 Ω (B) 10 Ω
(C) 15 Ω (D) 20 Ω
SOL 1.24 Power transferred to RL will be maximum when RL is equal to the thevenin
resistance. We determine thevenin resistance by killing all source as follows :

RTH = 10 # 10 + 10 = 15 Ω
10 + 10
Hence (C) is correct option.

MCQ 1.25 The value of the integral # − 3z + 4 dz where c is the circle z = 1 is given by
c
(z 2 + 4z + 5)
(A) 0 (B) 1/10
(C) 4/5 (D) 1

SOL 1.25 C R Integrals is # − 3z + 4 dz where C is circle z = 1


C
z 2 + 4z + 5

C
# f (z) dz = 0 if poles are outside C.

Now z 2 + 4z + 5 = 0

(z + 2) 2 + 1 = 0
Thus z1, 2 =− 2 ! j & z1, 2 > 1
So poles are outside the unit circle.
Hence (A) is correct option.

Brought to you by: Nodia and Company Visit us at: www.nodia.co.in


PUBLISHING FOR GATE
Page 13 GATE EC 2011 www.gatehelp.com

Q. No. 26 – 51 Carry Two Marks Each

MCQ 1.26 A current sheet J = 10uty A/m lies on the dielectric interface x = 0 between two
dielectric media with εr 1 = 5, μr 1 = 1 in Region − 1 (x < 0) and εr 2 = 5, μr 2 = 2 in
Region − 2 (x > 0). If the magnetic field in Region− 1 at x = 0− is Hv1 = 3utx + 30uty A/m
the magnetic field in Region-2 at x = 0+ is

(A) H2 = 1.5utx + 30uty − 10utz A/m


(B) Hv2 = 1.5utx + 30uty − 10utz A/m
(C) Hv2 = 1.5utx + 40uty A/m
(D) Hv2 = 3utx + 30uty + 10utz A/m
SOL 1.26 From boundary condition
Bn1 = Bn2
μ1 Hx1 = μ2 Hx2
or Hx2 = Hx1 = 1.5
2
or Hx2 = 1.5utx
Further if H z = 1.5utx + Auty + Buz
Then from Boundary condition
10ut
(3utx + 30uty) utx = (1.5utx + Auty + Butz ) xt + v y
J
=− 30utz =− Autz + Buty + 10uty
Comparing we get A = 30 and B =− 10
So H z = 1.5utx + 30uty − 10utz A/m
Hence (A) is correct option.
MCQ 1.27 A transmission line of characteristic impedance 50 W is terminated in a load
impedance ZL . The VSWR of the line is measured as 5 and the first of the voltage
maxima in the line is observed at a distance of λ from the load. The value of ZL is
4
(A) 10 Ω (B) 250 Ω
(C) (19.23 + j 46.15) Ω (D) (19.23 − j 46.15) Ω
SOL 1.27 Since voltage maxima is observed at a distance of λ/4 from the load and we know
that the separation between one maxima and minima equals to λ/4 so voltage
minima will be observed at the load, Therefore load can not be complex it must be
pure resistive.
Now Γ = s−1
s+1
Brought to you by: Nodia and Company Visit us at: www.nodia.co.in
PUBLISHING FOR GATE
Page 14 GATE EC 2011 www.gatehelp.com

also RL = R 0 (since voltage maxima is formed at the load)


s
RL = 50 = 10 Ω
5
Hence (A) is correct option.
MCQ 1.28 x (t) is a stationary random process with auto-correlation function. Rx (τ) = exp (πr 2) .
This process is passed through the system shown below. The power spectral density
of the output process y (t) is

(A) (4π 2 f 2 + 1) exp (− πf 2) (B) (4π 2 f 2 − 1) exp (− πf 2)


(C) (4π 2 f 2 + 1) exp (− πf ) (D) (4π 2 f 2 − 1) exp (− πf )
SOL 1.28 Hence (A) is correct option.
We have Sx (f) = F {Rx (τ)} = F {exp (− πτ2)}
2
= e− πf
The given circuit can be simplified as

Power spectral density of output is


Sy (f) = G (f) 2 Sx (f)
2
= j2πf − 1 2 e− πf
= ( (2πf) 2 + 1) 2 e− πf
2

Sy (f) = (4π2 f 2 + 1) e− πf
2
or
MCQ 1.29 The output of a 3-stage Johnson (twisted ring) counter is fed to a digital-to analog
(D/A) converter as shown in the figure below. Assume all the states of the counter
to be unset initially. The waveform which represents the D/A converter output vo is

Brought to you by: Nodia and Company Visit us at: www.nodia.co.in


PUBLISHING FOR GATE
Page 15 GATE EC 2011 www.gatehelp.com

SOL 1.29 All the states of the counter are initially unset.

State Initially are shown below in table :


Q2 Q1 Q0
0 0 0 0
1 0 0 4
1 1 0 6
1 1 1 7
0 1 1 3
0 0 1 1
0 0 0 0
Hence (A) is correct option.

Brought to you by: Nodia and Company Visit us at: www.nodia.co.in


PUBLISHING FOR GATE
Page 16 GATE EC 2011 www.gatehelp.com

MCQ 1.30 Two D flip-flops are connected as a synchronous counter that goes through the
following QB QA sequence 00 " 11 " 01 " 10 " 00 " ....
The combination to the inputs DA and DB are
(A) DA = QB; DB = QA
(B) DA = QA; DB = QB
(C) DA = (QA QB + QA QB); DB = QA
(D) DA = (QA QB + QA QB); DB = QB
SOL 1.30 The sequence is QB QA
00 " 11 " 01 " 10 " 00 " ...
QB QA QB (t + 1) QA (t + 1)
0 0 1 1
1 1 0 1
0 1 1 0
1 0 0 0
QB ^t + 1h

QB ^t + 1h = Q A

DA = Q A Q B + QA QB
Hence (D) is correct option.
MCQ 1.31 In the circuit shown below, for the MOS transistors, μn Cox = 100 μ/A/V 2 and the
threshold voltage VT = 1 V . The voltage Vx at the source of the upper transistor is

(A) 1 V (B) 2 V

Brought to you by: Nodia and Company Visit us at: www.nodia.co.in


PUBLISHING FOR GATE
Page 17 GATE EC 2011 www.gatehelp.com

(C) 3 V (D) 3.67 V


SOL 1.31 Given circuit is shown below.

For transistor M2 ,
VGS = VG − VS = Vx − 0 = Vx

VDS = VD − VS = Vx − 0 = Vx

Since VGS − VT = Vx − 1 < VDS , thus M2 is in saturation.


By assuming M1 to be in saturation we have
IDS (M ) = IDS (M )
1 2

μn C 0x μ C
(4) (5 − Vx − 1) 2 = n 0x 1 (Vx − 1) 2
2 2

4 (4 − Vx ) 2 = (Vx − 1) 2
or 2 (4 − Vx ) = ! (Vx − 1)
Taking positive root,
8 − 2Vx = Vx − 1
Vx = 3 V
At Vx = 3 V for M1,VGS = 5 − 3 = 2 V < VDS . Thus our assumption is true and
Vx = 3 V .
Hence (C) is correct option.
MCQ 1.32 An input x (t) = exp (− 2t) u (t) + δ (t − 6) is applied to an LTI system with impulse
response h (t) = u (t) . The output is
(A) [1 − exp (− 2t)] u (t) + u (t + 6) (B) [1 − exp (− 2t)] u (t) + u (t − 6)
(C) 0.5 [1 − exp (− 2t)] u (t) + u (t + 6) (D) 0.5 [1 − exp (− 2t)] u (t) + u (t − 6)
SOL 1.32 Hence (D) is correct option.
We have x (t) = exp (− 2t) μ (t) + s (t − 6) and h (t) = u (t)
Taking Laplace Transform we get
X (s) = b 1 + e−6s l and H (s) = 1
s+2 s
Now Y (s) = H (s) X (s)
−6s
= 1 : 1 + e−6sD = 1 +e
s s+2 s (s + 2) s

Brought to you by: Nodia and Company Visit us at: www.nodia.co.in


PUBLISHING FOR GATE
Page 18 GATE EC 2011 www.gatehelp.com

−6s
or Y (s) = 1 − 1 +e
2s 2 (s + 2) s
Thus y (t) = 0.5 [1 − exp (− 2t)] u (t) + u (t − 6)
MCQ 1.33 For a BJT the common base current gain α = 0.98 and the collector base junction
reverse bias saturation current Ico = 0.6 μA . This BJT is connected in the common
emitter mode and operated in the active region with a base drive current IB = 204 A
. The collector current Ic for this mode of operation is
(A) 0.98 mA (B) 0.99 mA
(C) 1.0 mA (D) 1.01 mA
SOL 1.33 Hence (D) is correct option.
We have α = 0.98
Now β = α = 4.9
1−α
In active region, for common emitter amplifier,
IC = βIB + (1 + β) ICO ...(1)
Substituting ICO = 0.6 μA and IB = 20 μA in above eq we have,
IC = 1.01 mA
2 (s + 1)
MCQ 1.34 If F (s) = L [f (t)] = then the initial and final values of f (t) are respectively
s2 + 4s + 7
(A) 0, 2 (B) 2, 0
(C) 0, 2/7 (D) 2/7, 0
SOL 1.34 Correct Option is ( )
MCQ 1.35 In the circuit shown below, the current I is equal to

(A) 14 0c A (B) 2.0 0c A


(C) 2.8 0c A (D) 3.2 0c A
SOL 1.35 From star delta conversion we have

Brought to you by: Nodia and Company Visit us at: www.nodia.co.in


PUBLISHING FOR GATE
Page 19 GATE EC 2011 www.gatehelp.com

Thus R1 = Ra Rb = 6.6 = 2Ω
Ra + Rb + Rc 6 + 6 + 6
Here R1 = R 2 = R 3 = 2 Ω
Replacing in circuit we have the circuit shown below :

Now the total impedance of circuit is


(2 + j4) (2 − j4)
Z = +2 = 7Ω
(2 + j4) (2 − j4)
14 0c
Current I = = 2 0c
7
Hence (B) is correct option.
MCQ 1.36 A numerical solution of the equation f (x) + x − 3 = 0 can be obtained using
Newton- Raphson method. If the starting value is x = 2 for the iteration, the value
of x that is to be used in the next step is
(A) 0.306 (B) 0.739
(C) 1.694 (D) 2.306
SOL 1.36 Hence (C) is correct option.
We have f (x) = x + x − 3 = 0
f l (x) = 1 + 1
2 x
Substituting x 0 = 2 we get
f l (x 0) = 1.35355 and f (x 0) = 2 + 2 − 3 = 0.414
Newton Raphson Method
f (x 0)
x1 = x 0 −
f l (x 0)
Substituting all values we have
x 1 = 2 − 0.414 = 1.694
1.3535

MCQ 1.37 The electric and magnetic fields for a TEM wave of frequency 14 GHz in a
homogeneous medium of relative permittivity εr and relative permeability μr = 1
are given by

Brought to you by: Nodia and Company Visit us at: www.nodia.co.in


PUBLISHING FOR GATE
Page 20 GATE EC 2011 www.gatehelp.com

Ev = E p e j (ωt − 280πy) utz V/m Hv = 3e j (ωt − 280πy) utx A/m


Assuming the speed of light in free space to be 3 # 108 m/s , the intrinsic impedance
of free space to be 120π , the relative permittivity εr of the medium and the electric
field amplitude E p are
(A) εr = 3, E p = 120π (B) εr = 3, Eb = 360π
(C) εr = 9, E p = 360π (D) εr = 9, E p = 120π
SOL 1.37 From the expressions of Ev & Hv , we can write,
β = 280 π
or 2 π = 280 π & λ = 1
λ 140
v
Wave impedance, Zw = E = p = 120 π
E
Hv 3 εr
again, f = 14 GHz
8
Now λ = C = 3 # 10 9 = 3
εr f εr 14 # 10 140 εr
or 3 = 1
140 εr 140
or εr = 9
= 120π = E p = 120π
Ep
Now
3 9
Hence (D) is correct option.
MCQ 1.38 A message signal m (t) = cos 200πt + 4 cos πt modulates the carrier c (t) = cos 2πfc t
where fc = 1 MHz to produce an AM signal. For demodulating the generated AM
signal using an envelope detector, the time constant RC of the detector circuit
should satisfy
(A) 0.5 ms < RC < 1 ms (B) 1 μs << RC < 0.5 ms
(C) RC << μs (D) RC >> 0.5 ms
SOL 1.38 Highest frequency component in m (t) is fm = 4000π/2π = 2000 Hz
Carrier frequency fC = 1 MHz
For Envelope detector condition
1/fC << RC << 1/fm
1 μs << RC << 0.5 ms
Hence (B) is correct option.
MCQ 1.39 The block diagram of a system with one input it and two outputs y1 and y2 is given
below.

Brought to you by: Nodia and Company Visit us at: www.nodia.co.in


PUBLISHING FOR GATE
Page 21 GATE EC 2011 www.gatehelp.com

A state space model of the above system in terms of the state vector x and the
output vector y = [y1 y2]T is
(A) xo = [2] x + [1] u ; y = [1 2] x
1
(B) xo = [− 2] x + [1] u; y = > H x
2

−2 0 1
(C) xo = > H x + > H u ; y = 81 2B x
0 −2 1

2 0 1 1
(D) xo = > H x + > Hu ; y = > Hx
0 2 1 2
SOL 1.39 Hence (B) is correct option.
dy
Here x = y1 and xo = 1
dx
y1 x 1
y = > H = > H = > Hx
y2 2x 2
Now y1 = 1 u
s+2
y1 (s + 2) =u
yo1 + 2y1 =u
xo + 2x =u
xo =− 2x + u
xo = [− 2] x + [1] u
Drawing SFG as shown below

Thus xo1 = [− 2] x1 + [1] u


y1 = x1 ; y2 = 2x1

y1 1
y = > H = > H x1
y2 2

Brought to you by: Nodia and Company Visit us at: www.nodia.co.in


PUBLISHING FOR GATE
Page 22 GATE EC 2011 www.gatehelp.com

Here x1 = x
MCQ 1.40 Two systems H1 (Z ) and H2 (Z ) are connected in cascade as shown below. The overall
output y (n) is the same as the input x (n) with a one unit delay. The transfer
function of the second system H2 (Z ) is

1 − 0.6z−1 z−1 (1 − 0.6z−1)


(A) (B)
z (1 − 0.4z−1)
−1
(1 − 0.4z−1)
z−1 (1 − 0.4z−1) 1 − 0.4 z−1
(C) (D)
(1 − 0.6z−1) z (1 − 0.6z−1)
−1

SOL 1.40 Hence (B) is correct option.


y (n) = x (n − 1)
or Y (z) = z−1 X (z)
Y (z)
or = H (z) = z−1
X (z)
Now H1 (z) H2 (z) = z−1
1 − 0.4z−1
c 1 − 0.6z−1 m H2 (z) = z
−1

z−1 (1 − 0.6z−1)
H2 (z) =
(1 − 0.4z−1)

MCQ 1.41 An 8085 assembly language program is given below. Assume that the carry flag is
initially unset. The content of the accumulator after the execution of the program
is

(A) 8 CH (B) 64 H
(C) 23 H (D) 15 H
SOL 1.41 Initially Carry Flag, C = 0
MVI A, 07 H ; A = 0000 0111
RLC ; Rotate left without carry. A = 0000 1110
MVO B, A ; B = A = 0000 1110

Brought to you by: Nodia and Company Visit us at: www.nodia.co.in


PUBLISHING FOR GATE
Page 23 GATE EC 2011 www.gatehelp.com

RLC ; A = 0001 1100


RLC ; A = 0011 1000
ADD B ; A = 0011 1000
; + 0000 1110
; 0100 0110
RRC Rotate Right with out carry, A = 0010 0011
;
Thus A = 23 H
Hence (C) is correct option.
MCQ 1.42 The first six points of the 8-point DFT of a real valued sequence are
5, 1 − j 3, 0, 3 − j 4, 0 and 3 + j 4..... The last two points of the DFT are respectively
(A) 0, 1 − j 3 (B) 0, 1 + j 3
(C) 1 + j 3, 5 (D) 1 − j3, 5
SOL 1.42 For 8 point DFT, x* [1] = x [7]; x* [2] = x [6]; x* [3] = x [5] and it is conjugate symmetric
about x [4], x [6] = 0 ; x [7] = 1 + j3
Hence (B) is correct option.
MCQ 1.43 For the BJT QL in the circuit shown below, β = 3, VBE on = 0.7 V, V = 0.7 V . The
CE sat

switch is initially closed. At time t = 0 , the switch is opened. The time t at which
Q1 leaves the active region is

(A) 10 ms (B) 25 ms
(C) 50 ms (D) 100 ms
SOL 1.43 Hence (C) is correct option
In active region VBEon = 0.7 V
Emitter voltage VE = VB − VBEon =− 5.7 V

VE − (− 10) − 5.7 − (− 10)


Emitter Current IE = = = 1 mA
4.3k 4.3k
Now IC . IE = 1 mA
Applying KCL at collector

Brought to you by: Nodia and Company Visit us at: www.nodia.co.in


PUBLISHING FOR GATE
Page 24 GATE EC 2011 www.gatehelp.com

i1 = 0.5 mA
Since i1 = C dVC
dt
or VC = 1 # i1 dt = i1 t ...(1)
C C

with time, the capacitor charges and voltage across collector changes from 0 towards
negative.
When saturation starts,VCE = 0.7 & VC =+ 5 V (across capacitor)

Thus from (1) we get, + 5 = 0.5 mA T


5 μA
−6
or T = 5 # 5 # 10 = 50 m sec
0.5 # 10−3
.
MCQ 1.44 In the circuit shown below, the network N is described by the following Y matrix:
0.1 S − 0.01 S
Y=>
0.1 S 0.1 S H
. the voltage gain V2 is
V1

(A) 1/90 (B) –1/90


(C) –1/99 (D) –1/11
SOL 1.44 From given admittance matrix we get
I1 = 0.1V1 − 0.01V2 and ...(1)
I2 = 0.01V1 + 0.1V2 ...(2)
Now, applying KVL in outer loop;
V2 =− 100I2

Brought to you by: Nodia and Company Visit us at: www.nodia.co.in


PUBLISHING FOR GATE
Page 25 GATE EC 2011 www.gatehelp.com

or I2 =− 0.01V2 ...(3)
From eq (2) and eq (3) we have
− 0.01V2 = 0.01V1 + 0.1V2
− 0.11V2 = 0.01V1
V2 = − 1
V1 11
Hence (D) is correct option.
MCQ 1.45 In the circuit shown below, the initial charge on the capacitor is 2.5 mC, with the
voltage polarity as indicated. The switch is closed at timet = 0 . The current i ^ t h at
a time t after the switch is closed is

(A) i (t) = 15 exp (− 2 # 103 t) A (B) i (t) = 5 exp ^− 2 # 103 t h A


(C) i (t) = 10 exp (− 2 # 103 t) A (D) i (t) =− 5 exp (− 2 # 103 t) A
SOL 1.45 Here we take the current flow direction as positive.
At t = 0− voltage across capacitor is
−3
Q
VC (0−) =− =− 2.5 # 10−6 =− 50 V
C 50 # 10
+
Thus VC (0 ) =− 50 V
In steady state capacitor behave as open circuit thus
V (3) = 100 V
Now, VC (t) = VC (3) + (VC (0+) − VC (3)) e−t/RC
−t
= 100 + (− 50 − 100) e 10 # 50 # 10
−6

= 100 − 150e− (2 # 10 t)
3

Now ic (t) = C dV
dt
= 50 # 10−6 # 150 # 2 # 103 e−2 # 10 t A
3

= 15e−2 # 10 t
3

ic (t) = 15 exp (− 2 # 103 t) A


Hence (A) is correct option.
MCQ 1.46 The system of equations
x+y+z = 6
x + 4y + 6y = 20

Brought to you by: Nodia and Company Visit us at: www.nodia.co.in


PUBLISHING FOR GATE
Page 26 GATE EC 2011 www.gatehelp.com

x + 4y + λz = μ
has NO solution for values of λ and μ given by
(A) λ = 6, μ = 20 (B) λ = 6, μ =
Y 20
(C) λ =
Y 6, μ = 20 (D) λ =
Y 6, μ = 20
SOL 1.46 Writing A: B we have
R V
S1 1 1 : 6 W
S1 4 6 : 20W
S W
S1 4 λ : μ W
T X
Apply R 3 " R 3 − R2
R V
S1 1 1 : 6 W
S1 4 6 : 20 W
S W
S0 0 λ − 6 : μ − 20W
T X
For equation to have solution, rank of A and A: B must be same. Thus for no
solution; λ = 6, μ ! 20
Hence (B) is correct option
MCQ 1.47 A fair dice is tossed two times. The probability that the second toss results in a
value that is higher than the first toss is
(A) 2/36 (B) 2/6
(C) 5/12 (D) ½
SOL 1.47 Total outcome are 36 out of which favorable outcomes are :
(1, 2), (1, 3), (1, 4), (1, 5), (1, 6), (2, 3), (2, 4), (2, 5), (2, 6);
(3, 4), (3, 5), (3, 6), (4, 5), (4, 6), (5, 6) which are 15.

Thus P (E) = No. of favourable outcomes = 15 = 5


No. of total outcomes 36 12
Hence (C) is correct option.

Common Data Questions: 48 & 49


The channel resistance of an N-channel JFET shown in the figure below is 600 Ω
when the full channel thickness (tch ) of 10μm is available for conduction. The built-
in voltage of the gate P+ N junction (Vbi ) is − 1 V . When the gate to source voltage
(VGS ) is 0 V, the channel is depleted by 1 μm on each side due to the built in voltage
and hence the thickness available for conduction is only 8 μm

Brought to you by: Nodia and Company Visit us at: www.nodia.co.in


PUBLISHING FOR GATE
Page 27 GATE EC 2011 www.gatehelp.com

MCQ 1.48 The channel resistance when VGS =− 3 V is


(A) 360 Ω (B) 917 Ω
(C) 1000 Ω (D) 3000 Ω
SOL 1.48 Full channel resistance is
ρ L
r # = 600 Ω ...(1)
W#a
If VGS is applied, Channel resistance is
ρ L
where b = a c1 −
Vp m
rl = # VGS
W#b
Pinch off voltage,
qN
Vp = D a2 ...(2)

If depletion on each side is d = 1 μm at VGS = 0 .
qN
Vj = D d2

qN qND
or 1 = D (1 # 10−6) 2 & = 1012
2ε 2ε
Now from equation (2), we have
Vp = 1012 # (5 # 10−6) 2

or Vp =− 25 V
At VGS =− 3 V ;
b = 5 b1 − − 3 μm = 3.26 μm
− 25 l
ρL ρL a = 600 5
rl = = # 3.26 = 917 Ω
W # b Wa # b
Hence (B) is correct option.
MCQ 1.49 The channel resistance when VGS = 0 V is
(A) 480 Ω (B) 600 Ω
(C) 750 Ω (D) 1000 Ω

Brought to you by: Nodia and Company Visit us at: www.nodia.co.in


PUBLISHING FOR GATE
Page 28 GATE EC 2011 www.gatehelp.com

SOL 1.49
At VGS = 0 V , b = 4 μm
since 2b = 8 μm
ρL a = 600 5 = 750 Ω
Thus rl =
Wa # b #4

Hence (C) is correct option.

Common Data Questions: 50 & 51


The input-output transfer function of a plant H (S ) = 100 . The plant is
s (s + 10) 2
placed in a unity negative feedback configuration as shown in the figure below.

MCQ 1.50 The gain margin of the system under closed loop unity negative feedback is
(A) 0 dB (B) 20 dB
(C) 26 dB (D) 46 dB
SOL 1.50 Hence (C) is correct option.
We have G (s) H (s) = 100
s (s + 10) 2
Now G (jω) H (jω) = 100
jω (jω + 10) 2
If ωp is phase cross over frequency G (jω) H (jω) = 180c
ωp
− 180c = 100 tan−1 0 − tan−1 3 − 2 tan−1 a
10 k
Thus

or − 180c =− 90 − 2 tan−1 (0.1ωp)


or 45c = tan−1 (0.1ωp)
or tan 45c = 0.1ωp = 1
or ωp = 10 rad/se
Now G (jω) H (jω) = 100
ω (ω + 100)
2

At ω = ωp
G (jω) H (jω) = 100 = 1
10 (100 + 100) 20
Gain Margin =− 20 log 10 G (jω) H (jω)
=− 20 log 10 b 1 l
20
= 26 dB

Brought to you by: Nodia and Company Visit us at: www.nodia.co.in


PUBLISHING FOR GATE
Page 29 GATE EC 2011 www.gatehelp.com

MCQ 1.51 The signal flow graph that DOES NOT model the plant transfer function H (S ) is

SOL 1.51 Hence (D) is correct option


From option (D) TF = H (s)
= 100 ! 100
s (s2 + 100) s (s + 10) 2

Linked Answer Questions: Q.52 to Q.55 Carry Two Marks Each

Statement for Linked Answer Questions: 52 & 53

MCQ 1.52 The bias current IDC through the diodes is


(A) 1 mA (B) 1.28 mA
(C) 1.5 mA (D) 2 mA
SOL 1.52 Hence (A) is correct option.
The current flows in the circuit if all the diodes are forward biased. In forward
biased there will be 0.7 V drop across each diode.
12.7 − 4 (0.7)
Thus IDC = = 1 mA
9900

Brought to you by: Nodia and Company Visit us at: www.nodia.co.in


PUBLISHING FOR GATE
Page 30 GATE EC 2011 www.gatehelp.com

MCQ 1.53 The ac output voltage Vac is


(A) 0.25 cos ^ωt h mV (B) 1 cos (ωt) mV
(C) 2 cos (ωt) mV (D) 22 cos (ωt) mV
SOL 1.53 Hence (B) is correct option.
The forward resistance of each diode is
r = VT = 25 mV = 25 Ω
IC 1 mA
4 (r)
Vac = Vi # e
4 (r) + 9900 o
Thus

= 100 mV cos (ωt) 0.01 = 1 cos (ωt) mV

Statement for Linked Answer Questions: 54 & 55


A four-phase and an eight-phase signal constellation are shown in the figure below.

MCQ 1.54 For the constraint that the minimum distance between pairs of signal points be d
for both constellations, the radii r 1 , and r 2 of the circles are
(A) r 1 = 0.707d, r2 = 2.782d (B) r 1 = 0.707d, r 2 = 1.932d
(C) r 1 = 0.707d, r 2 = 1.545d (D) r 1 = 0.707d, r 2 = 1.307d
SOL 1.54 Four phase signal constellation is shown below

Now d2 = r 12 + r 12
d2 = 2r 12
r1 = d/ 2 = 0.707d

Brought to you by: Nodia and Company Visit us at: www.nodia.co.in


PUBLISHING FOR GATE
Page 31 GATE EC 2011 www.gatehelp.com

θ = 2π = 2π = π
M 8 4
Applying Cooine law we have
d2 = r 22 + r 22 − 2r 22 cos π
4
= 2r 22 − 2r 22 1/ 2 = (2 − 2 ) r 22
or r2 = d = 1.3065d
2− 2
Hence (D) is correct option.
MCQ 1.55 Assuming high SNR and that all signals are equally probable, the additional average
transmitted signal energy required by the 8-PSK signal to achieve the same error
probability as the 4-PSK signal is
(A) 11.90 dB (B) 8.73 dB
(C) 6.79 dB (D) 5.33 dB
SOL 1.55 Here Pe for 4 PSK and 8 PSK is same because Pe depends on d . Since Pe is same,
d is same for 4 PSK and 8 PSK.

Additional Power SNR


= (SNR) 2 − (SNR) 1

= 10 log b ES2 l − 10 log b ES1 l


No No
= 10 log b ES2 l
ES1
= 10 log a r2 k & 20 log a r2 k = 20 log 1.3065d
2
r1 r1 0.707d

Brought to you by: Nodia and Company Visit us at: www.nodia.co.in


PUBLISHING FOR GATE
Page 32 GATE EC 2011 www.gatehelp.com

Additional SNR = 5.33 dB


Hence (D) is correct option.

Q. No. 56 – 60 Carry One Mark Each

MCQ 1.56 There are two candidates P and Q in an election. During the campaign, 40% of
the voters promised to vote for P, and rest for Q. However, on the day of election
15% of the voters went back on their promise to vote for P and instead voted forQ.
25% of the voters went back on their promise to vote for Q and instead voted for
P. Suppose, P lost by 2 votes, then what was the total number of voters?
(A) 100 (B) 110
(C) 90 (D) 95
SOL 1.56 Let us assume total voters are 100. Thus 40 voter (i.e. 40 %) promised to vote for
P and 60 (rest 60 % ) promised to vote fore Q.
Now, 15% changed from P to Q (15 % out of 40)
Changed voter from P to Q 15 40 = 6
100 #
Now Voter for P 40 − 6 = 34
Also, 25% changed form Q to P (out of 60%)
Changed voter from Q to P 25 60 = 15
100 #
Now Voter for P 34 + 15 = 49
Thus P P got 49 votes and Q got 51 votes, and P lost by 2 votes, which is given.
Therefore 100 voter is true value.
Hence (A) is correct option.
MCQ 1.57 Choose the most appropriate word from the options given below to complete the
following sentence:
It was her view that the country’s problems had been_________ by foreign
technocrats, so that to invite them to come back would be counter-productive.
(A) Identified (B) ascertained
(C) Texacerbated (D) Analysed
SOL 1.57 The sentence implies that technocrats are counterproductive (negative). Only (C)
can bring the same meaning.
Hence (C) is correct option
MCQ 1.58 Choose the word from the options given below that is most nearly opposite in
meaning to the given word:
Frequency
(A) periodicity (B) rarity
(C) gradualness (D) persistency

Brought to you by: Nodia and Company Visit us at: www.nodia.co.in


PUBLISHING FOR GATE
Page 33 GATE EC 2011 www.gatehelp.com

SOL 1.58 Periodicity is almost similar to frequency. Gradualness means something happening
with time. Persistency is endurance. Rarity is opposite to frequency.
Hence (B) is correct option.
MCQ 1.59 Choose the most appropriate word from the options given below to complete the
following sentence:
Under ethical guidelines recently adopted by the Indian Medical Association, human
genes are to be manipulated only to correct diseases for which______________
treatments are unsatisfactory.
(A) Similar (B) Most
(C) Uncommon (D) Available
SOL 1.59 Available is appropriate because manipulation of genes will be done when other
treatments are not useful.
Hence (D) is correct option.
MCQ 1.60 The question below consists of a pair of related words followed by four pairs of
words. Select the pair that best expresses the relation in the original pair:
Gladiator : Arena
(A) dancer : stage (B) commuter: train
(C) teacher : classroom (D) lawyer : courtroom
SOL 1.60 A gladiator performs in an arena. Commutators use trains. Lawyers performs,
but do not entertain like a gladiator. Similarly, teachers educate. Only dancers
performs on a stage.
Hence (A) is correct option.

Q. No. 61 – 65 Carry Two Marks Each

MCQ 1.61 The fuel consumed by a motorcycle during a journey while traveling at various
speeds is indicated in the graph below.

The distances covered during four laps of the journey are listed in the table below

Brought to you by: Nodia and Company Visit us at: www.nodia.co.in


PUBLISHING FOR GATE
Page 34 GATE EC 2011 www.gatehelp.com

Lap Distance (kilom- Average speed (kilometers per


eters) hour)
P 15 15
Q 75 45
R 40 75
S 10 10
From the given data, we can conclude that the fuel consumed per kilometre was
least during the lap
(A) P (B) Q
(C) R (D) S
SOL 1.61 Since fuel consumption/litre is asked and not total fuel consumed, only average
speed is relevant. Maximum efficiency comes at 45 km/hr, So least fuel consumer
per litre in lap Q
Hence (B) is correct option.
MCQ 1.62 Three friends, R, S and T shared toffee from a bowl. R took 1/3rd of the toffees,
but returned four to the bowl. S took 1/4th of what was left but returned three
toffees to the bowl. T took half of the remainder but returned two back into the
bowl. If the bowl had 17 toffees left, how many toffees-were originally there in the
bowl?
(A) 38 (B) 31
(C) 48 (D) 41
SOL 1.62 Let total no of toffees be x . The following table shows the all calculations.
Friend Bowl Status
= x −4 = 2x + 4
3 3

= 1 :2x + 4D − 3 = 2x + 4 − x + 2
4 3 3 6
= x +1−3 = x −2 = x +6
6 6 2

= 1 a x + 6k − 2 = x +6−x −1
2 2 2 4
= x +1 = x +5
4 4

Now, x + 5 = 17
4
or x = 17 − 5 = 12
4
x = 12 # 4 = 48

Brought to you by: Nodia and Company Visit us at: www.nodia.co.in


PUBLISHING FOR GATE
Page 35 GATE EC 2011 www.gatehelp.com

Hence (C) is correct option.


y
MCQ 1.63 Given that f ^y h = , and q is any non-zero real number, the value
y
of f ^q h − f ^− q h is
(A) 0 (B) − 1
(C) 1 (D) 2
SOL 1.63 Hence (D) is correct option.
y
f (y) =
y
−y
Now f (− y) = =− f (y)
y
or f (q) − f (− q) = 2f (q) = 2

MCQ 1.64 The sum of n terms of the series 4 + 44 + 444 + f. is


(A) (4/81) [10n + 1 − 9n − 1] (B) (4/81) [10n − 1 − 9n − 1]
(C) (4/81) [10n + 1 − 9n − 10] (D) (4/81) [10n − 9n − 10]
SOL 1.64 Hence (C) is correct option.
4 + 44 + 444 + .............. 4 (1 + 11 + 111 + .......)
= 4 (9 + 99 + 999 + ............)
9
= 4 [(10 − 1) + (100 − 1) + ........]
9
= 4 [10 (1 + 10 + 102 + 103) − n]
9

= 4 :10 # 10 − 1 − nD
n

9 10 − 1
= 4 610n + 1 − 10 − 9n@
81

MCQ 1.65 The horse has played a little known but very important role in the field of medicine.
Horses were injected with toxins of diseases until their blood built up immunities.
Then a serum was made from their blood. Serums to fight with diphtheria and
tetanus were developed this way.
It can be inferred from the passage that horses were
(A) given immunity to diseases
(B) generally quite immune to diseases
(C) given medicines to fight toxins
(D) given diphtheria and tetanus serums
SOL 1.65 Option B fits the sentence, as they built up immunities which helped humans

Brought to you by: Nodia and Company Visit us at: www.nodia.co.in


PUBLISHING FOR GATE
Page 36 GATE EC 2011 www.gatehelp.com

create serums from their blood.


Hence (B) is correct option.

Answer Sheet
1. (C) 13. (A) 25. (A) 37. (D) 49. (C) 61. (B)
2. (C) 14. (C) 26. (A) 38. (B) 50. (C) 62. (C)
3. (D) 15. (B) 27. (A) 39. (B) 51. (D) 63. (D)
4. (B) 16. (A) 28. (A) 40. (B) 52. (A) 64. (C)
5. (B) 17. (A) 29. (A) 41. (C) 53. (B) 65. (B)
6. (A) 18. (C) 30. (D) 42. (B) 54. (D)
7. (B) 19. (A) 31. (C) 43. (C) 55. (D)
8. (A) 20. (D) 32. (D) 44. (D) 56. (A)
9. (C) 21. (D) 33. (D) 45. (A) 57. (C)
10. (B) 22. (D) 34. (*) 46. (B) 58. (B)
11. (A) 23. (A) 35. (B) 47. (C) 59. (D)
12. (D) 24. (C) 36. (C) 48. (B) 60. (A)

Brought to you by: Nodia and Company Visit us at: www.nodia.co.in


PUBLISHING FOR GATE
EC-Paper Code-B GATE 2011 www.gateforum.com

Q. No. 1 – 25 Carry One Mark Each


1. Consider the following statements regarding the complex Poynting vector P for
the power radiated by a point source in an infinite homogeneous and lossless
()
 
medium. Re P denotes the real part of P . S denotes a spherical surface whose
ɵ denotes the unit surface normal on S. Which
centre is at the point source, and n
of the following statements is TRUE?

()

(A) Re P remains constant at any radial distance from the source

(B) Re (P ) increases with increasing radial distance from the source




∫∫ Re (P ) .ndS remains constant at any radial distance from the source



(C) 
s

∫∫ Re (P ) .ndS

(D)  decreases with increasing radial distance form the source
s

Answer: - (D)

()

Exp: - ∫∫
 S
ˆ
Re P .nds gives average power and it decreases with increasing radial
distance from the source

2. A transmission line of characteristic impedance 50Ω is terminated by a 50Ω load.


When excited by a sinusoidal voltage source at 10GHz, the phase difference
π
between two points spaced 2mm apart on the line is found to be radians. The
4
phase velocity of the wave along the line is
(A) 0.8 × 108 m / s (B) 1.2 × 108 m / s (C) 1.6 × 108 m / s (D) 3 × 108 m / s
Answer: - (C)
Exp: - Z0 = 50Ω ; ZL = 50Ω
π
For radians the distance is 2mm
4
ω 2 × π × 1010
The phase velocity vP = = = 16 × 10−7 = 1.6 × 108 m / s
β 2π
16 × 10−3

3. An analog signal is band-limited to 4kHz, sampled at the Nyquist rate and the
samples are quantized into 4 levels. The quantized levels are assumed to be
independent and equally probable. If we transmit two quantized samples per
second, the information rate is ________ bits / second.
(A) 1 (B) 2 (C) 3 (D) 4

Answer: - (D)
Exp: - Since two samples are transmitted and each sample has 2 bits of information,
then the information rate is 4 bits/sec.
© All rights reserved by Gateforum Educational Services Pvt. Ltd. No part of this document may be reproduced or utilized in any form without the
written permission. Discuss GATE 2011 question paper at www.gatementor.com. 1
EC-Paper Code-B GATE 2011 www.gateforum.com

4. The root locus plot for a system is given below. The open loop transfer function
corresponding to this plot is given by
s ( s + 1)
(A) G ( s ) H ( s) = k
( s + 2 ) ( s + 3)

(B) G ( s ) H ( s ) = k
( s + 1)
s ( s + 2 ) ( s + 3)
2

× × o × σ
1 −1
(C) G ( s ) H ( s ) = k −3 −2 0
s ( s − 1) ( s + 2 ) ( s + 3 )

( s + 1)
(D) G ( s ) H ( s ) = k
s ( s + 2) ( s + 3)
Answer: - (B)
Exp: - ' x ' → indicates pole
' O ' → indicates zero
The point on the root locus when the number of poles and zeroes on the real axis
to the right side of that point must be odd

5. A system is defined by its impulse response h (n) = 2nu (n − 2 ) . The system is


(A) stable and causal (B) causal but not stable
(C) stable but not causal (D) unstable and non-causal
Answer: - (B)
Exp: - h (n) = 2n u (n − 2 )
h (n) is existing for n>2 ; so that h (n) = 0;n < 0 ⇒ causal
∞ ∞ ∞

∑ h (n ) = ∑ 2 u (n − 2 ) = ∑ 2
n =−∞ n =∞
n

n=2
n
= ∞ ⇒ System is unstable

6. ( )
If the unit step response of a network is 1 − e−αt , then its unit impulse response
is
(A) αe−αt (B) α −1e−αt (
(C) 1 − α −1 e−αt ) (D) (1 − α ) e−αt

Answer: - (A)
Exp: - S ( t ) → step response

Impulse response h ( t ) =
d
dt
d
(S ( t ) ) = dt (
1 − eαt = αeαt )

7. The output Y in the circuit below is always ‘1’ when

Q
Y
R

© All rights reserved by Gateforum Educational Services Pvt. Ltd. No part of this document may be reproduced or utilized in any form without the
written permission. Discuss GATE 2011 question paper at www.gatementor.com. 2
EC-Paper Code-B GATE 2011 www.gateforum.com

(A) two or more of the inputs P,Q,R are ‘0’


(B) two or more of the inputs P,Q,R are ‘1’
(C) any odd number of the inputs P,Q,R is ‘0’
(D) any odd number of the inputs P,Q,R is ‘1’
Answer: - (B)
Exp: - The output Y expression in the Ckt
Y = PQ + PR + RQ
So that two or more inputs are ‘1’, Y is always ‘1’.

8. In the circuit shown below, capacitors C1 and C2 are very large and are shorts at
v
the input frequency. vi is a small signal input. The gain magnitude o at 10M
vi
rad/s is 5V

2kΩ
10µH
2kΩ

+
C2
Q1
+
+
2.7V vo
− 2kΩ
2kΩ C1 −
vi ~

(A) maximum (B) minimum (C) unity (D) zero


Answer: - (A)
Exp: - In the parallel RLC Ckt
L = 10µH and C = 1nF
1 1
ωg = = = 107 rad / s = 10Mrad / s
LC 10 × 10−6 × 10−9
So that for a tuned amplifier, gain is maximum at resonant frequency

9. Drift current in the semiconductors depends upon


(A) only the electric field
(B) only the carrier concentration gradient
(C) both the electric field and the carrier concentration
(D) both the electric field and the carrier concentration gradient
Answer: - (C)
Exp: - Drift current, J = σE
J = (nµn + pµP ) qE
So that it depends on carrier concentration and electric field.
© All rights reserved by Gateforum Educational Services Pvt. Ltd. No part of this document may be reproduced or utilized in any form without the
written permission. Discuss GATE 2011 question paper at www.gatementor.com. 3
EC-Paper Code-B GATE 2011 www.gateforum.com

10. A Zener diode, when used in voltage stabilization circuits, is biased in


(A) reverse bias region below the breakdown voltage
(B) reverse breakdown region
(C) forward bias region
(D) forward bias constant current mode
Answer: - (B)
Exp: -

V2

For Zener diode


Voltage remains constant in break down region and current carrying capacity in
high.

11. The circuit shown below is driven by a sinusoidal input vi = Vp cos ( t / RC ) . The
steady state output vo is

R C
+

+
vi ~ vo
− R C

(A) ( Vp / 3) cos ( t / RC ) (B) ( Vp / 3) sin ( t / RC )

(C) ( Vp / 2 ) cos ( t / RC ) (D) ( Vp / 2 ) sin ( t / RC )

Answer: - (A)
v0 z2 1 1
Exp: - = where z2 = R || and z1 = R +
vi z1 + z2 jωc jωc
R
z2 =
R ( jcw ) + 1

1   t  R
Given w = ∵ vi = vp cos    ⇒ z2 = 1 + j
RC   RC 
1 1
z1 = R + =R+ ⇒ R (1 − j)
jωc jR
R
v0 1+ j 1 1
= = =
vi R 1+2 3
+ R (1 − j)
1+ j
© All rights reserved by Gateforum Educational Services Pvt. Ltd. No part of this document may be reproduced or utilized in any form without the
written permission. Discuss GATE 2011 question paper at www.gatementor.com. 4
EC-Paper Code-B GATE 2011 www.gateforum.com


12. Consider a closed surface S surrounding volume V. If r is the position vector of a

ɵ the unit normal on S, the value of the integral ɵ
point inside S, with n ∫∫ 5r.ndS
 is
s

(A) 3V (B) 5V (C) 10V (D) 15V


Answer: - (D)
Exp: - Apply the divergence theorem
 
∫∫
 5r.n.dx = ∫∫∫ 5∇.rdV
S v
 
= 5 (3 ) ∫∫∫ dv = 15 V (∵ ∇.r = 3 and r is the position vector )
v

TEmn
13. The modes in a rectangular waveguide are denoted by where m and n are
TMmn
the eigen numbers along the larger and smaller dimensions of the waveguide
respectively. Which one of the following statements is TRUE?
(A) The TM10 mode of the wave does not exist
(B) The TE10 mode of the wave does not exist
(C) The TM10 and the TE10 modes both exist and have the same cut-off
frequencies
(D) The TM10 and TM01 modes both exist and have the same cut-off frequencies
Answer: - (A)
Exp: - TM10 mode doesn’t exist in rectangular waveguide.

dy
14. The solution of the differential equation = ky, y ( 0 ) = c is
dx
(A) x = ce−ky (B) x = kecy (C) y = cekx (D) y = ce −kx
Answer: - (C)
dy dy
Exp: - Given y ( 0 ) = C and = ky, ⇒ = kdx
dx y
ln y = kx + c ⇒ y = ekx ec
When y ( 0 ) = C , y = k1 e0 ∴ y = c ekx (∵ k1 = C)

15. The Column-I lists the attributes and the Column-II lists the modulation
systems. Match the attribute to the modulation system that best meets it
Column-I Column-II
P Power efficient transmission of signals 1 Conventional AM
Most bandwidth efficient transmission of
Q 2 FM
voice signals
R Simplest receiver structure 3 VSB
Bandwidth efficient transmission of
S 4 SSB-SC
signals with Significant dc component
(A) P-4;Q-2;R-1;S-3 (B) P-2;Q-4;R-1;S-3
(C) P-3;Q-2;R-1;S-4 (D) P-2;Q-4;R-3;S-1

© All rights reserved by Gateforum Educational Services Pvt. Ltd. No part of this document may be reproduced or utilized in any form without the
written permission. Discuss GATE 2011 question paper at www.gatementor.com. 5
EC-Paper Code-B GATE 2011 www.gateforum.com

Answer: - (B)
Exp: - Power efficient transmission → FM
Most bandwidth efficient → SSB-SC
Transmission of voice signal
Simplest receives structure → conventional AM
Bandwidth efficient transmission of → VSB
Signals with significant DC component

d2 y dy
16. The differential equation 100 − 20 + y = x ( t ) describes a system with an
dt2 dt
input x(t) and an output y(t). The system, which is initially relaxed, is excited by
a unit step input. The output y(t) can be represented by the waveform

(A) y (t) (B) y (t)

t t

y (t) y (t)

(C) (D)

t t

Answer: - (A)
100d2 y 20dy
Exp: - − + y = x (t )
dt2 dt
Apply L.T both sides
 1
(100s 2
)
− 20s + 1 Y ( s ) =
1
s ∵ x ( t ) = u ( t ) x ( s ) = 3 
 

1
Y (s) =
(
s 100s − 20s + 1
2
)
So we have poles with positive real part ⇒ system is unstable.

© All rights reserved by Gateforum Educational Services Pvt. Ltd. No part of this document may be reproduced or utilized in any form without the
written permission. Discuss GATE 2011 question paper at www.gatementor.com. 6
EC-Paper Code-B GATE 2011 www.gateforum.com

17. For the transfer function G ( jω) = 5 + jω , the corresponding Nyquist plot for
positive frequency has the form

jω jω
j5
(A) σ (B) σ
5

jω jω

1/5
(C) σ (D) 1/5 σ

Answer: - (A)
Exp: - As we increases real part ‘5’ is fixed only imaginary part increases.

18. The trigonometric Fourier series of an even function does not have the
(A) dc term (B) cosine terms
(C) sine terms (D) odd harmonic terms
Answer: - (C)
Exp: - f ( t ) is even function, hence bk = 0
Where 'bk ' is the coefficient of sine terms

19. When the output Y in the circuit below is ‘1’, it implies that data has

Y
Data
D Q D Q

Clock
Q Q

(A) changed from 0 to 1 (B) changed from 1 to 0


(C) changed in either direction (D) not changed
Answer: - (A)
Exp: - When data is ‘0’, Q is ‘0’
And Q’ is ‘1’ first flip flop
Data is changed to 1
Q is 1 → first ‘D’
Q’ is connected to 2nd flip flop
So that Q2 = 1
So that the inputs of AND gate is ‘1’ ⇒ y = '1'
© All rights reserved by Gateforum Educational Services Pvt. Ltd. No part of this document may be reproduced or utilized in any form without the
written permission. Discuss GATE 2011 question paper at www.gatementor.com. 7
EC-Paper Code-B GATE 2011 www.gateforum.com

20. The logic function implemented by the circuit below is (ground implies logic 0)
4 × 1MUX

I0
I1
I2 Y F

I3
S1 S0

P Q

(A) F = AND (P, Q ) (B) F = OR (P, Q ) (C) F = XNOR (P, Q ) (D) F = XOR (P, Q )
Answer: - (D)
Exp: - From the CKT
O is connected to ' I0 ' & ' I3 '
And ‘1’ is connected to I1 & I2 ∴ F = PQ + PQ = XOR (P, Q )

21. The circuit below implements a filter between the input current ii and the output
voltage vo. Assume that the opamp is ideal. The filter implemented is a

L1

R1
ii


+
+ vo

(A) low pass filter (B) band pass filter


(C) band stop filter (D) high pass filter
Answer: - (D)
Exp: - When W=0; inductor acts as a S.C ⇒ V0 = 0
And when ω = ∞ , inductor acts as a O.C ⇒ V0 = i1R1
So it acts as a high pass filter.

22. A silicon PN junction is forward biased with a constant current at room


temperature. When the temperature is increased by 10ºC, the forward bias
voltage across the PN junction
(A) increases by 60mV (B) decreases by 60mV
(C) increases by 25mV (D) decreases by 25mV
Answer: - (D)
Exp: - For Si forward bias voltage change by -2.5mv /0 C
For 100 C increases, change will be −2.5 × 10 = −25mV
© All rights reserved by Gateforum Educational Services Pvt. Ltd. No part of this document may be reproduced or utilized in any form without the
written permission. Discuss GATE 2011 question paper at www.gatementor.com. 8
EC-Paper Code-B GATE 2011 www.gateforum.com

23. In the circuit shown below, the Norton equivalent current in amperes with respect
to the terminals P and Q is
j30Ω
P

16 ∠ 0O A 25Ω − j50Ω

Q
15Ω

(A) 6.4 − j4.8 (B) 6.56 − j7.87 (C) 10 + j0 (D) 16 + j0


Answer: - (A)
Exp: - When terminals P & Q are S.C
Then the CKT becomes
j30Ω
P

16∠00 A
25Ω
IN = ISC

15Ω Q

16 ( 25 ) (16 ) (25) =
(16 ) (25) = 6.4 − j4.8
From current Division rules IN = =
25 + 15 + j30 40 + j30 10 ( 4 + j3 )

24. In the circuit shown below, the value of RL such that the power transferred to RL
is maximum is
(A) 5Ω (B) 10 Ω (C) 15 Ω (D) 20 Ω

10Ω 10Ω

10Ω RL

1A
+ +
5V 2V
− −

Answer: - (C)
Exp: - For maximum power transmission R L = R *TH
For the calculation of R TH
© All rights reserved by Gateforum Educational Services Pvt. Ltd. No part of this document may be reproduced or utilized in any form without the
written permission. Discuss GATE 2011 question paper at www.gatementor.com. 9
GATE ESE PSU’s 2019-20
ECE ENGINEERING
GATE ECE 2003-2019 SOLVED

GATE ECE 2003-2019 SOLVED Detail Solution

CONTENT COVERED:
1.Theory Notes
2.Explanation
3.Derivation
4.Example
5.Shortcut & Formula Summary
6.Previous year Paper Q. Sol.
Noted-: Single Source Follow, Revise
Multiple Time Best key of Success
1
Page

http://www.orbitmentor.com [email protected]
EC-Paper Code-B GATE 2011 www.gateforum.com

10Ω
10Ω

R TH
10Ω

Q
R TH = (10 || 10 ) + 10 = 15Ω

−3z + 4
25. The value of the integral ∫ ( z
c
2
+ 4z + 5 )
dz where c is the circle z = 1 is given by

(A) 0 (B) 1/10 (C) 4/5 (D) 1


Answer: - (A)

Exp: - ∫ z
C
−3z + 4
2
+ 4z + 5
(
dz = 0 ∵ z2 + 4z + 5 = ( z + 2 ) + 1 = 0
2
)
z = −2 ± j will be outside the unit circle
So that integration value is ‘zero’.

Q. No. 26 – 51 Carry Two Marks Each


26. A current sheet J = 10uɵ y A/m lies on the dielectric interface x=0 between two
dielectric media with εr1 = 5, µr1 = 1 in Region -1 (x<0) and εr2 = 5, µr2 = 2 in

Region -2 (x>0). If the magnetic field in Region-1 at x=0- is H1 = 3uɵ x + 30uɵ y A / m
the magnetic field in Region-2 at x=0+ is

x > 0 (Re gion − 2 ) : εr2 , µr2 = 2 x



J
x=0
y
x < 0 (Re gion − 1) : εr1 , µr1 = 1
 
(A) H2 = 1.5uɵ x ɵ y − 10uɵ z A / m
+ 30u (B) H2 = 3uɵ x + 30uɵ y − 10uɵ z A / m
 
(C) H2 = 1.5uɵ x + 40uɵ y A / m (D) H2 = 3uɵ x + 30uɵ y + 10uɵzA / m

Answer: - (A)
Exp: - Ht2 − Ht1 = J × an ⇒ Ht2 = Ht1 − 10u
ˆz = 30uy − 10u
ˆz
And Bn1 = Bn2
µ1
µ1H1 = µ2H2 ⇒ H2 = H2
µ2
Normal component in x direction
1
H2 = (3) uˆx = 1.5uˆx ; H2 = 1.5uˆx + 30uˆy − 10uz A / m
2
© All rights reserved by Gateforum Educational Services Pvt. Ltd. No part of this document may be reproduced or utilized in any form without the
written permission. Discuss GATE 2011 question paper at www.gatementor.com. 10
EC-Paper Code-B GATE 2011 www.gateforum.com

27. A transmission line of characteristic impedance 50W is terminated in a load


impedance ZL. The VSWR of the line is measured as 5 and the first of the voltage
λ
maxima in the line is observed at a distance of from the load. The value of ZL
4
is
(A) 10Ω (B) 250 Ω
(C) (19.23 + j46.15 ) Ω (D) (19.23 − j46.15 ) Ω

Answer: - (A)
λ
Exp: - Voltage maximum in the line is observed exactly at
4
Therefore ' zL ' should be real
z0 50
VSWR = ⇒ zL = = 10Ω (∵ Voltage minimum at load)
zL 5

28. X(t) is a stationary random process with autocorrelation function


( )
R x ( τ ) = exp πr2 . This process is passed through the system shown below. The
power spectral density of the output process Y(t) is
( ) ( )
(A) 4π2 f 2 + 1 exp −πf 2
H ( f ) = j2 π f
(B) ( 4π f 2 2
− 1) exp ( −πf ) 2
+
X (t ) Y (t )
(C) ( 4π f + 1) exp ( −πf )
2 2 ∑

(D) ( 4π f 2 2
− 1) exp ( −πf )

Answer: - (A)
Exp: - The total transfer function H(f) = ( j2πf − 1)

SX ( f ) = H ( f ) Sx ( f ) R x ( τ ) ← → Sx ( f )
2 F

( )
= 4π2 f 2 + 1 e−πf
2

(∵ e −πt2
←
F
→ e− πf
2

29. The output of a 3-stage Johnson (twisted ring) counter is fed to a digital-to-
analog (D/A) converter as shown in the figure below. Assume all the states of the
counter to be unset initially. The waveform which represents the D/A converter
output Vo is

Vref D/A
Converter Vo
D2 D1 D0

Q2 Q1 Q0
Johnson
Clock Counter

© All rights reserved by Gateforum Educational Services Pvt. Ltd. No part of this document may be reproduced or utilized in any form without the
written permission. Discuss GATE 2011 question paper at www.gatementor.com. 11
EC-Paper Code-B GATE 2011 www.gateforum.com

Vo Vo

(A) (B)

Vo
Vo

(C) (D)

Answer: - (A)
Exp: - For the Johnson counter sequence
D2D1D0 V0
0 0 0 − 1
1 0 0 − 4
11 0 − 6
111 − 7
0 11 − 3
0 0 1 − 1
0 0 0 −0

30. Two D flip-flops are connected as a synchronous counter that goes through the
following QBQA sequence 0011011000…
The combination to the inputs DA and DB are
(A) DA = QB ; DB = QA (B) DA = QA ; DB = QB

(
(C) DA = QA QB + QA QB ; DB = QA ) (
(D) DA = QA QB + QA QB ; DB = QB )
Answer: - (D)
Exp: - Q (present) Q(next)
QB QA Q1B Q1A DB DA
0 0 1 1 1 1
1 1 0 1 0 1
0 1 1 0 1 0
1 0 0 0 0 0

DA = QB DB
1 QB
QA 0 1
1 QA
0 1 0
0

1
1
1
1 0

© All rights reserved by Gateforum Educational Services Pvt. Ltd. No part of this document may be reproduced or utilized in any form without the
written permission. Discuss GATE 2011 question paper at www.gatementor.com. 12
EC-Paper Code-B GATE 2011 www.gateforum.com

31. In the circuit shown below, for the MOS transistors, µnCox = 100µA / V 2 and the
threshold voltage VT = 1V. The voltage Vx at the source of the upper transistor is

6V

5V W /L = 4

Vx

W /L = 1

(A) 1V (B) 2V (C) 3V (D) 3.67V


Answer: - (C)
W
Exp: - The transistor which has =4
L
VDS = 6 − Vx and VGS = 5 − Vx
VGS − VT = 5 − Vx − 1 = 4 − Vx
VDS > VGS − VT
So that transistor in saturation region.
W
The transistor which has =1
L
Drain is connected to gate
So that transistor in saturation
VDS > VGS > VT (∵ VDS = VGS )

The current flow in both the transistor is same

 ( VGS )1 − VT   ( VGS )2 − VT 
2 2
w w
µn c0x L    = µn c0x   . 
 1  2   L 2  2 
   

(5 − Vx − 1) ( V − 4)
2 2

4 =1 x (∵ VGS = Vx − 0 )
2 2
( )
4 Vx2 − 8Vx + 16 = Vx2 − 2Vx + 1 ⇒ 3Vx2 − 30Vx + 63 = 0 ⇒ Vx = 3V

32. An input x ( t ) = exp ( −2t ) u ( t ) + δ ( t − 6 ) is applied to an LTI system with impulse


response h ( t ) = u ( t ) . The output is is

(A) 1 − exp ( −2t )  u ( t ) + u ( t + 6 ) (B) 1 − exp ( −2t )  u ( t ) + u ( t − 6 )

(C) 0.5 1 − exp ( −2t )  u ( t ) + u ( t + 6 ) (D) 0.5 1 − exp ( −2t )  u ( t ) + u ( t − 6 )

© All rights reserved by Gateforum Educational Services Pvt. Ltd. No part of this document may be reproduced or utilized in any form without the
written permission. Discuss GATE 2011 question paper at www.gatementor.com. 13
EC-Paper Code-B GATE 2011 www.gateforum.com

Answer: - (D)
1 1
Exp: - x ( s ) = + e−6s and H ( s ) =
s+2 s
1 e−6s 11 1 e−6s
Y ( s ) = H ( s) × ( s ) = + = − +
s (s + 2) s 2 s 2 ( s + 2) s

( )
⇒ y ( t ) = 0.5 1 − e−2t u ( t ) + u ( t − 6 )

33. For a BJT the common base current gain α = 0.98 and the collector base junction
reverse bias saturation current ICO = 0.6µA. This BJT is connected in the common
emitter mode and operated in the active region with a base drive current
IB=20 A. The collector current IC for this mode of operation is
(A) 0.98mA (B) 0.99mA (C) 1.0mA (D) 1.01mA
Answer: - (D)
α 0.98
Exp: - IC = βIB + (1 + β ) ICB0 = β = = = 49
1 − α 1 − 0.98
IB = 20µA, ICB0 = 0.6µA ∴ IC = 1.01mA

2 ( s + 1)
34. If F ( s ) = L f ( t )  = then the initial and final values of f(t) are
s + 4s + 7
2

respectively
(A) 0,2 (B) 2,0 (C) 0,2/7 (D) 2/7,0
Answer: - (B)
s (2s + 1)
Exp: - Lt f ( t ) = Lt =2
t →0 s→∞ s2 + 4s + 7
s (2s + 1)
Lt f ( t ) = Lt =0
t →∞ s →0 s2 + 4s + 7

35. In the circuit shown below, the current I is equal to

I − j4 Ω
j4Ω
+
6Ω
14 0ºV ~

6Ω 6Ω

(A) 14 0ºA (B) 2.0 0ºA (C) 2.8 0ºA (D) 3.2 0ºA
Answer: - (B)
Exp: - Apply the delta – to – star conversion
The circuit becomes
© All rights reserved by Gateforum Educational Services Pvt. Ltd. No part of this document may be reproduced or utilized in any form without the
written permission. Discuss GATE 2011 question paper at www.gatementor.com. 14
EC-Paper Code-B GATE 2011 www.gateforum.com

I
+ − j4 Ω
j4Ω
14 0ºV ~

2Ω 2Ω

2Ω

4 + 16
The net Impedance = ( 2 + j4 ) || ( 2 − j4 ) + 2 = + 2 = 7Ω
4
14∠00
I= = 2∠00 A
7

36. A numerical solution of the equation f ( x ) = x + x − 3 = 0 can be obtained using


Newton- Raphson method. If the starting value is x = 2 for the iteration, the
value of x that is to be used in the next step is
(A) 0.306 (B) 0.739 (C) 1.694 (D) 2.306
Answer: - (C)
f ( xn )
Exp: - xn+1 = xn −
f 1 ( xn )

(
f (2) = 2 + 2 − 3 = ) 2 − 1 and f 1 ( 2 ) = 1 +
1
2 2
=
2 2 +1
2 2

⇒ xn+1 = 2 −
( 2 −1 ) = 1.694
x 2 +1
2 2

37. The electric and magnetic fields for a TEM wave of frequency 14 GHz in a
homogeneous medium of relative permittivity εr and relative permeability µr = 1
are given by
 
E = Ep e (
j ωt −280 πy )
H = 3e (
j ωt −280 π y )
ˆz V / m
u ˆ
ux A / m
Assuming the speed of light in free space to be 3 x 108 m/s, the intrinsic
impedance of free space to be 120π , the relative permittivity εr of the medium
and the electric field amplitude Ep are
(A) εr = 3, Ep = 120π (B) εr = 3, Ep = 360π
(C) εr = 9, Ep = 360π (D) εr = 9, Ep = 120π
Answer: - (D)
E µ µ
Exp: - =η= = 120π r
H ∈ ∈r

EP µ
= η = 120π r Only option ‘D’ satisfies
3 ∈r

© All rights reserved by Gateforum Educational Services Pvt. Ltd. No part of this document may be reproduced or utilized in any form without the
written permission. Discuss GATE 2011 question paper at www.gatementor.com. 15
EC-Paper Code-B GATE 2011 www.gateforum.com

38. A message signal m ( t ) = cos 200πt + 4 cos πt modulates the carrier


c ( t ) = cos 2π fc t where fc = 1 MHZ to produce an AM signal. For demodulating the
generated AM signal using an envelope detector, the time constant RC of the
detector circuit should satisfy
(A) 0.5 ms < RC < 1ms (B) 1µs << RC < 0.5 ms
(C) RC << µs (D) RC >> 0.5 ms
Answer: - (B)
1
Exp: - Time constant should be length than
fm
1
And time constant should be far greater than
fc
4000a
fm = = 2000
2a
1 1
<< Rc <
fC 2000
1µs << RC << 0.5ms

39. The block diagram of a system with one input it and two outputs y1 and y2 is given
below.

11
y1
ss ++ 2

22 y2
ss++22

A state space model of the above system in terms of the state vector x and
T
the output vector y = y1 y2  is

(A) x = 2 x + 1u; y = 1 2 x
• 1 
(B) x = −2 x + 1 u; y =  x
2 
•  −2 0  1
(C) x =   x +   u; y = 1 2 x
 0 − 2 1
• 2 0 1 1 
(D) x =   x +   u; y =  x
0 2 1 2 
Answer: - (B)
Exp: - Draw the signal flow graph

© All rights reserved by Gateforum Educational Services Pvt. Ltd. No part of this document may be reproduced or utilized in any form without the
written permission. Discuss GATE 2011 question paper at www.gatementor.com. 16
EC-Paper Code-B GATE 2011 www.gateforum.com

x 1/S
x

4 y1

−2

2 /S

y2

−1
From the graph
xɺ = −2x + 4 & y1 = x1 ; y2 = 2x1
y  1 
xɺ = −2 x + 1 u;  1  =   x
y2  2 

40. Two systems H1 (z) and H2 (z) are connected in cascade as shown below. The
overall output y(n) is the same as the input x(n) with a one unit delay. The
transfer function of the second system H2 (z) is

H1 ( z ) =
(1 − 0.4z )
−1

H2 ( z ) y (n )
X (n )
(1 − 0.6z )
−1

(A)
(1 − 0.6z ) −1

(B)
(
z−1 1 − 0.6z−1 )
z (1 − 0.4z )
−1 −1
(1 − 0.4z ) −1

z (1 − 0.4z )
−1 −1
(1 − 0.4z ) −1

(C) (D)
(1 − 0.6z ) −1
z (1 − 0.6z )
−1 −1

Answer: - (B)
Exp: - The overall transfer function = z −1 (∵ unit day T.F = z −1 )

H1 ( z ) H2 ( z ) = z ;−1
H2 ( z ) =
z −1
=z −1
(
1 − 0.6z −1 )
H1 ( z ) (
1 − 0.4z−1 )

41. An 8085 assembly language program is given below. Assume that the carry flag
is initially unset. The content of the accumulator after the execution of the
program is
MVI A,07H
RLC
MOV B,A
RLC
RLC
ADD B
RRC
(A) 8CH (B) 64H (C) 23H (D) 15H

© All rights reserved by Gateforum Educational Services Pvt. Ltd. No part of this document may be reproduced or utilized in any form without the
written permission. Discuss GATE 2011 question paper at www.gatementor.com. 17
EC-Paper Code-B GATE 2011 www.gateforum.com

Answer: - (C)
Exp: - MVI A, 07 H ⇒ 0000 0111 ← The content of ‘A’
RLC ⇒ 0000 1110 ← The content of ‘A’
MOV B, A ⇒ 0000 1110 ← The content of ‘B’
RLC ⇒ 0001 1100 ← The content of ‘B’
RLC ⇒ 0011 1000 ← The content of ‘B’
ADD B

A 0000 1110
+
B 0011 1000
0100 0110
0010 0011
RRC → 23H
2 3

42. The first six points of the 8-point DFT of a real valued sequence
are5, 1 − j3, 0,3 − j4, 0 and 3 + j4. . The last two points of the DFT are respectively
(A) 0, 1-j3 (B) 0, 1+j3 (C) 1+j3, 5 (D) 1 – j3, 5
Answer: - (C)
Exp: - DFT points are complex conjugates of each other and they one symmetric to the
middle point.
x ( 0 ) = x* (7 )
x (1) = x* ( 6 )
x (2 ) = x* (5 )
x (3 ) = x* ( 4 )

⇒ Last two points will be x* ( 0 ) and x* (1) = 1 + j3 and 5

43. For the BJT QL in the circuit shown below, β = ∞, VBEon=0.7V, VCEsat = 0.7V. The switch
is initially closed. At time t = 0, the switch is opened. The time t at which Q1
leaves the active region is

5V

0.5mA

−5V
Q1 5µF
t =0

4.3kΩ

−10V

(A) 10 ms (B) 25 ms (C) 50 ms (D) 100 ms


© All rights reserved by Gateforum Educational Services Pvt. Ltd. No part of this document may be reproduced or utilized in any form without the
written permission. Discuss GATE 2011 question paper at www.gatementor.com. 18
EC-Paper Code-B GATE 2011 www.gateforum.com

Answer: - (C)
Exp: - Apply KVL at the BE junction
−5 − 0.7 + 10 4.3
IE = = = 1mA
4.3kΩ 4.3kΩ
5V
Always IE = 1mA ; At collector junction
ICap + ( 0.5mA ) = 1mA (∵ β = ∞;IE = IC )
0.5mA
VC
ICap = 1 − 0.5 = 0.5mA always constant
−5V 5mF
VCE = VC − VE ⇒ VC = VCE + VE t =0

= 0.7 + ( 4.3 ) 3 × 1 × 10−3 = 0.7 + 4.3 (∵ VE = IERE )


4.3kΩ
VC = 5V = Vcap VE = IER E
−10V
t VCap ( C ) (5) × 5 × 10−6
Vcap = ICap Or t = = = 50ms
c ICap 0.5 × 10−3
44. In the circuit shown below, the network N is described by the following Y matrix:
0.1S − 0.01S  V2
Y =  . the voltage gain is
0.01S 0.1S  V1

25Ω I1 I2
+ +

+ V1 N
100V − V2 100Ω

− −

(A) 1/90 (B) –1/90 (C) –1/99 (D) –1/11


Answer: - (D)
Exp: - N1 = 100V + 25I1 ; V2 = −100I2
V2 −1
I2 = Y3 V1 + Y4 V2 ⇒ −0.01V2 = 0.01V1 + 0.1V2 ⇒ =
V1 11

45. In the circuit shown below, the initial charge on the capacitor is 2.5 mC, with the
voltage polarity as indicated. The switch is closed at time t=0. The current i(t) at
a time t after the switch is closed is
i (t )
(
(A) i ( t ) = 15 exp −2 × 103 t A )
(
(B) i ( t ) = 5 exp −2 × 103 t A ) +
10Ω

100V −
(
(C) i ( t ) = 10 exp −2 × 10 t A 3
) −
50µF
+
(
(D) i ( t ) = −5 exp −2 × 103 t A )
© All rights reserved by Gateforum Educational Services Pvt. Ltd. No part of this document may be reproduced or utilized in any form without the
written permission. Discuss GATE 2011 question paper at www.gatementor.com. 19
EC-Paper Code-B GATE 2011 www.gateforum.com

Answer: - (A)
Exp: - Q = 2.5mC

2.5 × 10−3 C
Vinitial = = 50V ⇒ Thus net voltage = 100 + 50 = 150V
50 × 10−6 F

i (t) =
150
10
( )
exp −2 × 10t t A =15 exp −2 × 10t t A ( )

46. The system of equations


x+y+z=6
x + 4y + 6z = 20
x + 4y + λz = µ

has NO solution for values of λ and µ given by


(A) λ = 6, µ = 20 (B) λ = 6, µ ≠ 20 (C) λ ≠ 6, µ = 20 (D) λ ≠ 6, µ ≠ 20

Answer: - (B)
Exp: - Given equations are x + y + z = 6 , x + 4y + 6z = 20 and x + 4y + λz = µ

If λ = 6 and µ = 20 , then x + 4y + 6z = 20

x + 4y + 6z = 20 infinite solution

If λ = 6 and µ ≠ 20 , the

x + 4y + 6z = 20
( µ ≠ 20 ) no solution
x + 4y + 6z = µ

If λ ≠ 6 and µ = 20

x + 4y + 6z = 20
will have solution
x + 4y + λz = 20

λ ≠ 6 and µ ≠ 20 will also give solution

47. A fair dice is tossed two times. The probability that the second toss results in a
value that is higher than the first toss is
(A) 2/36 (B) 2/6 (C) 5/12 (D) ½
Answer: - (C)
Exp: - Total number of cause = 36
Total number of favorable causes = 5+ 4 + 3 + 2 + 1 = 15
15 5
Then probability = =
36 12

(1,1) (2,1) (3,1) ( 4,1) (5,1)


( 6,1)
© All rights reserved by Gateforum Educational Services Pvt. Ltd. No part of this document may be reproduced or utilized in any form without the
written permission. Discuss GATE 2011 question paper at www.gatementor.com. 20
EC-Paper Code-B GATE 2011 www.gateforum.com

(1, 2) (2, 2) (3,2) ( 4, 2) (5,2 )


( 6,2)
(1,3) (2, 3) (3,3) ( 4, 3) (5,3)
( 6,3)
(1, 4) (2, 4) (3, 4) ( 4, 4) (5, 4 )
( 6, 4)
(1,5) (2,5) (3,5) ( 4,5) (5,5)
( 6,5)
(1, 6 ) (2, 6 ) (3, 6 ) ( 4, 6 ) (5, 6 )
( 6, 6 )

Common Data Questions: 48 & 49

The channel resistance of an N-channel JFET shown in the figure below is 600 Ω
when the full channel thickness (tch) of 10µm is available for conduction. The
built-in voltage of the gate P+ N junction (Vbi) is -1 V. When the gate to source
voltage (VGS) is 0 V, the channel is depleted by 1µm on each side due to the built-
in voltage and hence the thickness available for conduction is only 8µm

+
Gate
VGS P+
− Source Drain
t ch N

P+

48. The channel resistance when VGS = -3 V is


(A) 360Ω (B) 917Ω (C) 1000Ω (D) 3000Ω
Answer: - (C)
Exp: - Width of the depletion large W α Vbi + VGS

W2 −1 − 3
= ⇒ w2 = 2w1 = 2 (1µm) = 2µm
W1 −1
So that channel thickness = 10 – 4 = 6µm
8µm − 750
6µm − ?
8
rd = × 750 = 1000 Ω
6

49. The channel resistance when VGS = 0 V is


(A) 480Ω (B) 600Ω (C) 750Ω (D) 1000Ω
© All rights reserved by Gateforum Educational Services Pvt. Ltd. No part of this document may be reproduced or utilized in any form without the
written permission. Discuss GATE 2011 question paper at www.gatementor.com. 21
EC-Paper Code-B GATE 2011 www.gateforum.com

Answer: - (C)
1
Exp: - rdon α
t oh
At VGS= 0, t ch = 10µm ; ( Given rd = 600Ω )
10
rd = × 600 ← at 8µm = 750Ω
8

Common Data Questions: 50 & 51

100
The input-output transfer function of a plant H ( s) = . The plant is
s ( s + 10 )
2

placed in a unity negative feedback configuration as shown in the figure below.

r u 100 y
Σ H (s ) =
+ s ( s + 10 )
2

− plant

50. The gain margin of the system under closed loop unity negative feedback is
(A) 0dB (B) 20dB (C) 26 dB (D) 46 dB
Answer: - (C)
100
Exp: - H ( s) =
s ( s + 10 )
2

 ω
Phase cross over frequency= −90 − 2 tan−1   = −180
0

 10 
 ω  ω
⇒ −2 tan−1   = −900 ⇒ tan−1   = 45 ⇒ ω = 10 rad / sec
0

 10   10 
100 1 1
(H ( jw) ) = = =
j10 ( j10 + 10 )
2
10.2 20

1
GM = 20 log = 20 log20 = 26dB
1 / 20

51. The signal flow graph that DOES NOT model the plant transfer function H(s) is
−100
1 1/s 1/s 1/s 100
u y
(A) (B) 1/s 1/s 1/s 100
u y
−10 −10

−100 −20

−100
1/s 1/s 1/s 100
u y
(C) (D) 1/s 1/s 1/s 100
u y
−20

© All rights reserved by Gateforum Educational Services Pvt. Ltd. No part of this document may be reproduced or utilized in any form without the
written permission. Discuss GATE 2011 question paper at www.gatementor.com. 22
EC-Paper Code-B GATE 2011 www.gateforum.com

Answer: - (D)
Exp: -(D) Option (D) does not fix for the given transfer function.

Linked Answer Questions: Q.52 to Q.55 Carry Two Marks Each

Statement for Linked Answer Questions: 52 & 53

In the circuit shown below, assume that the voltage drop across a forward biased
diode is 0.7 V. The thermal voltage Vt = kT / q = 25mV. The small signal input
vi = Vp cos ( ωt ) where Vp = 100mV .

9900Ω
+

12.7V +

IDC + iac
VDC + v ac

vi

52. The bias current IDC through the diodes is


(A) 1 mA (B) 1.28 mA (C) 1.5 mA (D) 2 mA
Answer: - (A)
12.7 − ( 0.7 + 0.7 + 0.7 + 0.7 )
Exp: - IDC = = 1mA
9900

53. The ac output voltage vac is


(A) 0.25 cos ( ωt ) mV (B) 1 cos ( ωt ) mV

(C) 2 cos ( ωt ) mV (D) 22 cos ( ωt ) mV

Answer: - (C)
ηVT 2 × 25mV
Exp: - AC dynamic resistance, rd = = = 50Ω
ID 1mA
η = 2 for Si (∵ forward drop = 0.7V)

The ac dynamic resistance offered by each diode = 50Ω


 4 × 50Ω   100 
∴ Vac = Vi ( ac )   = 200 × 10−3 cos wt  
 9900 + 50  10000 
Vac = 2 cos ( wt ) mV
© All rights reserved by Gateforum Educational Services Pvt. Ltd. No part of this document may be reproduced or utilized in any form without the
written permission. Discuss GATE 2011 question paper at www.gatementor.com. 23
EC-Paper Code-B GATE 2011 www.gateforum.com

Statement for Linked Answer Questions: 54 & 55

A four-phase and an eight-phase signal constellation are shown in the figure


below. Q
Q
d
r2
d r1 I I

54. For the constraint that the minimum distance between pairs of signal points be d
for both constellations, the radii r1, and r2 of the circles are
(A) r1 = 0.707d, r2 = 2.782d (B) r1 = 0.707d, r2 = 1.932d
(C) r1 = 0.707d, r2 = 1.545d (D) r1 = 0.707d, r2 = 1.307d
Answer: - (D)
Exp:- For 1st constellation 450

r12 + r12 = d2 ⇒ r12 = d2 / 2 ⇒ r 1 = 0.707d r2

For 2nd constellation r2

d
= r2 cos 67.5
2 67.50

r2 = 1.307d d/2 d d/2

55. Assuming high SNR and that all signals are equally probable, the additional
average transmitted signal energy required by the 8-PSK signal to achieve the
same error probability as the 4-PSK signal is
(A) 11.90 dB (B) 8.73 dB (C) 6.79 dB (D) 5.33 dB
Answer: - (D)
(0.707d)
2
r12
Exp: - Energy = r12 and r22 ⇒ =
(1.307d)
2 2
r2

(1.307)
2

Energy (in dD) = 10 log = 5.33dB


(0.707 )
2

Q. No. 56 – 60 Carry One Mark Each

56. There are two candidates P and Q in an election. During the campaign, 40% of
the voters promised to vote for P, and rest for Q. However, on the day of election
15% of the voters went back on their promise to vote for P and instead voted for
Q. 25% of the voters went back on their promise to vote for Q and instead voted
for P. Suppose, P lost by 2 votes, then what was the total number of voters?
(A) 100 (B) 110 (C) 90 (D) 95
Answer: - (A)
© All rights reserved by Gateforum Educational Services Pvt. Ltd. No part of this document may be reproduced or utilized in any form without the
written permission. Discuss GATE 2011 question paper at www.gatementor.com. 24
EC-Paper Code-B GATE 2011 www.gateforum.com

Exp: - P Q
40% 60%
−6% + 6%
+15% − 15%
49% 51%
∴ 2% = 2
100% = 100

57. Choose the most appropriate word from the options given below to complete the
following sentence:
It was her view that the country's problems had been_________ by
foreign technocrats, so that to invite them to come back would be
counter-productive.
(A) Identified (B) ascertained (C) Texacerbated (D) Analysed
Answer: - (C)
Exp: -The clues in the question are ---foreign technocrats did something negatively to
the problems – so it is counter-productive to invite them. All other options are
non-negative. The best choice is exacerbated which means aggravated or
worsened.

58. Choose the word from the options given below that is most nearly opposite in
meaning to the given word:
Frequency
(A) periodicity (B) rarity
(C) gradualness (D) persistency
Answer: - (B)
Exp: - The best antonym here is rarity which means shortage or scarcity.

59. Choose the most appropriate word from the options given below to complete the
following sentence: Under ethical guidelines recently adopted by the
Indian Medical Association, human genes are to be manipulated only to
correct diseases for which______________ treatments are
unsatisfactory.
(A) Similar (B) Most (C) Uncommon (D) Available
Answer: - (D)
Exp: - The context seeks to take a deviation only when the existing/present/current/
alternative treatments are unsatisfactory. So the word for the blank should be a
close synonym of existing/present/current/alternative. Available is the closest of
all.

60. The question below consists of a pair of related words followed by four pairs of
words. Select the pair that best expresses the relation in the original pair:
Gladiator : Arena
(A) dancer : stage (B) commuter: train
(C) teacher : classroom (D) lawyer : courtroom
Answer: - (D)
© All rights reserved by Gateforum Educational Services Pvt. Ltd. No part of this document may be reproduced or utilized in any form without the
written permission. Discuss GATE 2011 question paper at www.gatementor.com. 25
EC-Paper Code-B GATE 2011 www.gateforum.com

Exp: - The given relationship is worker: workplace. A gladiator is (i) a person, usually a
professional combatant trained to entertain the public by engaging in mortal
combat with another person or a wild.(ii) A person engaged in a controversy or
debate, especially in public.

Q. No. 61 – 65 Carry Two Marks Each

61 The fuel consumed by a motorcycle during a journey while traveling at various


speeds is indicated in the graph below.

120
(kilometers per litre)

90
Fuel consumption

60

30

0
0 15 30 45 60 75 90
Speed

(kilometers per hour)

The distances covered during four laps of the journey are listed in the table below
Lap Distance (kilometers) Average speed
(kilometers per hour)
P 15 15
Q 75 45
R 40 75
S 10 10
From the given data, we can conclude that the fuel consumed per kilometre was
least during the lap
(A) P (B) Q (C) R (D) S
Answer: - (A)
Exp: - Fuel consumption Actual
15 1
P 60 km / l = l
60 4
75 5
Q 90 km / l = l
90 6
40 8
R 75 km / l = l
75 15
10 1
S 30 km / l = l
30 3

© All rights reserved by Gateforum Educational Services Pvt. Ltd. No part of this document may be reproduced or utilized in any form without the
written permission. Discuss GATE 2011 question paper at www.gatementor.com. 26
EC-Paper Code-B GATE 2011 www.gateforum.com

62. Three friends, R, S and T shared toffee from a bowl. R took 1/3rd of the toffees,
but returned four to the bowl. S took 1/4th of what was left but returned three
toffees to the bowl. T took half of the remainder but returned two back into the
bowl. If the bowl had 17 toffees left, how many toffees-were originally there in
the bowl?
(A) 38 (B) 31 (C) 48 (D) 41
Answer: - (C)
Exp: - Let the total number of toffees is bowl e x
1
R took of toffees and returned 4 to the bowl
3

1
∴ Number of toffees with R = x−4
3

2
Remaining of toffees in bowl = x+ 4
3

1 2 
Number of toffees with S = x + 4 − 3
4  3 
3 2 
Remaining toffees in bowl =  x + 4 + 4
4 3 

1  3 2  
Number of toffees with T =   x + 4 + 4 + 2
2  4 3  
1 3  2  
Remaining toffees in bowl =  x + 4 + 4 +2
2  4  3  
1 3  2   3 2 
Given, 
2  4  3
x + 4  + 4  + 2 = 17 ⇒
4  3 x + 4  = 27 ⇒ x = 48
   

63. Given that f(y) = | y | / y, and q is any non-zero real number, the value of
| f(q) - f(-q) | is
(A) 0 (B) -1 (C) 1 (D) 2
Answer: - (D)
y q −q −q
Exp: - Given, f(y) = ⇒ f ( q) = ; f ( −q ) = =
y q −q q

q q 2q
f ( q) − f ( q ) = + = =2
q q q

64. The sum of n terms of the series 4+44+444+.... is


(A) ( 4 / 81) 10n+1 − 9n − 1 (B) ( 4 / 81) 10n−1 − 9n − 1

(C) ( 4 / 81) 10n+1 − 9n − 10  (D) ( 4 / 81) 10n − 9n − 10

© All rights reserved by Gateforum Educational Services Pvt. Ltd. No part of this document may be reproduced or utilized in any form without the
written permission. Discuss GATE 2011 question paper at www.gatementor.com. 27
EC-Paper Code-B GATE 2011 www.gateforum.com

Answer: - (C)
4
Exp: - Let S=4 (1 + 11 + 111 + ……..) = (9 + 99 + 999 + .......)
9

=
4
9
{ ( ) (
(10 − 1) + 102 − 1 + 103 − 1 + ......... ) }
4  10n − 1  ( )
=
4
9
{(
10 + 10 + ......10 − n = 10
2 n

9
) } 9
− n =
4
10n+1 − 9n − 10 { }
  81

65. The horse has played a little known but very important role in the field of
medicine. Horses were injected with toxins of diseases until their blood built up
immunities. Then a serum was made from their blood. Serums to fight with
diphtheria and tetanus were developed this way.
It can be inferred from the passage that horses were
(A) given immunity to diseases (B) generally quite immune to diseases
(C) given medicines to fight toxins (D) given diphtheria and tetanus serums
Answer: - (B)
Exp: - From the passage it cannot be inferred that horses are given immunity as in (A),
since the aim is to develop medicine and in turn immunize humans. (B) is correct
since it is given that horses develop immunity after some time. Refer “until their
blood built up immunities”. Even (C) is invalid since medicine is not built till
immunity is developed in the horses. (D) is incorrect since specific examples are
cited to illustrate and this cannot capture the essence.

© All rights reserved by Gateforum Educational Services Pvt. Ltd. No part of this document may be reproduced or utilized in any form without the
written permission. Discuss GATE 2011 question paper at www.gatementor.com. 28
!" #$
% &" '$ ( )" $
% *" +$
#
! , &"

- , . /( )= ( )=
+

0 ( )≤+ =0 =
+
≤ ≤
+
0 ≤ ≤
+
=
+
$
+ $
+
# # '
= = × =
− + − + % % (

+ 1 = −
−π $+ π $+
!" &" )" *"
! , !"

( )

- , 2 3 / = − =

π
θ π π
4 5 =6 θ+ θ +
=6 + =
+ +

∴ () = ( π$
+
) = −π $
+

+
# 2 (7) = − 1 ) 6 6 65 7 6
7+ 7+#

7+ = (7) 7
+π8 )

!" + &" )" *" +


! , )"

7
- , (7 ) 7= 7− 7
+π ) +π ) 7+ ) 7+#
1 1+

7=− 9 6 73 # 6

7
& 6 6 : 9 1+ = 7=;
) 7+#

∴1 = 7 = /1 − 1+ =
) 7+

© All rights reserved by Gateforum Educational Services Pvt. Ltd. No part of this booklet may be reproduced or utilized in any form without the
written permission.
% 1 66 6 9 6

Ω 8 Ω
∠ ;!
∠ ;@ ∠ ;@
? ?
+ +
∠ ;!

−8 Ω Ω

+ −
!" ! &" ! )" ! *" ; !
+8 +8 +8
! , )"
- ,

Ω 8 Ω

;!
;@ ;@
;@ 8 Ω Ω
− + + −

;!
−8 Ω Ω

1< = ;× = !
+8 +8

= 6 5 9 + 9 66

5Ω ''

' 5Ω
;;Ω
+

!" =; Ω &" ;; Ω )" =>Ω *" ; 5Ω


! , !"
- ,
( 1 + '' ) 1

5
+ + +
'' '5Ω @ @+ ;;Ω @
'5 − − '' −

;;Ω

© All rights reserved by Gateforum Educational Services Pvt. Ltd. No part of this booklet may be reproduced or utilized in any form without the
written permission.
! 6 6 9 9 6 @
@ = @+ ( )( )
;5 − = ;; ( 1 + '' + )/
;;;; = ;;1 + ;; × ;; = ;;1 + ;;;;
;; 1
−+;;;; = ;;1 =− 1= −
+;;;; +;;
−1
@ = ;; 1 + '' + = ;; 1 + ;; = =;1
+;;
@ =;1
A = = = =;Ω
1 1

( ! 6

2( )=
( +
+' )( + +)
( + )( + #) ( + %)

6 (ω ) 7

!" ω = $ &" ω = + $
)" ω = # $ *" ω = % $
! , )"
- , B

( ) = 2 ( 8ω) (ω + 2 ( 8ω ) )
() 7
2 ( 8ω ) 6 7

2 ( 8ω ) =
( −ω+
+' ) ω+ + %
ω+ + ω+ + ' ω+ + (
ω=# $ 6
2 ( 8ω ) = ; ( )=;

C 1 6 6 "3 (+ # % =) 6

!" &" D D

)" D D *" D D D D
! , !"
- ,
7
;; ; ;
; ; ;

; ;

1 6 7 7 7 7
6

© All rights reserved by Gateforum Educational Services Pvt. Ltd. No part of this booklet may be reproduced or utilized in any form without the
written permission.
E 1 3 #"F F
$ $
+" 9 6 9 6 AG)" D
D

!" < 7 <# &" < 7 <


# # +

)" < 7 <# *" < 7


+ #
! , )"

- , = ( #) −
+

( $#) AG)
#
< 7 <#

( $+ ) AG) 7 >
+
6 AG) < 7 <#
+

' 6 6 9
H ( θ) = 6 %
θ/; ≤ θ ≤ π $+

6
!" ; & &" + ( & )" = & *" E &
! , !"
H (θ)
- , * 6 =
H ( θ)

H (θ) =
+ π π $+
H θ = 6 θ θ θ φ
%π ; ;
;
= +π − %
=
%π ;

* 6 = = ;

;
= ; 9;= ; &

; ! 6 6 +%
6 6 ; E' 2
;−'
µ ; = % π × ; −C I $ ε; = H$ 6 6 6 6 6
#(π
!" ##; Ω &" ;; Ω )" %# #Ω *" %# % Ω
! ,

© All rights reserved by Gateforum Educational Services Pvt. Ltd. No part of this booklet may be reproduced or utilized in any form without the
written permission.
- , ) 6 6 6
µ µ +%
D; = =
+π ∈ +π ∈
B D; = = # Ω

! ! "
"# $%&

! 6 6 J
9 ! 6 6 6
ε 6 6 ε !
6 9 6
!" 6 &"
)" 6 J3+ *" 6
! , *"
- , - K
1 9 9 6

+ 6 6 66 9 "
6 *
) () *+

+
6 (ω ) ? * )+

!" 6 (ω ) − &" (ω )
)" − 6 (ω ) *" − (ω )
! , )"
- ,
) @( ) *+

6 )+
*

) 0 5A 6

4 6 6 ω " 6 9 6 6 5 ;
9 5 L+ B 6 ω " L

© All rights reserved by Gateforum Educational Services Pvt. Ltd. No part of this booklet may be reproduced or utilized in any form without the
written permission.
1 +

π
; π 6
#π +π

+
+

@( ) ; 6 ( )−

−+

# 1 66
=@

&

! &

!" = ! & + ) &" = ( ! + &) 6

)" (
= ! + & ) ) *" = !& + )

! , !"
- , = ! + &) = ! + & + ) = !& + )

% 4 6 " 3 ;= K
+ =

+
+
!" = − &" = − )" = *" =
+ + + +
! , *"

- , 2 *- + = + =

1H = = 9
= / (1H ) = ( 1H )
+
= ⋅ = +6
/2 ( )=;= ;== +6 6= ;
+ +
+
∴ K 3 =
+ +
© All rights reserved by Gateforum Educational Services Pvt. Ltd. No part of this booklet may be reproduced or utilized in any form without the
written permission. '
= < 6 " +
< 6
+ +
"
+ +
!" − &" −
( ) ( )
+ +
+ +
+ + + +

+ +
)" *"
( ) ( )
+ +
+ +
+ + + +

! , *"

- , 1 ( ) ↔ H( ) ( )↔− H( )

H( )= +
+ +
+ +
( )→− =
+
+ + +
+ +

( 9 6 ( % − 8#) Ω 6
=6 ( ;;π + ;; ) !
!" %% +4 &" =;4 )" (+ =4 *" +=4
! , &"
- , D = % − 8# = A < − M ) /A < = %/1 3 =6 ( ;;π N ;; ) = 1 6 (ω + α)

0= 1 +A < = × =+ × % = =;4
+ +

C 1 9 9 ) )+ 6 6 ) 6 9 +
@ 6 B 6 3; 6 "

B =;

) )+
()

!" 7 &" 6
)" 6 9 6 *" 6
! , *"
- , 4 6 6 3;
) 6 ) 6 9 )+ 9 6 9 6 ) )+
6 66 6 9 9 6 9 9 6
7
6 5 6

© All rights reserved by Gateforum Educational Services Pvt. Ltd. No part of this booklet may be reproduced or utilized in any form without the
written permission. (
E 6 6 6 66 9
υ−; C
! υ ≥ ; C@
= =;;
; ! υ < ; C@

>Ω

+ +
;@ @
− −

6 66
!" ; ! &" ' # ! )" ( (C ! *" ( + !
! , *"
@−; C
=
=;;
=
@ =;;
= =;;Ω
B 6 9 6 ; C@

;−; C ; C@
=
;;; + =;; ;@ =;;Ω
'#
= =(+ !
=;;

' + 6 96 + !
9 + & 6 6
96

!" % &" ( )" E *" ;

! , &"
- , 1 ! 1 &

!+ ! &+ &
; ; ; ; ;
; ; ; ;
; ; ; ;
; ; ;
; ; ;

© All rights reserved by Gateforum Educational Services Pvt. Ltd. No part of this booklet may be reproduced or utilized in any form without the
written permission. )
; ; ;
; ; ;
; ;
; ; ;
; ;
; ; ;
; ;
; ;
;
;
;
(6 96

+; ) 9 66

)5

O
&

1 66 6
!" 66
&" 66 )<> 3 ;
)" 66 )<> 3 !3&3
*" 66 )<> 3 !3&3;

! , !"
- , !

)5

O
&

O = ! )<>O
= ! )<>+ O
O = ! )<>+ O
1 )<> 3 ! &3
O =
J 6
O =
1 )<> 3 !3&3;

© All rights reserved by Gateforum Educational Services Pvt. Ltd. No part of this booklet may be reproduced or utilized in any form without the
written permission. *
O =O
J 6
O =O
6 66 66

+ 6 6 6 9 6 6 9
6 9 (
- = ; P + 8P7 ) − 8+=
K 6
7 6
!" + 2I7 66 &" % I7 66
)" + 2I7 9 66 *" % I7 9 66
! , !"

- , = +=
λ

λ=
+=
-7
# × ;E
= = + 2I7

+=
-
< - =6 ω
π
-7 = 6 ω +
+
J 6 QR 66 7

++ ! 9 9 (
- = EP + (P − =P7 ) 8( ω + # − % )@ $

6 6 6 6 9 ≤; -
6

(
!" −EP − (P − =P7 ) 8( ω + # + % )@ $

(
&" −EP + (P − =P7 ) 8( ω + # + % )@ $

(
)" −EP − (P − =P7 ) 8( ω − # − % )@ $

(
*" −EP + (P − =P7 ) 8( ω − # − % )@ $

! , )"
- = E ( +( += 7 ) 8( ω + # − % )@ $

- ( 6 6 6 9 )
>; =; <;
© All rights reserved by Gateforum Educational Services Pvt. Ltd. No part of this booklet may be reproduced or utilized in any form without the
written permission. +
- 6 6 66 6 3;
∴- =;

+ H - - 6 6
0 3;
∴- +- = ;
8( ω − # − % )
- = - = − (E +( + = 7)

+# 1 6 6 5 K 6 #=;; I7
9 9 . 9 6 C=S 6
6 9 9
6
!" C=; &" +(+= )" %;;; *" =+=;
! , )"

- , & = A B (β + )
+
AB → B
+×β
AB =
β+
β = ; C=
+ × #=;;
AB = = %;;; $ 6
+ ; C=

+% 6 6 " K 6
- +
() - () 6

!" (;;; $π ;

&" (%;; $π ;

)" (%;; $π +; $ π + ( )

*" (;;; $π +; $ π + ( )
! , &"
- , 0B* "
(
%;; δ ω + ;% ) (
%;; δ ω − ;% )
(

− − ; −'
( )
' ;
;# $ 6

© All rights reserved by Gateforum Educational Services Pvt. Ltd. No part of this booklet may be reproduced or utilized in any form without the
written permission.
- +
() =A (;)

A ( ; ) = +π B ( ω) ω
−∞

A ( τ ) ↔ B ( ω)

= × + × ;# × ( + × + × ;# × ( + %;; + %;; = (%;; $π


+π + +
B 6 0B* " 6 *) 6 "
7

+= 6 9 6 +;
6 ( ;;;; π ) ! ! #;; > π 9

!" +=; Ω &" +C = Ω )" += Ω *" ++ = Ω


! , )"

@ += @
- , π = = = +=Ω
1& !

' ,

+( 2
−= −# ;
! = 1= !#
+ ; ;
!" = ! + +1 &" ' ! + #;1 )" C ! + =1 *" C! + + 1
! , &"
= #
- , 2 , !3 /
+ ;
−= − λ −#
) 6 6 K ! ! 1λ = ; =;
+ ;−λ
( −= − λ ) ( −λ ) + ( = ;
=λ + λ+ + ( = ;
λ = −=λ − (
+
λ = −=λ+ − (λ = −= ( −=λ − ( ) − (λ
#

λ = +=λ − (λ + #; = 'λ + #;
#

- 6 6 6 K
∴ !# = '! + #;1

© All rights reserved by Gateforum Educational Services Pvt. Ltd. No part of this booklet may be reproduced or utilized in any form without the
written permission.
# +
+C "3 L' N +% N = (
!" + &" += )" % *" %(
! , )"
- 0, 2 ( )= #
−' +
+ +% + =

:( )=; # + − E + +% = ; 3+ %
T( ) = ( − E/ T+ ( ) = + − E < ;/ T% ( ) = +% − E > ;
I 6 ( ) 3+
∴ +# − ' × ++ + +% × + + = = +=
& (
∴ ( ( ) = (# − ' × (+ + +% × ( + = = %

+E 1 @! − @& = ( @ @) − @*

!" =@

&" +@

)" #@

*" (@
! , !"
@! − @& (
- , 1= = = #!/B 6 6 9 5
+ +
9 @) − @* 6 1 3 #!
A @! +Ω @& A

A
A
A − ;@
A Ω
+
A

Ω @) @*
=@
+!

@) @*
≡ @)
− + #!
@*

+! +@

@* = + + # + @) = = + @) /@) − @* = −=@

+' 9 9 !@ 66

!" ! @ ≈ +;;

&" ! @ ≈ ;;

)" ! @ ≈ +;

*" ! @ ≈ ;

© All rights reserved by Gateforum Educational Services Pvt. Ltd. No part of this booklet may be reproduced or utilized in any form without the
written permission.
! , *" @

- ,
;;5 +5

;5
;;5
−!

>@< # C ( 1) + 1& ) +5 − ;;5 ( 1& ) − ; C = ;


1& = ' 'µ!/1) = β1& = ; '' !/1- = !
∴ =
+( !
= +(Ω/7 = β = + (5Ω/ ∴ ! =
( ;;5 FF +5 ) = % +
1- +(
;;5 7: ++
7 := 7 F F = ++ Ω/! = ! = ( % +)
+% + +
7 :A ++ + ;5
! ≈ ;

#; 9 966 6

! , *"
!=; =O
- , != 9
!= =O

!=;
!= ! =; !=
9 99
O=; O=

! =;
© All rights reserved by Gateforum Educational Services Pvt. Ltd. No part of this booklet may be reproduced or utilized in any form without the
written permission.
# < 6 9 3 $+"
9 6 K 6 1 ; 3 3 $
+ 9 K
!" ; &" $+ )" *" #$+
! , !"
- ,
∞ 5
= 9( − 5 )
5 =;
+
∞ 5
; = 9( −5 ) =
5 =;
+
G
9( ; ) =
+
B 6 9 " ) K 6
9( ; ) =
9− " 9−+" =;
∞ 5
= 9 −5
5 =;
+
;
9 + 9( ; ) =
+ + +
9 =;

#+ 66

!" = $
# &
(A + A+ ) )

&" 9 # & = $
A )

)" # & = $
A )

*" 9 = $
# &
(A + A+ ) )
! , &"

A+
- , @; B" = − υ ( )
A +
)B
A +)B
@G ( B ) = − @ (B)
(A )B + )
6 K 6 9
A )
© All rights reserved by Gateforum Educational Services Pvt. Ltd. No part of this booklet may be reproduced or utilized in any form without the
written permission.
## 9 6 9 9 6 6 9
9 6 6 9
I7 = # 6 (+ ;'% × ;+ ) 6 (+ ( E × ;+ ) 6 ( ( +E# × ; ;
− β7 )

6 @ 9

!" >6 &" =6 )" ; < <6 *" =;

! , *"
π π
- , I7 = IG 6 6 6 (ω − β7 )
β
2

I7 = # 6 (+ ;'% × ;+ ) 6 (+ ( E× ;
+

) 6 (( +E# × ; ;
− β7 )
) 9 K
9 3+ 3
= ; 2I7
+ + ; + +
) #× ; +
) = + = + = = 2I7
+ + # +
9 9 6 @0 3 ;

#% 9 " 5
6 " K 6 5 K 6 6 "
9 6 K 6

5 $
5 $
I7"
!" Eπ &" %π )" +π *" π
! , &"
- , 1
U 0 3> " => +

1 H K 6
+
U 0 3 +π 5 + = +π 5 × %

© All rights reserved by Gateforum Educational Services Pvt. Ltd. No part of this booklet may be reproduced or utilized in any form without the
written permission. '
50
J >0 + = +π > × % = %π
5

#= ! 66 &B)" $
E 1
6 0 3 ;" 3 '$ ;
6
!" C$
E; &" (#$
E; )" '$ ; *" $ ;
- .
- , $E
= =
C $E

C $E
=; =;
$E

( ) = (;) ( ;) + ( ) (; )
' C (# C C;
= × + ×C = + = = C
; E ; E E; E; E; E

#( 1 )UGB 6 6 6 K U
U+ K 7 6 U 9

!" @ < EC=@ &" EC@ < @ < # += @


)" @ > # +=@ *" ; < @ < =@
! , !"
- , 9 6 6 6 )UGB ,
@
U −
@GI = @**
U+ − B

U+ −
U+ − B

@
@1< @1I @**

@1< = (#@** + +@ ) = + +=@ I 6 G Q


!R
E
6 6
© All rights reserved by Gateforum Educational Services Pvt. Ltd. No part of this booklet may be reproduced or utilized in any form without the
written permission. (
#C ! 6
K
!" $
# &" $
+ )" +$
# *" V
! , )"
- , 0( ) = 0 ( I) + 0 ( I) + 0 ( I) +
# =

= + + +
+ + +
+ %

= + + +
+ + +
+
% +
= + + + = = × =
+ % % + + # #

%

#E 6 6 ! 9 ! =5
+ + + +
= + +7 5 6 6 ∇! = ;
!" + &" + )" *" ;
! , !"
- , 4 5 ∇ != +


( +
! )
J ∇ != +


( +
! )
= + ∂

(5 ) =
I+ 5
+
( + +) +

=5 ( + +) +

∴H ∇ !=; ( + +) = ; = −+

#' ) K
+
( ) ++ ( )+
+
( ) = δ( ) () = ;−
= −+ =;
= ;−

6
= ;+

!" + &" )" ; *"


! , *"
+
( )++ ( )+
- , + ( ) = δ( )
) 9 −
+
( )− ( ;) − :( ;) + + ( ) − (; ) + ( )=
+
++ + ( )++ +% =

© All rights reserved by Gateforum Educational Services Pvt. Ltd. No part of this booklet may be reproduced or utilized in any form without the
written permission. )
−# − +
( )=
( +
++ + )
H 6
( ) = −+ − − −
()
( ) =+ − + −
− −

() =+− =
= ;+

%; ! 9 9 6 A 6
66 ! 66 &

!" ; E Ω &" %Ω )" + Ω *" + E Ω


! , !"
C ( + ;A %+ + C;A
- , 0 66 ! 66 ! 3 @13 =
A ++ (A + + )
+
AN+

;−# C
1= =
++A ++A
CA ( + ;A
@3#N1A3#N =
+NA ++A
0 (A + + ) ( C; ) − ( %+ + C;A ) + (A + + )
+

= =;
(A + + )
%
A

+Ω A + A
1
+
+ + + +
;@ −8 Ω

#@ ≡ ;@ @

#@
− −

66 ! 66 &

C; (A + + ) = ( %+ + C;A ) + (A + + )
+

= (AN+ ) = + ( # + =A )
=A + ; = ( + ;A
%3=A
A3; EΩ

© All rights reserved by Gateforum Educational Services Pvt. Ltd. No part of this booklet may be reproduced or utilized in any form without the
written permission. *
% 6 < 1 9

; ; ;
+ = ; ; + + + ;
# ; ; #
#

= ( ; ;) +

6
!" ≠ ; + =; # ≠; &" = ; + ≠; # ≠;
)" = ; + ≠; # =; *" ≠ ; + ≠; # =;
! , *"
- , 6
= & !& !+&

; ;
!= ; ; +

# ; ;

;
&= ;

; ; +
6 = ; + ;
; ;

6
≠;
+ ≠;
6
# 6 7

%+ 9 " I ( 8 ω ) 3 (+ 6 ω) ( +ω) $ω
(;)
!" $
% &" $
+ )" *" +
! , )"
- ,

+
= ()
:
ω

−+ +

© All rights reserved by Gateforum Educational Services Pvt. Ltd. No part of this booklet may be reproduced or utilized in any form without the
written permission. +

+6 ω
ω ( )= ( −
: ) + :( + )
8ω − 8ω +ω
+
ω
+
−# #

%# 65 6 + $

!" > 3 + 3 ; C= &" > 3 # 3 ; C=


)" > 3 % 3;= *" > 3 + 3;=
! , !"
- , + 2 ( B ) I ( B ) = B# + +
+ (+ + 5 ) + +5
#
(+ + 5 )
+
(+ + 5 )
(+ + 5 ) − (+ + 5 ) ;
( + 5)
6
(+ + 5 ) − ( + 5) = ;
+5
=
++5
+5 +5
!- +
+ ( + 5) = ; = =+ = ++5 = % 5=+

= ; C=

%% " " " 3

−∞
( τ ) 6 (#τ ) τ

!"
&"
)"
*"
! , &"

- , "= τ" 6 #τ" τ


−∞

B 6 " " 6

© All rights reserved by Gateforum Educational Services Pvt. Ltd. No part of this booklet may be reproduced or utilized in any form without the
written permission.
< " 5

"= >6 #τ" τ < ∞


−∞

" B

%= ! 6 6 6 6 ;; Ω 6
=; Ω 6 +;; Ω 6 1 6 9 %+' UI7
2I7 9 6
!" E+ = 6 &" ;= )" =E *" C=
! , &"
- , ) 6 6 6 3 ;;Ω
6 6 3 =;Ω +;;Ω/ K 6 3 %+'UI7 2I7
( U 6
λ
9 6
9 9
9 3 λ$
%
)

=;Ω ;;Ω +;;Ω

6 9
6

λ )
%+'UI7 = = = ; C= /
% %
λ )
2I7 + = + = = ; ;C=
% %+
< 9 9 +
∴ 3 <)U + = ; =+=
I 6 ;=

%( ! &0B> 6 9 !42J 6 6
+-
J; $+ K 9 ( )= ( ω6 )
+-
+ ( )=− ( ω6 ) (; ) 1 6 6

6 6 %=G 6 6 9
9
+- - - -
!" O &" O )" O *" O
J; J; +J; %J;
! , &"
J
- , 2 , &0B> 6 !42J 6 J 6
+
-K 9
+- +-
( )= ( ω6 ) / + ( )=− ( ω6 )
6 6 6 6 %=W 6
6 9
© All rights reserved by Gateforum Educational Services Pvt. Ltd. No part of this booklet may be reproduced or utilized in any form without the
written permission.
2 6 6 6 6 9

+
φ ( )= ω6 ; ≤ ≤ 6 6 %=W

+
φ ( + %=W) = ( ω6 + %=W) ; ≤ ≤

A 6 6 ,

( ) +
( 9 6 )

φ ( + %=W)

+- +
= ( )φ ( + %=W) = ω6 ⋅ ( ω6 + %=W)
; ;

-
= /
+
-
+ =−
+
-
5 9
+
- - -
= + J/0 + 6
= 0 ( γ < ;) = 0 +J< ; =0 J< −
+ + +

1
6 9 6 9
+

1A
- -

+ +

-
5 9
+
-
= + J/
+

- -
0 + 6
= 0 ( γ < ;) = 0 +J< ; =0 J< −
+ +

11 2 A @
- J;
6
+ +

1A
- -

+ +

© All rights reserved by Gateforum Educational Services Pvt. Ltd. No part of this booklet may be reproduced or utilized in any form without the
written permission.
+
-
+ -
+
+ +
− +
; J ; −
- + ;
0 J< = +
= J;
/
+ −∞ J −∞ πJ;
+π ;
+

-
+ ∞ +
+ J; - − -
< = / 3 0 J< = +
=O
J; + -
+π J;
J;
+
( 9 6 ! 2 0 )
B K 0( )= 0 6
+0 + 6

+ +

- - -
∴ 0( )= O + O =O
+ J; + J; J;

%C 6 6 ( = ; ;
6 #
) 6 UGB
' #
Kµ µ 1 ; $6
6 9
C
!" ; &" ;; )" ; *" ;
! , *"
+
−#
- , = '
= ;6 / 3 ; E #
= ;− + 6 #

;
∴ 3 ; ≈;

// 0 1 )2 *

1 6 6 UGB
δ = +;
µ
8 6 ;
B B G+ 6 C #'
ε; = E ' × ; − +
H$

© All rights reserved by Gateforum Educational Services Pvt. Ltd. No part of this booklet may be reproduced or utilized in any form without the
written permission.
%E 6 8 6 6 6 6
!" + H &" C H )" + H *" C H
! , !"
C × µ × ; +µ × E ' × ; +
- , B 6− 8 6 6 6 6 3 =+H
;

%' 9 6 6 6 6
!" ; C H &" ; C H )" ; #= H *" ; +% H
! , !"
ε! ε ( B G+ ) × ! # ' × E ' × ;− +
× µ × +;
- , )= /)9 = = =;C H

// 0 1 +2

4 ;@ 66 6 ! 6 6 5
9 ,
" Ω 6 6 & 6 #!
" + =Ω 6 6 & 6 +!

=; H 5 (@ 66 6 ! Ω 6 6 &
C$
# ! 1 E@ 6 6 6 ! 66 9 &
!" (@ &" C@ )" E@ *" '@
! , &"

= 4 ;@ 66 6 ! 6 CΩ 6 6 &
!" #$
C! &" =$
C! )" ! *" '$
C!
! , )"
- , 9 5 6 6 K @ A
9 A 6
J 6 K
@ 3 #A N#
@ 3 +A N=
B 9 K 9 A 3+ @ 3'
J 9 K 6 5
' 3 1 × +NC × 1 13 !

© All rights reserved by Gateforum Educational Services Pvt. Ltd. No part of this booklet may be reproduced or utilized in any form without the
written permission.
3 ! 1 ,

4 / 3 ! 1 2

+
6 6 9 2) ( )=
+
=+ 2) B" 6
!" 3 3+ &" 3# 3+
)" 3 # 3 *" 3# 3
! , !"
− ω − ω
- , φ= −
β
φ +

− ω − ω
>
β
<
!" )"
)" 7
AIB
G !" 9

=# 6
!" + $ &" # $

)" ( $ *" $# $
! , !"
- , X3 9 6 6 K 6
= +× = + $ 6

4 / 3 ! 1 2

! 9 6 66
8

=% 9 6 6 6
!" < $+ > &" ; < $ >

)" < $ > *" ; < $+ >


! , )"
- , H 9 ; ≤ ≤
9! R6 6

© All rights reserved by Gateforum Educational Services Pvt. Ltd. No part of this booklet may be reproduced or utilized in any form without the
written permission. '
I = 1 6 = M

16 6 = M π ( +
)
I ( +π ) = Mπ +

M
I=
+
9 6 6
9 >

Iφ (+π ) = Mπ ( ) +

Mπ + M+
Iφ = =
+π +

U 9 6

== ! Y " 9 9 6
6

9 6
!"
&"
)"
*"
! , &"

' '+

=( 4 6 9 6 9 9
Z
3 1!
!" - 9 &" H )" ) 6 *" U 6
! , &"
© All rights reserved by Gateforum Educational Services Pvt. Ltd. No part of this booklet may be reproduced or utilized in any form without the
written permission. (
=C G ! & ) *" 6 9 6 -AAGA
4 6 9 55 ,Z
61 ! 1"
!# .7 ! 7 . ! !
/
!" K &" 9
)" 9 *"
! , &"

=E 1 ( ;; ) +=' = # =+ ( ;; )+;(+ = C E= ( ;; )##+ =

!" + +# &" % +# )" #C *" +C (%


! , *"
- , ;; 3
+='
= # =+ +;(+
= C E=
##+
= +=' +;(+
= # =+ C E= = +C (%

=' ) 9
6 9 6,

! "
! ! " ! 8 999999999 / 1
#"
!" 5
&" 5
)" 5
*" 5
! , )"

(; ) 9 6
9 6,

:7 1 1 ! 8 99
/ 7
!" 99
&" 6
)" 8
*" 6 6
! , *"

© All rights reserved by Gateforum Educational Services Pvt. Ltd. No part of this booklet may be reproduced or utilized in any form without the
written permission. )
' ' ,

( 9 9 7 9
9

. / 1 $5 &
H %;;;
) 9 +;;
A +;;;
B 9 =;;
G E;;

6 9 9 , 9
!" ;S &" %S )" E S *" E(S
! , *"
- , 9 3 ; =;;
- 9 3 ';;;
9000
I 6 = 86%
10500

(+ ! & 6 0U + 0U 9
6
=
!" $
% &" $ ( )" C$ ( *" '$ (
! , )"

+ ) A
&

=B

!
0U
0 = +0U

G& ! &
6 3 (; (; 3 #(;;
H 6 3! G!&) L ! 0OAB

© All rights reserved by Gateforum Educational Services Pvt. Ltd. No part of this booklet may be reproduced or utilized in any form without the
written permission. *
3 #(;; − + × × %= × %= 3 =C=
+
=C= C
∴ K 3 =
#(;; (

(# ". 5 / ". ! " ". 8


/" " 7 "! ! ! " #1 " 0 "
# " "! 1 #! 8 ! . .8 7 !!
! ! . /

4 6 9 9
9Z
!" 9 9 6 6 A
9 66 6
&" 9
)" * 6 R 6
*" 6 6 9 8 6
6 9 9
! , !"

(% A 8 % 6 6 65 6 9 A +; A
; A +#; A ;
A 8
!" = &" ( )" ' *" ;
! , !"
- , < A +; A ;
+; N ; 3 +#;
N 3 %
3' 3=
I 6 ; =

(= 9 9 6 5 9 5 6 K 9
9 9 9 6 6 6 . 9
6 9 9 K 9

!" + &" # )" % %" E


! , !"
< 6 9 7 9 9

! !+ ! # & &+ &# ) )+

9 9! &

© All rights reserved by Gateforum Educational Services Pvt. Ltd. No part of this booklet may be reproduced or utilized in any form without the
written permission. +
- -
! ! + ! # 3 & &+ &# ! ! + ! # [ & &+ &#

) )+ - ! & B ! \&"

!
. .
) )+ ! !+
1 ) \ )+ ) 1 ! 3 !+ !#
1 ) Y )+ )+ 1 ! \ !+ !
1 ! Y !+ !+

© All rights reserved by Gateforum Educational Services Pvt. Ltd. No part of this booklet may be reproduced or utilized in any form without the
written permission.
|EC-GATE-2013 PAPER| www.gateforum.com

Q. No. 1 – 25 Carry One Mark Each

1. A bulb in a staircase has two switches, one switch being at the ground floor
and the other one at the first floor. The bulb can be turned ON and also can
be turned OFF by any one of the switches irrespective of the state of the
other switch. The logic of switching of the bulb resembles
(A) an AND gate (B) an OR gate (C) an XOR gate (D) a NAND gate
Answer: (C)
Exp: (C)
Let Switches = p1 , p 2
p1 , p2 Z(o / p)
OFF OFF OFF
OFF ON ON
ON OFF ON
ON ON OFF
From Truth Table, it can be verified that Ex-OR logic is implemented.

   
2. Consider a vector field A ( r ) . The closed loop line integral ∫ A • d l can be
expressed as
 
∫∫
(A)  ∇ ×(A )
• ds over the closed surface bounded by the loop

(B) ∫∫∫ (∇ • A ) dv
 over the closed volume bounded by the loop

(C) ∫∫∫ ( ∇ • A ) dv over the open volume bounded by the loop
 
(D) ∫∫ ( ∇ × A ) • ds over the closed surface bounded by the loop

Answer: (D)
Exp: (D)
Stoke’s Theorem: “ The Line Integral of a vector A around a closed path L is
equal to the integral of curl of A over the open surface S enclosed by the closed
path L”.
∴∫ A.dl = ∫∫ ( ∇ × A ).ds

3. Two systems with impulse responses h1 ( t ) and h2 ( t ) are connected in cascade.


Then the overall impulse response of the cascaded system is given by
(A) Product of h1 ( t ) and h2 ( t )

(B) Sum of h1 ( t ) and h2 ( t )

(C) Convolution of h1 ( t ) and h2 ( t )

(D) Subtraction of h2 ( t ) from h1 ( t )

GATEFORUM- India’s No.1 institute for GATE training


1
|EC-GATE-2013 PAPER| www.gateforum.com

Answer: (C)
Exp: x(t) h1 (t) h2 (t) y(t)

x(t) h(t)
1 *h(t)
2 y(t)

4. In a forward biased pn junction diode, the sequence of events that best describes
the mechanism of current flow is
(A) injection, and subsequent diffusion and recombination of minority carriers
(B) injection, and subsequent drift and generation of minority carriers
(C) extraction, and subsequent diffusion and generation of minority carriers
(D) extraction, and subsequent drift and recombination of minority carriers
Answer: (A)

5. In IC technology, dry oxidation (using dry oxygen) as compared to wet oxidation


(using steam or water vapor) produces
(A) superior quality oxide with a higher growth rate
(B) inferior quality oxide with a higher growth rate
(C) inferior quality oxide with a lower growth rate
(D) superior quality oxide with a lower growth rate
Answer: (D)

6. The maximum value of θ until which the approximation sin θ ≈ θ holds to within 10%
error is
(A) 10o (B) 18o (C) 50o (D) 90o
Answer: (B)
θ3 θ5
Exp: sin θ = θ − + + .......
3! 5!
sin θ = θ + error
Error should 10% of θ
θ3
if ≤ 0.1 θ
6

θ3
if ≤ 0.1 θ Then higher order terms also going to be less than 0.1 θ
6
So, sinθ ≈ θ approximation is valid
θ3
< 0.1 θ
6
θ2 < 0.6
θ < 0.6

Consider θmax = 0.6 = 0.7746 radians


180
θ(in degrees) = 0.7746 ×  45°
π
Out of all possible options we need to go for max theta below 45° Hence it is
18°

GATEFORUM- India’s No.1 institute for GATE training


2
|EC-GATE-2013 PAPER| www.gateforum.com


7. The divergence of the vector field A = xa
ˆx + ya
ˆy + za
ˆz is
(A) 0 (B) 1/3 (C) 1 (D) 3
Answer: (D)
Exp: Given A = xax + yay + zaz
∂ ∂ ∂
∇.A =
∂x x
( )
( A ) + ∂y Ay + ∂z ( Az )
Ax = x, Ay = y, Az = z,
∂ ∂ ∂
=
∂x
( x) +
∂y
( y) +
∂z
( z)
= 1+1+1 = 3

8. The impulse response of a system is h ( t ) = t u ( t ) . For an input u ( t − 1) , the output


is
t ( t − 1) ( t − 1)
2
t2 t2 − 1
(A) u (t) (B) u ( t − 1) (C) u ( t − 1) (D) u ( t − 1)
2 2 2 2
Answer: (C)
Exp:
U(t) t u (t) t2
u(t)
2
LTI system

For LTI system, if input gets delayed by one unit, output will also get delayed by
one unit.
(t − 1)2
u(t − 1) → u(t − 1)
2

9. The Bode plot of a transfer function G (s) is shown in the figure below

40
Gain (dB)

32

20

0
1 10 100
−8 ω (rad / s )

(
The gain 20 log G ( s ) ) is 32 dB and –8 dB at 1 rad/s and 10 rad/s respectively.
The phase is negative for all ω. The G(s) is
39.8 39.8 32 32
(A) (B) (C) (D)
s s2 s s2

GATEFORUM- India’s No.1 institute for GATE training


3
|EC-GATE-2013 PAPER| www.gateforum.com

Answer: (B)
Exp: Any two paints on same line segment of Bode plot satisfies the equation of
straight line.

20 log G ( jω)

40

32

20

1 10

−8 w (rad)

w
G2 − G1
i.e, = slope of the line segment.
log ω2 − log ω1
For the initial straight line
G2 − G1
⇒ = − 40 dB
log ω2 − log ω1 dec
ω
⇒ 0 − 32 = − 40 log  
1
1
⇒ ω = 6.309 = k N ;Where N is type of system here initial slope is
− 40 dB Hence N = 2
dec
1
⇒ 6.309 = k 2

2
⇒ k = ( 6.309 )
k = 39.8

39.8
Hence G (s) =
s2

10. In the circuit shown below what is the output voltage ( Vout ) if a silicon transistor
Q and an ideal op-amp are used?

+15 V Q
1 kΩ

+ + Vout
5V −
−15 V

(A) –15V (B) –0.7V (C) +0.7V (D) +15V

GATEFORUM- India’s No.1 institute for GATE training


4
|EC-GATE-2013 PAPER| www.gateforum.com

Answer: (B)
Exp:
Q
+
+15 V 0.7v

1 kΩ

+ Vout
5V
+
− −15 V

Vout = −0.7 v

11. Consider a delta connection of resistors and its equivalent star connection as
shown below. If all elements of the delta connection are scaled by a factor k, k> 0,
the elements of the corresponding star equivalent will be scaled by a factor of

Ra RC RB
Rb Rc
RA

(A) k 2 (B) k (C) 1/k (D) k


Answer: (B)
R a Rb
Exp: RC = Ra RC RB
R a + Rb + R c
Ra Rc
RB = Rb Rc RA
R a + Rb + R c
RbR c
RA =
R a + Rb + R c

Above expression shown that if R a , Rb & R c is scaled by k, RA, RB & RC is scaled


by k only.

12. For 8085 microprocessor, the following program is executed


MVI A, 05H;
MVIB, 05H;
PTR: ADD B;
DCR B;
JNZ PTR;
ADI 03H;
HLT;
At the end of program, accumulator contains
(A) 17 H (B) 20 H (C) 23 H (D) 05 H

GATEFORUM- India’s No.1 institute for GATE training


5
|EC-GATE-2013 PAPER| www.gateforum.com

Answer: (A)
Exp: Accumulator changes as follows (05 + 05 + 04 +03 +02 +01)H
At the end of Loop accumulator contains = 14H
ADI O3H →A=(14+03)=17H

13. The bit rate of a digital communication system is R kbits/s. The modulation used is
32-QAM. The minimum bandwidth required for ISI free transmission is
(A) R/10 Hz (B) R/10 kHz (C) R/5 Hz (D) R/5 kHz
Answer: (B)
Exp: Bit rate given = R Kbits/second
Modulation = 32-QAM
No. of bits/symbol = 5 [log2 32]
R
Symbol rate = k symbols / sec ond
5
Finally we are transmitting symbols.
B T → transmission bandwidth
R(symbol rate)
BT =
(1 + α )
R
BT =
5 (1 + α )
For B T to be minimum, α has to be maximum
R R
⇒ BT = =
5x2 10
Maximum value of α is '1 ' which is a roll off factor

14. For a periodic signal v ( t ) = 30 sin100t + 10 cos 300t + 6 sin (500t + π / 4 ) , the
fundamental frequency in rad/s
(A) 100 (B) 300 (C) 500 (D) 1500
Answer: (A)
Exp: ωo = 100 rad / sec fundamental
3ωo = 300 rad / sec third harmonic
5ωo = 500 rad / sec fifth harmonic

15. In a voltage-voltage feedback as shown below, which one of the following


statements is TRUE if the gain k is increased?

+ v1 + +
v A0 v out
− in − −

+ +
v f = kv out k
− −

GATEFORUM- India’s No.1 institute for GATE training


6
|EC-GATE-2013 PAPER| www.gateforum.com

(A) The input impedance increases and output impedance decreases


(B) The input impedance increases and output impedance also increases
(C) The input impedance decreases and output impedance also decreases
(D) The input impedance decreases and output impedance increases
Answer: (A)
Exp: In voltage-voltage feedback

A0

R in R AMP R out
R AMPo

R in = R AMP (1 + A 0K)
R AMPo
R out =
1 + A 0K
as K ↑
R in ↑, R out ↓

16. A band-limited signal with a maximum frequency of 5 kHz is to be sampled.


According to the sampling theorem, the sampling frequency which is not valid is
(A) 5 kHz (B) 12 kHz (C) 15 kHz (D) 20 kHz
Answer: (A)
Exp: Given: fm = 5kHz
According to sampling frequency
fs ≥ 2fm
fs ≥ 10 kHz
So, only in option (a) it is less than 10KHz ie., (5KHz)

17. In a MOSFET operating in the saturation region, the channel length modulation
effect causes
(A) an increase in the gate-source capacitance
(B) a decrease in the Transconductance
(C) a decrease in the unity-gain cutoff frequency
(D) a decrease in the output resistance
Answer: (D)
Exp: No channel length modulation
1
IDS = × k(VGS − VT )2
2
∂IDS 1 1
= => = 0 => rds = ∞
∂VDS rds rds

GATEFORUM- India’s No.1 institute for GATE training


7
|EC-GATE-2013 PAPER| www.gateforum.com

under the presence of channel length modulation


∂IDS 1
= λ IDsat =
∂VDS r0
1
r0 = ∴ which is reduced from ∞ to finite value
λ IDsat

18. Which one of the following statements is NOT TRUE for a continuous time causal
and stable LTI system?
(A) All the poles of the system must lie on the left side of the jω axis

(B) Zeros of the system can lie anywhere in the s-plane


(C) All the poles must lie within s = 1

(D) All the roots of the characteristic equation must be located on the left side of
the jω axis
Answer: (C)
Exp: For an LTI system to be stable and causal all poles or roots of characteristic
equation must lie on LHS of s-plane i.e., left hand side of jω − axis
[Refer Laplace transform].

19. The minimum Eigen value of the following matrix is


3 5 2 
 
5 12 7 
2 7 5 

(A) 0 (B) 1 (C) 2 (D) 3


Answer: (A)

3 5 2  5 5 2
  C1+L3  
5 12 7  → 12 12 7 ⇒ det er minant = 0,
Exp: 2 7 5  7 7 5

So the matrix is singular


Therefore atleast one of the Eigen value is ‘0’
As the choices are non negative, the minimum Eigen value is ‘0’

20. A polynomial f ( x ) = a4 x 4 + a3 x3 + a2 x2 + a1x − a0 with all coefficients positive has

(A) no real roots


(B) no negative real root
(C) odd number of real roots
(D) at least one positive and one negative real root
Answer: (D) Use Routh Hurwitz Criteria to get the condition.

GATEFORUM- India’s No.1 institute for GATE training


8
|EC-GATE-2013 PAPER| www.gateforum.com

21. Assuming zero initial condition, the response y(t) of the system given below to a
unit step input u(t) is

U (s) 1 Y (s)
s

t2
(A) u ( t ) (B) tu ( t ) (C) u (t) (D) e− t u ( t )
2
Answer: (B)
Exp: (B)
Integration of unit step function is ramp output

1
u(s) s
y(s)

Writing in time domain

u(t) u(t) y(t)

y(t) = u(t)*u(t) = tu(t)

V2 ( s )
22. The transfer function of the circuit shown below is
V1 ( s )
100 µF

+ +
10kΩ
V1 ( s ) V2 ( s )
100 µF
− −

0.5s + 1 3s + 6 s+2 s +1
(A) (B) (C) (D)
s +1 s+2 s +1 s+2
Answer: (D)
Exp: (D)

+ Z 1 (s) +

V1 (s) Z2(s) V2(s)

− −

1 s +1
z1(s) = −4
, z2(s) = −4
10 s 10 s
V2(s) Zq(s) s +1
= =
V(s)
1 Z1(s) + Zq(s) s + 2

GATEFORUM- India’s No.1 institute for GATE training


9
|EC-GATE-2013 PAPER| www.gateforum.com

23. A source vs ( t ) = V cos100πt has an internal impedance of ( 4 + j3) Ω . If a purely


resistive load connected to this source has to extract the maximum power out of the
source, its value in Ω should be
(A) 3 (B) 4 (C) 5 (D) 7
Answer: (C)
Exp: For maximum power Transfer
RL = Zs

= 42 + 32
= 5Ω

24. The return loss of a device is found to be 20 dB. The voltage standing wave ratio
(VSWR) and magnitude of reflection coefficient are respectively
(A) 1.22 and 0.1 (B) 0.81 and 0.1 (C) –1.22 and 0.1 (D) 2.44 and 0.2
Answer: (A)
Exp: The reflection co-efficient is −20 log Γ = 20dB
⇒ log Γ = −1dB
⇒ Γ = 10−1 => Γ = 0.1
Relation between Γ and VSWR is
1+ Γ
S=
1− Γ
1 + 0.1 1.1
S= =
1 − 0.1 0.9
s = 1.22
Let g ( t ) = e−πt , and h(t) is a filter matched to g(t). If g(t) is applied as input to h(t),
2
25.
then the Fourier transform of the output is
2 2 −π f 2
(A) e−πf (B) e−πf /2
(C) e (D) e−2 πf
Answer: (D)
Exp:
The concept of matched filter assumes that the input signal is of the same form
g(t) as the transmitted signal(except difference in amplitude).this requires that the
shape of the transmitted signal not change on reflection.
h(t) = g(− t) ⇔ H(f) = G * (f)
G * (f) = G(f) ∴ G(f) is real
2 2
g(t) = e − πt ↔ e − πf (fourier transform)
2 2 2
⇒ y(t) ↔ e − πf × e − πf = e −2 πf

g(t) h( t ) g(t)

y(t) = h(t) * g(t) [convolution]

GATEFORUM- India’s No.1 institute for GATE training


10
|EC-GATE-2013 PAPER| www.gateforum.com

Q. No. 26 – 55 Carry Two Marks Each

26. Let U and V be two independent zero mean Gaussian random variables of
1 1
variances and respectively,. The probability P (3V ≥ 2U) is
4 9
(A) 4/9 (B) 1/2 (C) 2/3 (D) 5/9
Answer: (B)
p(3V ≥ 2U) = p(3V − 2 ≥ 0) = p(W ≥ 0),W = 3V − 2U
Exp:
 1
U, V are independent random variables and U ∼ N  0, 
 4

 1
V ∼ N  0, 
 9

 1 1
∴ W = 3V − 2U ∼ N  0,9 × + 4 × 
 4 9
W ∼ N(0,2) ie., W has mean µ = 0 and var iance, σ2 = 2

w − µ 0 − µ
∴ p(W ≥ 0) = p  ≥
 σ σ 
= p(Z ≥ 0), Z is s tandard normal var iants
1
= 0.5 =
2

27. Let A be an m x n matrix and B an n x m matrix. It is given that determinant


(Im + AB ) = determinant (In + BA ) , where Ik is the k × k identity matrix. Using the
above property, the determinant of the matrix given below is
2 1 1 1
 
1 2 1 1
1 1 2 1
 
1 1 1 2

(A) 2 (B) 5 (C) 8 (D) 16


Answer: (B)
1 1 0 0 0
   
1 0 1 0 0
Let A = 1 1 1 11x4 ; B =   ; I1 = 1; I4 = 
Exp: 1 0 0 1 0
   
1 4x1 0 0 0 1

1 1 1 1
 
1 1 1 1
then AB = 4 ; BA =  Here m = 1; n = 4
1 1 1 1
 
1 1 1 1

GATEFORUM- India’s No.1 institute for GATE training


11
|EC-GATE-2013 PAPER| www.gateforum.com

and det(I1 + AB) = det(I4 + BA)


2 1 1 1
 
1 2 1 1
⇒ det of 5 = det of 
1 1 2 1
 
1 1 1 2
2 1 1 1
 
1 2 1 1
∴ det of 
1 1 2 1
 
1 1 1 2

28. In the circuit shown below, if the source voltage VS = 100 ∠53.13O V then the
Thevenin’s equivalent voltage in Volts as seen by the load resistance RL is

3Ω j4 Ω j6Ω 5Ω
+ −
VL1
+ +
VS ~ j40i2 RL = 10 Ω
− − 10VL1
I1
I2

(A) 100 ∠ 90O (B) 800 ∠ 0O (C) 800 ∠ 90O (D) 100 ∠ 60O
Answer: (C)
Exp: VTH = 10 VL1
 4
VC 100∠53.13 −1 
VL1 = = − Tan  8  × j4
3 + j4 5
VL1 = 80 ∠90°
VTH = 800 ∠90°

ω (s) 10
29. The open-loop transfer function of a dc motor is given as = , when
Va ( s ) 1 + 10s
connected in feedback as shown below, the approximate value of Ka that will
reduce the time constant of the closed loop system by one hundred times as
compared to that of the open-loop system is

R ( s) Va ( s ) 10 ω (s)
± Ka
1 + 10s

(A) 1 (B) 5 (C) 10 (D) 100

GATEFORUM- India’s No.1 institute for GATE training


12
|EC-GATE-2013 PAPER| www.gateforum.com

Answer: (C)

Exp:
τ openloop = 10

10 10
τ closed loop = = ⇒ Ka = 9.9.  10.
100 1 + 10ka
10k 10Ka
CLTF = =
10Ka 10s
1+ 1+ (1 + 10ka )
1 + 10s 1 + 10k a
= 10

30. In the circuit shown below, the knee current of the ideal Zener diode is 10mA. To
maintain 5V across RL , the minimum value of RL in Ω and the minimum power
rating of the Zener diode in mW, respectively, are

100Ω

10 V ILoad

Vz = 5V RL

(A) 125 and 125 (B) 125 and 250


(C) 250 and 125 (D) 250 and 250
Answer: (B)
5
Exp R L min =
IL max

10 − 5 5
I100 = = = 50 mA
100 100
ILMax = I100 − Iknee = 40 mA
5
RLmin = ×1000 = 125Ω
40
Minimum power rating of Zener should = 50 mA x 5V
= 250 mW

31. The following arrangement consists of an ideal transformer and an attenuator


which attenuates by a factor of 0.8 An ac voltage Vwx1 = 100V is applied across
WX to get an open circuit voltage VYZ1 across YZ. Next, an ac voltage
VYZ2 = 100V is applied across YZ to get an open circuit voltage VWX2 across WX.
Then, VYZ1 / VWX1 , VWX2 / VYZ2 are respectively.

GATEFORUM- India’s No.1 institute for GATE training


13
|EC-GATE-2013 PAPER| www.gateforum.com

(A) 125 / 100 and 80 / 100 W


1:1.25
(B) 100 / 100 and 80 / 100 Y

(C) 100 / 100 and100 / 100


X Z
(D) 80 / 100 and 80 / 100
Answer: (C)
Exp:
W
+ 1 : 1.25 +
Y
V1 V2
− −
X N1 : N2 Z

For a transform
V2 N
= 2
V1 N1

⇒ V2 = 1.125 × V1

The potentiometer gives an attenuation factor of 0.8 over υ2


Vyz
Hence Vyz = 0.8υ2 ⇒ = υ2
0.8
Vyz
⇒ = 1.125 × Vwx
0.8
VYZ1 100
⇒ VYZ = VWX ⇒ =
VWX1 100
Vwx2 100
Since potentiometer and transformer are bilateral elements. Hence =
Vyz1 100

32. Two magnetically uncoupled inductive coils have Q factors q1 and q2 at the
chosen operating frequency. Their respective resistances are R1 and R2. When
connected in series, their effective Q factor at the same operating frequency is
(A) q1 + q2 (B) (1 / q1 ) + (1 / q2 )

(C) ( q1R1 + q2R2 ) / (R1 + R2 ) (D) ( q1R2 + q2R1 ) / (R1 + R2 )

Answer: (C)
Exp: Q Factor of a inductive coil.
wL wL1 wL 2
Q= ⇒ Q1 = & Q2 =
R R2 R2

L R

GATEFORUM- India’s No.1 institute for GATE training


14
|EC-GATE-2013 PAPER| www.gateforum.com

When such two coils are connected in series individual inductances and
resistances are added.
Hence, L eq = L1 + L2

R eq = R1 + R 2

ωL1 ωL2
+
ωL eq ω (L1 + L2 ) R R R1 R 2
Hence Qeq = = = 1 2
R eq (R1 + R2 ) R1
+
R2
R1 R 2 R1R 2

Q1 Q2
+
R 2 R1 Q1 R1 + Q2 R 2
= =
1 1 R1 + R 2
+
R 2 R1

33. The impulse response of a continuous time system is given by


h ( t ) = δ ( t − 1) + δ ( t − 3) . The value of the step response at t = 2 is

(A) 0 (B) 1 (C) 2 (D) 3


Answer: (B)
Exp: h ( t ) = δ ( t − 1) + δ ( t − )

u (t)
h (t) y (t)

y ( t ) = u ( t − 1) + u ( t − 3 )
y (2 ) = u (1) + u ( −1)
=1

34. The small-signal resistance (i.e., dVB / dID ) in kΩ offered by the n-channel
MOSFET M shown in the figure below, at a bias point of VB = 2 V is (device data
for M: device Transconductance parameter kN = µnC'ox ( W / L ) = 40µA / V,2
threshold voltage VTN = 1 V, and neglect body effect and channel length
modulation effects)

VB
ID

(A) 12.5 (B) 25 (C) 50 (D) 100

GATEFORUM- India’s No.1 institute for GATE training


15
|EC-GATE-2013 PAPER| www.gateforum.com

Answer: (B)
Exp:
dVB
= ?
dID
VB = VDs = VGS
∴ M is in saturation
1
ID = × 40 × 10 −6 (VDS − VT )2
2
∂ID
= 40 × 10−6 (2 − 1) = 40 × 10−6
∂VDS
∂VDS ∂VB
= = 25kΩ
∂ID ∂ID

35. The ac schematic of an NMOS common-source stage is shown in the figure below,
where part of the biasing circuits has been omitted for simplicity. For the n-
channel MOSFET M, the Transconductance gm = 1mA / V, and body effect and
channel length modulation effect are to be neglected. The lower cutoff frequency
in Hz of the circuit is approximately at

RD C
10kΩ 1µF
V0

Vi M
R L 10 kΩ

(A) 8 (B) 32 (C) 50 (D) 200


Answer: (A)
Exp:

RD
1µF 10 kΩ = RD
RL

Vi
RL = 10kΩ

1
fcut =
2π ( R D + R L ) C
= 8Hz

GATEFORUM- India’s No.1 institute for GATE training


16
|EC-GATE-2013 PAPER| www.gateforum.com

d2 y dy
36. A system is described by the differential equation 2
+5 + 6y ( t ) = x ( t ) .
dt dt
Let x(t) be a rectangular pulse given by
1 0 < t <2
x (t) = 
0 otherwise
dy
Assuming that y(0)=0 and = 0 at t = 0, the Laplace transform of y(t) is
dt
e−2s 1 − e−2s
(A) (B)
s ( s + 2 ) ( s + 3) s ( s + 2 ) ( s + 3)

e−2s 1 − e−2s
(C) (D)
( s + 2) ( s + 3) ( s + 2) ( s + 3)
Answer: (B)
Exp: Writing in terms of laplace transform.
S2 y ( s ) + 5sy ( s ) + 6y ( s ) = x ( s )

x (s)
⇒ y ( s) = 2
s + 5s + 6
 1 − e−2s 
x ( s) =  
 s
 

1
x (t) = = U ( t ) − U ( t − 2)

0 2 t

1 − e−2s
x (s) =
s
1 − e−2s
⇒ Y (s) =
s ( s + 2 ) ( s + 3)

37. A system described by a linear, constant coefficient, ordinary, first order


differential equation has an exact solution given by y(t) for t>0, when the forcing
function is x(t) and the initial condition is y(0). If one wishes to modify the
system so that the solution becomes -2y(t) for t>0, we need to
(A) change the initial condition to –y(0) and the forcing function to 2x(t)
(B) change the initial condition to 2y ( 0 ) and the forcing function to −x ( t )

(C) change the initial condition to j 2y ( 0 ) and the forcing function to j 2x ( t )

(D) change the initial condition to -2y(0) and the forcing function to −2x ( t )
Answer: (D)
dy ( t )
Exp: + ky ( t ) = x ( t ) ≤
dt

GATEFORUM- India’s No.1 institute for GATE training


17
|EC-GATE-2013 PAPER| www.gateforum.com

SY ( s ) − y ( 0 ) + k Y ( s ) = X ( s )
Y ( s ) s + k  = X ( s ) − Y ( 0 )
X ( s ) − Y (0 )
Y (s) =
S+K
X ( s ) Y (0 )
Y (s) = −
s+k s+k
y ( t ) = e−kt x ( t ) − y ( 0 ) e−kt

So if we want −2y ( t ) as a solution both x(t) and y(0) has to be doubled and
multiplied by –ve sign
x ( t ) → −2x ( t )
y ( 0 ) → −2y ( 0 )

38. Consider two identically distributed zero-mean random variables U and V. Let the
cumulative distribution functions of U and 2V be F(x) and G(x) respectively.
Then, for all values of x
(A) F ( x ) − G ( x ) ≤ 0 (B) F ( x ) − G ( x ) ≥ 0

(C) (F ( x ) − G ( x ) ) .x ≤ 0 (D) (F ( x ) − G ( x ) ) .x ≥ 0

Answer: (D)
Exp: F ( x ) = P {X ≤ x}

G ( x ) = P {2X ≤ x}

{
=P X≤ x
2 }
For positive value of x,
F ( x ) − G ( x ) is always greater than zero

For negative value of x.


F ( x ) − G ( x ) is − ve

but . F ( x ) − G ( x )  . x ≥ 0

39. The DFT of vector a b c d is the vector α β γ δ  . Consider the product

a b c d
 
p q r s  = a b c d d a b c
c d a b
 
b c d a

The DFT of the vector p q r s  is a scaled version of

(A) α2 β2 γ2 δ2  (B)  α β γ δ


   
(C) α + β β + δ δ + γ γ + α  (D) α β γ δ 

GATEFORUM- India’s No.1 institute for GATE training


18
|EC-GATE-2013 PAPER| www.gateforum.com

Answer: (A)
T
Exp: x(n) = a b c d

a dd c a
   
N= 4 b ca d b
x(n) ⊗ × (n) 
c db a c 
   
d c b
a 4× 4 d 4×1
a b c d
 
4 d a b c
x(n) ⊗(n) = a b c d 
c d a b
 
b c d a
p q r s = x(n) ⊗ x(n)
DFT

x(n)

 x(k) α β γ δ

DFT {p q r s} = x(k)x(k) = x2(ck) =  α2 β2 γ2 δ2 

Y (s)
40. The signal flow graph for a system is given below. The transfer function for
U (s)
1
this system is
s +1
(A) 2
5s + 6s + 2
1 s −1 s −1 1 Y (s)
s +1 U (s)
(B)
s2 + 6s + 2
s +1
(C) 2
−4
s + 4s + 2
−2
1
(D) 2
5s + 6s + 2
Answer: (A)
Exp: By using Mason’s gain formula
y(s) s−2 + s−1 s−2 s + 1 s +1
= = =
u(s) 1 − −2s−2 − 4s−1 − 2s−1 − 4 + 0 252 + 6s−1 + 5 5s2
+ 6s + 2
 

41. In the circuit shown below the op-amps are ideal. The Vout in Volts is
1kΩ 1kΩ
−2V
(A) 4 +15 V
+15 V

+ Vout
(B) 6 +

−15 V
1kΩ
(C) 8 −15 V

1kΩ
+1V 1kΩ
(D) 10

GATEFORUM- India’s No.1 institute for GATE training


19
|EC-GATE-2013 PAPER| www.gateforum.com

Answer: (C)
Exp: 1kΩ 1kΩ
−2V


+ Vout
+ V1

Vout = ? 1kΩ

1kΩ
+1V 1kΩ

V1 = 1 (1 + 1) − 2 ( −1) = 4V
Vout = 2V1 = 8V

42. In the circuit shown below, Q1 has negligible collector-to-emitter saturation


voltage and the diode drops negligible voltage across it under forward bias. If
Vcc is + 5 V, X and Y are digital signals with 0 V as logic 0 and VCC as logic 1,
then the Boolean expression for Z is
+ Vcc

R1
Z
R2
X Q1
Diode

(A) XY (B) X Y (C) X Y (D) X Y


Answer: (B)
Exp: (B)
X Y Z
0 0 0
0 1 1
1 0 0
1 1 0

43. A voltage 1000sin ωt Volts is applied across YZ. Assuming ideal diodes, the
voltage measured across WX in Volts, is

1kΩ
(A) sin ωt

( )
(B) sin ωt + sin ωt / 2 W X
Z
( )
(C) sin ωt − sin ωt / 2

(D) 0 for all t + 1kΩ −

GATEFORUM- India’s No.1 institute for GATE training


20
|EC-GATE-2013 PAPER| www.gateforum.com

Answer: (D)

Exp:. Vyz

0
ωt

During the half cycle


All Diodes OFF & Hence

V1 V2
− + + −

+ V +

V1 = V2
V = 0V
During –Ve half cycle

V
V = 0V.

44. Three capacitors C1 , C2 and C3 whose values are 10µF, 5µF, and 2µF
respectively, have breakdown voltages of 10V, 5V, and 2V respectively. For the
interconnection shown below, the maximum safe voltage in Volts that can be
applied across the combination, and the corresponding total charge in µC stored
in the effective capacitance across the terminals are respectively.
(A) 2.8 and 36 C2 C3

(B) 7 and 119


(C) 2.8 and 32
(D) 7 and 80 C1

Answer: (C)
Vc3 2V
Exp: ≤ 5V ⇒ ≤ 5V..................(1)
c2 + c3 7

Vc2 5V
≤2⇒ ≤ 2V..................(2)
c2 + c3 7

V ≤ 10V ⇒ V ≤ 10V.......................(3)

From (1), V ≤ 17.5 Volts

From (2), V ≤ 2.8 Volts

GATEFORUM- India’s No.1 institute for GATE training


21
|EC-GATE-2013 PAPER| www.gateforum.com

From (3), V ≤ 10 Volts


To operate Circuit safe, V should be minimum of those =2.8V
10 80
ceff = c1 + (c2 C3 ) = 10µF + µF = µF
7 7
Q = ceff × 2.8V = 32µc

45. There are four chips each of 1024 bytes connected to a 16 bit address bus as
shown in the figure below. RAMs 1,2,3 and 4 respectively are mapped to
addresses
RAM# 4
1024B

RAM#3
A0 − A9

8 bit data bus


1024B
E

RAM# 2
1024B
E
A10
A11 RAM#1
1024B
A12
E
A13
11
Input

A14 10
01
00
A15 S1 S0

(A) 0C00H − 0FFFH,1C00H − 1FFFH, 2C00H − 2FFFH,3C00H − 3FFFH


(B) 1800H − 1FFFH,2800H − 2FFFH, 3800H − 3FFFH, 4800H − 4FFFH
(C) 0500H − 08FFH,1500H − 18FFH, 3500H − 38FFH,5500H − 58FFH
(D) 0800H − 0BFFH,1800H − 1BFFH, 2800H − 2BFFH, 3800H − 3BFFH
Answer: (D)
Exp: (D)

A 14 A 13 (s1 ) A 12 (s 0 ) A 11 A 10 A9 A0

Chip #1
{ 0
0
0
0
0
0
0
0
1
1
0
0
0
1
…………………
…………………
0=0800H
0=0BFFH

{
0 0 0 1 1 0 0 ………………… 0=1800H
Chip # 2
0 0 0 1 1 0 1 ………………… 1=1BFFH
0 0 1 0 1 0 0 ………………… 0=2800H
Chip # 3
{ 0
0
0
0
1
1
0
1
1
1
0
0
1
0
…………………
…………………
1=2BFFH
0=3800H
Chip # 4
{ 0 0 1 1 1 0 1 ………………… 0=3BFFH

GATEFORUM- India’s No.1 institute for GATE training


22
|EC-GATE-2013 PAPER| www.gateforum.com

46. In the circuit shown below, the silicon npn transistor Q has a very high value of
β . The required value of R2 in kΩ to produce IC = 1mA is
Vcc
3V

R1 Ic
60 kΩ

R2
RE
500 Ω

(A) 20 (B) 30 (C) 40 (D) 50


Answer: (C)
Exp VCC = 3V

IC
R1 = 60 kΩ

R2 VB = 1.2
3. = 1.2 0.7
60 + R 2
R2 RE = 500 Ω
R 2 = 40 kΩ

47. Let U and V be two independent and identically distributed random variables such
1
that P (U = + 1) = P (U = −1) = . The entropy H(U + V) in bits is
2
(A) 3/4 (B) 1 (C) 3/2 (D) log23
Answer: (C)
Exp:

U V (U + V )
+1 +1 +2
+1 -1 0
-1 +1 0
-1 -1 -2

P {U + V = +2} = 1 . 1 = 1
2 2 4
1 1 1
P {U + V = 0} = + =
4 4 2
P {U + V = −2} = 1 . 1 = 1
2 2 4
1 1
⇒ H {U + V} = log2 2 + 2 × log2 4
2 4
1
= +1 = 3
2 2

GATEFORUM- India’s No.1 institute for GATE training


23
|EC-GATE-2013 PAPER| www.gateforum.com

Common Data Questions: 48 & 49

Bits 1 and 0 are transmitted with equal probability. At the receiver, the pdf of the
respective received signals for both bits are as shown below.

pdf of received
1 signal for bit 0
pdf of received
signal for bit 1
0.5

−1 0 1 2 4

48. If the detection threshold is 1, the BER will be


1 1 1 1
(A) (B) (C) (D)
2 4 8 16
Answer: (D)
Exp: Pe = P ( 0 ) P (1 / 0 ) + P (1) P ( 0 / 1)

If Detection threshold = 1
1
P ( 0 ) = P (1) =
2
 Y = 1 1 Z
P
X = 0
= ∫ 0
f   dZ
1

0.5
1

1
4
0.2

Z
−1 0 4 1 2 4
5

1 1 1
P (1 / 0 ) = × ×1 =
2 4 8
P ( 0 / 1) = 0
1 1 1 1
Pe = × + ×0 =
2 8 2 16

49. The optimum threshold to achieve minimum bit error rate (BER) is
1 4 3
(A) (B) (C) 1 (D)
2 5 2
Answer: (B)
Exp: Optimum threshold = The point of intersection of two pdf’s

GATEFORUM- India’s No.1 institute for GATE training


24
|EC-GATE-2013 PAPER| www.gateforum.com

z
f  =1− z z ≤1
0
z z
f  = 0<z<2
1 4
The point of intersection which decides optimum threshold
z
⇒1−z =
4
z
1= +z
4
5z
1=
4
4
z=
5
Common Data Questions: 50 & 51

Consider the following figure

1Ω
+ 5Ω
Vs 10V 2Ω
− 2A

50. The current IS in Amps in the voltage source, and voltage VS in Volts across the
current source respectively, are
(A) 13, -20 (B) 8, -10 (C) −8, 20 (D) −13, 20
Answer: (D)
Exp:
2A 10A

− 8A 5A 10A
10V 5Ω
IS
+ +
2Ω 1Ω
+ 10V

2A

IS + 8 + 5 = 0
IS = −13A

51. The current in the 1 Ω resistor in Amps is


(A) 2 (B) 3.33 (C) 10 (D) 12
Answer: (C)
Exp: I1Ω = 10A we can use principle of superposition to determine the current across 1
ohm resistance.

GATEFORUM- India’s No.1 institute for GATE training


25
|EC-GATE-2013 PAPER| www.gateforum.com

Linked Answer Questions: Q.52 to Q.55 Carry Two Marks Each

Statement for Linked Answer Questions: 52 & 53

A monochromatic plane wave of wavelength λ = 600 µm is propagating in the


  
direction as shown in the figure below. Ei , Er , and Et denote incident, reflected,
and transmitted electric field vectors associated with the wave.

Ei Er
Hi Kr
Ki θi θr Hr
εr = 1.0
0 εr = 4.5 x
Ht
19.2 Et

z kt


52. The angle of incidence θ1 and the expression for Ei are
π×104 ( x + z )
E0 −j
0
(A) 60 and (ˆax − ˆaz ) e 3 2 V /m
2
π×104 z
E0 j
0
(B) 45 and (ˆax + ˆaz ) e 3 V /m
2
π×104 ( x + z )
E0 −j
(C) 450 and (ˆax − ˆaz ) e 3 2 V /m
2
π×104 z
E0 −j
0
(D) 65 and (ˆax − ˆaz ) e 3 V /m
2
Answer: (C)
Exp: (C)
The given oblique incidence is an vertical polarization ie., Ei is parallel to the
plane of incidence, Hi is perpendicular to the plane of incident.
− jβ1 x sin θi + z cos θi 
Ei = Eo cos θiax − sin θia2  × e .......(1)

From the given problem


n1 sin θi = n2 sin θt ................(2)

⇒ ε r1 sin θi = ε r2 sin θt
⇒ 1.sin θi = 2.1213 × sin(19.2)
sin θi = 0.6976
θi = sin−1(0.6976)
θi ≈ 45°

GATEFORUM- India’s No.1 institute for GATE training


26
|EC-GATE-2013 PAPER| www.gateforum.com

∴ the angle of incidence is 45°


2π 2π π
β= = −6
= 104 θi = 45, θr = 19.2
λ 600 × 10 3
substituting equation (1) we get
π
− j 104 [x sin(45) + z cos(45)]
Ei = Eo [cos(45) ax − sin(45)az ] e 3

− jπ104
 1 1  (x + z)
Ei = Eo  ax − az  × e 3 2

 2 2 
π104 (x + z)
Eo −j
Ei =  ax − az  e 3 2


53. The expression for Er is
π×104 ( x − z )
E0 −j
(A) 0.23 (ˆax + ˆaz ) e 3 2 V /m
2
π×104 z
E0 j
(B) − (ˆax + ˆaz ) e 3 V /m
2
π×104 ( x − z )
E0 −j
(C) 0.44 (ˆax + ˆaz ) e 3 2 V /m
2
π×104 ( x + z )
E0 −j
(D) (ˆax + ˆaz ) e 3 V /m
2
Answer: (A)
Exp: (A)

The reflection co-efficient for parallel polarization is given by


εr2 εr2
cos θi − − sin2 θi
Er εr1 εr1
Γ = =
Ei εr2 εr2
cos θi + − sin2 θi
εr1 εr1
4.5 4.5
cos(45) − − sin2 (45)
Γ = 1 1
4.5 4.5
cos(45) + − sin2 (45)
1 1
Γ = 0.23
− jβ x sin θr − z cos θr 
Er = ΓEo  cos θr ax + sin θr ax  e

But θr = θi since reflected ray and incident ray lies in the same medium by using
snell’s law
− jβ  x sin θi − z cos θi 
Er = Γ Eo cos θiax + sin θiaz  e
π×104  1 1 
−j x −z 
3  2i
Er = 0.23 × Eo cos(45)ax + sin(45)az  e 2
Hence
π×104 (x − z)
Eo −j
Er = 0.23 ×  ax + az  e 3 2
v /m
2

GATEFORUM- India’s No.1 institute for GATE training


27
|EC-GATE-2013 PAPER| www.gateforum.com

Statement for Linked Answer Questions: 54 & 55

The state diagram of a system is shown below. A system is described by the


state-variable equations
.
X = AX + Bu; y = CX + Du
1 −1 1 −1 1
u y

1 1
S S
54. The state-variable equations of the system shown in the figure above are
.  −1 0   −1 .  −1 0   −1
X=   X +  u X=   X +  u
(A)  1 − 1 1 (B)  − 1 − 1 1
y = 1 − 1 X + u y = −1 − 1 X + u
.  −1 0   −1 .  −1 −1  −1
X=  X+  u X=   X +  u
(C)  −1 −1 1 (D)  0 − 1  1
y = 1 − 1 X − u y =  −1 − 1 X − u
Answer: (A)
Exp: . .
1 x1 −1 x 1 1 x2 −1 x 2 1
u y

1 1
.
S S
x 1 = − x1 − 4
.
x 2 = − x2 − 4
y = − x2 + x1 + 4
.  −1 0   −1
x =  x +   u ; y = 1 − 1 x + 4
 1 −1 1

55. The state transition matrix eAt of the system shown in the figure above is
 e− t 0   e− t 0  e − t 0  e − t −te−t 
(A)   (B)   (C)   (D)  
−t
te e− t   −te
−t
e−t  e
−t
e−t   0 e−t 
Answer: (A)
 −1 0 
Exp: A=  
 1 −1

eAt = L−1 ( SI − A ) 
−1
 
e − t
0 
eAt =  
−t
 te e− t 

GATEFORUM- India’s No.1 institute for GATE training


28
|EC-GATE-2013 PAPER| www.gateforum.com

Q. No. 56 – 60 Carry One Mark Each

56. Choose the grammatically CORRECT sentence:


(A) Two and two add four (B) Two and two become four
(C) Two and two are four (D) Two and two make four
Answer: (D)

57. Statement: You can always give me a ring whenever you need.
Which one of the following is the best inference from the above statement?
(A) Because I have a nice caller tune
(B) Because I have a better telephone facility
(C) Because a friend in need in a friend indeed
(D) Because you need not pay towards the telephone bills when you give me a
ring
Answer: (C)

58. In the summer of 2012, in New Delhi, the mean temperature of Monday to
Wednesday was 41ºC and of Tuesday to Thursday was 43ºC. If the temperature
on Thursday was 15% higher than that of Monday, then the temperature in ºC on
Thursday was
(A) 40 (B) 43 (C) 46 (D) 49
Answer: (C)
Explanations:- Let the temperature of Monday be TM
Sum of temperatures of Tuesday and Wednesday = T and
Temperature of Thursday =TTh
Now, Tm + T = 41 × 3 = 123
& Tth + T = 43 × 3 = 129
∴ TTh − Tm = 6, Also TTh = 1.15Tm
∴0.15Tm = 6 ⇒ Tm = 40
∴ Temperature of thursday = 40 + 6 = 46O C

59. Complete the sentence:


Dare ____________ mistakes.
(A) commit (B) to commit (C) committed (D) committing
Answer: (B)

60. They were requested not to quarrel with others.


Which one of the following options is the closest in meaning to the word quarrel?
(A) make out (B) call out (C) dig out (D) fall out
Answer: (D)

GATEFORUM- India’s No.1 institute for GATE training


29
|EC-GATE-2013 PAPER| www.gateforum.com

Q. No. 61 – 65 Carry Two Marks Each

61. A car travels 8 km in the first quarter of an hour, 6 km in the second quarter and
16km in the third quarter. The average speed of the car in km per hour over the
entire journey is
(A) 30 (B) 36 (C) 40 (D) 24
Answer: (C)
Total dis tan ce
Explanations:-Average speed =
Total time
8 + 6 + 16
= = 40 km / hr
1 1 1
+ +
4 4 4

62. Find the sum to n terms of the series 10 + 84 + 734 + …

(A)
(
9 9n + 1 ) +1 (B)
(
9 9n − 1 ) +1 (C)
(
9 9n − 1 ) +n (D)
(
9 9n − 1 ) +n
2

10 8 8 8
Answer: (D)
Explanations:-Using the answer options, substitute n = 2. The sum should add up to 94
Alternative Solution:
The given series is 10+84+734+………………………..+n terms
( ) ( ) ( )
= ( 9 + 1) + 92 + 3 + 93 + 5 + 94 + 7 + ................. n terms

( )
= 9 + 92 + 93 + ....................nterms + (1 + 3 + 5 + 7 + ............ n terms )

(
9 9n − 1 )+n 2

 sn =
(
a rn − 1 )
(r > 1) and


=  r −1 
9 −1  Sum of first n odd natural numbers is n2 
 

63. Statement: There were different streams of freedom movements in colonial India
carried out by the moderates, liberals, radicals, socialists, and so on.
Which one of the following is the best inference from the above statement?
(A) The emergence of nationalism in colonial India led to our Independence
(B) Nationalism in India emerged in the context of colonialism
(C) Nationalism in India is homogeneous
(D) Nationalism in India is heterogeneous
Answer: (D)

64. The set of values of p for which the roots of the equation 3x2 + 2x + p (p − 1) = 0
are of opposite sign is
(A) ( −∞, 0 ) (B) ( 0,1) (C) (1, ∞ ) (D) ( 0, ∞ )
Answer: (B)
Explanation:
Since the roots are of opposite sign, the product of roots will be negative.
p ( p − 1)
∴ < 0 ⇒ p ( p − 1) < 0 ⇒ ( p − 0 ) ( p − 1) < 0 ⇒ 0 < p < 1
3
Thus the required set of values is ( 0, 1)

GATEFORUM- India’s No.1 institute for GATE training


30
|EC-GATE-2013 PAPER| www.gateforum.com

65. What is the chance that a leap year, selected at random, will contain 53
Sundays?
(A) 2/7 (B) 3/7 (C) 1/7 (D) 5/7
Answer: (A)
Explanations:-There are 52 complete weeks in a calendar year  852 × 7 = 364 days
Number of days in a leap year = 366
2
∴ Probability of 53 Saturdays =
7

GATEFORUM- India’s No.1 institute for GATE training


31
EC-GATE-2014 PAPER-01| www.gateforum.com

Q. No. 1 – 5 Carry One Mark Each

1. Choose the most appropriate phrase from the options given below to complete the following
sentence.
The aircraft_______ take off as soon as its flight plan was filed.
(A) is allowed to (B) will be allowed to
(C) was allowed to (D) has been allowed to
Answer: (C)

2. Read the statements:


All women are entrepreneurs.
Some women are doctors
Which of the following conclusions can be logically inferred from the above statements?
(A) All women are doctors (B) All doctors are entrepreneurs
(C) All entrepreneurs are women (D) Some entrepreneurs are doctors
Answer: (D)

3. Choose the most appropriate word from the options given below to complete the following
sentence.
Many ancient cultures attributed disease to supernatural causes. However, modern science
has largely helped _________ such notions.
(A) impel (B) dispel (C) propel (D) repel
Answer: (B)

4. The statistics of runs scored in a series by four batsmen are provided in the following table,
Who is the most consistent batsman of these four?

Batsman Average Standard deviation


K 31.2 5.21
L 46.0 6.35
M 54.4 6.22
N 17.9 5.90

(A) K (B) L (C) M (D) N


Answer: (A)
Exp: If the standard deviation is less, there will be less deviation or batsman is more consistent

5. What is the next number in the series?


12 35 81 173 357 ____
Answer: 725

 India’s No.1 institute for GATE Training  1 Lakh+ Students trained till date  65+ Centers across India
1
EC-GATE-2014 PAPER-01| www.gateforum.com

Exp: 12 35 81 173 357 ________

23 46 92 184 368
difference

357
⇒ 368
725
Q. No. 6 – 10 Carry One Mark Each

6. Find the odd one from the following group:


W,E,K,O I,Q,W,A F,N,T,X N,V,B,D
(A) W,E,K,O (B) I,Q,W,A (B) F,N,T,X (D) N,V,B,D
Answer: (D)
Exp: 1 Q W A
W E K O F N T X N V B D

8 6 4 8 6 4 8 6 4 8 6 2

Difference of position: D

7. For submitting tax returns, all resident males with annual income below Rs 10 lakh should fill
up Form P and all resident females with income below Rs 8 lakh should fill up Form All
people with incomes above Rs 10 lakh should fill up Form R, except non residents with
income above Rs 15 lakhs, who should fill up Form S. All others should fill Form T. An
example of a person who should fill Form T is
(A) a resident male with annual income Rs 9 lakh
(B) a resident female with annual income Rs 9 lakh
(C) a non-resident male with annual income Rs 16 lakh
(D) a non-resident female with annual income Rs 16 lakh
Answer: (B)
Exp: Resident female in between 8 to 10 lakhs haven’t been mentioned.

8. A train that is 280 metres long, travelling at a uniform speed, crosses a platform in 60 seconds
and passes a man standing on the platform in 20 seconds. What is the length of the platform
in metres?
Answer: 560
Exp: For a train to cross a person, it takes 20 seconds for its 280m.
So, for second 60 seconds. Total distance travelled should be 840. Including 280 train length
so length of plates =840-280=560

 India’s No.1 institute for GATE Training  1 Lakh+ Students trained till date  65+ Centers across India
2
EC-GATE-2014 PAPER-01| www.gateforum.com

9. The exports and imports (in crores of Rs.) of a country from 2000 to 2007 are given in the
following bar chart. If the trade deficit is defined as excess of imports over exports, in which
year is the trade deficit 1/5th of the exports?

120
Exports Im ports
110
100
90
80
70
60
50
40
30
20
10
0 2006
2000 2001 2002 2003 2004 2005 2007

(A) 2005 (B) 2004 (C) 2007 (D) 2006


Answer: (D)
imports − exp orts 10 1
Exp: 2004, = =
exp orts 70 7
26 2
2 0 0 5, =
76 7
20 1
2 0 0 6, =
100 5
10 1
2007, =
100 11

10. You are given three coins: one has heads on both faces, the second has tails on both faces,
and the third has a head on one face and a tail on the other. You choose a coin at random and
toss it, and it comes up heads. The probability that the other face is tails is
(A) 1/4 (B) 1/3 (C) 1/2 (D) 2/3
Answer: (B)

 India’s No.1 institute for GATE Training  1 Lakh+ Students trained till date  65+ Centers across India
3
EC-GATE-2014 PAPER-01| www.gateforum.com

Q. No. 1 – 25 Carry One Mark Each

1. For matrices of same dimension M, N and scalar c, which one of these properties DOES NOT
ALWAYS hold?
(A) (MT)T = M (B) (cMT)T = c(M)T
(C) (M + N)T = M T + NT (D) MN = NM
Answer: (D)
Exp: Matrix multiplication is not commutative in general.

2. In a housing society, half of the families have a single child per family, while the remaining
half have two children per family. The probability that a child picked at random, has a sibling
is _____
Answer: 0.667
Exp: Let E1 = one children family
E 2 = two children family and
A = picking a child then by Baye’s theorem, required probability is
1

( )
.x
E 2 2
P 2 = = = 0.667
A 1 x 1
. + .x 3
2 2 2
(Here ‘x’ is number of families)

 z2 − z + 4 j 
3. ∫C  z + 2 j 
C is a closed path in the z-plane given by z = 3. The value of the integral → 

dz is
(A) −4π (1 + j2 ) (B) 4 π ( 3 − j2 ) (C) −4π ( 3 + j2 ) (D) 4 π (1 − j2 )
Answer: (C)
Exp: Z = −2 j is a singularity lies inside C : Z = 3
∴ By Cauchy’s integral formula,
Z2 − Z + 4 j
∫ C Z + 2 j dz = 2πj.  Z − Z + 4 j Z=−2 j
2

= 2πj[ −4 + 2 j + 4 j] = −4π [3 + j2]

4. A real (4 × 4) matrix A satisfies the equation A2 = I, where I is the (4 × 4) identity matrix.


The positive eigen value of A is __________.
Answer: 1
1
Exp: A 2 = I ⇒ A = A −1 ⇒ if λ is on eigen value of A then is also its eigen value. Since, we
λ
require positive eigen value. ∴λ = 1 is the only possibility as no other positive number is self
inversed

 India’s No.1 institute for GATE Training  1 Lakh+ Students trained till date  65+ Centers across India
4
EC-GATE-2014 PAPER-01| www.gateforum.com

5. Let X1, X2, and X3 be independent and identically distributed random variables with the
uniform distribution on [0, 1]. The probability P{X1 is the largest} is ________
Answer: 0.32-0.34

6. For maximum power transfer between two cascaded sections of an electrical network, the
relationship between the output impedance Z1 of the first section to the input impedance Z2 of
the second section is
(A) Z2 = Zl (B) Z2 = − Zl (C) Z2 = Z1∗ (D) Z2 = − Z1∗
Answer: (C)
Exp: Two cascaded sections

Section Section
Z1 Z L Z 2
1 2

Z1 = Output impedance of first section


Z2 = Input impedance of second section
For maximum power transfer, upto 1st section is
ZL = Z1*
ZL = Z2 ⇒ Z1*

7. Consider the configuration shown in the figure which is a portion of a larger electrical
network

i5
i2

R R
i3
R
i4
i1 i6

For R = 1Ω and currents i1 = 2A, i4 = -1A, i5 = -4A, which one of the following is TRUE?
(A) i6 = 5 A
(B) i 3 = −4A
(C) Data is sufficient to conclude that the supposed currents are impossible
(D) Data is insufficient to identify the current i 2 ,i3 , and i 6
Answer: (A)

 India’s No.1 institute for GATE Training  1 Lakh+ Students trained till date  65+ Centers across India
5
EC-GATE-2014 PAPER-01| www.gateforum.com

Exp: Given i1 = 2A i5
i 4 = −1A B
i2
i5 = − 4A
1Ω 1Ω
KCL at node A, i1 + i 4 = i 2
⇒ i 2 = 2 − 1 =1A i3
1. KCL at node B, i 2 + i5 = i3 i4
⇒ i3 =1 − 4 = − 3A 1Ω C
A i1 i6
KCL at node C, i3 + i6 = i1
⇒ i6 = 2 − ( −3) = 5A

8. When the optical power incident on a photodiode is 10µW and the responsivity is 0.8 A / W,
the photocurrent generated ( in µA ) is ________.
Answer: 8
Ip
Exp: Responsivity ( R ) =
P0
Ip
0.8 =
10 × 10−6
⇒ I8 = 8µA

9. In the figure, assume that the forward voltage drops of the PN diode D1 and Schottky diode
D2 are 0.7 V and 0.3 V, respectively. If ON denotes conducting state of the diode and OFF
denotes non-conducting state of the diode, then in the circuit,
1kΩ 20 Ω

10 Ω D1 D2

(A) both D1 and D2 are ON (B) D1 is ON and D2 is OFF


(C) both D1 and D2 are OFF (D) D1 is OFF and D2 is ON
Answer: (D) 1K 20Ω
Exp: Assume both the diode ON.
Then circuit will be as per figure (2)
D1 D2
10 − 0.7 10V
∴I = = 9.3mA
1k
0.7 − 0.3 Figure (1)
I D2 = = 20mA
20
1K 20Ω
Now, I D1 = I − ID2 I

= −10.7 mA ( Not possible ) ID1 ID2

∴ D1is OFF and hense D 2 − ON 10V 0.7V 0.3V

 India’s No.1 institute for GATE Training  1 Lakh+ Students trained till date  65+ Centers across India
6
EC-GATE-2014 PAPER-01| www.gateforum.com

10. If fixed positive charges are present in the gate oxide of an n-channel enhancement type
MOSFET, it will lead to
(A) a decrease in the threshold voltage (B) channel length modulation
(C) an increase in substrate leakage current (D) an increase in accumulation capacitance
Answer: (A)

11. A good current buffer has


(A) low input impedance and low output impedance
(B) low input impedance and high output impedance
(C) high input impedance and low output impedance
(D) high input impedance and high output impedance
Answer: (B)
Exp: Ideal current Buffer has Zi = 0
Z0 = ∞

12. In the ac equivalent circuit shown in the figure, if i in is the input current and RF is very large,
the type of feedback is

RD
RD
υout
M2
M1

small signal
RF
input i in

(A) voltage-voltage feedback (B) voltage-current feedback


(C) current-voltage feedback (D) current-current feedback
Answer: (B)
Exp: Output sample is voltage and is added at the input or current
∴ It is voltage – shunt negative feedback i.e, voltage-current negative feedback

13. In the low-pass filter shown in the figure, for a cut-off frequency of 5kHz, the value of R2
( in kΩ ) is ____________.
R2

C
1kΩ 10nF
Vi −
R1 Vo
+

 India’s No.1 institute for GATE Training  1 Lakh+ Students trained till date  65+ Centers across India
7
EC-GATE-2014 PAPER-01| www.gateforum.com

Answer: 3.18
Exp: f = 5KHz
1
Cut off frequency ( LPF ) = = 5KHz
2πR 2 C
1
⇒ R2 = = 3.18 kΩ
2π × 5 × 103 × 10 × 10−9

14. In the following circuit employing pass transistor logic, all NMOS transistors are identical
with a threshold voltage of 1 V. Ignoring the body-effect, the output voltages at P, Q and R
are,

5V 5V 5V
5V

P Q R

(A) 4 V, 3 V, 2 V (B) 5 V, 5 V, 5 V
(C) 4 V, 4 V, 4 V (D) 5 V, 4 V, 3 V
Answer: (C)
5V
Exp: Assume al NMOS are in saturation
∴ VDS ≥ ( VGS − VT )
5V
For m1
M1
(5 − Vp ) ≥ ( 5 − Vp − 1)
(5 − V ) > ( 4 − V ) ⇒ Sat
P
p p
5V
M2
∴ ID1 = k ( VGS − VT )
2

ID1 = K ( 4 − Vp ) ........ (1)


2 Q
5V
M3
For m 2 ,
I D1 = K ( 5 − VQ − 1)
2 For m 3 ,
I D3 = K ( 5 − VR − 1)
2
R
I D2 = K ( 4 − VQ ) ......( 2 )
2

∴ I D 2 = I D3
∴ ID1 = ID2
(4 − V ) = (4 − V )
2 2

(4 − V ) = (4 − V )
2 2 Q R
p Q
⇒ VR = VQ = 4V
⇒ Vp = VQ & Vp + VQ = 8
∴ Vp = VQ = VR = 4V
⇒ Vp = VQ = 4V

 India’s No.1 institute for GATE Training  1 Lakh+ Students trained till date  65+ Centers across India
8
EC-GATE-2014 PAPER-01| www.gateforum.com

15. ( ) ( )
The Boolean expression ( X + Y ) X + Y + X + Y + X simplifies to
(A) X (B) Y (C) XY (D) X+Y
Answer: (A)
Exp: Given Boolean Expression is ( X + Y ) ( X + Y ) + XY + X
As per the transposition theorem
( A + BC ) = ( A + B )( A + C )
so, ( X + Y ) ( X + Y ) = X + YY = X+0

( X + Y ) ( X + Y ) + XY + X ( )
= X + XY .X

= X + ( X + Y ) .X = X + XX. + Y.X = X + 0 + Y.X


Apply absorption theorem = X (1 + Y ) = X.1= X

16. Five JK flip-flops are cascaded to form the circuit shown in Figure. Clock pulses at a
frequency of 1 MHz are applied as shown. The frequency (in kHz) of the waveform at Q3 is
__________ .

1 1 1 J1 Q1 1 J0
J4 Q4 1 J3 Q3 J2 Q2
> clk > clk > clk > clk > clk
1 K2 1 K1
1 K4 1 K2 1 K0

clock

Answer: 62.5
Exp: Given circuit is a Ripple (Asynchrnous) counter. In Ripple counter, o/p frequency of each
flip-flop is half of the input frequency if their all the states are used otherwise o/p frequency
input frequency
of the counter is =
modulus of the counter
input frequency
So, the frequency at Q3 =
16
1×10 6
= H z = 62.5 kHz
16

17. A discrete-time signal x [ n ] = sin ( π2 n ) ,n being an integer,is

(A) periodic with period π . (B) periodic with period π 2 .


(C) periodic with period π / 2 . (D) not periodic
Answer: (D)

 India’s No.1 institute for GATE Training  1 Lakh+ Students trained till date  65+ Centers across India
9
EC-GATE-2014 PAPER-01| www.gateforum.com

Exp: Assume x [ n ] to be periodic, (with period N)


⇒ x [n ] = x [n + N]
⇒ sin ( π 2 n ) = sin ( π2 ( n + N ) )
Every frigonometric function repeate after 2π interval.
⇒ sin ( π2 n + 2πk ) = sin ( π 2 h + π 2 N )
 2k 
⇒ 2πk = π2 N ⇒ N =  
 π 
Since ‘k’ is any integer, there is no possible value of ‘k’ for which ‘N’ can be an integer, thus
non-periodic.

18. Consider two real valued signals, x(t) band-limited to [ −500 Hz, 500 Hz ] and y ( t ) band-
limited to [ −1kHz, 1kHz ] . For z ( t ) = x ( t ) . y ( t ) , the Nyquist sampling frequency (in kHz) is
__________
Answer: 3
Exp: x ( t ) is band limited to [ −500Hz, 500Hz ]
y ( t ) is band limited to [ −1000Hz, 1000Hz ]
z ( t ) = x ( t ) .y ( t )
Multiplication in time domain results convolution in frequency domain.
The range of convolution in frequency domain is [ −1500Hz, 1500 Hz ]
So maximum frequency present in z(t) is 1500Hz Nyquist rate is 3000Hz or 3 kHz

19. A continuous, linear time-invariant filter has an impulse response h(t) described by

h ( t ) = { 30 for 0≤t ≤3
otherwise

When a constant input of value 5 is applied to this filter, the steady state output is _______.
Answer: 45
x (t )
Exp: h (t ) y (t )

y(t) = x (t)* h (t)


x(t) = 5

h (t) =
3

t
3
3
y ( t ) = ∫ 3.5.dτ = 45 ( steady state output )
0

 India’s No.1 institute for GATE Training  1 Lakh+ Students trained till date  65+ Centers across India
10
EC-GATE-2014 PAPER-01| www.gateforum.com

20. The forward path transfer function of a unity negative feedback system is given by
K
G ( s) =
( s + 2 )( s − 1)
The value of K which will place both the poles of the closed-loop system at the same
location, is ______.
Answer: 2.25
K
Exp: Given G ( s ) =
( )( s − 1)
s + 2
H (s) = 1

Characteristic equation: 1 + G ( s ) H ( s ) = 0
K
1+ = 0
( s + 2 )( s − 1)
9
The poles are s1,2 = − 1 ± − 4K
4
9
If − K = 0, then both poles of the closed loop system at the same location.
4
9
So, K = ⇒ 2.25
4

21. Consider the feedback system shown in the figure. The Nyquist plot of G(s) is also shown.
Which one of the following conclusions is correct?

+ Im G ( jω )
k G (s)

−1 +1 Re G ( jω )

(A) G(s) is an all-pass filter


(B) G(s) is a strictly proper transfer function
(C) G(s) is a stable and minimum-phase transfer function
(D) The closed-loop system is unstable for sufficiently large and positive k
Answer: ( D)
Exp: For larger values of K, it will encircle the critical point (-1+j0), which makes closed-loop
system unstable.

 India’s No.1 institute for GATE Training  1 Lakh+ Students trained till date  65+ Centers across India
11
EC-GATE-2014 PAPER-01| www.gateforum.com

22. In a code-division multiple access (CDMA) system with N = 8 chips, the maximum number
of users who can be assigned mutually orthogonal signature sequences is ________
Answer: 7.99 to 8.01
chip rate
Exp: Spreading factor(SF)=
symbol rate
This if a single symbol is represented by a code of 8 chips
Chip rate =80×symbol rate
8 × symbol rate
S.F (Spreading Factor) = =8
symbol rate
Spread factor (or) process gain and determine to a certain extent the upper limit of the total
number of uses supported simultaneously by a station.

23. The capacity of a Binary Symmetric Channel (BSC) with cross-over probability 0.5 is
________
Answer: 0
Exp: Capacity of channel is 1-H(p)
H(p) is entropy function
With cross over probability of 0.5
1 1 1 1
H ( p ) = log 2 + log 2 =1
2 0.5 2 0.5
⇒ Capacity = 1 − 1 = 0

S S12 
24. A two-port network has sattering parameters given by [ S] =  11  . If the port-2 of the
S21 S22 
two-port is short circuited, the S11 parameter for the resultant one-port network is
s11 − s11 s22 + s12s21 s11 − s11 s22 − s12s21
( A) ( B)
1 + s22 1 + s22
s11 − s11 s22 + s12s21 s11 − s11 s22 + s12s21
( C) ( D)
1 − s22 1 − s22
Answer:(B)
Exp:
a1 Two port a2
b1 Network b2

b1 = s11a1 + s12 a 2
b 2 = s 21a1 + s 22 a 2
 b1   s11 s12   a1  b1
 b  = s   ; s1 =
 2   21 s 22  a 2  a1 a 2 =0

By verification Answer B satisfies.

 India’s No.1 institute for GATE Training  1 Lakh+ Students trained till date  65+ Centers across India
12
EC-GATE-2014 PAPER-01| www.gateforum.com

25. The force on a point charge +q kept at a distance d from the surface of an infinite grounded
metal plate in a medium of permittivity ∈ is
q2
(A) 0 (B) away from the plate
16π ∈ d 2
q2 q2
(C) towards the plate (D) towards the plate
16π ∈ d 2 4π ∈ d 2
Answer:(C)
+q
1 Q1Q 2
Exp: F=
4π∈ R 2 d

1 92 92
F= =
4π ∈ ( 2d ) 2 16π∈ d 2
metal plate

Since the charges are opposite polarity d

the force between them is attractive.


−q

Q.No. 26 – 55 Carry Two Marks Each

26. The Taylor series expansion of 3 sin x + 2 cos x is


x3 x3
( A ) 2 + 3x − x 2 − + ....... ( B) 2 − 3x + x 2 − + .......
2 2
x3 x3
( C) 2 + 3x + x 2 + + ....... ( D ) 2 − 3x − x 2 + + .......
2 2
Answer: (A)
 x3   x2 
Exp: 3sin x + 2cos x = 3  x − + ...  + 2 1 − + ... 
 3!   2! 
x3
= 2 + 3x − x 2 − + ...
2

+∞
∫ g ( t ) e − jωt dt = ωe −2 ω for any real value ω . If
2
27. For a Function g(t), it is given that
−∞
+∞
y ( t ) = ∫ g ( τ ) dτ, then ∫ y ( t ) dt is
t

−∞ −∞

j j
(A)0 (B)-j (C) - (D)
2 2
Answer: (B)
Exp: Given

∫ g ( t ).e
− jwt
dt = ω.e−2w ( let G ( jω) )
2

−∞

⇒ ∫ g ( t ) dt = 0
−∞

 India’s No.1 institute for GATE Training  1 Lakh+ Students trained till date  65+ Centers across India
13
EC-GATE-2014 PAPER-01| www.gateforum.com

t
y(t) = ∫ g ( z ).dz ⇒ y ( t ) = g ( t ) * u ( t ) u ( t ) in unit step function 
−∞

⇒ Y ( jω) = G ( jω) .U ( jω)



Y ( jω) = ∫ y ( t ).e
− jω t
dt
−∞

 1 
⇒ Y ( j0 ) = ∫ y ( t ) dt = ω.e
−2w 2
 jω + πδ ( ω)   ω = 0
−∞  
1
= = −j
j

28. The volume under the surface z(x, y) = x + y and above the triangle in the x-y plane defined
by {0 ≤ y ≤ x and 0 ≤ x ≤ 12} is___________.
Answer: 864
12 x

Exp: Volume = ∫∫ Z ( x, y ) dydx = ∫ ∫ ( x + y ) dydx


R x =0 y=0

x 12

12
y2 
12
3 3  x3 
= ∫  xy +  .dx = ∫ x 2 dx =   = 864
x =0 
2 0 0
2 2  3 0

29. Consider the matrix:


0 0 0 0 01
0 0 0 0 10
0 0 0 1 00
J6 =
0 0 10 0 0
0 1 0 0 0 0
1 0 0 0 0 0
Which is obtained by reversing the order of the columns of the identity matrix I 6 .
Let P = I 6 + αJ 6 , where α is a non-negative real number. The value of α for which det(P) =
0 is ___________.
Answer: 1
1 0   0 1 1 α 
Exp: Consider, ( i ) Let P = I 2 + αJ 2 =   + α = 
0 1  1 0   α 1 
⇒ P = 1 − α2
1 0 0 α 
0 1 α 0 
( ii ) Let P = I4 + α J 4 =  
0 α 1 0
 
α 0 0 1

 India’s No.1 institute for GATE Training  1 Lakh+ Students trained till date  65+ Centers across India
14
EC-GATE-2014 PAPER-01| www.gateforum.com

1 α 0 0 1 α
P = (1 ) α 1 0 − (α ) 0 α 1
0 0 1 α 0 0

= (1 − α 2 ) − ( α )  α (1 − α 2 )  = (1 − α 2 )
2

S im ilarly , if P = I 6 + α J 6 th e n w e g et

P = (1 − α 2 )
3

∴ P = 0 ⇒ α = − 1, 1
∵ α is n o n n e g ativ e
∴α =1

30. A Y-network has resistances of 10Ω each in two of its arms, while the third arm has a
resistance of 11Ω in the equivalent ∆ − network, the lowest value ( in Ω ) among the three
resistances is ______________.
Answer: 29.09Ω
Exp:
10Ω 10Ω
X Z

10Ω 11Ω
11Ω
10Ω
Y
Star Connection Delta Connection

X = 29.09Ω
y = 32Ω
(10 )(10 ) + (10 )(11) + (10 )(11)
X= Ω
11
z = 32 Ω
( )( ) ( )(11) + (10 )(11)
10 10 + 10
y= Ω
10
(10 )(10 ) + (10 )(11) + (10 )(11)
z= Ω
10
i.e, lowest value among three resistances is 29.09Ω

31. A 230 V rms source supplies power to two loads connected in parallel. The first load draws
10 kW at 0.8 leading power factor and the second one draws 10 kVA at 0.8 lagging power
factor. The complex power delivered by the source is
(A) (18 + j 1.5) kVA (B) (18 - j 1.5) kVA
(C) (20 + j 1.5) kVA (D) (20 - j 1.5) kVA

 India’s No.1 institute for GATE Training  1 Lakh+ Students trained till date  65+ Centers across India
15
EC-GATE-2014 PAPER-01| www.gateforum.com

Answer: (B)
Exp:
+

L L
o o
230V a a
d d
I II

Load 1:
P = 10 kw 

cos φ = 0.8  SI = P − jQ = 10 − j7.5 KVA
Q = P tan φ = 7.5 KVAR 

Load 2: S = 10 KVA
Q
cos φ = 0.8 sin φ =
S
P
cos φ =
S
P
0.8 = → P = 8kw Q = 6KVAR
10
SI = P + jQ = 8 + j6
Complex power delivered by the source is SI + SII = 18 − j1.5 KVA

32. A periodic variable x is shown in the figure as a function of time. The root-mean-square (rms)
value of x is_______.

x
1

0
t
T/2 T/2

Answer: 0.408

 India’s No.1 institute for GATE Training  1 Lakh+ Students trained till date  65+ Centers across India
16
EC-GATE-2014 PAPER-01| www.gateforum.com

T
1
( x ( t ) ) dt
2
Exp: x rms =
T0∫
2 X
 t 0≤ t ≤ 2
T
x(t) = T
0 T ≤ t ≤T
 2 1

T2 2 T 
1 2 
 .t  .dt + ∫ ( 0 ) .dt 
T  ∫0  T 
=
2
t
 ( 0,0) T T
 T
2  2
T
1 4  t3  2
= .  
T T2  3 0
4 T3 1
x rms = . ⇒ ⇒ 0.408
3T 3 8 6

33. In the circuit shown in the figure, the value of capacitor C(in mF) needed to have critically
damped response i(t) is____________.

40 Ω 4H C
+ −
i (t) VO

Answer: 10mF
Exp: By KVL,
di ( t )
1
v ( t ) = Ri ( t ) + L. i ( t ) dt
C∫
+
dt
Differentiate with respect to time,
R .di ( t ) R di ( ti ) i ( t )
0 = + . + = 0
dt 2 L dt LC
d 2i ( t ) R di ( t ) i ( t )
+ . + = 0
dt 2 L dt LC
−R
2
R 4
±   −
L  L  LC
D1,2 =
2
−R
2
 R  1
D1,2 = ±   −
2L  2L  LC
For critically damped response,
2
 R  1 4L
  = ⇒ C= 2 F
 2L  LC R
Given, L=4H; R= 40Ω
4× 4
C = ⇒ 10mF
( 40 )
2

 India’s No.1 institute for GATE Training  1 Lakh+ Students trained till date  65+ Centers across India
17
EC-GATE-2014 PAPER-01| www.gateforum.com

34. A BJT is biased in forward active mode, Assume VBE = 0.7V, kT / q = 25mV and reverse
saturation current IS = 10−13 A. The transconductance of the BJT (in mA/V) is ________.
Answer: 5.785
KT
Exp: VBE = 0.7V, = 25mV, Is = 10−13
q
IC
Transconductance, g m =
VT
IC = IS  eVBE /VT − 1
= 10−13 e0.7/ 25mV − 1 = 144.625mA
IC 144.625 mA
∴gm = = = 5.785 A / V
VT 25 mV

35. The doping concentrations on the p-side and n-side of a silicon diode are 1 × 1016 cm −3 and
1 × 1017 cm −3 , respectively. A forward bias of 0.3 V is applied to the diode. At T = 300K, the
kT
intrinsic carrier concentration of silicon n i = 1.5 × 1010 cm −3 and = 26mV. The electron
q
concentration at the edge of the depletion region on the p-side is
(A) 2.3 × 109 cm −3 (B) 1 × 1016 cm −3 (C) 1 × 1017 cm −3 (D) 2.25 × 106 cm −3
Answer:(A)
n i 2 Vbi /VT
Exp: Electron concentration, n  e
NA

=
(1.5 × 10 ) 10 2

e0.3/26mV
1 × 1016
= 2.3 × 109 / cm 3

36. A depletion type N-channel MOSFET is biased in its linear region for use as a voltage
controlled resistor. Assume threshold voltage
−8
VTH = 0.5V, VGS = 2.0 V, VDS = 5V, W / L = 100, COX = 10 F / cm and µ n = 800cm / V − s .
2 2

The value of the resistance of the voltage controlled resistor ( in Ω ) is ________.


Answer:500
Exp: Given VT = −0.5V; VGS = 2V; VDS = 5V; W = 100; Cθx = 10−8 f / cm
L
µ n = 800cm 2 / v − s
1 W
I D = µ n C0 x  2 ( VGS − VT ) VDS − VDS2 
2 L 
−1 −1
 ∂I D   ∂ 1 W 
  = rds   µ n C0x  2 ( VGS − VT ) VDS − VDS2   
 ∂VDS   ∂VDS  2 L 

 India’s No.1 institute for GATE Training  1 Lakh+ Students trained till date  65+ Centers across India
18
GATE ESE PSU’s 2019-20
ECE ENGINEERING
GATE ECE 2003-2019 SOLVED

GATE ECE 2003-2019 SOLVED Detail Solution

CONTENT COVERED:
1.Theory Notes
2.Explanation
3.Derivation
4.Example
5.Shortcut & Formula Summary
6.Previous year Paper Q. Sol.
Noted-: Single Source Follow, Revise
Multiple Time Best key of Success
1
Page

http://www.orbitmentor.com [email protected]
EC-GATE-2014 PAPER-01| www.gateforum.com

−1
 W W 
= µ n C0 x ( VGS − VT ) − µ n C0x VDS 
 L L 

1
⇒ rds =
W
µ n C0 x ( VGS − VT − VDs )
L
1
= = 500Ω
800 × 10 × 100 ( 2 + 0.5 − 5 )
−8

37. In the voltage regulator circuit shown in the figure, the op-amp is ideal. The BJT has
VBE = 0.7 V and β = 100, and the zener voltage is 4.7V. For a regulated output of 9 V, the
value of R ( in Ω ) is ______ .

VI = 12 V V0 = 9 V

+ 1kΩ
1kΩ

Vz = 4.7 V R

V = 12V 9V
i
Answer:1093
Exp: Given VBE = 0.7V, β = 100, VZ = 4.7V, V0 = 9V + 1K

R
VR = 9 ×
R + 1k VR
R
4.7 = 9 × (∵ VR = Vz ) Vz R
R + 1k
R = 1093 Ω

38. In the circuit shown, the op-amp has finite input impedance, infinite voltage gain and zero
input offset voltage. The output voltage Vout is
R2
(A) − I 2 ( R 1 + R 2 )

(B) I 2 R 2 R1 l1

Vout
(C) I1 R 2 l2 +

(D) − I1 ( R 1 + R 2 )

 India’s No.1 institute for GATE Training  1 Lakh+ Students trained till date  65+ Centers across India
19
EC-GATE-2014 PAPER-01| www.gateforum.com

Answer: (C)
Exp: Given, Zi = ∞
A 0L = ∞
Vi0 = 0 R2

V2 = ( R1 / /R 2 ) I1
R1 I1
R 1R 2
= I1 ...... (1) −
R1 + R 2 V2 V0
+
KCL at inverting node V1

V2 V2 − V0
+ =0 (∴ Zi = ∞ )
R1 R2
V0 1 1 
= V2  + 
R2  R1 R 2 
V0  R 1R 2   R 2 + R 1 
=  I1  
R 2  R 1 + R 2   R 1R 2 
⇒ V0 = I1R 2

39. For the amplifier shown in the figure, the BJT parameters are VBE = 0.7 V, β = 200, and
thermal voltage VT = 25mV. The voltage gain ( v 0 / v i ) of the amplifier is _______.

VCC = +12V

RC
R1 5kΩ
33kΩ vo
1 µF 1 µF
vi

R2 RS
11kΩ 10Ω

R E1 CE
1mF
1k Ω

Answer: -237.76
Exp: VBE = 0.7V, β = 200, VT = 25mV
DC Analysis:
11k
VB = 12 × = 3V
11k + 33k
VE = 3 − 0.7 = 2.3V
2.3
IE = = 2.277 mA
10 + 1k

 India’s No.1 institute for GATE Training  1 Lakh+ Students trained till date  65+ Centers across India
20
EC-GATE-2014 PAPER-01| www.gateforum.com

I B = 11.34 µA
IC = 2.26 mA
25mV
re = = 10.98 Ω
2.277 mA
V −βR C −200 × 5k
AV = 0 = =
Vi β re + (1 + β )( R s ) 200 × 10.98 + ( 201)10
A V = −237.76

40. The output F in the digital logic circuit shown in the figure is

XOR
X
Y AND

Z
XNOR

( A ) F = XYZ + XYZ ( B) F = XYZ + XYZ


( C) F = XYZ + XYZ ( D ) F = XYZ + XYZ
Answer: (A)
Exp: XOR
X
Y

Z
XNOR

Assume dummy variable K as a output of XOR gate K = X ⊕ Y = XY + XY


F = K. ( K  Z )
= ( KZ + K.Z )
= K. KZ + K.K.Z
(
= 0 + K.Z ∵ K. K = 0 and K.K = K )
Put the value of K in above expression
F = ( XY + XY ) Z
= XYZ + XYZ

 India’s No.1 institute for GATE Training  1 Lakh+ Students trained till date  65+ Centers across India
21
EC-GATE-2014 PAPER-01| www.gateforum.com

41. Consider the Boolean function, F ( w, x, y,z ) = wy + xy + wxyz + wxy + xz + xyz. which one
of the following is the complete set of essential prime implicants?
(A) w, y, xz, x z (B) w, y, xz (C) y, x y z (D) y, xz,xz
Answer: (D)
Exp: Given Boolean Function is
F ( w, x, y, z ) = wy + xy + wxyz + wxy + xz + xyz
By using K-map

xz
yz
wx
00 01 11 10

00 1 1 1
01 1 1 1

11 1 1 1

10 1 1 1 y

xz
So, the essential prime implicants (EPI ) are y, xz, xz

42. The digital logic shown in the figure satisfies the given state diagram when Q1 is connected
to input A of the XOR gate.

S=0

S =1
A 00 01
D1 Q1
D2 Q2 S=0
S =1 S =1
> S > S=0
Q1 Q2
CLK
10 11
S =1

S=0

Suppose the XOR gate is replaced by an XNOR gate. Which one of the following options
preserves the state diagram?
(A) Input A is connected to Q 2
(B) Input A is connected to Q2
(C) Input A is connected to Q1 and S is complemented
(D) Input A is connected to Q1
Answer: (D)

 India’s No.1 institute for GATE Training  1 Lakh+ Students trained till date  65+ Centers across India
22
EC-GATE-2014 PAPER-01| www.gateforum.com

Exp: The input of D2 flip-flop is


D 2 = Q1s + Q1 s (∵ A = Q1 )

The alternate expression for EX-NOR gate is = A ⊕ B = A ⊕ B = A ⊕ B


So, if the Ex-OR gate is substituted by Ex-NOR gate then input A should be connected to Q1

D 2 = Q1S + Q1 S = Q1S + Q1 .S (∵ A = Q1 )
= Qi S + Q1 .S

n n
 1   1
43. Lex x [ n ] =   u ( n ) −  −  u ( − n − 1) . The Region of Convergence (ROC) of the z-
 −9   3
transform of x[n]
1 1 1 1
(A) is z > (B) is z < (C) is > z > (D) does not exist.
9 3 3 9
Answer: (C)
 −1   −1 
n n

Exp: Given x [ n ] =   u [ n ] −   u [ −n − 1]
 9   3 
 −1 
h
1
for   u [ n ] R oc in z >
 9  9
(Right sided sequence, R oc in exterior of circle of radius 1 )
9
1 1
Thus overall R oc in <z<
9 3
 πn 
44. Consider a discrete time periodic signal x[n] = sin   . Let a k be the complex Fourier
 s 
series coefficients of x[n]. The coefficients { a k } are non-zero when k = Bm ± 1, where m is
any integer. The value of B is_________.
Answer: 10
 πn 
Exp: Given x [ n ] = sin   ; N = 10
 5 
⇒ Fourier series co-efficients are also periodic with period N = 10
1 j 210π n −1 − i 210π n
x [n] = e e
2j 2j
1 −1 −1
a1 = ; a −1 = ⇒ a −1 = a −1+10 = a 9 =
2j 2j 2j
a1 = a1 + 10  a1 = a1 + 20
 or
a −1 = a −1 + 10  a −1 = a −1 + 20
⇒ k = 10 m + 1 or k = 10.m − 1 ⇒ B = 10

 India’s No.1 institute for GATE Training  1 Lakh+ Students trained till date  65+ Centers across India
23
EC-GATE-2014 PAPER-01| www.gateforum.com

45. A system is described by the following differential equation, where u(t) is the input to the
system and y(t) is the output of the system.

y ( t ) + 5y ( t ) = u ( t )
.

When y(0) = 1 and u(t) is a unit step function, y(t) is


(A) 0.2 + 0.8e −5t (B) 0.2 − 0.2e −5t (C) 0.8 + 0.2e −5t (D) 0.8 − 0.8e −5t
Answer: (A)
Exp: Given y ( t ) + 5y ( t ) = u ( t ) and y ( 0 ) =1; u ( t ) is a unit step function.
Apply Laplace transform to the given differential equation.
1
S y ( s) − y ( 0) + 5 y (s ) =
s
1   dy  
y ( s ) [ s + 5] = + y ( 0) L   = s y ( s ) − y ( 0)  L u ( t ) = 1  
s   dt     s 
1
+1
y (s) = s
( s + 5)
( s +1) A B
y (s) = ⇒ +
s ( s + 5) s s+ 5
A= 1 ; B = 4
5 5
1 4
y (s) = +
5s 5 ( s + 5 )
Apply inverse Laplace transform,
1 4 −5t
y(t) = + e
5 5
y ( t ) = 0.2 + 0.8e −5t

46. Consider the state space model of a system, as given below


. 
 x 1   −1 1 0   x1  0  x1 
.    x  + 4  u; y = 1 1 1  x 
 x 2  =  0 −1 0   2   [ ] 2
 .   0 0 −2   x 3   0   x 3 
x 3   
 
The system is
(A) controllable and observable
(B) uncontrollable and observable
(C) uncontrollable and unobservable
(D) controllable and unobservable
Answer: (B)

 India’s No.1 institute for GATE Training  1 Lakh+ Students trained till date  65+ Centers across India
24
EC-GATE-2014 PAPER-01| www.gateforum.com

Exp: From the given state model,


 −1 1 0  0 
A =  0 −1 0  B =  4 
  c = [1 1 1]
 0 0 −2   0 

Controllable: Q c = c =  B AB A 2 B 

if Q c ≠ 0 → controllable

 0 4 −8
Q c =  4 −4 4  ⇒ Q c = 0
 0 0 0 
∴ uncontrollable
 C 
Observable : Q 0 =  CA 
 CA 2 

If Q 0 ≠ 0 → observable

1 1 1
Q 0 =  −1 0 −2  ⇒ Q 0 =1
 1 −1 4 
∴ Observable
The system is uncontrollable and observable

10
47. The phase margin in degrees of G ( s ) = calculated using the
( s + 0.1)( s + 1) + ( s + 10)
asymptotic Bode plot is_______.
Answer: 48
10
Exp: G (s) =
( s + 0.1)( s +1)( s +10 )
10
G (s) =
 s   s
0.1 1 +  [ 1+ s ]  1+  .10
 0.1   10 
10
G (s) =
[1+10s][ 1+ s][ 1+ 0.1s]
10
By Approximation, G ( s ) =
[10s +1]

 India’s No.1 institute for GATE Training  1 Lakh+ Students trained till date  65+ Centers across India
25
EC-GATE-2014 PAPER-01| www.gateforum.com

10
Phase Margin = θ =180 + GH ω=ωgc ωgc = 1 =
100ω2 + 1
 10 × 0.99 
= 180 − tan −1   99
 1  =100ω2 =

Phase Margin = 95° .73
99
⇒ ω2 ⇒ ωgc = 0.9949r / sc

Asymptotic approximation, Phase margin = φ − 45°  48

1
48. For the following feedback system G ( s ) = . The 2% settling time of the step
( s + 1) + ( s + 2 )
response is required to be less than 2 seconds.

r + C ( s) G ( s)
y

Which one of the following compensators C(s) achieves this?


 1   0.03   s + 8
( A) 3
 s + 5 
( B) 5
 s
+ 1

( C) 2 ( s + 4) ( D) 4
 s + 3
Answer: (C)
Exp: By observing the options, if we place other options, characteristic equation will have 3rd order
one, where we cannot describe the settling time.
If C ( s ) = 2 ( s + 4 ) is considered
The characteristic equation, is
s 2 + 3s + 2 + 2s + 8 = 0
⇒ s 2 + 5s +10 = 0
Standard character equation s 2 + 2 ξωn s + ω2n = 0
ω2n = 10; ξωn = 2.5
4
Given, 2% settling time, < 2 ⇒ ξw n > 2
ξw n
49. Let x be a real-valued random variable with E[X] and E[X2] denoting the mean values of X
and X2, respectively. The relation which always holds true is

( A ) ( E [ X ]) ( B) E  X 2  ≥ ( E [ X ])
2 2
> E  X 2 

( C) E  X 2  = ( E [ X ]) ( D ) E  X 2  > ( E [ X ])
2 2

Answer: (B)
Exp: V ( x ) = E ( x 2 ) − {E ( x )} ≥ 0 i.e., var iance cannot be negative
2

∴ E ( x 2 ) ≥ {E ( x )}
2

 India’s No.1 institute for GATE Training  1 Lakh+ Students trained till date  65+ Centers across India
26
EC-GATE-2014 PAPER-01| www.gateforum.com

50. Consider a random process X ( t ) = 2 sin ( 2 πt + ϕ ) , where the random phase ϕ is uniformly
distributed in the interval [ 0, 2 π] . The auto-correlation E  X ( t1 ) X ( t 2 ) 

( A ) cos ( 2π ( t1 + t 2 ) ) ( B ) sin ( 2π ( t1 − t 2 ) )
( C ) sin ( 2π ( t1 + t 2 ) ) ( D ) cos ( 2π ( t1 − t 2 ) )
Answer: (D)
Exp: Given X(t) = 2 sin ( 2πt + φ )
f φ (θ)
φ in uniformly distributed in the interval [0, 2π ]
1


E [ x(t1 )x(t 2 ) ] = ∫ 2 sin(2 πt1 + θ) 2 sin ( 2πt 2 + θ ) f φ (θ)dθ
0

2π 1
= 2 ∫ sin ( 2πt1 + θ ) sin ( 2πt 2 + θ ).
.dθ
0 2π 0 2π θ
1 2π 1 2π
= ∫
2π 0
sin(2π(t1 + t 2 ) + 2θ)dθ +
2π ∫0
cos(2π(t1 − t 2 )dθ

First integral will result into zero as we are integrating from 0 to 2 π.


Second integral result into cos {2π(t1 − t 2 )}

⇒ E [ X(t1 )X(t 2 )] = cos ( 2π(t1 − t 2 )

51. Let Q ( γ) be the BER of a BPSK system over an AWGN channel with two-sided noise
power spectral density N0/2. The parameter γ is a function of bit energy and noise power
spectral density.
A system with tow independent and identical AWGN channels with noise power spectral
density N0/2 is shown in the figure. The BPSK demodulator receives the sum of outputs of
both the channels.
AWGN
Channel1
0 /1 BPSK BPSK 0 /1
Modulator +
Demodulator
AWGN
Channel 2

( )
If the BER of this system is Q b γ , then the value of b is _____________.
Answer: 1.414
  
 2E    E 
Exp: Bit error rate for BPSK = Q   . Q  
 NO    N O  
  2 
2E
⇒Y=
NO

 India’s No.1 institute for GATE Training  1 Lakh+ Students trained till date  65+ Centers across India
27
EC-GATE-2014 PAPER-01| www.gateforum.com

φ2 ( t )
NO
Function of bit energy and noise PSD
2
Counterllation diagram of BPSK
−a a φ1 ( t )
Channel is A WGN which implies noise sample as independent
Let 2x + n1 + n 2 = x1 + n1

where x1 = 2x
x + n1
+
n = n1 + n 2
1

x 2x + n1 + n2
+
 2E  1
noise in channel 1
Now Bit error rate = Q  
 N O1  x + n2
  +

E1 is energy in x1
noise in channel 2
N O1 is PSD of h1
E1 = 4E [as amplitudes are getting doubled]
N O1 = N O [independent and identical channel]
 4E   2E 
⇒ Bit error rate = Q   = Q 2  ⇒ b = 2 or 1.414
 N   N
 O   O 

52. A fair coin is tossed repeatedly until a ‘Head’ appears for the first time. Let L be the number
of tosses to get this first ‘Head’. The entropy H(L) in bits is _________.
Answer: 2
Exp: In this problem random variable is L
L can be 1, 2,..............

1
P {L = 1} =
2
1
P {L = 2} =
4
1
P {L = 3} =
8
1 1 1 1 1 1 1 1 1
H {L} = log 2 + lg o 2 + log 2 + ......... = 0 + 1. + 2. + 3. + .........
2 1 4 1 8 1 2 4 8
2 4 8
[ Arithmatic gemometric series summation]
1 .1
2 2
= + =2
1− 1
2
 1
2 1 −  
 2

 India’s No.1 institute for GATE Training  1 Lakh+ Students trained till date  65+ Centers across India
28
EC-GATE-2014 PAPER-01| www.gateforum.com

53. In spherical coordinates, let aˆ θ .aˆ φ denote until vectors along the θ, φ directions.
100
E= sin θ cos ( ωt − βr ) aˆ θ V / m and
r
0.265
H= sin θ cos ( ωt − βr ) aˆ φ A / m
r
represent the electric and magnetic field components of the EM wave of large distances r
from a dipole antenna, in free space. The average power (W) crossing the hemispherical shell
located at r = 1km,0 ≤ θ ≤ π / 2 is _______
Answer: 55.5
100
Exp: Eθ = sin θ e − Jβr
r
0.265
HQ = sin θ e− Jβr
r
1
Pavg = ∫ E θ H*Q .ds
2 s
1 100 ( 0.265 ) 2 2
= ∫ sin θ r sin θ dθ dφ
2 s r2
1
Pavt = ∫ ( 26.5 ) sin 2 dθ dφ
2 s
π

( 3 ) ( 2π )
2
=13.25 ∫
θ= 0
sin 3 θdθ ∫
Q=0
dφ = 13.25. 2

P = 55.5 w

54. For a parallel plate transmission line, let v be the speed of propagation and Z be the
characteristic impedance. Neglecting fringe effects, a reduction of the spacing between the
plates by a factor of two results in
(A) halving of v and no change in Z (B) no change in v and halving of Z
(C) no change in both v and Z (D) halving of both v and Z
Answer: (B)
276 d
Exp: Zo = log  
∈r r
d → distance between the two plates
so, zo – changes, if the spacing between the plates changes.
1
V= → independent of spacing between the plates
LC

 India’s No.1 institute for GATE Training  1 Lakh+ Students trained till date  65+ Centers across India
29
EC-GATE-2014 PAPER-01| www.gateforum.com

λ
55. The input impedance of a section of a lossless transmission line of characteristic
8
impedance 50Ω is found to be real when the other end is terminated by a load
Z L ( = R + jX ) Ω. if X is 30 Ω, the value of R ( in Ω ) is _________
Answer: 40
Exp: Given,  = λ
s
Zo = 50Ω

(
Zin  = λ
8 )  Z + JZo 
= Zo  L 
 Zo + KZL 
 Z + J50   Z L + J50 50 − JZ L 
Zin = 50  L  = 50  × 
 50 + JZ L   50 + JZL 50 − JZ L 
 50ZL + 50 ZL + J ( 502 − Z2L ) 
Zin = 50  
 502 + Z2L 
Given , Zin → Re al
So, I mg ( Zin ) = 0
502 − Z2L = 0
Z2L = 502
R 2 + X 2 = 502
R 2 = 502 − X 2 = 502 − 302
R = 40Ω

 India’s No.1 institute for GATE Training  1 Lakh+ Students trained till date  65+ Centers across India
30
EC-GATE-2014 PAPER-02| www.gateforum.com

Q. No. 1 – 5 Carry One Mark Each

1. Choose the most appropriate word from the options given below to complete the following
sentence.
Communication and interpersonal skills are_____ important in their own ways.
(A) each (B) both (C) all (D) either
Answer: (B)

2. Which of the options given below best completes the following sentence?
She will feel much better if she ________________.
(A) will get some rest (B) gets some rest
(C) will be getting some rest (D) is getting some rest
Answer: (B)

3. Choose the most appropriate pair of words from the options given below to complete the
following sentence.
She could not _____ the thought of _________ the election to her bitter rival.
(A) bear, loosing (B) bare, loosing (C) bear, losing (D) bare, losing
Answer: (C)

4. A regular die has six sides with numbers 1 to 6 marked on its sides. If a very large number of
throws show the following frequencies of occurrence: 1 → 0.167; 2 → 0.167; 3 → 0.152; 4 →
0.166; 5 → 0.168; 6 → 0.180. We call this die
(A) irregular (B) biased (C) Gaussian (D) insufficient
Answer: (B)
Exp: For a very large number of throws, the frequency should be same for unbiased throw. As it
not same, then the die is baised.

5. Fill in the missing number in the series.


2 3 6 15 ___ 157.5 630
Answer: 45
Exp:
2 3 6 15 45 157.5 630

1.5 2 2.5 3 3.5 4

2nd number
is in increa sin g order as shown above
1st number

 India’s No.1 institute for GATE Training  1 Lakh+ Students trained till date  65+ Centers across India
1
EC-GATE-2014 PAPER-02| www.gateforum.com

Q. No. 6 – 10 Carry One Mark Each

6. Find the odd one in the following group


Q,W,Z,B B,H,K,M W,C,G,J M,S,V,X
(A) Q,W,Z,B (B) B,H,K,M (C) W,C,G,J (D) M,S,V,X
Answer: (C)
Exp: a W Z B
B H K N
17 23 26 2
W C G J M S V X
6 3 2 6 4 3 6 3 2
6 3 2

7. Lights of four colors (red, blue, green, yellow) are hung on a ladder. On every step of the
ladder there are two lights. If one of the lights is red, the other light on that step will always
be blue. If one of the lights on a step is green, the other light on that step will always be
yellow. Which of the following statements is not necessarily correct?
(A) The number of red lights is equal to the number of blue lights
(B) The number of green lights is equal to the number of yellow lights
(C) The sum of the red and green lights is equal to the sum of the yellow and blue lights
(D) The sum of the red and blue lights is equal to the sum of the green and yellow lights
Answer: (D)

8. The sum of eight consecutive odd numbers is 656. The average of four consecutive even
numbers is 87. What is the sum of the smallest odd number and second largest even number?
Answer: 163
Exp: Eight consecutive odd number =656
a-6, a-1, a-2, a ,a+2 ,a+4, a+6
a+8=656
a=81
Smallest m=75 … (1)
Average consecutive even numbers
a −2+a +a +2+a +4
⇒ = 87
4
⇒ a = 86
Second largest number =88
1+2=163

9. The total exports and revenues from the exports of a country are given in the two charts
shown below. The pie chart for exports shows the quantity of each item exported as a
percentage of the total quantity of exports. The pie chart for the revenues shows the
percentage of the total revenue generated through export of each item. The total quantity of
exports of all the items is 500 thousand tonnes and the total revenues are 250 crore rupees.
Which item among the following has generated the maximum revenue per kg?

 India’s No.1 institute for GATE Training  1 Lakh+ Students trained till date  65+ Centers across India
2
EC-GATE-2014 PAPER-02| www.gateforum.com

Exports Revenues

Item 6 Item 1
Item 1
Item 6
16% 11%
12%
Item5 Item2 19%
Item 2
12% 20% Item 5
20%
Item3
Item4 20%
19% Item3
22% Item 4
6% 23%

(A) Item 2 (B) Item 3 (C) Item 6 (D) Item 5


Answer: (D)
Exp: Item:2 Item:3
20 23 × 250 × 107
× 250 × 107
100 19 × 500 × 103
20
× 500 × 103
100 1.2 = Item 3
0.5 × 104 = 5 × 103 1 = Item 2
Item: 6 Item:5
19 20 5
= 1.18 = Item 6 = = 1.6 ⇒ 1.6 = Item 5
16 12 3

10. It takes 30 minutes to empty a half-full tank by draining it at a constant rate. It is decided to
simultaneously pump water into the half-full tank while draining it. What is the rate at which
water has to be pumped in so that it gets fully filled in 10 minutes?
(A) 4 times the draining rate (B) 3 times the draining rate
(C) 2.5 times the draining rate (D) 2 times the draining rate
Answer: (A)
Exp: Vhalf = 30(s) drawing rate = s
Total volume =60 S tank
(s1 )(10) − (s)10 = 30s
s1 (s) − s = 3s
s1 = 4s
s1 = 4drawing rate

 India’s No.1 institute for GATE Training  1 Lakh+ Students trained till date  65+ Centers across India
3
EC-GATE-2014 PAPER-02| www.gateforum.com

Q. No. 1 – 25 Carry One Mark Each

1. The determinant of matrix A is 5 and the determinant of matrix B is 40. The determinant of
matrix AB is ________.
Answer: 200
Exp: AB = A . B = ( 5 ) . ( 40 ) = 200

2. Let X be a random variable which is uniformly chosen from the set of positive odd numbers
less than 100. The expectation E[X]is __________.
Answer:50
Exp: X = 1,3,5,....,99 ⇒ n = 50 ( number of observations )
1 n 1 1
∴E(x) = ∑ x i = [1 + 3 + 5 + .... + 99] = ( 50 ) = 50
2

n i =1 50 50

3. For 0 ≤ t < ∞, the maximum value of the function f ( t ) = e − t − 2e −2 t occurs at


(A) t = loge4 (B) t = loge2 (C) t = 0 (D) t = loge8
Answer: (A)
Exp: f ' ( t ) = −e − t + 4e−2t = 0
1
⇒ e − t  4e − t − 1 ⇒ e− t = ⇒ t = log e4
4
and f ( t ) < 0 at t = log e
'' 4

x
 1
4. The value of lim  1 +  is
x →∞  x
(A) ln2 (B) 1.0 (C) e (D) ∞
Answer: (C)
x
 1
Exp: lim  1 +  = e ( standard limit )
x →∞
 x

5. If the characteristic equation of the differential equation


d2y dy
2
+ 2α + y = 0
dx dx
has two equal roots, then the values of α are
( A) ± 1 ( B) 0,0 ( C) ± j ( D ) ± 1/ 2
Answer: (A)
Exp: For equal roots, Discriminant B2 − 4AC = 0
⇒ 4α 2 − 4 = 0
⇒ α = ±1

 India’s No.1 institute for GATE Training  1 Lakh+ Students trained till date  65+ Centers across India
4
EC-GATE-2014 PAPER-02| www.gateforum.com

6. Norton’s theorem states that a complex network connected to a load can be replaced with an
equivalent impedance
(A) in series with a current source (B) in parallel with a voltage source
(C) in series with a voltage source (D) in parallel with a current source
Answer: (D)
Exp: Norton’s theorem

IN Zequ Load

7. In the figure shown, the ideal switch has been open for a long time. If it is closed at t=0, then
the magnitude of the current (in mA) through the 4 kΩ resistor at t = 0+ is _______.
5kΩ 4 kΩ 1kΩ
i
10 V −+ 10µF 1mH
t=0

5 kΩ 4 kΩ
Answer: 1.2 mA
Exp: For t = o+
10V + •
10 − •
i (o +) = ⇒ 1.11mA i (0+)
9K
i ( o + )  1.2 mA

8. A silicon bar is doped with donor impurities ND = 2.25 x 1015 atoms / cm3. Given the intrinsic
carrier concentration of silicon at T = 300 K is ni = 1.5 x 1010 cm-3. Assuming complete
impurity ionization, the equilibrium electron and hole concentrations are
(A) n0 = 1.5 x 1016 cm-3, p0 = 1.5 x 105 cm-3
(B) n0 = 1.5 x 1010 cm-3, p0= 1.5 x 1015 cm-3
(C) n0 = 2.25 x 1015 cm-3, p0 = 1.5 x 1010 cm-3
(D) n0 = 2.25 x 1015 cm-3, p0 = 1 x 105 cm-3
Answer: (D)
N D = 2.25 × 1015 Atom / cm3
Exp:
h i = 1.5 × 1010 / cm 3
Since complete ionization taken place,
h 0 = N D = 2.25 × 1015 / cm3

n i 2 (1.5 × 10 )
10 2

P0 = = = 1 × 105 / cm 3
n0 2.25 × 1015

 India’s No.1 institute for GATE Training  1 Lakh+ Students trained till date  65+ Centers across India
5
EC-GATE-2014 PAPER-02| www.gateforum.com

9. An increase in the base recombination of a BJT will increase


(A) the common emitter dc current gain β
(B) the breakdown voltage BVCEO
(C) the unity-gain cut-off frequency fT
(D) the transconductance gm
Answer: (B)

10. In CMOS technology, shallow P-well or N-well regions can be formed using
(A) low pressure chemical vapour deposition
(B) low energy sputtering
(C) low temperature dry oxidation
(D) low energy ion-implantation
Answer: (D)

11. The feedback topology in the amplifier circuit (the base bias circuit is not shown for
simplicity) in the figure is VCC

Rc Io
(A) Voltage shunt feedback
Vo

(B) Current series feedback


RS
(C) Current shunt feedback
RE
VS ~
(D) Voltage series feedback
Answer: (B)
Exp: By opening the output feed back signed becomes zero. Hence it is current sampling.
As the feedback signal vf is subtracted from the signal same vs it is series mixing.
12. In the differential amplifier shown in the figure, the magnitudes of the common-mode and
differential-mode gains are Acm and Ad, respectively. If the resistance RE is increased, then
(A) Acm increases
VCC
(B) common-mode rejection ratio increases
RC RC
(C) Ad increases
(D) common-mode rejection ratio decreases + V0 −
Answer: (B)
Exp: A d does not depend on R E +
Vi
A cm decreases as R E is increased −

Ad RE
∴ CMRR = = Increases I0
A cm
− VEE

 India’s No.1 institute for GATE Training  1 Lakh+ Students trained till date  65+ Centers across India
6
EC-GATE-2014 PAPER-02| www.gateforum.com

13. A cascade connection of two voltage amplifiers A1 and A2 is shown in the figure. The open-
loop gain Av0, input resistance Rin, and output resistance RO for A1 and A2 are as follows:
A1:A v 0 = 10, R in = 10kΩ, R 0 = 1kΩ
A2 : A v0 = 5, R in = 5kΩ, R 0 = 200 Ω
The approximate overall voltage gain Vout / Vin is __________.

+ +

Vin A1 A2 R L Vout
1kΩ
− −

Answer: 34.722
V0  Zi 2   R L 
Exp: Overall voltage gain, A v = = A V1 A V2   
Vi  Zi2 + Z01   R L + Z02 
 5k   1k 
= 10 × 5   
 5k + 1k  1k + 200 
A V = 34.722

14. For an n-variable Boolean function, the maximum number of prime implicants is
(A) 2(n-1) (B) n/2 (C) 2n (D) 2(n-1)
Answer: (D)
Exp: For an n-variable Boolean function, the maximum number of prime implicants = 2 ( n −1)

15. The number of bytes required to represent the decimal number 1856357 in packed BCD
(Binary Coded Decimal) form is __________ .
Answer: 4
Exp: In packed BCD (Binary Coded Decimal) typically encoded two decimal digits within a single
byte by taking advantage of the fact that four bits are enough to represent the range 0 to 9.
So, 1856357 is required 4-bytes to stored these BCD digits

16. In a half-subtractor circuit with X and Y as inputs, the Borrow (M) and Difference (N = X - Y)
are given by
(A) M = X, ⊕ Y, N = XY (B) M = XY, N = X⊕ Y
(C) M = X Y , ⊕ N = X ⊕ Y (D) M = XY N = X⊕Y
Answer: (C)

 India’s No.1 institute for GATE Training  1 Lakh+ Students trained till date  65+ Centers across India
7
EC-GATE-2014 PAPER-02| www.gateforum.com

Exp: Function Table for Half-subtractor is

X Y Difference (N) Borrow (M)


0 0 0 0
Hence, N = X ⊕ Y and m = XY
0 1 1 1
1 0 1 0
1 1 0 0

Hence, N = X ⊕ Y and m = XY

17. An FIR system is described by the system function


7 3
H ( z ) = 1 + z −1 + z −2 The system is
2 2
(A) maximum phase (B) minimum phase (C) mixed phase (D) zero phase
Answer: (C)
Exp: Minimum phase system has all zeros inside unit circle maximum phase system has all zeros
outside unit circle mixed phase system has some zero outside unit circle and some zeros
inside unit circle.
7 3
for H ( s ) = 1 + z −1 + z −2
2 2
One zero is inside and one zero outside unit circle hence mixed phase system

18. Let x[n] = x[-n]. Let X(z) be the z-transform of x[n]. If 0.5 + j 0.25 is a zero o X(z), which
one of the following must also be a zero of X(z).
(A) 0.5 - j0.25 (B) 1/(0.5 + j0.25)
(C) 1/(0.5 - j0.25) (D) 2 + j4
Answer: (B)
Exp: Given x [ n ] = x [ −n ]
⇒ x ( z ) = x ( z −1 ) [ Time reversal property in z − transform ]
⇒ if one zero is 0.5 + j0.25
1
then other zero will be
0.5 + j0.25

19. Consider the periodic square wave in the figure shown.


x
1

0 1 2 3 4 t

−1

 India’s No.1 institute for GATE Training  1 Lakh+ Students trained till date  65+ Centers across India
8
EC-GATE-2014 PAPER-02| www.gateforum.com

The ratio of the power in the 7th harmonic to the power in the 5th harmonic for this waveform
is closest in value to _______.
Answer: 0.5
1
Exp: For a periodic sequence wave, nth harmonic component is α
n
1
⇒ power in nth harmonic component is α 2
n
⇒ Ratio of the power in 7 harmonic to power in 5th harmonic for given waveform is
th

1 2
7 = 25 ≈ 0.5
1 2 49
5

20. The natural frequency of an undamped second-order system is 40 rad/s. If the system is
damped with a damping ratio 0.3, the damped natural frequency in rad/s is ________.
Answer: 38.15 r / sec
Exp: Given ωn = 40 r / sec
ξ = 0.3
ωd = ωn 1 − ξ 2

ωd = 40 1 − ( 0.3)
2

ωd = 38.15 r / sec

21. For the following sytem,


x 2 (s)
+
x1 ( s ) + s + 1 Y (s)
s +1
− s

y ( s)
When X1 ( s ) = 0 , the transfer function is
x 2 ( s)
s +1 1 s+2 s +1
( A) ( B) ( C) ( D)
s2 s +1 s ( s + 1) s ( s + 2)
Answer: (D)
Exp: If X1 ( s ) = 0
+
Y (s) X2 ( s )
1 Y (s)
; The block diagram becomes s
X2 (s ) −

1 1
Y (s ) s s ( s + 1) +S
= = ⇒
X2 (s ) 1 s (s + 2) / s + 1 s (s + 2) ( S + 1)
1+ .
s ( s + 1)

 India’s No.1 institute for GATE Training  1 Lakh+ Students trained till date  65+ Centers across India
9
EC-GATE-2014 PAPER-02| www.gateforum.com

22. The capacity of a band-limited additive white Gaussian noise (AWGN) channel is given by
 P 
C = W log 2  1 + 2  bits per second (bps), where W is the channel bandwidth, P is the
 σ w
average power received and σ2 is the one-sided power spectral density of the AWGN. For a
P
fixed 2 = 1000, , the channel capacity (in kbps) with infinite bandwidth ( W → ∞ ) is
σ
approximately
(A) 1.44 (B) 1.08 (C) 0.72 (D) 0.36
Answer: (A)
 P 
ω ln 1 + 2 
Exp:
 P 
C = lim ω log 2 1 + 2  = lim  σ ω
w →∞
 σ ω ω→∞ ln 2

 P   P 
ln 1 + 2  ln 1 + 2 
 σ ω . σ ω
lim 
1 P P
= lim = 2
ln 2 ω→∞ P σ 2
σ ln 2 ω→∞ P
σω2
σ2 ω 


This lim it is equivalent to

ln [1 + x ] P P
lim =1 = = ln 2 e 2 = 1.44 KGpa
ω→∞ x σ .ln 2
2
σ

23. Consider sinusoidal modulation in an AM system. Assuming no overmodulation, the


modulation index ( µ ) when the maximum and minimum values of the envelope,
respectively, are 3 V and 1 V, is ________.
Answer: 0.5
A ( t )max − A ( t ) min
Exp: µ=
A ( t ) max + A ( t ) min
3 −1 1
µ= = = 0.5
3 +1 2

24. To maximize power transfer, a lossless transmission line is to be matched to a resistive load
impedance via a λ / 4 transformer as shown.

lossless transmission line


λ / 4 transformer

Z L = 50Ω Z L = 100Ω

The characteristic impedance ( in Ω ) of the λ / 4 transformer is _________.


Answer: 70.7Ω

 India’s No.1 institute for GATE Training  1 Lakh+ Students trained till date  65+ Centers across India
10
EC-GATE-2014 PAPER-02| www.gateforum.com

Exp: Here impedance is matched by using QWT λ ( 4)


∴ Z'0 = Z L Zin
= 100 × 50 = 50 2
= Z'0 = 70.7Ω
25. Which one of the following field patterns represents a TEM wave travelling in the positive x
direction?
( A ) E = +8y,ˆ H = −4zˆ ( B) E = −2y,
ˆ H = −3zˆ

( C) E + 2z,ˆ H = +2yˆ ( D) E = −3y,


ˆ H = +4zˆ
Answer: (B)
Exp: For TEM wave
Electric field (E), Magnetic field (H) and
Direction of propagation (P) are orthogonal to each other.
Here P = + a x
By verification
E = − 2a y , H = − 3a z
E × H = − a y ×− a z = + a x → P

Q. No. 26 – 55 Carry Two Marks Each

26. The system of linear equations


 2 1 3  a   5 
 3 0 1  b =  −4 has
    
 1 2 5  c   14 
(A) a unique solution (B) infinitely many solutions
(C) no solution (D) exactly two solutions
Answer: (B)
2 1 3 5 
Exp: [ A / B] = 3 0 1 −4
1 2 5 14 

2 1 3 5  2 1 3 5 
R 2 → 2R 2 − 3R 1   R3 +R 2  
 0 − 3 − 7 −23 → 0 − 3 − 7 −23
R 3 → 2R 3 − R 1
0 3 7 23  0 0 0 0 
Since, rank ( A ) = rank ( A / B ) < number of unknowns
∴ Equations have infinitely many solutions.

 India’s No.1 institute for GATE Training  1 Lakh+ Students trained till date  65+ Centers across India
11
EC-GATE-2014 PAPER-02| www.gateforum.com

27. The real part of an analytic function f(z) where z =.x + jy is given by e-y cos(x). The
imaginary part of f(z) is
(A) eycos(x) (B) e-ysin(x) (C) -eysin(x) (D) –e-ysin(x)
Answer: (B)
Exp: real part u = e− y cos x and V = ?
∂v ∂v
dv = dx + dy
∂x ∂y
∂u ∂u
= − dx + dy ( U sin g C − R equations ) = e − y cos xdx − e − y sin xdy = d  e − y sin x 
∂y ∂x
Integrating, we get V = e − y sin x

28. The maximum value of the determinant among all 2×2 real symmetric matrices with trace 14
is ________.
Answer: 49
y x
Exp: General 2 × 2 real symmetric matrix is  
x z 
⇒ det = yz − x 2 and trace is y + z = 14 ( given )
⇒ z = 14 − y .............. ( *)
Let f = yz − x 2 ( det ) = − x 2 − y 2 + 14y ( u sin g *)
Using maxima and minima of a function of two variables, we have f is maximum at
x = 0, y = 7 and therefore, maximum value of the determinant is 49


29. If r = xaˆ x + yaˆ y + zaˆ z and r = r, then div ( r 2 ∇ ( ln r ) ) = _______.

Answer: 3

∇ ( ln r ) = 2 ⇒ div ( r 2∇ ( ln r ) ) = div ( r ) = 3
r 
Exp:
r

 ∂  1  x  1 r
∇ ( ln r ) = ∑ aˆ x ∂x ( ln r ) = ∑ aˆ x  r  r  = r 2 ∑ aˆ x x = r 2 
   

30. A series LCR circuit is operated at a frequency different from its resonant frequency. The
operating frequency is such that the current leads the supply voltage. The magnitude of
current is half the value at resonance. If the values of L, C and R are 1 H, 1 F and 1Ω ,
respectively, the operating angular frequency (in rad/s) is ________.
Answer: 0.45 r/sec
Exp: The operating frequency (wx), at which current leads the supply.
i.e., ωx <ωr
again magnitude of current is half the value at resonance

 India’s No.1 institute for GATE Training  1 Lakh+ Students trained till date  65+ Centers across India
12
EC-GATE-2014 PAPER-02| www.gateforum.com

V
i,e.,. at ω = ωx ⇒ I x =
z
V
at ω= ωx ⇒ Iresonance =
R
I resonance
Ix =
2
V V
i.e., = = Z = 2R
z 2R
Given R = 1Ω; L=1H; C=1F
2
 1 
Z = R2 + − ωL  = 2
 ωc 
2
 1 
=R + 
2
− ωL  = 4
 ωc 
By substituting R, L & C values,
2
1  1
⇒ 1 +  − ω  = 4 ⇒ ω2 = 2 = 5
ω  ω
1
Assume ω2 = x, then, x + = 5
x
⇒ x 2 − 5x + 1= 0
x1, 2 = 4.791, 0.208
if x = 4.791 ⇒ ω= 2.18 r sec
if x = 0.208 ⇒ ω= 0.45r sec
But ωx < ωr
So, operating frequency ωx = 0.45 r sec

31. In the h-parameter model of the 2-port network given in the figure shown, the value of h22 (in
S) is _____ .

3Ω

3 Ω 3Ω
1 2

2Ω
1' 2'
2Ω 2Ω

Answer: 1.24
Exp: If two, π − n ws are connected in parallel,
The y-parameter are added

 India’s No.1 institute for GATE Training  1 Lakh+ Students trained till date  65+ Centers across India
13
EC-GATE-2014 PAPER-02| www.gateforum.com

i.e., y equ = y1 + y 2
 2 −1   1 −1 
 3 3  2
y1 = y2 =
− 1 2   −1 1 
 3 3   2 
5 −5 
3 6
y equ = 
 −5 5 
 6 3
 1 − y12 
 y11 y11 
h =  
 y 21 ∆y 
 y11 y11 
where ∆y = y11 y 22 − y12 − y 21
 5  5     −5  −5  
The value of h 22 = ∆y =     −     
 3  3     6  6  
∆y = 2.0833
y11 = 5 ∴ h 22 = 1.24
3

32. In the figure shown, the capacitor is initially uncharged. Which one of the following
expressions describes the current I(t) (in mA) for t > 0?
R1

1kΩ
I
5V +−
C
R2 1 µF
2 kΩ

( A) I ( t ) =
5
3
( ) 2
1 − e − t / τ , τ = msec
3
( B) I ( t ) =
5
2
( ) 2
1 − e − t / τ , τ = msec
3

( C) I ( t ) =
5
2
( )
1 − e − t / τ , τ = 3msec ( D) I ( t ) =
5
2
( )
1 − e − t / τ , τ = 3 msec

Answer: (A)
−t
Exp: ν c ( t ) = VR 2 ( t ) = Vfinal + [ Vinitial − Vfinal ] e τ

2
τ = R equ .Cequ ⇒ ×103 ×10−6
3
2
R equ = 2K  1K ⇒ KΩ
3
Cequ = 1µF

 India’s No.1 institute for GATE Training  1 Lakh+ Students trained till date  65+ Centers across India
14
EC-GATE-2014 PAPER-02| www.gateforum.com

2
τ = msec
3
Vinitial = 0volts
2 10
Vfinal = Vs.s = 5. = volts
3 3
10 10 − t τ
νR 2 ( t ) = − e
3 3
10  −t ν (t) 5 −t
νR 2 ( t ) = 1 − e τ  volts ⇒ i R 2 (t) = R 2 =  1 − e τ  mA
3 
 2K 3 

33. In the magnetically coupled circuit shown in the figure, 56 % of the total flux emanating from
one coil links the other coil. The value of the mutual inductance (in H) is ______ .
M
10 Ω

4H 5H (1/16) F
(
60 cos 4t + 300 V ~ )
Answer: 2.49 Henry
Exp: Given 56% of the total flux emanating from one coil links to other coil.
i.e, K = 56% ⇒ 0.56
M
We have, K =
L1L 2
L1 = 4H; L 2 = 5H
M = ( 0.56 ) 20 ⇒ m = 2.50H

34. Assume electronic charge q = 1.6×10-19 C, kT/q = 25 mV and electron mobility µ n = 1000
cm2/V-s. If the concentration gradient of electrons injected into a P-type silicon sample is
1×1021/cm4, the magnitude of electron diffusion current density (in A/cm2) is _________.
Answer: 4000
kJ
Exp: Given q = 1.6 × 10−19 ; = 2.5 mV, µ n = 1000cm 2 / v − s
q
D n kJ
From Einstein relation, =
µn q
⇒ D n = 25mV × 1000cm / v − S
2

⇒ 25cm 2 / s
dn
Diffuion current Density J = q D n
dx
= 1.6 × 10−19 × 25 × 1 × 1021
= 4000 A / cm 2

 India’s No.1 institute for GATE Training  1 Lakh+ Students trained till date  65+ Centers across India
15
EC-GATE-2014 PAPER-02| www.gateforum.com

35. Consider an abrupt PN junction (at T = 300 K) shown in the figure. The depletion region
width Xn on the N-side of the junction is 0.2 µm and the permittivity of silicon ( ε si ) is
1.044×10-12 F/cm. At the junction, the approximate value of the peak electric field (in kV/cm)
is _________.

P + − region N − region
X11
N A >> N D N D = 1016 / cm3

Answer: 30.66
Exp: Given x n = 0.2 µm, ∈Si = 1.044 × 10−12 F / µ n
N D = 1016 / cm3
q ND xn
Peak Electric field, E =

1.6 × 10−19 × 1016 × 0.00002
= = 30.66 KV / cm
1.044 × 10−12

36. When a silicon diode having a doping concentration of NA = 9 × 1016 cm-3 on p-side and ND =
1 × 1016 cm-3 on n-side is reverse biased, the total depletion width is found to be 3µm . Given
that the permittivity of silicon is 1.04 × 10–12 F/cm, the depletion width on the p-side and the
maximum electric field in the depletion region, respectively, are
(A) 2.7 µm and 2.3 × 105 V/cm (B) 0.3 µm and 4.15 × 105 V/cm
(C) 0.3 µm and 0.42 × 105 V/cm (D) 2.1 µm and 0.42 × 105 V/cm
Answer: (B)
Exp: Given N A = 9 × 1016 / cm3 ; N D = 1 × 1016 / cm3
Total depletion width x = x n + x p = 3 µm
∈= 1.04 × 10−12 F / cm
x n N A 9 × 1016
= =
x p N D 1 × 1016
x n = 9x p ......... (1)
Total Depletion width, x n + x p = 3µm
9x p + x p = 3 µm
x p = 0.3 µm
qN A x p 1.6 × 10−19 × 9 × 1016 × 0.3 µm
Max. Electric field, E = =
∈ 1.04 × 10−12
= 4.15 × 105 V / cm

 India’s No.1 institute for GATE Training  1 Lakh+ Students trained till date  65+ Centers across India
16
EC-GATE-2014 PAPER-02| www.gateforum.com

37. The diode in the circuit shown has Von = 0.7 Volts but is ideal otherwise.
If Vi = 5sin ( ωt ) Volts, the minimum and maximum values of VO (in Volts) are, respectively,

1kΩ
Vi Vo
R1
R2
1kΩ

+
− 2V

(A) -5 and 2.7 (B) 2.7 and 5 (C) -5 and 3.85 (D) 1.3 and 5
Answer: (C)
Exp: When Vi makes Diode 'D' OFF, V0 = Vi
∴ V0 ( min ) = −5V

When Vi makes diode 'D ' ON,

V0 =
( Vi − 0.7 − 2 ) + V + 2V
R1 + R 2
on

∴ V0 ( max ) =
( 5 − 0.7 − 2 )1k + 0.7 + 2V
1k + 1k
= 3.85V

38. For the n-channel MOS transistor shown in the figure, the threshold voltage VTh is 0.8 V.
Neglect channel length modulation effects. When the drain voltage VD = 1.6 V, the drain
current ID was found to be 0.5 mA. If VD is adjusted to be 2 V by changing the values of R
and VDD, the new value of ID (in mA) is

VDD

G
S

(A) 0.625 (B) 0.75 (C) 1.125 (D) 1.5


Answer: (C)

 India’s No.1 institute for GATE Training  1 Lakh+ Students trained till date  65+ Centers across India
17
EC-GATE-2014 PAPER-02| www.gateforum.com

Exp: Given VTh = 0.8V


1 w
When VD = 1.6V, ID = 0.5mA = µ n cos ( VDS − VTh )
2

2 L
[∵ Device is in sat ]
1 ω
⇒ µ n cos = 0.78125 × 10−3 A / V 2
2 L
When VD = 2V
1 ω
I D = µ n cos ( VDS − VTh )
2

2 L
= 078125 × 10−3 ( 2 − 0.8 )1.125mA

39. For the MOSFETs shown in the figure, the threshold voltage Vt = 2V and
1  W
K= µC∞   = 0.1 mA / V 2 . The value of ID (in mA) is ________.
2  L

VDD = +12 V

R1
10 kΩ

R2
ID
10 kΩ

VDD = −5V

Answer: 0.9
1 W 12V
Exp: Given Vt = 2V, K = µ cos = 0.1A / V 2
2 L

( )
1 W 2 10 K
I D1 = I D2 = µ n cos VGs1 − Vt
2 L
= 0.1mA / V 2 ( 5 − 2 )
2
10 K
ID
= 0.9 mA

−5V

 India’s No.1 institute for GATE Training  1 Lakh+ Students trained till date  65+ Centers across India
18
EC-GATE-2014 PAPER-02| www.gateforum.com

40. In the circuit shown, choose the correct timing diagram of the output (y) from the given
waveforms W1, W2, W3 and W4.

X1 D Q
FF1
Clk > Q
output ( y )

X2 D Q

> Q

ClK

X1

X2

W1

W2

W3

W4

(A) W1 (B) W2 (C) W3 (D) W4


Answer: (C)
Exp: This circuit has used negative edge triggered, so output of the D-flip flop will changed only
when CLK signal is going from HIGH to LOW (1 to 0)
1 1 1 1

CLK 0 0 0 0

X1

X2

Y( w 3 )

 India’s No.1 institute for GATE Training  1 Lakh+ Students trained till date  65+ Centers across India
19
EC-GATE-2014 PAPER-02| www.gateforum.com

This is a synchronous circuit, so both the flip flops will trigger at the same time and will
respond on falling edge of the Clock. So, the correct output (Y) waveform is associated to w3
waveform.

41. The outputs of the two flip-flops Q1, Q2 in the figure shown are initialized to 0,0. The
sequence generated at Q1 upon application of clock signal is

Q1
J1 Q1 J2 Q2
> >
K1 Q1 K2Q2

CLK

(A) 01110… (B) 01010… (C) 00110… (D) 01100…


Answer: (D)
Exp:

J1 ( Q 2 ) K1 ( Q 2 ) J 2 ( Q1 ) K 2 ( Q1 ) Q1 Q2
Clock
Initial → - - - - 0 0
1st CP → 1 0 0 1 1 0
2nd CP → 1 0 1 0 1 1
3rd CP →
0 1 1 0 0 1
4th CP→
0 1 0 1 0 0
So, the output sequence generated at Q1 is 01100….

42. For the 8085 microprocessor, the interfacing circuit to input 8-bit digital data (DI0 – DI7)
from an external device is shown in the figure. The instruction for correct data transfer is
3 − to − 8 7
Decoder 6 I / O Device
A2 C 5
(A) MVI A, F8H 4 Data Bus
A1 B Digital
3 DI 0 − DI 7 DO0 − D7 ( D − D )
0 7
A0 A 2 Inputs
1
0
(B) IN F8H G 2A G 2B G1 DS1 DS2
10 / M
RD
A3 A8
A
(C) OUT F8H A5 4 A9
A6 A10
A7 A11
A12 A
13
A14
(D) LDA F8F8H A15

 India’s No.1 institute for GATE Training  1 Lakh+ Students trained till date  65+ Centers across India
20
EC-GATE-2014 PAPER-02| www.gateforum.com

Answer: (D)
Exp: This circuit diagram indicating that it is memory mapped I/O because to enable the 3-to-8
decoder G 2A is required active low signal through ( Io m ) and G 2B is required active
low through (R )
D it means I/o device read the status of device LDA instruction is
appropriate with device address.
Again to enable the decoder o/p of AND gate must be 1 and Ds 2 signal required is 1 which is
the o/p of multi-i/p AND gate to enable I/O device.
So,
A15 A14 A13 A12 A11 A10 A 9 A 8 A 7 A 6 A 5 A 4 A 3 A 2 A1 A 0
1 1 1 1 1 0 0 0 1 1 1 1 1 0 0 0
   
F 8 F 8
Device address = F8F8H
The correct instruction used → LDA F8F8H

43. Consider a discrete-time signal


n for 0 ≤ n ≤ 10
x [n] = 
0 otherwise
If y[n] is the convolution of x[n] with itself, the value of y[4] is _________.
Answer: 10
 n for 0 ≤ n ≤ 10 
Exp: Given x [ n ] =  
0 elsewhere 

y [n ] = x [n ] * x [n ]
n
y [ n ] = ∑ x [ k ].x [ n − k ]
k =0

4
⇒ y [ 4] = ∑ x [ k ].x [ G − k ]
k =0

= x ( 0 ) .x ( 4 ) + x (1) x ( 3) + x ( 2 ) x ( 2 ) + x ( 3) x (1) + x ( 4 ) .x ( 0 )
= 0 + 3 + 4 + 3 + 0 = 10

44. The input-output relationship of a causal stable LTI system is given as


y [ n ] = αy [ n − 1] + β x [ n ]

If the impulse response h[n] of this system satisfies the condition Σ ∞n =0 h [ n ] = 2, the
relationship between α and β is

( A) α = 1− β/ 2 ( B) α = 1+ β / 2 ( C) α = 2β ( D) α = −2β

 India’s No.1 institute for GATE Training  1 Lakh+ Students trained till date  65+ Centers across India
21
EC-GATE-2014 PAPER-02| www.gateforum.com

Answer: (A)
Exp: Given system equation as
y [ n ] = α y [ n − 1] + β x [ n ]

y (z) β
⇒ =
x (z) 1 − α z −1

β
⇒ H (z) =
1 − α z −1

h [n] = β (α ) u [n] [ causal system ]


h


Also given that ∑ h [n] = 2
h =0

 1 
β =2
1 − α 
β
1− α =
2
β
α =1−
2


45. The value of the integral ∫ sin c 2 ( 5t ) dt is ____________.
−∞

Answer: 0.2
Exp: We can use pasrevalis theorem
sin 5πt
Let x ( t ) sin ( 5t ) =
5πt
⇒ in frequency domain
1 X (f )
5

−2.5 −2.5 f

∞ ∞ 2.5 2
1
Now, ∫ x ( t ) dt = ∫ x ( t ) df = ∫  
2 2

−2.5  
−∞ −∞
5

1 1
= × 5 = = 0.2
25 5

 India’s No.1 institute for GATE Training  1 Lakh+ Students trained till date  65+ Centers across India
22
EC-GATE-2014 PAPER-02| www.gateforum.com

46. An unforced liner time invariant (LTI) system is represented by

 x 1   −1 0   x1 
 =    
 x 2   0 −2   x 2 
If the initial conditions are x1(0) = 1 and x2(0) = -1, the solution of the state equation is

( A) x1 ( t ) = −1, x 2 ( t ) = 2 ( B) x1 ( t ) = −e − t , x 2 ( t ) = 2e − t

(C) x1 ( t ) = e − t , x 2 ( t ) = −e−2t ( D) x1 ( t ) = −e − t , x 2 ( t ) = −2e − t

Answer: (C)

Exp: Solution of state equation of X ( t ) = L−1 SI − A −1  .X ( 0 )

 1  −1 0 
X ( 0) =   A =  
 −1  0 − 2

−1
S + 1 0 
[SI − A ]
−1
=
 0 S + 2 

1 S + 2 0
=
(S + 1)(S + 2 )  0 S + 1

 1 
 0 
[SI − A ] =  S + 1
−1

 0 1 
 S + 2 

 −1  1  
L  S + 1 0 
 
L ( SI − A )  = 
−1
−1  
   −1  1 

 0 L   
 S + 2 

e− t 0 
L−1 ( SI − A )  = 
−1
  0 
 e−2t 

 X1 ( t )  e − t 0   1
 =  
 X 2 ( t )   0 e −2t   −1

 X1 ( t )   −e t  X1 ( t ) = e− t
  =  −2t  ∴
 X 2 ( t )   −e  X 2 ( t ) = −e −2t

 India’s No.1 institute for GATE Training  1 Lakh+ Students trained till date  65+ Centers across India
23
EC-GATE-2014 PAPER-02| www.gateforum.com

47. The Bode asymptotic magnitude plot of a minimum phase system is shown in the figure.

26.02

G ( jω )
6.02
( dB)
0
0.1 1 2 10 20
− 6.02
ω ( rad / s ) in log scale

If the system is connected in a unity negative feedback configuration, the steady state error of
the closed loop system, to a unit ramp input, is_________.
Answer: 0.50
Exp:

( −20 dB / dec )
26.02
G ( jw )

dB
6.02
20
0.1 1 2 10 w (r / sec )

−6.02

( −20 db (dec ) )

→ Due to initial slope , it is a type-1 system, and it has non zero velocity error coefficient
(KV )
→ The magnitude plot is giving 0dB at 2r/sec.
Which gives k v
∴kv = 2

A
The steady state error ess =
kv

given unit ramp input; A = 1

1
ess =
2

ess = 0.50

 India’s No.1 institute for GATE Training  1 Lakh+ Students trained till date  65+ Centers across India
24
EC-GATE-2014 PAPER-02| www.gateforum.com

48. Consider the state space system expressed by the signal flow diagram shown in the figure.

C3

1 S−1 S−1 S−1


u x3 x2 x1 c1 y
a3
a2
a1

The corresponding system is


(A) always controllable (B) always observable
(C) always stable (D) always unstable
Answer: (A)
Exp: From the given signal flow graph, the state model is
X  0 1
1 0   X1   0 
      
 X 2  = 0 0 1   X 2  + 0 u
X  a a a1   X 3  1 
 3  3 2
 X1 
Y = [ C1 C 2 C3 ]  X 2 
 X 3 
0 1 0 0
A = 0 0 1  ; B =  0  ;C = [ C1 C 2 C3 ]
 
a 3 a 2 a1  1 
Controllability:
Q c =  B AB A 2 B
0 0 1 
 
QC = 0 1 a1 
1 a1 a 2 + a12 

QC = 1 ≠ 0

Observability
C   C1 C2 C3 
   
Q O =  CA  ⇒  a 3c3 c1 + a 2 c3 c 2 + a1c3 
 
 2
 CA  c 2 a 3 + c3 ( a1a 3 ) a 2 c 2 + c 3 ( a 1a 2 + a 3 ) c1 + a1c 2 + c3 ( a1 + a 2 ) 
2

Q 0 ⇒ depends on a1 ,a 2 ,a 3 & c1 & c 2 & c3


.
It is always controllable

 India’s No.1 institute for GATE Training  1 Lakh+ Students trained till date  65+ Centers across India
25
EC-GATE-2014 PAPER-02| www.gateforum.com

49. The input to a 1-bit quantizer is a random variable X with pdf f x ( x ) = 2e −2x for x ≥ 0 and
f x ( x ) = 0 for x < 0 , for x < 0 For outputs to be of equal probability, the quantizer threshold
should be _____.
Answer: 0.35
Exp: one bit
X
Quantizer Q (x )

One bit quantizer will give two levels.


1
Both levels have probability of
2
Pd of input X is

fx ( π )

xT

Let x T be the thsuhold

 x1 x ≥ xT 
Q(x) = 
x2 x < x T 

Where x1 and x 2 are two levels

P {Q ( r ) = x1 } =
1
2

1
⇒ ∫ 2.e −2x dx =
xT 2

e −2x 1
2. =
−2 xT
2
1
−e −2∞ + e −2x T =
2
1
e −2x T =
2
1
− 2x T = ln
2
− 2x T = −0.693
x T = 0.35

 India’s No.1 institute for GATE Training  1 Lakh+ Students trained till date  65+ Centers across India
26
EC-GATE-2014 PAPER-02| www.gateforum.com

50. Coherent orthogonal binary FSK modulation is used to transmit two equiprobable symbol
waveforms s1 ( t ) = α cos 2 πf1t and s2 ( t ) = cos 2 πf 2 t, where α = 4mV. Assume an AWGN
N0
channel with two-sided noise power spectral density = 0.5 × 10−12 W / Hz. Using an
2
1 ∞
optimal receiver and the relation Q ( v ) = ∫ e− u
2
/2
du the bit error probability for a data
2π v

rate of 500 kbps is


( A) Q ( 2) ( B) (
Q 2 2 ) ( C) Q ( 4) ( D) (
Q 4 2 )
Answer: (C)
Exp: For Binary FSK
 E 
Bit error probability = Q 
 N 
 O 

E → Energy per bit [No. of symbols = No. of bits]


A2T 1
E= , A = 4 × 10−3 ,T = [inverse of data rate]
2 500 × 103
16 × 10−6 × 2 × 10−6
⇒E= = 16 × 10−12
2
N 0 = 1 × 10−12
 16 × 10−12 
⇒ Pe = Q   = Q ( 4)
 1 × 10−12 
 

51. The power spectral density of a real stationary random process .X(t) is given by
 1,
Sx ( f ) =  0,w f ≤w
f >w

  1 
The value of the expectation E  π X ( t )  t −   is ___________.
  4w  
Answer: 4
1 
 , f ≤ w
Exp: Given Sx ( f ) =  w 
0 , f ≥ w 
 
w
1
R x ( τ) = ∫ w .e
j2 πft
df
−w

1 e j2 πwt − e − j2 πwt 1  sin ( 2πwt ) 


= =  
w j2πt w πt 
 1 
sin  2πw .  4
  1   1  1  4w =
Now, E  π × ( t ) .x  t −  = π R x   ⇒ π. .
  4w    4w  w 1 1
π.
4w

 India’s No.1 institute for GATE Training  1 Lakh+ Students trained till date  65+ Centers across India
27
EC-GATE-2014 PAPER-02| www.gateforum.com

52. In the figure, M(f) is the Fourier transform of the message signal .m(t) where A = 100 Hz and
B = 40 Hz. Given v(t) = cos ( 2πf c t ) and w ( t ) = cos ( 2 π ( f c + A ) t ) , where f c > A The cutoff
frequencies of both the filters are f C

M (f )
−1

−A −B B A f
m (t) High Pass s (t)
Low Pass
Filter Filter
v (t) w (t)

The bandwidth of the signal at the output of the modulator (in Hz) is _____.
Answer: 60
Exp: m ( t ) ↔ M ( f )
M (f )

f
−A −B B A

After multiplication with V ( t ) = cos ( 2πf c t )

Let w1 ( t ) = m ( t ) .V ( t )
⇒ W1 ( f ) ( specturm of w1 ( t ) ) is

−fc − A − fc − B − fc + B fc + A −fc − A − fc − B − fc + B fc + A
−fc fc

After high pass filter

− fC − A − fC − B fC fC + B fC + A

After multiplication with cos ( 2π ( f c + A ) t ) and low pass filter of cut off f c

 India’s No.1 institute for GATE Training  1 Lakh+ Students trained till date  65+ Centers across India
28
EC-GATE-2014 PAPER-02| www.gateforum.com

f
− (A − B) (A − B)
O
Bandwidth = A − B
= 100 − 40 = 60

53. If the electric field of a plane wave is




) ( )
E ( Z, t ) = x3cos ωt − kz + 30O − y4sin ωt − kz + 45O ( mV / m ) ,
the polarization state of the plane wave is
(A) left elliptical (B) left circular
(C) right elliptical (D) right circular
Answer: (A)
Exp: E ( z1t ) = 3cos ( cot − kz + 3o° ) a x − 4 − sin ( ωt − kz + 45° ) a y
E x = 3cos ( ωt − kz + 30° )
E y = − 4cos ( ωt − kz + 45° )
At z = 0 E x = 3cos ( ωt + 30o )
E y = −4sin ( ωt + 45o )
E x ≠ E y → so Elliptical polarization
Q = 30° −135° = − 105°
∴ left hand elliptical (LEP)

54. In the transmission line shown, the impedance Zin (in ohms) between node A and the ground
is _________.

Z0 = 50Ω, L = 0.5λ
100 Ω 50 Ω
Zin = ?

Answer: 33.33Ω
λ
Exp: Here =
2
(
Zin = λ
2 )=Z L = 50Ω
100
∴ Zin = (100  50 ) = = 33.33Ω
3

 India’s No.1 institute for GATE Training  1 Lakh+ Students trained till date  65+ Centers across India
29
EC-GATE-2014 PAPER-02| www.gateforum.com

55. For a rectangular waveguide of internal dimensions a × b ( a > b ) , the cut-off frequency for
the TE11 mode is the arithmetic mean of the cut-off frequencies for TE10 mode and TE20
mode. If a = 5 cm, the value of b (in cm) is _____.
Answer: 2
2
C 1
Exp: t c10 =  
2 a
1 2
t c10 = K   ; t c20 = K  
a a
1 1
t c11 = K +
a 2 b2
f c10 + f c20
Given t c11 =
2
1 1 K 1 2
K + = +
a 2 b2 2  a a 
1 1 3
2
+ 2 =
a b 2a
1 1 9 1 9 1
+ 2 = ⇒ − + = 2
5 b 4 ( 5) 5 20 b
1
− 0.2 + 0.45 =
b2
1 1
∴ 2
= 2 ⇒ b = 2cm
b 2

 India’s No.1 institute for GATE Training  1 Lakh+ Students trained till date  65+ Centers across India
30
EC-GATE-2014 PAPER-03| www.gateforum.com

Q. No. 1 – 5 Carry One Mark Each

1. “India is a country of rich heritage and cultural diversity.” Which one of the following facts
best supports the claim made in the above sentence?
(A) India is a union of 28 states and 7 union territories.
(B) India has a population of over 1.1 billion.
(C) India is home to 22 official languages and thousands of dialects.
(D) The Indian cricket team draws players from over ten states.
Answer: C
Exp: Diversity is shown in terms of difference language

2. The value of one U.S. dollar is 65 Indian Rupees today, compared to 60 last year. The Indian
Rupee has ____________.
(A) Depressed (B) Depreciated (C) Appreciated (D) Stabilized
Answer: B

3. 'Advice' is ________________.
(A) a verb (B) a noun
(C) an adjective (D) both a verb and a noun
Answer: B

4. The next term in the series 81, 54, 36, 24 … is ________


Answer: 16
2
Exp: 81 − 54 = 27;27 × = 18
3
2
54 − 36 = 18;18 × = 12
3
2
36 − 24 = 12;12 × = 8
3
∴ 24 − 8 = 16

5. In which of the following options will the expression P < M be definitely true?
(A) M < R > P > S (B) M > S < P < F
(C) Q < M < F = P (D) P = A < R < M
Answer: D

Q. No. 6 – 10 Carry Two Marks Each

6. Find the next term in the sequence: 7G, 11K, 13M, ___
(A) 15Q (B) 17Q (C) 15P (D) 17P
Answer: B

 India’s No.1 institute for GATE Training  1 Lakh+ Students trained till date  65+ Centers across India
1
EC-GATE-2014 PAPER-03| www.gateforum.com

7. The multi-level hierarchical pie chart shows the population of animals in a reserve forest. The
correct conclusions from this information are:

Beetle Tiger
Re d − ant

Elephant
Mammal
Honey
− bee In sec t
Leopard
Re ptile
Snake
Moth Bird
Crocadile
Hawk Drongo
Bulbul
Butterfly

(i) Butterflies are birds


(ii) There are more tigers in this forest than red ants
(iii) All reptiles in this forest are either snakes or crocodiles
(iv) Elephants are the largest mammals in this forest
(A) (i) and (ii) only (B) (i), (ii), (iii) and (iv)
(C) (i), (iii) and (iv) only (D) (i), (ii) and (iii) only
Answer: D
Exp: It is not mentioned that elephant is the largest animal

8. A man can row at 8 km per hour in still water. If it takes him thrice as long to row upstream,
as to row downstream, then find the stream velocity in km per hour.
Answer: 4
Exp: 4 km/hr.
Speed of man=8
Left distance =d
d
Time taken=
8
Upstream:
Speed of stream=s
⇒ speed upstream = S' = (8 − s)
 d 
t' =  
8−s
Downstream:
d
Given speed downstream = t '' =
8+s

 India’s No.1 institute for GATE Training  1 Lakh+ Students trained till date  65+ Centers across India
2
EC-GATE-2014 PAPER-03| www.gateforum.com

⇒ 3t ' = t ''
3d d
⇒ =
8−s 8+s
3d d
⇒ =
8−s 8+s
⇒ s = 4km / hr

9. A firm producing air purifiers sold 200 units in 2012. The following pie chart presents the
share of raw material, labour, energy, plant & machinery, and transportation costs in the total
manufacturing cost of the firm in 2012. The expenditure on labour in 2012 is Rs. 4,50,000. In
2013, the raw material expenses increased by 30% and all other expenses increased by 20%.
If the company registered a profit of Rs. 10 lakhs in 2012, at what price (in Rs.) was each air
purifier sold?

Trans
Labour
portat
ion 15%

10%
Plant and Raw Material
machinery 20%
30%
Energy
25%

Answer: 20,000
15
Exp: Total expenditure= = x = 4,50,000
100
x=3×106
Profit=10 lakhs
So, total selling price =40,00,000 … (1)
Total purifies=200 … (2)
S.P of each purifier=(1)/(2)=20,000

10. A batch of one hundred bulbs is inspected by testing four randomly chosen bulbs. The batch
is rejected if even one of the bulbs is defective. A batch typically has five defective bulbs.
The probability that the current batch is accepted is _________
Answer: 0.8145
95
Exp: Probability for one bulb to be non defective is
100
4
 95 
∴ Probabilities that none of the bulbs is defectives   = 0.8145
 100 

 India’s No.1 institute for GATE Training  1 Lakh+ Students trained till date  65+ Centers across India
3
EC-GATE-2014 PAPER-03| www.gateforum.com

Q.No. 1 – 25 Carry One Mark Each

1. The maximum value of the function f(x) = ln(1 + x)- x (where .x > - 1) occurs at x=______.
Answer: 0
1
Exp: f1 (x) = 0 ⇒ −1 = 0
1+ x
−x
⇒ =0⇒x =0
1+ x
−1
and f 11 ( x ) = < 0 at x = 0
(1 + x )
2

2. Which ONE of the following is a linear non-homogeneous differential equation, where x and
y are the independent and dependent variables respectively?
dy dy
( A) + xy = e − x ( B) + xy = 0
dx dx
dy dy − y
( C) + xy = e − y ( D) + e = e− y = 0
dx dx
Answer: A
dy
Exp: (A) + xy = e − x is a first order linear equation (non-homogeneous)
dx
dy
(B) + xy = 0 is a first order linear equation (homogeneous
dx
(C), (D) are non linear equations

3. Match the application to appropriate numerical method.


Application Numerical |Method
P1: Numerical integration M1: Newton-Raphson Method
P2: Solution to a transcendental equation M2: Runge-Kutta Method
P3: Solution to a system of linear equations M3: Simpson’s 1/3-rule
P4: Solution to a differential equation M4: Gauss Elimination Method
(A) P1—M3, P2—M2, P3—M4, P4—M1 (B) P1—M3, P2—M1, P3—M4, P4—M2
(C) P1—M4, P2—M1, P3—M3, P4—M2 (D) P1—M2, P2—M1, P3—M3, P4—M4
Answer: B
Exp: P1 − M3,P2 − M1, P3 − M4,P4 − M2

4. An unbiased coin is tossed an infinite number of times. The probability that the fourth head
appears at the tenth toss is
(A) 0.067 (B) 0.073 (C) 0.082 (D) 0.091

 India’s No.1 institute for GATE Training  1 Lakh+ Students trained till date  65+ Centers across India
4
EC-GATE-2014 PAPER-03| www.gateforum.com

Answer: C
Exp: P[fourth head appears at the tenth toss] = P [getting 3 heads in the first 9 tosses and one
head at tenth toss]
  1 9   1  21
=  9 C3 .    ×   = = 0.082
  2    2  256

5. If z = xyln(xy), then
∂z ∂z ∂z ∂z
( A) x +y =0 ( B) y =x
∂x ∂y ∂x ∂y
∂z ∂z ∂z ∂z
( C) x =y ( D) y +x =0
∂x ∂y ∂x ∂y
Answer: C
∂z  1 
Exp: = y  x × × y + ln xy  = y (1 + ln xy )
∂x  xy 
∂z ∂z ∂z
and = x (1 + ln xy ) ⇒ x =y
∂y ∂x ∂y

6. A series RC circuit is connected to a DC voltage source at time t = 0. The relation between


the
source voltage VS, the resistance R, the capacitance C, and the current i(t) is given below:
1 t
Ve = Ri ( t ) + i ( u ) du
c ∫0
Which one of the following represents the current f(t)?

( A) ( B)

i (t) i (t)

0 t
0 t

( C) ( D)
i (t) i (t)

0 t 0 t

 India’s No.1 institute for GATE Training  1 Lakh+ Students trained till date  65+ Centers across India
5
EC-GATE-2014 PAPER-03| www.gateforum.com

Answer: A
Exp: In a series RC circuit,
VS
→ Initially at t = 0, capacitor charges with a current of and in steady state at t = ∞ ,
R
capacitor behaves like open circuit and no current flows through the circuit
→ So the current i(t) represents an exponential decay function

i (t )

0 →t

7. In the figure shown, the value of the current I (in Amperes) is __________.
5Ω 5Ω
I

5V ± ↑ 1A 10 Ω

Answer: 0.5
Exp:
V
I
5Ω 5Ω
+
5V − 1A 10Ω

V−5 V
Apply KCL at node V, −1 + = 0
5 15
30
⇒ V = volts
4
V 2
⇒ current I = ⇒ ⇒ 0.50 Amperes
15 4

8. In MOSFET fabrication, the channel length is defined during the process of


(A) Isolation oxide growth
(B) Channel stop implantation
(C) Poly-silicon gate patterning
(D) Lithography step leading to the contact pads
Answer: C

 India’s No.1 institute for GATE Training  1 Lakh+ Students trained till date  65+ Centers across India
6
EC-GATE-2014 PAPER-03| www.gateforum.com

9. A thin P-type silicon sample is uniformly illuminated with light which generates excess
carriers. The recombination rate is directly proportional to
(A) The minority carrier mobility
(B) The minority carrier recombination lifetime
(C) The majority carrier concentration
(D) The excess minority carrier concentration
Answer: D
Exp: (
Recombination rate, R = B n n o + n 'n )( P no + Pn' )
n n0 & Pn0 = Electron and hole concentrations respectively under thermal equilibrium
n 'n & p 'n = Excess elements and hole concentrations respectively

kT
10. At T = 300 K, the hole mobility of a semiconductor µ P = 500cm 2 / V − s and = 26 mV.
q
The hole diffusion constant D P in cm2/s is ________
Answer: 13
Exp: From Einstein relation,
D P kJ
=
µp q
⇒ D P = 26 mV × 500cm 2 / v − s = 13cm 2 / s

11. The desirable characteristics of a transconductance amplifier are


(A) High input resistance and high output resistance
(B) High input resistance and low output resistance
(C) Low input resistance and high output resistance
(D) Low input resistance and low output resistance
Answer: A
Exp: Transconductance amplifier must have z i = ∞ and z 0 = ∞ ideally

12. In the circuit shown, the PNP transistor has VBE = 0.7 and β = 50. Assume that R B = 100kΩ
For V0 to be 5 V, the value of R C ( in kΩ ) _________________

RC

V0

RB VEE = 10 V

 India’s No.1 institute for GATE Training  1 Lakh+ Students trained till date  65+ Centers across India
7
EC-GATE-2014 PAPER-03| www.gateforum.com

Answer: 1.075
Exp: KVL in base loop gives,
10 − 0.7
IB = = 93 µA
100K
⇒ IC = β IB = 50 × 93 µA = 4.65mA

from figure, V0 = IC R C

V0 5V
⇒ RC = = = 1.075 Ω
IC 4.65 mA

13. The figure shows a half-wave rectifier. The diode D is ideal. The average steady-state current
(in Amperes) through the diode is approximately ____________.

D
C
10sin ωt ~ R
4 mF
f = 50 Hz 100 Ω

Answer: 0.09
Idc
Exp: Vdc = Vm −
4fc
Idc
Idc R L = Vm −
4fc
 1 
Idc  R L + = Vm
 4fc 
10
⇒ Idc = = 0.09A
1
100 +
4 × 50 × 4 × 10−3

14. An analog voltage in the range 0 to 8 V is divided in 16 equal intervals for conversion to 4-bit
digital output. The maximum quantization error (in V) is _________________
Answer: 0.25
step − size
Exp: Maximum quantization error is
2
8−0 1
step − size = = = 0.5V
16 2
Quantization error = 0.25 V

 India’s No.1 institute for GATE Training  1 Lakh+ Students trained till date  65+ Centers across India
8
EC-GATE-2014 PAPER-03| www.gateforum.com

15. The circuit shown in the figure is a

D Q Q
D Latch D Latch
En Q En Q

Clk

(A) Toggle Flip Flop (B) JK Flip Flop


(C) SR Latch (D) Master-Slave D Flip Flop
Answer: D
Exp: Latches are used to construct Flip-Flop. Latches are level triggered, so if you use two latches
in cascaded with inverted clock, then one latch will behave as master and another latch which
is having inverted clock will be used as a slave and combined it will behave as a flip-flop. So
given circuit is implementing Master-Slave D flip-flop

16. Consider the multiplexer based logic circuit shown in the figure.

W 0
MUX 0
1 MUX F
1
S1
S2

Which one of the following Boolean functions is realized by the circuit?


( A) F = WS1 S2 ( B) F = WS1 + WS2 + S1S2

( C) F = W + S1 + S2 ( D) F = W ⊕ S1 ⊕ S2
Answer: D
Exp:
W
0
MUX
Y
1 0
MUX F
1

S1
S2

Output of first MUX = ws1 + ws1 = w ⊕ s1


Let Y = w ⊕ s1
Output of second MUX = Ys2 + Y s 2
= Y ⊕ s2
= w ⊕ s1 + s 2

 India’s No.1 institute for GATE Training  1 Lakh+ Students trained till date  65+ Centers across India
9
EC-GATE-2014 PAPER-03| www.gateforum.com

17. Let x(t)= cos (10πt ) + cos ( 30πt ) be sampled at 20 Hz and reconstructed using an ideal low-
pass filter with cut-off frequency of 20 Hz. The frequency/frequencies present in the
reconstructed signal is/are
(A) 5 Hz and 15 Hz only (B) 10 Hz and 15 Hz only
(C) 5 Hz, 10 Hz and 15 Hz only (D) 5 Hz only
Answer: (A)
Explanation: x ( t ) = cos (10πt ) + cos ( 30πt ) , Fs = 20Hz
Spectrum of x(t)

−15 − 5 5 15 t

Spectrum of sampled version of x(t)

−35 25 − 15 − 5 5 15 25 35

After LPF, signal will contain 5 and 15Hz component only

( z − b) , where H e = 1,
−1

18. For an all-pass system H (z) =


(1 − az ) −1 ( ) − jω
for all ω .If

Re ( a ) ≠ 0, Im ( a ) ≠ 0, then b equals
(A) a (B) a* (C) 1/a* (D) 1/a
Answer: (B)
1 1
Exp: For an all pass system, pole = or zero =
zero * pole*
pole = a
1
zero =
b
1 1
⇒ = or b = a *
b a*

19. A modulated signal is y(t) = m.(t)cos(40000 πt ), where the baseband signal m(t) has
frequency components less than 5 kHz only. The minimum required rate (in kHz) at which
y(t) should be sampled to recover m(t) is __________________.

 India’s No.1 institute for GATE Training  1 Lakh+ Students trained till date  65+ Centers across India
10
EC-GATE-2014 PAPER-03| www.gateforum.com

Answer: 10 KHz.
Exp: Since m(t) is a base band signal with maximum frequency 5 KHz, assumed spreads as
follows:

M(f )

f
m(t) M(f )

−5k +5k f (Hz)

*1
∵ y(t) = m(t) cos(40000π t) 
7
→ m(f ) [δ(f − 20k) + δ(f + 20k)]
2
1
∴ y(f ) = [ M(f − 20k) + M(f + 20 k)]
2
Thus the spectrum of the modulated signal is as follows:

y(f )

15k 25k f (Hz)


−25k −20k −15k 20k

If y(t) is sampled with a sampling frequency ‘fs’ then the resultant signal is a periodic
extension of successive replica of y(f) with a period ‘fs’.
It is observed that 10 KHz and 20 KHz are the two sampling frequencies which causes a
replica of M(f) which can be filtered out by a LPF.
Thus the minimum sampling frequency (fs) which extracts m(t) from g(f) is 10 KHz.

20. Consider the following block diagram in the figure.

R ( s) + + C ( s)
G1 G2
+ +

C ( s)
The transfer function is
R ( s)
G1G 2 G1
( A) ( B) G1G 2 + G1 + 1 ( C) G1G 2 + G 2 + 1 ( D)
1 + G1G 2 1 + G1G 2

 India’s No.1 institute for GATE Training  1 Lakh+ Students trained till date  65+ Centers across India
11
EC-GATE-2014 PAPER-03| www.gateforum.com

Answer: C
Exp: By drawing the signal flow graph for the given block diagram
G1 G2 1
C (s)
R (s) 1
1
1
Number of parallel paths are three
Gains P1 = G1G 2 , P2 = G 2 , P3 = 1
By mason’s gain formula,
C(s)
= P1 + P2 + P3
R (s)
⇒ G1G 2 + G 2 + 1

21. The input −3e2 t u ( t ) , where u(t) is the unit step function, is applied to a system with transfer
s−2
function . . If the initial value of the output is -2, then the value of the output at steady
s+3
state is__________________.
Answer: 0
Exp: 1
Y (s ) S− 2
=
X (s ) S+3
⇒ SY ( s ) + 3Y ( s ) = S × ( s ) − 2X ( s )
Due to initial condition, we can write above equation as
Sy ( s ) − y ( 0 ) + 3y ( s ) = sx ( s ) − x ( 0 − ) − 2x ( s )

y ( 0− ) = −2, x ( 0− ) = 0  x ( t ) = 3e2t u ( t ) 
 −3 
⇒ Sy ( s ) + 2 + 3y ( s ) = ( s − 2 )  
s−2
−5
( s + 3) y ( s ) = −3 − 2 ⇒ y ( s ) =
5+3
⇒ y ( t ) = −5e u ( t )
−3t

y ( ∞ )( steady sate ) = 0
Exp: 2
s−2
H (s) = ;X ( t ) = −3e2 t .u ( t )
s+3
−3 −3
∴ X (s) = ⇒ Y (s) =
s−2 s+3
−3s
y ( t ) at t =∞ ⇒ y ( ∞ ) = lim S.y ( s ) = lim
s →0 s →0 s+3
y(∞) = 0

 India’s No.1 institute for GATE Training  1 Lakh+ Students trained till date  65+ Centers across India
12
EC-GATE-2014 PAPER-03| www.gateforum.com

22. The phase response of a passband waveform at the receiver is given by


φ ( f ) = −2πα ( f − f c ) − 2πβ f c
Where fc is the centre frequency, and α and β are positive constants. The actual signal
propagation delay from the transmitter to receiver is
α −β αβ
(A) ( B) ( C) α ( D) β
α+β α +β
Answer: C
Exp: Phase response of pass band waveform
φ ( f ) = −2πα ( f − f c ) − 2πβ f c
− dφ ( f )
Group delay t y = =α
2π df
Thus ' α ' is actual signal propagation delay from transmitter to receiver

23. Consider an FM signal f ( t ) = cos [ 2 πf c t + β1 sin 2 πf1t + β 2 sin 2πf 2 t.] . The maximum
deviation of the instantaneous frequency from the carrier frequency fc is
( A) β1f1 + β2f 2 ( B) β1f 2 + β 2f1 ( C) β1 + β2 ( D) f1 + f 2
Answer: A
Exp: Instantaneous phase φi ( t ) = 2π f c t + β1 sin 2πf1 + β 2 sin 2πf 2 t
d 1
Instantaneous frequency f i ( t ) = φi ( t ) ×
dt 2π
= f c + β1f1 cos 2πf1t + β2 f 2 cos 2πf 2 t
Instantaneous frequency deviation = β1f1 cos 2 πf1 t + β2 f 2 cos 2 πf 2 t
Maximum ∆f = β1f1 + β2 f 2

24. Consider an air filled rectangular waveguide with a cross-section of 5 cm × 3 cm. For this
waveguide, the cut-off frequency (in MHz) of TE21 mode is _________.
Answer: 7810MHz.
2 2
C 2 1
Exp: f c ( TE 21 ) =   + 
2 9 b

3 × 1010  2   1 
2 2

=   + 
2  5 3
= 1.5 × 1010 0.16 + 0.111
= 0.52 × 1.5 × 1010
= 7.81 GHz
= 7810 MHz.

 India’s No.1 institute for GATE Training  1 Lakh+ Students trained till date  65+ Centers across India
13
EC-GATE-2014 PAPER-03| www.gateforum.com

25. In the following figure, the transmitter Tx sends a wideband modulated RF signal via a
coaxial cable to the receiver Rx. The output impedance ZT of Tx, the characteristic
impedance Z0 of the cable and the input impedance ZR of Rx are all real.

Transmitter Re ceiver
Characteristic Im pedance = Z0

ZT ZR
TX RX

Which one of the following statements is TRUE about the distortion of the received signal
due to impedance mismatch?
(A) The signal gets distorted if ZR ≠ Z0, irrespective of the value of ZT
(B) The signal gets distorted if ZT ≠ Z0, irrespective of the value of ZR
(C) Signal distortion implies impedance mismatch at both ends: ZT ≠ Z0 and ZR ≠ Z0
(D) Impedance mismatches do NOT result in signal distortion but reduce power transfer
efficiency
Answer: C
Exp: Signal distortion implies impedance mismatch at both ends. i.e.,
ZT ≠ Z 0
ZR ≠ Z0

Q. No. 26 – 55 Carry Two Marks Each

26. The maximum value of f(x)=2x3 – 9x2 +12x - 3 in the interval 0 ≤ x ≤ 3 is _______.
Answer: 6
Exp: f 1 ( x ) = 6x 2 − 18x + 12 = 0 ⇒ x = 1, 2 ∈ [ 0,3]

Now f ( 0 ) = −3 ; f ( 3) = 6 and f (1) = 2 ; f ( 2 ) = 1

Hence, f(x) is maximum at x = 3 and the maximum value is 6

27. Which one of the following statements is NOT true for a square matrix?
(A) If A is upper triangular, the eigenvalues of A are the diagonal elements of it
(B) If A is real symmetric, the eigenvalues of A are always real and positive
(C) If A is real, the eigenvalues of A and AT are always the same
(D) If all the principal minors of A are positive, all the eigenvalues of A are also positive
Answer: B

 India’s No.1 institute for GATE Training  1 Lakh+ Students trained till date  65+ Centers across India
14
EC-GATE-2014 PAPER-03| www.gateforum.com

 −1 1
Exp: Consider, A  which is real symmetric matrix
1 − 1

Characteristic equation is A − λI = 0 ⇒ (1 + λ ) − 1 = 0
2

⇒ λ + 1 = ±1
∴λ = 0, − 2 ( not positive )

(B) is not true


(A), (C), (D) are true using properties of eigen values

28. A fair coin is tossed repeatedly till both head and tail appear at least once. The average
number of tosses required is __________________.
Exp: Let the first toss be Head.
Let x denotes the number of tosses( after getting first head) to get first tail.
We can summarize the even as:
Event x Probability(p(x))
(After getting first H)
T 1 1/2
HT 2 1/2*1/2=1/4
HHT 3 1/8
and so on………..

1 1 1
E ( x ) = ∑ xp ( x ) = 1x + 2x + 3x 
x =1 2 4 8
1 1 1
Let, S =1x + 2x + 3x  ( I)
2 4 8
1 1 1 1
⇒ S= + 2x + 3x  ( II )
2 4 8 16
( I − II ) gives
 1 1 1 1 1
1 −  S = + + + + 
 2 2 4 8 16
1
1
⇒ S = 2 =1
2 1
1−
2
⇒S= 2
⇒ E(x) = 2

i.e. The expected number of tosses (after first head) to get first tail is 2 and same can be
applicable if first toss results in tail.

Hence the average number of tosses is 1+2 = 3.

 India’s No.1 institute for GATE Training  1 Lakh+ Students trained till date  65+ Centers across India
15
EC-GATE-2014 PAPER-03| www.gateforum.com

29. Let X1, X2, and X3 be independent and identically distributed random variables with the
uniform distribution on [0, 1]. The probability P{X1+ X2 ≤ X3} is ________.
Answer: 0.16
Exp: Given x1 x 2 and x 3 be independent and identically distributed with uniform distribution on
[0,1]
Let z = x1 + x 2 − x 3
⇒ P {x1 + x 2 ≤ x 3 } = P {x1 + x 2 − x 3 ≤ 0}
= P {z ≤ 0}
Let us find probability density function of random variable z.
Since Z is summation of three random variable x1 , x 2 and − x 3
Overall pdf of z is convolution of the pdf of x1 x 2 and − x 3
pdf of {x1 + x 2 } is

O 1 2

1
pdf of − x 3 is

−1

3 0
( z + 1) ( z + 1)
0 2
1
P {z ≤ 0} = ∫ dz = = = 0.16
−1
2 6 6
−1

30. Consider the building block called ‘Network N’ shown in the figure.
Let C = 100µF and R = 10kΩ
Network N

+ +
C
V1 ( s) R V2 ( s)

− −

Two such blocks are connected in cascade, as shown in the figure.

+ + +
V1 ( s) Network N Network N V3 ( s )
− − −

 India’s No.1 institute for GATE Training  1 Lakh+ Students trained till date  65+ Centers across India
16
EC-GATE-2014 PAPER-03| www.gateforum.com

v 3 ( s)
The transfer function of the cascaded network is
v1 ( s )
2
s s2  s  s
( A) ( B) ( C)   ( D)
1+ s 1 + 3s + s2 1+ s 2+s
Answer: B
Exp: Two blocks are connected in cascade, Represent in s-domain,

+ 1 1 +
SC SC
V1 ( s ) R R V3 ( s )

− −

V3 ( s ) R . R
=
V1 ( s ) 1 1   1 
R+R+ +R + R
sc  SC   SC 
R. R
=
1 1 R
.
SC SC
 2R ( SC ) + 1 +
 SC
[1 + RSC]
S2 C 2 .R.R
=
1 + 2R ( SC )  + RSC + R S C
2 2 2

S2 .100 × 100 × 10−6 × 10−6 × 10 × 10 × 103 × 103


=
S2 × 100 × 106 × 104 × 10−12 + 3S + 100 × 10−6 × 104 + 1
V3 ( s ) S2
=
V1 ( s ) 1 + 3S + S2

31. In the circuit shown in the figure, the value of node voltage V2 is

10∠0O V
+ −
V1 V2
4Ω
4∠0OA 6Ω j6Ω
− j3Ω

(A) 22 + j 2 V (B) 2 + j 22 V (C) 22 – j 2 V (D) 2 – j 22 V

 India’s No.1 institute for GATE Training  1 Lakh+ Students trained till date  65+ Centers across India
17
EC-GATE-2014 PAPER-03| www.gateforum.com

Answer: D
4Ω
Exp: Super node
0
V 10 0 V2
V
+ −

4 0o A 6Ω j6 Ω
− j3Ω

KVL for V1 & V2 :


V1
V2
+ −
+ +
− 10 0 o −

V1 − V2 = 10 0o ....(1)
V1 = V2 + 10 0o

KCL at super node:


V1 V2 V2
−4 0o + + + =0 ... ( 2 )
− j3 6 j6
V1 V2 V2
+ + = 4 0o
− j3 V6 j6
V2 + 10 0o V2 V2
from (1) & ( 2 ) , + + = 4 0o
− j3 6 j6
 1 1 1 10
V2  + +  = 4 0o +
 − j3 6 j6  j3
∴ V2 = ( 2 − j22 ) Volts

32. In the circuit shown in the figure, the angular frequency ω (in rad/s), at which the Norton
equivalent impedance as seen from terminals b-b' is purely resistive, is
__________________.
1Ω 1F
b
+
10cos ωt ~
( Volts) − 0.5 H

b'

Answer: 2 r/sec

 India’s No.1 institute for GATE Training  1 Lakh+ Students trained till date  65+ Centers across India
18
EC-GATE-2014 PAPER-03| www.gateforum.com

Exp: Norton’s equivalent impedance


1
1* jω. b
ZN = 2 + 1 1Ω 1F
1 jω.1
1 + jω. 0.5H
2
jω 1
= + b1
Zu
2 + jω jω
( 2 − ω ) + jω ⇒ Z ( ω − 2 ) − jω .  ω + 2 jω
2  2   2 
ZN = =
 2 jω − ω2   ω4 + 4ω2 
N

Equating imaginary term to zero i.e., ω3 − 4ω = 0


⇒ ω ( ω2 − 4 ) = 0 ⇒ ω = 2 r / sec

33. For the Y-network shown in the figure, the value of R 1 ( in Ω ) in the equivalent ∆ -network is
_____________________.
R1

5Ω 3Ω

7.5Ω

Answer: 10Ω R1
Exp:
5Ω 3Ω

7.5Ω

R1 =
( 7.5)( 5) + ( 3)( 5 ) + ( 7.5)( 3) Ω
7.5
R 1 = 10Ω

34. The donor and accepter impurities in an abrupt junction silicon diode are 1 x 1016 cm-3 and 5
x 1018 cm-3, respectively. Assume that the intrinsic carrier concentration in silicon ni = 1.5 x
kT
1010 cm-3 at 300 K, = 26 mV and the permittivity of silicon εsi = 1.04 × 10−12 F / cm. The
q
built-in potential and the depletion width of the diode under thermal equilibrium conditions,
respectively, are
(A) 0.7 V and 1 x 10-4 cm (B) 0.86 V and 1 x 10-4 cm
(C) 0.7 V and 3.3 x 10-5 cm (D) 0.86 V and 3.3 x 10-5 cm

 India’s No.1 institute for GATE Training  1 Lakh+ Students trained till date  65+ Centers across India
19
EC-GATE-2014 PAPER-03| www.gateforum.com

Answer: D
 
NA ND  5 × 1018 × 1 × 1016 
Exp: Vbi = VT ln = 26 mv ln
ni2  (1.5 × 1010 )2 
 
= 0.859V
2εS Vbi  N A + N D  −5
W=   = 3.34 × 10 cm
q  NA ND 

35. The slope of the ID vs VGS curve of an n-channel MOSFET in linear regime is 10−3 Ω −1 at
VDS = 0.1V. . For the same device, neglecting channel length modulation, the slope of the
( )
I D vs VGS curve in A / V under saturation regime is approximately ___________.

Answer: 0.07
 V 2
Exp: In linear region, I D = k ( VGS − VT ) VDS − DS 
 2 

∂I D VDS2
= 10−3 = kVDS ∵ VDS is small, is neglected
∂VGS 2
10−3
⇒K= = 0.01
0.1
1
In saturation region, I D = k ( VGS − VT )
2

2
k
ID = ( VGS − VT )
2

∂ ID k 0.01
= = = 0.07
∂VGS 2 2

36. An ideal MOS capacitor has boron doping-concentration of 1015 cm-3 in the substrate. When a
gate voltage is applied, a depletion region of width 0.5 µm is formed with a surface (channel)
potential of 0.2 V. Given that εo = 8.854 × 10-14 F/cm and the relative permittivities of silicon
and silicon dioxide are 12 and 4, respectively, the peak electric field (in V/µm) in the oxide
region is __________________.
Answer: 2.4
2 × 0.2
Exp: Es = = 0.8 v / µm
0.5
Es
E ox = E s = 2.4 v / µm
E ox

 India’s No.1 institute for GATE Training  1 Lakh+ Students trained till date  65+ Centers across India
20
EC-GATE-2014 PAPER-03| www.gateforum.com

37. In the circuit shown, the silicon BJT has β = 50 . Assume VBE =0.7 V and VCE(sat) = 0.2 V.
Which one of the following statements is correct?
10 V

RC

50kΩ
5V
RB

(A) For RC = 1 kΩ, the BJT operates in the saturation region


(B) For RC = 3 kΩ, , the BJT operates in the saturation region
(C) For RC =20 kΩ, , the BJT operates in the cut-off region
(D) For RC =20 kΩ, , the BJT operates in the linear region
Answer: B
Exp: KVL in base loop,
5 − I B ( 50 k ) − 0.7 = 0
5 − 0.7
IB = = 80 µA
50 k
⇒ IC = β IB = 50 × 86 µA = 4.3mA
10 − VCE ( sat ) 10 − 0.2
∴RC = =
IC 4.3mA
R C = 2279 Ω and the BJT is in saturation

38. Assuming that the Op-amp in the circuit shown is ideal, VO is given by

3R
V1 −
R VO
V2 +
2R R

5 5 3 7 11
( A) V1 − 3V2 ( B) ZV1 − V2 ( C) − V1 + V2 ( D) − 3V1 + V2
2 2 2 2 2
Answer: D
Exp: Virtual ground and KCL at inverting terminal gives
3R
V2 − V1 V2 V2 − V0
+ + =0
R 2R 3R R V2
• • −
V1 • Vo
V0 V2 V2 V2 V1 • +
= + + − 2R V2 R
3R R 3R 2R R
11
V0 = −3V1 + V2
2

 India’s No.1 institute for GATE Training  1 Lakh+ Students trained till date  65+ Centers across India
21
EC-GATE-2014 PAPER-03| www.gateforum.com

39. For the MOSFET M1 shown in the figure, assume W/L = 2, VDD = 2.0 V, µ n Cox = 100µA / V 2
and VTH = 0.5 V. The transistor M1 switches from saturation region to linear region when Vin
(in Volts) is_________________.

VDD

R = 10 kΩ

Vout

Vin M1

Answer: 1.5
Exp: Transistor m1 switch from saturation to linear
⇒ VDS = VGS − VT ; where VDS = V0 and VGS = Vi

∴ VDS = V0 = Vi − VT

1 w
Drain current ID = µ n cos ( VGS − VT )
2

2 L
VDD − Vo 1
= × 100 × 10−6 × 2 ( VGS − 0.5 )
2

10K 2
2 − ( Vi − 0.5 )
= 100 × 10−6 ( Vi − 0.5 )
2

10 K

⇒ Vi = 1.5V

40. If WL is the Word Line and BL the Bit Line, an SRAM cell is shown in

WL
WL
VDD
VDD BL
BL BL
BL

(A) ( B)

 India’s No.1 institute for GATE Training  1 Lakh+ Students trained till date  65+ Centers across India
22
EC-GATE-2014 PAPER-03| www.gateforum.com

WL WL
VDD VDD
BL BL BL
BL

( C) ( D)

Answer: B
Exp: For an SRAM construction four MOSFETs are required (2-PMOS and 2-NMOS) with
interchanged outputs connected to each CMOS inverter. So option (B) is correct.

41. In the circuit shown, W and Y are MSBs of the control inputs. The output F is given by

4 :1 MUX 4 :1 MUX

I0 I0

I1 I1 F
VCC Q
Q I2
I2

I3 I3

W X Y Z

(A) F = W X + WX + Y Z ( B) F = W X + WX + YZ

( C) F = W XY + WXY ( D) (
F = W + X YZ )
Answer: C
Exp: I0 I0

I1 4 :1 I1 4 :1
Vcc MUX Q MUX Q F
I2 I2

I3 I3

W X Y Z

The output of the first MUX = W × Vcc + WX.Vcc


WX + WX (∵ Vcc = log ic1)
=W ⊕ X
Let Q = W ⊕ X

 India’s No.1 institute for GATE Training  1 Lakh+ Students trained till date  65+ Centers across India
23
EC-GATE-2014 PAPER-03| www.gateforum.com

The output of the second MUX = Q.Y Z + Q. Y Z


= Q.Y ( Z + Z )
= Q.Y.1 = Q.Y
Put the value of Q in above expression
= ( WX + WX ) .Y
= W X.Y + WX.Y

42. If X and Y are inputs and the Difference (D = X – Y) and the Borrow (B) are the outputs,
which
one of the following diagrams implements a half-subtractor?

Y I0 X I0
2 :1 2 :1
MUX D MUX D
I1
( A) I1
( B)
S S
X Y
S S
Y I0 X I0
2 :1 2 :1
MUX B MUX B
I1 I1

Y I0 X I0
2 :1 2 :1
MUX B MUX B
( C) I1
( B)
I1
S S
X Y
S S
Y I0 X I0
2 :1 2 :1
MUX D MUX D
I1 I1

Answer: A
Exp:
X Y D B
0 0 0 0
0 1 1 1
1 0 1 0
1 1 0 0

So, D = X ⊕ Y = XY + XY and B = X.Y

 India’s No.1 institute for GATE Training  1 Lakh+ Students trained till date  65+ Centers across India
24
EC-GATE-2014 PAPER-03| www.gateforum.com

Y I0
2 :1
D = X. Y + XY
MUX
=X ⊕ Y
I1

Y I0
2 :1
B = X. Y + X.0
MUX
= X.Y + 0
I1
= X .Y

Let H1 ( z ) = (1 − pz −1 ) , H 2 ( z ) = (1 − qz −1 ) , H ( z ) = H1 ( z ) + r H 2 ( z ) . The quantities p, q, r


−1 −1
43.
1 1
are real numbers. Consider p = ,q = , r < 1. If the zero of H(z) lies on the unit circle, then
2 4
r = ________
Answer: -0.5

H1 ( z ) = (1 − Pz −1 )
−1
Exp:

H 2 ( z ) = (1 − qz −1 )
−1

1 1 1 − qz −1 + r (1 − Pz −1 ) (1 + r ) − ( q + rp ) z −1
H (z) = +r = =
1 − Pz −1 (1 − qz −1 ) (1 − Pz −1 )(1 − Pz −1 ) (1 − Pz −1 )(1 − Pz −1 )
q + rp
zero of H ( z ) =
1+ r
Since zero is existing on unit circle
q + rp q + rp
⇒ = 1 or = −1
1+ r 1+ r
1 r 1 r
− + − +
4 2 = 1 or 4 2 = −1
1+ r 1+ r
1 r 1 r
− + =1+ r or − + = −1 − r
4 2 4 2
5 r 5 3 −3r
⇒r=− ⇒ =− or = r = − 1 ⇒ r = −0.5
2 2 4 4 2 2

5
r=− is not possible
2

 India’s No.1 institute for GATE Training  1 Lakh+ Students trained till date  65+ Centers across India
25
EC-GATE-2014 PAPER-03| www.gateforum.com

1
44. Let h(t) denote the impulse response of a causal system with transfer function . Consider
s +1
the following three statements.
S1: The system is stable.
h ( t + 1)
S2: is independent of t for t 0.
h ( t)
S3: A non-causal system with the same transfer function is stable.
For the above system,
(A) Only S1 and S2 are true (B) only S2 and S3 are true
(C) Only S1 and S3 are true (D) S1, S2 and S3 are true
Answer: A
1
Exp: h ( t ) ↔ H (s) = ⇒ h ( t ) = e− t u ( t )
s +1
S1 : System is stable (TRUE)
Because h(t) absolutely integrable
h ( t + 1)
S2 : is independent of time (TRUE)
h (t)
− ( t +1)
e
⇒ e −1 (independent of time)
e− t
S3 : A non-causal system with same transfer function is stable
1
↔ −e − t u ( − t ) (a non-causal system) but this is not absolutely integrable thus
s +1
unstable.
Only S1 and S2 are TRUE

1
45. The z-transform of the sequence x[n] is given by X ( z ) = , , with the region of
(1 − 2z )
−1 2

convergence z > 2. Then, x [ 2 ] is ______.


Answer: 12
Exp(1):
1 1 1
X ( z) = =
(1 − 2z ) −1 2
(1 − 2z ) (1 − 2z−1 )
−1

x [ n ] = 2n u [ n ] * 2n u [ n ]
n
x [ n ] = ∑ 2K.2(
n −k )

k =0
2
⇒ x [ 2] = ∑ 2k.2(
2− k )
= 20.22 + 21.21 + 22.20 = 4 + 4 + 4 = 12
k =0

 India’s No.1 institute for GATE Training  1 Lakh+ Students trained till date  65+ Centers across India
26
EC-GATE-2014 PAPER-03| www.gateforum.com

Exp(2):
1 Z2
X (z) = =
(1 − 2Z ) ( Z − 2)
−1 2 2

 
 
−1  Z Z 
X(n) = Z .
Z−2  Z − 2
 ↓ ↓ 
 u ( z ) 
v( z ) 

= ∑ u m .Vn − m ( u sin g conduction theorem and u n = 2n ; v n = 2n )


n

m =0

n
= ∑ 2m.2n − m = 2n ( n + 1)
m =0

∴ x ( 2 ) = 12

46. The steady state error of the system shown in the figure for a unit step input is _______.

R ( s) + E ( s) 1 C ( s)
K=4
r (t) − e (t) s+2 C(t)
2
s+4

Answer: 0.5
4 2
Exp: Given G ( s ) = ;H ( s ) =
s+2 s+4
For unit step input,

k p = lim G ( s ) H ( s )
s→0

 4  2 
k p = lim   
s→0 s + 2
  s + 4 

kp = 1

A
Steady state error ess =
1 + kp

1
ess =
1+1
1
ess = ⇒ 0.50
2

 India’s No.1 institute for GATE Training  1 Lakh+ Students trained till date  65+ Centers across India
27
EC-GATE-2014 PAPER-03| www.gateforum.com

47. The state equation of a second-order linear system is given by


x ( t ) = Ax ( t ) , x ( 0) = x 0

1  e− t  0  e − t − e −2 t 
For x 0 =   , x ( t ) =  − t  and for x 0 =   , x ( t ) =  − t −2 t 
 −1   −e  1   − e + 2e 
3
when x 0 =   , x ( t ) is
5
 −8e − t + 11e −2 t   11e − t − 8e −2t 
( A)  −t −2 t 
( B)  −t −2t 
 8e − 22e   −11e + 16e 
 3e − t − 5e −2t   5e − t − 3e −2 t 
(C)  −t −2t 
( D)  −t −2 t 
 −3e + 10e   −5e + 6e 
Answer: B
Exp: Apply linearity principle,
3   1 0
5 = a  −1 + b 1  s
     
a =3; b =8

 e− t  e − t − e −2 t 
⇒ x ( t ) = 3  −t  +  −t −2t 
 −e   −e + 2e 
 11e− t − 8e−2t 
⇒ x(t) =  −t −2 t 
 −11e + 16e 

48. In the root locus plot shown in the figure, the pole/zero marks and the arrows have been
removed. Which one of the following transfer functions has this root locus?

1 σ

s +1 s+4
( A) ( B)
( s + 2 )( s + 4)( s + 7 ) ( s + 1)( s + 2)( s + 7)
s+7 ( s + 1)( s + 2)
( C) ( D)
( s + 1)( s + 2)( s + 4) ( s + 7)( s + 4)

 India’s No.1 institute for GATE Training  1 Lakh+ Students trained till date  65+ Centers across India
28
EC-GATE-2014 PAPER-03| www.gateforum.com

Answer: B

Exp:: For transfer function


(s + 4)
( s + 1)( s + 2 )( s + 3)
From pole zero plot

σ
−7 −4 −2 −1

49. Let X(t) be a wide sense stationary (WSS) random process with power spectral density SX(f).
If Y(t) is the process defined as Y ( t ) = X ( 2t − 1) , the power spectral density SY(f) is
1 f  1 f 
(A) SY ( f ) = SX   e− jπf ( B) SY ( f ) = SX   e − jπf / 2
2 2 2 2
1 f  1 f 
(C) SY ( f ) = SX   ( D) SY ( f ) = SX   e − j2 πf
2 2 2 2
Answer: C
Exp: Shifting in time domain does not change PSD. Since PSD is Fourier transform of
autocorrelation function of WSS process, autocorrelation function depends on time
difference.
X ( t ) ↔ R x ( z ) ↔ Sx ( f )
1 f 
Y ( t ) = X ( 2t − 1) ↔ R y ( 2ζ ) ↔ Sx  
2 2
[time scaling property of Fourier transform]

50. A real band-limited random process X(t) has two-sided power spectral density

Sx (f ) = { 10 − 6 ( 3000 − f
0
) W atts / H z for f ≤ 3 kH z
otherw ise

Where f is the frequency expressed in Hz. The signal X(t)modulates a carrier cos16000 πt
and the resultant signal is passed through an ideal band-pass filter of unity gain with centre
frequency of 8 kHz and band-width of 2 kHz. The output power (in Watts) is _______.
Answer: 2.5
Exp:

Sx ( f ) 3 × 10−3 watts
Sx ( f )

−3 +3 f (in KHz )

 India’s No.1 institute for GATE Training  1 Lakh+ Students trained till date  65+ Centers across India
29
EC-GATE-2014 PAPER-03| www.gateforum.com

After modulation with cos (16000πt )


1
Sy ( f ) =Sx ( f − f c ) + Sx ( f + f c ) 
4
This is obtain the power spectral density Random process y(t), we shift the given power
spectral density random process x(t) to the right by fc shift it to be the left by fc and the two
shifted power spectral and divide by 4.

1.5 × 10 −3 / 2

−11 −8 −5 5 8 11 f (in KHz )

After band pass filter of center frequency 8 KHz and BW of 2 kHz

1.5 × 10 −3 / 2

−9 − 8 − 7 7 8 9

Total output power is area of shaded region


= 2 [ Area of one side portion ]
= 2 [ Area of triangle + Area of rec tan gle ]
 1 
2  − × 2 × 103 × 0.5 × 10−3 + 2 × 103 × 1 × 10−3 
=  
2
2
= [ 0.5 + 2] = 2.5 watts

51. In a PCM system, the signal m ( t ) = { sin (100πt ) + cos (100πt )} V is sampled at the Nyquist
rate. The samples are processed by a uniform quantizer with step size 0.75 V. The minimum
data rate of the PCM system in bits per second is _____.
Answer: 200
Exp: Nyquist rate = 2 × 50 Hz
= 100 samples / sec

∆=
m ( t ) max − m ( t ) min
⇒L=
(
2− − 2 )
L 0.75
2 2
L= = 3.77 = 4
0.75
No. of bits required to encode ‘4’ levels = 2 bits/level
Thus data rate = 2 × 100 = 200 bits / sec

 India’s No.1 institute for GATE Training  1 Lakh+ Students trained till date  65+ Centers across India
30
EC-GATE-2014 PAPER-03| www.gateforum.com

52. A binary random variable X takes the value of 1 with probability 1/3. X is input to a cascade
of 2 independent identical binary symmetric channels (BSCs) each with crossover probability
1/2. The outputs of BSCs are the random variables Y1 and Y2 as shown in the figure.

X Y1 Y2
BSC BSC

The value of H(Y1) + H(Y2) in bits is___________.


Answer: 2
1 2
Exp: Let P {x = 2} = , P {x = 0} =
3 3
to find H ( Y1 ) we need to know P { y1 = 0} and P { y 2 = 1}

P {Y1 = 0} = P {Y1 = 0 / x1 = 0} P {x1 = 0} + P { y1 = 0 / x1 = 1} P {x1 = 1}

1 1 1 2 1
= . + × =
2 3 2 3 2
1
P { y1 = 1} =
2
1 1
⇒ H ( y1 ) = log 2 2 + log 2 2 = 1
2 2
Similarly
1 1
P {y 2 = 0} = and P { y 2 = 1} =
2 2
⇒ H {y 2 } = 1

⇒ H {y1} + H {y 2 } = 2 bits

53. Given the vector A = ( cos x )( sin y ) aˆ x + ( sin x )( cos y ) aˆ y , where â x , â y denote unit vectors
along x,y directions, respectively. The magnitude of curl of A is ________
Answer: 0
Exp (1):
aˆ x aˆ y aˆ z
 ∂ ∂ ∂
Curl A =
∂x ∂y ∂z
cos x sin y sin x cos y 0

=0

∴ Curl A = 0

 India’s No.1 institute for GATE Training  1 Lakh+ Students trained till date  65+ Centers across India
31
EC-GATE-2014 PAPER-03| www.gateforum.com

Exp(2):
Given A = cos x sin yaˆ x + sin x cos y aˆ y
ax ay az
∇×A = ∂ / ∂x ∂ / ∂y ∂ / ∂z
cos x sin y sin x cos y 0
= a x ( 0 ) − a y ( 0 ) + a z ( cos x cos y − cos x cos y ) = 0
∴ ∇×A = 0

54. A region shown below contains a perfect conducting half-space and air. The surface current
  
K s on the surface of the perfect conductor is K s = x2
ˆ amperes per meter. The tangential H
field in the air just above the perfect conductor is
y


KS Air
x

Perfect conductor

(A) ( xˆ + zˆ ) 2 amperes per meter (B) x̂2 amperes per meter


(C) − ẑ2 amperes per meter (D) ẑ2 amperes per meter
Answer: D
Exp: Given medium (1) is perfect conductor
Medium (2) is air
∴ H1 = 0
From boundary conditions
( H1 − H 2 ) × a n = K S
H1 = 0
K S = 2aˆ x
an = ay
− H 2 × a y = 2aˆ x
− ( H x a x + H y a y + H z a z ) × a y = 2a x
− H x a z + H z a x = 2a x
∴ Hz = 2
H = 2a z

 India’s No.1 institute for GATE Training  1 Lakh+ Students trained till date  65+ Centers across India
32
EC-GATE-2014 PAPER-03| www.gateforum.com

( )

55. Assume that a plane wave in air with an electric field E = 10cos ωt − 3x − 3z aˆ y V/m is
incident on a non-magnetic dielectric slab of relative permittivity 3 which covers the region.
Z > 0 The angle of transmission in the dielectric slab is _________________ degrees.
Answer: 30
Exp: (
Given E = 10cos ωt − 3x − 3z a y )
(
− Jβ x cos θx + y cos θ y + z cos θz )
E = E 0e
So, β x = β cos θx = 3
β y = β cos θ y = 0
βz = β cos θz = 3
β x + β y 2 + βz 2 = β2
2

9 + 3 = β2 ⇒ β = 13
3
β cos θz = 3 ⇒ cos θz = ⇒ θz = 61.28 = θi
13
sin θi E2 sin 61.28 3 0.8769
= ⇒ = ⇒ = sin θt
sin θt E1 sin θt 1 3
θt = 30.4 ⇒ θt  30o

 India’s No.1 institute for GATE Training  1 Lakh+ Students trained till date  65+ Centers across India
33
EC-GATE-2014 PAPER-04| www.gateforum.com

Q. No. 1 – 5 Carry One Mark Each

1. Which of the following options is the closest in meaning to the word underlined in the
sentence
below? In a democracy, everybody has the freedom to disagree with the government.
(A) Dissent (B) Descent (C) Decent (D) Decadent
Answer: A
Exp: Dissent is to disagree

2. After the discussion, Tom said to me, 'Please revert!’ He expects me to _________.
(A) Retract (B) Get back to him
(C) Move in reverse (D) Retreat
Answer: B
Exp: Revert means set back

3. While receiving the award, the scientist said, "I feel vindicated". Which of the following is
closest in meaning to the word ‘vindicated’?
(A) Punished (B) Substantiated (C) Appreciated (D) Chastened
Answer: B
Exp: Vindicate has 2 meanings
1. Clear of blain
2. Substantiate, justify

4. Let f ( x, y ) = x n y m = P. If x is doubled and y is halved, the new value of f is

( A) 2n−m P ( B) 2 m− n P ( C) 2 ( n − m) P ( D) 2 ( m − n ) P
Answer: A
m
y
Exp: (2x) ×   = 2n − m × x n y m
n

2

5. In a sequence of 12 consecutive odd numbers, the sum of the first 5 numbers is 425. What is
the sum of the last 5 numbers in the sequence?
Answer: 495
Exp: Let consecutive odd numbers be a-10, a-8, a-6, a-4, a-2, a, ……a+12
Sum of 1st 5 number = 5a-30=425 ⇒ a=91
Last 5 numbers=(a+4)+(a+6)+…….+(a+12)
=(95+97+99+101+103)= 495

 India’s No.1 institute for GATE Training  1 Lakh+ Students trained till date  65+ Centers across India
1
EC-GATE-2014 PAPER-04| www.gateforum.com

Q. No. 6 – 10 Carry Two Marks Each

6. Find the next term in the sequence: 13M, 17Q, 19S, ___
(A) 21W (B) 21V (C) 23W (D) 23V
Answer: C
Exp:
13 M

17(13 + 4) Q(M + 4)
19(17 + 2) S(Q + 2)
23(19 + 4) W = (s+ 4)
⇒ 23W

7. If ‘KCLFTSB’ stands for ‘best of luck’ and ‘SHSWDG’ stands for ‘good wishes’, which of
the following indicates ‘ace the exam’?
(A) MCHTX (B) MXHTC (C) XMHCT (D) XMHTC
Answer: B
Exp: KCLFTSB SHSWDG
Reverse order: Reverse order:
BCS TOF LUCK GO OD W I S HES
Ace the exam
Reverse order should be
MAXE EHT ECA
Looking at the options we have M X H T C

8. Industrial consumption of power doubled from 2000-2001 to 2010-2011. Find the annual rate
of increase in percent assuming it to be uniform over the years.
(A) 5.6 (B) 7.2 (C) 10.0 (D) 12.2
Answer: B
Exp:
n
 r 
A = P 1 + 
 100 
A = 2P
10
 r 
2 = 1+ 
 100 
∴ r = 7.2

 India’s No.1 institute for GATE Training  1 Lakh+ Students trained till date  65+ Centers across India
2
EC-GATE-2014 PAPER-04| www.gateforum.com

9. A firm producing air purifiers sold 200 units in 2012. The following pie chart presents the
share of raw material, labour, energy, plant & machinery, and transportation costs in the total
manufacturing cost of the firm in 2012. The expenditure on labour in 2012 is Rs. 4,50,000. In
2013, the raw material expenses increased by 30% and all other expenses increased by 20%.
What is the percentage increase in total cost for the company in 2013?

Transpor Labour
− tation
30% 15%

Plant and Raw Material


machinery 20%
30%
Energy
25%

Answer: 22%
Exp: Let total cost in 2012 is 100
Raw material increases in 2013 to 1.3 x 20=26
Other Expenses increased in 2013 to 1.2 x 80=96
Total Cost in 2013 =96+26=122
Total Cost increased by 22%
Hint:Labour cost (i.e, 4,50,000) in 2012 is redundant data.

10. A five digit number is formed using the digits 1,3,5,7 and 9 without repeating any of them.
What is the sum of all such possible five digit numbers?
(A) 6666660 (B) 6666600 (C) 6666666 (D) 6666606
Answer: B
Exp: 1 appears in units place in 4! Ways
Similarly all other positions in 4! Ways
Same for other digits.
Sum of all the numbers = (11111) X 4! (1+3+5+7+9) = 6666600

 India’s No.1 institute for GATE Training  1 Lakh+ Students trained till date  65+ Centers across India
3
EC-GATE-2014 PAPER-04| www.gateforum.com

Q.No. 1 – 25 Carry One Mark Each


1
1. The series ∑ n! converges to
n =0

(A) 2 ln2 (B) 2 (C) 2 (D) e


Answer: D

1 1 1
Exp: ∑ n! = 1 + 1! + 2! + .........
n =0

x x2
= e as e x = 1 + + + ......., ∀ x in R
1! 2!

2. The magnitude of the gradient for the function f ( x, y, z ) = x 2 + 3y 2 + z 3 at the point (1,1,1) is
_________.
Answer: 7

( )(
  
Exp: ( ∇f )P(1,1,1) = i ( 2x ) + j ( 6y ) + k ( 3z 2 )
P 1,1,1)
  
= 2i + 6 j + 3k
( ∇f ) P = 4 + 36 + 9 = 7

3. Let X be a zero mean unit variance Gaussian random variable. E  X  is equal to _____
Answer: 0.8
1 − x2 2
Exp: X ~ N ( 0,1) ⇒ f ( x ) = e , −∞ < x < ∞

∴ E{ x } = ∫

x .f ( x ) dx
−∞

1 ∞ −x2
= x2 ∫ x e 2
dx
2π 0

2 ∞ 2
= ∫ e− u du = = 0.797  0.8
2π 0 π

4. If a and b are constants, the most general solution of the differential equation
d2x dx
2
+ 2 + x = 0 is
dt dt
( A ) ae − t ( B) ae− t + bte − t ( C) ae t + bte − t ( D ) ae −2t
Answer: B
A.E : − m 2 + 2m + 1 = 0 ⇒ m = −1, −1
Exp:
∴ general solution is x = ( a + bt ) e − t

 India’s No.1 institute for GATE Training  1 Lakh+ Students trained till date  65+ Centers across India
4
EC-GATE-2014 PAPER-04| www.gateforum.com

xy
5. The directional derivative of f ( x, y ) = ( x + y ) at (1,1) in the direction of the unit vector at
2
π
an angle of with y-axis, is given by _____ .
4
Answer: 3
  2xy + y 2    x 2 + 2xy 
Exp: f =
1
( x 2 y + xy 2 ) ⇒ ∇f = i   + j 
2  2   2 
3  3 
at (1,1) , ∇f = i+ j
2 2
π
ê = unit vector in the direction i.e., making an angle of with y-axis
4
 π   π 
=  sin  i +  cos  j
 4  4
 3  1 
∴ directional derivative = e.
ˆ ∇f = 2   =3
 2  2

6. The circuit shown in the figure represents a

Ii A l Ii R

(A) Voltage controlled voltage source (B) Voltage controlled current source
(C) Current controlled current source (D) Current controlled voltage source
Answer: C
Exp:

A1I1

The dependent source represents a current controlled current source

7. The magnitude of current (in mA) through the resistor R2 in the figure shown is_______.
R2

1kΩ
R1 R3
10 mA 4 kΩ 2 mA
2 kΩ

R 4 3kΩ

 India’s No.1 institute for GATE Training  1 Lakh+ Students trained till date  65+ Centers across India
5
EC-GATE-2014 PAPER-04| www.gateforum.com

Answer: 2.8
Exp: By source transformation
R 2 = 1kΩ 4 kΩ

I
2 kΩ
+ −
20V − + 8V

3kΩ

By KVL,
20 − 10k.I + 8 = 0
28
⇒I=
10 k
⇒ I = 2.8mA

8. At T = 300 K, the band gap and the intrinsic carrier concentration of GaAs are 1.42 eV and
106 cm-3, respectively. In order to generate electron hole pairs in GaAs, which one of the
wavelength ( λ C ) ranges of incident radiation, is most suitable? (Given that: Plank’s constant
is 6.62 × 10-34 J-s, velocity of light is 3 × 1010 cm/s and charge of electron is 1.6 × 10-19 C)
(A) 0.42 µm < λ C < 0.87 µm (B) 0.87 µm < λ C < 1.42 µm
(C) 1.42 µm < λ C < 1.62 µm (D) 1.62 µm < λ C < 6.62 µm
Answer: A
hC 6.62 × 10−34 × 3 × 108
Exp: E = ⇒λ= = 0.87 µm
λ 1.42 × 1.6 × 10−19

9. In the figure ln ( ρi ) is plotted as a function of 1/T, where ρi the intrinsic resistivity of silicon,
T is is the temperature, and the plot is almost linear.
ln ( ρi )

1/ T
The slope of the line can be used to estimate
(A) Band gap energy of silicon (Eg)
(B) Sum of electron and hole mobility in silicon µ n + µ p ( )
( )
−1
(C) Reciprocal of the sum of electron and hole mobility in silicon µ n + µ p

(D) Intrinsic carrier concentration of silicon ( n i )

 India’s No.1 institute for GATE Training  1 Lakh+ Students trained till date  65+ Centers across India
6
EC-GATE-2014 PAPER-04| www.gateforum.com

Answer: A
3
n i α T 2 e − Eg /kT and
Exp: 1
ρι α
ηi
∴ From the graph, Energy graph of Si can be estimated

10. The cut-off wavelength (in µm) of light that can be used for intrinsic excitation of a
semiconductor material of bandgap Eg= 1.1 eV is ________
Answer: 1.125
hC
Exp: E=
λ
6.6 × 10−34 × 3 × 108
⇒λ= = 1.125 µm
1.1 × 1.6 × 10−19

11. If the emitter resistance in a common-emitter voltage amplifier is not bypassed, it will
(A) Reduce both the voltage gain and the input impedance
(B) Reduce the voltage gain and increase the input impedance
(C) Increase the voltage gain and reduce the input impedance
(D) Increase both the voltage gain and the input impedance
Answer: B
Exp: When a CE amplifier’s emitter resistance is not by passed, due to the negative feedback the
voltage gain decreases and input impedance increases

12. Two silicon diodes, with a forward voltage drop of 0.7 V, are used in the circuit shown in the
figure. The range of input voltage Vi for which the output voltage V0 = Vi , is
R
+ +

D1 D2
Vi VO

−1V ± ± 2V
− −

( A ) − 0.3V < Vi < 1.3V ( B) − 0.3V < Vi < 2V


( C) − 1.0 V < Vi < 2.0V ( D ) − 1.7 V < Vi < 2.7V
Answer: D
Exp: When Vi < −1.7V ; D1 − ON and D 2 − OFF
∴ V0 = −1.7V
When Vi > 2.7V; D1 − OFF & D 2 − ON
∴ V0 = 2.7V
When − 1.7 < Vi < 2.7V, Both D1 & D 2 OFF
∴ V0 = Vi

 India’s No.1 institute for GATE Training  1 Lakh+ Students trained till date  65+ Centers across India
7
EC-GATE-2014 PAPER-04| www.gateforum.com

13. The circuit shown represents:

C2 + 12 V
vi −
v0
R2 + − 12 V

R1
−2 V C1

(A) A bandpass filter (B) A voltage controlled oscillator


(C) An amplitude modulator (D) A monostable multivibrator
Answer: D

14. For a given sample-and-hold circuit, if the value of the hold capacitor is increased, then
(A) Droop rate decreases and acquisition time decreases
(B) Droop rate decreases and acquisition time increases
(C) Droop rate increases and acquisition time decreases
(D) Droop rate increases and acquisition time increases
Answer: B
dv
Exp: Capacitor drop rate =
dt
dv 1
For a capacitor, ∝
dt c
∴ Drop rate decreases as capacitor value is increased
For a capacitor, Q = cv = i × t ⇒ t ∝ c
∴ Acquisition time increases as capacitor value increased

15. In the circuit shown in the figure, if C=0, the expression for Y is

C
A
B
Y

A
B

(A) Y = A B + A B ( B) Y = A + B
( C) Y = A + B ( D) Y = A B

 India’s No.1 institute for GATE Training  1 Lakh+ Students trained till date  65+ Centers across India
8
EC-GATE-2014 PAPER-04| www.gateforum.com

Answer: A
Exp: C=0 1

A A⋅B
B
A⋅B Y

A ⋅ B + A ⋅ B = AB
A
B A⋅B

Y = 1.A  B
=A  B
= A ⊕ B = A B + AB + AB

16. The output (Y) of the circuit shown in the figure is

VD0
( A) A+ B+C

A B C

( B) A + B . C + A .C
A Output ( Y )

B
( C) A+ B+ C

( D) A.B .C
Answer: A
Exp: VDD

A B C

output ( Y )
A

 India’s No.1 institute for GATE Training  1 Lakh+ Students trained till date  65+ Centers across India
9
EC-GATE-2014 PAPER-04| www.gateforum.com

This circuit is CMOS implementation


If the NMOS is connected in series, then the output expression is product of each input with
complement to the final product.
So, Y = A.B .C
= A + B +C

17. A Fourier transform pair is given by


n − j6 πf
2 FT Ae
  u [ n + 3] ⇔
 
3 2
1 −   e − j2 πf
3
where u[n] denotes the unit step sequence. The value of A is _________.
Answer: 3.375
n
2
Exp: x [ n ] =   u [ n + 3]
3
−3
2 j3Ω
∞ n   .e
X ( e jΩ ) =
2  3
∑  3  .e − jΩn =
2
n =−3
1 − e − jΩ
3
3
 3  27
⇒A=  = = 3.375
2 8

w
18. A real-valued signal x(t) limited to the frequency band f ≤ is passed through a linear time
2
invariant system whose frequency response is
 − j4 πf w
e , f ≤
H (f ) = 
2
 0, f > w
 2
The output of the system is
( A ) x ( t + 4) ( B) x ( t − 4 ) ( C) x ( t + 2) ( D) x ( t − 2)
Answer: D
Exp: Let x ( t ) Fourier transform be x ( t )

x (t )
h (t ) y (t )

y ( t ) = x ( t ) * h ( t ) [ convolution ]
⇒ Y ( f ) = X ( f ) .H ( f )
⇒ Y ( f ) = e − j4 πf .X ( f )
⇒ y ( t ) = x ( t − 2)

 India’s No.1 institute for GATE Training  1 Lakh+ Students trained till date  65+ Centers across India
10
EC-GATE-2014 PAPER-04| www.gateforum.com

19. The sequence x[n] = 0.5n u[n], where u[n] is the unit step sequence, is convolved with itself to

obtain y[n]. Then ∑ y ( n ) _________________.
n =−∞

Answer: 4
Exp: y [n ] = x [n ] * x [n ]

Let Y ( e jΩ ) is F.T. pair with y [ n ]


⇒ Y ( e jΩ ) = X ( e jΩ ) .X ( e jΩ )

Y ( e jΩ ) =
1 1
− jΩ
.
1 − 0.5e 1 − 0.5e − jΩ

also Y ( e jΩ ) = ∑ y [ n ].e − j Ωn

h =−∞

∑ y [ n ] = Y ( e ) = 0.5 . 0.5 = 4
1 1
⇒ j0

n =−∞

20. In a Bode magnitude plot, which one of the following slopes would be exhibited at high
frequencies by a 4th order all-pole system?
(A) – 80 dB/decade (B) – 40 dB/decade
(C) +40 dB/decade (D) +80 dB/decade
Answer: A
Exp: → In a BODE diagram, in plotting the magnitude with respect to frequency, a pole introduce a
line 4 slope −20dB / dc
→ If 4th order all-pole system means gives a slope of ( −20 ) * 4 dB / dec i.e. − 80dB / dec

21. For the second order closed-loop system shown in the figure, the natural frequency (in rad/s)
is
U (s) + 4 Y (s)
S (S + 4 )

(A) 16 (B) 4 (C) 2 (D) 1


Answer: C
Y (s) 4
Exp: Transfer function =
U (s) S + 4s + 4
2

If we compare with standard 2nd order system transfer function


wn2
i.e.,
s 2 + 2ξw n s + w n 2
w n 2 = 4 ⇒ w n = 2 rad / sec

 India’s No.1 institute for GATE Training  1 Lakh+ Students trained till date  65+ Centers across India
11
EC-GATE-2014 PAPER-04| www.gateforum.com

22. If calls arrive at a telephone exchange such that the time of arrival of any call is independent
of the time of arrival of earlier or future calls, the probability distribution function of the total
number of calls in a fixed time interval will be
(A) Poisson (B) Gaussian (C) Exponential (D) Gamma
Answer: A
Exp: Poisson distribution: It is the property of Poisson distribution.

23. In a double side-band (DSB) full carrier AM transmission system, if the modulation index is
doubled, then the ratio of total sideband power to the carrier power increases by a factor of
_________________.
Answer: 4
Ratio of total side band power
Exp: α µ2
Carrier power
If it in doubled, this ratio will be come 4 times
24. For an antenna radiating in free space, the electric field at a distance of 1 km is found to be 12
mV/m. Given that intrinsic impedance of the free space is 120 π Ω, the magnitude of average
power density due to this antenna at a distance of 2 km from the antenna (in nW/m2 )
is________________.
Answer: 47.7
Exp: Electric field of an antenna is
 
 
ηI0 dl  Jβ 1 J 
Eθ = sin θ  + 2 −
4π r r βr 3 
 ↓ ↓ ↓ 
 field
Radiation inductive Electrostatic

 field field 
1
∴E α
r
E1 r2
= ⇒ E 2 = 6 mv / m
E 2 r1
1 E 2 1 36 × 10−8
P= = = 47.7 nw / m 2
2 η 2 120π

25. Match column A with column B.


Column A Column B
(1) Point electromagnetic source (P) Highly directional
(2) Dish antenna (Q) End free
(3) Yagi-Uda antenna (R) Isotropic
1→ P 1→ R 1→ Q 1→ R
(A) 2 → Q (B) 2 → P (C) 2 → P (D) 2 → Q
3→ R 3→ Q 3→ R 3→ P

 India’s No.1 institute for GATE Training  1 Lakh+ Students trained till date  65+ Centers across India
12
EC-GATE-2014 PAPER-04| www.gateforum.com

Answer: B
Exp: 1. Point electromagnetic source, can radiate fields in all directions equally, so isotropic.
2. Dish antenna → highly directional
3. Yagi – uda antenna → End fire

R F

Figure: Yagi-uda antenna

Q. No. 26 – 55 Carry Two Marks Each

26. With initial values y(0) = y’(0)=1 the solution of the differential equation
d2y dy
2
+ 4 + 4y = 0
dx dx
at x = 1 is ________
Answer: 0.54
Exp: A.E : m 2 + 4m + 4 = 0 ⇒ m = −2, −2
∴ solutions is y = ( a + bx ) e −2x .......(1)
y ' = ( a + bx ) ( −2e −2 x ) + e −2 x ( b ) ........ ( 2 )
u sin g y ( 0 ) = 1; y ( 0 ) = 1, (1) and ( 2 ) gives
'

a = 1 and b = 3
∴ y = (1 + 3x ) e−2x
at x = 1, y = 4e −2 = 0.541  0.54

27. Parcels from sender S to receiver R pass sequentially through two post-offices. Each post-
1
office has a probability of losing an incoming parcel, independently of all other parcels.
5
Given that a parcel is lost, the probability that it was lost by the second post-office
is_________.
Answer: 0.44
Exp: Parcel will be lost if
a. it is lost by the first post office
b. it is passed by first post office but lost by the second post office
1 4 1 9
Prob(parcel is lost) = + x =
5 5 5 25
P (Parcel lost by second post if it passes first post office)= P (Parcel passed by first post
office) x P(Parcel lost by second post office)

 India’s No.1 institute for GATE Training  1 Lakh+ Students trained till date  65+ Centers across India
13
EC-GATE-2014 PAPER-04| www.gateforum.com

4 1 4
= × =
5 5 25
4 / 25 4
Prob(parcel lost by 2nd post office | parcel lost)= = = 0.44
9 / 25 9

1
28. The unilateral Laplace transform of f ( t ) is . Which one of the following is the
s + s +1
2

unilateral Laplace transform of g(t) = t. f(t)?


−s − ( 2s + 1) S 2S + 1
( A) ( B) ( C) ( D)
(s ) (s ) (s ) (s )
2 2 2 2
2
+ s +1 2
+ s +1 2
+ s +1 2
+ s +1
Answer: D
Exp: (1)
If f ( t ) ↔ F ( s )
−d
Then tf ( t ) ↔ F(s)
ds
−d  1 
=  2 
ds  s + s + 1 
− ( 2s + 1) 2s + 1
=− =
( s2 + s + 1) ( s2 + s + 1)
2 2

Exp: (2)
1
F (s) =
s + s +12

d
L  g ( t ) = t.f ( t )  = −  F ( s )  ( u sin g multiplication by t )
ds 
2s + 1
=
(s + s + 1)
2 2

29. For a right angled triangle, if the sum of the lengths of the hypotenuse and a side is kept
constant, in order to have maximum area of the triangle, the angle between the hypotenuse
and the side is
(A) 12O (B) 36O (C) 60O (D) 45O
Answer: (C) ( As per IIT Website)
Exp: Let x (opposite side), y (adjacent side) and z (hypotenuse) of a right angled triangle.
Given Z + y = K ( cons tan t ) ......(1) and angle between them say ' θ ' then Area,

1 1 z2
A= xy = ( z sin θ )( z cos θ ) = sin 2θ
2 2 4
k
Now (1) ⇒ z + z sin θ = k ⇒ z =
1 + sin θ

 India’s No.1 institute for GATE Training  1 Lakh+ Students trained till date  65+ Centers across India
14
EC-GATE-2014 PAPER-04| www.gateforum.com

k2  sin 2θ 
∴A =  
 (1 + sin θ ) 
2
4
dA
In order to have max imum area, =0

k 2  (1 + sin θ ) ( 2cos 2θ ) − sin 2θ ( cos θ ) .2 (1 + sin θ ) 
2

⇒  =0
(1 + sin θ )
4
4  
π
⇒θ= = 30o , Answer obtained is different than official key
6

30. The steady state output of the circuit shown in the figure is given by
y ( t ) = A ( ω) sin ( ωt + φ ( ω) ) . If the amplitude A ( ω ) = 0.25, then the frequency ω is

R C y (t)
± sin ( ωt ) C
C

1 2 1 2
( A) ( B) ( C) ( D)
3RC 3RC RC RC
Answer: B
Exp: V

y (t )
R C
+
− sin ωt C
C

V − 1 0o V V
+ + =0
( ) ( )
By nodal method,
R 1 2
j ωc j ωc
1 j ωc  1 0 o
V  + j ωc + =
R 2  R
2
V=
2 + 3jωRC
V 1
Y= ⇒
2 2 + jω3RC
1 1
given A ( ω) = ⇒
4 4 + 9R 2 c2 .ω2
2
⇒ ω=
3 RC

 India’s No.1 institute for GATE Training  1 Lakh+ Students trained till date  65+ Centers across India
15
EC-GATE-2014 PAPER-04| www.gateforum.com

31. In the circuit shown in the figure, the value of V0(t) (in Volts) for t → ∞ is ______.
ix
2H

+ 2i +
10 u ( t ) A − x 5Ω Vo ( t )

5Ω

Answer: 31.25
Exp: B ix

2H
+ +
− 2ix
10x ( t ) 5Ω V0 ( t )
A 5Ω −

For t → ∞ , i.e., at steady state, inductor will behave as a shot circuit and hence VB = 5.i X
50
By KCL at node B, − 10 + VB − 2i x + i x = 0 ⇒ i x =
8
250
V0 ( t ) = 5i x ( t ) ⇒ V0 ( t ) = = 31.25 volts
8

32. The equivalent resistance in the infinite ladder network shown in the figure is Re.

2R R R R

Re R R R R

The value of Re/R is ________


Answer: 2.618
Exp:
R R R R

R R R R

R e qu

→ For an infinite ladder network, if all resistance are having same value of R
1+ 5 
Then equivalent resistance is   .R
 2 
→ For the given network, we can split in to R is in series with Requivalent

 India’s No.1 institute for GATE Training  1 Lakh+ Students trained till date  65+ Centers across India
16
EC-GATE-2014 PAPER-04| www.gateforum.com

Re q =
(1 + 5 ) R
2

R e qu

R equ
⇒ R equ = R + 1.618R ⇒ = 2.618
R

33. For the two-port network shown in the figure, the impedance (Z) matrix (in Ω ) is

30 Ω
1 2
+ +
10 Ω 60 Ω
− −
1' 2'

 6 24  9 8  9 6   42 6 
( A)  42 9  ( B) 8 24  ( C) 6 24  ( D)  6 60
       
Answer: C
Exp: For the two-part network
1 1 1 
 30 + 10 −
30 
Y matrix =  
 −1 1
+
1
 30 60 30 
Zmatrix = [ Y ]
−1

−1
 0.1333 − 0.0333
Z=
 −0.0333 0.05 
9 6 
Z= 
 6 24 

34. Consider a silicon sample doped with ND = 1×1015/cm3 donor atoms. Assume that the
intrinsic carrier concentration ni = 1.5×1010/cm3. If the sample is additionally doped with NA
= 1×1018/cm3 acceptor atoms, the approximate number of electrons/cm3 in the sample, at
T=300 K, will be _________________.
Answer: 225.2
Exp: P = N A − N D = 1 × 1018 − 1 × 1015 = 9.99 × 1017

η 2 (1.5 × 10 )
10 2

η= i = = 225.2 / cm3
P 9.99 × 1017

 India’s No.1 institute for GATE Training  1 Lakh+ Students trained till date  65+ Centers across India
17
EC-GATE-2014 PAPER-04| www.gateforum.com

35. Consider two BJTs biased at the same collector current with area A1 = 0.2µm × 0.2 µm and
A 2 = 300 µm × 300 µm . Assuming that all other device parameters are identical, kT/q = 26
mV, the intrinsic carrier concentration is 1 × 1010 cm-3, and q = 1.6 × 10-19 C, the difference
between the base-emitter voltages (in mV) of the two BJTs (i.e., VBE1 – VBE2) is
_________________.
Answer: 381
Exp: IC1 = IC2 ( Given )
VBE1

IS1 e VT = IS2 e BE2


V /VT

( VBE1 − VBE2 ) IS
e VT
= 2
IS1
IS2  300 × 300 
VBE1 − VBE 2 = VT ln = 26 × 10−3 ln   ∵ I S αA
IS1  0.2 × 0.2 
(V BE1 )
− VBE2 = 381mV

36. An N-type semiconductor having uniform doping is biased as shown in the figure.
V

N − type semiconductor

If EC is the lowest energy level of the conduction band, EV is the highest energy level of the
valance band and EF is the Fermi level, which one of the following represents the energy
band diagram for the biased N-type semiconductor?
EC EC
EF
( A) ( B) EF

EV EV

EC
EF
( D)
( C)
EV EC
EF
EV
Answer: D

37. Consider the common-collector amplifier in the figure (bias circuitry ensures that the
transistor operates in forward active region, but has been omitted for simplicity). Let IC be the
collector current, VBE be the base-emitter voltage and VT be the thermal voltage. Also, g m
and r0 are the small-signal transconductance and output resistance of the transistor,
respectively. Which one of the following conditions ensures a nearly constant small signal
voltage gain for a wide range of values of RE?

 India’s No.1 institute for GATE Training  1 Lakh+ Students trained till date  65+ Centers across India
18
EC-GATE-2014 PAPER-04| www.gateforum.com

VCC

Vin Vout

RE

( A) g m R E << 1 ( B) I C R E >> VT ( C) g m r0 >> 1 ( D) VBE >> Vr


Answer: B
RE RE IE R E
Exp: AV = = =
re + R E VT
+ R E VT + I E R E
IE
IC R E
∴ AV  (∵ IC  IE )
VT + IC R E
∴ IC R E >> U T ⇒ A V in almost cons tan t

38. A BJT in a common-base configuration is used to amplify a signal received by a 50Ω


antenna. Assume kT/q = 25 mV. The value of the collector bias current (in mA) required to
match the input impedance of the amplifier to the impedance of the antenna is________.
Answer: 0.5
VI
Exp: Input impedance of CB amplifier, z i = re =
IE
25 mV
⇒ 50 = (∵ signal is received from 50Ω antenna and VT = 25 mV )
IE
25mV
⇒ IE = = 0.5 mA
50Ω

39 . For the common collector amplifier shown in the figure, the BJT has high β , negligible
VCE(sat), and VBE = 0.7 V. The maximum undistorted peak-to-peak output voltage vo (in Volts)
is______.

VCC = +12V

R1
5kΩ
1 µF
vi
1 µF
R2 vo
10 kΩ RE
1kΩ

 India’s No.1 institute for GATE Training  1 Lakh+ Students trained till date  65+ Centers across India
19
EC-GATE-2014 PAPER-04| www.gateforum.com

Answer: 9.4
Exp: ∵ β = high, I B is neglected
10 k
∴ VB = 12 × = 8V
10 k + 5k
VE = VB − 0.7 = 7.3V
∴ VCE = 12 − 7.3 = 4.7V
∴ Maximum undistorted V0 ( p − p ) = 2 × 4.7V = 9.4V

40. An 8-to-1 multiplexer is used to implement a logical function Y as shown in the figure. The
output Y is given by

0 I0
D I1
0 I2
D I3
0 I4 Y
0 I5
1 I6
0 I7 S2 S1 S0

A B C

( A) Y= A BC + ACD ( B) Y= A BC + A BD

( C) Y= A BC + ACD ( D) Y= A BD + A BC
Answer: C
Exp: Y = ABCD + ABCD + ABC 0 Io
D I1
I2
Remaining combinations of the select 0
D I3 8 :1 Y
lines will produce output 0. 0 I4
MUX
I5
So, Y = ACD ( B + B ) + ABC
0
1 I6
0 I7
S 2 S1 S0
= ACD + ABC
= ABC + ACD B C
A

41. A 16-bit ripple carry adder is realized using 16 identical full adders (FA) as shown in the
figure. The carry-propagation delay of each FA is 12 ns and the sum-propagation delay of
each FA is 15 ns. The worst case delay (in ns) of this 16-bit adder will be __________.
A0 B0 A1 B1 A14 B14 A15 B15

C0 C1 C14 C15
FA 0 FA1 FA14 FA15

S0 S1 S14 S15

 India’s No.1 institute for GATE Training  1 Lakh+ Students trained till date  65+ Centers across India
20
EC-GATE-2014 PAPER-04| www.gateforum.com

Answer: 195
Exp: A1 A14 A15
A0 B0 B1 B14 B15

C1 C14 C15
FA 0
C0
FA1 ........ . FA14 FA15

S0 S1 S14 S15

This is 16-bit ripple carry adder circuit, in their operation carry signal is propagating from 1st
stage FA0 to last state FA15, so their propagation delay is added together but sum result is
not propagating. We can say that next stage sum result depends upon previous carry.
So, last stage carry (C15) will be produced after 16 ×12 ns = 192 ns
Second last stage carry (C14) will be produced after 180 ns.
For last stage sum result (S15) total delay = 180ns + 15ns = 195ns
So, worst case delay = 195 ns

42. An 8085 microprocessor executes “STA 1234H” with starting address location 1FFEH (STA
copies the contents of the Accumulator to the 16-bit address location). While the instruction
is fetched and executed, the sequence of values written at the address pins A15-A8 is
(A) 1FH, 1FH, 20H, 12H (B) 1FH, FEH, 1FH, FFH, 12H
(C) 1FH, 1FH, 12H, 12H (D) 1FH, 1FH, 12H, 20H, 12H
Answer: A
Exp: Let the opcode of STA is XXH and content of accumulator is YYH.
Instruction: STA 1234 H
Starting address given = 1FFEH
So, the sequence of data and addresses is given below:

Address (in hex) : Data (in hex)


A 15 − A 8 A 7 − A 0

1F FE H → XXH
1F FF H → 34H
20 00 H → 12 H
12 34 H → YYH

1
43. A stable linear time invariant (LTI) system has a transfer function H ( s) = . To make
s2 + s − 6
this system causal it needs to be cascaded with another LTI system having a transfer function
H1(s). A correct choice for H1(s) among the following options is
(A) s + 3 (B) s - 2 (C) s - 6 (D) s + 1

 India’s No.1 institute for GATE Training  1 Lakh+ Students trained till date  65+ Centers across India
21
EC-GATE-2014 PAPER-04| www.gateforum.com

Answer: B
1 1
Exp: Given, H ( s ) = =
s + s − 6 ( s + 3)( s − 2 )
2

It is given that system is stable thus its ROC includes jω axis . This implies it cannot be
causal, because for causal system ROC is right side of the rightmost pole.
⇒ Poles at s = 2 must be removes so that it can be become causal and stable
simultaneously.
1 1
⇒ ( s − 2) =
( s + 3)( s − 2) s+3

Thus H1 ( s ) = s − 2

44. A causal LTI system has zero initial conditions and impulses response h(t). Its input x(t) and
output y(t) are related through the linear constant-coefficient differential equation
d2 y ( t ) dy ( t )
2
+a + a2y ( t) = x ( t)
dt dt
Let another signal g(t) be defined as
dh ( t )
g ( t ) = a 2 ∫ h ( τ ) dτ + + ah ( t )
t

0 dt
If G(s) is the Laplace transform of g(t), then the number of poles of G(s) is _______.
Answer: 1
Exp: Given differential equation
s 2 y ( s ) + α sy ( s ) + α 2 y ( s ) = x ( s )

y (s ) 1
⇒ = = H (s )
x (s ) s + αs + α 2
2

t
d
g ( t ) = α 2 ∫ h ( z ) dz + h ( t ) + αh ( t )
0 dt

H (s )
= α2 + SH ( s ) + α H ( s )
s
1 1 α
= α2 +s 2 + 2
s ( s + αs + α )
22
( s + 2s + α )
2
s + α s + α2

α 2 + αs + s 2 1
= =
s ( s + αs + α ) s
2 2

No. of poles = 1

 India’s No.1 institute for GATE Training  1 Lakh+ Students trained till date  65+ Centers across India
22
EC-GATE-2014 PAPER-04| www.gateforum.com

45. The N-point DFT X of a sequence x [ n ] ,0 ≤ n ≤ N − 1 is given by


N −1 2π
1 −j
X[k] = ∑ x [ n]e
nk,
N
0 ≤ k ≤N − L
N n=0

Denote this relation as X = DFT(x). For N = 4, which one of the following sequences
satisfies DFT(DFT (x))=x ?
( A) x = [1 2 3 4 ] ( B) x = [1 2 3 2 ]
( C) x = [1 3 2 2 ] ( D) x = [1 2 2 3]
Answer: B
Exp: This can be solve by directly using option and satisfying the condition given in question
X = DFT ( x )
N −1 2π
D FT ( D FT ( x ) ) = DFT ( X ) =
1 −j
∑ X [n ] e
nk
N

N n =0

DFT y [1 2 3 4]

1 1 1 1  1   10 
1 − j − 1 j   2   
X=
1     = 1  2 + 2 j
4 1 − 1 1 − 1  3  4 2 
    
1 + j − 1 − j  4   −2 − j2 
DFT of ( x ) will not result in [1 2 3 4]

Try with DFT of y [1 2 3 2]

1 1 1 1  1   8  4 
1 − j − 1 j   2     
X=
1     = 1  −2  =  −1
4 1 − 1 1 − 1  3  4  0  0 
      
1 + j − 1 − j  2   −2   −1

 4 1 1 1   4
1  2  1 
−   − j − 1 j   −1 1  4   2 
  
  1 1
DFT of = = =
 0 4 1 − 1 1 − 1   0  2  6  3 
        
 −1 1 + j − 1 − j  −1 4 2
Same as x
Then ‘B’ is right option

. 
x1  0 1   x1 
46. The state transition matrix φ ( t ) of a system  .  = 
  0 0  x 2 
x 2 
 t 1 1 0 0 1 1 t 
( A)   ( B)   ( C)   ( D)  
1 0  t 1 1 t  0 1

 India’s No.1 institute for GATE Training  1 Lakh+ Students trained till date  65+ Centers across India
23
EC-GATE-2014 PAPER-04| www.gateforum.com

Answer: D
Exp: Given state model,
 x 1 ( t )   0 1   x1 ( t ) 
 =  
 x 2 ( t )   0 0   x 2 ( t ) 
0 1 
A= 
0 0
φ ( t ) ⇒ state transistion matrix
φ ( t ) = L−1 ( SI − A ) 
−1
 
−1
s − 1 1 s 1
[SI − A ] = 
−1
 ⇒ 2 0
0 s  s  s 
1 1 2
s
φ ( t ) = L−1  s 
0 1 
 s 
1 t 
φ( t ) =  
0 1 

ps2 + 3ps − 2
47. Consider a transfer function G p ( s ) = with p a positive real parameter.
s2 + ( 3 + p ) s + ( 2 − p )
The maximum value of p until which GP remains stable is ________.
Answer: 2
ps 2 + 3ps − 2
Exp: Given G p ( s ) =
s2 + ( 3 + p ) s + ( 2 − p )
By R − H criteria
The characteristic equation is s 2 + ( 3 + p ) s + ( 2 − p ) = 0
i.e. s 2 + ( 3 + p ) s + ( 2 − p ) = 0
By forming R-H array,

s2 1 (2 − p)
s1 ( 3 + φ ) 0
s0 ( 2 − p )

For stability, first column elements must be positive and non-zero


i.e. (1)( 3 + p ) > 0 ⇒ p > −3
and ( 2 )( 2 − p ) > 0 ⇒ p < 2
i.e. −3 < p < 2

The maximum value of p unit which G p remains stable is 2

 India’s No.1 institute for GATE Training  1 Lakh+ Students trained till date  65+ Centers across India
24
GATE ESE PSU’s 2019-20
ECE ENGINEERING
GATE ECE 2003-2019 SOLVED

GATE ECE 2003-2019 SOLVED Detail Solution

CONTENT COVERED:
1.Theory Notes
2.Explanation
3.Derivation
4.Example
5.Shortcut & Formula Summary
6.Previous year Paper Q. Sol.
Noted-: Single Source Follow, Revise
Multiple Time Best key of Success
1
Page

http://www.orbitmentor.com [email protected]
EC-GATE-2014 PAPER-04| www.gateforum.com

48. The characteristic equation of a unity negative feedback system 1 + KG(s) = 0. The open loop
transfer function G(s) has one pole at 0 and two poles at -1. The root locus of the system for
varying K is shown in the figure.

ζ = 0.5
A

x x
( 0,0) σ
−1 1
O

3

The constant damping ratio line, for ζ = 0.5 , intersects the root locus at point A. The distance
from the origin to point A is given as 0.5. The value of K at point A is ________ .
Answer: 0.375
Exp: We know that the co-ordinate of point A of the given root locus i.e., magnitude condition
G (s) H (s) = 1
Here, the damping factor ξ = 0.5 and the length of 0A = 5
ξ = 0.5
A
Then in the right angle triangle *
OX OX 1
cos θ = ⇒ cos 60 = ⇒ OX = θ
OA 0.5 4 * * *O
−1 −2 −1
X
3 3
AX AX 3
⇒ sin θ = ⇒ sin 60 = ⇒ AX =
OA 0.5 4

So, the co-ordinate of point A is −1 + j 3


4 4
Substituting the above value of A in the transfer function and equating to 1
i.e. by magnitude condition,

k
=1
s ( s + 1)
2
s = −1 + j 3
4 4

2
1 3  9 3 
k= + . + 
16 16  16 16 

k = 0.375

 India’s No.1 institute for GATE Training  1 Lakh+ Students trained till date  65+ Centers across India
25
EC-GATE-2014 PAPER-04| www.gateforum.com

49. Consider a communication scheme where the binary valued signal X satisfies P{X = +
1}=0.75 and P{X = -1}= 0.25. The received signal Y = X + Z, where Z is a Gaussian random
variable with zero mean and variance σ 2 . The received signal Y is fed to the threshold
detector. The output of the threshold detector X̂ is:

X̂ = { +−1.1. Y>τ
Y ≤ τ.

ˆ ≠ X , the threshold τ should be


To achieve a minimum probability of error P X { }
(A) Strictly positive
(B) Zero
(C) Strictly negative
(D) Strictly positive, zero, or strictly negative depending on the nonzero value of σ 2
Answer: C
Exp: C

H1 : x = +1; H 0 : x = −1
P ( H1 ) = 0.75; P ( H 0 ) = 0.25
Received signal γ =X+Z
1
Where Z ∼ N ( 0, −2 ) ; f Z ( z ) = e− Z 2 σ2
2

σ 2π
 1+ Z if X = 1
Received signal γ = 
 −1 + Z if X = −1
1
1 − ( γ−1)2
f γ ( y H1 ) = e 2 σ2

σ 2π
1
1 − ( γ+1)2
f γ ( y H0 ) = e 2 σ2

σ 2π

At optimum threshold yopt: for minimum probability of error

f γ ( y H1 ) P ( H0 )
=
f γ ( y H0 ) P ( H1 )
y = yopt


1 
2 σ 2 
( γ−1)2 − ( γ+1)2  P ( H0 )
e =
yopt
P ( H1 )

+ 2 yopt σ2 P ( H0 )
e =
P ( H1 )
σ 2  P ( H 0 )  −1.1σ 2
y opt = ln   = = −0.55σ 2
2  P ( H1 )  2
y opt = Optimum threshold
y opt < 0 ∴ Threshold is negative.

 India’s No.1 institute for GATE Training  1 Lakh+ Students trained till date  65+ Centers across India
26
EC-GATE-2014 PAPER-04| www.gateforum.com

50. Consider the Z-channel given in the figure. The input is 0 or 1 with equal probability.

1.0
0 0

INPUT 0.25 OUTPUT

1 1
0.75

If the output is 0, the probability that the input is also 0 equals ______________
Answer: 0.8
Exp: Given channel
1.0
X=0 Y =0

input output
0.25

X =1 Y =1
0.75

We have to det er min e, P {x = 0 / y = 0}

P {y = 0 / x = 0} P {x = 0} 1. 1 4
P {x = 0 / y = 0} = = 2 = = 0.8
P {y = 0} 1. 1 + 0.25 ×
1 5
2 2

51. An M-level PSK modulation scheme is used to transmit independent binary digits over a
band-pass channel with bandwidth 100 kHz. The bit rate is 200 kbps and the system
characteristic is a raised-cosine spectrum with 100% excess bandwidth. The minimum value
of M is ________.
Answer: 16
1
Exp: Bandwidth requirement for m-level PSK = (1 + α )
T
[Where T is symbol duration. α is roll of factor]
1
⇒ (1 + α ) = 100 × 103
T
α = 1 [100% excess bandwidth ]
1
⇒ ( 2 ) = 100 × 103
T
Bit duration
2
⇒T= 1
100 × 103 = = 0.5 × 10−5 = 5 × 10−6 sec
= 20 µ sec 200 × 10 3

Symbol duration 20 × 10−6 sec


Bit duration = ⇒ log 2 m = = 4 ⇒ M = 16
log 2 m 5 × 10−6

 India’s No.1 institute for GATE Training  1 Lakh+ Students trained till date  65+ Centers across India
27
EC-GATE-2014 PAPER-04| www.gateforum.com

52. Consider a discrete-time channel Y =- X + Z, where the additive noise z is signal-dependent.


In particular, given the transmitted symbol X ∈{ − a, + a} . at any instant, the noise sample Z is
chosen independently from a Gaussian distribution with mean β X and unit variance. Assume
a threshold detector with zero threshold at the receiver.
When β = 0, the BER was found to be Q(a) = 1 × 10-8
 1 ∞ 
 Q ( v ) = ∫ e − u / 2du, and for v > 1, use Q ( v ) ≈ e − v
2 2
/2

2π v

When β = −0.3, the BER is closest to


(A) 10-7 (B) 10-6 (C) 10-4 (D) 10-2
Answer: C
Exp: X∈[-a,a] and P(x = -a) = P(x = a) = ½

γ =X+Z → Received signal

Z ∼ N ( βX,1)
Q ( a ) = 1 × 10−8
1
− ( Z −βX )
2
1
fZ ( z ) = Q ( a ) ≈ e−ϑ
2
2 2
e

 −a + z if x = −a
γ=
 a + z if x = + a
H1 : x = + a
H 0 : x = −a
and Threshold = 0
1 −
1
( y − a (1+β ) )2
f γ ( y H1 ) = e 2


1 −
1
( y + a (1+β ) )2
fγ ( y H0 ) = e 2


BER :
Pe = P ( H1 ) P ( e H1 ) + P ( H 0 ) P ( e H 0 )

1 − 12 ( y − a (1+β) )2 1 − 12 ( y + a (1+β ))2
0
dy = Q ( a (1 + β ) )
1 1
= ∫ 2π
2 −∞
e dy +
2 −∫0 2π
e

β=0
Pe = Q ( a ) = 1 × 10−8 = e− a ⇒ a = 6.07
2
2

β = −0.3
Pe = Q ( 6.07 (1 − 0.3) ) = Q ( 4.249 )
− ( 4.249 ) 2
2
Pe = e = 1.2 × 10−4
Pe  10−4.

 India’s No.1 institute for GATE Training  1 Lakh+ Students trained till date  65+ Centers across India
28
EC-GATE-2014 PAPER-04| www.gateforum.com

53. The electric field (assumed to be one-dimensional) between two points A and B is shown. Let
ψ A and ψ B be the electrostatic potentials at A and B, respectively. The value of ψ B − ψ A in
Volts is ________.

40 kV / cm
20 kV / cm

0 kV / cm
A B
5µ m
Answer: -15
Exp: A B

( 0 kV / cm, 20kV / cm ) (5 × 10 −4
kV / cm, 40 kV / cm )

40 − 20
−4 (
E − 20 = x − 0 ) ⇒ E = 4 × 104 x + 20
5 × 10
( 4 × 10 x + 20 ) dx
B 5×10−4 / cm
VAB = − ∫ E.dl = − ∫ 4
A 0
5×10−4
 
= − ( 2 × 104 × 25 × 10−8 + 20 × 5 × 10−4 )
x2
= −  4 × 104 + 20x 
 2 0
= − ( 50 × 10−4 + 100 × 10−4 ) = −150 × 10−4 kV
⇒ VAB = −15V


54. Given F = zaˆ x + xaˆ y + yaˆ z . If S represents the portion of the sphere x 2 + y 2 + z 2 = 1 for
 
z ≥ 0 , then ∫ ∇ × F . ds is ___________.
S

Answer: 3.14
   
Exp: ∫ ∇ × F.ds = ∫
S C
F.dr(u sin g stoke 's theorem and C is closed curve i.e.,

x 2 + y 2 = 1, z = 0
⇒ x = cos θ, y = sin θ and θ : 0 to 2π

= ∫ zdx + xdy + ydz


C


= ∫ xdy = ∫ cos θ ( cos θ dθ )
C 0


1 sin 2θ 
= θ +  = π  3.14
2 2 0

 India’s No.1 institute for GATE Training  1 Lakh+ Students trained till date  65+ Centers across India
29
EC-GATE-2014 PAPER-04| www.gateforum.com

55. If E = − ( 2y3 − 3yz 2 ) xˆ − ( 6xy 2 − 3xz 2 ) yˆ + ( 6xyz ) zˆ is the electric field in a source free
region, a valid expression for the electrostatic potential is
( A) xy 3 − yz 2 ( B) 2xy 3 − xyz 2 (C) y3 + xyz 2 ( D) 2xy 3 − 3xyz 2
Answer: D
Exp: Given E = − ( 2y3 − 3yz 2 ) a x − ( 6xy 2 − 3xz 2 ) a y + 6xyz.a z
By verification option (D) satisfy
E = −∇V

 India’s No.1 institute for GATE Training  1 Lakh+ Students trained till date  65+ Centers across India
30
GATE 2016 General Aptitude - GA Set-1

Q. 1 – Q. 5 carry one mark each.


Q.1 Which of the following is CORRECT with respect to grammar and usage?

Mount Everest is ____________.


(A) the highest peak in the world

(B) highest peak in the world

(C) one of highest peak in the world

(D) one of the highest peak in the world

Q.2 The policeman asked the victim of a theft, “What did you ?”
(A) loose (B) lose (C) loss (D) louse

Q.3 Despite the new medicine’s ______________ in treating diabetes, it is not ______________widely.
(A) effectiveness --- prescribed (B) availability --- used
(C) prescription --- available (D) acceptance --- proscribed

Q.4 In a huge pile of apples and oranges, both ripe and unripe mixed together, 15% are unripe fruits. Of
the unripe fruits, 45% are apples. Of the ripe ones, 66% are oranges. If the pile contains a total of
5692000 fruits, how many of them are apples?

(A) 2029198 (B) 2467482 (C) 2789080 (D) 3577422

Q.5 Michael lives 10 km away from where I live. Ahmed lives 5 km away and Susan lives 7 km away
from where I live. Arun is farther away than Ahmed but closer than Susan from where I live. From
the information provided here, what is one possible distance (in km) at which I live from Arun’s
place?

(A) 3.00 (B) 4.99 (C) 6.02 (D) 7.01

Q. 6 – Q. 10 carry two marks each.

Q.6 A person moving through a tuberculosis prone zone has a 50% probability of becoming infected.
However, only 30% of infected people develop the disease. What percentage of people moving
through a tuberculosis prone zone remains infected but does not show symptoms of disease?

(A) 15 (B) 33 (C) 35 (D) 37

1/2
GATE 2016 General Aptitude - GA Set-1

Q.7 In a world filled with uncertainty, he was glad to have many good friends. He had always assisted
them in times of need and was confident that they would reciprocate. However, the events of the
last week proved him wrong.

Which of the following inference(s) is/are logically valid and can be inferred from the above
passage?

(i) His friends were always asking him to help them.

(ii) He felt that when in need of help, his friends would let him down.

(iii) He was sure that his friends would help him when in need.

(iv) His friends did not help him last week.

(A) (i) and (ii) (B) (iii) and (iv) (C) (iii) only (D) (iv) only

Q.8 Leela is older than her cousin Pavithra. Pavithra’s brother Shiva is older than Leela. When Pavithra
and Shiva are visiting Leela, all three like to play chess. Pavithra wins more often than Leela does.

Which one of the following statements must be TRUE based on the above?
(A) When Shiva plays chess with Leela and Pavithra, he often loses.

(B) Leela is the oldest of the three.

(C) Shiva is a better chess player than Pavithra.

(D) Pavithra is the youngest of the three.

1 1 1
If 𝑞𝑞 −𝑎𝑎 and 𝑟𝑟 −𝑏𝑏 and 𝑠𝑠 −𝑐𝑐
Q.9
= = = , the value of abc is .
𝑟𝑟 𝑠𝑠 𝑞𝑞

(A) (𝑟𝑟𝑟𝑟𝑟𝑟)−1 (B) 0 (C) 1 (D) r+q+s

Q.10 P, Q, R and S are working on a project. Q can finish the task in 25 days, working alone for 12
hours a day. R can finish the task in 50 days, working alone for 12 hours per day. Q worked 12
hours a day but took sick leave in the beginning for two days. R worked 18 hours a day on all days.
What is the ratio of work done by Q and R after 7 days from the start of the project?

(A) 10:11 (B) 11:10 (C) 20:21 (D) 21:20

END OF THE QUESTION PAPER

2/2
GATE 2016 Electronics and Communication Engineering (Set 1)

Q. 1 – Q. 25 carry one mark each.


Q.1 Let M4 = I, (where I denotes the identity matrix) and M ≠ I, M2 ≠ I and M3 ≠ I. Then, for any
natural number k, M−1 equals:
(A) M4k + 1 (B) M4k + 2 (C) M4k + 3 (D) M4k

Q.2 The second moment of a Poisson-distributed random variable is 2. The mean of the random
variable is _______

Q.3 Given the following statements about a function 𝑓: ℝ → ℝ, select the right option:

P: If f(x) is continuous at 𝑥 = 𝑥0 , then it is also differentiable at 𝑥 = 𝑥0 .


Q: If f(x) is continuous at 𝑥 = 𝑥0 , then it may not be differentiable at 𝑥 = 𝑥0 .
R: If f(x) is differentiable at 𝑥 = 𝑥0 , then it is also continuous at 𝑥 = 𝑥0 .

(A) P is true, Q is false, R is false (B) P is false, Q is true, R is true


(C) P is false, Q is true, R is false (D) P is true, Q is false, R is true

Q.4 Which one of the following is a property of the solutions to the Laplace equation: ∇2 𝑓 = 0?
(A) The solutions have neither maxima nor minima anywhere except at the boundaries.
(B) The solutions are not separable in the coordinates.
(C) The solutions are not continuous.
(D) The solutions are not dependent on the boundary conditions.

EC (1) 1/17
GATE 2016 Electronics and Communication Engineering (Set 1)

Q.5 Consider the plot of 𝑓(𝑥) versus 𝑥 as shown below.

𝑥
Suppose 𝐹(𝑥) = ∫−5 𝑓(𝑦)𝑑𝑦. Which one of the following is a graph of 𝐹(𝑥)?

(A) (B)

(C) (D)

Q.6 Which one of the following is an eigen function of the class of all continuous-time, linear, time-
invariant systems (𝑢(𝑡) denotes the unit-step function)?
(A) 𝑒 𝑗𝜔0 𝑡 𝑢(𝑡) (B) cos (𝜔0 𝑡)

(C) 𝑒 𝑗𝜔0 𝑡 (D) sin (𝜔0 𝑡)

Q.7 A continuous-time function x(t) is periodic with period T. The function is sampled uniformly with
a sampling period 𝑇𝑠 . In which one of the following cases is the sampled signal periodic?
(A) 𝑇 = √2 𝑇𝑠 (B) 𝑇 = 1.2 𝑇𝑠
(C) Always (D) Never

Q.8 Consider the sequence 𝑥[𝑛] = 𝑎𝑛 𝑢[𝑛] + 𝑏 𝑛 𝑢[𝑛], where 𝑢[𝑛] denotes the unit-step sequence and
0 < |𝑎| < |𝑏| < 1. The region of convergence (ROC) of the z-transform of 𝑥[𝑛] is
(A) |𝑧| > |𝑎| (B) |𝑧| > |𝑏| (C) |𝑧| < |𝑎| (D) |𝑎| < |𝑧| < |𝑏|

EC (1) 2/17
GATE 2016 Electronics and Communication Engineering (Set 1)

Q.9 𝐴 𝐵
Consider a two-port network with the transmission matrix: 𝑇 = � �. If the network is
𝐶 𝐷
reciprocal, then
(A) 𝑇 −1 = 𝑇 (B) 𝑇 2 = 𝑇
(C) Determinant (T) = 0 (D) Determinant (T) = 1

Q.10 A continuous-time sinusoid of frequency 33 Hz is multiplied with a periodic Dirac impulse train of
frequency 46 Hz. The resulting signal is passed through an ideal analog low-pass filter with a cutoff
frequency of 23 Hz. The fundamental frequency (in Hz) of the output is _________

Q.11 A small percentage of impurity is added to an intrinsic semiconductor at 300 K. Which one of the
following statements is true for the energy band diagram shown in the following figure?

(A) Intrinsic semiconductor doped with pentavalent atoms to form n-type semiconductor
(B) Intrinsic semiconductor doped with trivalent atoms to form n-type semiconductor
(C) Intrinsic semiconductor doped with pentavalent atoms to form p-type semiconductor
(D) Intrinsic semiconductor doped with trivalent atoms to form p-type semiconductor

Q.12 Consider the following statements for a metal oxide semiconductor field effect transistor
(MOSFET):

P: As channel length reduces, OFF-state current increases.


Q: As channel length reduces, output resistance increases.
R: As channel length reduces, threshold voltage remains constant.
S: As channel length reduces, ON current increases.

Which of the above statements are INCORRECT?


(A) P and Q (B) P and S (C) Q and R (D) R and S

EC (1) 3/17
GATE 2016 Electronics and Communication Engineering (Set 1)

Q.13 Consider the constant current source shown in the figure below. Let 𝛽 represent the current gain of
the transistor.


Vref


The load current 𝐼0 through RL is

𝛽+1 𝑉𝑟𝑒𝑓 𝛽 𝑉𝑟𝑒𝑓 𝛽+1 𝑉𝑟𝑒𝑓 𝛽 𝑉𝑟𝑒𝑓


(A) 𝐼0 = � 𝛽
� 𝑅 (B) 𝐼0 = �𝛽+1� 𝑅
(C) 𝐼0 = � 𝛽
� 2𝑅 (D) 𝐼0 = �𝛽+1� 2𝑅

EC (1) 4/17
GATE 2016 Electronics and Communication Engineering (Set 1)

Q.14 The following signal Vi of peak voltage 8 V is applied to the non-inverting terminal of an ideal
opamp. The transistor has VBE = 0.7 V, β =100; VLED = 1.5 V, VCC = 10 V and −VCC = −10 V.

The number of times the LED glows is ________

EC (1) 5/17
GATE 2016 Electronics and Communication Engineering (Set 1)

Q.15 Consider the oscillator circuit shown in the figure. The function of the network (shown in dotted
lines) consisting of the 100 kΩ resistor in series with the two diodes connected back-to-back is to:

(A) introduce amplitude stabilization by preventing the op amp from saturating and thus producing
sinusoidal oscillations of fixed amplitude
(B) introduce amplitude stabilization by forcing the opamp to swing between positive and negative
saturation and thus producing square wave oscillations of fixed amplitude
(C) introduce frequency stabilization by forcing the circuit to oscillate at a single frequency
(D) enable the loop gain to take on a value that produces square wave oscillations

EC (1) 6/17
GATE 2016 Electronics and Communication Engineering (Set 1)

Q.16 The block diagram of a frequency synthesizer consisting of a Phase Locked Loop (PLL) and a
divide-by-𝑁 counter (comprising ÷ 2 ,÷ 4, ÷ 8, ÷ 16 outputs) is sketched below. The
synthesizer is excited with a 5 kHz signal (Input 1). The free-running frequency of the PLL is set to
20 kHz. Assume that the commutator switch makes contacts repeatedly in the order 1-2-3-4.

The corresponding frequencies synthesized are:


(A) 10 kHz, 20 kHz, 40 kHz, 80 kHz
(B) 20 kHz, 40 kHz, 80 kHz, 160 kHz
(C) 80 kHz, 40 kHz, 20 kHz, 10 kHz
(D) 160 kHz, 80 kHz, 40 kHz, 20 kHz

Q.17 The output of the combinational circuit given below is

(A) A+B+C (B) A(B+C) (C) B(C+A) (D) C(A+B)

EC (1) 7/17
GATE 2016 Electronics and Communication Engineering (Set 1)

Q.18 What is the voltage Vout in the following circuit?


VDD

(A) 0 V (B) (|VT of PMOS| + VT of NMOS) / 2


(C) Switching threshold of inverter (D) VDD

Q.19 Match the inferences X, Y, and Z, about a system, to the corresponding properties of the elements
of first column in Routh’s Table of the system characteristic equation.

X: The system is stable … P: … when all elements are positive


Y: The system is unstable … Q: … when any one element is zero
Z: The test breaks down … R: … when there is a change in sign of coefficients

(A) X→P, Y→Q, Z→R (B) X→Q, Y→P, Z→R


(C) X→R, Y→Q, Z→P (D) X→P, Y→R, Z→Q

Q.20 A closed-loop control system is stable if the Nyquist plot of the corresponding open-loop transfer
function
(A) encircles the s-plane point (−1 + j0) in the counterclockwise direction as many times as the
number of right-half s-plane poles.
(B) encircles the s-plane point (0 − j1) in the clockwise direction as many times as the number of
right-half s-plane poles.
(C) encircles the s-plane point (−1 + j0) in the counterclockwise direction as many times as the
number of left-half s-plane poles.
(D) encircles the s-plane point (−1 + j0) in the counterclockwise direction as many times as the
number of right-half s-plane zeros.

Q.21 Consider binary data transmission at a rate of 56 kbps using baseband binary pulse amplitude
modulation (PAM) that is designed to have a raised-cosine spectrum. The transmission bandwidth
(in kHz) required for a roll-off factor of 0.25 is ________

EC (1) 8/17
GATE 2016 Electronics and Communication Engineering (Set 1)

Q.22 A superheterodyne receiver operates in the frequency range of 58 MHz − 68 MHz. The
intermediate frequency 𝑓𝐼𝐹 and local oscillator frequency 𝑓𝐿𝑂 are chosen such that 𝑓𝐼𝐹 ≤ 𝑓𝐿𝑂 . It is
required that the image frequencies fall outside the 58 MHz − 68 MHz band. The minimum
required 𝑓𝐼𝐹 (in MHz) is ________

Q.23 The amplitude of a sinusoidal carrier is modulated by a single sinusoid to obtain the amplitude
modulated signal 𝑠(𝑡) = 5 cos 1600𝜋𝑡 + 20 cos 1800𝜋𝑡 + 5 cos 2000𝜋𝑡. The value of the
modulation index is __________

Q.24 Concentric spherical shells of radii 2 m, 4 m, and 8 m carry uniform surface charge densities of
20 nC/m2, −4 nC/m2 and ρs, respectively. The value of ρs (nC/m2) required to ensure that the
�⃗ at radius 10 m is _________
�⃗ = 0
electric flux density 𝐷

Q.25 The propagation constant of a lossy transmission line is (2 + 𝑗5) m−1 and its characteristic
impedance is (50 + 𝑗0) Ω at 𝜔 = 106 rad s−1. The values of the line constants L, C, R, G are,
respectively,
(A) L = 200 µH/m, C = 0.1 µF/m, R = 50 Ω/m, G = 0.02 S/m
(B) L = 250 µH/m, C = 0.1 µF/m, R = 100 Ω/m, G = 0.04 S/m
(C) L = 200 µH/m, C = 0.2 µF/m, R = 100 Ω/m, G = 0.02 S/m
(D) L = 250 µH/m, C = 0.2 µF/m, R = 50 Ω/m, G = 0.04 S/m

Q. 26 – Q. 55 carry two marks each.


1
Q.26 The integral ∬ (𝑥
2𝜋 𝐷
+ 𝑦 + 10)𝑑𝑥 𝑑𝑦, where 𝐷 denotes the disc: 𝑥 2 + 𝑦 2 ≤ 4, evaluates to_____

Q.27 A sequence x[n] is specified as

𝑥[𝑛] 1 1𝑛 1
� �= � � � �, for 𝑛 ≥ 2.
𝑥[𝑛 − 1] 1 0 0

The initial conditions are x[0] = 1, x[1] = 1, and x[n] = 0 for n < 0. The value of x[12] is ______

Q.28 In the following integral, the contour C encloses the points 2πj and −2πj
1 𝑠𝑖𝑛 𝑧
− � 𝑑𝑧
2𝜋 𝐶 (𝑧 − 2𝜋𝑗)3
The value of the integral is ________

π π
Q.29 The region specified by {( 𝜌, 𝜑, 𝑧): 3 ≤ 𝜌 ≤ 5, 8 ≤ 𝜑 ≤ 4 , 3 ≤ z ≤ 4.5} in cylindrical coordinates
has volume of _______

EC (1) 9/17
GATE 2016 Electronics and Communication Engineering (Set 1)

Q.30 The Laplace transform of the causal periodic square wave of period T shown in the figure below is

1 1
(A) 𝐹(𝑠) = (B) 𝐹(𝑠) = 𝑠𝑇
1+𝑒 −𝑠𝑇/2 −
𝑠�1+𝑒 2 �
1 1
(C) 𝐹(𝑠) = (D) 𝐹(𝑠) =
𝑠(1−𝑒 −𝑠𝑇 ) 1−𝑒 −𝑠𝑇

Q.31 A network consisting of a finite number of linear resistor (R), inductor (L), and capacitor (C)
elements, connected all in series or all in parallel, is excited with a source of the form
3

� 𝑎𝑘 cos(𝑘𝜔0 𝑡) , where 𝑎𝑘 ≠ 0, 𝜔0 ≠ 0.
𝑘=1

The source has nonzero impedance. Which one of the following is a possible form of the output
measured across a resistor in the network?
(A) (B)
3 4

� 𝑏𝑘 cos(𝑘𝜔0 𝑡 + 𝜙𝑘 ) , where 𝑏𝑘 ≠ 𝑎𝑘 , ∀ 𝑘 � 𝑏𝑘 cos(𝑘𝜔0 𝑡 + 𝜙𝑘 ) , where 𝑏𝑘 ≠ 0, ∀ 𝑘


𝑘=1 𝑘=1
(C) (D)
3 2

� 𝑎𝑘 cos(𝑘𝜔0 𝑡 + 𝜙𝑘 ) � 𝑎𝑘 cos(𝑘𝜔0 𝑡 + 𝜙𝑘 )
𝑘=1 𝑘=1

Q.32 A first-order low-pass filter of time constant T is excited with different input signals (with zero
initial conditions up to t = 0). Match the excitation signals X, Y, Z with the corresponding time
responses for t ≥ 0:

X: Impulse P: 1 − 𝑒 −𝑡/𝑇
Y: Unit step Q: t − T(1 − 𝑒 −𝑡/𝑇 )
Z: Ramp R: 𝑒 −𝑡/𝑇

(A) X→R, Y→Q, Z→P (B) X→Q, Y→P, Z→R


(C) X→R, Y→P, Z→Q (D) X→P, Y→R, Z→Q

EC (1) 10/17
GATE 2016 Electronics and Communication Engineering (Set 1)

Q.33 An AC voltage source V = 10 sin(t) volts is applied to the following network. Assume that R1 = 3
kΩ, R2 = 6 kΩ and R3 = 9 kΩ, and that the diode is ideal.

Irms ↑

RMS current Irms (in mA) through the diode is ________

Q.34 In the circuit shown in the figure,


the maximum power (in watt) delivered to the resistor R is __________

3 kΩ 10 kΩ
100 v0
5V 2 kΩ v0 40 kΩ R

Q.35 Consider the signal


𝑥[𝑛] = 6 𝛿[𝑛 + 2] + 3𝛿[𝑛 + 1] + 8𝛿[𝑛] + 7𝛿[𝑛 − 1] + 4𝛿[𝑛 − 2] .

If 𝑋(𝑒 𝑗𝜔 ) is the discrete-time Fourier transform of x[n] ,


1 𝜋
then 𝜋 ∫−𝜋 𝑋�𝑒 𝑗𝜔 � sin2(2𝜔) 𝑑𝜔 is equal to _________

EC (1) 11/17
GATE 2016 Electronics and Communication Engineering (Set 1)

Q.36 Consider a silicon p-n junction with a uniform acceptor doping concentration of 1017 cm−3 on the p-
side and a uniform donor doping concentration of 1016 cm−3 on the n-side. No external voltage is
applied to the diode. Given: kT/q = 26 mV, ni =1.5 ×1010 cm−3, εSi = 12ε0, ε0 = 8.85 × 10−14 F/m,
and q = 1.6 ×10−19 C.
The charge per unit junction area (nC cm−2) in the depletion region on the p-side is ___________

Q.37 Consider an n-channel metal oxide semiconductor field effect transistor (MOSFET) with a gate-to-
𝑊
source voltage of 1.8 V. Assume that 𝐿 = 4, 𝜇𝑁 𝐶𝑜𝑥 = 70 × 10−6 AV −2 , the threshold voltage is
0.3V, and the channel length modulation parameter is 0.09 V−1. In the saturation region, the drain
conductance (in micro seimens) is ________

Q.38 The figure below shows the doping distribution in a p-type semiconductor in log scale.

The magnitude of the electric field (in kV/cm) in the semiconductor due to non uniform doping
is _________

Q.39 Consider a silicon sample at T = 300 K, with a uniform donor density 𝑁𝑑 = 5 × 1016 cm−3,
illuminated uniformly such that the optical generation rate is 𝐺𝑜𝑝𝑡 = 1.5 × 1020 cm−3 𝑠 −1
throughout the sample. The incident radiation is turned off at 𝑡 = 0. Assume low-level injection to
be valid and ignore surface effects. The carrier lifetimes are 𝜏𝑝0 = 0.1 µs and 𝜏𝑛0 = 0.5 µs.

The hole concentration at 𝑡 = 0 and the hole concentration at 𝑡 = 0.3 µs, respectively, are
(A) 1.5 × 1013 cm−3 and 7.47 × 1011 cm−3
(B) 1.5 × 1013 cm−3 and 8.23 × 1011 cm−3
(C) 7.5 × 1013 cm−3 and 3.73 × 1011 cm−3
(D) 7.5 × 1013 cm−3 and 4.12 × 1011 cm−3

EC (1) 12/17
GATE 2016 Electronics and Communication Engineering (Set 1)

Q.40 An ideal opamp has voltage sources V1, V3, V5, …, VN-1 connected to the non-inverting input and V2,
V4, V6, …, VN connected to the inverting input as shown in the figure below (+VCC = 15 volt,
−VCC = −15 volt). The voltages V1, V2, V3, V4, V5, V6,… are 1, − 1/2, 1/3, −1/4, 1/5, −1/6, … volt,
respectively. As N approaches infinity, the output voltage (in volt) is ___________

Q.41 A p-i-n photodiode of responsivity 0.8A/W is connected to the inverting input of an ideal opamp as
shown in the figure, +Vcc = 15 V, −Vcc = −15V, Load resistor RL = 10 kΩ. If 10 µW of power is
incident on the photodiode, then the value of the photocurrent (in µA) through the load is ________

EC (1) 13/17
GATE 2016 Electronics and Communication Engineering (Set 1)

Q.42 Identify the circuit below.

(A) Binary to Gray code converter (B) Binary to XS3 converter


(C) Gray to Binary converter (D) XS3 to Binary converter

Q.43 The functionality implemented by the circuit below is

(A) 2-to-1 multiplexer (B) 4-to-1 multiplexer


(C) 7-to-1 multiplexer (D) 6-to-1 multiplexer

EC (1) 14/17
GATE 2016 Electronics and Communication Engineering (Set 1)

Q.44 In an 8085 system, a PUSH operation requires more clock cycles than a POP operation. Which one
of the following options is the correct reason for this?

(A) For POP, the data transceivers remain in the same direction as for instruction fetch (memory to
processor), whereas for PUSH their direction has to be reversed.

(B) Memory write operations are slower than memory read operations in an 8085 based system.

(C) The stack pointer needs to be pre-decremented before writing registers in a PUSH, whereas a
POP operation uses the address already in the stack pointer.

(D) Order of registers has to be interchanged for a PUSH operation, whereas POP uses their natural
order.

Q.45 The open-loop transfer function of a unity-feedback control system is


𝐾
𝐺(𝑠) = 𝑠2 +5𝑠+5

The value of K at the breakaway point of the feedback control system’s root-locus plot is ________

Q.46 The open-loop transfer function of a unity-feedback control system is given by

𝐾
𝐺(𝑠) =
𝑠(𝑠 + 2)

For the peak overshoot of the closed-loop system to a unit step input to be 10%, the value of K is
____________

Q.47 The transfer function of a linear time invariant system is given by

H(s) = 2𝑠 4 − 5𝑠 3 + 5𝑠 − 2

The number of zeros in the right half of the s-plane is ________

Q.48 Consider a discrete memoryless source with alphabet 𝑆 = {𝑠0 , 𝑠1 , 𝑠2 , 𝑠3 , 𝑠4 , … } and respective
1 1 1 1 1
probabilities of occurrence P = �2 , 4 , 8 , 16 , 32 , … �. The entropy of the source (in bits) is _______

Q.49 A digital communication system uses a repetition code for channel encoding/decoding. During
transmission, each bit is repeated three times (0 is transmitted as 000, and 1 is transmitted as 111).
It is assumed that the source puts out symbols independently and with equal probability. The
decoder operates as follows: In a block of three received bits, if the number of zeros exceeds the
number of ones, the decoder decides in favor of a 0, and if the number of ones exceeds the number
of zeros, the decoder decides in favor of a 1. Assuming a binary symmetric channel with
crossover probability p = 0.1, the average probability of error is ________

EC (1) 15/17
GATE 2016 Electronics and Communication Engineering (Set 1)

Q.50 An analog pulse s(t) is transmitted over an additive white Gaussian noise (AWGN) channel. The
received signal is r(t) = s(t) + n(t), where n(t) is additive white Gaussian noise with power spectral
𝑁
density 0 . The received signal is passed through a filter with impulse response h(t). Let 𝐸𝑠 and 𝐸ℎ
2
denote the energies of the pulse s(t) and the filter h(t), respectively. When the signal-to-noise ratio
(SNR) is maximized at the output of the filter (SNRmax), which of the following holds?
2𝐸𝑠 𝐸
(A) 𝐸𝑠 = 𝐸ℎ ; SNR max = 𝑁0
(B) 𝐸𝑠 = 𝐸ℎ ; SNR max = 2𝑁𝑠
0
2𝐸𝑠 2𝐸ℎ
(C) 𝐸𝑠 > 𝐸ℎ ; SNR max > 𝑁0
(D) 𝐸𝑠 < 𝐸ℎ ; SNR max = 𝑁0

Q.51 The current density in a medium is given by


400 𝑠𝑖𝑛𝜃
�𝐽⃗ = 𝑎� Am−2
2𝜋(𝑟 2 + 4) 𝑟
The total current and the average current density flowing through the portion of a spherical surface
𝜋 𝜋
r = 0.8 m,
12
≤ 𝜃 ≤ , 0 ≤ 𝜙 ≤ 2𝜋 are given, respectively, by
4

(A) 15.09 A, 12.86 Am-2 (B) 18.73 A, 13.65 Am-2


(C) 12.86 A, 9.23 Am-2 (D) 10.28 A, 7.56 Am-2

Q.52 An antenna pointing in a certain direction has a noise temperature of 50 K. The ambient
temperature is 290 K. The antenna is connected to a pre-amplifier that has a noise figure of 2 dB
and an available gain of 40 dB over an effective bandwidth of 12 MHz. The effective input noise
temperature Te for the amplifier and the noise power Pao at the output of the preamplifier,
respectively, are
(A) Te = 169.36 K and Pao = 3.73×10-10 W (B) Te = 170.8 K and Pao = 4.56×10-10 W
(C) Te = 182.5 K and Pao = 3.85×10-10 W (D) Te = 160.62 K and Pao = 4.6×10-10 W

Q.53 Two lossless X-band horn antennas are separated by a distance of 200λ. The amplitude reflection
coefficients at the terminals of the transmitting and receiving antennas are 0.15 and 0.18,
respectively. The maximum directivities of the transmitting and receiving antennas (over the
isotropic antenna) are 18 dB and 22 dB, respectively. Assuming that the input power in the lossless
transmission line connected to the antenna is 2 W, and that the antennas are perfectly aligned and
polarization matched, the power ( in mW) delivered to the load at the receiver is ________

EC (1) 16/17
GATE 2016 Electronics and Communication Engineering (Set 1)

Q.54 The electric field of a uniform plane wave travelling along the negative z direction is given by the
following equation:
𝐸�⃗𝑤𝑖 = �𝑎�𝑥 + 𝑗𝑎�𝑦 �𝐸0 𝑒 𝑗𝑘𝑧

This wave is incident upon a receiving antenna placed at the origin and whose radiated electric field
towards the incident wave is given by the following equation:

1
𝐸�⃗𝑎 = �𝑎�𝑥 + 2𝑎�𝑦 �𝐸𝐼 𝑒 −𝑗𝑘𝑟
𝑟

The polarization of the incident wave, the polarization of the antenna and losses due to the
polarization mismatch are, respectively,
(A) Linear, Circular (clockwise), −5dB (B) Circular (clockwise), Linear, −5dB
(C) Circular (clockwise), Linear, −3dB (D) Circular (anti clockwise), Linear, −3dB

Q.55 The far-zone power density radiated by a helical antenna is approximated as:
1
𝑊���⃗𝑟𝑎𝑑 = 𝑊
���⃗𝑎𝑣𝑒𝑟𝑎𝑔𝑒 ≈ 𝑎
�𝐶 4
𝑟 0 2 𝑐𝑜𝑠 θ
𝑟
The radiated power density is symmetrical with respect to φ and exists only in the upper
𝜋
hemisphere: 0 ≤ 𝜃 ≤ ; 0 ≤ 𝜙 ≤ 2𝜋; 𝐶0 is a constant. The power radiated by the antenna (in
2
watts) and the maximum directivity of the antenna, respectively, are
(A) 1.5C0 , 10dB (B) 1.256C0 , 10dB (C) 1.256C0 , 12dB (D) 1.5C0 , 12dB

END OF THE QUESTION PAPER

EC (1) 17/17
GATE 2016 General Aptitude - GA Set-3

Q. 1 – Q. 5 carry one mark each.


Q.1 Based on the given statements, select the appropriate option with respect to grammar and usage.

Statements
(i) The height of Mr. X is 6 feet.
(ii) The height of Mr. Y is 5 feet.

(A) Mr. X is longer than Mr. Y.

(B) Mr. X is more elongated than Mr. Y.

(C) Mr. X is taller than Mr. Y.

(D) Mr. X is lengthier than Mr. Y.

Q.2 The students ___________ the teacher on teachers’ day for twenty years of dedicated teaching.

(A) facilitated (B) felicitated (C) fantasized (D) facillitated

Q.3 After India’s cricket world cup victory in 1985, Shrotria who was playing both tennis and cricket
till then, decided to concentrate only on cricket. And the rest is history.

What does the underlined phrase mean in this context?


(A) history will rest in peace (B) rest is recorded in history books

(C) rest is well known (D) rest is archaic

Q.4 ½ ½
Given (9 inches) = (0.25 yards) , which one of the following statements is TRUE?

(A) 3 inches = 0.5 yards (B) 9 inches = 1.5 yards

(C) 9 inches = 0.25 yards (D) 81 inches = 0.0625 yards

Q.5 S, M, E and F are working in shifts in a team to finish a project. M works with twice the efficiency
of others but for half as many days as E worked. S and M have 6 hour shifts in a day, whereas E
and F have 12 hours shifts. What is the ratio of contribution of M to contribution of E in the
project?

(A) 1:1 (B) 1:2 (C) 1:4 (D) 2:1

1/3
GATE 2016 General Aptitude - GA Set-3

Q.10 A wire of length 340 mm is to be cut into two parts. One of the parts is to be made into a square and
the other into a rectangle where sides are in the ratio of 1:2. What is the length of the side of the
square (in mm) such that the combined area of the square and the rectangle is a MINIMUM?

(A) 30 (B) 40 (C) 120 (D) 180

END OF THE QUESTION PAPER

3/3
GATE 2016 Electronics and Communication Engineering (Set 2)

Q. 1 – Q. 25 carry one mark each.


Q.1
3 2 4
The value of 𝑥 for which the matrix 𝐴 = � 9 7 13 �
−6 −4 −9 + 𝑥
has zero as an eigenvalue is ________

Q.2 Consider the complex valued function 𝑓(𝑧) = 2𝑧 3 + 𝑏 |𝑧|3 where 𝑧 is a complex variable.
The value of 𝑏 for which the function 𝑓(𝑧) is analytic is ________

Q.3 As 𝑥 varies from −1 to +3, which one of the following describes the behaviour of the function
𝑓(𝑥) = 𝑥 3 – 3𝑥 2 + 1?
(A) 𝑓(𝑥) increases monotonically.
(B) 𝑓(𝑥) increases, then decreases and increases again.
(C) 𝑓(𝑥) decreases, then increases and decreases again.
(D) 𝑓(𝑥) increases and then decreases.

Q.4 How many distinct values of 𝑥 satisfy the equation sin(𝑥) = 𝑥/2, where 𝑥 is in radians?
(A) 1 (B) 2 (C) 3 (D) 4 or more

Q.5 Consider the time-varying vector 𝐈 = 𝐱� 15 cos(𝜔𝑡) + 𝐲� 5 sin(𝜔𝑡) in Cartesian coordinates, where
𝜔 > 0 is a constant. When the vector magnitude |𝐈| is at its minimum value, the angle 𝜃 that 𝐈
makes with the 𝑥 axis (in degrees, such that 0 ≤ 𝜃 ≤ 180) is ________

Q.6 In the circuit shown below, 𝑉𝑆 is a constant voltage source and 𝐼𝐿 is a constant current load.

The value of 𝐼𝐿 that maximizes the power absorbed by the constant current load is
𝑉𝑆 𝑉𝑆 𝑉𝑆 (D) ∞
(A) (B) (C)
4𝑅 2𝑅 𝑅

EC (2) 1/14
GATE 2016 Electronics and Communication Engineering (Set 2)

Q.7 The switch has been in position 1 for a long time and abruptly changes to position 2 at 𝑡 = 0.

If time 𝑡 is in seconds, the capacitor voltage VC (in volts) for 𝑡 > 0 is given by
(A) 4(1 − exp(−𝑡/0.5))
(B) 10 − 6 exp(−𝑡/0.5)
(C) 4(1 − exp(−𝑡/0.6))
(D) 10 − 6 exp(−𝑡/0.6)

Q.8 The figure shows an RLC circuit with a sinusoidal current source.

At resonance, the ratio |𝐈𝐋 |/|𝐈𝐑 |, i.e., the ratio of the magnitudes of the inductor current phasor and
the resistor current phasor, is ________

Q.9 The z-parameter matrix for the two-port network shown is

2𝑗𝜔 𝑗𝜔
� �,
𝑗𝜔 3 + 2𝑗𝜔

where the entries are in Ω. Suppose 𝑍𝑏 (𝑗𝜔) = 𝑅𝑏 + 𝑗𝜔 .

Then the value of 𝑅𝑏 (in Ω) equals ________

Q.10 sin(4𝜋𝑡)
The energy of the signal 𝑥 (𝑡 ) = is ________
4𝜋𝑡

EC (2) 2/14
GATE 2016 Electronics and Communication Engineering (Set 2)

Q.11 The Ebers-Moll model of a BJT is valid


(A) only in active mode
(B) only in active and saturation modes
(C) only in active and cut-off modes
(D) in active, saturation and cut-off modes

Q.12 A long-channel NMOS transistor is biased in the linear region with 𝑉𝐷𝑆 =50 mV and is used as a
resistance. Which one of the following statements is NOT correct?
(A) If the device width 𝑊 is increased, the resistance decreases.
(B) If the threshold voltage is reduced, the resistance decreases.
(C) If the device length 𝐿 is increased, the resistance increases.
(D) If 𝑉𝐺𝑆 is increased, the resistance increases.

Q.13 Assume that the diode in the figure has 𝑉𝑜𝑛 = 0.7 𝑉, but is otherwise ideal.

The magnitude of the current i2 (in mA) is equal to ________

Q.14 Resistor R1 in the circuit below has been adjusted so that I1 = 1 mA. The bipolar transistors Q1 and
Q2 are perfectly matched and have very high current gain, so their base currents are negligible. The
supply voltage Vcc is 6 V. The thermal voltage 𝑘𝑇/𝑞 is 26 mV.

The value of R 2 (in Ω) for which I2 =100 µA is ________

EC (2) 3/14
GATE 2016 Electronics and Communication Engineering (Set 2)

Q.15 Which one of the following statements is correct about an ac-coupled common-emitter amplifier
operating in the mid-band region?
(A) The device parasitic capacitances behave like open circuits, whereas coupling and bypass
capacitances behave like short circuits.
(B) The device parasitic capacitances, coupling capacitances and bypass capacitances behave like
open circuits.
(C) The device parasitic capacitances, coupling capacitances and bypass capacitances behave like
short circuits.
(D) The device parasitic capacitances behave like short circuits, whereas coupling and bypass
capacitances behave like open circuits.

Q.16 Transistor geometries in a CMOS inverter have been adjusted to meet the requirement for worst
case charge and discharge times for driving a load capacitor C. This design is to be converted to
that of a NOR circuit in the same technology, so that its worst case charge and discharge times
while driving the same capacitor are similar. The channel lengths of all transistors are to be kept
unchanged. Which one of the following statements is correct?

(A) Widths of PMOS transistors should be doubled, while widths of NMOS transistors should be
halved.
(B) Widths of PMOS transistors should be doubled, while widths of NMOS transistors should not
be changed.
(C) Widths of PMOS transistors should be halved, while widths of NMOS transistors should not be
changed.
(D) Widths of PMOS transistors should be unchanged, while widths of NMOS transistors should be
halved.

Q.17 Assume that all the digital gates in the circuit shown in the figure are ideal, the resistor 𝑅 = 10 𝑘Ω
and the supply voltage is 5 𝑉. The D flip-flops D1, D2, D3, D4 and D5 are initialized with logic
values 0,1,0,1 and 0, respectively. The clock has a 30% duty cycle.

The average power dissipated (in mW) in the resistor 𝑅 is ________

EC (2) 4/14
GATE 2016 Electronics and Communication Engineering (Set 2)

Q.18 A 4:1 multiplexer is to be used for generating the output carry of a full adder. A and B are the bits
to be added while 𝐶in is the input carry and 𝐶out is the output carry. A and B are to be used as the
select bits with A being the more significant select bit.

Which one of the following statements correctly describes the choice of signals to be connected to
the inputs I0 , I1 , I2 and I3 so that the output is Cout ?
(A) I0 =0, I1 =Cin, I2 =Cin and I3 =1
(B) I0 =1, I1 =Cin, I2 =Cin and I3 =1
(C) I0 =Cin, I1 =0, I2 =1 and I3 =Cin
(D) I0 =0, I1 =Cin, I2 =1 and I3 =Cin

Q.19 𝑠−2
The response of the system 𝐺(𝑠) = (𝑠+1)(𝑠+3) to the unit step input 𝑢(𝑡) is 𝑦(𝑡).
𝑑𝑦
The value of 𝑑𝑡
at 𝑡 = 0+ is _________

Q.20 The number and direction of encirclements around the point −1 + 𝑗0 in the complex plane by the
1−𝑠
Nyquist plot of 𝐺(𝑠) = 4+2𝑠 is

(A) zero. (B) one, anti-clockwise.


(C) one, clockwise. (D) two, clockwise.

Q.21 A discrete memoryless source has an alphabet {𝑎1 , 𝑎2 , 𝑎3 , 𝑎4 } with corresponding probabilities
1 1 1 1
�2 , 4 , 8 , 8 �. The minimum required average codeword length in bits to represent this source for
error-free reconstruction is ________

Q.22 A speech signal is sampled at 8 kHz and encoded into PCM format using 8 bits/sample. The PCM
data is transmitted through a baseband channel via 4-level PAM. The minimum bandwidth (in kHz)
required for transmission is ________

Q.23 A uniform and constant magnetic field 𝐁 = 𝐳�𝐵 exists in the 𝐳� direction in vacuum. A particle of
mass 𝑚 with a small charge 𝑞 is introduced into this region with an initial velocity 𝐯 = 𝐱�𝑣𝑥 + 𝐳�𝑣𝑧 .
Given that 𝐵, 𝑚, 𝑞, 𝑣𝑥 and 𝑣𝑧 are all non-zero, which one of the following describes the eventual
trajectory of the particle?
(A) Helical motion in the 𝐳� direction.
(B) Circular motion in the 𝑥𝑦 plane.
(C) Linear motion in the 𝐳� direction.
(D) Linear motion in the 𝐱� direction.

EC (2) 5/14
GATE 2016 Electronics and Communication Engineering (Set 2)

Q.24 Let the electric field vector of a plane electromagnetic wave propagating in a homogenous medium
be expressed as 𝐄 = 𝐱�𝐸𝑥 𝑒 −𝑗(𝜔𝑡−𝛽𝑧) , where the propagation constant 𝛽 is a function of the angular
frequency 𝜔. Assume that 𝛽(𝜔) and 𝐸𝑥 are known and are real. From the information available,
which one of the following CANNOT be determined?
(A) The type of polarization of the wave.
(B) The group velocity of the wave.
(C) The phase velocity of the wave.
(D) The power flux through the 𝑧 = 0 plane.

Q.25 Light from free space is incident at an angle 𝜃𝑖 to the normal of the facet of a step-index large core
optical fibre. The core and cladding refractive indices are 𝑛1 = 1.5 and 𝑛2 = 1.4, respectively.

The maximum value of 𝜃𝑖 (in degrees) for which the incident light will be guided in the core of the
fibre is ________

Q. 26 – Q. 55 carry two marks each.

Q.26 The ordinary differential equation


𝑑𝑥
𝑑𝑡
= −3 𝑥 + 2, with 𝑥(0) = 1
is to be solved using the forward Euler method. The largest time step that can be used to solve the
equation without making the numerical solution unstable is ________

Q.27 Suppose C is the closed curve defined as the circle 𝑥 2 + 𝑦 2 = 1 with C oriented anti-clockwise.
The value of ∮(𝑥𝑦 2 𝑑𝑥 + 𝑥 2 𝑦 𝑑𝑦) over the curve C equals ________

Q.28 Two random variables 𝑋 and 𝑌 are distributed according to

(𝑥 + 𝑦), 0 ≤ 𝑥 ≤ 1, 0 ≤ 𝑦 ≤ 1
𝑓𝑋,𝑌 (𝑥, 𝑦) = �
0, otherwise.

The probability 𝑃(𝑋 + 𝑌 ≤ 1) is ________

EC (2) 6/14
GATE 2016 Electronics and Communication Engineering (Set 2)

Q.29 a 0 3 7
2 5 1 3
The matrix A =  has det(𝐴) = 100 and trace(𝐴) = 14.
0 0 2 4
 
0 0 0 b

The value of |𝑎 − 𝑏| is ________

Q.30 In the given circuit, each resistor has a value equal to 1 Ω.

What is the equivalent resistance across the terminals 𝑎 and 𝑏?


(A) 1/6 Ω (B) 1/3 Ω (C) 9/20 Ω (D) 8/15 Ω

Q.31 In the circuit shown in the figure, the magnitude of the current (in amperes) through R 2 is ___

Q.32 2𝑠 + 6
A continuous-time filter with transfer function 𝐻(𝑠) = is converted to a discrete-
𝑠2 + 6𝑠 + 8
2𝑧 2 − 0.5032 𝑧
time filter with transfer function 𝐺(𝑧) = so that the impulse response of the
𝑧2 − 0.5032 𝑧 + 𝑘
continuous-time filter, sampled at 2 Hz, is identical at the sampling instants to the impulse response
of the discrete time filter. The value of 𝑘 is ________

EC (2) 7/14
GATE 2016 Electronics and Communication Engineering (Set 2)
Q.33 The Discrete Fourier Transform (DFT) of the 4-point sequence
𝑥[𝑛] = {𝑥[0], 𝑥[1], 𝑥[2], 𝑥[3]} = {3, 2, 3, 4} is
𝑋[𝑘] = {𝑋[0], 𝑋[1], 𝑋[2], 𝑋[3]} = {12, 2𝑗, 0, −2𝑗}.
If X1[k] is the DFT of the 12-point sequence 𝑥1 [𝑛] = {3, 0, 0, 2, 0, 0, 3, 0, 0, 4, 0, 0},
𝑋1 [8]
the value of � � is ________
𝑋1 [11]

Q.34 The switch S in the circuit shown has been closed for a long time. It is opened at time 𝑡 = 0 and
remains open after that. Assume that the diode has zero reverse current and zero forward voltage
drop.

The steady state magnitude of the capacitor voltage 𝑉C (in volts) is ______

Q.35 A voltage 𝑉𝐺 is applied across a MOS capacitor with metal gate and p-type silicon substrate at
T=300 K. The inversion carrier density (in number of carriers per unit area) for 𝑉𝐺 = 0.8 V is
2 × 1011 cm−2. For 𝑉𝐺 = 1.3 V, the inversion carrier density is 4 × 1011 cm−2 . What is the value
of the inversion carrier density for 𝑉𝐺 = 1.8 V?
(A) 4.5 × 1011 cm−2 (B) 6.0 × 1011 cm−2
(C) 7.2 × 1011 cm−2 (D) 8.4 × 1011 cm−2

Q.36 Consider avalanche breakdown in a silicon 𝑝+ 𝑛 junction. The 𝑛-region is uniformly doped with a
donor density 𝑁𝐷 . Assume that breakdown occurs when the magnitude of the electric field at any
point in the device becomes equal to the critical field 𝐸𝑐𝑟𝑖𝑡 . Assume 𝐸𝑐𝑟𝑖𝑡 to be independent of 𝑁𝐷 .
If the built-in voltage of the 𝑝+ 𝑛 junction is much smaller than the breakdown voltage, 𝑉𝐵𝑅 , the
relationship between 𝑉𝐵𝑅 and 𝑁𝐷 is given by

(A) 𝑉𝐵𝑅 × �𝑁𝐷 = constant (B) 𝑁𝐷 × �𝑉𝐵𝑅 = constant


(C) 𝑁𝐷 × 𝑉𝐵𝑅 = constant (D) 𝑁𝐷 /𝑉𝐵𝑅 = constant

EC (2) 8/14
GATE 2016 Electronics and Communication Engineering (Set 2)
Q.37 Consider a region of silicon devoid of electrons and holes, with an ionized donor density of
𝑁𝑑+ = 1017 cm−3 . The electric field at 𝑥 = 0 is 0 V/cm and the electric field at 𝑥 = 𝐿 is
50 kV/cm in the positive 𝑥 direction. Assume that the electric field is zero in the 𝑦 and 𝑧 directions
at all points.

Given 𝑞 = 1.6 × 10−19 coulomb, 𝜖0 = 8.85 × 10−14 F/cm, 𝜖𝑟 = 11.7 for silicon, the value of 𝐿 in
nm is ________

Q.38 Consider a long-channel NMOS transistor with source and body connected together. Assume that
the electron mobility is independent of 𝑉𝐺𝑆 and 𝑉𝐷𝑆 . Given,

𝑔𝑚 = 0.5 𝜇A/V for 𝑉𝐷𝑆 = 50 mV and 𝑉𝐺𝑆 = 2 V,


𝑔𝑑 = 8 𝜇A/V for 𝑉𝐺𝑆 = 2 V and 𝑉𝐷𝑆 = 0 V,

𝜕𝐼 𝜕𝐼
where 𝑔𝑚 = 𝜕𝑉 𝐷 and 𝑔𝑑 = 𝜕𝑉 𝐷
𝐺𝑆 𝐷𝑆

The threshold voltage (in volts) of the transistor is ________

Q.39 The figure shows a half-wave rectifier with a 475 𝜇F filter capacitor. The load draws a constant
current 𝐼𝑂 = 1 A from the rectifier. The figure also shows the input voltage 𝑉𝑖 , the output voltage
𝑉𝐶 and the peak-to-peak voltage ripple 𝑢 on 𝑉𝐶 . The input voltage 𝑉𝑖 is a triangle-wave with an
amplitude of 10 V and a period of 1 ms.

The value of the ripple 𝑢 (in volts) is ________

EC (2) 9/14
GATE 2016 Electronics and Communication Engineering (Set 2)
Q.40 In the opamp circuit shown, the Zener diodes 𝑍1 and 𝑍2 clamp the output voltage 𝑉𝑂 to +5 V or
−5 V. The switch S is initially closed and is opened at time 𝑡 = 0.

The time 𝑡 = 𝑡1 (in seconds) at which 𝑉𝑂 changes state is ________

Q.41 An opamp has a finite open loop voltage gain of 100. Its input offset voltage Vios (= +5mV) is
modeled as shown in the circuit below. The amplifier is ideal in all other respects. Vinput is 25 mV.

The output voltage (in millivolts) is ________

Q.42 An 8 Kbyte ROM with an active low Chip Select input (CS ���) is to be used in an 8085
microprocessor based system. The ROM should occupy the address range 1000H to 2FFFH. The
address lines are designated as 𝐴15 to 𝐴0 , where 𝐴15 is the most significant address bit.
Which one of the following logic expressions will generate the correct ���CS signal for this ROM?
(A) 𝐴15 + 𝐴14 + (𝐴13 ⋅ 𝐴12 + 𝐴 ����� �����
13 ⋅ 𝐴12 )
(B) 𝐴15 ⋅ 𝐴14 ⋅ (𝐴13 + 𝐴12 )
(C) �����
𝐴 ����� ����� �����
15 ⋅ 𝐴14 ⋅ (𝐴13 ⋅ 𝐴12 + 𝐴13 ⋅ 𝐴12 )
(D) �����
𝐴 �����
15 + 𝐴14 + 𝐴13 ⋅ 𝐴12

EC (2) 10/14
GATE 2016 Electronics and Communication Engineering (Set 2)
Q.43 In an N bit flash ADC, the analog voltage is fed simultaneously to 2𝑁 − 1 comparators. The output
of the comparators is then encoded to a binary format using digital circuits. Assume that the analog
voltage source Vin (whose output is being converted to digital format) has a source resistance of
75 Ω as shown in the circuit diagram below and the input capacitance of each comparator is 8 pF.
The input must settle to an accuracy of 1/2 LSB even for a full scale input change for proper
conversion. Assume that the time taken by the thermometer to binary encoder is negligible.

If the flash ADC has 8 bit resolution, which one of the following alternatives is closest to the
maximum sampling rate ?
(A) 1 megasamples per second
(B) 6 megasamples per second
(C) 64 megasamples per second
(D) 256 megasamples per second

Q.44 The state transition diagram for a finite state machine with states A, B and C, and binary inputs X,
Y and Z, is shown in the figure.

Which one of the following statements is correct?


(A) Transitions from State A are ambiguously defined.
(B) Transitions from State B are ambiguously defined.
(C) Transitions from State C are ambiguously defined.
(D) All of the state transitions are defined unambiguously.

EC (2) 11/14
GATE 2016 Electronics and Communication Engineering (Set 2)
Q.45 1
In the feedback system shown below 𝐺(𝑠) = .
(𝑠 2 +2𝑠)
The step response of the closed-loop system should have minimum settling time and have no
overshoot.

The required value of gain 𝑘 to achieve this is ________

Q.46 1
In the feedback system shown below 𝐺(𝑠) = (𝑠+1)(𝑠+2)(𝑠+3) .

The positive value of 𝑘 for which the gain margin of the loop is exactly 0 dB and the phase margin
of the loop is exactly zero degree is ________

Q.47 𝑘
The asymptotic Bode phase plot of 𝐺(𝑠) = (𝑠+0.1)(𝑠+10)(𝑠+𝑝 , with 𝑘 and 𝑝1 both positive,
1)
is shown below.

The value of 𝑝1 is ________

Q.48 An information source generates a binary sequence {𝛼𝑛 }. 𝛼𝑛 can take one of the two possible
values −1 and +1 with equal probability and are statistically independent and identically
distributed. This sequence is precoded to obtain another sequence {𝛽𝑛 }, as 𝛽𝑛 = 𝛼𝑛 + 𝑘 𝛼𝑛−3 .
The sequence {𝛽𝑛 } is used to modulate a pulse 𝑔(𝑡) to generate the baseband signal
1, 0≤𝑡≤𝑇
𝑋(𝑡) = ∑∞ 𝑛=−∞ 𝛽𝑛 𝑔(𝑡 − 𝑛𝑇), where 𝑔(𝑡) = � 0, otherwise.
1
If there is a null at 𝑓 = 3𝑇 in the power spectral density of 𝑋(𝑡), then 𝑘 is ________

EC (2) 12/14
GATE 2016 Electronics and Communication Engineering (Set 2)
Q.49 An ideal band-pass channel 500 Hz - 2000 Hz is deployed for communication. A modem is
designed to transmit bits at the rate of 4800 bits/s using 16-QAM. The roll-off factor of a pulse with
a raised cosine spectrum that utilizes the entire frequency band is ________

Q.50 Consider a random process 𝑋(𝑡) = 3𝑉(𝑡) − 8, where 𝑉(𝑡) is a zero mean stationary random
process with autocorrelation 𝑅𝑣 (𝜏) = 4𝑒 −5|𝜏| . The power in 𝑋(𝑡) is ________

Q.51 A binary communication system makes use of the symbols “zero” and “one”. There are channel
errors. Consider the following events:
𝑥0 : a "zero" is transmitted
𝑥1 : a "one" is transmitted
𝑦0 : a "zero" is received
𝑦1 : a "one" is received
1 3 1
The following probabilities are given: 𝑃(𝑥0 ) = , 𝑃(𝑦0 |𝑥0 ) = , and 𝑃(𝑦0 |𝑥1 ) = . The
2 4 2
information in bits that you obtain when you learn which symbol has been received (while you
know that a “zero” has been transmitted) is ________

Q.52 The parallel-plate capacitor shown in the figure has movable plates. The capacitor is charged so that
the energy stored in it is 𝐸 when the plate separation is 𝑑. The capacitor is then isolated electrically
and the plates are moved such that the plate separation becomes 2𝑑.

At this new plate separation, what is the energy stored in the capacitor, neglecting fringing effects?
(A) 2𝐸 (B) √2𝐸 (C) 𝐸 (D) 𝐸 ⁄2

Q.53 A lossless microstrip transmission line consists of a trace of width 𝑤. It is drawn over a practically
infinite ground plane and is separated by a dielectric slab of thickness 𝑡 and relative permittivity
𝜀𝑟 > 1. The inductance per unit length and the characteristic impedance of this line are 𝐿 and 𝑍0 ,
respectively.

Which one of the following inequalities is always satisfied?

𝐿𝑡 𝐿𝑡
(A) 𝑍0 > �𝜀 (B) 𝑍0 < �𝜀
0 𝜀𝑟 𝑤 0 𝜀𝑟 𝑤

𝐿𝑤 𝐿𝑤
(C) 𝑍0 > �𝜀 (D) 𝑍0 < �𝜀
0 𝜀𝑟 𝑡 0 𝜀𝑟 𝑡

EC (2) 13/14
GATE 2016 Electronics and Communication Engineering (Set 2)
Q.54 A microwave circuit consisting of lossless transmission lines T1 and T2 is shown in the figure.
The plot shows the magnitude of the input reflection coefficient Γ as a function of frequency 𝑓.
The phase velocity of the signal in the transmission lines is 2 × 108 m/s.

The length 𝐿 (in meters) of T2 is ________

Q.55 A positive charge 𝑞 is placed at 𝑥 = 0 between two infinite metal plates placed at 𝑥 = −𝑑 and at
𝑥 = +𝑑 respectively. The metal plates lie in the 𝑦𝑧 plane.

The charge is at rest at 𝑡 = 0, when a voltage +𝑉 is applied to the plate at – 𝑑 and voltage −𝑉 is
applied to the plate at 𝑥 = +𝑑. Assume that the quantity of the charge 𝑞 is small enough that it does
not perturb the field set up by the metal plates. The time that the charge 𝑞 takes to reach the right
plate is proportional to
(A) 𝑑 / 𝑉 (B) √𝑑 / 𝑉 (C) 𝑑 / √𝑉 (D) �𝑑/𝑉

END OF THE QUESTION PAPER

EC (2) 14/14
GATE 2016 General Aptitude - GA Set-4

Q. 1 – Q. 5 carry one mark each.


Q.1 An apple costs Rs. 10. An onion costs Rs. 8.

Select the most suitable sentence with respect to grammar and usage.
(A) The price of an apple is greater than an onion.

(B) The price of an apple is more than onion.

(C) The price of an apple is greater than that of an onion.

(D) Apples are more costlier than onions.

Q.2 The Buddha said, “Holding on to anger is like grasping a hot coal with the intent of throwing it at
someone else; you are the one who gets burnt.”

Select the word below which is closest in meaning to the word underlined above.

(A) burning (B) igniting (C) clutching (D) flinging

Q.3 M has a son Q and a daughter R. He has no other children. E is the mother of P and daughter-in-
law of M. How is P related to M?

(A) P is the son-in-law of M. (B) P is the grandchild of M.

(C) P is the daughter-in law of M. (D) P is the grandfather of M.

Q.4 The number that least fits this set: (324, 441, 97 and 64) is ________.

(A) 324 (B) 441 (C) 97 (D) 64

Q.5 It takes 10 s and 15 s, respectively, for two trains travelling at different constant speeds to
completely pass a telegraph post. The length of the first train is 120 m and that of the second train is
150 m. The magnitude of the difference in the speeds of the two trains (in m/s) is ____________.

(A) 2.0 (B) 10.0 (C) 12.0 (D) 22.0

1/3
GATE 2016 General Ap
ptitude - GA Set-44

Q. 6 – Q. 10 carry tw
wo marks each.
Q.6 Thee velocity V of a vehiclle along a sttraight line is
i measured in m/s and plotted as shown
s with
resppect to timee in seconds. At the endd of the 7 seconds,
s howw much willl the odomeeter reading
incrrease by (in m)?

(A)) 0 (B) 3 (C) 4 (D) 5

Q.7 Thee overwhelm ming numberr of people innfected with h rabies in Inndia has beenn flagged by
y the World
Heaalth Organizzation as a soource of conncern. It is esstimated thatt inoculatingg 70% of petts and stray
doggs against rabbies can leadd to a significcant reductio
on in the num
mber of peoplle infected with
w rabies.

Whhich of the foollowing can be logicallyy inferred from


m the above sentences?
(A)) The numbber of people in India infeected with raabies is high.

(B)) The number of people in other partts of the world who are innfected with rabies is low
w.

(C)) Rabies cann be eradicated in India by


b vaccinatin
ng 70% of strray dogs.

(D)) Stray dogss are the main source of rabies


r worldw
wide.

Q.8 f is shared by four firstt year underggraduate stud


A flat dents. They agreed
a to alloow the oldestt of them to
enjoy some exttra space in the flat. Maanu is two months
m olderr than Sravann, who is thhree months
youunger than Trrideep. Pavaan is one monnth older than
n Sravan. Who
W should occcupy the ex xtra space in
the flat?

(A)) Manu (B) Sravan (C) Trideep (D) Pavan

Q.9 Finnd the area boounded by thhe lines 3x+22y=14, 2x-3y


y=5 in the firrst quadrant.
(A)) 14.95 (B) 15.25 (C) 15.70 (D) 20.35

2/33
GATE 2016 General Aptitude - GA Set-4

Q.10 A straight line is fit to a data set (ln x, y). This line intercepts the abscissa at ln x = 0.1 and has a
slope of −0.02. What is the value of y at x = 5 from the fit?

(A) −0.030 (B) −0.014 (C) 0.014 (D) 0.030

END OF THE QUESTION PAPER

3/3
GATE 2016 Electronics and Communication Engineering (Set 3)

Q. 1 – Q. 25 carry one mark each.


Q.1 Consider a 2 × 2 square matrix
σ x 
A= ,
ω σ 
where x is unknown. If the eigenvalues of the matrix A are (σ + jω ) and (σ − jω ) , then x is
equal to

(A) + jω (B) − jω (C) +ω (D) −ω

Q.2 sin ( z )
For f ( z ) = , the residue of the pole at z = 0 is __________
z2

Q.3 The probability of getting a “head” in a single toss of a biased coin is 0.3. The coin is tossed
repeatedly till a “head” is obtained. If the tosses are independent, then the probability of getting
“head” for the first time in the fifth toss is __________

Q.4 1
dx
The integral ∫
0 (1 − x)
is equal to __________

Q.5 Consider the first order initial value problem

y' = y + 2x − x2, y(0) = 1, (0 ≤ x < ∞)

with exact solution y(x) = x2 + ex. For x = 0.1, the percentage difference between the exact solution
and the solution obtained using a single iteration of the second-order Runge-Kutta method with
step-size h = 0.1is __________

Q.6 Consider the signal 𝑥(𝑡) = cos(6𝜋𝑡) + sin(8𝜋𝑡), where 𝑡 is in seconds. The Nyquist sampling
rate (in samples/second) for the signal 𝑦(𝑡) = 𝑥(2𝑡 + 5) is

(A) 8 (B) 12 (C) 16 (D) 32

sin(𝑡) sin(𝑡)
Q.7 If the signal 𝑥(𝑡) = ∗ with ∗ denoting the convolution operation, then x(t) is equal to
𝜋𝑡 𝜋𝑡

sin(𝑡) sin(2𝑡) 2 sin(𝑡) sin(𝑡) 2


(A)
𝜋𝑡
(B)
2𝜋𝑡
(C)
𝜋𝑡 (D) � �
𝜋𝑡

EC (3) 1/19
GATE 2016 Electronics and Communication Engineering (Set 3)

Q.8 A discrete-time signal 𝑥[𝑛] = 𝛿[𝑛 − 3] + 2 𝛿[𝑛 − 5] has z-transform X(z). If 𝑌(𝑧) = 𝑋(−𝑧) is the
z-transform of another signal y[n], then

(A) 𝑦[𝑛] = 𝑥[𝑛] (B) 𝑦[𝑛] = 𝑥[−𝑛]


(C) 𝑦[𝑛] = −𝑥[𝑛] (D) 𝑦[𝑛] = −𝑥[−𝑛]

Q.9 In the RLC circuit shown in the figure, the input voltage is given by

𝑣𝑖 (𝑡) = 2 cos(200𝑡) + 4 sin(500𝑡).


The output voltage 𝑣𝑜 (𝑡) is

0.25 H 100 µF

+ +
2Ω
0.4 H 10 µF

vi(t) vo(t)
2Ω

- -

(A) cos(200𝑡) + 2 sin(500𝑡) (B) 2cos(200𝑡) + 4 sin(500𝑡)


(C) sin(200𝑡) + 2 cos(500𝑡) (D) 2sin(200𝑡) + 4 cos(500𝑡)

Q.10 The I-V characteristics of three types of diodes at the room temperature, made of semiconductors
X, Y and Z, are shown in the figure. Assume that the diodes are uniformly doped and identical in
all respects except their materials. If EgX, EgY and EgZ are the band gaps of X, Y and Z, respectively,
then

X Y Z

(A) EgX > EgY > EgZ (B) EgX = EgY = EgZ
(C) EgX < EgY < EgZ (D) no relationship among these band gaps exists.

EC (3) 2/19
GATE 2016 Electronics and Communication Engineering (Set 3)
Q.11 The figure shows the band diagram of a Metal Oxide Semiconductor (MOS). The surface region of
this MOS is in

SiO2
EM
ΦB EC
EFS
ΦB
Ei

EV

(A) inversion (B) accumulation (C) depletion (D) flat band

Q.12 The figure shows the I-V characteristics of a solar cell illuminated uniformly with solar light of
power 100 mW/cm2. The solar cell has an area of 3 cm2 and a fill factor of 0.7. The maximum
efficiency (in %) of the device is __________

I
ISC = 180mA

(0, 0) V
VOC=0.5V

EC (3) 3/19
GATE 2016 Electronics and Communication Engineering (Set 3)
Q.13 The diodes D1 and D2 in the figure are ideal and the capacitors are identical. The product RC is
very large compared to the time period of the ac voltage. Assuming that the diodes do not
breakdown in the reverse bias, the output voltage VO (in volt) at the steady state is __________

D1

C +
10sinωt
R Vo

ac -
C

D2

Q.14 Consider the circuit shown in the figure. Assuming VBE1 = VEB2 = 0.7 volt, the value of the dc
voltage VC2 (in volt) is __________

VCC = 2.5V

β1 =100
Q2
Q1
β2 = 50
10 kΩ
VC2
1V
1 kΩ

EC (3) 4/19
GATE 2016 Electronics and Communication Engineering (Set 3)
Q.15 In the astable multivibrator circuit shown in the figure, the frequency of oscillation (in kHz) at the
output pin 3 is __________

Vcc

RA = 2.2 kΩ 8 4
Vcc Res
7
Disch

RB = 4.7 kΩ
555 Timer
6 3
Thresh Out

2
Trig

Gnd
C = 0.022 µF 1

Q.16 In an 8085 microprocessor, the contents of the accumulator and the carry flag are A7 (in hex) and 0,
respectively. If the instruction RLC is executed, then the contents of the accumulator (in hex) and
the carry flag, respectively, will be

(A) 4E and 0 (B) 4E and 1 (C) 4F and 0 (D) 4F and 1

Q.17 The logic functionality realized by the circuit shown below is

(A) OR (B) XOR (C) NAND (D) AND

Q.18 The minimum number of 2-input NAND gates required to implement a 2-input XOR gate is

(A) 4 (B) 5 (C) 6 (D) 7

EC (3) 5/19
GATE 2016 Electronics and Communication Engineering (Set 3)
Q.19 The block diagram of a feedback control system is shown in the figure. The overall closed-loop
gain G of the system is

+ +
X G2 G1 Y
− −
H1

G1G2 G1G2
(A) G = (B) G =
1 + G1 H1 1 + G1G2 + G1 H1
G1G2 G1G2
(C) G = (D) G =
1 + G1G2 H1 1 + G1G2 + G1G2 H1

Q.20 For the unity feedback control system shown in the figure, the open-loop transfer function G(s) is
given as
2
G (s) = .
s ( s + 1)

The steady state error ess due to a unit step input is

+ e(t)
x(t) G(s) y(t)

(A) 0 (B) 0.5 (C) 1.0 (D) ∞

Q.21 For a superheterodyne receiver, the intermediate frequency is 15 MHz and the local oscillator
frequency is 3.5 GHz. If the frequency of the received signal is greater than the local oscillator
frequency, then the image frequency (in MHz) is __________

Q.22 An analog baseband signal, bandlimited to 100 Hz, is sampled at the Nyquist rate. The samples are
quantized into four message symbols that occur independently with probabilities p1 = p4 = 0.125
and p2 = p3. The information rate (bits/sec) of the message source is __________

EC (3) 6/19
GATE 2016 Electronics and Communication Engineering (Set 3)
Q.23 A binary baseband digital communication system employs the signal

 1
 , 0 ≤ t ≤ TS
p ( t ) =  TS
0,
 otherwise

for transmission of bits. The graphical representation of the matched filter output y(t) for this signal
will be

(A) (B)
y(t) y(t)

1/Ts 0.5

t t
0 Ts 2Ts 0 Ts 2Ts
(C) (D)
y(t) y(t)

1 1

t t
0 Ts 2Ts 0 Ts/2 Ts

Q.24 If a right-handed circularly polarized wave is incident normally on a plane perfect conductor, then
the reflected wave will be

(A) right-handed circularly polarized


(B) left-handed circularly polarized
(C) elliptically polarized with a tilt angle of 450
(D) horizontally polarized

Q.25 Faraday’s law of electromagnetic induction is mathematically described by which one of the
following equations?
 
(A) ∇ • B = 0 (B) ∇ • D = ρV
 
 ∂B   ∂D
(C) ∇ × E = − (D) ∇ × H = σE +
∂t ∂t

EC (3) 7/19
GATE 2016 Electronics and Communication Engineering (Set 3)

Q. 26 – Q. 55 carry two marks each.

Q.26 The particular solution of the initial value problem given below is

d2y dy dy
2
+ 12 + 36 y =
0 with y (0) =
3 and =
−36
dx dx dx x =0

(A) (3 – 18x) e−6x (B) (3 + 25x) e−6x


(C) (3 + 20x) e−6x (D) (3 − 12x) e−6x

Q.27 If the vectors e1 = (1, 0, 2), e2 = (0, 1, 0) and e3 = (−2, 0, 1) form an orthogonal basis of the three-
dimensional real space ℝ3 , then the vector u = (4, 3,−3) ∈ ℝ3 can be expressed as

2 11 2 11
(A) u = − e1 − 3e 2 − e3 (B) u = − e1 − 3e 2 + e3
5 5 5 5
2 11 2 11
(C) u = − e1 + 3e 2 + e3 (D) u = − e1 + 3e 2 − e3
5 5 5 5

Q.28 A triangle in the xy-plane is bounded by the straight lines 2x = 3y, y = 0 and x = 3. The volume
above the triangle and under the plane x + y + z = 6 is __________

Q.29 1 ez
2π j ∫c
The values of the integral dz along a closed contour c in anti-clockwise direction for
z−2

(i) the point z0 = 2 inside the contour c, and


(ii) the point z0 = 2 outside the contour c,

respectively, are

(A) (i) 2.72, (ii) 0 (B) (i) 7.39, (ii) 0

(C) (i) 0, (ii) 2.72 (D) (i) 0, (ii) 7.39

EC (3) 8/19
GATE 2016 Electronics and Communication Engineering (Set 3)
Q.30 2𝜋
A signal 2 cos � 3 𝑡� − cos (𝜋𝑡) is the input to an LTI system with the transfer function

𝐻(𝑠) = 𝑒 𝑠 + 𝑒 −𝑠 .
If 𝐶𝑘 denotes the kth coefficient in the exponential Fourier series of the output signal, then 𝐶3 is
equal to

(A) 0 (B) 1 (C) 2 (D) 3

Q.31 The ROC (region of convergence) of the z-transform of a discrete-time signal is represented by the
=
shaded region in the z-plane. If the signal x[n] ( 2.0 ) n , −∞ < n < +∞ , then the ROC of its
z-transform is represented by

(A) (B)
Im Im

Unit circle z-plane Unit circle z-plane

×0.5 ×2 Re ×0.5 ×2 Re

(C) (D)
Im Im

Unit circle z-plane Unit circle z-plane

×0.5 ×2 Re ×0.5 ×2 Re

(ROC does not exist)

EC (3) 9/19
GATE 2016 Electronics and Communication Engineering (Set 3)
Q.32 Assume that the circuit in the figure has reached the steady state before time 𝑡 = 0 when the 3 Ω
resistor suddenly burns out, resulting in an open circuit. The current 𝑖(𝑡) (in ampere) at 𝑡 = 0+ is
__________

3F 2Ω
1Ω

i(t) 2Ω
12 V

2F 3Ω

Q.33 In the figure shown, the current 𝑖 (in ampere) is __________

1A 5Ω
8V
1Ω

1Ω 1Ω
8V

i
1Ω

EC (3) 10/19
GATE 2016 Electronics and Communication Engineering (Set 3)

Q.34  z11 z12 


The z-parameter matrix   for the two-port network shown is
 z 21 z 22 

3Ω

input port output port

6Ω

2 − 2  2 2 9 − 3 9 3
(A)   (B)   (C)   (D)  
− 2 2  2 2 6 9  6 9

Q.35 A continuous-time speech signal xa(t) is sampled at a rate of 8 kHz and the samples are
subsequently grouped in blocks, each of size N. The DFT of each block is to be computed in real
time using the radix-2 decimation-in-frequency FFT algorithm. If the processor performs all
operations sequentially, and takes 20 µs for computing each complex multiplication (including
multiplications by 1 and −1) and the time required for addition/subtraction is negligible, then the
maximum value of N is __________

Q.36 The direct form structure of an FIR (finite impulse response) filter is shown in the figure.

Unit Unit
x[n]
Delay Delay

5 5


y[n]
+
The filter can be used to approximate a

(A) low-pass filter (B) high-pass filter


(C) band-pass filter (D) band-stop filter

EC (3) 11/19
GATE 2016 Electronics and Communication Engineering (Set 3)
Q.37 The injected excess electron concentration profile in the base region of an npn BJT, biased in the
active region, is linear, as shown in the figure. If the area of the emitter-base junction is 0.001 cm2,
µn = 800 cm2/(V-s) in the base region and depletion layer widths are negligible, then the collector
current IC (in mA) at room temperature is __________

(Given: thermal voltage VT = 26 mV at room temperature, electronic charge q = 1.6 × 10−19 C)

IB

n p n
1014 cm-3
Excess electron
IE profile
IC

0
0.5µm

Q.38 Figures I and II show two MOS capacitors of unit area. The capacitor in Figure I has insulator
materials X (of thickness t1 = 1 nm and dielectric constant ε1 = 4) and Y (of thickness t2 = 3 nm and
dielectric constant ε2 = 20). The capacitor in Figure II has only insulator material X of thickness tEq.
If the capacitors are of equal capacitance, then the value of tEq (in nm) is __________

Metal Metal
t2 ε2
t1 ε1 ε1 tEq

SiSi SiSi

Figure I Figure II

EC (3) 12/19
GATE 2016 Electronics and Communication Engineering (Set 3)
Q.39 The I-V characteristics of the zener diodes D1 and D2 are shown in Figure I. These diodes are used
in the circuit given in Figure II. If the supply voltage is varied from 0 to 100 V, then breakdown
occurs in

-80V
-70V D1
V

0-100V D2
D1
D2

Figure I Figure II

(A) D1 only (B) D2 only


(C) both D1and D2 (D) none of D1 and D2

Q.40 For the circuit shown in the figure, R1 = R2 = R3 = 1 Ω, L = 1 µH and C = 1 µF. If the input
( )
Vin = cos 106 t , then the overall voltage gain (Vout /Vin) of the circuit is __________

R1
R3
L

-
- Vout
+ R2
C
+
Vin

EC (3) 13/19
GATE 2016 Electronics and Communication Engineering (Set 3)
Q.41 In the circuit shown in the figure, the channel length modulation of all transistors is non-zero
(λ ≠ 0). Also, all transistors operate in saturation and have negligible body effect. The ac small
signal voltage gain (Vo/Vin) of the circuit is

VDD

M3 M2

VG

vo

vin M1

(A) −𝑔𝑚1 (𝑟𝑜1 ||𝑟𝑜2 || 𝑟𝑜3 ) 1


(B) −𝑔𝑚1 (𝑟𝑜1 || || 𝑟𝑜3 )
𝑔𝑚3

1 1
(C) −𝑔𝑚1 �𝑟𝑜1 || �𝑔 ||𝑟𝑜2 � || 𝑟𝑜3 � (D) −𝑔𝑚1 (𝑟𝑜1 || �𝑔 ||𝑟𝑜3 � || 𝑟𝑜2 )
𝑚2 𝑚3

Q.42 In the circuit shown in the figure, transistor M1 is in saturation and has transconductance
gm = 0.01 siemens. Ignoring internal parasitic capacitances and assuming the channel length
modulation λ to be zero, the small signal input pole frequency (in kHz) is __________

VDD

1kΩ

vo

50pF
vin
M1
5kΩ

EC (3) 14/19
GATE 2016 Electronics and Communication Engineering (Set 3)

Q.43 Following is the K-map of a Boolean function of five variables P, Q, R, S and X. The minimum
sum-of-product (SOP) expression for the function is

PQ PQ
00 01 11 10 00 01 11 10
RS RS

00 0 0 0 0 00 0 1 1 0

01 1 0 0 1 01 0 0 0 0

11 1 0 0 1 11 0 0 0 0

10 0 0 0 0 10 0 1 1 0

X=0 X=1

(A) P Q S X + P Q S X + Q R S X + Q R S X

(B) Q S X + Q S X

(C) Q S X + Q S X

(D) Q S + Q S

Q.44 For the circuit shown in the figure, the delays of NOR gates, multiplexers and inverters are 2 ns,
1.5 ns and 1 ns, respectively. If all the inputs P, Q, R, S and T are applied at the same time instant,
the maximum propagation delay (in ns) of the circuit is __________

Q
0 0

R MUX MUX Y

S 1 1
S0 S0

EC (3) 15/19
GATE 2016 Electronics and Communication Engineering (Set 3)

Q.45 For the circuit shown in the figure, the delay of the bubbled NAND gate is 2 ns and that of the
counter is assumed to be zero.

Q0 (LSB)

3-bit Q1
Synchronous
Counter
Q2 (MSB)

Clk

RESET

If the clock (Clk) frequency is 1 GHz, then the counter behaves as a

(A) mod-5 counter (B) mod-6 counter (C) mod-7 counter (D) mod-8 counter

Q.46 The first two rows in the Routh table for the characteristic equation of a certain closed-loop control
system are given as

s3 1 (2K + 3)

s2 2K 4

The range of K for which the system is stable is

(A) −2.0 < K < 0.5 (B) 0 < K < 0.5 (C) 0 < K < ∞ (D) 0.5 < K < ∞

Q.47 A second-order linear time-invariant system is described by the following state equations

x1 (t ) + 2 x1 (t ) = 3u (t )
d
dt

x2 (t ) + x2 (t ) = u (t )
d
dt
where x1(t) and x2(t) are the two state variables and u(t) denotes the input. If the output c(t) = x1(t),
then the system is

(A) controllable but not observable


(B) observable but not controllable
(C) both controllable and observable
(D) neither controllable nor observable

EC (3) 16/19
GATE 2016 Electronics and Communication Engineering (Set 3)
Q.48 The forward-path transfer function and the feedback-path transfer function of a single loop negative
feedback control system are given as

K ( s + 2)
G ( s) = and H ( s ) = 1 ,
s 2 + 2s + 2

respectively. If the variable parameter K is real positive, then the location of the breakaway point on
the root locus diagram of the system is __________

Q.49 A wide sense stationary random process 𝑋(𝑡) passes through the LTI system shown in the figure. If
the autocorrelation function of 𝑋(𝑡) is 𝑅𝑋 (𝜏), then the autocorrelation function 𝑅𝑌 (𝜏) of the output
𝑌(𝑡) is equal to

X(t) Y(t)

Delay = T0

(A) 2𝑅𝑋 (𝜏) + 𝑅𝑋 (𝜏 − 𝑇0 ) + 𝑅𝑋 (𝜏 + 𝑇0 ) (B) 2𝑅𝑋 (𝜏) − 𝑅𝑋 (𝜏 − 𝑇0 ) − 𝑅𝑋 (𝜏 + 𝑇0 )

(C) 2𝑅𝑋 (𝜏) + 2𝑅𝑋 (𝜏 − 2𝑇0 ) (D) 2𝑅𝑋 (𝜏) − 2𝑅𝑋 (𝜏 − 2𝑇0 )

Q.50 A voice-grade AWGN (additive white Gaussian noise) telephone channel has a bandwidth of
η
4.0 kHz and two-sided noise power spectral density = 2.5 ×10 −5 Watt per Hz. If information at
2
the rate of 52 kbps is to be transmitted over this channel with arbitrarily small bit error rate, then the
minimum bit-energy Eb (in mJ/bit) necessary is __________

Q.51 The bit error probability of a memoryless binary symmetric channel is 10−5. If 105 bits are sent over
this channel, then the probability that not more than one bit will be in error is __________

Q.52 Consider an air-filled rectangular waveguide with dimensions a = 2.286 cm and b = 1.016 cm. At
10 GHz operating frequency, the value of the propagation constant (per meter) of the corresponding
propagating mode is __________

EC (3) 17/19
GATE 2016 Electronics and Communication Engineering (Set 3)

Q.53 Consider an air-filled rectangular waveguide with dimensions a = 2.286 cm and b = 1.016 cm. The
increasing order of the cut-off frequencies for different modes is

(A) TE01 < TE10 < TE11 < TE20 (B) TE20 < TE11 < TE10 < TE01

(C) TE10 < TE20 < TE01 < TE11 (D) TE10 < TE11 < TE20 < TE01

Q.54 A radar operating at 5 GHz uses a common antenna for transmission and reception. The antenna has
a gain of 150 and is aligned for maximum directional radiation and reception to a target 1 km away
having radar cross-section of 3 m2. If it transmits 100 kW, then the received power (in µW) is
__________

EC (3) 18/19
GATE 2016 Electronics and Communication Engineering (Set 3)

Q.55 Consider the charge profile shown in the figure. The resultant potential distribution is best
described by

(B)
(A)

(C) (D)

END OF THE QUESTION PAPER

EC (3) 19/19
|EC| GATE-2017-PAPER-I www.gateforum.com

Electronics and Communication Engineering


Q. No. 1 to 25 Carry One Mark Each

1. The clock frequency of an 8085 microprocessor is 5 MHz. If the time required to execute an
instruction is 1.4 s, then the number of T-states needed for executing the instruction is
(A) 1 (B) 6 (C) 7 (D) 8
Key: (C)
Exp: fclock  5MHz; Tclock  0.2 106 sec
Texecution  1.4s
1.4
No.of T  state required  7
0.2

2. Consider a single input single output discrete-time system with x[n] as input and y[n] as
output, where the two are related as

 n x n  for 0  n  10
yn  
 x  n   x  n  1
 otherwise
Which one of the following statements is true about the system?
(A) It is causal and stable (B) It is causal but not stable
(C) It is not causal but stable (D) It is neither causal nor stable
Key: (A)
Exp: For an input-output relation if the present output depends on present and past input values then
the given system is “Causal”.
For the given relation,

 n x n 0  n  10
yn  
 x  n   x  n  1 otherwise

For n ranging from 0 to 10 present output depends on present input only.
At all other points present output depends on present and past input values.
Thus the system is “Causal”.
Stability
If x[n] is bounded for the given finite range of n i.e. 0  n  10 y  n  is also bounded.
Similarly x  n   x  n  1 is also bounded at all other values of n
Thus the system is “stable”.

3. Consider the following statement about the linear dependence of the real valued functions
y1  1, y2  x and y3  x 2 , over the field of real numbers.
I. y1 , y2 and y3 are linearly independent on  1  x  0
II. y1 , y2 and y3 are linearly dependent on 0  x  1
III. y1 , y2 and y3 are linearly independent on 0  x  1
IV. y1 , y2 and y3 are linearly dependent on  1  x  0
Which one among the following is correct?

 ICP–Intensive Classroom Program  eGATE-Live Internet Based Classes DLP  TarGATE-All India Test Series
Leaders in GATE Preparations  65+ Centers across India
© All rights reserved by Gateforum Educational Services Pvt. Ltd. No part of this booklet may be reproduced or utilized in any form without the written permission.

1
|EC| GATE-2017-PAPER-I www.gateforum.com

(A) Both I and II are true (B) Both I and III are true
(C) Both II and IV are true (D) Both III and IV are true
Key: (B)
Exp: y1  1, y2  x, y3  x 2
y1 y 2 y3 1 x x 2
1 x
Consider y1 y2 y3  0 1 2x  2 20
0 1
y1 y2 y3 0 0 2
 y1 , y2 , y3 are linearly independent  x

4. Consider the 5 × 5 matrix


1 2 3 4 5
5 1 2 3 4 

A  4 5 1 2 3
 
3 4 5 1 2
 2 3 4 5 1 
It is given that A has only one real eigen value. Then the real eigen value of A is
(A) 2.5 (B) 0 (C) 15 (D) 25
Key: (C)
1 2 3 4 5
5 1 2 3 4 

Exp: A  4 5 1 2 3
 
3 4 5 1 2
 2 3 4 5 1 

For eigen values    , A  I  0

1  2 3 4 5
5 1  2 3 4
 4 5 1  2 3 0
3 4 5 1  2
2 3 4 5 1 
R1  R1  R 2  R 3  R 4  R 5
15   15   15   15   15  
5 1  2 3 4
 4 5 1  2 3 0
3 4 5 1  2
2 3 4 5 1 

 ICP–Intensive Classroom Program  eGATE-Live Internet Based Classes DLP  TarGATE-All India Test Series
Leaders in GATE Preparations  65+ Centers across India
© All rights reserved by Gateforum Educational Services Pvt. Ltd. No part of this booklet may be reproduced or utilized in any form without the written permission.

2
|EC| GATE-2017-PAPER-I www.gateforum.com

1 1 1 1 1
5 1  2 3 4
 15    4 5 1  2 3 0
3 4 5 1  2
2 3 4 5 1 
 15    0
   15

5. The voltage of an electromagnetic wave propagating in a coaxial cable with uniform


characteristic impedance is V     e    jt volts, Where  is the distance along the length of
the cable in meters.    0.1  j40  m 1 is the complex propagation constant, and
  2 109 rad s is the angular frequency. The absolute value of the attenuation in the cable
in dB/meter is __________.
Key: (0.85 to 0.88)
Exp: Given    0.1  j40  m 1
p
Here   0.1
m
p 
WE know that, 1  8.686 dB  0.1 p  0.8686 dB
m m m m

6. A bar of Gallium Arsenide (GaAs) is doped with Silicon such that the Silicon atoms occupy
Gallium and Arsenic sites in the GaAs crystal. Which one of the following statement is true?
(A) Silicon atoms act as p-type dopants in Arsenic sites and n-type dopants in Gallium sites
(B) Silicon atoms act as n-type dopants in Arsenic sites and p-type dopants in Gallium sites
(C) Silicon atoms act as p-type dopants in Arsenic as well as Gallium sites
(D) Silicon atoms act as n-type dopants in Arsenic as well as Gallium sites
Key: (A)
Exp: Silicon atoms act as P- type dopants in Arsenic sites and n- type dopants in Gallium sites.

5 10 10 
7. The rank of the matrix M  1 0 2  is
3 6 6 
(A) 0 (B) 1 (C) 2 (D) 3
Key: (C)
5 10 10
Exp: M  1 0 2  5  0  12   10  6  6   10  6  0   60  0  60  0
3 6 6

5 10
But a 2 × 2 minor,  0  10  10  0  Rank  2
1 0

 ICP–Intensive Classroom Program  eGATE-Live Internet Based Classes DLP  TarGATE-All India Test Series
Leaders in GATE Preparations  65+ Centers across India
© All rights reserved by Gateforum Educational Services Pvt. Ltd. No part of this booklet may be reproduced or utilized in any form without the written permission.

3
|EC| GATE-2017-PAPER-I www.gateforum.com

8. For a narrow base PNP BJT, the excess minority carrier concentration  n E for emitter,
p B for base. n C for collector  normalized to equilibrium minority carrier concentration
 n E0for emitter, p B0 for base, n C0for collector)
in the quasi-neutral emitter, base and collector
regions are shown below. Which one of the following biasing modes is the transistor operating in?

105

Carrier Concentration
Normalized excess pB
pB0
n C
n C0

0 n
E

n E0
Emitter  P  Base  N  Collector  P 
X and Y axes are not to scale

(A) Forward active (B) Saturation (C) Inverse active (D) Cutoff
Key: (C)
Exp: As per the change carrier profile, base – to – emitter junction is reverse bias and base to
collector junction is forward bias, so it works in Inverse active.

9. The Miller effect in the context of a Common Emitter amplifier explains


(A) an increase in the low-frequency cutoff frequency
(B) an increase in the high-frequency cutoff frequency
(C) a decrease in the low-frequency cutoff frequency
(D) a decrease in the high-frequency cutoff frequency
Key: (D)
Exp: Miller effect increase input capacitance, so that there will be decrease in gain in the high
frequency cutoff frequency.

10. Consider the D-Latch shown in the figure, which is transparent when its clock input CK is
high and has zero propagation delay. In the figure, the clock signal CLK1 has a 50% duty
cycle and CLK2 is a one-fifth period delayed version of CLK1. The duty cycle at the output
latch in percentage is ___________.

TCIK
CLK1 D Q Output
CLK1 D-Latch
CLK2 CK

CLK2
TCIK 5

Key: (29.9 to 30.1)

 ICP–Intensive Classroom Program  eGATE-Live Internet Based Classes DLP  TarGATE-All India Test Series
Leaders in GATE Preparations  65+ Centers across India
© All rights reserved by Gateforum Educational Services Pvt. Ltd. No part of this booklet may be reproduced or utilized in any form without the written permission.

4
|EC| GATE-2017-PAPER-I www.gateforum.com

TCLK
Exp: TCLK
2
CLK  2

CLK  1

t CLK
5

TCLK TCLK

 Dutyde of O / p  2 5  100  30%
TCLK

11. Which of the following can be pole-zero configuration of a phase-lag controller (lag
compensator)? j
j
(A) (B) Pole
Pole
s-plane s-plane
Zero Zero

 

(C) j (D) j
Pole Pole
s-plane s-plane
Zero Zero

 

Key: (A)
Exp: In phase lag compensator pole is near to j axis,

j

12. In the latch circuit shown, the NAND gates have non-zero, but unequal propagation delays.
The present input condition is: P = Q = „0‟. If the input condition is changed simultaneously to
P = Q = „1‟, the outputs X and Y are

 ICP–Intensive Classroom Program  eGATE-Live Internet Based Classes DLP  TarGATE-All India Test Series
Leaders in GATE Preparations  65+ Centers across India
© All rights reserved by Gateforum Educational Services Pvt. Ltd. No part of this booklet may be reproduced or utilized in any form without the written permission.

5
|EC| GATE-2017-PAPER-I www.gateforum.com

P
(A) X = „1‟, Y = „1‟ X
(B) either X = „1‟, Y = „0‟ or X = „0‟, Y = „1‟
(C) either X = „1‟, Y = „1‟ or X = „0‟, Y = „0‟
(D) X = „0‟, Y = „0‟
Y
Q

Key: (B) gate 1


Exp: Unequal propagation delay P  0 x 1

Q0 y 1
gate  2 
Case I: Case II:
Gate 1  2ns Gate 1  1nsec
Gate 2  1ns Gate 2  2nsec
2ns 1n sec
P 1 x  1 P 1 x0

Q 1 y0 Q 1 y 1
1ns 2n sec
 Either x = 1, y = 0 or x = 0, y =1

13. Three fair cubical dice are thrown simultaneously. The probability that all three dice have the
same number of dots on the faces showing up is (up to third decimal place) __________.
Key: (0.027 to 0.028)
1 1 1 1
Exp: Required probability  6       0.028
 6 6 6  36

14. A periodic signal x(t) has a trigonometric Fourier series expansion



x  t   a 0    a n cos n0 t  bn sin n0 t .
n 1

If x  t    x   t    x  t   0  , we can conclude that


(A) an are zero for all n and bn are zero for n even
(B) an are zero for all n and bn are zero for n odd
(C) an are zero for n even and bn are zero for n odd
(D) an are zero for n odd and bn are zero for n even
Key: (A)
Exp: If x  t    x   t  the given periodic signal is odd symmetric. For an odd symmetric signal
a n  0 for all n.

 ICP–Intensive Classroom Program  eGATE-Live Internet Based Classes DLP  TarGATE-All India Test Series
Leaders in GATE Preparations  65+ Centers across India
© All rights reserved by Gateforum Educational Services Pvt. Ltd. No part of this booklet may be reproduced or utilized in any form without the written permission.

6
|EC| GATE-2017-PAPER-I www.gateforum.com

    To
If x  t    x  t   ,   where T0 is fundamental period then the given
 0  0 2
condition satisfies half-wave symmetry.
For half-wave symmetrical signal all coefficients an and bn are zero for even value of n.

15. The open loop transfer function G  s  


 s  1
s p
 s  2  s  3
Where p is an integer, is connected in unity feedback configuration as shown in figure.

G s

Given that the steady state error is zero for unit step input and is 6 for unit ramp input, the
value of the parameter p is _________.
Key: (0.99 to 1.01)
s 1
Exp: G s 
s  s  2  s  3
p

If p  1, ess  for ramp input   6


1
kv 
6
p 1 ess  for step input   0
1
k p  , ess  0
1  kp

16. An n   n Silicon device is fabricated with uniform and non-degenerate donor doping
concentrations of N D1  11018 cm3 and ND2  11015 cm3 corresponding to the n  and n
regions respectively. At the operational temperature T, assume complete impurity ionization,
3
kT/q = 25 mV, and intrinsic carrier concentration to be n i  110 cm . What is the
10

magnitude of the built-in potential of this device?


(A) 0.748V (B) 0.460V (C) 0.288V (D) 0.173V
Key: (D)
N 
Exp: Vbi  VT n  1 
 N2 
 1018 
 0.25n  15   0.173V.
 10 

17. For the operational amplifier circuit shown, the output saturation voltages are 15V. The
upper and lower threshold voltages for the circuit are, respectively.

 ICP–Intensive Classroom Program  eGATE-Live Internet Based Classes DLP  TarGATE-All India Test Series
Leaders in GATE Preparations  65+ Centers across India
© All rights reserved by Gateforum Educational Services Pvt. Ltd. No part of this booklet may be reproduced or utilized in any form without the written permission.

7
|EC| GATE-2017-PAPER-I www.gateforum.com

 
Vin Vout

 10 k

5 k
 3V

(A) 5V and  5V (B) 7V and  3V


(C) 3V and  7V (D) 3V and 3V

Key: (B)

Vsat  15V Vsat  15V


Exp: Given Vsat  15V,  Vsat  15V
10 k
15  3  5  3  12  3  7V
10 k
VUTP  VUTP VLTP
15 3 5 k
5 k

VLTP 
 15  3  5  3  18  3  6  3  3V  3V  3V
 
15 3

18. In the circuit shown, the positive angular frequency  (in radians per second) at which

magnitude of the phase difference between the voltages V1 and V2 equals radians, is
4
__________. V2

1 1H
100 cos t ~ 1 V1

Key: (0.9 to 1.1)


 1  V2
Exp: V1     10
 2  j 
100
V1    tan 1  / 2 1 j
4 2
100cos t 1 V1
1  j 1000
V2  1000
2  j
 1  2  
V2     1  tan 1  tan 1 
 4  2  2 V2
 
V2  V1   / 4
 
 tan 1  tan 1   tan 1   / 4
2 2 I
  tan  / 4  1 rad / sec V1

 ICP–Intensive Classroom Program  eGATE-Live Internet Based Classes DLP  TarGATE-All India Test Series
Leaders in GATE Preparations  65+ Centers across India
© All rights reserved by Gateforum Educational Services Pvt. Ltd. No part of this booklet may be reproduced or utilized in any form without the written permission.

8
|EC| GATE-2017-PAPER-I www.gateforum.com

19. In a digital communication system, the overall pulse shape p(t) at the receiver before the
sampler has the Fourier transform P(f). If the symbols are transmitted at the rate of 2000
symbols per second, for which of the following cases is inter symbol interference zero?
(A) P f  (B) P f 

1 1

f  kHz  1.2 0.8 f  kHz 


1.2 0 1.2 0 0.8 1.2

(C) P f  (D) P f 

1
1

f  kHz  f  kHz 
1.2  1 0 1 1.2 1.2 0 1.2
Key: (B)
Exp: For ISI free pulse,
If P(t) is having spectrum P(f)

Then  P  f  kR   constant
k 
S

R S  2 KSpa
Thin condition is met by pulse given in option B.

20. Consider a stable system with transfer function


sp  b1sp1    bp
G s 
sq  a1sq 1    a q
Where b1 ,, b p and a1 , a q are real valued constants. The slope of the Bode log magnitude
curve of G(s) converges to 60 dB decade as   . A possible pair of values for p and q is
(A) p  0 and q  3 (B) p  1and q  7
(C) p  2 and q  3 (D) p  3 and q  5
Key: (A)
1  1  b1 s    b ps 
1 p

Exp: G s   
sq  p 1  q1 s 1    a q s  q 
q p
If s  s , when p  0 and q  3, then
3

It have 60dB d ec at   

21. A good transconductance amplifier should have


(A) high input resistance and low output resistance
(B) low input resistance and high output resistance

 ICP–Intensive Classroom Program  eGATE-Live Internet Based Classes DLP  TarGATE-All India Test Series
Leaders in GATE Preparations  65+ Centers across India
© All rights reserved by Gateforum Educational Services Pvt. Ltd. No part of this booklet may be reproduced or utilized in any form without the written permission.

9
|EC| GATE-2017-PAPER-I www.gateforum.com

(C) high input and output resistances


(D) low input and output resistance
Key: (C)
Exp: A good trans conductance amplifier should have high input and output resistance.

22. Let  X1 , X 2  be independent random variables. X1 has mean 0 and variance 1, while X2 has
mean 1 and variance 4. The mutual information I  X1 ;X 2  between X1 and X2 in bits is
_________.
Key: (0.0 to 0.0)
Exp: For two independent random variable
I  X;Y   H  X   H  X Y 
H  X Y   H  X  for independent X and Y
 I  X;Y   0

23. Consider the following statements for continuous-time linear time invariant (LTI) systems.
I. There is no bounded input bounded output (BIBO) stable system with a pole in the right
half of the complex plane.
II. There is non causal and BIBO stable system with a pole in the right half of the complex
plane.
Which one among the following is correct?
(A) Both I and II are true (B) Both I and II are not true
(C) Only I is true (D) Only II is true
Key: (D)
Exp: If a system is non-causal then a pole on right half of the s-plane can give BIBO stable system.
But for a causal system to be BIBO all poles must lie on left half of the complex plane.

24. Which one of the following statements about differential pulse code modulation (DPCM) is true?
(A) The sum of message signal sample with its prediction is quantized
(B) The message signal sample is directly quantized, and its prediction is not used
(C) The difference of message signal sample and a random signal is quantized
(D) The difference of message signal sample with its predictions is quantized
Key: (D) x n  en eq  n 
Exp: DPCM Block diagram Quantized

e q [n] is quantized e  n 
x̂  n  
e  n  is difference of message
signal sample with its prediction. Prediction

25. Consider a wireless communication link between a transmitter and a receiver located in free
space, with finite and strictly positive capacity. If the effective areas of the transmitter and the

 ICP–Intensive Classroom Program  eGATE-Live Internet Based Classes DLP  TarGATE-All India Test Series
Leaders in GATE Preparations  65+ Centers across India
© All rights reserved by Gateforum Educational Services Pvt. Ltd. No part of this booklet may be reproduced or utilized in any form without the written permission.

10
|EC| GATE-2017-PAPER-I www.gateforum.com

receiver antennas, and the distance between them are all doubled, and everything else remains
unchanged, the maximum capacity of the wireless link
(A) increases by a factor of 2 (B) decrease by a factor 2
(C) remains unchanged (D) decreases by a factor of 2
Key: (C)
Exp:
 S 
C  Blog 2 1  
 N0B 
PG A
where S  t t 2 er
4r
P A . 4 
S1  t er2 2 A e t
4r 
PA A t 4A er .A e t
 t 2 er 2 e  Pt
 r  4. 2 r 2
 Pt .A er .A e t
S
A2r2
Channelcpacity remain same.

Q. No. 26 to 55 Carry Two Marks Each

26. Starting with x = 1, the solution of the equation x 3  x  1, after two iterations of Newton-
Raphson‟s method (up to two decimal places) is _________.
Key: (0.65 to 0.72)
Exp: Let f  x   x 3  x  1  f   x   3x 2  1
Given x o  1
By Newton Raphson method,
f  xo  f 1 1 3
1st iteration, x1  x o  1  1    0.75
f  xo  f  1 4 4

f  x1  f  0.75  0.17
2nd iteration, x 2  x1   0.75   0.75   0.69
f   x1  f   0.75  2.69

27. In binary frequency shift keying (FSK), the given signal waveform are
u 0  t   5cos  20000t  ;0  t  T, and
u1  t   5cos  22000t  ; 0  t  T,
Where T is the bit-duration interval and t is in seconds. Both u 0  t  and u1  t  are zero outside
the interval 0  t  T. With a matched filter (correlator) based receiver, the smallest positive
value of T (in milliseconds) required to have u 0  t  and u1  t  uncorrelated is
(A) 0.25 ms (B) 0.5 ms (C) 0.75 ms (D) 1.0 ms
Key: (B)

 ICP–Intensive Classroom Program  eGATE-Live Internet Based Classes DLP  TarGATE-All India Test Series
Leaders in GATE Preparations  65+ Centers across India
© All rights reserved by Gateforum Educational Services Pvt. Ltd. No part of this booklet may be reproduced or utilized in any form without the written permission.

11
|EC| GATE-2017-PAPER-I www.gateforum.com

Exp: u o  t   5cos  20000t 


f o  10 kHz
u1  t   5cos  22000t 
f1  11 kHz
For u o  t  and u1  t  to be orthogonal, it is necessary that
n 1
; 11  10   103 
f1  f o 
2T 2T
1
T  0.5 msec
2  103

28. For the DC analysis of the Common-Emitter amplifier shown, neglect the base current and
assume that the emitter and collector current are equal. Given that VT  25mV, VBE  0.7V,
and the BJT output resistance r0 is practically infinite. Under these conditions, the midband
voltage gain magnitude. Ac  Vo Vi V V, is _________.
VCC  12V

RC 2 k
73 k 10 F
R1 
10 F C2

RL Vo
C1 2 k
8 k
V1 ~ 47 k R2 RE
CE
100 F

Key: (127.0 to 129.0)
Vo R c
Exp: AV  
Vi re VCC  12V
VT
re  RC 2 k
IE 73 k R1
12  47
VG   4.7V C2
120 VG
VG  VEE  I E R E
4.7  0.7
IE   2mA
2  103 47 k R2 RE 2 k
25
re   12.5
2
R || R L 2  103 || 8  103
AV  c   128
re 12.5

29. The figure shows an RLC circuit exited by the sinusoidal voltage 100cos  3t  volts, where t is
amplitude of V2
in seconds. The ratio is _________.
amplitude of V1

 ICP–Intensive Classroom Program  eGATE-Live Internet Based Classes DLP  TarGATE-All India Test Series
Leaders in GATE Preparations  65+ Centers across India
© All rights reserved by Gateforum Educational Services Pvt. Ltd. No part of this booklet may be reproduced or utilized in any form without the written permission.

12
|EC| GATE-2017-PAPER-I www.gateforum.com

V1

4 1H
5
100 cos t ~ V2
1
F
36

Key: (2.55 to 2.65)


 4  j3   4  j3 
Exp: V1     1000  V1     1000
 4  j3  5  12 j   9  9j 
 5  12 j   5  12 j 
V2     1000  V2     1000
 4  j3  5  12 j   9  9j 
V2 5  12 j 52  122 13
    2.6
V1 4  j3 42  32 5

30. Which one of the following is the general solution of the first order differential equation
dy
  x  y  1 , where x, y are real?
2

dx
(A) y  1  x  tan 1  x  c  , where c is a constant
(B) y  1  x  tan  x  c  , where c is a constant

(C) y  1  x  tan 1  x  c  , where c is a constant


(D) y  1  x  tan  x  c  , where c is a constant
Key: (D)
dy
  x  y  1
2
Exp: ...(1)
dx
Put x  y  1  t
dy dt
1 
dx dx
dy dt
  1
dx dx
dt
From (1), 1  t2
dx
dt
  1 t2
dx
1
 dt   dx
1  t2
 tan 1  t   x  C
 tan 1  x  y  1  x  C

 ICP–Intensive Classroom Program  eGATE-Live Internet Based Classes DLP  TarGATE-All India Test Series
Leaders in GATE Preparations  65+ Centers across India
© All rights reserved by Gateforum Educational Services Pvt. Ltd. No part of this booklet may be reproduced or utilized in any form without the written permission.

13
|EC| GATE-2017-PAPER-I www.gateforum.com

 x  y  1  tan  x  C 
 y  1  x  tan  x  C 

31. A linear time invariant (LTI) system with the transfer function
K  s 2  2s  2 
G s 
s 2
 3s  2 
is connected in unity feedback configuration as shown in the figure.

G s

For the closed loop system shown, the root locus for 0  K   intersects the imaginary axis
for K = 1.5. The closed loop system is stable for
(A) K  1.5 (B) 1  K  1.5 (D) 0  K  1 (D) no positive value of K
Key: (A)
k  s 2  2s  2 
Exp: Given G  s  
s 2
 3s  2 

C.E  1  G  s   s 2  3s  2  ks 2  2ks  2k  0
 s2 1  k   s  2k  s   2k  2  0
If closed loop system to be stable all coefficients to positive
k  1  k  1.5  k  1
So, k  1.5

32. Let I    2z dx  2y dy  2x dz  where x, y, z are real, and let C be the straight line segment
C

from point A :  0, 2,1 to point B :  4,1, 1 . The value of I is _________.


Key: (-11.1 to -10.9)
Exp: The straight line joining A(0, 2, 1) and B(4, 1, -1) is
x  0 y  2 z 1
 
4  0 1  2 1  1
x y  2 z 1
    t  say 
4 1 2
 x  4t, y  2  t, z  1  2t
 dx  4dt, dy  dt, dz  2dt
For x  0  t  0
For x  4  t  1
I    2zdx  2ydy  2xdz 
C
1
  2 1  2t  4dt  2  2  t  dt   2  4t  2dt 
t 0

 ICP–Intensive Classroom Program  eGATE-Live Internet Based Classes DLP  TarGATE-All India Test Series
Leaders in GATE Preparations  65+ Centers across India
© All rights reserved by Gateforum Educational Services Pvt. Ltd. No part of this booklet may be reproduced or utilized in any form without the written permission.

14
|EC| GATE-2017-PAPER-I www.gateforum.com

1
30t 2
1
    30t  4  dt   4t  11
t 0
2 0

33. As shown, two Silicon (Si) abrupt p-n junction diodes are fabricated with uniform donor
doping concentration of N D1  1014 cm 3 and N D2  1016 cm3 in the n-regions of the diodes,
and uniform acceptor doping concentration of N A1  1014 cm3 and N A2  1016 cm3 in the p-
regions of the diodes, respectively. Assuming that the reverse bias voltage is >> built-in
potentials of the diodes, the ratio C2 C1 of their reverse bias capacitances for the same
applied reverse bias, is __________.
p n p n

1014 1014 1016 1016


cm3 cm3 cm3 cm3

C1 C2
Diode 1 Diode 2

Key: (10.0 to 10.0)


A
Exp: C
W
1 1
C and W 
W doping
C  doping
C2  doping 2 1016
   100  10
C1  doping 1 1014

34. An optical fiber is kept along the ẑ direction. The refractive indices for the electric fields
along x̂ and ŷ directions in the fiber are n x  1.5000 and n y  1.5001, respectively (n x  n y
due to the imperfection in the fiber cross-section). The free space wavelength of a light wave
propagating in the fiber is 1.5m. If the light wave is circularly polarized at the input of the
fiber, the minimum propagation distance after which it becomes linearly polarized, in
centimeter, is ___________.
Key: (0.36 to 0.38)
Exp: For circular polarization the phase difference between E x &E y is  / 2
 The phase difference for linear polarization should be 
So the wave must travel a minimum distance such that the extra phase difference of  / 2
must occur.
 y  min  x  min   / 2
 2  min
  min  n y  n x    / 2   n y  n x    / 2
c o 
o 1.5 106 1.5
  min     102  0.375  102 m  0.375cm
4  n y  n x  40.0001 4

 ICP–Intensive Classroom Program  eGATE-Live Internet Based Classes DLP  TarGATE-All India Test Series
Leaders in GATE Preparations  65+ Centers across India
© All rights reserved by Gateforum Educational Services Pvt. Ltd. No part of this booklet may be reproduced or utilized in any form without the written permission.

15
|EC| GATE-2017-PAPER-I www.gateforum.com

35. Two discrete-time signals x[n] and h[n] are both non-zero for n = 0, 1, 2 and are zero
otherwise. It is given that
x  0  1, x 1  2, x  2  1, h 0  1.
Let y[n] be the linear convolution of x[n] and h[n]. Given that y[1] = 3 and y[2] = 4, the value
of the expression 10y 3  y  4 is _________.
Key: (31.00 to 31.00)
Exp: Given
x  n   1, 2,1;  h 0  1
h  n   1,a, b
1 2 1
1 1 2 1
a a 2a a
b b 2b b
y  n   1, 2  a, 2a  b  1, 2b  a, b

It is given that y 1  3


 2  a  3 a 1
Similarly 2a  b  1  4  b  3  2 1  1
b 1
 y 3  2 1  1  3

y  4  b  1

10y 3  y  4  30  1  31

36. Which one of the following options correctly describes the locations of the roots of the
equation s4  s2  1  0 on the complex plane?
(A) Four left half plane (LHP) roots
(B) One right half plane (RHP) root, one LHP root and two roots on the imaginary axis
(C) Two RHP roots and two LHP roots
(D) All four roots are on the imaginary axis
Key: (C)
Exp: F  s   s4  s2  1  0

Let take s2  t
t2  t 1  0
1  i 3
t
2
1  j 3
Where t  s2 
2
j2   j2 
1  j 3 1  j 3
s2   e 3  s2  e 3
2 2

 ICP–Intensive Classroom Program  eGATE-Live Internet Based Classes DLP  TarGATE-All India Test Series
Leaders in GATE Preparations  65+ Centers across India
© All rights reserved by Gateforum Educational Services Pvt. Ltd. No part of this booklet may be reproduced or utilized in any form without the written permission.

16
|EC| GATE-2017-PAPER-I www.gateforum.com

j2   j2 
s  e 6
and s  e 6

Hence two roots contain RHS and two roots contain LHS plane.

37. The dependence of drift velocity of electrons on electric field in a semiconductor is shown
below. The semiconductor has a uniform electron concentration of n  11016 cm3 and
electronic charge q  1.6 1019 C. If a bias of 5V is applied across a 1 m region of this
semiconductor, the resulting current density in this region, in kA/cm2, is _________.
Drift velocity  cm s 
constant
107

linear

0
5 105 Electric field  V cm 

Key (1.5 to 1.7)


Exp:  d  n 
d 107 cm 2
n   20
 5  105 V  sec
V 5
E  V / cm
d 1  104
J drift  nq d  nq n 
J drift  nq a  nq n   1016  1.6  10 19  20  5  10 4  1.6 KA
cm 2

38. For the circuit shown, assume that the NMOS transistor is in saturation. Its threshold voltage
W
Vtn  1V and its transconductance parameter n Cox    1mA V2 . Neglect channel length
L
modulation and body bias effects. Under these conditions, the drain current ID in mA is
___________.
VDD  8V
1k
R1 RD
3 M ID

R2 RS 1k
5 M

Key: (1.9 to 2.1)


8 5
Exp: VG   5V
8

 ICP–Intensive Classroom Program  eGATE-Live Internet Based Classes DLP  TarGATE-All India Test Series
Leaders in GATE Preparations  65+ Centers across India
© All rights reserved by Gateforum Educational Services Pvt. Ltd. No part of this booklet may be reproduced or utilized in any form without the written permission.

17
|EC| GATE-2017-PAPER-I www.gateforum.com

VGS  VG  ID R S  5  103 ID
1 W
I D   n Cox    VGS  VT 
2

2  L
1
I D   1  103  VGS  VT 
2

2
1 2 103 2
I D   103 5  103 I D  1   4  103 I D 
2 2
3
10
ID  16  106 I 2D  8  103 
2 
0.5  103 I 2D  5I D  8  103  0
I D  8mA, 2mA
ID must be least value
So ID  2mA

39. Let X(t) be a wide sense stationary random process with the power spectral density S X(f) as
shown in Figure (a), where f is in Hertz (Hz). The random process X(t) is input to an ideal low
pass filter with frequency response
 1
1, f  Hz
H f   
2
0, 1
f  Hz
 2
As shown in Figure (b). The output of the lowpass filter is Y  t  .
SX  f 

exp   f  X(t) ideal lowpass Y(t)


filter h  t 
f cutoff  1 2 Hz
0
a  b
Let E be the expectation operator and consider the following statements.
I. E  X  t   E  Y  t 

II. E  X2  t    E  Y2  t  

III. E  Y2  t    2
Select the correct option:
(A) only I is true (B) only II and III are true
(C) only I and II are true (D) only I and III are true

Key: (A)

 ICP–Intensive Classroom Program  eGATE-Live Internet Based Classes DLP  TarGATE-All India Test Series
Leaders in GATE Preparations  65+ Centers across India
© All rights reserved by Gateforum Educational Services Pvt. Ltd. No part of this booklet may be reproduced or utilized in any form without the written permission.

18
|EC| GATE-2017-PAPER-I www.gateforum.com

Exp: SX  f 

exp   f 

H f 
1

1 2 12 f
SY  f 

1 2 12 f
Since DC components in same in Sx  f  and Sy  f 

 E  x  t   E  y  t 
 
E  x 2  t    Area under Sx  f    e df  2 e df  2
f f

 0

12
e f
1/2
E  y2  t   Area under Sy  f   2  e f df  2  2 1  e 1/2 
0
1 0

 E  x 2  t    E  y 2  t  
E  y 2  t    2

40. As shown a uniformly doped Silicon (Si) bar of length L = 0.1 m with a donor concentration
ND  1016 cm3 is illuminated at x = 0 such that electron and hole pairs are generated at the
 x 3 1
rate of G L  G L0 1   ,0  x  L, where G L0  10 cm s . Hole lifetime is 104 s,
17

 L 
19
electronic charge q  1.6 10 C, hole diffusion coefficient D p  100 cm 2 s and low level
injection condition prevails. Assuming a linearly decaying steady state excess hole
concentration that goes to 0 at x = L, the magnitude of the diffusion current density at x = L/2,
in A/cm2, is _________.
Light

Si  ND  1016 cm3 

x0 L  0.1 m
 ICP–Intensive Classroom Program  eGATE-Live Internet Based Classes DLP  TarGATE-All India Test Series
Leaders in GATE Preparations  65+ Centers across India
© All rights reserved by Gateforum Educational Services Pvt. Ltd. No part of this booklet may be reproduced or utilized in any form without the written permission.

19
|EC| GATE-2017-PAPER-I www.gateforum.com

Key: (15.9 to 16.1)

 L
  1 1 1
Exp: P  n  G Lo 1  2  P  G Lo    P  1017   104   1013 / cm 3
 L  2 2 2
 
1
 1013
dp 19 2
J P1 diff  qD P  1.6  10  100  4
 16A / cm 2
dx 0.1  10
2

41. The Nyquist plot of the transfer function


K
G s 
 s  2s  2   s  2 
2

does not encircle the point 1  j0  for K = 10 but does encircle the point  1  j0  for K =
100. Then the closed loop system (having unity gain feedback) is
(A) stable for K = 10 and stable for K = 100
(B) stable for K = 10 and unstable for K = 100
(C) unstable for K = 10 and stable for K =100
(D) unstable for K = 10 and unstable for K = 100
Key: (B)
k
Exp: G s 
 s  2s  2   s  2 
2

C.E  s3  4s2  76s  4  k  0


If system to stable
24  k  4  k  4  0
k  4  k  20
(i) Stable condition 4  k  20
Means If k  10 system stable
k  100 system unstable
k
Or G  j 
 2    2 j   2  j  
2

   2 
G  j    tan 1    tan 1  2 
 
2 2 

Im  G  j 
k
If   0 G  j  0
4
   G  j  0  270
x
Re G  j 
1
20

 ICP–Intensive Classroom Program  eGATE-Live Internet Based Classes DLP  TarGATE-All India Test Series
Leaders in GATE Preparations  65+ Centers across India
© All rights reserved by Gateforum Educational Services Pvt. Ltd. No part of this booklet may be reproduced or utilized in any form without the written permission.

20
|EC| GATE-2017-PAPER-I www.gateforum.com

So If k  10 touching point  0.5


If k  100 touching point  5
N  P  Z, Here P  0
N  Z
If closed loop system to be stable, then Z  0,  N  0,
So, k  10 is stable system

42. In the figure shown, the npn transistor acts as a switch


5V

4.8 k

Vin  t 
12 k
2V

T Vin  t 
0V

t  in seconds 

For the input Vin  t  as shown in the figure, the transistor switches between the cut-off and
saturation regions of operation, when T is large. Assume collector-to-emitter voltage
saturation VCE  sat   0.2V and base-to-emitter voltage VBE  0.7V. The minimum value of the
common-base current gain    of the transistor for the switching should be _________.
Key: (0.89 to 0.91)

IB 
 2  0.7   0.108mA
Key:
12  103
5  0.2
IC   1mA
4.8  103
I 1
 C   9.259
I B 0.108
 9.259
   0.903
1   1  9.259

A three dimensional region R of finite volume is described by x  y  z ;0  z  1,


2 2 3
43.
Where x, y, z are real. The volume of R (up to two decimal places) is ___________.

Key: (0.70 to 0.85)


Exp: PQ  x 2  y2 is the radius of variable circle at some Z.
 PQ2  x 2  y 2  z3  Given 

 ICP–Intensive Classroom Program  eGATE-Live Internet Based Classes DLP  TarGATE-All India Test Series
Leaders in GATE Preparations  65+ Centers across India
© All rights reserved by Gateforum Educational Services Pvt. Ltd. No part of this booklet may be reproduced or utilized in any form without the written permission.

21
|EC| GATE-2017-PAPER-I www.gateforum.com

1

1 1
z4
 Volume of region revolved around z  axis     PQ  dz   z dz     0.79
2 3

z 0 0
4 0 4

P Q

x y

44. The expression for an electric field in free space is E  E 0  x  y  j2z e 


 j t  kx  ky 

, where x, y, 
z represent the spatial coordinates, t represents time, and ,k are constants. This electric field
(A) does not represent a plane wave
(B) represents a circular polarized plane wave propagating normal to the z-axis
(C) represents an elliptically polarized plane wave propagating along x-y plane.
(D) represents a linearly polarized plane wave
Key: (C)
Exp: Given the direction of propagation is aˆ x  aˆ y

The orientation of E field is aˆ x  aˆ y  j2aˆ z


The dot product between above two is  1  1  0  0
 It is a plane wave
We observed that
P  aˆ x  aˆ y , aˆ x  aˆ y and j2aˆ z are normal to each other.

So electric field can be resolved into two normal component along aˆ x  aˆ y and j2aˆ x


The magnitude are 2 and 2 and  
2
So elliptical polarization.

45. A finite state machine (FSM) is implemented using the D flip-flops A and B, and logic gates, as
shown in the figure below. The four possible states of the FSM are QA QB  00,01,10 and 11.

QA
D Q D Q QB

A B
XIN Q
CK Q CK

CLK

 ICP–Intensive Classroom Program  eGATE-Live Internet Based Classes DLP  TarGATE-All India Test Series
Leaders in GATE Preparations  65+ Centers across India
© All rights reserved by Gateforum Educational Services Pvt. Ltd. No part of this booklet may be reproduced or utilized in any form without the written permission.

22
|EC| GATE-2017-PAPER-I www.gateforum.com

Assume that X1N is held at a constant logic level throughout the operation of the FSM. When
the FSM is initialized to the state QA QB  00 and clocked, after a few clock cycles, it starts
cycling through
(A) all of the four possible states if X1N  1
(B) three of the four possible states if X1N  0
(C) only two of the four possible states if XIN  1
(D) only two of the four possible states if X1N  0
Key: (D)

Exp: In given diagram

Xin=0 Next State Xin=1 Next State


Prsent State DA DB Xin Xin
 A  B  A  B

00 0 1 0 1 0 1 0 1
01 1 1 0 1 1 1 1 1
11 0 1 0 1 0 1 0 0
01 1 1 0 1 1 1 0 1
When Xin  0 2State
When Xin=1 3 State

46. Let x(t) be a continuous time periodic signal with fundamental period T = 1 seconds. Let {ak}
be the complex Fourier series coefficients of x(t), where k is integer valued. Consider the
following statements about x(3t):
I. The complex Fourier series coefficients of x(3t) are {ak} where k is integer valued
II. The complex Fourier series coefficients of x(3t) are {3ak} where k is integer valued
III. The fundamental angular frequency of x(3t) is 6 rad/s
For the three statements above, which one of the following is correct?
(A) only II and III are true (B) only I and III are true
(C) only III is true (D) only I is true
Key: (B)
Exp: Fourier series coefficient ak is unaffected by scaling operating. Thus (I) is true and (II) is false.
1
T  1sec for x(t) and if it compressed by „3‟ then the resultant period T 
3
2
 Fundamental frequency   6 rad/sec.
T1
Thus (III) is correct.

47. A 4-bit shift register circuit configured for right-shift operation, i.e, Din  A, A  B, B  C,
C  D, is shown. If the present state of the shift register is ABCD = 1101, the number of
clock cycles required to reach the state ABCD = 1111 is _________.

 ICP–Intensive Classroom Program  eGATE-Live Internet Based Classes DLP  TarGATE-All India Test Series
Leaders in GATE Preparations  65+ Centers across India
© All rights reserved by Gateforum Educational Services Pvt. Ltd. No part of this booklet may be reproduced or utilized in any form without the written permission.

23
|EC| GATE-2017-PAPER-I www.gateforum.com

Din A B C D
Clock

Key: (10.0 to 10.0)


Exp:

CLK A B C D
Din  A  B A  B B  C C  D
0 1 1 0 1  initial state
1 0 1 1 0
2 0 0 1 1
3 1 0 0 1
4 0 1 0 0
5 0 0 1 0
6 0 0 0 1
7 1 0 0 0
8 1 1 0 0
9 1 1 1 0
10 1 1 1 1  Final state
10 clock pulse required.

Let f  x   ex  x for real x. From among the following, choose the Taylor series
2
48.
approximation of f(x) around x = 0, which included all powers of x less than or equal to 3.
3 2
(A) 1  x  x2  x3 (B) 1  x  x  x
3

2
3 2 7 3
(C) 1  x  x  x (D) 1  x  3x 2  7x3
2 6
Key: (C)
x2 x3
Exp: We have Taylor series of f(x) around x = 0 is f  x   f  0   xf '  0   f ''  0   f '''  0 
2! 3!
(upto powers of „x‟ less than or equal to „3‟)
Given
f  x   ex  x  f  0  1
2

f '  x   e x  x 1  2x   f '  0   1
2

f ''  x   e x  x 1  2x   2e x  x  f ''  0   3
2 2 2

f '''  x   e x  x 1  2x   e x  x 4 1  2x   2 1  2x  e x  x  f '''  0   7
 3 2 2

x2 x3 3 7
 f  x   e x  x  1  x.1   3   7   1  x  x 2  x 3
2

2! 3! 2 6

 ICP–Intensive Classroom Program  eGATE-Live Internet Based Classes DLP  TarGATE-All India Test Series
Leaders in GATE Preparations  65+ Centers across India
© All rights reserved by Gateforum Educational Services Pvt. Ltd. No part of this booklet may be reproduced or utilized in any form without the written permission.

24
|EC| GATE-2017-PAPER-I www.gateforum.com

49. The following FIVE instructions were executed on an 8085 microprocessor.


MVI A, 33H
MVI B, 78H
ADD B
CMA
ANI 32H
The Accumulator value immediately after the execution of the fifth instruction is
(A) 00H (B) 10H (C) 11H (D) 32H
Key: (B)
Exp: MVI A, 33H A  33H
MVI B, 78H B  78H
ADD B B  ABH
CMA A  54H
ANI 32H A  10H
A  0011 0011 A  1010 1011 0101 0100
B  0111 1000 B  0101 0100 0011 0010
1010 1011 0001 0000

50. In the circuit shown, the voltage V1N  t  is described by: 1 I



 0, for t  0
VIN  
15 volts for t  0 VIN (t) 1H 2H

Where t is in seconds. The time (in seconds) at which the current



I in the circuit will reach the value 2 Amperes is ___________.
Key: (0.30 to 0.40)
Exp: Under dc condition inductor acts as short all
15
 I total   15A
1 Itotal 1 I  t   2A

i  t   i 0   i  e 
t
 2
 i  

i  0   i  0   0A Vin
1H 24
i  t    0  15  e
3 t
2
 15

 

i total  t   15 1  e
3 t
2
A
i  0   0A
Vin  15
i  t  total 
I total 15
3

3
1 e 2
3 t


2  5 1 e
3 t
2
  t  0.34sec
51. A half wavelength dipole is kept in the x-y plane and oriented along 45o from the x-axis.
Determine the direction of null in the radiation pattern for 0    . Here the angle
  0      is measured from the z-axis, and the angle   0    2  is measured from the
x-axis in the x-y plane.

 ICP–Intensive Classroom Program  eGATE-Live Internet Based Classes DLP  TarGATE-All India Test Series
Leaders in GATE Preparations  65+ Centers across India
© All rights reserved by Gateforum Educational Services Pvt. Ltd. No part of this booklet may be reproduced or utilized in any form without the written permission.

25
|EC| GATE-2017-PAPER-I www.gateforum.com

(A)   90o ,   45o (B)   45o ,   90o


(C)   90o ,   135o (D)   45o ,   135o
Key: (A)
Exp: The null occurs along axis of the antenna which is   90o and   45o

52. The amplifier circuit shown in the figure is R 2  79k


implemented using a compensated operational
amplifier (op-amp), and has an open-loop voltage R1  1k
gain, A o  105 V V and an open-loop cut-off 
frequency f c  8Hz. The voltage gain of the amplifier Vo
at 15 kHz, in V/V is __________. 

V1 ~
Key: (43.3 to 45.3)
Exp: Given Amplifier is using –ve feed back
Ao
Af 
1  A o
1
 ; A o  105
80
105
Af   79.93
1  105 / 80
f cut  8Hz  1  A o   10008Hz
Af
A f   
1   f / f cut 
2

79.93

2
 15  103 
1  
 10008 
 44.3

53. Let h[n] be the impulse response of a discrete-time linear time invariant (LTI) filter. The
impulse response is given by
1 1 1
h  0  ; h 1  ; h  2  ; and h  n   0 for n  0 and n  2.
3 3 3
Let H   be the discrete-time Fourier system transform (DTFT) of h[n], where  is the
normalized angular frequency in radians. Given that H  0   0 and 0  0  , the value of
0 (in radians) is equal to __________.
Key: (2.05 to 2.15)
Exp: It is given that,

 ICP–Intensive Classroom Program  eGATE-Live Internet Based Classes DLP  TarGATE-All India Test Series
Leaders in GATE Preparations  65+ Centers across India
© All rights reserved by Gateforum Educational Services Pvt. Ltd. No part of this booklet may be reproduced or utilized in any form without the written permission.

26
|EC| GATE-2017-PAPER-I www.gateforum.com

1 1 1
h  0  ;h 1  ;h  2  &
3 3 3
h  n   0for n  0and n  2.
 h  n   h  0   n   h 1   n  1  h  2   n  2
1
 [n]    n  1    n  2
3
Apply DTFT on both sides,
1
 H    1  e  j  e 2 j 
3
Given that H  0   0 & 0  0  
1   j0 
3 j0 j0
 H  0   1  e 2  e 2  e 2    0
3    
3 j0
0
1  2e 2
cos 0
2
0 1
consider H  0   cos 
2 2
2
0 
3
0  2.094

54. Which one of the following gives the simplified sum of products expression for the Boolean
function F  m0  m2  m3  m5 , where m0 ,m2 ,m3 and m5 are minterms corresponding to
the inputs A, B and C with A as the MSB and C as the LSB?
(A) AB  ABC  ABC (B) AC  AB  ABC
(C) AC  AB  ABC (D) ABC  AC  ABC
Key: (B)
Exp: F  Mo  M2  M3  M5  minterm
BC
A 00 01 11 10
AC

0 1 1 1
AB

1 1

AB C

55. A continuous time signal x  t   4cos  200t   8cos  400t  , where t is in seconds, is the
input to a linear time invariant (LTI) filter with the impulse response
 2sin  300t 
 , t0
ht   t
 t 0
 600,

Let y(t) be the output of this filter. The maximum value of y  t  is ________.

 ICP–Intensive Classroom Program  eGATE-Live Internet Based Classes DLP  TarGATE-All India Test Series
Leaders in GATE Preparations  65+ Centers across India
© All rights reserved by Gateforum Educational Services Pvt. Ltd. No part of this booklet may be reproduced or utilized in any form without the written permission.

27
|EC| GATE-2017-PAPER-I www.gateforum.com

Key: (7.90 to 8.10)


Exp: Given
 2sin 300t
 , t0
ht   t

 600 , t 0
Thus h  t   600sin c  300t 

 f 
 H  f   2rect  .
 300 
Given x  t   4cos 200t  8cos 400t
In f-domain,
X  f   2 
  f  100    f  100  4 
  f  200    f  200

H f 
X f 

4
2
 multiply 

150 150 f 200 100 100 200 f

Y f 


100 100 f

General Aptitude

Q. No. 1 - 5 Carry One Mark Each

1. She has a sharp tongue and it can occasionally turn _______.


(A) hurtful (B) left (C) methodical (D) vital
Key: (A)
2. Some table are shelves. Some shelves are chairs. All chairs are benches. Which of the
following conclusion can be deduced from the preceding sentences?
(i) At least one bench is a table
(ii) At least one shelf is a bench
(iii) At least one chair is a table
(iv) All benches are chairs
(A) only (i) (B) only (ii) (C) only (ii) and (iii) (D) only (iv)

Key: (B)

 ICP–Intensive Classroom Program  eGATE-Live Internet Based Classes DLP  TarGATE-All India Test Series
Leaders in GATE Preparations  65+ Centers across India
© All rights reserved by Gateforum Educational Services Pvt. Ltd. No part of this booklet may be reproduced or utilized in any form without the written permission.

28
|EC| GATE-2017-PAPER-I www.gateforum.com

Exp: B T S S
T S C
B T B
C C

3. 40% of deaths on city roads may be attributed to drunken driving. The number of degree
needed to represent this as a slice of a pie chart is
(A) 120 (B) 144 (C) 160 (D) 212
Key: (B)
Exp: Given 40% of deaths on city roads are drunken driving
 360  360
w.k.t. in pie chart 100%  360 1%     40%   40  40% 144o
 100  100

4. In the summer, water consumption is known to decrease overall by 25%. A water Board
official states that in the summer household consumption decreases by 20%, while other
consumption increases by 70%.
Which of the following statement is correct?
(A) The ratio of household to other consumption is 8/17
(B) The ratio of household to other consumption is 1/17
(C) The ratio of household to other consumption is 17/8
(D) There are errors in the official‟s statement
Key: (D)
Exp: Let H is house hold consumption and P is the other consumption.
Given
H  0.8  P  1.7   H  P   0.75
 Ratio is negative.

5. I ________ made arrangements had I _________informed earlier.


(A) could have, been (B) would have, being
(C) had, have (D) had been, been
Key: (A)

Q. No. 6- 10 Carry Two Marks Each

6. “If you are looking for a history of India, or for an account of the rise and fall of the British
Raj, or for the reason of the cleaving of the subcontinent into two mutually antagonistic parts
and the effects this mutilation will have in the respective section, and ultimately on Asia, you
will not find it in these pages; for though I have spent a lifetime in the country. I lived too near
the seat of events, and was too intimately associated with the actors, to get the perspective
needed for the impartial recording of these matters”.
Here, the word „antagonistic‟ is closest in meaning to
(A) impartial (B) argumentative (C) separated (D) hostile
Key: (D)

 ICP–Intensive Classroom Program  eGATE-Live Internet Based Classes DLP  TarGATE-All India Test Series
Leaders in GATE Preparations  65+ Centers across India
© All rights reserved by Gateforum Educational Services Pvt. Ltd. No part of this booklet may be reproduced or utilized in any form without the written permission.

29
|EC| GATE-2017-PAPER-I www.gateforum.com

7. There are 3 Indians and 3 Chinese in a group of 6 people. How many subgroups of this group
can we choose so that every subgroup has at least one Indian?
(A) 56 (B) 52 (C) 48 (D) 44
Key: (A)
Exp: No. of sub groups such that every sub group has at least one Indian
 3C1  3C2  3C3  3C1  3C2  3C1  3C1  3C3
   
Onlyindians Oneindian&remaining chinese

 3C2  3C1  3C2  3C2  3C2  3C3  3C3  3C1  3C3  3C2  3C3  3C3
   
2indians & remainnig chinese 3indians & remaining chinese

 7  9  9  3  9  9  3  3  3  1  56.
Alternate method
Sub groups containing only Indians = 3C1  3C2  3C3  3  3  1  7

Subgroups containing one Indian and rest chinese = 3C 1 3C1  3C2  3C2   33  3  1  21

Sub groups containing two Indian and remaining Chinese  3C2 3C1  3C2  3C3   21

Sub groups containing three Indian and remaining Chinese  3C3 3C1  3C2  3C3   7

 Total no. of sub groups = 7+21+21+7 =56.

8. A contour line joints locations having the same height above


the mean sea level. The following is a contour plot of a
geographical region. Contour lines are shown at 25 m
intervals in this plot.
The path from P to Q is best described by
(A) Up-Down-Up-Down
(B) Down-Up-Down-Up
(C) Down-Up-Down
(D) Up-Down-Up
Key: (C)
Exp: Down- up-Down
 between 475&500 
 between 525&550 
Q
down down
 more than 575
up
P up

At p,height  575  between 500 &525


9. Trucks (10m long) and cars (5 m long) go on a single lane bridge. There must be a gap of
atleast 20 m after each truck and a gap of atleast 15m after each car. Trucks and cars travel at a
speed of 36 km/h. If cars and trucks go alternatively, what is the maximum number of vehicles
that can use the bridge in one hour?
(A) 1440 (B) 1200 (C) 720 (D) 600

 ICP–Intensive Classroom Program  eGATE-Live Internet Based Classes DLP  TarGATE-All India Test Series
Leaders in GATE Preparations  65+ Centers across India
© All rights reserved by Gateforum Educational Services Pvt. Ltd. No part of this booklet may be reproduced or utilized in any form without the written permission.

30
|EC| GATE-2017-PAPER-I www.gateforum.com

Key: (A)
Exp: Given speeds both car & Truck = 36 km/hour

They travel in 1 hr = 36 km = 36000 m.


5m 5m truck 20m
car gap 10m gap 1hr  36km  36000m
Maximum no.of vehicles than can use the bridge
36000m
in1hour   720sets  720  2  1440 vechicles
50m
Alternate method
Length of truck + gap required = 10+20 = 30m
Length of car + gap required = 5+15 = 20m
Alternative pairs of Truck and car needs 30+ 20 = 50 m.
Let 'n' be the number of repetition of (Truck + car) in 1 hour (3600 sec).
Given speed  36 km hr  10m sec
50m  n
 36 km hr
3600sec
50n
 m sec  10 m sec
3600
36000
n   720  Truck  car 
50
So, 720  Truck  car  passes  720  2  1440 vehicles

10. S, T, U, V, W, X, Y and Z are seated around a circular table. T‟s neighbours are Y and V. Z is
seated third to the left of T and second to the right of S.U‟s neighbours are S and Y; and T and
W are not seated opposite each other. Who is third to the left of V?
(A) X (B) W (C) U (D) T
Key: (A)
Exp: Following circular seating arrangement can be drawn.
X
Z S

W U

Y V
T
Only one such arrangement can be drawn.
The person on third to the left of V is X.

 ICP–Intensive Classroom Program  eGATE-Live Internet Based Classes DLP  TarGATE-All India Test Series
Leaders in GATE Preparations  65+ Centers across India
© All rights reserved by Gateforum Educational Services Pvt. Ltd. No part of this booklet may be reproduced or utilized in any form without the written permission.

31
|EC| GATE-2017-PAPER-II www.gateforum.com

Electronics and Communication Engineering


Q. No. 1 to 25 Carry One Mark Each

1. Consider the circuit shown in the figure.


Y 0
MUX 0

0 MUX F
1
1
X
Z
The Boolean expression F implemented by the circuit is
(A) XYZ  XY  YZ (B) XYZ  XZ  YZ
(C) XYZ  XY  YZ (D) XYZ  XY  YZ
Key (B)

 
y 0
Exp: F  xyz  z xy
xy
F  xyz   x  y  z
0
F

F  xyz  xz  yz 0 1 1

x z

2. An LTI system with unit sample response h  n   5  n   7  n  1  7  n  3  5  n  4 is a


(A) Low – pass filter (B) high – pass filter (C) band – pass filter (D) band – stop filter
Key: (C)
Exp: h  n   5  n   7  n  1  7  n  3  5  n  4
Obtain
h e j   5  7e j  7e3 j  54 j


At   0 and ;
2
h e   0
j


For 0    at a frequency 0 maximum value of h e j  is obtained
2
h e j 


0
  

2 2
Thus Ideal behaviour of h[n] is Band pass filter.

 ICP–Intensive Classroom Program  eGATE-Live Internet Based Classes DLP  TarGATE-All India Test Series
Leaders in GATE Preparations  65+ Centers across India
© All rights reserved by Gateforum Educational Services Pvt. Ltd. No part of this booklet may be reproduced or utilized in any form without the written permission.

1
|EC| GATE-2017-PAPER-II www.gateforum.com

3. In the circuit shown, V is a sinusoidal voltage source. The current I is in phase with voltage V.
amplitude of voltage across the capacitor
The ratio is __________.
amplitude of voltage across the resistor

5 5H I

5F
V

Key: (0.19 to 0.21)


Exp: If I &V are in phase then the circuit is in resonance
At resonance
VC 1 L 1 5
Q   0.2
VR R C 5 5

4. In a DRAM,
(A) periodic refreshing is not required
(B) information is stored in a capacitor
(C) information is stored in a latch
(D) both read and write operations can be performed simultaneously
Key: (B)

5. Consider an n-channel MOSFET having width W, length L, electron mobility in the channel
n and oxide capacitance per unit area Cox . If gate-to-source voltage VGS=0.7V, drain-to-
source voltage VDS=0.1V,  n Cox   100A / V 2 , threshold voltage VTH=0.3 V and (W/L) =50,
then the transconductance gm (in mA/V) is ___________.
Key: (0.45 to 0.55)
Exp: Here, VDS  VGS  VPH , so n-channel MOSFET is working in linear region.
W 2
VDS 
I D   n Cox 
 GSV  VTH  .VDS  
L  2 

So, transconductance g m is in linear region and is given by

ID W
gm    n Cox  .VDS  100 106  50  0.1  5 104  0.5 mA V
VGS VDS  const
L

6. Two conducting spheres S1 and S2 of radii a and b (b>a) respectively, are placed far apart and
connected by a long, thin conducting wire, as shown in the figure.

S2
S1
Wire

Radius a
Radius b

 ICP–Intensive Classroom Program  eGATE-Live Internet Based Classes DLP  TarGATE-All India Test Series
Leaders in GATE Preparations  65+ Centers across India
© All rights reserved by Gateforum Educational Services Pvt. Ltd. No part of this booklet may be reproduced or utilized in any form without the written permission.

2
|EC| GATE-2017-PAPER-II www.gateforum.com

For some charge placed on this structure, the potential and surface electric field on S1 are Va
and Ea, and that on S2 are Vb and Eb, respectively, which of the following is CORRECT?
(A) Va  Vb and Ea  Eb (B) Va  Vb and Ea  Eb
(C) Va  Vb and Ea  Eb (D) Va  Vb and Ea  Eb
Key: (C)
Exp:  bc3 two spheres are joined with a conducting wire, the voltage on two spheres is same.
 Va  Vb  The capacitance of sphere  radius
Ca a

Cb b
We know Q  CV
Q a Ca a
 
Q b Cb b
1 a
E a 4o a2 b
  1
Eb 1 b a
4o b2
Ea  E b

7. For the circuit shown in the figure, P and Q are the inputs and Y is the output.

PMOS

Y
P NMOS
Q

The logic implemented by the circuit is


(A) XNOR (B) XOR (C) NOR (D) OR
Key: (B)

8. An n-channel enhancement mode MOSFET is biased at VGS  VTH and VDS   VGS  VTH  , where
VGS is the gate-to-source voltage, VDS is the drain-to-source voltage and VTH is the threshold
voltage. Considering channel length modulation effect to be significant, the MOSFET behaves
as a
(A) voltage source with zero output impedance
(B) voltage source with non-zero output impedance
(C) current source with finite output impedance
(D) current source with infinite output impedance
Key: (C)
Exp: If channel length modulation is considered and significant it means   0

 ICP–Intensive Classroom Program  eGATE-Live Internet Based Classes DLP  TarGATE-All India Test Series
Leaders in GATE Preparations  65+ Centers across India
© All rights reserved by Gateforum Educational Services Pvt. Ltd. No part of this booklet may be reproduced or utilized in any form without the written permission.

3
|EC| GATE-2017-PAPER-II www.gateforum.com

1 V
VA  early voltage   and re  A
 ID

 

Vgs g m v gs re Vo


If VAS  VTH and VDS   VDS  VTH  then it indicates that MOSFET is working in saturation
region and it can be used as an amplifier. So it can act as current source with finite output
impedance.

9. A connection is made consisting of resistance A in series with a parallel combination of


resistances B and C. Three resistors of value 10, 5, 2 are provided. Consider all possible
permutations of the given resistors into the positions A, B, C, and identify the configurations
with maximum possible overall resistance, and also the ones with minimum possible overall
resistance. The ratio of maximum to minimum values of the resistances (up to second decimal
place) is ____________.
Key: (2.12 to 2.16)
Exp: The maximum resistance
5
10
R max  11.428

5
The minimum resistance
10
2
R min  5.333

R max 5
 2.14
R min

10. An npn bipolar junction transistor (BJT) is operating in the active region. If the reverse bias
across the base – collector junction is increased, then
(A) the effective base width increases and common – emitter current gain increases
(B) the effective base width increases and common – emitter current gain decreases
(C) the effective base width decreases and common – emitter current gain increases
(D) the effective base width decreases and common – emitter current gain decreases
Key: (C)
Exp: If the reverse bias voltage across the base collector junction is increased, then their effective
base width will decrease and collector current will increase, therefore their common-emitter
current gain increases.

 ICP–Intensive Classroom Program  eGATE-Live Internet Based Classes DLP  TarGATE-All India Test Series
Leaders in GATE Preparations  65+ Centers across India
© All rights reserved by Gateforum Educational Services Pvt. Ltd. No part of this booklet may be reproduced or utilized in any form without the written permission.

4
GATE ESE PSU’s 2019-20
ECE ENGINEERING
GATE ECE 2003-2019 SOLVED

GATE ECE 2003-2019 SOLVED Detail Solution

CONTENT COVERED:
1.Theory Notes
2.Explanation
3.Derivation
4.Example
5.Shortcut & Formula Summary
6.Previous year Paper Q. Sol.
Noted-: Single Source Follow, Revise
Multiple Time Best key of Success
1
Page

http://www.orbitmentor.com [email protected]
|EC| GATE-2017-PAPER-II www.gateforum.com

11. Consider the state space realization


 
 x1  t    0 0   x1  t    0  u  t  , with the initial condition  x1  0    0 
  0 9   x  t    45   
 x 2  0    0 
 2   
x t  2  ,

where u(t) denotes the unit step function. The value of lim x12  t   x 22  t  is _______.
t 

Key: (4.99 to 5.01)


Exp: x 1  t   0 ...1
x .2  t   9x 2  t   45u  t  ... 2 
Apply L.T to above equation
x1  t   0  because initial conditions are zero
45
Sx 2  s   x 2  0   9X 2  s  
S
45
X 2  s  s  9 
3
45
X2 s  
s s  9
S 5
X2 s   
5 s9
X 2  t   5u  t   5e 9t u  t 

It x12  t   x 22  t   It x 2  t   5
t  t 

1 1 0 0 0 
0 0 1 1 0 

12. The rank of the matrix 0 1 1 0 0  is ___________.
 
 1 0 0 0 1 
0 0 0 1 1

Key: (4 to 4)
Exp:  1 1 0 0 0 
 0 0 1 1 0 
 
 0 1 1 0 0 
 
 1 0 0 0 1 
 0 0 0 1 1

1 1 0 0 0 
0 0 1 1 0 
 
R 4  R 4  R 1 ~ 0 1 1 0 0 
 
0 1 0 0 1 
0 0 0 1 1

 ICP–Intensive Classroom Program  eGATE-Live Internet Based Classes DLP  TarGATE-All India Test Series
Leaders in GATE Preparations  65+ Centers across India
© All rights reserved by Gateforum Educational Services Pvt. Ltd. No part of this booklet may be reproduced or utilized in any form without the written permission.

5
|EC| GATE-2017-PAPER-II www.gateforum.com

1 1 0 0 0 
0 0 1 1 0 
 
R 4  R 4  R 3 ~ 0 1 1 0 0 
 
0 0 1 0 1 
0 0 0 1 1

1 1 0 0 0 
0 1 1 0 0 
 
R 2  R 3 ~ 0 0 1 1 0 
 
0 0 1 0 1 
0 0 0 1 1

1 1 0 0 0 
0 1 1 0 0 
 
R 4  R 4  R 3 ~ 0 0 1 1 0 
 
0 0 0 1 1 
0 0 0 1 1

1 1 0 0 0
0 1 1 0 0 

R 5  R 5  R 4 ~ 0 0 1 1 0
 
0 0 0 1 1
0 0 0 0 0 

Which is in Echelon form  Rank  No. of non zero rows  4

13. A two – wire transmission line terminates in a television set. The VSWR measured on the line
is 5.8. The percentage of power that is reflected from the television set is ______________
Key: (48.0 to 51.0)
Percentage of power reflected is   100
2
Exp:
VSWR  1 5.8  1 4.8
    0.7058
VSWR  1 5.8  1 6.8
% Power reflected =  100  49.82%
2

14. The input x(t) and the output y (t) of a continuous-time system are related as
yt   x  u  du. The system is
t

t T

(A) Linear and time-variant (B) Linear and time-invariant


(C) Non-linear and time-variant (D) Non-linear and time-invariant
Key: (B)
Exp: Given Input-output relationship describes integration over a fundamental period T. The
integration over one period is linear and time-invariant.

 ICP–Intensive Classroom Program  eGATE-Live Internet Based Classes DLP  TarGATE-All India Test Series
Leaders in GATE Preparations  65+ Centers across India
© All rights reserved by Gateforum Educational Services Pvt. Ltd. No part of this booklet may be reproduced or utilized in any form without the written permission.

6
|EC| GATE-2017-PAPER-II www.gateforum.com

15. Which of the following statements is incorrect?


(A) Lead compensator is used to reduce the settling time.
(B) Lag compensator is used to reduce the steady state error.
(C) Lead compensator may increase the order of a system.
(D) Lag compensator always stabilizes an unstable system.
Key: (D)
Exp: The phase-lead controller adds zero and a pole, with the zero to the right of the pole, to the
forward-path transfer function. The general effect is to add more damping to the closed-loop
system. The rise time and settling time are reduced in general.
 Reduces the steady state error
 Reduces the speed of response (i.e  decreases)
 Increases the gain of original network without affecting stability
 Permits the increases of gain if phase margin is acceptable
 System becomes lesser stable
 Reduces the effect of noise
 Decrease the bandwidth

1
16. The residues of a function f  z   are
 z  4 z  1
3

1 1 1 1 1 1 1 1
(A) and (B) and (C) and (D) and
27 125 125 125 27 5 125 5
Key: (B)
Exp: Z = 4 is a pole of order „1‟ (or) simple pole
 1  1 1
Residue of f(z) at z= 4 = Res f  z   lim  z  4  .  3  and
 z  4  z  1  5 125
3
z4
z 4

z = – 1 is a pole of order „3‟.

1  d 2  1  
 Res f  z    z  1 .
3
lim  2 
3 
z 1  3  1! z1  dz   z  4  z  1  
1  d2  1  1
 lim  2    
2 z 1  dz  z  4   125

17. A sinusoidal message signal is converted to a PCM signal using a uniform quantizer. The
required signal-to-quantization noise ratio (SQNR) at the output of the quantizer is 40dB. The
minimum number of bits per sample needed to achieve the desired SQNR is _______
Key: (7 to 7)
Exp: For sinusoidal signal
 SNR Q in dB  6.0 n  1.75
Given required  SNR Q  40dB

 ICP–Intensive Classroom Program  eGATE-Live Internet Based Classes DLP  TarGATE-All India Test Series
Leaders in GATE Preparations  65+ Centers across India
© All rights reserved by Gateforum Educational Services Pvt. Ltd. No part of this booklet may be reproduced or utilized in any form without the written permission.

7
|EC| GATE-2017-PAPER-II www.gateforum.com

 6.0 n  1.75  40dB


 6.0n  40  11.75
40  11.75
n
6.02
 n  7  Since ' n ' must be an int eger 

d2 y dy
18. The general solution of the differential equation 2
2  5y  0 in terms of arbitrary
dx dx
constants K1 and K2 is
(A) K1e   1 6  x (B) K1e   1 8  x
1 6 x 1 8 x
 K2 e  K2 e

(C) K1e   2 6  x (D) K1e   2 8  x


2  6 x 2  8 x
 K2e  K2 e

Key: (A)
Exp: D2  2D  5  0
 D  1  6  roots are real and distinct 
 1 6  x  1 6  x
 y  k1 .e  k 2e
Where k1, k2 are arbitrary constants.

19. Which one of the following graphs shows the Shannon capacity (channel capacity) in bits of a
memory less binary symmetric channel with crossover probability P?
(A) (B)
1 1
Capacity Capacity

p p
0 1 0 1
(C) (D)
1 1
Capacity Capacity

p p
0 1 0 1

Key: (C)
Exp: For memory less binary Symmetric channel
Channel capacity
C
C  1  H p
1  1  1
H  p   p log 2  1  p  log 2  
p 1 p 
p  Cross over probability
 C  1  p log 2 p  1  p  log 2 1  p 
At p  0; C  1 P
0 1
At p 1 C 1
At p 1 2 C  0
 ICP–Intensive Classroom Program  eGATE-Live Internet Based Classes DLP  TarGATE-All India Test Series
Leaders in GATE Preparations  65+ Centers across India
© All rights reserved by Gateforum Educational Services Pvt. Ltd. No part of this booklet may be reproduced or utilized in any form without the written permission.

8
|EC| GATE-2017-PAPER-II www.gateforum.com

20. The output V0 of the diode circuit shown in the figure is connected to an averaging DC
voltmeter. The reading on the DC voltmeter in Volts, neglecting the voltage drop across the
diode, is ____________.

10sin t 1k V0
f  50Hz

Key: (3.15 to 3.21)
Vm 10
Exp: Vo    3.1847V
 

21. Consider the random process X  t   U  Vt, where U is a zero-mean Gaussian random variable
and V is a random variable uniformly distributed between 0 and 2. Assume that U and V are
statistically independent. The mean value of the random process at t = 2 is ____________
Key: (2)
Exp: Given x  t   U  Vt
x  2   U  2V
E  x  2    E  U  2V   E  U   2E  V   0  2  1  2

22. For the system shown in the figure, Y (s) / X (s) = __________.

  
X s  G s   2 Y s 


Key: (0.95 to 1.05) X s   Y s
G s
Y s 2 1 
Exp:  1
X s  1 2

23. The smaller angle (in degrees) between the planes x + y + z =1 and 2x – y + 2z = 0 is
________.
Key: (54.0 to 55.0)
Exp: x  y  2 1
2x  y  2z  0
We have angle between two planes
a1x  b1 y  c1 y  d1  0
a x x  b2 y  c2 y  d 2  0

a1a 2  b1b2  c1c2


is cos  
a12  b12  c12 a 22  b 22  c22
2 1 2 3 1
 cos        54.73
111 4 1 4 3 9 3

 ICP–Intensive Classroom Program  eGATE-Live Internet Based Classes DLP  TarGATE-All India Test Series
Leaders in GATE Preparations  65+ Centers across India
© All rights reserved by Gateforum Educational Services Pvt. Ltd. No part of this booklet may be reproduced or utilized in any form without the written permission.

9
|EC| GATE-2017-PAPER-II www.gateforum.com

24. Consider the circuit shown in the figure. Assume base-to- emitter voltage VBE=0.8 V and
common base current gain    of the transistor is unity.
18V

44k 4k

16k 2k

The value of the collector- to – emitter voltage VCE (in volt) is _______.
Key: (5.5 to 6.5)
Exp: Given VBE  0.8V;   1
As   1;  is very large
So IE  IC
18  16
VB   4.8V
60
4.8  08
IC   2mA
2  103
VCE  18  6  103  2  103
 18  12  6V

25. In the figure, D1 is a real silicon pn junction diode with a drop of 0.7V under forward bias
condition and D2 is a zener diode with breakdown voltage of -6.8 V. The input Vin(t) is a
periodic square wave of period T, whose one period is shown in the figure.

Vin  t  10F

14V
D1
0 10 Vout  t 
t  sec onds  D2
14V

Assuming 10  T. where  is the time constant of the circuit, the maximum and minimum
values of the output waveform are respectively,
(A) 7.5 V and –20.5V (B) 6.1 V and –21.9V
(C) 7.5 V and –21.2 V (D) 6.1 V and –22.6 V
Key: (A)
Exp: When Vi  14V, the equivalent circuit is

 ICP–Intensive Classroom Program  eGATE-Live Internet Based Classes DLP  TarGATE-All India Test Series
Leaders in GATE Preparations  65+ Centers across India
© All rights reserved by Gateforum Educational Services Pvt. Ltd. No part of this booklet may be reproduced or utilized in any form without the written permission.

10
|EC| GATE-2017-PAPER-II www.gateforum.com

VC
 
0.7V
Vi 10 Vo
6.8V

Vi  VC  Vo
VC  Vi  Vo
VC  14   6.8  0.7   14  7.5  6.5V
Maximum Vo  7.5V
When Vi  14V, the equivalent circuit is
VC
 

Vi 10 Vo

Vo  Vi  VC  14V  6.5  20.5V


Minimum Vo  20.5V

Q. No. 26 to 55 Carry Two Marks Each



26. If the vector function F  ax  3y  k1z   ay  k 2 x  2z   az  k 3 y  z  is irrotational, then the values
of the constants k1, k2 and k3 respectively, are
(A) 0.3, –2.5, 0.5 (B) 0.0, 3.0, 2.0 (C) 0.3, 0.33, 0.5 (D) 4.0, 3.0, 2.0
Key: (B)

Exp: curl F  0

i j k
  
 0
x y z
3y  k1z k 2 x  2z k 3 y  z
 i   k 3  2   j  0  k1   k  k 2  3   0
 k1  0, k 2  3, k 3  2

27. The un-modulated carrier power in an AM transmitter is 5kW. This carrier is modulated by a
sinusoidal modulating signal. The maximum percentage of modulation is 50%. If it is reduced
to 40%, then the maximum un-modulated carrier power (in kW) that can be used without
overloading the transmitter is ___________
Key: (5.19 to 5.23)
Exp: Total power when   50% is
 2 
PT  PC 1  
 2

 ICP–Intensive Classroom Program  eGATE-Live Internet Based Classes DLP  TarGATE-All India Test Series
Leaders in GATE Preparations  65+ Centers across India
© All rights reserved by Gateforum Educational Services Pvt. Ltd. No part of this booklet may be reproduced or utilized in any form without the written permission.

11
|EC| GATE-2017-PAPER-II www.gateforum.com

  0.5 2 
PT  5 1    5 1  0.125  5 1.125
 2 
PT  5.625
When   40%
Total power remains 5.625
  0.4 2 
 5.625  PC 1    5.625  PC 1  0.08
 2 
PC  5.22

28. Consider an LTI system with magnitude response


 |f |
1  , | f | 20
| H  f  |  20
0, | f | 20
And phase response Arg H  f   2f .
If the input to the system is
     
x  t   8cos  20t    16sin  40t    24 cos  80t   .
 4  8  16 
Then the average power of the output signal y (t) is _________.
Key: (7.95 to 8.05)
Exp: Consider an LTI system with magnitude response
 |f |
1  , | f | 20
| H  f  |  20
0, | f | 20

And phase response Arg H  f   2f .


If the input to the system is
     
x  t   8cos  20t    16sin  40t    24 cos  80t   .
 4   8   16 
Then the average power of the output signal y (t) is _________.
28. Obtain X  f  for the given x(t)

X  f   4 
  f  10     f  10   8 
  f  20     f  20 
  f  40     f  40  
12 

Xf 
12 Hf 
8 1
4  multiply  1 1
2 2

40 20 10 10 20 40 40 20 10 10 20


f f

 ICP–Intensive Classroom Program  eGATE-Live Internet Based Classes DLP  TarGATE-All India Test Series
Leaders in GATE Preparations  65+ Centers across India
© All rights reserved by Gateforum Educational Services Pvt. Ltd. No part of this booklet may be reproduced or utilized in any form without the written permission.

12
|EC| GATE-2017-PAPER-II www.gateforum.com

4
 Y f     f  10     f  10  
2
1 
     f  10     f  10   
 2 
y  t   4cos 2t
16
Thus max power  8
2

29. A MOS capacitor is fabricated on p-type Si (silicon) where the metal work function is 4.1 eV
and electron affinity of Si is 4.0 eV. EC-FF=0.9 eV, where EC and EF are the conduction band
minimum and the Fermi energy levels of Si, respectively. Oxide r  3.9, 0  8.85 1014 F / cm.
oxide thickness tox = 0.1 m and electronic charge q = 1.6×10-19 C. If the measured flat band
voltage of the capacitor is –1V, then the magnitude of the fixed charge at the oxide-
semiconductor interface, in nC/cm2, is __________.

Key: (6.85 to 6.95)


QF
Exp: VFB MS 
Cox
q MS  qM  qS
 qM  q xo  Ec  E f   4.1  4.0  0.9  0.1  0.9   0.8eV

0.8  q 1V
MS    0.8V
q
E
Cox  ox  34.5 109 F cm 2
t ox
QF
1  0.8 
34.5  109
QF
0.2  
34.5 109
Q F  6.9 nc cm 2

30. An electron (q1) is moving in free space with velocity 105 m/s towards a stationary electron
(q2) far away. The closest distance that this moving electron gets to the stationary electron
before the repulsive force diverts its path is ___________ ×10-8m.
[Given, mass of electron m = 9.11×10-31kg, charge of electron e = -1.6×10-19 C , and
permittivity 0  1/ 36  109 F / m]
Key: (4.55 to 5.55)
Exp: Work done due to field and external agent must be zero
1
qV  MV 2
2
1.6  1019 1
 m  105 
2
 1.6  1019 
4o  2
   5.058  108 m

 ICP–Intensive Classroom Program  eGATE-Live Internet Based Classes DLP  TarGATE-All India Test Series
Leaders in GATE Preparations  65+ Centers across India
© All rights reserved by Gateforum Educational Services Pvt. Ltd. No part of this booklet may be reproduced or utilized in any form without the written permission.

13
|EC| GATE-2017-PAPER-II www.gateforum.com

31. The values of the integrals


1 1 xy  11
xy 
0  0  x  y 3 
 dy  dx and 0  0  x  y 3  dy are
 dx
   
(A) same and equal to 0.5 (B) same and equal to -0.5
(C) 0.5 and – 0.5, respectively (D) - 0.5 and 0.5, respectively
Key: (C)
11 
xy
Exp: To find    dy  dx

0  0  x  y
3


xy
1 1 1
1 y
Consider   x  y
0
3
dy  x 
0  x  y
3
dy  
0  x  y
3
dy

31 1
x. x  y  1
 x  y   x dy
 
0  x  y
3  1 3
0

x  1 1
1 1
1 1
 
2   x  1 2
   
x  0  x  y 
2 2
dy  x 
0  x  y
3
dy

1
x  1 1   x  y x 1 1
2 1

        
2   x  12 x 2  2  1 2   x  12 x 2 
0

 1 1  1 1
 x   2  
  x  1 2
x  x 1 x
 
1

 x  1
2

11  1
xy 1  1
1
1
  dy  dx     dx  

0  0  x  y
3
 
0   x  1 
2
 x 1 0 2

11 
xy 1
Similarly    dx  dy 

0  0  x  y
3
 2

32. Passengers try repeatedly to get a seat reservation in any train running between two stations
until they are successful. If there is 40% chance of getting reservation in any attempt by a
passenger, then the average number of attempts that passengers need to make to get a seat
reserved is _________.
Key: (2.4 to 2.6)
Exp: Let „X‟ is a random variable which takes number of attempts
Given probability of any attempts to be successful,
40 2 2 3
p  40%   , q 1 
100 5 5 5
X 1 2 3 4 
2 3  2 3 3  2 3 3 3  2
pX                 
5 5 5 5 5 5 5 5 5 5

 ICP–Intensive Classroom Program  eGATE-Live Internet Based Classes DLP  TarGATE-All India Test Series
Leaders in GATE Preparations  65+ Centers across India
© All rights reserved by Gateforum Educational Services Pvt. Ltd. No part of this booklet may be reproduced or utilized in any form without the written permission.

14
|EC| GATE-2017-PAPER-II www.gateforum.com

2 3
 2  3 2  3  2  3  2
E  X    Xp  X   1    2     3      4      ...
 5   5 5     
5 5 5  5
2  2  3 2

 
2 3
3 3 3
 1  x   1  2x  3x 2  4x 3  ....
2
 1  2    3    4    .....  1  
5  5 5 5  5  5 
2 25
   2.5
5 4
 Average number of attempts that passengers need to make to get seat reserved is „2.5‟

33. Figure I shows a 4-bits ripple carry adder realized using full adders and Figure II shows the
circuit of a full-adder (FA). The propagation delay of the XOR, AND and OR gates in Figure
II are 20 ns, 15 ns and 10 ns respectively. Assume all the inputs to the 4-bit adder are initially
reset to 0.

Xn 
Y3 X 3 Y2 X 2 Y1 X1 Y0 X 0 
Yn  Sn

Z3 Z2 Z1
Z4 FA FA FA FA Z0
 Zn 1
Zn
S3 S2 S1 S0
Figure  I Figure  II

At t=0, the inputs to the 4-bit adder are changed to


X3X2 X1X0  1100, Y3Y2 Y1Y0  0100 and Z0  1.
The output of the ripple carry adder will be stable at t (in ns) = ___________
Key: (70.0 to 70.0)

34. The permittivity of water at optical frequencies is 1.75  0 . It is found that an isotropic light
source at a distance d under water forms an illuminated circular area of radius 5m, as shown in
the figure. The critical angle is c .

5m
Air

Water C d


Light
Source

The value of d (in meter) is _____________


Key: (4.2 to 4.4)
  2   1 
Exp: C  sin 1    sin 1    49.106
 1   1.75 
 
5 5
 tan C  d   4.33m
d tan C

 ICP–Intensive Classroom Program  eGATE-Live Internet Based Classes DLP  TarGATE-All India Test Series
Leaders in GATE Preparations  65+ Centers across India
© All rights reserved by Gateforum Educational Services Pvt. Ltd. No part of this booklet may be reproduced or utilized in any form without the written permission.

15
|EC| GATE-2017-PAPER-II www.gateforum.com

35. A unity feedback control system is characterized by the open-loop transfer function
10K  s  2 
G s  
s3  3s 2  10
The Nyquist path and the corresponding Nyquist plot of G(s) are shown in the figures below.

j
s  plane  j5.43K G  s   plane
 j
 j
s  Re j
R  0
 Re G
0 K 2K
 j

 j  j5.43K
Nyquist path for G  s  Nyquist Plot of G  s 

If 0 < K < 1, then the number of poles of the closed-loop transfer function that lie in the right –
half of the s-plane is
(A) 0 (B) 1 (C) 2 (D) 3
Key: (C)
Exp: N=0, Because 0 < L < 1
There are no encircles around (Y, 0)
And
10K  S  2  10K  S  2 
G  S  
S  3S  10
3 2
S  3.72  S   0.31  1.598i 
So, P2
NPZ
Z2
OR
C.E  S3  3S2  10KS  20K  10
If stable 30 K > 20K+10
K>1
Here, in the question asking 0 < K < 1
So, System is unstable

36. The signal x  t   sin 14000t  , where t is in seconds is sampled at a rate of 9000 samples per
second. The sampled signal is the input to an ideal low pass filter with frequency response
H(f) as follows :
1, | f | 12kHz
H f   
0, | f | 12kHz.
What is the number of sinusoids in the output and their frequencies in kHz?
(A) Number = 1, frequency = 7 (B) Number = 3, frequencies= 2,7,11
(C) Number = 2, frequencies = 2, 7 (D) Number = 2, frequencies = 2, 11
Key: (B)

 ICP–Intensive Classroom Program  eGATE-Live Internet Based Classes DLP  TarGATE-All India Test Series
Leaders in GATE Preparations  65+ Centers across India
© All rights reserved by Gateforum Educational Services Pvt. Ltd. No part of this booklet may be reproduced or utilized in any form without the written permission.

16
|EC| GATE-2017-PAPER-II www.gateforum.com

Exp: Given input signal x  t   sin 1400t 


Input signal spectrum
Xf 
1
2
7
7
1  f  kHz 

2

Sampled signal spectrum is the spectrum of X(f) which repeats with integer multiples of 9
kHz.
Sampled signal spectrum: Xs  f 

16 7
11 2
2 7 11 16
9 9 f

The sampled signal spectrum is passed through a LPF of cutoff frequency 12 KHz. Thus the
filtered out sinusoids are of 2 KHz 7 KHz and 11 KHz frequency.

37. A unity feedback control system is characterized by the open-loop transfer function
2  s  1
G s  
s  ks 2  2s  1
3

The value of k for which the system oscillates at 2 rad/s is ________.


Key: (0.74 to 0.76)
2  s  1
Exp: G s 
s  ks2  2s  1
3

  2 rad sec
K  ??
1  G s H s  0

2  k  1
1  s3  ks2  4s  3  0
s  ks  2s  1
3 2

3
s 1 4
s2 k 3
4k  3
s1 0
k
s0 3
For marginal stable
4k  3 3
 0  k   0.75
k 4
Cross check

 ICP–Intensive Classroom Program  eGATE-Live Internet Based Classes DLP  TarGATE-All India Test Series
Leaders in GATE Preparations  65+ Centers across India
© All rights reserved by Gateforum Educational Services Pvt. Ltd. No part of this booklet may be reproduced or utilized in any form without the written permission.

17
|EC| GATE-2017-PAPER-II www.gateforum.com

Take auxiliary equation


ks 2  3  0
3 2
s 30
4
s   j2
  2 rad sec
3
k
4

38. Consider the circuit shown in the figure.

 
3i 0

P
1 i0

10V 1 1

1
Q

The Thevenin equivalent resistance (in  ) across P – Q is _________.


Key: (-1.01 to -0.99)
V
Exp: To find R th   
I io

Here i o 
V
V 1 
 V
1
Nodal at V
V  3i o V
 I0 1
I I
V  3V  V  I  0
R th  1

39. The transfer function of a causal LTI system is H(s) = 1/s. If the input to the system is x(t) =
[sin(t) / t]u(t), where u(t) is a unit step function, the system output y(t) as t   is ______.
Key: (0.45 to 0.55)
sin t u  t 
Exp: Given x  t  
t
By using frequency integration property,
xt s

L
  X1  u du
t 

1
Consider x1  t   sin t u  t    X1  s 
L

s 1
2

 1   1
s
 2  du  tan  s 
 
u 1 2

 ICP–Intensive Classroom Program  eGATE-Live Internet Based Classes DLP  TarGATE-All India Test Series
Leaders in GATE Preparations  65+ Centers across India
© All rights reserved by Gateforum Educational Services Pvt. Ltd. No part of this booklet may be reproduced or utilized in any form without the written permission.

18
|EC| GATE-2017-PAPER-II www.gateforum.com

1  
 L  x  t     tan 1  s   X  s 
 2 
1 1
 Y  s   X  s  H  j   tan 1 s
2s s
By using final value theorem,
1 1  1
lim y  t   lim sY  s   lim   tan 1  s  
t  s 0

s 0 2 2  2

z2  1 z
40. An integral I over a counter clock wise circle C is given by I   e dz.
C z2  1
If C is defined as |z| = 3, then the value of I is
(A) i sin 1 (B) 2i sin 1 (C) 3i sin 1 (D) 4i sin 1
Key: (D)
 z2  1  z
Exp:  C  z 2  1 e dz
I

 z2  1  z  z2  1 
Consider f  z   ez  2   e  
 z 1   z  i  z  i  
 z   i are simple poles of f(z) which lie inside |z|=3

Residue of f(z) at z  i  lim  z  i   e z
 z 2  1   iei
z i   z  i  z  i  
 

& Residue of f(z) at z  i  lim  z  i  e
 z  z 2  1 
  ie  i
z  i   z  i  z  i  
 

 z2  1  z
 By residue theorem, I     e dz  2i  ie  ie   2  e  e 
i i i i

C
z 1
2

 ei  e  i 
 4i    4isin 1
 2i 

41. Consider a binary memory less channel characterized by the transition probability diagram
shown in the figure.
0.25
0 0
0.25

0.75
1 1
0.75
The channel is
(A) Lossless (B) Noiseless (C) Useless (D) Deterministic
Key: (C)

 ICP–Intensive Classroom Program  eGATE-Live Internet Based Classes DLP  TarGATE-All India Test Series
Leaders in GATE Preparations  65+ Centers across India
© All rights reserved by Gateforum Educational Services Pvt. Ltd. No part of this booklet may be reproduced or utilized in any form without the written permission.

19
|EC| GATE-2017-PAPER-II www.gateforum.com

Exp: It is a useless channel as


MAP criteria cannot decide anything on receiving „0‟ we cannot decide what is transmitted.

42. An abrupt pn junction (located at x = 0) is uniformly doped on both p and n sides. The width
of the depletion region is W and the electric field variation in the x-direction is E(x). Which of
the following figures represents the electric field profile near the pn junction?
(A) Ex (B) Ex
n  side p  side
n  side p  side
 0, 0
 0, 0 x x
W W

Ex
(C) (D) Ex
n  side p  side
W x n  side p  side
W
 0,0 x
 0,0

Key: (A)
Exp: If left side is p-region and right side is n-region then electric field triangle will be down
warded and if the left side is n-region and right side is p-region, then electric field triangle will
be upward.

43. A second – order LTI system is described by the following state equations,
d
x1  t   x 2  t   0
dt
d
x 2  t   2x1  t   3x 2  t   r  t 
dt
Where x1  t  and x 2  t  are the two state variables and r(t) denotes the input. The output c(t) =
x1(t). The system is.
(A) Undamped (oscillatory) (B) Under damped
(C) Critically damped (D) Over damped
Key: (D)
Exp: x 1 (t)  x 2 (t)
sX1 (s)  X 2 (s)  1
x 2 (t)  2 x1 (t)  3x 2 (t)  r(t)
sX 2 (s)  2X1 (s)  3X 2 (s)  R(s)
s2  2  3s  X1 (s)  R(s)
R(s)
C(s)  X1 (s)  2
s  3s  2
C(s) 1

R(s) (s  1)(s  2)
system is over damped

 ICP–Intensive Classroom Program  eGATE-Live Internet Based Classes DLP  TarGATE-All India Test Series
Leaders in GATE Preparations  65+ Centers across India
© All rights reserved by Gateforum Educational Services Pvt. Ltd. No part of this booklet may be reproduced or utilized in any form without the written permission.

20
|EC| GATE-2017-PAPER-II www.gateforum.com

44. Consider the parallel combination of two LTI systems shown in the figure.

h1  t 

x t  yt

h2  t 

The impulse responses of the systems are


h1  t   2  t  2   3  t  1
h2  t     t  2.
If the input x(t) is a unit step signal, then the energy of y(t) is ____________.
Key: (7.0 to 7.0)
Exp: Since h1  t  and h 2  t  are connected in parallel the resultant system can be given as follows.

xt yt
h1  t   h 2  t 

 y  t   x  t  * h1  t   h 2  t 

From the given h1 (t) & h 2 (t) y(t)


h1  t   h 2  t   2  t  2  3  t  1    t  2 2
xt  ut
2
y  t   2u  t  2   3u  t  1  u  t  2 
2  1 t
1 2
1
 Energy of y  t     2 dt    1 dt  4 1  13  7
2 2

2 1

45. Assuming that transistors M1 and M2 are identical and have a threshold voltage of 1V, the state
of transistors M1 and M2 are respectively. 3V
(A) Saturation, Saturation
2.5V M2
(B) Linear, Linear

(C) Linear, Saturation 2V M1


(D) Saturation, Linear

Key: (C)
Exp: If VD  VG  VTH , then transistor is working in saturation region.
So, For M2 transistor
VD2  VG2  VTH
3V   2.5  1 V
Assume that M1 is working in saturation, so that

 ICP–Intensive Classroom Program  eGATE-Live Internet Based Classes DLP  TarGATE-All India Test Series
Leaders in GATE Preparations  65+ Centers across India
© All rights reserved by Gateforum Educational Services Pvt. Ltd. No part of this booklet may be reproduced or utilized in any form without the written permission.

21
|EC| GATE-2017-PAPER-II www.gateforum.com

ID1  ID2
VGS1  VTH  VGS2  VTH
2V  VG 2  VS2
 2.5V  VS2
VS2  VD1  0.5V.
Now, for M1, transistor to work in saturation VD1  VG1  VTH , but it is not satisfied by M1
transistor and VG1  VTH , so, transistor M1 is ON but working in linear region.

46. A programmable logic array (PLA) is shown in the figure.


P P Q Q R R

* * *
* * * F
* * *
P

The Boolean function F implemented is


(A) P Q R  P Q R  P Q R (B)  P  Q  R  P  Q  R  P  Q  R 

(C) P Q R  P Q R  P Q R (D)  P  Q  R  P  Q  R  P  Q  R 

Key: (C)
Exp: PQR  PQR  PQR

47. A modulating signal given By x(t) = 5 sin  4103 t  10 cos 2103 t  V is fed to a phase modulator
with phase deviation constant kp=0.5rad/V. If the carrier frequency is 20 kHz, the
instantaneous frequency (in kHz) at t = 0.5 ms is __________
Key: (69.9 to 70.1)
Exp: 
x  t   5sin 4  103 t  10 cos  2  102 t  
Transform theorem frequency
1 d
f1  t   f c  kp. m  t 
2 dt
d
dt
 
m  t   5cos 4  103 t  10 cos  2  103 t  4  103  10sin  2  103 t  .2  103 
 5cos  2  10    4  103  0   20  103 cos 12   20  103
d
mt
dt t  0.5

1
f i  t   20   5  20  70 kHz
2

 ICP–Intensive Classroom Program  eGATE-Live Internet Based Classes DLP  TarGATE-All India Test Series
Leaders in GATE Preparations  65+ Centers across India
© All rights reserved by Gateforum Educational Services Pvt. Ltd. No part of this booklet may be reproduced or utilized in any form without the written permission.

22
|EC| GATE-2017-PAPER-II www.gateforum.com

The minimum value of the function f  x   x  x 2  3 in the int erval  100  x  100 occurs at x =
1
48.
3
________.
Key: (-100.01 to -99.99)
x3
f  x   x  x 2  3 
1
Exp: x
3 3
3x 2
f  x   1  x2 1
3
 x2 1  0
 x  1
f   x   2x

f  1  2  0  at x  1, f  x  has local minimum.

f   1  2  0  at x  1, f  x  has local maximum


1 2
For x = 1, local minimum value  f 1   1 
3 3
Finding f  100   333433.33
f 100   333233.33

 x  100, 100 are end points of interval 


 Minimum occurs at x  100

49. The switch in the circuit, shown in the figure, was open for a long time and is closed at t = 0.
The current i(t) (in ampere) at t = 0.5 seconds is ________

it 5

10A t0
5
2.5H

Key: (8.0 to 8.3)


Exp: At t=0-
5
10A  5 iL  0   5A

i L  O 
At t  0

it
5
5

2.5H  5A

 ICP–Intensive Classroom Program  eGATE-Live Internet Based Classes DLP  TarGATE-All India Test Series
Leaders in GATE Preparations  65+ Centers across India
© All rights reserved by Gateforum Educational Services Pvt. Ltd. No part of this booklet may be reproduced or utilized in any form without the written permission.

23
|EC| GATE-2017-PAPER-II www.gateforum.com

L 2.5 1
    iL     0
R 5 2
2t
i L  5e
i  t   10  5e 2t
At t  0.5s
5
i  0.5   10   8.16A
e

50. In the voltage reference circuit shown in the figure, the op-amp is ideal and the transistors Q1
Q2….., Q32 are identical in all respects and have infinitely large values of common – emitter
current the relation IC=IS exp ((VBE/VT), where Is is the saturation current. Assume that the
voltage VP shown in the figure is 0.7 V and the thermal voltage VT=26mV.

20k 15V
20k

Vout

5k 15V
Vp 

Q1 Q2 Q3 Q32

The output voltage Vout(in volts) is __________.

Key: (1.1 to 1.2)


Exp: KCL at node „a‟ 20k
Vo  Vi Vx  0.7

20 5 5k
VC  Vi  4Vx  2.8 VS 
a
31I Vo
Vo  5Vx  2.8 Vx 
Now, Is e VX VT
 31 s e VS VT

Vx V
 n31  S 20k
VT VT

Vx  VS
 n31
VT
 Vx  0.789V
From equation (i) Vo  5  0.789  2.8  1.145V

 ICP–Intensive Classroom Program  eGATE-Live Internet Based Classes DLP  TarGATE-All India Test Series
Leaders in GATE Preparations  65+ Centers across India
© All rights reserved by Gateforum Educational Services Pvt. Ltd. No part of this booklet may be reproduced or utilized in any form without the written permission.

24
|EC| GATE-2017-PAPER-II www.gateforum.com

51. The state diagram of a finite state machine (FSM) designed to detect an overlapping sequence
of three bits is shown in the figure. The FSM has an input „In‟ and an output „Out‟. The initial
state of the FSM is S0.
In  0
00 Out  0
S0
In  1
In  1 Out  0
Out  0
In  0 01
Out  0 In  0
S1 Out 0

10 In  1
In  0 S2 In  1 Out  0
Out  0 Out  1

11
S3

If the input sequence is 10101101001101, starting with the left-most bit, then the number
times „Out‟ will be 1 is __________.
Key: (4 to 4)
Exp: From the state diagram, let us obtain the transition of states and out when IN channel.
Initial state is
So, the input sea is 10101101001101
When IN  1 then S0  S1 , with out  0
Next IN  0 then S1  Sz with out  0
IN  1 then S1  Sn with out  1
IN  0 then S3  S2 with out  0
IN  1, then S2  S3 with out  1
IN  1, then S3  S1 with out  0
IN  0, then S1  S2 with out  0
IN  1, then S2  S3 with out  1
IN  0, then S3  S2 with out  0
IN  0, then S2  Su with out  0
IN  1, then S0  S1 with out  0
IN  1, then S1  S1 with out  0
IN  0, then S1  S2 with out  0
IN  1, then S2  S3 with out  1
 The ticketed mark now corresponding to output = 1.
So output will be 1 „4‟ times.

52. Standard air – filled rectangular waveguides of dimensions a = 2.29 cm and b= 1.02 cm are
designed for radar applications. It is desired that these waveguides operate only in the
dominatnt TE10 mode but not higher than 95% of the next higher cutoff frequency. The range
of the allowable operating frequency f is.
(A) 8.19 GHz  f  13.1 GHz (B) 8.19 GHz  f  12.45 GHz
(C) 6.55 GHz  f  13.1 GHz (D) 1.64 GHz  f  10.24 GHz
Key: (B)

 ICP–Intensive Classroom Program  eGATE-Live Internet Based Classes DLP  TarGATE-All India Test Series
Leaders in GATE Preparations  65+ Centers across India
© All rights reserved by Gateforum Educational Services Pvt. Ltd. No part of this booklet may be reproduced or utilized in any form without the written permission.

25
|EC| GATE-2017-PAPER-II www.gateforum.com

c 1 3  108 1
Exp: Cut off frequency of TE10 is f c     65.5  108 Hz
2a 2 2.29  10 2
a
Since, b   next higher mode is TE 20
2
c 2
fC    13.1GHz
TE 20
2 a
f  0.95  13.1  12.45GHz

53. For a particular intensity of incident light on a silicon pn junction solar cell, the photocurrent
density (JL) is 2.5 mA/cm2 and the open-circuit voltage (Voc) is 0.451 V. consider thermal
voltage (VT) to be 25mV. If the intensity of the incident light is increased by 20 times,
assuming that the temperature remains unchanged. Voc (in volts) will be ______.
Key: (0.51 to 0.54)
JL 2.5  103
Exp: JS    3.6  1011 A cm 2
  VOC     0.025
0.451 
 
e VT   1 e   1
   
 
Now if the intensity of the light is increased by 20 times it means their photocurrent will also
increased by 20 times.

KT  J L 
Voc  n   1
q  JS 
 20  2.5  103 
25 103 n  11
 1
 3.6  10 
 0.5262 Volt.

54. In the circuit shown, transistors Q1 and Q2 are biased at a collector current of 2.6mA.
Assuming that transistor current gains are sufficiently large to assume collector current equal
to emitter current and thermal voltage of 26 mV, the magnitude of voltage gain V0/Vs in the
mid-band frequency range is _____________ (up to second decimal place).
5V

1k

VO

Q1
VS

Q2
R B2

5V
Key: (49.0 to 51.0)
Exp: a.c equivalent circuit for the given figure

 ICP–Intensive Classroom Program  eGATE-Live Internet Based Classes DLP  TarGATE-All India Test Series
Leaders in GATE Preparations  65+ Centers across India
© All rights reserved by Gateforum Educational Services Pvt. Ltd. No part of this booklet may be reproduced or utilized in any form without the written permission.

26
|EC| GATE-2017-PAPER-II www.gateforum.com

RC 1k
I 2.6  10 3
gm  C   100 m
VT 26  10 3
1
R C  1k; RE  ;
gm

AV 
g m R E

100  1
 50
VS ~ 1
1  gmR E 11 RE
gm
A V  50

55. Two n-channel MOSFETs, T1 and T2, are identical in all respects except that the width of T2
is double that of T1. Both the transistors are biased in the saturation region of operation, but
the gate overdrive voltage (VGS-VTH) of T2 is double that of T1, where VGS and VTH are the
gate – to – source voltage and threshold voltage of the transistors, respectively. If the drain
current and transconductance of T1 are ID1 and gm1 respectively, the corresponding values of
these two parameters for T2 are
(A) 8ID1 and 2gm1 (B) 8ID1 and 4gm1 (C) 4ID1 and 4gm1 (D) 4ID1 and 2gm1

Key: (B)
Exp: Drain current in saturation is
1 W
I D   n Cox  VGS  VTH 
2

2 L
For transistor T1
I 
ID  ID1 and g m  g m1  D1  n Cox  VGs  VTH 
VGS L
For transistor T2
W2  2W1  2W
 VGS  VTh 2  2  VGS  VTh 1  2  VGS  VTh 
1 2W
 2  VGS  VTH    8I D1
2
I D2   n Cox
2 L
ID2 2W
g m2    n Cox  2  VGs  VTh   4g m 1
VGS2 L

General Aptitude
Q. No. 1 - 5 Carry One Mark Each

1. The ninth and the tenth of this month are Monday and Tuesday ___________.
(A) figuratively (B) retrospectively (C) respectively (D) rightfully
Key: (C)

2. 500 students are taking one or more courses out of Chemistry, Physics, and Mathematics.
Registration records indicate course enrolment as follows: Chemistry (329). Physics (186).

 ICP–Intensive Classroom Program  eGATE-Live Internet Based Classes DLP  TarGATE-All India Test Series
Leaders in GATE Preparations  65+ Centers across India
© All rights reserved by Gateforum Educational Services Pvt. Ltd. No part of this booklet may be reproduced or utilized in any form without the written permission.

27
|EC| GATE-2017-PAPER-II www.gateforum.com

Mathematics (295). Chemistry and Physics (83), Chemistry and Mathematics (217), and
Physics and Mathematics (63). How many students are taking all 3 subjects?
(A) 37 (B) 43 (C) 47 (D) 53
Key: (D)
Exp: Given
A  x 2  83 ... 1
A  y 2  63 ...  2  C  329  P 186 
A  x 3  217 ...  3 y1
x1 x2
And A
x3 y2
x1  x 2  A  x 3  329 ...(4)
z1
x 2  A  y1  y 2  186 ...(5) M  295 
x 3  A   y 2  z1  295 ...  6 
x1  x 2  x 3  y1  y 2  z1  A  500 ...(7)
1   2    3  x 2  y 2  x 3  363  3A ...(8)
 4    5   6   3A  2  363  3A    x1  y1  z1   810
 3A  2  363  3A    x1  y1  z1   810  From 8 
 3A  726   500  x 2  x 3  y1  A   810  From  7  
 3A  726  500   363  3A   A  810
 863  A  810  A  53
Alternate method
n  C   329, n  P   186, n  M   295, n  C  P   83;

n  C  M   217,  P  M   63
n  P  C  M   n C  n  P  n M   n C  P  n C  m   n P  M 
n  P  C  M .
 500  329  186  295  83  217  63  n  P  C  m 
 n  P  C  m   500  447  53.

3. It is _________ to read this year‟s textbook __________ the last year‟s.


(A) easier, than (B) most easy, than (C) easier, from (D) easiest, from
Key: (A)

4. Fatima starts from point P, goes North for 3 km, and then East for 4km to reach point Q. She
then turns to face point P and goes 15km in that direction. She then goes North for 6km. How
far is she from point P, and in which direction should she go to reach point P?
(A) 8km, East (B) 12 km, North (C) 6km, East (D) 10km, North
Key: (A)

 ICP–Intensive Classroom Program  eGATE-Live Internet Based Classes DLP  TarGATE-All India Test Series
Leaders in GATE Preparations  65+ Centers across India
© All rights reserved by Gateforum Educational Services Pvt. Ltd. No part of this booklet may be reproduced or utilized in any form without the written permission.

28
|EC| GATE-2017-PAPER-II www.gateforum.com

Exp: The required distance


FP  x  100  36  64 4 km Q
x  8, East
3 km
'x ' 5 km
F
P

15 km
6 km
10

5. A rule states that in order to drink beer one must be over 18 years old. In a bar, there are 4
people. P is 16 years old, Q is 25 years old, R is drinking milkshake and S is drinking beer.
What must be checked to ensure that the rule is being followed?
(A) Only P‟s drink (B) Only P‟s drink and S‟s age
(C) Only S‟s age (D) Only P‟s drink, Q‟s drink and S‟s age
Key: (B)
Exp: For rules to be followed, we need to check P's drink and S's age.

Q. No. 6- 10 Carry Two Marks Each

6. Each of P, Q, R, S, W, X, Y and Z has been married at most once. X and Y are married and
have two children P and Q. Z is the grandfather of the daughter S of P. Z and W are married
and are parents of R. Which one of the following must necessarily be FALSE?
(A) X is the mother-in-law of R (B) P and R are not married to each other
(C) P is a son of X and Y (D) Q cannot be married to R
Key: (D)

7. The number of 3-digit numbers such that the digit 1 is never to the immediate right of 2 is
(A) 781 (B) 791 (C) 881 (D) 891
Key: (C)
Exp: Total no. of 3 digit no‟s = 91010 = 900
The no. of 3-digit numbers in which „1‟ is to the immediate right of 2 = 19
2 1 10 choices
2 1
9 choices 19 choices
 The no. of 3-digit no‟s such that the digit 1 is never to immediate right of 2 is
900  19  881

 ICP–Intensive Classroom Program  eGATE-Live Internet Based Classes DLP  TarGATE-All India Test Series
Leaders in GATE Preparations  65+ Centers across India
© All rights reserved by Gateforum Educational Services Pvt. Ltd. No part of this booklet may be reproduced or utilized in any form without the written permission.

29
|EC| GATE-2017-PAPER-II www.gateforum.com

Alternate method
Total no. of 3 digit numbers are  9 10 10  9 10 10  900.
Numbers with digit 1 is to the immediate right of 2 are

2 1 x  x 2 1  19
1 1 10 9 11
So, no. of 3 digit numbers such that the digit 1 is never to the immediate right of 2 are = 900-
19= 881

8. A contour line joins locations having the same height above the mean sea level. The following is
a contour plot of a geographical region. Contour lines are shown at 25m intervals in this plot.
R

425
450 550
575
Q P S
575
550

500
 T
500
475

0 1 2km
Which of the following is the steepest path leaving from P?
(A) P to Q (B) P to R (C) P to S (D) P to T
Key: (B)
Exp: Closer lines represents steepest path
Alternate method
The steepest path will be the path which is deepest from sea level. So, P to R is the steepest
path.

9. 1200 men and 500 women can build a bridge in 2weeks. 900men and 250 women will take 3
weeks to build the same bridge. How many men will be needed to build the bridge in one week?
(A) 3000 (B) 3300 (C) 3600 (D) 3900
Key: (C)
Exp: Given 1200 Men + 500 Women can build a bridge in 2 weeks. And
900 Men + 250 Women will take 3 weeks to build the same bridge
 To complete in a week; there are 2400 Men + 1000W required in the first equation and
2700 Men + 750 Women required in the second equation.
 2400 M + 1000W = 2700M + 750W
6M
 1W 
5
 ICP–Intensive Classroom Program  eGATE-Live Internet Based Classes DLP  TarGATE-All India Test Series
Leaders in GATE Preparations  65+ Centers across India
© All rights reserved by Gateforum Educational Services Pvt. Ltd. No part of this booklet may be reproduced or utilized in any form without the written permission.

30
|EC| GATE-2017-PAPER-II www.gateforum.com

 The no. of men required to build the bridge in one week


 6M 
 2400M  1000    3600 Men
 5 
Alternate method
Let a man can build the bridge in x weeks and a woman can build the bridge in y weeks.

120 500
So,  1 2
x y
900 250
 1 3
x y
By equations  i  and  ii  ; weget
x  3600; y  3000
 A man build the bridge3600 weeks
 Required men  3600 to build in a week.

10. “If you are looking for a history of India, or for an account of the rise and fall of the British
Raj, or for the reason of the cleaving of the subcontinent into two mutually antagonistic parts
and the effects this mutilation will have in the respective section, and ultimately on Asia, you
will not find it in these pages; for though I have spent a lifetime in the country, I lived too near
the seat of events, and was too intimately associated with the actors, to get the perspective
needed for the impartial recording of these matters.”
Which of the following statements best reflects the author‟s opinion?
(A) An intimate association does not allow for the necessary perspective.
(B) Matters are recorded with an impartial perspective.
(C) An intimate association offers an impartial perspective.
(D) Actors are typically associated with the impartial recording of matters.
Key: (A)

 ICP–Intensive Classroom Program  eGATE-Live Internet Based Classes DLP  TarGATE-All India Test Series
Leaders in GATE Preparations  65+ Centers across India
© All rights reserved by Gateforum Educational Services Pvt. Ltd. No part of this booklet may be reproduced or utilized in any form without the written permission.

31
GATE 2018 General Aptitude (GA) Set-5

Q. 1 – Q. 5 carry one mark each.


Q.1 “By giving him the last ______ of the cake, you will ensure lasting _____ in our house
today.”

The words that best fill the blanks in the above sentence are

(A) peas, piece (B) piece, peace


(C) peace, piece (D) peace, peas

Q.2 “Even though there is a vast scope for its ________, tourism has remained a/an _________
area.”

The words that best fill the blanks in the above sentence are

(A) improvement, neglected


(B) rejection, approved
(C) fame, glum
(D) interest, disinterested

Q.3 If the number 715∎423 is divisible by 3 (∎ denotes the missing digit in the thousandths
place), then the smallest whole number in the place of ∎ is _________.

(A) 0 (B) 2 (C) 5 (D) 6

Q.4 1 1 1 1
What is the value of 1 + + + + + ⋯?
4 16 64 256

(A) 2 7 3 4
(B) (C) (D)
4 2 3

Q.5 A 1.5 m tall person is standing at a distance of 3 m from a lamp post. The light from the
lamp at the top of the post casts her shadow. The length of the shadow is twice her height.
What is the height of the lamp post in meters?

(A) 1.5 (B) 3 (C) 4.5 (D) 6

Q. 6 – Q. 10 carry two marks each.

Q.6 Leila aspires to buy a car worth Rs. 10,00,000 after 5 years. What is the minimum amount
in Rupees that she should deposit now in a bank which offers 10% annual rate of interest, if
the interest was compounded annually?

(A) 5,00,000 (B) 6,21,000


(C) 6,66,667 (D) 7,50,000
GA 1/3
GATE 2018 General Aptitude (GA) Set-5

Q.7 Two alloys A and B contain gold and copper in the ratios of 2:3 and 3:7 by mass,
respectively. Equal masses of alloys A and B are melted to make an alloy C. The ratio of
gold to copper in alloy C is ______.

(A) 5:10 (B) 7:13 (C) 6:11 (D) 9:13

Q.8 The Cricket Board has long recognized John’s potential as a leader of the team. However,
his on-field temper has always been a matter of concern for them since his junior days.
While this aggression has filled stadia with die-hard fans, it has taken a toll on his own
batting. Until recently, it appeared that he found it difficult to convert his aggression into
big scores. Over the past three seasons though, that picture of John has been replaced by a
cerebral, calculative and successful batsman-captain. After many years, it appears that the
team has finally found a complete captain.

Which of the following statements can be logically inferred from the above paragraph?
(i) Even as a junior cricketer, John was considered a good captain.
(ii) Finding a complete captain is a challenge.
(iii) Fans and the Cricket Board have differing views on what they want in a captain.
(iv) Over the past three seasons John has accumulated big scores.

(A) (i), (ii) and (iii) only


(B) (iii) and (iv) only
(C) (ii) and (iv) only
(D) (i), (ii), (iii) and (iv)

Q.9 A cab was involved in a hit and run accident at night. You are given the following data
about the cabs in the city and the accident.
(i) 85% of cabs in the city are green and the remaining cabs are blue.
(ii) A witness identified the cab involved in the accident as blue.
(iii) It is known that a witness can correctly identify the cab colour only 80% of the
time.

Which of the following options is closest to the probability that the accident was caused by
a blue cab?

(A) 12% (B) 15% (C) 41% (D) 80%

GA 2/3
GATE 2018 General Aptitude (GA) Set-5
Q.10 A coastal region with unparalleled beauty is home to many species of animals. It is dotted
with coral reefs and unspoilt white sandy beaches. It has remained inaccessible to tourists
due to poor connectivity and lack of accommodation. A company has spotted the
opportunity and is planning to develop a luxury resort with helicopter service to the nearest
major city airport. Environmentalists are upset that this would lead to the region becoming
crowded and polluted like any other major beach resorts.

Which one of the following statements can be logically inferred from the information given
in the above paragraph?

(A) The culture and tradition of the local people will be influenced by the tourists.
(B) The region will become crowded and polluted due to tourism.
(C) The coral reefs are on the decline and could soon vanish.
(D) Helicopter connectivity would lead to an increase in tourists coming to the region.

END OF THE QUESTION PAPER

GA 3/3
GATE 2018 Electronics and Communication Engineering

Q. 1 – Q. 25 carry one mark each.


Q.1 Two identical nMOS transistors M1 and M2 are connected as shown below. The circuit is
used as an amplifier with the input connected between G and S terminals and the output taken
between D and S terminals. Vbias and VD are so adjusted that both transistors are in saturation.
𝜕𝑖𝐷
The transconductance of this combination is defined as 𝑔𝑚 = while the output
𝜕𝑣𝐺𝑆
𝜕𝑣𝐷𝑆
resistance is 𝑟𝑜 = , where 𝑖𝐷 is the current flowing into the drain of M2 . Let 𝑔𝑚1 ,
𝜕𝑖𝐷
𝑔𝑚2 be the transconductances and 𝑟𝑜1 , 𝑟𝑜2 be the output resistances of transistors M1 and
M2 , respectively.

Which of the following statements about estimates for 𝑔𝑚 and 𝑟𝑜 is correct?


(A) 𝑔𝑚 ≈ 𝑔𝑚1 . 𝑔𝑚2 . 𝑟𝑜2 and 𝑟𝑜 ≈ 𝑟𝑜1 + 𝑟𝑜2 .
(B) 𝑔𝑚 ≈ 𝑔𝑚1 + 𝑔𝑚2 and 𝑟𝑜 ≈ 𝑟𝑜1 + 𝑟𝑜2 .
(C) 𝑔𝑚 ≈ 𝑔𝑚1 and 𝑟𝑜 ≈ 𝑟𝑜1 . 𝑔𝑚2 . 𝑟𝑜2.
(D) 𝑔𝑚 ≈ 𝑔𝑚1 and 𝑟𝑜 ≈ 𝑟𝑜2 .

Q.2 In the circuit shown below, the op-amp is ideal and Zener voltage of the diode is 2.5 volts.
At the input, unit step voltage is applied, i.e. 𝑣𝐼𝑁 (𝑡) = 𝑢(𝑡) volts. Also, at 𝑡 = 0, the
voltage across each of the capacitors is zero.

The time 𝑡, in milliseconds, at which the output voltage 𝑣𝑂𝑈𝑇 crosses −10 V is
(A) 2.5 (B) 5 (C) 7.5 (D) 10

Q.3 A good transimpedance amplifier has


(A) low input impedance and high output impedance.
(B) high input impedance and high output impedance.
(C) high input impedance and low output impedance.
(D) low input impedance and low output impedance.

EC 1/17
GATE 2018 Electronics and Communication Engineering

Q.4 Let the input be u and the output be y of a system, and the other parameters are real
constants. Identify which among the following systems is not a linear system:
d3y d2y dy du d 2u
(A)  a1 2  a2  a3 y  b3 u  b2  b1 2 (with initial rest conditions)
dt 3 dt dt dt dt
t
(B) y  t    e t    u   d
0
(C) y  au  b, b  0
(D) y  au

Q.5 The Nyquist stability criterion and the Routh criterion both are powerful analysis tools for
determining the stability of feedback controllers. Identify which of the following statements
is FALSE:
(A) Both the criteria provide information relative to the stable gain range of the system.
(B) The general shape of the Nyquist plot is readily obtained from the Bode magnitude plot
for all minimum-phase systems.
(C) The Routh criterion is not applicable in the condition of transport lag, which can be
readily handled by the Nyquist criterion.
(D) The closed-loop frequency response for a unity feedback system cannot be obtained
from the Nyquist plot.

Q.6 Consider 𝑝(𝑠) = 𝑠 3 + 𝑎2 𝑠 2 + 𝑎1 𝑠 + 𝑎0 with all real coefficients. It is known that its
derivative 𝑝′(𝑠) has no real roots. The number of real roots of 𝑝(𝑠) is
(A) 0 (B) 1 (C) 2 (D) 3

Q.7 In a p-n junction diode at equilibrium, which one of the following statements is
NOT TRUE?
(A) The hole and electron diffusion current components are in the same direction.
(B) The hole and electron drift current components are in the same direction.
(C) On an average, holes and electrons drift in opposite direction.
(D) On an average, electrons drift and diffuse in the same direction.

EC 2/17
GATE 2018 Electronics and Communication Engineering

Q.8 The logic function f ( X , Y ) realized by the given circuit is

VDD

X X

Y Y
f (X , Y )

(A) NOR (B) AND (C) NAND (D) XOR

Q.9 A function 𝐹(𝐴, 𝐵, 𝐶) defined by three Boolean variables A, B and C when expressed as sum
of products is given by
𝐹 = 𝐴̅ ⋅ 𝐵̅ ⋅ 𝐶̅ + 𝐴̅ ⋅ 𝐵 ⋅ 𝐶̅ + 𝐴 ⋅ 𝐵̅ ⋅ 𝐶̅
̅ ̅ ̅
where, 𝐴, 𝐵 , and 𝐶 are the complements of the respective variables. The product of sums
(POS) form of the function F is

(A) 𝐹 = (𝐴 + 𝐵 + 𝐶) ⋅ (𝐴 + 𝐵̅ + 𝐶) ⋅ (𝐴̅ + 𝐵 + 𝐶)
(B) 𝐹 = (𝐴̅ + 𝐵̅ + 𝐶̅ ) ⋅ (𝐴̅ + 𝐵 + 𝐶̅ ) ⋅ (𝐴 + 𝐵̅ + 𝐶̅ )
(C) 𝐹 = (𝐴 + 𝐵 + 𝐶̅ ) ⋅ (𝐴 + 𝐵̅ + 𝐶̅ ) ⋅ (𝐴̅ + 𝐵 + 𝐶̅ ) ⋅ (𝐴̅ + 𝐵̅ + 𝐶) ⋅ (𝐴̅ + 𝐵̅ + 𝐶̅ )
(D) 𝐹 = (𝐴̅ + 𝐵̅ + 𝐶) ⋅ (𝐴̅ + 𝐵 + 𝐶) ⋅ (𝐴 + 𝐵̅ + 𝐶) ⋅ (𝐴 + 𝐵 + 𝐶̅ ) ⋅ (𝐴 + 𝐵 + 𝐶)

EC 3/17
GATE 2018 Electronics and Communication Engineering

Q.10 The points P, Q, and R shown on the Smith chart (normalized impedance chart) in the
following figure represent:

P Q R

(A) P: Open Circuit, Q: Short Circuit, R: Matched Load


(B) P: Open Circuit, Q: Matched Load, R: Short Circuit
(C) P: Short Circuit, Q: Matched Load, R: Open Circuit
(D) P: Short Circuit, Q: Open Circuit, R: Matched Load

Q.11 Let M be a real 4  4 matrix. Consider the following statements:


S1: M has 4 linearly independent eigenvectors.
S2: M has 4 distinct eigenvalues.
S3: M is non-singular (invertible).
Which one among the following is TRUE?
(A) S1 implies S2 (B) S1 implies S3
(C) S2 implies S1 (D) S3 implies S2

Q.12 ax 2  by 2 f f
Let f ( x, y )  , where a and b are constants. If  at x  1 and y  2 , then
xy x y
the relation between a and b is
b b
(A) a  (B) a  (C) a  2b (D) a  4b
4 2

Q.13 A discrete-time all-pass system has two of its poles at 0.250 and 230 . Which one of the
following statements about the system is TRUE?
(A) It has two more poles at 0.530 and 40 .
(B) It is stable only when the impulse response is two-sided.
(C) It has constant phase response over all frequencies.
(D) It has constant phase response over the entire z-plane.

EC 4/17
GATE 2018 Electronics and Communication Engineering

Q.14 Let 𝑥(𝑡) be a periodic function with period 𝑇 = 10. The Fourier series coefficients for this
series are denoted by 𝑎𝑘 , that is

2𝜋
𝑥(𝑡) = ∑ 𝑎𝑘 𝑒 𝑗𝑘 𝑇 𝑡
𝑘=−∞

The same function 𝑥(𝑡) can also be considered as a periodic function with period 𝑇 ′ = 40.
Let 𝑏𝑘 be the Fourier series coefficients when period is taken as 𝑇′. If ∑∞
𝑘=−∞|𝑎𝑘 | = 16, then

∑𝑘=−∞|𝑏𝑘 | is equal to
(A) 256 (B) 64 (C) 16 (D) 4

Q.15 Consider the following amplitude modulated signal:

𝑠(𝑡) = cos(2000 𝜋𝑡) + 4 cos(2400 𝜋𝑡) + cos(2800 𝜋𝑡).

The ratio (accurate to three decimal places) of the power of the message signal to the power
of the carrier signal is __________.

Q.16 Consider a binary channel code in which each codeword has a fixed length of 5 bits. The
Hamming distance between any pair of distinct codewords in this code is at least 2. The
maximum number of codewords such a code can contain is _________.

Q.17 A binary source generates symbols 𝑋 ∈ {−1,1} which are transmitted over a noisy channel.
The probability of transmitting 𝑋 = 1 is 0.5. Input to the threshold detector is 𝑅 = 𝑋 + 𝑁.
The probability density function 𝑓𝑁 (𝑛) of the noise 𝑁 is shown below.

𝑓𝑁 (𝑛)

0.5

−2 2 𝑛

If the detection threshold is zero, then the probability of error (correct to two decimal
places) is __________.

Q.18 A p-n step junction diode with a contact potential of 0.65 V has a depletion width of 1μm
at equilibrium. The forward voltage (in volts, correct to two decimal places) at which this
width reduces to 0.6 μm is _______.

EC 5/17
GATE 2018 Electronics and Communication Engineering

Q.19 A traffic signal cycles from GREEN to YELLOW, YELLOW to RED and RED to GREEN.
In each cycle, GREEN is turned on for 70 seconds, YELLOW is turned on for
5 seconds and the RED is turned on for 75 seconds. This traffic light has to be implemented
using a finite state machine (FSM). The only input to this FSM is a clock of 5 second period.
The minimum number of flip-flops required to implement this FSM is _______.

Q.20 There are two photolithography systems: one with light source of wavelength λ1 = 156 nm
(System 1) and another with light source of wavelength λ2 = 325 nm (System 2). Both
photolithography systems are otherwise identical. If the minimum feature sizes that can be
realized using System1 and System2 are Lmin1 and Lmin2 respectively, the ratio Lmin1/Lmin2
(correct to two decimal places) is__________.

Q.21 A lossy transmission line has resistance per unit length R  0.05  / m . The line is
distortionless and has characteristic impedance of 50 . The attenuation constant (in Np/m,
correct to three decimal places) of the line is _______.

Q.22  k 2k  x 
Consider matrix A   2 2
and vector x   1  . The number of distinct real values
k  k k   x2 
of k for which the equation Ax  0 has infinitely many solutions is _______.

Q.23 Let X 1 , X 2 , X 3 and X 4 be independent normal random variables with zero mean and unit
variance. The probability that X 4 is the smallest among the four is _______.

Q.24 𝑡2
𝑥 −( )
Taylor series expansion of 𝑓(𝑥) = ∫0 𝑒 2 𝑑𝑡 around 𝑥 = 0 has the form

𝑓(𝑥) = 𝑎0 + 𝑎1 𝑥 + 𝑎2 𝑥 2 + ⋯

The coefficient 𝑎2 (correct to two decimal places) is equal to _______.

EC 6/17
GATE 2018 Electronics and Communication Engineering

Q.25 The ABCD matrix for a two-port network is defined by:

V1   A B   V2 
 I   C D    I 
 1   2

I1 2 2 I2
 

V1 5 V2
 

The parameter B for the given two-port network (in ohms, correct to two decimal places)
is _______.

EC 7/17
GATE 2018 Electronics and Communication Engineering

Q. 26 – Q. 55 carry two marks each.

Q.26 The circuit shown in the figure is used to provide regulated voltage (5 V) across the 1k
resistor. Assume that the Zener diode has a constant reverse breakdown voltage for a current
range, starting from a minimum required Zener current, I Z min  2 mA to its maximum
allowable current. The input voltage VI may vary by 5% from its nominal value of 6 V. The
resistance of the diode in the breakdown region is negligible.
R

VI 5V 1kΩ

The value of R and the minimum required power dissipation rating of the diode,
respectively, are
(A) 186  and 10 mW (B) 100  and 40 mW

(C) 100  and 10 mW (D) 186  and 40 mW

Q.27 Let c(t )  Ac cos(2 f c t ) and m(t )  cos(2 f mt ). It is given that f c  5 f m . The signal
c(t )  m(t ) is applied to the input of a non-linear device, whose output vo (t ) is related to
the input vi (t ) as vo (t )  avi (t )  bvi (t ) , where a and b are positive constants. The
2

output of the non-linear device is passed through an ideal band-pass filter with center
frequency f c and bandwidth 3 f m , to produce an amplitude modulated (AM) wave. If it is
desired to have the sideband power of the AM wave to be half of the carrier power, then
a b is

(A) 0.25 (B) 0.5 (C) 1 (D) 2

Q.28 Consider a white Gaussian noise process 𝑁(𝑡) with two-sided power spectral density
2
𝑆𝑁 (𝑓) = 0.5 W/Hz as input to a filter with impulse response 0.5𝑒 −𝑡 /2 (where 𝑡 is in
seconds) resulting in output 𝑌(𝑡). The power in 𝑌(𝑡) in watts is
(A) 0.11 (B) 0.22 (C) 0.33 (D) 0.44

EC 8/17
GATE 2018 Electronics and Communication Engineering

Q.29 The state equation and the output equation of a control system are given below:

 4 1.5  2
x  x  u ,
4 0  0
y  1.5 0.625 x .

The transfer function representation of the system is


3s  5 3s  1.875
(A) (B)
s  4s  6
2
s 2  4s  6
4s  1.5 6s  5
(C) (D)
s  4s  6
2
s  4s  6
2

Q.30 Red (R), Green (G) and Blue (B) Light Emitting Diodes (LEDs) were fabricated using p-n
junctions of three different inorganic semiconductors having different band-gaps. The built-
in voltages of red, green and blue diodes are VR, VG and VB, respectively. Assume donor and
acceptor doping to be the same (NA and ND, respectively) in the p and n sides of all the three
diodes.
Which one of the following relationships about the built-in voltages is TRUE?
(A) VR > VG > VB
(B) VR < VG < VB
(C) VR = VG = VB
(D) VR > VG < VB

Q.31 A four-variable Boolean function is realized using 4 1 multiplexers as shown in the


figure.

I0 I0 F (U , V ,W , X )
I1 4 1 I1
VCC 4 1
I2 MUX MUX
I2

I3 S1 S0 I3 S1 S0

U V W X

The minimized expression for F (U , V ,W , X ) is

(A) (U V  U V ) W (B) (U V  U V )(W X  W X )

(C) (U V  U V ) W (D) (U V  U V )(W X  W X )

EC 9/17
GATE 2018 Electronics and Communication Engineering

Q.32 A 2  2 ROM array is built with the help of diodes as shown in the circuit below. Here W0
and W1 are signals that select the word lines and B0 and B1 are signals that are output of the
sense amps based on the stored data corresponding to the bit lines during the read operation.

B0 B1
Sense amps
B0 B1
W0
W0 D00 D01
W1 D10 D11
W1
Bits stored in the ROM Array
VDD

During the read operation, the selected word line goes high and the other word line is in a
high impedance state. As per the implementation shown in the circuit diagram above, what
are the bits corresponding to Dij (where i = 0 or 1 and j = 0 or 1) stored in the ROM?
(A) 1 0 (B) 0 1 (C) 1 0 (D) 1 1
0 1 1 0 1 0 0 0

Q.33 The distance (in meters) a wave has to propagate in a medium having a skin depth of 0.1 m
so that the amplitude of the wave attenuates by 20 dB, is
(A) 0.12 (B) 0.23 (C) 0.46 (D) 2.3

Q.34 A curve passes through the point  x  1, y  0  and satisfies the differential equation
dy x 2  y 2 y
  . The equation that describes the curve is
dx 2y x

 2
 1  y2 
(A) ln 1  y2   x  1 (B) ln 1    x  1
 x  2  x2 

 y 1  y
(C) ln 1    x  1 (D) ln 1    x  1
 x 2  x

EC 10/17
GATE 2018 Electronics and Communication Engineering

Q.35 For the circuit given in the figure, the voltage VC (in volts) across the capacitor is

(A) 1.25 2 sin(5t  0.25 ) (B) 1.25 2 sin(5t  0.125 )

(C) 2.5 2 sin(5t  0.25 ) (D) 2.5 2 sin(5t  0.125 )

Q.36 For the circuit given in the figure, the magnitude of the loop current (in amperes, correct to
three decimal places) 0.5 second after closing the switch is _______.

1V
1

1 1H

Q.37 A dc current of 26 μA flows through the circuit shown. The diode in the circuit is forward
biased and it has an ideality factor of one. At the quiescent point, the diode has a junction
capacitance of 0.5 nF . Its neutral region resistances can be neglected. Assume that the room
temperature thermal equivalent voltage is 26 mV.

5 sin( t ) mV 100 
V

For   2 106 rad/s , the amplitude of the small-signal component of diode current (in μA,
correct to one decimal place) is _______.

EC 11/17
GATE 2018 Electronics and Communication Engineering

Q.38 An op-amp based circuit is implemented as shown below.

31 k

+15 V
1 k
+ –
A +
1V
+ vo


–15 V

In the above circuit, assume the op-amp to be ideal. The voltage (in volts, correct to one
decimal place) at node A, connected to the negative input of the op-amp as indicated in the
figure is _________.

Q.39 The input 4sinc(2𝑡) is fed to a Hilbert transformer to obtain 𝑦(𝑡), as shown in the figure
below:

sin(𝜋𝑥) ∞
Here sinc(𝑥) = . The value (accurate to two decimal places) of ∫−∞|𝑦(𝑡)|2 𝑑𝑡
𝜋𝑥
is _______.

Q.40 1 3
A random variable 𝑋 takes values −0.5 and 0.5 with probabilities and , respectively.
4 4
The noisy observation of 𝑋 is 𝑌 = 𝑋 + 𝑍, where 𝑍 has uniform probability density over
the interval (−1, 1). 𝑋 and 𝑍 are independent. If the MAP rule based detector outputs 𝑋̂ as

−0.5, 𝑌<𝛼
𝑋̂ = {
0.5, 𝑌 ≥ 𝛼,

then the value of 𝛼 (accurate to two decimal places) is _______.

EC 12/17
GATE 2018 Electronics and Communication Engineering

Q.41 For a unity feedback control system with the forward path transfer function

K
G s 
s  s  2

The peak resonant magnitude M r of the closed-loop frequency response is 2. The


corresponding value of the gain K (correct to two decimal places) is _________.

Q.42 The figure below shows the Bode magnitude and phase plots of a stable transfer function
𝑛0
𝐺 (𝑠 ) = 3 2 .
𝑠 +𝑑2 𝑠 +𝑑1 𝑠+𝑑0

Consider the negative unity feedback configuration with gain 𝑘 in the feedforward path.
The closed loop is stable for 𝑘 < 𝑘0 . The maximum value of 𝑘0 is ______.

Q.43 In the circuit shown below, the (𝑊/𝐿) value for M2 is twice that for M1. The two nMOS
transistors are otherwise identical. The threshold voltage 𝑉𝑇 for both transistors is 1.0V.
Note that 𝑉𝐺𝑆 for M2 must be > 1.0 V.

Current through the nMOS transistors can be modeled as


𝑊 1 2
𝐼𝐷𝑆 = 𝜇𝐶𝑜𝑥 ( ) ((𝑉𝐺𝑆 − 𝑉𝑇 )𝑉𝐷𝑆 − 𝑉𝐷𝑆 ) for 𝑉𝐷𝑆 ≤ 𝑉𝐺𝑆 − 𝑉𝑇
𝐿 2

𝑊
𝐼𝐷𝑆 = 𝜇𝐶𝑜𝑥 ( ) (𝑉𝐺𝑆 − 𝑉𝑇 )2 /2 for 𝑉𝐷𝑆 ≥ 𝑉𝐺𝑆 − 𝑉𝑇
𝐿

The voltage (in volts, accurate to two decimal places) at 𝑉𝑥 is _______.

EC 13/17
GATE 2018 Electronics and Communication Engineering

Q.44 A solar cell of area 1.0 cm 2 , operating at 1.0 sun intensity, has a short circuit current of
20 mA, and an open circuit voltage of 0.65 V. Assuming room temperature operation and
thermal equivalent voltage of 26 mV, the open circuit voltage (in volts, correct to two
decimal places) at 0.2 sun intensity is _______.

Q.45 A junction is made between p Si with doping density NA1 = 1015 cm3 and p Si with doping
density NA2 = 1017 cm3.

Given: Boltzmann constant k = 1.38 x 1023 JK1, electronic charge q = 1.6 x 1019 C.
Assume 100% acceptor ionization.

At room temperature (T = 300K), the magnitude of the built-in potential (in volts, correct to
two decimal places) across this junction will be _________________.

Q.46 In the circuit shown below, a positive edge-triggered D Flip-Flop is used for sampling input
data 𝐷𝑖𝑛 using clock 𝐶𝐾. The XOR gate outputs 3.3 volts for logic HIGH and 0 volts for
logic LOW levels. The data bit and clock periods are equal and the value of Δ𝑇/𝑇𝐶𝐾 = 0.15,
where the parameters Δ𝑇 and 𝑇𝐶𝐾 are shown in the figure. Assume that the Flip-Flop and the
XOR gate are ideal.

If the probability of input data bit (𝐷𝑖𝑛 ) transition in each clock period is 0.3, the average
value (in volts, accurate to two decimal places) of the voltage at node 𝑋, is _______.

EC 14/17
GATE 2018 Electronics and Communication Engineering

Q.47 The logic gates shown in the digital circuit below use strong pull-down nMOS transistors for
LOW logic level at the outputs. When the pull-downs are off, high-value resistors set the
output logic levels to HIGH (i.e. the pull-ups are weak). Note that some nodes are
intentionally shorted to implement “wired logic”. Such shorted nodes will be HIGH only if
the outputs of all the gates whose outputs are shorted are HIGH.

The number of distinct values of 𝑋3 𝑋2 𝑋1 𝑋0 (out of the 16 possible values) that give
𝑌 = 1 is _______.

Q.48 The cutoff frequency of TE01 mode of an air filled rectangular waveguide having inner
dimensions a cm  b cm ( a  b ) is twice that of the dominant TE10 mode. When the
waveguide is operated at a frequency which is 25% higher than the cutoff frequency of the
dominant mode, the guide wavelength is found to be 4 cm. The value of b (in cm, correct to
two decimal places) is _______.

Q.49 A uniform plane wave traveling in free space and having the electric field

𝐸⃗ = (√2𝑎̂𝑥 − 𝑎̂𝑧 ) cos[6√3𝜋 × 108 𝑡 − 2𝜋(𝑥 + √2𝑧)] 𝑉/𝑚

is incident on a dielectric medium (relative permittivity > 1, relative permeability = 1) as


shown in the figure and there is no reflected wave.
z
Free space
Dielectric
Medium (r >1)

The relative permittivity (correct to two decimal places) of the dielectric medium is
___________.

EC 15/17
GATE 2018 Electronics and Communication Engineering

Q.50 The position of a particle 𝑦(𝑡) is described by the differential equation:

𝑑2𝑦 𝑑𝑦 5𝑦
= − − .
𝑑𝑡 2 𝑑𝑡 4
𝑑𝑦
The initial conditions are 𝑦(0) = 1 and | = 0. The position (accurate to two
𝑑𝑡 𝑡=0
decimal places) of the particle at 𝑡 = 𝜋 is _______.

Q.51 The contour C given below is on the complex plane z  x  jy , where j  1 .


C y

1 1 x

1 dz
 j C
The value of the integral is _______.
z 1
2

Q.52 Let 𝑟 = 𝑥 2 + 𝑦 − 𝑧 and 𝑧 3 − 𝑥𝑦 + 𝑦𝑧 + 𝑦 3 = 1. Assume that 𝑥 and 𝑦 are independent


𝜕𝑟
variables. At (𝑥, 𝑦, 𝑧) = (2, −1,1), the value (correct to two decimal places) of is
𝜕𝑥
_________ .

Q.53 Consider the network shown below with 𝑅1 = 1Ω, 𝑅2 = 2Ω and 𝑅3 = 3Ω. The network is
connected to a constant voltage source of 11V.

The magnitude of the current (in amperes, accurate to two decimal places) through the
source is _______.

EC 16/17
GATE 2018 Electronics and Communication Engineering

Q.54 A band limited low-pass signal 𝑥(𝑡) of bandwidth 5 kHz is sampled at a sampling rate 𝑓𝑠 .
The signal 𝑥(𝑡) is reconstructed using the reconstruction filter 𝐻(𝑓) whose magnitude
response is shown below:

The minimum sampling rate 𝑓𝑠 (in kHz) for perfect reconstruction of 𝑥(𝑡) is _______.

Q.55 Let 𝑋[𝑘] = 𝑘 + 1, 0 ≤ 𝑘 ≤ 7 be 8-point DFT of a sequence 𝑥[𝑛],

−𝑗2𝜋𝑛𝑘/𝑁
where 𝑋[𝑘] = ∑𝑁−1
𝑛=0 𝑥[𝑛]𝑒 .

The value (correct to two decimal places) of ∑3𝑛=0 𝑥[2𝑛] is ______.

END OF THE QUESTION PAPER

EC 17/17
EC-GATE 2018

Section-I: General Ability

1. “By giving him the last ______of the cake, you will ensure lasting ____in our house today.”
The words that best fill the blanks in the above sentence are
(A) peas, piece (B) piece, peace (C) peace, piece (D) peace, peas
Key: (B)

2. “Even though there is a vast scope for its _____, tourism has remained a/an____ area.”
The words that best fill the blanks in the above sentence are
(A) improvement, neglected (B) rejection, approved
(C) fame, glum (D) interest, disinterested
Key: (A)

1 1 1 1
3. What is the value of 1      ....?
4 16 64 256
7 3 4
(A) 2 (B) (C) (D)
4 2 3
Key: (D)
1 1 1 1
Exp: Given series is 1      ........ ...1
4 16 64 256
Clearly, (1) is infinite Geometric series (∴ The first term a =1, common ratio (r)=1/4)
1 1 1 1 1 4
∴ The sum of infinite Geometric series 1      .......  
4 16 64 256 1 3
1
4
a
(∵sum of infinite geometric series  )
1 r

4. If the number 715 ? 423 is divisible by 3 (? Denotes the missing digit in the thousands place), then
the smallest whole number in the place of ? is ______.
(A) 0 (B) 2 (C) 5 (D) 6
Key: (B)
Exp: We know that, if the sum of the digits is a multiple of 3 then the number is divisible by 3
∴ 7  1  5  x  4  2  3  22  x where 'x' is the missing digit
If x=0; then 22 is not multiple of 3
If x=2; then 24 is a multiple of 3

© All rights reserved by Thinkcell Learning Solutions Pvt. Ltd. No part of this booklet may be reproduced or utilized in any form without the written permission.

2
EC-GATE 2018

If x=5; then 27 is a multiple of 3


If x=6; then 28 is not multiple of 3
∴ The minimum value of x=2.

5. A 1.5 m tall person is standing at a distance of 3 m from a lamp post. The light from the lamp at the
top of the post casts her shadow. The length of the shadow is twice the height. What is the height of
the lamp post in meters?
(A) 1.5 (B) 3 (C) 4.5 (D) 6
Key: (B) A
Exp: B  D ( Since lamp post as wellas the man are man height
standing vertical to the ground )
E E Lamp x C
so  ABE  CDE  AA Similarity criterion 
le le height
1.5m
AB BE X 6
     X  3m. B 3m D 3m
E
CD DE 1.5 3

6. A coastal region with unparalleled beauty is home to many species of animals. It is dotted with
coral reefs and unspoilt white sandy beaches. It has remained inaccessible to tourists due to poor
connectivity and lack of accommodation. A company has spotted the opportunity and is planning to
develop a luxury resort with helicopter service to the nearest major city airport. Environmentalists
are upset that this would lead to the region becoming crowded and polluted like any other major
beach resorts.
Which one of the following statements can be logically inferred from the information given in the
above paragraph?
(A) The culture and tradition of the local people will be influenced by the tourists.
(B) The region will become crowded and polluted due to tourism.
(C) The coral reefs are on the decline and could soon vanish.
(D) Helicopter connectivity would lead to an increase in tourists coming to the region.
Key: (B)

7. A cab was involved in a hit and run accident at night. You are given the following data about the
cabs in the city and the accident.
(i) 85% of cabs in the city are green and the remaining cabs are blue.
(ii) A witness indentified the cab involved in the accident as blue.
(iii) It is known that a witness can correctly identify the cab colour only 80% of the time.
Which of the following is closest to the probability that the accident was caused by a blue cab?
(A) 12% (B) 15% (C) 41% (D) 80%

© All rights reserved by Thinkcell Learning Solutions Pvt. Ltd. No part of this booklet may be reproduced or utilized in any form without the written permission.

3
EC-GATE 2018

Key: (A)
Exp: P [ Green cab] = P[G] = 85%, P [Blue cab] =P[B] = 15%
Let A denote that a witness can correctly identify the cab colour as blue.
Given

A A
P    80%, P    20%
B G 
By Baye‟s theorem we have
P  B P  A B
P B A 
P  B P  A B  P G  P  A G 
15%  80%

15%  80%  85%  20%
0.41  41%

8. Leila aspires to buy a car Rs. 10,00,000 after 5 years. What is the minimum amount in Rupees that
she should deposit now in a bank which offers 10% annual rate of interest, if the interest was
compounded annually?
(A) 5,00,000 (B) 6,21,000 (C) 6,66,667 (D) 7,50,000
Key: (B)
Exp: Let Principal = x
Time period = 5 years
R=10% per year
After 5 years, Amount = P + Interest =10,000,00
Principal =x = 2
By compound interest, we have
5
 10 
Amount  P 1   
 100 
110  110  110  110  110 
10,00,000  P  P
121 121  11
       
100  100  100  100  100  100000
10,000,00 1,00,000
P  6, 20,921  6, 21,000
12112111

9. Two alloys A and B contain gold and copper in the ratios of 2:3 and 3:7 by mass, respectively.
Equal masses of alloys A and B are melted to make an alloy C. The ratio of gold to copper in alloy
C is _____.
(A) 5:10 (B) 7:13 (C) 6:11 (D) 9:13

© All rights reserved by Thinkcell Learning Solutions Pvt. Ltd. No part of this booklet may be reproduced or utilized in any form without the written permission.

4
EC-GATE 2018

Key: (B)
Exp: Given
A B
2:3 3:7
   
G C G C
Total parts in A =5 Total parts in B =10
The part of gold in A=2/5 The part of gold in B=3/10
The part of copper in A =3/5 The part of copper in B =7/10
We assume that, the equal quantities of A & B = L.C.M of 5 and 10 =10.

∴ The required ratio of gold & copper in C is


G : C
2 3 3 7  4  3 : 6  7  7 :13
10  10  : 10   10 

5  10 5  10
A B A B

10. The Cricket Board has long recognized John‟s potential as a leader of the team. However, his on-
field temper has always been a matter of concern for them since his junior days. While this
aggression has filled stadia with die-hard fans, it has taken a toll on his own batting. Until recently,
it appeared that he found it difficult to convert his aggression into big scores. Over the past three
seasons though, that picture of John has been replaced by a cerebral, calculative and successful
batsman-captain. After many years, it appears that the team has finally found a complete captain.
Which of the following statements can be logically inferred from the above paragraph?
(i) Even as a junior cricketer, John was considered a good captain.
(ii) Finding a complete captain is challenge.
(iii) Fans and the Cricket Board have differing views on what they want in a captain.
(iv) Over the past three seasons John has accumulated big scores.
(A) (i), (ii) and (iii) only (B) (iii) and (iv) only
(C) (ii) and (iv) only (D) (i), (ii), (iii) and (iv)
Key: (C)

© All rights reserved by Thinkcell Learning Solutions Pvt. Ltd. No part of this booklet may be reproduced or utilized in any form without the written permission.

5
EC-GATE 2018

Section-II: Electronics & Communication Engineering

1. A binary source generates symbols X {1,1} which are transmitted over a noisy channel. The
probability of transmitting X = 1 is 0.5. Input to the threshold detector is R = X + N. The probability
density function fN(n) of the noise N is shown below.

f N (n)

0.5

2 2 n

If the detection threshold is zero, then the probability of error (correct to two decimal places)
is____.

Key: (0.125)
Exp: Given
Px  1  Px  1  0.5, threshold is 0V
When x =1 is transmitted, error is going to take place if received voltage is less than 0V.
When x= -1 is transmitted, error is going is take place if received voltage is greater than 0V.
 Perror  PR  0 x 1 .Px  1  PR  0 x 1 .Px  0
Given that R= X+M, where M is a Random noise
 Perror  Px  N  0 x 1 Px  1  P R  0 x 1 P x  1
 PM  1.Px  1  P M  1 P x  1
Using pdf figure,
fM  x 

0.5
PM  1
0.25 0.25 PM  1

2 1 1 2 n

Shaded portion represent the PM  1 and PM  1


1 1 1 
 Pe   1 0.25    1 0.25   0.125
2 2 2 2

© All rights reserved by Thinkcell Learning Solutions Pvt. Ltd. No part of this booklet may be reproduced or utilized in any form without the written permission.

6
EC-GATE 2018

2. In the circuit shown below, the op-amp is ideal and Zener voltage of the diode is 2.5 volts. At the
input, unit step voltage is applied, i.e.,  IN (t)  u(t) volts. Also, at t = 0, the voltage across each of
the capacitors is zero.
1F

1k 1F

v1N  t  
1V vOUT (t)
0V
t 0

The time t, in milliseconds, at which the output voltage OUT crosses  10V is
(A) 2.5 (B) 5 (C) 7.5 (D) 10

Key: (C)
Exp: Given, VIN  t   u  t  , also, t = 0 voltage across each capacitor is zero.

Vz  2.5V 1F

C.VC 1m t
iC  ; VC 
t 1 1k 1F

Voltage across at second capacitor is 2.5, v1N  t   vOUT (t)

By K.V.L, VC1  2.5  V0  0, given V0  10V


VC1  7.5
1
t  7.5  7.5msec
1m

3. Consider the following amplitude modulated signal:


s(t)  cos(2000 t)  4cos(2400 t)  cos(2800 t)
The ratio (accurate to three decimal places) of the power of the message signal to the power of the
carrier signal is____.
Key: (*)
Exp: Given,
cos  2000 t  4cos  2400t  cos  2800t 
st   
side band carrier side band
 4cos  2400t   2cos  400  t  cos  2400  t 

© All rights reserved by Thinkcell Learning Solutions Pvt. Ltd. No part of this booklet may be reproduced or utilized in any form without the written permission.

7
EC-GATE 2018

 1 
 4 1  cos  400 t   cos  2400t 
 2 
Now at this point, some data is missing,
if we compare this with standard AM signal equation
s  t   A c 1  k a m  t   cos  2400 t 
if k a  1 2, m  t   cos  400t  , PM  1 2
1
if k a  1, m  t   cos  400t  , PM  1 8
2
1
if k a  2, m  t   cos  400t  , PM  1 32
4
Like this infinite possibilities exist
Hence many answers are possible.

4. A p-n step junction diode with a contact potential of 0.65 V has a depletion width of 1m at
equilibrium. The forward voltage (in volts, correct to two decimal places) at which this width
reduces to 0.6 m is____.
Key: (0.416)
Exp: We know that in pn-junction W  Vbi  Vf

Given, W1  1m , Vf  0
W2  0.6m, Vf  ?
W1 5 0.65
 
W2 3 0.65  x
25 25
 0.65  x  0.65
9 9
x  0.416

5. The points P, Q, and R shown on the Smith chart (normalized impedance chart) in the following
figure represent:
(A) P: Open Circuit, Q: Short Circuit, R: Matched Load
(B) P: Open Circuit, Q: Matched Load, R: Short Circuit
(C) P: Short Circuit, Q: Matched Load, R: Open Circuit P Q R

(D) P: Short Circuit, Q: Open Circuit, R: Matched Load


Key: (C)
Exp: Constant resistance circle are given by
2 2
 r   1 
x   y 
2

 1  r  1 r 

© All rights reserved by Thinkcell Learning Solutions Pvt. Ltd. No part of this booklet may be reproduced or utilized in any form without the written permission.

8
EC-GATE 2018

 r  1
center  ,0  , radius  , r is normalized resistance constant reactance circle are given by
1 r  1 r
2
y
 x  1   y    2 , x in normalized reactance
2 1 1
r0 r 1
 x x
r 5
 P is short circuit R in open circuit, Q is matched load. P Q R x
At point P, normalized reactance is zero r
At point Q, normalized reactance is zero
At point R, normalized reactance is zero

6. A lossy transmission line has resistance per unit length R = 0.05  / m. The line is distortionless and
characteristic impedance of 50 . The attenuation constant (in Np/m, correct to three decimal
places) of the line is_____.
Key: (0.001)
Exp: Given R  0.05  m.
Condition for distortion less transmission line
R L
RC LG  
G C
R
  50  50  R  G
G
L L
 z o  50   502
C C
Alternation constant  RG
0.05  0.05 0.05
   0.001
50  50 50

7. Three are two photolithography systems: one with light source of wavelength 1  156nm (System 1)
and another with light source of wavelength  2  325 nm(System 2). Both photolithography system
are otherwise identical. If the minimum feature sizes that can be realized System 1 and System 2 are
Lmin1 and Lmin 2 respectively, the ratio Lmin 1 / Lmin 2 (correct to two decimal places) is ____.
Key: (0.48)

Exp: CD  K1  L min 
NA
CD is the minimum feature size
NA=Numerical aperture
Lmin1 1 156
  k1  k 2 and NA1  NA2    0.48
Lmin 2 2 325

© All rights reserved by Thinkcell Learning Solutions Pvt. Ltd. No part of this booklet may be reproduced or utilized in any form without the written permission.

9
EC-GATE 2018

8. Let x(t) be a periodic function with period T = 10. The Fourier series coefficients for this series are
denoted by ak, that is
 2


jk t
x(t)  ake T

k  

The same function x(t) can also be considered as periodic function with period T  40. Let b k be
 
 
the Fourier series coefficients when period is taken as T . If k 
a k  16, then k
b k is
equal to
(A) 256 (B) 64 (C) 16 (D) 4
Key: (C)
Exp: Only for case study let us consider the periodic waveform x(t) as a square wave.

x(t)
A

    

t
T  10

T '  40
 2
xt  a e
jk t
T
k
k 

We know ak is the exponential Fourier series coefficient and is given by


T
1
x  t  e jk0 t dt
T 0
ak 

Since the waveform repeat itself, we can say if we evaluate ak using the period, as any integer
multiple of To like 2To, 3To…… the coefficient will not change, we can notice this in the above
integration.
So we can say,
if T=10, the E.F.S coefficient is a k , then

if T=40, the E.F.S coefficient is b k  a k


 
Since, a k  bk  a
k 
k  
k 
bk  16

For further more clarity, if we find the D.C. component (k=0) of x(t), then if we consider
T  10, T  20, T  40, etc, the value of a o remains same.

© All rights reserved by Thinkcell Learning Solutions Pvt. Ltd. No part of this booklet may be reproduced or utilized in any form without the written permission.

10
EC-GATE 2018

9. Let M be a real 4 × 4 matrix. Consider the following statements:


S1: M has 4 linearly independent eigenvectors
S2: M has 4 distinct eigenvalues.
S3: M is non-singular (invertible)
Which one among the following is TRUE?
(A) S1 implies S2 (B) S1 implies S3
(C) S2 implies S1 (D) S3 implies S2
Key: (C)
Exp: We know that if a matrix A has „n‟ distinct eigen values then A has „n‟ linearly independent
eigenvectors.
S2 implies S1.

10. A discrete-time all-pass system has two of its poles at 0.25 0 and 230. Which one of the
following statements about the system is TRUE?

(A) It has two more poles at 0.530 and 40.


(B) It is stable only when the impulse response is two-sided.
(C) It has constant phase response over all frequencies.
(D) It has constant phase response over the entire z-plane.
Key: (B)
Exp: For the given all pass system the 2 poles are located at z  0.250 and z  230

P  230
x

x
P  0.250

If we are interested for stability of the given system, then the associated ROC must include the unit
circle as shown in the figure.
For the above highlighted ROC, with respect to the pole z  0.250 , the ROC is outside the circle.
So its impulse response will be right sided.

© All rights reserved by Thinkcell Learning Solutions Pvt. Ltd. No part of this booklet may be reproduced or utilized in any form without the written permission.

11
EC-GATE 2018

Similarly for the pole z  230 , the ROC is inside the circle of radius 2, so its impulse response is
left sided, will contain u  n  1 term. So, overally the impulse response should be both sided for
the given stability.
So Option B is Correct.
By observing the pole pattern of an All Pass System, we can comment the location of zeros. But we
never can comment regarding rest of the poles, if present. Hence option A is wrong.
We know for an All Pass System, the magnitude response is constant for all frequency or in entire
z-plane but the phase response changes, Hence option C and option D are wrong.

11. A traffic signal cycles from GREEN to YELLOW. YELLOW to RED and RED to GREEN. In each
cycle, GREEN is turned on for 70 seconds. YELLOW is turned on for 5 seconds and the RED is
turned on for 75 seconds. This traffic light has to be implemented using a finite state machine
(FSM). The only input to this FSM is a clock of 5 second period. The minimum number of flip-
flops required to implement this FSM is____.
Key: (5)
Exp:  Green  70 seconds
Yellow  5 seconds
Red  75 seconds
So to complete one cycle i.e.,
Green  yellow Red Green  yellow  red…….
Then required time is 70+5+75=150 sec.
Since clock period is 5 sec; the number of clock needed for one cycle of 150 sec is 30, so we
can regard the FSM as MOD 30 counter.
2n  30  n min  5
so, minimum 5 flip flops are required.

12. Two identical nMOS transistor M1 and M 2 are connected as shown below. The circuit is used as
amplifier with the input connected between G and S terminals and the output taken between D and
S terminals. Vbias and VD are so adjusted that both transistors are in saturation. The
i
transconductance of this combination is defined as g m  D while the output resistance is
vGS
 v DS
r0  , where iD is the current flowing into the drain of M2. Let g m1 , gm2 be transconductances
i D
and r01 , r02 be the output resistance of transistor M1 and M2, respectively.
Which of the following statements about estimates for gm and r0 is correct?

© All rights reserved by Thinkcell Learning Solutions Pvt. Ltd. No part of this booklet may be reproduced or utilized in any form without the written permission.

12
EC-GATE 2018

(A) g m  g m1g m2 .r02 and r0  r01  r02 . VD D

(B) g m  g m1  g m2 and r0  r01  r02 . Vbias


M2
(C) g m  g m1 and r0  r01.g m2 .r02 .

(D) g m  g m1 and r0  r02 G M1


Key: (C)
S
i v
Exp: g m  D  ro  DS
 rms I D
The given circuit is MOS cascode
i
gm  o
vi
AC equivalent circuit of MOS cascade is
d2 iD

Vgs2
 g m2 vgs2 ro2

d1s 2

Vgs1  vi ro1
g m1vi

Writing a node equation for the node, we have


Vgs2 Vgs2
g m2 Vgs2    g m1vi ...1
ro1 ro2
Observe that the voltage  d1,s 2    v gs2
Equation (1) can be simplified as
 1 1 
 g m2    vgs2  g m vi ...  2 
 ro1 ro2 
1 1
Since g m2  ,
ro1 ro2
g m vgs2  g m1vi ...(3)
Node equation at node d2 is
vgs2  1 
i0  g m2 vgs2    g m2   vgs2
ro2  ro2 
i0 g m2 vgs2 ...  4 

© All rights reserved by Thinkcell Learning Solutions Pvt. Ltd. No part of this booklet may be reproduced or utilized in any form without the written permission.

13
EC-GATE 2018
ix

Vgs2 g m2 vgs2 ro2  v2

s2

ix
ro1

From equation (3) and (4)


i I
gm  0  D gm1
vi vgs
vx vgs
Similarly R 0  
I x ID
Voltage at the source node, which is vgs2 can expressed in terms of i x is
vgs2  i x ro1 and ...1
vx   i x  g m2 vgs2  ro2  i x ro1 ...  2 

Substituting (1) in (2)


v x  i x  ro1  ro2  g m2 ro2 ro1 
R o  ro1  ro2  ro1ro2 g m2

In above expression the last term will dominate, thus R o  gm2 .ro2  ro1

 k 2k  x 
13. Consider matrix A   2 2
and vector x   1  . The number of distinct real values of k for
k  k k  x2 
which the equation Ax = 0 has infinitely many solutions is____.
Key: (2)
Exp: Given that AX=0 has infinitely many solutions.
Then A  0
K 2K
 0
K  K K2
2

 K 3  2K  K 2  K   0
 K 3  2K 3  2K 2  0  K 3  2K 2  0
 K 2  K  2  0  K  0,0, 2
 Number of distinct values of K = 2. [i.e., 0 and 2].

© All rights reserved by Thinkcell Learning Solutions Pvt. Ltd. No part of this booklet may be reproduced or utilized in any form without the written permission.

14
EC-GATE 2018

14. Considered a binary channel code in which each codeword has a fixed length of 5 bits. The
Hamming distance between any pair of distance codewords in this code is at least 2. The maximum
number of codewords such a code can contain is ____.
Key: (16)

 t2 
 
Taylor series expansion of f(x)   e
x
2
15. dt around x = 0 has the form
0

f (x)  a 0  a1x  a 2 x 2  .....


The coefficient a2 (correct to two decimal places) is equal to _____.
Key: (0)
 dt
x
 t2 2
Exp: f x   e
0

 f '  x   e x
2
2

 2x 
 f ''  x   e x      x.e
 x2 2
 f ''  0   0
2
2

 2 
By Taylor‟s series, of f(x) around x=0, we have
x 2 '' f ''  0 
f  x   f  0  x f '  0  f  0   ........ ; where the coefficient of x 2 
2! 2!
But given the coefficient of x 2 in f  x   a 2
f ''  0 
a 2 
2!
0
 a 2   0  f ''  0   0  a 2  0.
2!

16. A good transimpedance amplifier has


(A) low input impedance and high output impedance
(B) high input impedance and high output impedance
(C) high input impedance and low output impedance
(D) low input impedance and low output impedance
Key: (D)
Exp: Trans impedance amplifier contain, input of Norton's circuit and output of Thevenin circuit
Ro

I1

Rs Ri  R m I1 RL
Is 

A good transimpedance amplifier has low input impedance

© All rights reserved by Thinkcell Learning Solutions Pvt. Ltd. No part of this booklet may be reproduced or utilized in any form without the written permission.

15
EC-GATE 2018

17. Consider p(s)  s3  a 2s 2  a1s  a 0 with all real coefficients. It is known that its derivative p'(s) has
no real roots. The number of real roots of p(s) is
(A) 0 (B) 1 (C) 2 (D) 3
Key: (B)
Exp: Given p'(s) has no real roots.
 Discriminant of p'(s) < 0.
 p(s) will have exactly one real root.

18. Let X1 ,X2 , X3 and X4 be independent normal random variables with zero mean and unit variance.
The probability that X4 is the smallest among the four is _____.
Key: (0.25)
Exp: Since X1 , X2 , X3 and X4 are independent normal random variables with zero mean and unit
variance, then
P  X1is the smallest   P  X 2is the smallest   P  X3 is the smallest 
1
 P  X 4 is the smallest    0.25.
4

19. The ABCD matrix for a two-port network I1 2 2 I2


 
is defined by:
 V1   A B   V2  V1 5 V2
 I    C D   I 
 1   2
 

The parameter B for the given two-port network (in ohms, correct to two decimal places) is ____.

Key: (4.8)
Exp: The ABCD matrix equation is
V1  AV2  BI2
I1  CV2  DI2
If we make output port short i.e. V2  0, then
V1
B
I2
Making the output short, the network becomes

© All rights reserved by Thinkcell Learning Solutions Pvt. Ltd. No part of this booklet may be reproduced or utilized in any form without the written permission.

16
EC-GATE 2018

I1
2 X 2


  2I2  

V1  2I2  5 I 2
 2
I2
 5

Writing KVL on the input loop


V1  2I1   2I2   0
 V1  2I1  2I2 ...1
Writing KCL at node X
 2 
I1   I2    I2   0
 5 
 I1  0.4I2  I2
 I1  1.4I2 ... 2 

Putting equation (2) in equation (1)


V1  2  1.4I2   2I2
 2.8I2  2I2
V1
  B  4.8
I 2

20. In a p-n junction diode at equilibrium, which one of the following statements is NOT TRURE?
(A) The hole and electron diffusion current components are in the same direction.
(B) The hole and electron drift current components are in the same direction.
(C) ON an average, holes and electrons drift in opposite direction.
(D) ON an average, electrons drift and diffuse in the same direction.
Key: (D)
Exp: Since the concentration of holes on the p side is much greater than that in the n side, a very large
hole diffusion current tends to flow across the junction from the p to the n material. Hence an
electric field must build up across the junction in such a direction that a hole drift current will tend
to flow across the junction from n to p side in order to counterbalance the diffusion current.

© All rights reserved by Thinkcell Learning Solutions Pvt. Ltd. No part of this booklet may be reproduced or utilized in any form without the written permission.

17
EC-GATE 2018

21. The logic function f(X,Y) realized by the given circuit is


VDD

X   X

Y   Y

f (X,Y)

(A) NOR (B) AND (C) NAND (D) XOR

Key: (D)
Exp: If we focus the N-MOS section
f  X,Y 

X X

Y Y

XY XY

f  X.Y   X Y  XY  X Y XY

22. The Nyquist stability criterion and the Routh criterion both are powerful analysis tools for
determining the stability of feedback controllers. Identify which of the following statements is
FALSE:
(A) Both the criteria provide information relative to the stable gain range of the system.
(B) The general shape of the Nyquist plot is readily obtained from the Bode magnitude plot for all
minimum-phase systems.
(C) The Routh criterion is not applicable in the condition of transport lag, which can be readily
handled by the Nyquist criterion.
(D) The closed-loop frequency response for a unity feedback system cannot be obtained from the
Nyquist plot.

© All rights reserved by Thinkcell Learning Solutions Pvt. Ltd. No part of this booklet may be reproduced or utilized in any form without the written permission.

18
EC-GATE 2018

Key: (B)
Exp:  Analysis of option A
By observing the encirclement about the point -1+j0 in Nyquist plot we find the range of k for
which system will be stable similarly in Routh criterion by observing the sign change we find the
range of k for which system will be stable
Hence option A represents a correct statement.
Analysis of option B
Shape of Nyquist plot can‟t be commented readily form bode plot, first we have to find the
transfer function and by using this we have to draw the Nyquist plot, so readily we can‟t comment.
Hence option B represents a false statement.
Analysis of option C
For transportation log e  s Nyquist plot is suitable as its magnitude is 1 for all frequency and
phase is , so no approximation required but in Routh criterion we need to do approximation
first, so its not suitable.
Hence option C represents a correct statement.
Analysis of option D
With the help of Nyquist plot we cannot comment closed loop frequency response for this we need
M, N circles, Nichols chart etc.
Hence option D represents a correct statement.

23. A function F(A, B, C) defined by three Boolean variables A, B and C when expressed as sum of
products is given by
F  A.B.C A.B.C A.B.C
Where, A, B, and C and the complements of the respective variables. The product of sums (POS)
form of the function F is
(A) F  (A B C). A  B  C. A  B  C 

(B) F  A  B  C. A  B  C. A  B  C

(C) F  (A B C).(A B C).(A B C). A  B  C.(A B C)

(D) f   A  B  C. A  B  C.(A  B  C).(A B C).(A B C)


Key: (C)
Exp: F  A.B.C   ABC  A B C  A B C  m  0,2,4 
 M 1,3,5,6,7 
  A  B  C  A  B  C  A  B  C  A  B  C  A  B  C 

© All rights reserved by Thinkcell Learning Solutions Pvt. Ltd. No part of this booklet may be reproduced or utilized in any form without the written permission.

19
EC-GATE 2018

24. Let the input be u and the output be y of a system, and the other parameters are real constants.
Identify which among the following systems is not a linear system:
d3 y d2 y dy du d2u
(A)  a1  a 2  a 3 y  b3 u  b 2  b1 (with initial rest conditions)
dt 3 dt 2 dt dt dt 2

(B) y(t)   a  (t )u()d


t

(C) y = au + b, b  0
(D) y = au
Key: (C)
Exp: If we observe all the 4 options carefully, we can notice option C (i.e., y  au  b, b  0 ) is non
linear system due to the addition of non-zero constant b, since it does not satisfy homogeneity and
additivity as follows
au1  b  au 2  b   a  u1  u 2   b
a  ku   b  k au  b

Option A is linear differential equation with constant co-efficient, so it is linear.


Integrator system is generally linear, so option B represents a linear system.
y  au, is a perfect linear system.

ax 2  by2 f f
25. Let f(x,y)  , where a and b are constants. If  at x = 1 and y = 2, then the relation
xy x y
between a and b is
b b
(A) a  (B) a  (C) a  2b (D) a = 4b
4 2
Key: (D)
ax 2  by2 ax 2 by2
Exp: Let f  x, y    
xy xy xy
ax by
 f  x, y  

y x
f a by f ax b
   2&  
x y x y y 2 x
f f
Given 
x y x 1,y  2
a by ax b
   2 
y x2 y x x 1,y  2

© All rights reserved by Thinkcell Learning Solutions Pvt. Ltd. No part of this booklet may be reproduced or utilized in any form without the written permission.

20
EC-GATE 2018

a a
  2b    b
2 4
a a
   2b  b
2 4
 2a  a  4  3b 
 3a  12b  a  4b.

26. In the circuit shown below, the (W/L) value for M2 is twice that for M1. The two nMOS transistors
are otherwise identical. The threshold voltage VT for both transistors is 1.0 V.
Note that VGS for M2 must be > 1.0 V.
Current through the nMOS transistors can be modeled as
 W  1 2 
IDS  Cox    VGS  VT  VDS  VDS  for VDS  VGS  VT
 L  2 

W
I DS  Cox    VGS  VT  / 2
2
for VDS  VGS  VT
 L
3.3V

2.0V
M2

Vx

M1

The voltage (in volts, accurate to two decimal places) at Vx is ________.


Key: (0.4266)
 n Cox W
Exp: Let K n   
2  L

W W
Given that    2  
 L 2  L 1
So, K n 2  2K n1
For M1 , VGS1  VT  2  1  1V
For M s , VGS2  VT  2  Vx  1  1V  Vx  1V
VDS2   3.3  Vx    VGS2  VT 

So, M1 will be in linear region and M2 will be in saturation region.

© All rights reserved by Thinkcell Learning Solutions Pvt. Ltd. No part of this booklet may be reproduced or utilized in any form without the written permission.

21
EC-GATE 2018

ID1  ID2
K n1  2  VGS1  VT  VDS1  VDS1   K n 2  VGS2  VT 
2 2

K n1  2  2  1 Vx  Vx2   2K n1  2  Vx  1
2

2Vx  Vx2  2 1  Vx2  2Vx   2Vx2  4Vx  2


2
3Vx2  6Vx  2  0; Vx2  2V  0
3

8
4
Vx  1  3  1  1V
4 3
VGS2   2  Vx   VT  1  Vx 
1
Vx  1   0.4226V
3

27. A junction is made between p- Si with doping density N A1  1015 cm 3 and p Si with doping density
N A 2  1017 cm 3 .
Given: Boltzmann constant k = 1.38 × 10–23 J.K–1, electronic charge q = 1.6 × 10–19 C.
Assign 100% acceptor ionization.
At room temperature (T = 300 K), the magnitude of the built-in potential (in volts, correct to two
decimal places) across this junction will be_____.
Key: (0.1192)
Exp: Setting JP = 0
VT dP
E
P dx
 dP dV 
dV  VT  E   dx 
V1 V2
dx
P 
V21  V2  V1  VT ln  1 
 P2  P1   P2
Built in potential,

kT  N A2  1.38  3
Vbi  n  n 100  V  0.1192V
q  N A1  1.6 100

28. In the circuit shown below, a positive edge-triggered D Flip-Flop is used for sampling input data Din
using clock CK. The XOR gate output 3.3 volts for logic HIGH and 0 volts for logic LOW levels.
The data bit and clock periods are equal and the value of T / TCK  0.15, where the parameters
T and TCK are shown in the figure. Assume that the Flip-Flop and the XOR gate are ideal.

© All rights reserved by Thinkcell Learning Solutions Pvt. Ltd. No part of this booklet may be reproduced or utilized in any form without the written permission.

22
EC-GATE 2018

TCK
X
Din D Q
DFlip-Flop CK
CLK Din
CK
T T T

If the probability of input data bit (Din) transition in each clock period is 0.3, the average value (in
volts, accurate to two decimal places) of the voltage at node X, is_____.

Key: (0.8415)
Exp: If
P1: Probability of Output Vx  1 ON state 
P0: Probability of Output Vx  0  OFF state 
VX1: Voltages corresponding to logic 1 i.e. 3.3V
VX0: Voltage corresponding to logic i.e. 0 V
TON:The period when Output Vx  1
TOFF: The period when Output Vx  0
Then we can say the average value of Vx is

 Vx avg  P1  Vx 1
 
 TON  P0 Vx 0  TOFF 
The above relation is somewhat similar to the average probability of error calculation in digital
modulation Schemes.

 Let us draw the wave form of Vx and find the unknown parameter of the above equation.
 Form the given logic circuit we can say that the Output of the ex or Din and previous state of
the D flip flop, because the elements are given as ideal.
TCLK

Din

 T   T   T   T 
3.3V

 When there 
 T  T  Vx is a transition 
 1  
 TCLK  TCLK  of Din 
© All rights reserved by Thinkcell Learning Solutions Pvt. Ltd. No part of this booklet may be reproduced or utilized in any form without the written permission.

23
EC-GATE 2018

  
  Vx avg   P1 Vx1  TON    P1 Vxo  TOFF 
  
   T     T  
 0.3 3.3  1     0.7  0  
   TCLK     TCLK  
 0.3  3.3  1  0.15    0.7  0 
 0.3  3.3  0.85  0.8415 V.

29. The circuit shown in the figure is used to provide regulated voltage (5V) across the 1k resistor.
Assume that the Zener diode has a constant reverse breakdown voltage for a current range, starting
from a minimum required Zener current, IZmin  2mA to its maximum allowable current. The input
voltage VI may vary by 5% from its nominal value of 6V. The resistance of the diode in the
breakdown region is negligible.
R

1k
VI 5V

The value of R and the minimum required power dissipation rating of the diode, respectively, are
(A) 186  and 10 mW (B) 100  and 40 mW
(C) 100  and10mW (D) 186  and 40mW
Key: (B)
Exp: Vz  6V  5%  6V  0.3V  5.7V to 6.3V
5V
IL   5mA I R I2
1k
Is min   IL  I Z min   5mA
Iz
V  Vz
Is  I ; V1
5V 1k
R
VI min   Vz
Is min    7mA
R max
5.7  5
So, R max   100
7
6.3  5
Is max    13mA
100
Iz max   Is max   IL  13mA
Pz min   Vz Iz max   5  8mW  40mW

© All rights reserved by Thinkcell Learning Solutions Pvt. Ltd. No part of this booklet may be reproduced or utilized in any form without the written permission.

24
EC-GATE 2018

30. For the circuit given in the figure, the magnitude of the loop current (in amperes, correct to three
decimal places) 0.5 second after closing the switch is _____.
1V
  1

1H
1

Key: (0.316)
1V t 0
Exp:   1

assume switching 
1 1H is happening at t  0 
 

We can redraw the above circuit as


t 0
2

 1H
1V
 iL  t 

i L  t   i L     i L  0   i L     e  t /  ; t  0

At t  0 , switch is in open state, so the loop is not closed hence, i L 0  0A  


At t   switch is closed and inductor is short Circuited
2

1V
 1
iL     A
 2

L
Time constant  
R tn 2

R1n  2

© All rights reserved by Thinkcell Learning Solutions Pvt. Ltd. No part of this booklet may be reproduced or utilized in any form without the written permission.

25
EC-GATE 2018

L 1
 , 
R tn 2

 
So we have i L 0  0  i L    
1
2
 
1
2
 i L  t   i L     i L  0   i L     e  t /  ; t  0
1  1  2t
  0  e ; t  0
2  2

 1  e2t  ; t  0
1
2
1 1
 i L  t  0.5  1  e20.5   1  e 1 
2 2
1
 1  0.368  0.5  0.632  0.316amp
2

31. Consider a white Gaussian noise process N(t) with two-sided power spectral density SN(f) = 0.5
W/Hz as input to a filter with impulse response 0.5 e  t / 2 (where t is in seconds) resulting in output
2

Y(t). The power in Y(t) in watts is


(A) 0.11 (B) 0.22 (C) 0.33 (D) 0.44
Key: (B)
Exp: Given, SM  t   0.5 w / Hz. Nt h  t   0.5.e
 t  2 2
yt

Sy  t   H  jt  .SM  t 
2

PSD PSD
Sy  t   0.5 H  t 
2
SM  f  Sy  f 
Power in y  t   area under s y  t 



 s  t  dt
y

 

 0.5 H  t  dt  0.5  H  t  dt
2 2

 

Applying parsevals theorem,


 

 Ht  h t
2 2
dt  dt
 
 2 
 Power in y  t   0.5   0.5e  dt  0.5  0.25 e  t 2 dt
2
t

 
2

 

t2

 1
1 1 2
 0.5  0.25  2 . 
2  1
e 2
dt
2a.
2
 0.22
Gaussian function
Area  1

© All rights reserved by Thinkcell Learning Solutions Pvt. Ltd. No part of this booklet may be reproduced or utilized in any form without the written permission.

26
EC-GATE 2018

32. The position of a particle y(t) is described by the differential equation:


d2 y dy 5y
2
  .
dt dt 4
dy
The initial conditions are y(0) = 1 and  0.
dt t  0

The position (accurate to two decimal places) of the particle at t   is _____.


Key: (-0.21)
d 2 y dy 5y
Exp: Given,   ;
dt 2 dt 4
where y(t) represents the position of a particle
d2 y dy 5
   y  0  1
dt 2 dt 4
 The auxiliary equation of the D.E is
m2  m  5 4  0
1  1  4 1 5 4  1  i  2  1
m    i  complex 
2 1 2 2
 Roots are complex
 The required solution of equation (1) is
1
 t
yt  e 2
C1cost +C2sint    2 
dy
Given y  0   1& 0
dt t 0
From  2  ;if y  0   1; then C1  1
dy
if  0at t  0;
dt
dy  1 
then  e t 2  C1 sin t  C2 cos t   C1 cos t  C 2 sin t   e  t 2 
dt 2 
 1 
 0  C2  C1  
 2
C 1
 C2  1   C1  1
2 2
 The position of the particle at t   is
 1 
y     e t 2 cos t  sin t 
 2  t 
 e 2  1  e 2  0.21

© All rights reserved by Thinkcell Learning Solutions Pvt. Ltd. No part of this booklet may be reproduced or utilized in any form without the written permission.

27
EC-GATE 2018

1 3
33. A random variable X takes value –0.5 and 0.5 with probabilities and , respectively. The noisy
4 4
observation of X is Y = X + Z, where Z has uniform probability density over the interval (–1, 1). X
and Z are independent. If the MAP rule based detector outputs X̂ as
0.5, Y  
X̂  
 0.5, Y  ,
Then the value of  (accurate to two decimal places) is _____.
Key: (-0.75)
1
Exp: Given, P x  0.5  , T x  0.3  3 4 , y  x  m
4
Noise is with uniform density function as shown
fz  g 
12

z
1 1

pdf of y when x  0.5 is transmitted

 0.5 px  0.5  1 4


y
12 f y/x 0.5

y
1.5 x1 0.5

pdf of y when x  0.5 is transmitted

12 fy x0.5  y px  0.5  3 4

y
0.5xg 1.5

In a general rule, threshold shift towards that side where probability is less
Using MAP rule,
x 0.5
P y  x T 0.5  P x  0.5 

x  0.5


P y  xT x T  0.5  Px  0.5
© All rights reserved by Thinkcell Learning Solutions Pvt. Ltd. No part of this booklet may be reproduced or utilized in any form without the written permission.

28
EC-GATE 2018

Equality can be used to find threshold


1 1 1 3
1.5  x T  x  1.5  x T   
2 4 2 4
1.5  x T  4.5  3x T
3  4x T  x T  3 4  0.75

34. The state equation and the output equation of a control system are given below:
 4 1.5  2
x  x    u,
4 0  0
y  1.5 0.625 x.

The transfer function representation of the system is


3s  5 3s  1.875 4s  1.5 6s  5
(A) (B) (C) (D)
s  4s  6
2
s 2  4s  6 s  4s  6
2
s  4s  6
2

Key: (A)
 4 1.5  2
Exp:  A    , B    , C  1.5 0.62512
4 0  22 0  21
 T F  C  sI  A  B
1
 
s  4 1.5
 sI  A   
 4 s 
1  s 1.5
sI  A
1

s  s  4   6  4 s  4 
 s 1.5 
 s 2  4s  6 s 2  4s  6 
 
 4 s4 
 s 2  4s  6 s 2  4s  6  22
 2
  s I  A  B   s I  A    
1 1
  22 0 
21

 2s 
 s 2  4s  6 
 
 8 
 s 2  4s  6  21
 C  s I  A   B
1
 
 2s 
 s2  4s  6 
 1.5 0.62512  
 8 
 s2  4s  6  21

© All rights reserved by Thinkcell Learning Solutions Pvt. Ltd. No part of this booklet may be reproduced or utilized in any form without the written permission.

29
EC-GATE 2018

 3s 5  3s  5
 2  2   2
 s  4s  6 s  4s  6 11 s  4s  6
3s  5
 So the closed loop transfer function is T  s  
s  4s  6
2

35. The figure below shows the Bode magnitude and phase plots of a stable transfer function
n0
G(s)  3
s  d 2s  d1s  d 0
2

G  j

36 dB
20 dB  k G s

0 dB 

G  j

0

180

270

Consider the negative unity feedback configuration with gain k in the feedforward path. The closed
loop is stable for k < k0. The maximum value of k0 is _____.

Key: (0.1)
Exp: Basically here we are supposed to find the value of K  K 0 for which system will be marginally
stable.
We know for marginal stable, pc  gc , if we notice both the magnitude and phase plot
carefully we can say, the frequency of which the phase is –180° in the phase plot the corresponding
gain is 20dB in the magnitude plot.
So for marginal stable, at this frequency, the gain plot should touch 0 db line, so this to happens,
if we select K  0.1  20dB, then each point in the gain plot will be subtracted by + 20dB,
including the point where gain is + 20dB, it becomes 0 dB & the system becomes marginal stable.

© All rights reserved by Thinkcell Learning Solutions Pvt. Ltd. No part of this booklet may be reproduced or utilized in any form without the written permission.

30
EC-GATE 2018

Since K=0.1, it is positive phase plot remains unaffected so no change in pc but gc changed
so that we have pc  gc .

So if K  0.1, pc  gc which is the condition for marginal stable.
 if K  0.1, system stable
K  0.1, unstable
K  0.1, marginal stable

36. The contour C given blow is on the complex plane z  x  jy, where j  1.
y
C

x
1 1

1 dz
 j C z 2  1
The value of the integral is ____.

Key: (2)
1 1
Exp: Let f  z   
z2  1  z  1 z  1
 The singular points of f(z) are z = -1,1 both lies inside „C‟.
By Cauchy‟s integral formula,
1 dz 1 dz C1 C2
  
 j C z  1 j C  z  1 z  1
2

1 1 1 1 1  1 11
   dz   dz 
j  2 C1 z  1 2 C2 z  1 
1 1 1 
   2j  1   2j  1 
j  2 2 
(Since the closed curve „C1‟ is clock wise & 'C2 ' is anti clock wise)
1 2j
  j  j   2
j j
H(f )
37. A band limited low –pass signal x(t) of bandwidth
5 kHz is sampled at a sampling rate fs. The signal K

x(t) is reconstructed using the reconstruction filter


H(f) whose magnitude response is shown below:
The minimum sampling rate fs( in kHz) for perfect 8  6 0 6 8 f (kHz)
reconstruction of x(t) is ____.

© All rights reserved by Thinkcell Learning Solutions Pvt. Ltd. No part of this booklet may be reproduced or utilized in any form without the written permission.

31
EC-GATE 2018

Key: (13)

Exp: It is given that x  t  is a low pass signal, and is band limited to 5 kHz, let its spectrum X  f  is as
follows:
Xf 

5 5 f (kHz)


If we sample x  t  ideally, the sampled signal is Xs  f    X  f  nfs , where fs is the sampling
n 

frequency.
If we think to sample it on critical sampling rate
fs  2f m  10kHz, Then the spectrum of the sampled signal will be
Xs (f )
fs  13kHz

f (kHz)
15 10 8 6 5 0 5 6 8 10 15

If we pass now Xs  f  through the non ideal filter (shown in dotted line), at the output of the filter
along with the original signal, we get some undesired frequency of 5 to 8 kHz.
However, if we select fs  13kHz, then the filter will capture only the desired part, as follows, (this
idea we can get by observing the spectrum when we have selected fs = 10 kHz).

xs  f 

fs  13kHz

f (kHz)
18 13 8 6 5 0 5 6 8 13 18

© All rights reserved by Thinkcell Learning Solutions Pvt. Ltd. No part of this booklet may be reproduced or utilized in any form without the written permission.

32
EC-GATE 2018

38. Consider the network shown below with R1  1, R 2  2 and R 3  3 . The network is connected
to a constant voltage source of 11V.

R1 R1

R2

R1 R1  11V
R3 R3 

R2

R1 R1

The magnitude of the current (in amperes, accurate to two decimal places) through the source is
____.
Key: (8)
I
Exp: By observing the network we can say, it holds a symmetry
and hence we can say Node A and Node B are R1 R1
equipotential. Similarly Node C and Node D are also R2
A B
equipotential.
R3
When 2 nodes are equipotential, the element connected R1 R1  11V
R3 R3 
across these 2 nodes (R2 in this case) can be replaced by
either short circuit, or open circuit or any other value of C D
R2
resistor without affecting the network. R1
R1
If we replace the R2 by short circuit, then the network can
be redrawn as

R eq   R1 R1    R1 R 3 R1 R 3    R1 R1 
I R eq

R1  R1 R 3  R1  R R3 
     R1   1
2 
R1 R1
2 2 2  2 2
1 3
 1    R1 , R 2 , R 3 values are given 
R1 R3 R3 R1 
 11V 2 2
3/4 3 11
 1 1   
4/2 8 8
R1 R1 11 11
I   8A
R eq 11 / 8

© All rights reserved by Thinkcell Learning Solutions Pvt. Ltd. No part of this booklet may be reproduced or utilized in any form without the written permission.

33
EC-GATE 2018

39. A dc current of 26 A flows through the circuit shown. The diode in the circuit is forward biased
and it has an ideality factor of one. At the quiescent point, the diode has a junction capacitance of
0.5 nF. Its neutral region resistances can be neglected. Assume that the room temperature thermal
equivalent voltage is 26 mV.

5sin(t)mV
100 

For   2 106 rad/s, the amplitude of the small-signal component of diode current (in A, correct
to one decimal place) is _____.
Key: (6.4)
nVT 1 26m
Exp: rd   6
 103  1k
ID 26 10
Cd  0.5109 F
rd
A.C equivalent

j
Xd    j103 Cd
C
r .X  j106 103
Z  rd X d  d d  3 
rd  Xd 10  103 j 1  j
5m 5 106 5106
I 
103 0.1 1 0.6  0.5j
0.1103 
1 j 1 j
 6.4 106   39.8  I  6.4A

40. Let X[K]  k  1,0  k  7 be 8-point DFT of a sequence x[n],

Where X[k]  n 0 x[n] e j2nk/ N.


N 1


3
The value (correct to two decimal places) of n 0
x[2n] is _______.

© All rights reserved by Thinkcell Learning Solutions Pvt. Ltd. No part of this booklet may be reproduced or utilized in any form without the written permission.

34
EC-GATE 2018

Key: (3)
Exp: It is given that
X  k   k  1, 0  k  7 for 8 point DFT of x  n 

So X  k    1, 2, 3, 4, 5, 6, 7, 8
  

We are asked to find


3

 x  2n   x  0  x  2  x  4  x  6
n 0

For N-point DFT we know that


N 1

X  0   x  n   Centralordinate
n 0 
N 1 
N n  Propertyof DFT
X     x  n  1
 2  n 0 

If we put N=8 in the above 2 equations


8 1
X  0    x  n   x  0   x 1  x  2   ....x  7 
n 0
8 1
X  4    x  n  1  x  0   x 1  x  2   x  3  ....x  6   x  7 
n

n 0

 X  0   X  4   2  x  0   x  2   x  3  x  6 
1
  x  0   x  2   x  4   x  6    X  0   X  4 
2
3
1
  x  2n   1  5  3
n 0 2

41. For the circuit given in the figure, the voltage VC (in volts) across the capacitor is
100 k



5sin(5t)V 1F VC
 

100 k

(A) 1.25 2 sin(5t  0.25) (B) 1.25 2 sin(5t  0.125)

(C) 2.5 2 sin(5t  0.25) (D) 2.5 5 sin(5t  0.125)

© All rights reserved by Thinkcell Learning Solutions Pvt. Ltd. No part of this booklet may be reproduced or utilized in any form without the written permission.

35
EC-GATE 2018

Key: (C)
Exp: Using sine function as reference phasor, if we transform the given time domain network into its
equivalent phasor domain, it will be as follows
R

 
1
50 VC
 j C

Here R  100k, C  1F and   5rad / sec


Using the voltage division rule for VC we have

 1 / jC   1 
VC    50    50
 2R 1 / jC  1 j2RC 
 1 
6  
 50
1  j 2  5100  10  1 10 
3

 1 
  50
1  j 
 1  5
   45 50    45
 2  2
5 2
   45  2.5 2  0.25
2 2
VC  t   2.5 2 sin 5t  0.25

42. An op-amp based circuit is implemented as shown below.


31k

15V
1k

A 

1V  o

15V 

In the above circuit, assume the op-amp to be ideal. The voltage (in volts, correct to one decimal
place) at node A, connected to the negative input of the op-amp as indicated in the figure is____.

© All rights reserved by Thinkcell Learning Solutions Pvt. Ltd. No part of this booklet may be reproduced or utilized in any form without the written permission.

36
EC-GATE 2018

Key: (0.5)
R f  31k
R f 31k
Exp: Given V0    31
R 1k
Vsat  15V and Vsat  15V 15V
As V0   Vsat  V0  15V R  1k

Applying KVL at point A VA Vo

VA  1 VA  15 1V
 0
1 31 15V
31VA  31  VA  15  0
16
32VA  16  VA   0.5V
32

43. A solar cell of area 1.0 cm2, operating at 1.0 sun intensity, has a short circuit current of 20 mA, and
an open circuit voltage of 0.65 V. Assuming room temperature operation and thermal equivalent
voltage of 26 mV, the open circuit voltage (in volts, correct to two decimal places) at 0.2 sun
intensity is ____.
Key: (0.608)
Exp: A  1.0cm 2 ; I L  20mA; Voc  0.65; VT  26mV

JL 
IL
A
 J  Voc
 20m A/cm2  Voc  VT n 1  L  ; J L  JS VT
 Js  e 1  
J L'  0.2  20mA/cm 2  4mA/cm 2

 Voc1

J 1 ; n  1 
e 5  e   7.2 10
VT
 e  
Voc2 0.65 10

e  1
L
J' Voc2 VT
1 26m
1
L
VT

Voc2
 23.39  Voc2  26m  23.39  0.608
VT

44. The cutoff frequency of TE01 mode of an air filled rectangular waveguide having inner dimensions
a cm × b cm (a > b) is twice that of the dominant TE 10 mode. When the waveguide is operated at
frequency which is 25% higher than the cutoff frequency of the dominant mode, the guide
wavelength is found to be 4 cm. The value of b (in cm, correct to two decimal places) is ____.
Key: (0.75)
C
Exp: Cut off frequency for TE 01 
2b
C
Cut off frequency for TE10 
2a

© All rights reserved by Thinkcell Learning Solutions Pvt. Ltd. No part of this booklet may be reproduced or utilized in any form without the written permission.

37
EC-GATE 2018

C C a
It is given that 2  2
2b 2a b
5 C 
Given waveguide frequency f   
4  2a 
(25% higher than the cut off frequency of dominant mode)
5
f  fc
4
 01
 guide 
2
 fc 
1  
f 
 01 5
 guide   01  4cm
4
2 3
1  
5
3 4
  01   2.4cm
5
 10  2.4  1.25  3
10
a   1.5cm
2
 b  0.75cm

45. The distance (in meters) a wave has to propagate in a medium having a skin depth of 0.1m so that
the amplitude of the wave attenuates by 20 dB, is
(A) 0.12 (B) 0.23 (C) 0.46 (D) 2.3
Key: (B)
1
Exp: Skin depth   0.1m
f
1
Attenuation constant f   10
0.1
Amplitude of wave varies as e z
1
 e z 
10
1
 z  n    10 z  2.3
 10 
2.3
 z  0.23m
10

46. The logic gates shown in the digital circuit below use strong pull-down nMOS transistors for LOW
logic levels at the outputs. When the pull-downs are off, high-value resistors set the output logic

© All rights reserved by Thinkcell Learning Solutions Pvt. Ltd. No part of this booklet may be reproduced or utilized in any form without the written permission.

38
EC-GATE 2018

levels to HIGH (i.e., the pull-ups are weak). Note that some nodes are intentionally shorted to
implement “wired logic”. Such shorted nodes will be HIGH only if the outputs of all the gates
whose outputs are shorted are HIGH.
X0

X1
X2
Y
X3

The numbers of distinct values of X3 X 2 X1X 0 (out of the 16 possible values) that give Y = 1 ____ .
Key: (8)
Exp:
X0 X0
X0
X0 G
G F
0  fixed 
X1 X1  X2
Y 1
X2
G   X1  X2 .X3 1

X3 X3

 If we evaluate F by referring the above logic circuit


F  X0  X0 G   0
So the node F is always 0. Irrespective any possible combinations of X0 , X1 , X 2 , X3
 Since already one of the input to the OR gate is logic 0 (fixed) and we are expecting Y=1, so
we must need the other input of the or gate is logic 1 for which X3  1.
 So for Y=1. We need the fixed value of X3  1, with this X0 , X1 , X 2 can take any possible
combinations,
 So the number of possible combination of X0 , X1 , X 2 is 23  8, for each of the combination
X3  1.
 Hence there are 8 combinations of X0 , X1 , X 2 , X3 for which output Y=1

47. A 2 × 2 ROM array is built with the help of diodes as shown in the circuit below. Here W0 and W1
are signals that select the word lines and B0 and B1 are signals that are output of the sense amps
based on the stored data corresponding to the bit lines during the read operation. During the read
operation, the selected word line goes high and the other word line is in a high impedance state.

© All rights reserved by Thinkcell Learning Solutions Pvt. Ltd. No part of this booklet may be reproduced or utilized in any form without the written permission.

39
EC-GATE 2018

B0 B1

Sense amps

W0
B0 B1
W0  D00 D01 
W1  D10 D11 
W1 Bits stored in the ROM Array

VDD

As per the implementation shown in the circuit diagram above, what are the bits corresponding to
Dij (where i = 0 or 1 and j = 0 or 1) stored in the ROM?
1 0  0 1  1 0 1 1 
(A)   (B)   (C)   (D)  
0 1  1 0  1 0 0 0
Key: (A)
Exp: Form the given statement; it is clear that, if we want to read the stored information in the ROM
word line value should be logic 1.
So in all the 4 options, the top row of matrix indicate if W0  1, what are the value of
B0 and B1 similarly the bottom row indicates if W1  1. what are the value of B0 & B1 .
If we make W0  1, it‟s corresponding diode will be ON. And VDD will be available on B0 ,
since the diode corresponding to W1 is in off state  W1  1 now  , by default B1  0.
So when W0  1, then B0  1and B1  0.
Similarly when W1  1, then B0  0 and B1  1
So the matrix is
B0 B1 B0 B1
W0  1 1 0 or W0 1 0
W1  1 0 1  W1 0 1 

48. A curve passed through the point (x = 1, y = 0) and satisfies the differential equation
dy x 2  y2 y
  . The equation that describes the curve is
dx 2y x
 y2  1  y2 
(A) ln 1  2   x  1 (B) ln 1    x  1
 x  2  x2 

 y 1  y
(C) ln 1    x  1 (D) ln 1    x  1
 x 2  x

© All rights reserved by Thinkcell Learning Solutions Pvt. Ltd. No part of this booklet may be reproduced or utilized in any form without the written permission.

40
EC-GATE 2018

Key: (A)
Exp: Given D.E is
dy x 2  y 2 y
 
dx 2y x
dy  1 1  x2
   y 
dx  x 2  2y
dy  1 1  2 x 2
y   y   1
dx  x 2  2
dy dt dy 1 dt
Let y 2  t  2y  y 
dx dx dx 2 dx
1 dt  1 1  x2
From 1    t 
2 dx  x 2  2
dt 1 1
  2    t  x 2   2  ; which is linear D.E.
dx  x 2
1 1

2    dx  1 1 2 e x
I.F  e  x 2
 P  2  x  2  & Q  x   I.F  x 2
   
 The solution of eq  2  is
e x 2 e
x

x2  x2
t.  x dx

e x
 t. 2  e x  c
x
e x
 y 2 2  e  x  c   3 
x
Given that eq(3) passes through the point (1,0).
 C  e1  from  3 
From  3 ;
y2 e x
2
 e x  e1
x
 y2  1 y2  y2 
  2  1 e x   2  1  e x 1  n  2  1  x  1.
x  e x x 

49. A uniform plane wave traveling in free space and having the electric field

E  
2aˆ x  aˆ z cos 6 3  108 t  2(x  2 z)  V / m E

is incident on a dielectric medium (relative permittivity > 1, relative permeability = 1) as shown in


the figure and there is no reflected wave.

© All rights reserved by Thinkcell Learning Solutions Pvt. Ltd. No part of this booklet may be reproduced or utilized in any form without the written permission.

41
EC-GATE 2018

z
Dielectric
Free space
Medium( r  1)

The relative permittivity (correct to two decimal places of the dielectric medium is ____.
Key: (2)

Exp: Given E    
2 a11  a z cos  3 108 t  2 x  2z V m   z
The wave is parallel to the x, z plane
Since, there is no reflection, Free space
Angle of incidence in Brewster angle is x
i 2
2 i
i  B  tan 1  tan 1  2
1
1
tan i  2
 2  2  2  2

50. For a unity feedback control system with the forward path transfer function
K
G(s) 
s(s  2)

The peak resonant magnitude Mr of the closed-loop frequency response is 2. The corresponding
value of the gain K (correct to two decimal places) is _____.
Key: (15.02)
Exp: The O.L.T.F of the unity –ve feed back system is
K
G s 
s  s  2
The C.L.T.F is
G s K
T s   2
1  G  s  s  2s  K
By comparing with standard 2nd order equation

© All rights reserved by Thinkcell Learning Solutions Pvt. Ltd. No part of this booklet may be reproduced or utilized in any form without the written permission.

42
EC-GATE 2018

s 2  2ns  n2 , we have


n  K and
2n  2  n  1  2 2n  1
1
K
2
Let us obtain  for the resonant peak
1
Mr 
2 1  2
1 1
2  4 
2 1   2
1  2
1
 162 
1  2
 162 1  2   1
let 2  x  16x 1  x   1
 16x  16x 2  1  0
 16x 2  16x  1  0, by solving this we get
 x  0.067  2
1 1
K 2   15.02
 0.067

51. A four-variable Boolean function is realized using 4 × 1 multiplexers as shown in the figure.

I0 I0 F(U,V, W,X)

I1 I1
4 1 4 1
VCC
MUX MUX
I2 I2

I3 S1 S0 I3 S1 S0

U V W X

The minimized expression for F(U,V,W,X) is

(A)  UV  UV  W (B)  UV  UV  WX  WX 
(C)  UV  UV  W (D)  UV  UV  WX  WX 

© All rights reserved by Thinkcell Learning Solutions Pvt. Ltd. No part of this booklet may be reproduced or utilized in any form without the written permission.

43
EC-GATE 2018

Key: (C)
Exp:
0 I Y Y
0 I0
1 Y
I1 4 1 I1 4 1
Vcc MUX MUX F  U,V, W,X 
I2 0 I
2
1 0
I3 S S2 I3
0 1

U V W X

Output of the first MUX


Y  UV.0  UV.1  U V.1  UV.0  UV U V
Output of second MUX
F  W X Y  WXY  WX.0  WX.0
 W Y X  X   WY  YW

  U V U V  W
Although option C and option D are representing same function, but we can say the minimized
expression is option C.

52. Let c(t)  Ac cos(2f c t)and m(t)  cos(2f m t). It is given that f c  5 f m . The signal c(t) + m (t) is
applied to the input of a non-linear device, whose output v0 (t) is related to the input
vi (t) as v0 (t)  avi (t)  bvi2 (t), where a and b are positive constants. The output of the non-linear
device is passed through an ideal band-pass filter with center frequency fc and bandwidth 3fm, to
produce an amplitude modulated (AM) wave. If it is desired to have the sideband power of the AM
wave to the half of the carrier power, then a/b is
(A) 0.25 (B) 0.5 (C) 1 (D) 2
Key: (D)
Exp: Given
c  t   AC cos  2fc t  , m  t   cos  2f m t 
fc  5f m1

m t  vi  t 
 avi  t   b vi2  t  v0  t 

ct „a‟ and „b‟ are constants

© All rights reserved by Thinkcell Learning Solutions Pvt. Ltd. No part of this booklet may be reproduced or utilized in any form without the written permission.

44
EC-GATE 2018

v0  t   a  A c cos  2f c t   cos  2fmt    b A c cos  2 f c t   cos  2 f m t 


2

bA2c bA c2 b b
 a cos  2f m t   a A c cos  2f c t    cos  4 f c t    cos  4 f m t 
2 2 2 2
 2b A c cos  2 f m t  cos  2f c t .
After passing through BPF of center frequency fc and bandwidth 3f m ,
a Ac cos  2f c t  2b Ac cos  2f m t  cos  2 f c t 
yt  
carrier side bands
a 2 Ac2
Carrier power  , side band power = b 2 A c2
2
side band power 1
It is given that 
carrier power 2
b2 Ac2 1 b2 1 a
 2 2
  2
  2
a Ac 2 a 4 b
2

53. The input 4 sinc (2t) is fed to a Hilbert transformer to obtain y(t), as shown in the figure below:

Hilbert
4sinc(2t) y(t)
Transform

sin(x) 

2
Here sinc(x) = . The value (accurate to two decimal places) of y(t) dt is____.
x 

Key: (8)
Exp: By Parseval‟s theorem we can say
 
1
 y  t  dt   Y   d
2 2

 2 

1
The impulse response of the Hilbert transform is h  t   , and Fourier transform
t
H     jsgn()  j

j
X  
If x  t   k sinc(at)
k    k/a
then X    rect  
a  2a 

-a a

© All rights reserved by Thinkcell Learning Solutions Pvt. Ltd. No part of this booklet may be reproduced or utilized in any form without the written permission.

45
EC-GATE 2018

 4sin c  2t 
2
1
4sin c2t h  t2  yt
t
-2 2

1  
y  t    4sin c 2t      Y    2rect   .   jsgn  x  t  * h  t   X  .H  
 t   4 
Y  
 Y     j2
2 j
*

 2
2

j
-2 2   j2

4
 Y   
2


-2 2
 Finally,
 
1 1  1
 y  t  dt   Y   d  area of Y      4   4  8
2 2 2


2  2   2

54. Red (R), Green (G) and Blue (B) Light Emitting Diodes (LEDs) were fabricated using p-n junctions
of three different inorganic semiconductors having different band-gaps. The built-in voltage of red,
green and blue diodes are VR , VG and VB , respectively. Assume donor and acceptor doping to be
the same (NA and ND, respectively) in the p and n sides of all the three diodes. Which one of the
following relationships about the built-in voltages in TRUE?
(A) VR  VG  VB (B) VR  VG  VB

(C) VR  VG  VB (D) VR  VG  VB

Key: (B)
Exp: Built in potential of pn diode
 N .N 
Vbi  kT n  A D 
 n2 
 i 
Vbi  kT n N A N D  2kT n n i2 ... 1

Where n i can be defined as

© All rights reserved by Thinkcell Learning Solutions Pvt. Ltd. No part of this booklet may be reproduced or utilized in any form without the written permission.

46
EC-GATE 2018

Eg

n  N C .N V e
2
i
kT

Eg

ni  k ' e 2kT
...  2 
So substitute equation (2) in (1)
Vbi  kT n NA ND  2kT n k '  Eg
Vbi  M  Eg ;where Eg is band gap

All diodes N A and N D are same, so that M same

hc
Vbi  E g ; E g 

We know that Eg Red   Eg green   Eg blue  because
 Red  green   blue
 VR  VG  VB

55. Let r  x 2  y  z and z3  xy  yz  y3  1. Assume that x and y are independent variables. At


r
(x,y,z) = (2, –1, 1 ), the value (correct to two decimal places) of is ____.
x
Key: (4.50)
Exp: Given, r  x 2  y  z...  i 

and z3  xy  yz  y3  1...  ii 
r z
  2x  ...1  x, y are independent 
x x
 from  i  
z z
3z 2  y  y  0  form  ii  
x x
z
 3z 2  y   y
x
z y
  2   2
x 3z  y
r  y 
form 1 &  2  ;  2x   2 
x  3z  y 
r  1 
  2  2   
x  2, 1,1  3 1   1 
1
4  4.50
2

© All rights reserved by Thinkcell Learning Solutions Pvt. Ltd. No part of this booklet may be reproduced or utilized in any form without the written permission.

47
EC-GATE 2018

© All rights reserved by Thinkcell Learning Solutions Pvt. Ltd. No part of this booklet may be reproduced or utilized in any form without the written permission.

48
India’s Best Institute for IES, GATE & PSUs

ESE 2019
Mains Mains
Classroom Test
Program Series
Conventional Ques. Conventional
Practice Program Test Series

Commencing from Commencing from


th th
20 Feb, 2019 17 Mar, 2019
These batches are offered only Test series will be conducted at
at Delhi Centre. all MADE EASY centres across India.

Features :
Very useful to develop numerical solving approach & improving writing skills.
Special focus on improving answer layout specially for theory questions.
Classes will be delivered by senior faculties.
Updated Mains workbook for every subject having varied practice question sets.
Test series will be conducted on every Sunday in synchronisation with the subject taught in class.
Discussion on probable questions.
Comprehensive and in-depth discussion on variety of conventional questions, thus strengthening fundamental concepts.

Batch Course Duration Class Duration Test Series


Details 90 days | 300 - 350 hours 5-6 days a week and 6-7 hours a day Every Sunday

Streams Batch Code Batch Commencing Date Venue (Delhi) Timing

ME A 20-Feb-2019 Ghitorni Centre 7:30 AM to 1:30 PM


ME B 20-Feb-2019 Ghitorni Centre 3:00 PM to 9:00 PM
ME C 20-Feb-2019 Saket Centre 7:30 AM to 1:30 PM
CE A 21-Feb-2019 Ignou Road Centre 7:30 AM to 1:30 PM Hostel
CE B Kalu Sarai Centre 3:00 PM to 9:00 PM
EE A
21-Feb-2019
22-Feb-2019 Lado Sarai Centre 7:30 AM to 1:30 PM
facility
EE B 22-Feb-2019 Kalu Sarai Centre 3:00 PM to 9:00 PM
will be
EC A 22-Feb-2019 Lado Sarai Centre 7:30 AM to 1:30 PM
arranged

Ex. MADE EASY Students


Program Non MADE EASY
Enrolled in Postal, Rank Improvement, Mains, GS,
Post-GATE, ESE+ GATE, GATE Batches
students
Fee Mains Exclusive Batch
Structure (Inclusive of ESE-2019 Mains Offline Test Series)
` 18,500 ` 22,500

ESE 2019 Mains Offline Test Series Rs. 3,500/- Rs. 2,500/- Rs. 4,500/- Rs. 3,500/-

Rs. 1000/- Discount on test series valid till 15-02-2019

Corporate office : 44-A/1, Kalu Sarai, New Delhi - 1100 16 011-45124612, 9958995830 Click here to enroll
GATE ESE PSU’s 2019-20
ECE ENGINEERING
GATE ECE 2003-2019 SOLVED

GATE ECE 2003-2019 SOLVED Detail Solution

CONTENT COVERED:
1.Theory Notes
2.Explanation
3.Derivation
4.Example
5.Shortcut & Formula Summary
6.Previous year Paper Q. Sol.
Noted-: Single Source Follow, Revise
Multiple Time Best key of Success
1
Page

http://www.orbitmentor.com [email protected]
*As per ASSOCHAM & Brands Academy

Announcing

NEW BATCHES
for ESE 2020 & GATE 2020
1 Yr/2Yrs CLASSROOM COURSES

Early start Extra edge

Batches Commencement Dates


Regular Batches Weekend Batches
Delhi Delhi Noida
CE 14-Feb-2019 CE 09-Feb-2019 16-Feb-2019
ME 14-Feb-2019 ME NA 03-Feb-2019
EE 18-Feb-2019 EE 23-Feb-2019 16-Feb-2019
EC Mid-Feb, 2019 EC 23-Feb-2019 16-Feb-2019
CS 16-May-2019
CS 17-Feb-2019

Rest of India (Regular Batches)


Patna Lucknow Bhopal Indore Jaipur
25-Feb'19 20-Feb'19 25-Feb'19 20-Feb'19 17-Feb'19

Pune Hyderabad Bhubaneswar Kolkata


11-Feb'19 17-Feb'19 25-Feb'19 2-Mar'19

Admission open
Corporate office : 44-A/1, Kalu Sarai, New Delhi - 1100 16 011-45124612, 9958995830
Delhi Hyderabad Noida Jaipur Bhopal Lucknow Indore Bhubaneswar Pune Kolkata Patna
Centres 011-45124612 040-66774612 0120-6524612 0141-4024612 0755-4004612 09919111168 0731-4029612 09040299888 020-26058612 8981888880 0612-2356615
RRB-JE
Examination, 2019
India’s Best Institute for IES, GATE & PSUs

Admission Open in
Classroom Course for RRB-JE
(CBT-1)
Classroom Centres : Delhi Lucknow Patna

Batches commencing from


th
15 Feb, 2019
All streams are eligible

Fee Structure : Classroom Course (RRB-JE)

Exam Course Duration Timing Freshers Ex. MADE EASY Students


Enrolled in any classroom/postal courses

CBT 1 50 Days 7 days a week Rs. 10,170 + GST Rs. 7,628 + GST
(only) (180-200 hours) 4 hours per day = Rs. 12,000/- = Rs. 9,000/-

Delhi : Lucknow : Patna :


7303231082, 09919111168, 0612-2356615,
7303212325 08400029422 9955991166

www.madeeasy.in
India’s Best Institute for IES, GATE & PSUs

GATE 2019
RANK PREDICTOR
www.gaterankpredictor.madeeasy.in

Step 1 Step 2 Step 3


Select Your Stream Select the session Paste the URL of
you appeared for your Response Sheet

and predict your


• All India Rank
• Normalized Marks
Click here to redirect at MADE EASY GATE Rank Predictor
General Studies &
Engineering Aptitude
Batches for ESE 2020 India’s Best Institute for IES, GATE & PSUs

(Preliminary Examination)

Syllabus Covered
1. Current issues of national and international importance relating to social economic and industrial development.

2. Engineering Aptitude covering Logical reasoning and Analytical ability.

3. Engineering Mathematics and Numerical Analysis.

4. General Principles of Design, Drawing, Importance of Safety.

5. Standards and Quality practices in production, construction, maintenance and services.

6. Basic of Energy and Environment : Conservation, Environmental pollution and degradation, Climate Change,
Environmental impact assessment.

7. Basic of Project Management.

8. Basics of Material Science and Engineering.

9. Information and Communication Technologies (ICT) based tools and their applications in Engineering such as
networking, e-governance and technology based education.

10. Ethics and values in engineering profession.

Course Duration Timings Teaching Hours


Regular Batches : 2.5 months Regular : 6 to 7 days a week and 4-6 hours a day 250-300
Weekend Batches : 4 months Weekend : Sat, Sun & public holiday, 8 hours each day hours

Batch Type Commencing Dates Venue Timing


Regular Batch 20th Feb, 2019 Ghitorni (Delhi) 8:00 AM to 12:00 PM
Weekend Batch 24th Feb, 2019 Ghitorni (Delhi) 8:00 AM to 5:00 PM
Weekend Batch 24th Feb, 2019 Noida Centre 8:00 AM to 5:00 PM

Fee Structure
Non-MADE EASY Students Ex. MADE EASY Students
Enrolled in Postal, Rank Improvement, Mains, GS, GATE, GATE + ESE Batches

` 25,000 • GS & Engg Aptitude Books will NOT be issued.


• GS & Engg Aptitude Books will be issued.
` 18,000 • Interested students can avail books by paying the fee of Rs. 2,000/-

Corporate office : 44-A/1, Kalu Sarai, New Delhi - 1100 16 011-45124612, 9958995830
ADMISSION
Noida office : D-28, Sector - 63, Noida, Uttar Pradesh - 201301 0120-6524612, 08860378009 OPEN
GATE - 2019 Forenoon
Session
ELECTRONICS & COMMUNICATION
09/02/19
ENGINEERING

Subject wise weightage

S. No. Name of the Subject 1 Mark (Q) 2 Marks (Q) Total Marks

1 Verbal Ability 3 1 5

2 Numerical Ability 2 4 10

3 Engineering Mathematics 5 3 11

4 Network Theory 1 2 5

5 Control Systems 2 4 10

6 Digital Circuits 3 1 5

7 Signals & System 2 3 8

8 EDC & VLSI 4 8 20

9 Analog Circuits 1 2 5

10 EMT 3 3 9

11 Communication Systems 4 4 12

Total No. of. Marks 30 70 100

Subject Experts,
ACE Engineering Academy
2 Electronics & Communication Engineering

Section : General Aptitude


01. The boat arrived ______ dawn.
(a) Under (b) at (c) on (d) in
01. Ans: (b)
Sol: Use preposition ‘at’ dawn
End of Solution

02. When he did come home, she ______ him lying dead on the roadside somewhere.
(a) concluded (b) notice (c) looked (d) pictured
02. Ans: (d)
Sol: Pictured means to have a thought, understanding or idea about something or someone.
End of Solution

03. Five different books (P, Q, R, S, T) are to be arranged on a shelf. The books R and S are to be arranged first and
second, respectively from the right side of the shelf. The number of different orders in which P, Q and T may be
arranged is ______.
(a) 6 (b) 2 (c) 12 (d) 120
03. Ans: (a)
Sol: Five different books = P, Q, R, S and T from the given data, the above books are arranged as follows
__ __ __ S R
P Q T _bb
b
P T Q bbb
b
Q P T bb
` 6 ways
Q T P bbb
b
T P Q bbb
T Q P bb
a
Another Method:
From the given data R and S places are fixed and remaining three books can be arranged in 3! ways (i.e.) 3 × 2 ×1
= 6 ways.
Hence Option (1) is correct.
End of Solution

04. The strategies that the company _______ to sell its products ______ house-to-house marketing.
(a) use, includes (b) uses, include (c) uses, including (d) used, includes
04. Ans: (b)
Sol: Sub + Verb agreement
‘The Company’ is singular, so ‘uses’ and the ‘strategies’ is plural, so ‘include’(The company uses, the strategies
include).
3 GATE-19 EXAM PAPER
4 Electronics & Communication Engineering

05. It would take one machine 4 hours to complete a production order and another machine 2 hours to complete the
same order. If both machines work simultaneously at their respective constant rates, the time taken to complete
the same order is _____ hours.
(a) 3/4 (b) 2/3 (c) 7/3 (d) 4/3

05. Ans: (d)

Sol: Machine one (M1) can take to complete production = 4 hours

Second Machine (M2) can take to complete production = 2 hours


1 th
M1 = 4 hours, 1 hours = 4 production
1 th
M2 = 2 hours, 1 hours = 2 production

In one hour ^M1  M2h  4  2  4  4


1 1 1  2 3 th

4
∴ M1 and M2 can take to complete production = 3 hours
End of Solution
06. Five people P, Q, R, S and T work in a bank. P and Q don’t like each other but have to share an office till T gets
a promotion and moves to the big office next to the garden. R, who is currently sharing an office with T wants to
move to the adjacent office with S, the handsome new intern. Given the floor plan, what is the current location of
Q, R and T?
(O = Office, WR = Washroom)

(a) WR O1 O2 O3 O4 (b) WR O1 O2 O3 O4
P, Q R, T S P, Q T R, S

Manager Teller Teller Manager Teller Teller


1 2 1 2
Entry Entry
Garden Garden

(c) WR O1 O2 O3 O4 (c) WR O1 O2 O3 O4
P Q R S P, Q R S

Manager Teller Teller Manager Teller Teller


1 2 T 1 2
Entry Entry
Garden Garden
5 GATE-19 EXAM PAPER

06. Ans: (a)


Sol: Before getting promotion ‘T’ sharing with R and P and Q’s are working together means they are in same office.
Option ‘2’ is not correct due to T is sharing with R (i.e.) before getting promotion T is not worked alone.
Option ‘3’ is not correct due to ‘T’ place of work is not defined.
Option ‘4’ is also not correct due to after ‘T’ getting promotion P and Q is are not working together.
End of Solution

07. Four people are standing in a line facing you. They are Rahul, Mathew, Seema and Lohit. One is an engineer, one
is a doctor, one a teacher and another a dancer. You are told that:
1. Mathew is not standing next to Seema
2. There are two people standing between Lohit and the engineer
3. Rahul is not a doctor
4. The teacher and the dancer are standing next to each other
5. Seema is turning to her right to speak to the doctor standing next to her
Who among them is an engineer?
(a) Mathew (b) Rahul
(c) Seema (d) Lohit
07. Ans: (a)
Sol: Four peoples are Rahu, Mathew, Seema and Lohit and in the group one engineer, one is a doctor, one a teacher
and another a dancer.
Statement 1:
Seema Mathew
Statement 2:
Lohit ___ ___ ___ ___
Engineer
Statement 3:
Rahul ≠ doctor
Statement 4:
Teacher (or) Dancer Dancer (or) Teacher

Statement 5:
____ Seema
Doctor
From above conditions, the following line can be formed
Lohit Seema Rahul Mathew
↓ ↓ ↓ ↓
Doctor Teacher/Dancer Teacher/Dancer Engineer
From above, an engineer in the group is Mathew.

Hence option ‘1’ is correct.


6 Electronics & Communication Engineering

08. The bar graph in Panel(a) shows the propagation of male and female illiterates in 2001 and 2011. The proportions
of males and females in 2001 and 2011 are given in Panel(b) and (c), respectively. The total population did not
change during this period.
The percentage increase in the total number of literates from 2001 to 2011 is _____.
Proportion of illiterates (%)

100
80
60
60 50
40 40
40
20
0
Female Male

Panel (a)

2001 2011

Female Female
Male 40% Male 50%
60% 50%

Panel (b) Panel (c)

(a) 35.43 (b) 30.43 (c) 34.43 (d) 33.43


08. Ans: (b)
Sol: Panel (a), Bar diagram represents, proportion of illiterates (%) dark shaded represents female and male illiterates
in 2001 and light shaded represents female and male illiterates in 2011. Panel (b) and panel (c) male and females
in 2001 and 2011 respectively.
Assume population in 2001 = 100 nos
from given data, population in 2011 also 100.
From the given Bar charts and pie charts, the following table can be possible.
2001 2011
Males Females Males Females
Total 60 40 50 50
Illiterates 50% of 60 = 30 60% of 40 = 24 40% of 50 = 20 40% of 50 = 20
Literates 60 - 30 = 30 40 - 24 = 16 50 - 20 = 30 50 - 20 = 30

Total literates in 2001 = 30 + 16 = 46


Total literates in 2011 = 30 + 30 = 60
60 − 46 #
∴The percentage increase in the total number of literates from 2001 to 2011 = 46 100
= 14 # =
60 100 30.43%
7 GATE-19 EXAM PAPER

09. “Indian history was written by British historians – extremely well documented and researched, but not always
impartial. History had to serve its purpose: Everything was made subservient to the glory of the Union Jack.
Latter-day Indian scholars presented a contrary picture.”
From the text above, we can infer that:
Indian history written by British historians _____
(a) was not well documented and researched and was always biased
(b) was well documented and not researched but was always biased
(c) was well documented and researched but was sometimes biased
(d) was not well documented and researched and was sometimes biased
09. Ans: (c)
Sol: Other choices are irrelevant
End of Solution

10. Two design consultants, P and Q, started working from 8 Am for a client. The client budgeted a total of USD
3000 for the consultants. P stopped working when the hour hand moved by 210 degrees on the clock. Q stopped
working when the hour hand moved by 240 degrees. P took two tea breaks of 15 minutes each during her shift,
but took no lunch break. Q took only one lunch break for 20 minutes, but no tea breaks. The market rate for
consultants is USD 200 per hour and breaks are not paid. After paying the consultants, the client shall have USD
____ remaining in the budget.
(a) 000.00
(b) 166.67
(c) 300.00
(d) 433.33
10. Ans: (b)
Sol: P and Q started work at 8 am
P makes an angle of 210°
Q makes an angle of 240°
360 o
hours hand makes an angle of 30° for each hours (i.e.) 12 = 30 o
P →210° and Q →240°
Tea break time of P = 15 min × 2 = 30 min
Lunch break time of Q = 20 min
210
P’s working hours including breaks = 30 = 7 hours
240
Q’s working hours including breaks = 30 = 8 hours
P’s net working hours (excluding breaks) = 7 hrs - 30 min = 6 hrs 30 min
Q’s net working hours (excluding breaks) = 8 hrs - 20 min = 7 hrs 40 min
P’s and Q’s total working hours = 6 hrs 30 min + 7 hrs 40 min = 14 hrs 10 min
10
Total amount payed to consultants = 14 hrs # 200  60 # 200  2800  33.34  2833.33
The remaining amount with client from the budget = 3000 - 2833.33 = 166.67 USD
8 Electronics & Communication Engineering

Section : Electronics & Communication Engineering

01. For an LTI system, the Bode plot for its gain is as illustrated in the figure shown. The number of system poles Np
and the number of system zeros Nz in the frequency range 1 Hz ≤ ƒ ≤ 107 Hz is
Gain (dB)

-20
100 dB
/de
c
-60
dB

-40
/de

dB
/de
c

c
104 105 106 107 f
0
101 102 103 (in Hz)
-40
dB
/de
c
-60
dB
/de
c

(a) Np = 4, Nz = 2 (b) Np = 6, Nz = 3
(c) Np = 7, Nz = 4 (d) Np = 5, Nz = 2
01. Ans: (b)
Sol:

Gain (dB)

-20
100 dB
/de
c
-60
dB/

-40
dec

dB
/de
c
104 105 106 107 f
0
101 102 103 (in Hz)
-40
dB
/de
c
-60
dB
/de
c


9 GATE-19 EXAM PAPER

At f = 10Hz → 1pole (– 20 dB/dec)


f = 102 Hz → (–40 dB/dec) → 2 poles
f = 103 Hz → (+20 dB/dec) → 1 zero
f = 104 Hz → (+40 dB/dec) → 2 zeros
f = 105 Hz → (–40 dB/dec) → 2 poles
f = 106 Hz → (–20 dB/dec) → 1 pole
N p = 6 , NZ = 3
End of Solution
02. Let Y(s) be the unit-step response of a causal system having a transfer function G ] s g = ]
3-s
s + 1g]s + 3g

G]s g
that is, Y ] s g = s . The forced response of the system is
(a) u(t)
(b) 2u(t)
(c) 2u(t) – 2e–t u(t) + e–3t u(t)
(d) u(t) – 2e–t u(t) + e–3t u(t)
02. Ans: (d)
Sol: Y(s) → Unit step response of a causal system
G (s) 3s
Y (s)  s  
 (s 1) (s  3) s
 2  1 1
s1 s3 s
↓ILT

y(t) = –2 e–t u(t) + e–3t u(t) + u(t)
End of Solution
03. Let H(z) be the z-transform of a real-valued discrete-time signal h[n]. If P(z) = H(z) H b 1z l has a zero at
1 1
z = 2 + 2 j , and P(z) has a total of four zeros, which one of the following plots represents all the zeros correctly?

(a) z-plane Imaginary


axis
2

1
= 1
|z|
0.5

1 Real axis
-2 0.5 2
-0.5

-1
-2
10 Electronics & Communication Engineering

(b) z-plane Imaginary


axis
2

=1
|z|
0.5

-0.5 Real axis


-2 0.5 2

-1
-2

(c) z-plane Imaginary


axis
2

=1
|z|
0.5

Real axis
-2 0.5 2

-0.5

-1
-2

(d) z-plane Imaginary


axis
2

=1
|z|
0.5

-0.5 Real axis


-2 0.5 2

-0.5

-2
11 GATE-19 EXAM PAPER

03. Ans: (a)


Sol: H(z) is Z-transform of real valued discrete time signal h(n)
1 j
If H(z) has a zero at z1  2  2
The remaining zeros are z 2  z1*  1  j
2 2
z  1  1  j
3
z1

z 4  d z1 n  1  j
*

1
The pole zero plot is

z-plane Imaginary
axis
2

1
= 1
|z|
0.5

1 Real axis
-2 0.5 2
-0.5

-1
-2

End of Solution
12 Electronics & Communication Engineering

04. The correct circuit representation of the structure shown in the figure is

B E C

n+
p+

n++
n

n+

(a) (b)
C C

B B

E E

(c) (d)
C C

B B

E E

04. Ans: (a)


Sol: As per the fabrication structure given BJT is npn transistor with base to collector of nn+ material means act as P-N
diode from base to collector i.e at base usually ‘n’ material acts as p-type with respect to n++ and n++ act as n-type
material with respect to n-material
End of Solution
13 GATE-19 EXAM PAPER
14 Electronics & Communication Engineering

The families of curves represented by the solution of the equation dx =-c xy m


n
dy
05.
For n = –1 and n = +1, respectively, are
(a) Circles and Hyperbolas
(b) Parabolas and Circles
(c) Hyperbolas and Circles
(d) Hyperbolas and Parabolas
05. Ans: (c)

Sol: Given dx =-c xy m


n
dy

⇒ yn dy +xn dx = 0 ( use variable separable method)

yn+1 xn+1 cn+1


By Integrating n + 1 + n + 1 = n + 1 For (n ≠−1)

⇒ yn+1 + xn+1 = cn+1
Clearly for n = 1 x2 + y2 = c2 ⇒ circle

c x m =-
-1
dy y
For n = −1 dx =- y x

⇒ x dy + y dx = 0
⇒ d(xy) = 0
By Integrating → xy = c → Hyperbola

End of Solution

06. The baseband signal m(t) shown in the figure is phase-modulated to generate the PM signal
ϕ(t) = cos(2πfct + k m(t)). The time t on the x-axis in the figure is in milliseconds. If the carrier frequency
is fc = 50 kHz and k = 10π, then the ratio of the minimum instantaneous frequency (in kHz) to the maximum
instantaneous frequency (in kHz) is ______ (rounded off to 2 decimal places).

m(t)

1 2 3 4 5 6 7 8 9
0
t (in ms)

-1

15 GATE-19 EXAM PAPER

06. Ans: 0.75

Sol: fi = fc + 2r dt m ] t g
k d

fmax = fc + 2r dt m ] t gmax
k d

fmin = fc + 2r dt m ] t gmin
k d

k
given fc = 50kHz, k = 10r & 2r = 5

dm ] t g dm ] t g
dt min =- 1k, dt max = 2k
fmin 50k + 5k 45k
fmax = 50k + 10k = 60k = 0.75

End of Solution

07. he figure shows the high-frequency C-V curve of a MOS capacitor (at T = 300K) with Φms = 0 V and no oxide
charges. The flat-band, inversion, and accumulation conditions are represented, respectively, by the points

P Q

0 VG


(a) Q, R, P
(b) Q, P, R
(c) R, P, Q
(d) P, Q, R
07. Ans: (a)
Sol: Given C-V characteristics of MOS capacitor with p-type substrate for high frequencies.
Point-P possible in accumulation mode
Point-Q possible in flat band mode
Point-R possible in inversion mode
So, option (1) is correct
16 Electronics & Communication Engineering

# #
r r
sin x
08. The value of the integral x dx dy , is equal to _______.
0 y

08. Ans: 2

Sol: #y =r0 ; #x =ry sinx x dxEdy y


(�, �)
Change the order of integration, then
x: 0 to π y
x= x=�
y: 0 to x
#x = 0 ; #y = 0 x dyEdx
r y = x sin x
x
0

= #
r sin x
- r
x .x dx = cos x | 0
x=0

= -(-1-1)

=
2
End of Solution
RS V
SS2 2 3 3WWW
S0 1 1 1WW
09. The number of distinct eigenvalues of the matrix A = SSS W is equal to _______.
SS0 0 3 3WWW
SS0 0 0 2WW
09. Ans: 3 T X
RS VW
SS 2 2 3 3 W
Sol:
SS0 1 1 1WWW
Given that A = SS W
SS0 0 3 3WWW
SS0 0 0 2WW
T X

The given matrix is an upper triangular matrix. It’s eigen values are Just diagonal
elements only.
∴ Eigen values are 2, 1, 3, 2
∴ Number of distinct eigen values = 3

End of Solution

10. A standard CMOS inverter is designed with equal rise and fall times (βn = βp). If the width of the pMOS transistor
in the inverter is increased. What would be the effect on the LOW noise margin (N ML) and the HIGH noise
marging N MH?
(a) No change in the noise margins.
(b) N ML decreases and N MH increases.
(c) N ML increases and N MH decreases.
(d) Both N ML and N MH increase.
17 GATE-19 EXAM PAPER

10. Ans: (c)


Sol: VDD
Vo
VDD
VOH

V0
Vin

VOL
fig: CMOS inverter VIH
VTon VIL VIH VDD

Vton VDD-|VTop|
fig: VTC or the CMOS inverter

The behavior of the CMOS inverter for static conditions of operation is described by the voltage transfer
characteristics (VTC) and for dynamic operation condition is described by the time response during switching
conditions.

VOH = VDD
2Vo - VTop - VDD + k r VTon
VIL = 1 + kr
Vo = Vin - VTop + _ Vin - VDD - VTop i + k r _ Vin - Vtop i
2 2

 n  n c ox b L ln
W
kr =  =
 p c ox b L l
p W
p

VDD + VTop + k r ^2Vo + VToph



VIH = 1 + kr

Vo = Vin - VTon + ^Vin - VTonh + k 7Vin - VDD - VTopA
2 1 2
r
VOL = 0
NML = VIL - VOL
NMH = VOH - VIH
∴ Wp↑→NML↑
Wp↑→NMH↓
End of Solution
11. In the circuit shown, Vs is a square wave of period T with maximum and minimum values of 8 V and −10V,
respectively. Assume that the diode is ideal and R1 = R2 = 50Ω.
The average value of VL is _____ volts (rounded off to 1 decimal place).
R1

+8 +
Vs
T T +
− R2 VL
-10 2
-

18 Electronics & Communication Engineering

11. Ans: -3
Sol:
T 50Ω
0 t 2
+ +4V
Vs
+
− 50Ω VL =

T
2 t T

+
10V +- 50Ω VL =
-10V
-

So, output waveform is


VL

+4V
0 T t
T
2
-10V

RS T/2 VW
SS T
W
1
VL  T SS 4 dt 
(Avg) SS0
# #
( 10) dtWWW
WW
T/2
T X
 1 >4 d T  0 n  10 d T  T nH
T 2 2

 1 . T 84  10B
T 2
VL  3 Volts
(Avg)
End of Solution

# b z + 1z l dz
2
1
12. The value of the contour integral 2rj evaluated over the unit circle |z| = 1 is _______ .
12. Ans: 0

Sol: Given 2rJ # b z + 1z l dz C is |z| = 1


2
1
C

]z2 + 1g2
# b z + 1z l dz = #
2
dz
C C
z2
19 GATE-19 EXAM PAPER

z = 0 is singular point lies inside of the curve C, |z| = 1


By Cauchy’s Integral formula

]z2 + 1g2 1 f1 ] z g
# z 1+1 = 2rJ 1!
at z = 0
C

Here f(z) = (z2 + 1)­2



f1(z) = 2(z2 + 1) 2z ⇒ f1(0) = 0
]z2 + 1g2

1
∴ 2rJ # z2
1
dz = 2rJ # 0 = 0
C

End of Solution

13. Consider the signal f(t) = 1 + 2 cos(πt) + 3 sin b 3 l + 4 cos b 2 t + 4 l , where t is in seconds. Its fundamental
2rt r r
time period, in seconds, is _______.
Ans: 12

Sol: f (t)  1  2cos _t i  3sin d 3 t n  4cos d 2 t  4 n


2  

0  GCD d , 3 , 2 n
2 

 GCD d 6 , 4 , 3 n
6 6 6

 
6
2
Time period T  0  12secs

Alternate method

2 
(i) 1    2  3 3  2

2   2  2  2   
T1  & T1 2 T2 3 & T2 3 T3 2 & T3 4
T 2
(ii) T1 = 3
2

T= 2= 1
1
T3 4 2

(iii) L.C.M of denominators of step (ii)

= L.C.M (3, 2) = 6

(iv) T = (L. C. M) T1 = (2) (6) = 12 secs


20 Electronics & Communication Engineering

14. Let Z be an exponential random variable with mean 1. That is, the cumulative distribution function of Z is given
by
Fz ]xg = )
1 - e -x if x $ 0

0 if x < 0

Then Pr (Z > 2 |Z > 1), rounded off to two decimal places, is equal to ______ .
14. Ans: 0.3679
Sol: Given that Z is exponential distribution with cumulative function Fz (x) = 1 - e -x x ≥ 0
0 x < 0
We know that probability density function
fz (x) = Flz (x) = e -x x $ 0

0 x < 0

P ]z > 2/z > 1g =
P ((z > 2) + (z > 1))
P ]z > 1g

3

# e -x dx
= 2
3

# e -x dx
1

]- e -xg3
= ] -xg23
-e 1
- ]e - 3 - e -2g e -2
- ]e - 3 - e -1g e -1
= =
= e −1

= 0.3679
End of Solution

15. In the circuit shown, what are the values of F for EN = 0 and EN = 1, respectively?

Vdd

EN
F


(a) Hi-Z and D
(b) 0 and D
(c) Hi-Z and D
(d) 0 and 1
21 GATE-19 EXAM PAPER

15. Ans: (a)


Sol: Case-1: When enable = 0, both MOSFETs are off, hence F = Hi-Z
Case-2: When enable = 1, inputs of both MOSFETs is D i.e.,
→ If D = 0 is ⇒ n-channel MOSFET is on, p-channel MOSFET is off so F = 0,
→ If D = 1 ⇒ PMOS-on, NMOS-off. thus, F = VDD = logic 1
i.e, when E = 1 ⇒ F = D
End of Solution
16. In the circuit shown, the clock frequency, i.e., the frequency of the CLK signal, is 12 kHz. The frequency of the
signal at Q2 is _____ kHz.

D1 Q1 D2 Q2

Clk Q1 Clk Q 2
12kHz

16. Ans: 4
Sol: Given D1 = Q1 Q2 , D2 = Q1
CLK Present state Flipflops inputs Next state
Q1 Q2 D1 D2 Q1 Q2
0 0 1 0 1 0
1 0 0 1 0 1
0 1 0 0 0 0
So, the circuit counts 00,10,01,........
f 12
Hence, the frequency at Q2 is CLK3 = 3 = 4kHz
End of Solution

17. What is the electric flux a # E.dat k through a quarter-cylinder of height H (as shown in the figure) due to an
infinitely long line charge along the axis of the cylinder with a charge density of Q ?

ε0


Q
H

4H HQ Hf HQ
(a) Qf (b) 4f (c) 4Q0 (d) f 0
0 0
22 Electronics & Communication Engineering

17. Ans: (b)


Sol: Electric flux = # E v .dav
S
we know electric field due to line charge
v = t L at t
E 2rf 0 t
Given ρL = Q
v = Q at t
E 2rf 0 t
So
#S Ev .dav = #S 2rfQ0 t at t .dav
= 2rf t # da
Q

0 S

Q
= 2rf t (surface area of the given portion)
0

#S Ev .dav = 2rf0 t b 2rt4 H l


Q

QH
= 4f .
0
End of Solution
18. Which one of the following functions is analytic over the entire complex plane?
(a) e1/z (b) ln(z)
1
(c) 1 - z (d) cos(z)
18. Ans: (d)
Sol: (a) e1/z is NOT analytic at z = 0
(b) ℓnz is NOT analytic in Domain D = {z / x ≤ 0 , y = 0}
1
(c) 1 - z is NOT analytic at z = 1

∴ cosz is analytic every where in the complex plane.


End of Solution

19. A linear Hamming code is used to map 4-bit messages to 7-bit codewords. The encoder mapping is linear. If the
message 0001 is mapped to the codeword 0000111, and the message 0011 is mapped to the codeword 1100110,
then the message 0010 is mapped to
(a) 1100001
(b) 1111000
(c) 0010011
(d) 1111111
19. Ans: (a)
Sol: A code is said to be linear if the algebraec sum of two codes is also another code
0001 → 0000111
0011 → 1100110
0010 1100001
So, 0010 is mapped into 1100001
23 GATE-19 EXAM PAPER
24 Electronics & Communication Engineering

20 In the table shown, List-I and List II, respectively, contain terms appearing on the left-hand side and the right-hand
side of Maxwell’s equations (in their standard form). Match the left-hand side with the corresponding right-hand
side.
List I List II
1 ∇.D P 0
2 ∇×E Q ρ
3 ∇.B R 2B
- 2t

4 ∇×H S 2D
J + 2t

(a) 1 − Q, 2 − R, 3 − P, 4 − S (b) 1 − Q, 2 − S, 3 − P, 4 − R
(c) 1 − P, 2 − R, 3 − Q, 4 − S (d) 1 − R, 2 − Q, 3 − S, 4 − P
20. Ans: (a)
Sol: 1. d.Dv =  (Q) → Gauss’s Law
v
2. d # E v = - 2B (R) → Faraday’s Law
2t
3. v
d.B = 0 (P) → Gauss’s Law for magnetic fields
v
4. d # H v = J + 2D (S) → Ampere’s Law
2t
End of Solution

21. Consider the two-port resistive network shown in the figure. When an excitation of 5 V is applied across Port 1,
and Port 2 is shorted, the current through the short circuit at Port 2 is measured to be 1 A (sec(a) in the figure).
Now, if an excitation of 5 V is applied across Port 2, and Port 1 is shorted (sec(b) in the figure). What is the current
through the short circuit at Port 1 ? 1Ω 2Ω

Port 1 RΩ Port 2

1Ω 2Ω 1Ω 2Ω

+
− 5V
5V +
− RΩ 1A ? RΩ

(a) (b)

(a) 1 A (b) 0.5 A (c) 2.5 A (d) 2 A


25 GATE-19 EXAM PAPER

21. Ans: (a)


Sol: This is direct application of reciprocity theorem - which states, in any linear passive bilateral network excited by
a single source. The ratio of response to excitation remains constant even if the position of source and Load are
interchanged.

i
So, V = Constant


1Ω 2Ω

RΩ
+
− 5V
1A

So, answer is 1 Amp

End of Solution

22. Radiation resistance of a small dipole current element of length ℓ at a frequency of 3 GHz is 3 ohms. If the length
is changed by 1%, then the percentage change in the radiation resistance, rounded off to two decimal places, is
________ %
22. Ans: 2

Sol: Given f = 3GHz

Rrad = 3Ω
T,
, # 100 = 1
TR
R # 100 = ?

R rad = 802 b  l
, 2

R rad ,2
TR T,
R rad # 100 = 2 , # 100
rad

= 2(1)
TR
R rad # 100 = 2%
rad
26 Electronics & Communication Engineering

23. Which one of the following options describes correctly the equilibrium band diagram at T = 300 K of a Silicon
pnn +p++ configuration shown in the figure?

p n n+ p++

EC

(a) EF

EV

EC

EF
(b)

EV

EC

EV
(c)
EF

EC

EF
(d)

EV


27 GATE-19 EXAM PAPER

23. Ans: (c)


Sol: → The device is not biased hence fermi level should be constant.
→ In P type fermi level should be closer to Ev.
→ In N type fermi level should be closer to Ec.
→ In P++ fermi level penetrates into valence band.
End of Solution
24. In the circuit shown, A and B are the inputs and F is the output. What is the functionality of the circuit?
Vdd

A B


(a) XNOR (b) SRAM Cell (c) Latch (d) XOR
24. Ans: (b)
Sol: Vdd

P1

P2

N1 N2

A B

28 Electronics & Communication Engineering

A B N1 N2 P1 P2 F(Output)
0 0 OFF OFF ON ON 1
1 0 ON OFF OFF ON 0
0 1 OFF ON ON OFF 0
1 1 ON ON OFF OFF Hi-Z
∴ Given diagram is S-RAM Cell.
End of Solution

25. If X and Y are random variables such that E[2X+Y] = 0 and E[X + 2Y] = 33, then E[X] + E[Y] = _______ .
25. Ans: 11
Sol: Given X,Y are Random variables
E(2X + Y) = 0 and E(X+2Y) = 33
⇒ 2E(X) + E(Y) = 0.........(1)
⇒ E(X) + 2 E(Y) = 33......(2)
By solving (1) and (2) E(Y) = 22
E(X) = −11
∴E(X) + E(Y) = 22 − 11 = 11
End of Solution
26. The dispersion equation of a waveguide, which relates the wavenumber k to the frequency ω, is
k ]~g = ]1/cg ~2 - ~20
where the speed of light c = 3 × 108 m/s, and ω0 is a constant. If the group velocity is 2 × 108 m/s, then the phase
velocity is
(a) 2 × 108 m/s
(b) 4.5 × 108 m/s
(c) 3 × 108 m/s
(d) 1.5 × 108 m/s
26. Ans: (b)
Sol: Given

wave number  = k ]g = c 2 - 20


1

Group velocity (vg) = 2 ×108 m/s

Phase velocity ^v ph = 


ωc
vP =
ω2 - ω20
we know
d 1
v g = d =
c m
d
1 d
 = c 2 - 20
29 GATE-19 EXAM PAPER

d 1 1
d = c 2 2 - 20 2
d 
d = c 2 - 20
1 ω
v g = c ω2 - ω20
ωv g
ω2 - ω20 = c
.2 # 108
=
3 # 108

ω2 - ω20 = 3
ωc = 2ωωc
so, v p =
ω2 - ω20
3.
v p = 2 c = 2 ]3 # 10 g = 4.5 # 108 m/s.
3 3
8

(or)
we know the relation
vp vg = c2
c2
v p = vg
]3 # 108g2
vp =
2 # 108
vp = 4.5×108 m/s.

End of Solution

27. It is desired to find three-tap causal filter which gives zero signal as an output to an input of the form
x (n) = c1 exp d  n + c2 exp d n
jn jn
2 2

Where c1 and c2 are arbitrary real numbers. The desired three-tap filter is given by
h[0] = 1, h[1] = a, h[2] = b
And
h[n] = 0 for n < 0 or n > 2.
What are the values of the filter taps a and b if the output is y[n] = 0 for all n, when x[n] is as given above?

x[n] n=0 y[n]=0


h[n]={1,a,b}

(a) a = 1, b = 1 (b) a = –1, b = 1


(c) a = 0, b = 1 (d) a = 0, b = –1
30 Electronics & Communication Engineering

27. Ans: (c)

Sol: Given x (n) = c1 exp d  n + c2 exp d n


jn jn
2 2
and h(0) = 1; h(1) = a; h(2) = b

H (e j~) = 1 + ae -j~ + be -j2~


Given that y(n) = 0 for all n

From the given options by trail & errors of we make a = 0 & b = 1


jn j
& output  1  0.e  2  e j2d  2 n  1  1  0

Due to c1 e  2

jn j
& output  1  0.e  2  e j2d 2 n  1  1  0

c2 e 2

End of Solution

28. Let the state-space representation of an LTI system be xo (t) = A x(t) + B u(t), y (t) = C x(t) + d u(t) where A,B,C
are matrices, d is a scalar, u(t) is the input to the system and y(t) is its output. Let B = [ 0 0 1]T and d = 0. which
one of the following options for A and C will ensure that the transfer function of this LTI system is
1
H (s) = 3 ?
s + 3s2 + 2s + 1
RS V
SS 0 1 0 WW
0 1 WW and C = 60 0 1@
W
(a) A = SSS 0 W
SS- 3 - 2 - 1WW
TR XV
SS 0 1 0 WW
0 1 WW and C = 60 0 1@
S W
(b) A = SSS 0 W
SS- 1 - 2 - 3WW
RST VX
SS 0 1 0 WW
0 1 WW and C = 61 0 0@
W
(c) A = SSS 0 W
SS- 3 - 2 - 1WW
TR XV
SS 0 1 0 WW
0 1 WW and C = 61 0 0@
S W
(d) A = SSS 0 W
SS- 1 - 2 - 3WW
T X
28. Ans: (d)
1
Sol: H (s) 
s3  3s 2  2s  1
RS VW RS VR V R V
SSxo 1WW SS 0 1 0 WWW SSSx1WWW SSS0WWW
SSxo 2WW  SS 0 0 1 WW SSx 2WW  SS0WW 7uA
SS WW SS WS W S W
Sxo 3W S 1  2  3WW SSx3WW SS1 WW
T X T RS VWX T X T X
SSx1WW
7YA = 81 0 0B SSSx 2WWW
SSx WW
3
T X
31 GATE-19 EXAM PAPER
32 Electronics & Communication Engineering

29. A germanium sample of dimensions 1 cm × 1 cm is illuminated with a 20 mW, 600 nm laser light source as shown
in the figure. The illuminated sample surface has a 100 nm of loss-less Silicon dioxide layer that reflects one-
fourth of the incident light. From germanium interface, one-third is absorbed in the germanium layer, and one-
third is transmitted through the other isde of the sample. If the absorption coefficient of germanium at 600 nm is
3 × 104 cm–1 and the bandgap is 0.66 eV, the thickness of the germanium layer, rounded off to 3 decimal places,
is _____ µm.
20mW, 600nm

1 cm
1 cm
Silicon dioxide 100 nm
Germanium T

29. Ans: 0.231
Sol: 1 - e - ax = 0.5
e - ax = 0.5

- ln ]0.5g - ln ]0.5g
x= a = # 4 cm
3 10

= 0.231 mm

End of Solution

30. In the circuit shown, the breakdown voltage and the maximum current of the Zener diode are 20 V and 60 mA.
respectively. The values of R1 and RL are 200 Ω and 1 kΩ, respectively. What is the range of Vi that will maintain
the Zener diode in the ‘on’ state?
R1

+ RL
Vi −


(a) 18 V to 24 V (b) 22 V to 34 V (c) 24 V to 36 V (d) 20 V to 28 V
30. Ans: (c)
Sol: VI,min - 20 20
0.2k = 1k
VI,min = 24V

VI,max - 20
0.2k = 60mA + 20mA
VI,max = 36V
33 GATE-19 EXAM PAPER

31. Let h[n] be a length-7 discrete-time finite impulse response filter, given by
h[0] =4, h[1] = 3, h[2] = 2, h[3] = 1,
h[-1] = -3, h[-2] = -2, h[-3] = -1,
and h[n] is zero for |n| ≥4. A length-3 finite impulse response approximation g[n] of h[n] has to be obtained such
that
_ h, g i = # _e j~ i - G _e j~ i d
r 2

E H
-r

is minimized, where H(e ) and G(ejω) are the discrete-time Fourier transforms of h[n] and g[n], respectively. For

the filter that minimizes E(h, g), the value of 10g[-1] + g[1], rounded off to 2 decimal places, is ______.
Ans: Insufficient data

End of Solution

32. In the circuit shown, the threshold voltages of the pMOS (|Vtp|) and nMOS (Vtn) transistors are both equal to 1 V.
All the transistors have the same output resistance rds of 6 MΩ. The other parameters are listed below:
n n C ox = 60nA/V2; b L l
W =5
nMOS

n n C ox = 30 nA/V2; b L l
W = 10
pMOS

µn and µp are the carrier mobilities, and Cox is the oxide capacitance per unit area. Ignoring the effect of channel
length modulation and body bias, the gain of the circuit is _____
(rounded off to 1 decimal place).
Vdd = 4V

Vout
Vin


32. Ans: -900
Sol: I DC = 2 n P C OX b L l ^VSGP - VTP h2
1 W
P
1 # nA # # ] g2 2
= 2 30 V2 10 2 - 1 V
= 150µA

2I DC n n C OX b L l =
W nA
g mn = 2 # 150nA # 60 #5
N v2
= 300µS
Av = −gm(rds || rds) = − 300 × 3 = −900
34 Electronics & Communication Engineering

33. Consider a causal second-order system with the transfer function


1
G (s) =
1 + 2s + s2
1
with a unit-step R (s) = s as an input. Let C(s) be the corresponding output. The time taken by the sytem output

c(t) to reach 94% of its steady-state value lim c (t), rounded off to two decimal places, is
t"3

(a) 4.50 (b) 3.89 (c) 2.81 (d) 5.25


33. Ans: (a)

1
Sol: G (s) 
1  2s  s 2
1
G (s) 
(s  1) 2
1 1 1  1
G (s) 
s (s  1) 2 s (s  1) 2 s  1
C (t)  `1  te t  e t j u (t)
⇒ 94% of ss value = 0.94

0.94 = (1 – te–t – e–t)

0.06 = e–t (t+1)

⇒ By option verification method

Let t = 4.5

e–t (t+1) = e–4.5 ( 1 + 4.5) = 0.06109 ≈ 0.06

Thus option 4.5 is the Answer


End of Solution

34. Consider the line integral


# ^xdy - ydxh
C

the integral being taken in a counterclockwise direction over the closed curve C that forms the boundary of the
region R shown in the figure below. The region R is the area enclosed by the union of a 2 × 3 rectangle and a
semi-circle of radius 1. The line integral evaluates to
y

C
3

2 R


0 x
1 2 3 4 5
(a) 6 + π/2 (b) 8 + π (c) 12 + π (d) 16 + 2π
35 GATE-19 EXAM PAPER

34. Ans: (c)


Sol: By Greens theorem

# xdy - ydx = ## c 22Nx - 22My mdxdy


C
W
N
W
M R

⇒ # ^xdy - ydxh = ## ]1 + 1gdxdy


C R

= # # 2dxdy

⇒ # ^xdy - ydxh = 2 # Area of the region


C

= 2(Area of a rectangle + Area of a semi circle)


r ]1 g2 D
= 2 :2 # 3 + = 2 b6 + 2 l
r
2
]12 + rg
= 2 2

= 12 + π

End of Solution
36 Electronics & Communication Engineering

35. The state transition diagram for the circuit shown is

D Q 1
Q 0

CLK
A

A= 0 A= 1 A= 0
(a)
Q=0 Q=1

A= 1

A= 0
A= 0 A= 1
(b)
Q=0 Q=1

A= 1

A= 1 A= 0 A= 0
(c)
Q=0 Q=1

A= 1

A= 0
A= 1 A= 0
(d)
Q=0 Q=1

A= 1

35. Ans: (d)


Sol: Q(t + 1) = D.........(1)
D = ^ A Q + AQh^Qh ........(2)
Substitute (2) in (1)
Q ]t + 1g = A.Q
So, if A = 0 ⇒ Q(t + 1) = 1
A = A ⇒ Q(t + 1) = Q
If A = 0, D = Q. Q = 0 = 1
If A = 1, D = QQ = Q
37 GATE-19 EXAM PAPER

36. In an ideal pn junction with an ideality factor of 1 at T = 300 K, the magnitude of the reverse-bias voltage required
to reach 75% of its reverse saturation current, rounded off to 2 decimal places, is _____mV.
[k = 1.38 × 10-23 JK-1, h = 6.625 × 10-34 J-s, q = 1.602 × 10-19C]
36. Ans: 35.87
Sol: IR = 0.75 IS

ID = −0.75 IS
IS ]e V /V - 1g =- 0.75 I S
D T

e V /V = 0.25
D T

VD = VT ln(0.25)
VR = − VT ln(0.25)

1.38 # 10 -23 # 300 #


= - - 1.386
1.6 # 10 -19

= 35.87 mV
End of Solution

37. The block diagram of a system is illustrated in the figure shown, where X(s) is the input and Y(s) is the output.
Y]s g
The transfer function H ] s g =
X]s g
is

s + Y(s)
X(s) + − 1
Σ Σ s
− 1 +
s

(a) H ] s g =
s+1

s2 + s + 1
(b) H ] s g = 3
s2 + 1
s + 2s2 + s + 1
s2 + 1
(c) H (s) = 2
2s + 1
(d) H ] s g = 3 2
s2 + 1
s +s +s+1
37. Ans: (b)
Sol:

s + Y(s)
X(s) + − 1
Σ Σ s
− 1 +
s

bs + 1 l = b s + 1 l
2

s s

38 Electronics & Communication Engineering

X(s) + − Y(s)
⇒ Σ s2 + 1
s
1
s

b s +2 1 l
2

s+s +1

Y (s) `s 2  1 j s2  1
 
X (s) s `s 2  s  1 j  s 2  1 s  2s 2  s  1
3

End of Solution

38. Consider a six-point decimation-in-time Fast Fourier Transform (FFT) algorithm, for which the signal-flow graph

corresponding to X[1] is shown in the figure. Let W6 = exp c - m . In the figure, what should be the values of
j2r
6
the coefficients a1,a2,a3 in terms of W6 so that X[1] is obtained correctly?
x[0] X[0]
a1
x[3] X[1]
-1
a2
x[1] X[2]
a3
x[4] X[3]
-1
x[2] X[4]

x[5] X[5]
-1

(a) a1 = 1, a2 = W6 .a3 = W 62
(b) a1 = - 1, a2 = W 62, a 3 = W6
(c) a1 = 1, a2 = W 62, a 3 = W6
(d) a1 =- 1, a2 = W6, a 3 = W 62

38. Ans: (a)


N1  5
Sol: X (k)  / x (n) W 6
kn

n0

 x (0)  x (1) W6k  x (2) W62k  x (3) W63k  x (4) W64k  x (5) W65k

X (1)  x (0)  x (1) W61  x (2) W62  x (3) W63  x (4) W64  x (5) W65
39 GATE-19 EXAM PAPER

Based on symmetry
N
k 2
WN  W Nk W63  W60  1
W64  W61
W65  W62
From the SFG
We can say a1 = 1
a2 = W6
a3 = W62
End of Solution
39. Two identical copper wires W1 and W2, placed in parallel as shown in the figure, carry currents I and 2I, respec-
tively, in opposite directions. If the two wires are separated by a distance of 4r, then the magnitude of the magnetic
v between the wires at a distance r from W1 is
field B
W1
r

W2

µ 0I
(a)
6πr

µ20I2
(b)
2πr2

6µ0I
(c)
5πr

5µ0I
(d)
6πr

39. Ans: (d)

Sol: W1 I
r

3r

W
2 2I
40 Electronics & Communication Engineering

v =B v1 + Bv2
B
v 1 = 0 I 7
B 2r
v 2 =  0 2I 7
2 ]3rg
B
v v
B = B1 + B v2

v =  0 I :1 + 2 D 7
B 2r 3
v = 5 o I 7
B 6r
End of Solution

40. Consider a differentiable function f(x) on the set of real numbers such that f(-1) = 0 and f l]xg # 2. Given these
conditions, which one of the following inequalities is necessarily true for all x ∈[-2, 2]?
(a) f ]xg # 2 x + 1 (b) f ]xg # 2 x
1 1

(c) f ]xg # 2 x + 1 (d) f ]xg # 2 x


40. Ans: (c)
Sol: From the option (3)
if the max of f(x) = 2|x+1| then

2 ]x + 1g if x + 1 $ 0 & x $ - 1
f ]x g = ) 3
- 2 ]x + 1g if x + 1 < 0 & x < - 1


f l]xg = )
2 if x $ - 1
3
- 2 if x < - 1

f l]xg # 2 and f(−1) = 0


So, option (c) is correct

End of Solution

41. Consider the homogeneous ordinary differential equation


d2 y dy
x2 2 - 3x dx + 3y = 0, x>0
dx
with y(x) as a general solution. Given that
y(1) = 1 and y(2) = 14
the value of y(1.5), rounded off to two decimal places, is _____
41. Ans: 5.25
d2 y dy
Sol: x2 2 - 3x dx + 3y = 0
dx
Given that y(1) = 1
y(2) = 14
Put x = e (or) z = ln x
z
41 GATE-19 EXAM PAPER

d
i = dz , xDy = θy
x2D2y = θ(θ − 1)y

∴ The given DE is equivalent to
θ(θ − 1) −3θy + 3y = 0
θ2 y − 4 θy + 3y = 0

d2 y dy
∴ - 4 dz + 3y = 0
dz2

Axillary equation m2 − 4m + 3 = 0
(m − 1) (m − 3) = 0
∴ m = 1, 3
The solution is y = C1ez + C2e3z
∴ y = C1x + C2x3
y(1) = 1 ⇒ 1 = C1 + C2

y(2) = 14 ⇒ 14 = 2C1 + 8C2

By solving C2 = 2, C1 = -1
∴ y = −x + 2x3
∴ y(1.5) = −1.5 + 2(1.5)3
= 5.25

End of Solution

42. In the circuit shown, V1 = 0 and V2 = Vdd. The other relevant parameters are mentioned in the figure. Ignoring the
effect of channel length modulation and the body effect, the value of Iout is _____mA (rounded off to 1 decimal
place).

Vdd

W/L=10 W/L=10 W/L=40

Iout
V1 W/L=5 W/L=5 V2
Vdd

1mA

W/L=2 W/L=3


42 Electronics & Communication Engineering

42. Ans: 6
Sol:
Vdd

W/L=10 W/L=10 W/L=40

Iout
V1 W/L=5 W/L=5 V2
Vdd

1mA
I1

W/L=2 W/L=3


bWl
L 2
I1 = # 1mA = 32 mA
b l
W
L 1
bWl
L 4 3
I out = # 2 mA
b l
W
L 3

40 # 3 =
= 10 2 6mA
End of Solution

43. The RC circuit shown below has a variable resistance R(t) given by the following expression:
R ] t g = R 0 b1 - T l for 0 # t # T
t

where R0 = 1Ω, and C = 1 F. We are also given that T = 3 R0C and the source voltage is Vs = 1V. If the current at
time t = 0 is 1 A, then the current I(t), in amperes, at time t = T/2 is _____ (rounded off to 2 decimal places).

I(t) R(t)

Vs + C

t=0

43. Ans: 0.1
Sol: In general for RC circuit with state variable voltage across capacitor VC (t)  VS 81  e t/B
dVC (t)
But current =I (t) i C=
(t) C dt

I (t)  C dt :VS `1  e t/jD
d
43 GATE-19 EXAM PAPER

 CVS <0  e t/# 1 F


T
VS t/R(t). C
I (t)  R (t) e

Given R (t)  R 0 <1  Tt F


R 0  1
4 R (t)  1 <1  3t F
T  3R 0 C  3

e t/d1  3 n
1 t
I (t) 
d1  t n
3
T 3
At =t 2= 2 sec

Id t  2 n 
3 1
e  (3/2) (1/2)

d1  1 n
2

I = 2 e–3A
= 0.099 ≈ 0.1 Amps

End of Solution

44. A single bit, equally likely to be 0 and 1, is to be sent across an additive white Gaussian noise (AWGN) channel
with power spectral density N0/2. Binary signaling, with 0 → p(t) and 1 → q(t), is used for the transmission, along
with an optimal receiver that minimizes the bit-error probability.
Let ϕ1(t), ϕ2(t) form an orthonormal signal set.
If we choose p(t) = ϕ1(t) and q(t) = −ϕ1(t), we would obtain a certain bit-error probability Pb.
If we keep p(t) = ϕ1(t), but take q (t) = E {2 (t), for what value of E would we obtain the same bit-error probabil-
ity Pb?
(a) 3
(b) 2
(c) 1
(d) 0
44. Ans: (a)
Sol: 0 is represented by p(t)
1 is represented by q(t)
φ1(t) and φ2(t) form are orthonormal signal set
p(t) = φ1(t)
q(t) = −φ1(t)
The signal space diagram is
44 Electronics & Communication Engineering

q(t) p(t)
φ1(t)
-1 1


dmin = 2
If p(t) = φ1(t) and q ] t g = E 2 ] t g
The signal space diagram is
φ2(t)

E
E+1

φ1(t)
1


bit error probability will be same if the dmin is same
1+E = 2
1+E = 4
E = 3

End of Solution


K
45. Consider a unity feedback system, as in the figure shown, with an integral compensator s and open-loop transfer
function
G]s g = 2
1

s + 3s + 2
where K > 0. The positive value of K for which there are exactly two poles of the unity feedback system on the
jω axis is equal to _____ (rounded off to two decimal places).

X(s) + K Y(s)
Σ G(s)
S


45 GATE-19 EXAM PAPER

45. Ans: 6
K
Sol: G C (s) 
s (s 2  3s  2)

CE → 1 + GC(s) = 0
CE → s3 + 3s2 + 2s + k = 0
s3 1 2

s2 3 k

s1 0
d6kn 0
3
s0 k

AE → 3s2 + 6 = 0
s=! j 2
k=6

End of Solution

46. In the circuit shown, Vs is a 10 V square wave of period T = 4 ms with R = 500 Ω and C = 10 µF. The capacitor is
initially uncharged at t = 0, and the diode is assumed to be ideal. The voltage across the capacitor (Vc) at 3 ms is
equal to ____ volts (rounded off to one decimal place).
R

+
VS
+ C VC
+10 −

0 −
T T
2
−10

46. Ans: 3.3
Sol: VS = 10V
Vc(t) = 10V ]1 - e -t/R g C

VC ]t = 2msg = 10V _1 - e - 500 # 10 # 10 i


-3


2 10 #
-6

= 3.3V
T/2↔T: Diode is OFF
Vc(t = 3ms) = 3.3V
46 Electronics & Communication Engineering
47 GATE-19 EXAM PAPER

47. In the circuits shown the threshold voltage of each nMOS transistor is 0.6 V. Ignoring the effect of channel length
modulation and body bias, the values of Vout1 and Vout2, respectively, in volts, are
3V

3V +
− Vout1
3V +

3V 3V 3V

Vout2

3V +−

(a) 1.8 and 2.4 (b) 2.4 and 2.4


(c) 1.8 and 1.2 (d) 2.4 and 1.2
47. Ans: (a)
Sol: Given, VTN = 0.6V, λ = 0 & VSB = 0
∴Vth = Vtho (or) Vt = Vto
Since, N-MOS, current always flow from drain to source,
⇒from fig (1)
∴ VG1 = 3V, VD1 = 3V, VTN = 0.6V
Since, to ON N-MOS [E - mode],
VGS ≥ VTh
Let VGS = VTh ⇒ VG1 - Vs1 = VTh
∴VS1 = VG1 - VTh = 2.4V
Since, VS1 = VG2 ⇒ VG2 = 2.4V,
& VD2 = 3V
∴ Let VGS2 = VTh ⇒ VG2 - VS2 = VTh
∴ VS2 = Vout1 = 2.4 - 0.6
= 1.8 V.
∴ Vout1 = 1.8V.
Similarly from fig (2)
VG1 = VG2 = VG3 = 3V & VD1 = 3V
∴ VS1 = VG1 - VTh = 3 - 0.6 = 2.4V
∴ VS1 = VO2 = 2.4V
∴VS2 = VG2 - VTh = 3 - 0.6 = 2.4 V
48 Electronics & Communication Engineering

& VS3 = VG3 - VTh = 2.4V


Vout2 = 2.4V
∴Vo1 =1.8V, Vo2 = 2.4 V
End of Solution


48. A rectangular waveguide of width w and height h has cut-off frequencies for TE10 and TE11 modes in the ratio 1:2.
The aspect ratio w/h, rounded off to two decimal places, is ______
48. Ans: 1.73
c1
Sol: fc TE = 2 a
10

fc TE = 2c 1 + 1
11
a2 b2
Given
fcTE 1
fcTE = 2
10

11

fcTE = 2fcTE
11 10

c 1 1 c1
2 a2 + b2 = 2 2 a .
1 1 4
2 + 2 = 2
a b a
1 3
2 = 2
b a
bal =3
2
b
a
b = 3
a
b = 1.732

End of Solution

49. Let a random process Y(t) be described as Y(t) = h(t) * X(t) + Z(t), where X(t) is a white noise process with power
spectral density Sx(ƒ) = 5 W/Hz. The filter h(t) has a magnitude response given by |H(ƒ)| = 0.5 for
-5 ≤ ƒ≤ 5, and zero elsewhere. Z(t) is a stationary random process, uncorrelated with X(t), with power spectral
density as shown in the figure. The power in Y(t), in watts, is equal to____ W (rounded off to two decimal places).

1 SZ(f) (W/Hz)

−5 5 f(Hz)

49 GATE-19 EXAM PAPER

49. Ans: 17.5

Sol:
x(t)∗h(t)
h(t)
x(t)

∑ y(t)

z(t)

Let us assume x(t)*h(t) = w(t)


y(t) = w(t) + z(t)

RYY(τ) = RWW(τ) + RZZ(τ) +RWZ(τ) + RZW(τ)

x(t) & z(t) are uncorrelated

Rwz(τ) = Rzw(τ) = 0

RYY(τ) = Rww(τ) + Rzz(τ)


SYY(ƒ) =Sww(ƒ) + Szz(ƒ)

# S YY ]f g df = # S WW ]f g df + # S ZZ ]f g df
3 3 3
The power of y(t) is PY =
-3 -3 -3

SWW(f) = |H(f)|2 SX(f)

|H(f)|2 1
4

–5 0 5 f

Sz(f)
SWW(t) =|H(f)|2Sx(f)
5 1
4

–5 0 5 f –5 5
0 f

PY = 4 ]10g + 2 ]10g]1 g = 12.5 + 5 = 17.5 W


5 1

End of Solution
50 Electronics & Communication Engineering

50. In the circuit shown, if v(t) = 2 sin(1000 t) volts, R = 1 kΩ and C = 1 µF, then the steady-state current i(t), in
milliAmperes (mA), is
i(t)

C
+
v(t) R
R

C C

R

(a) 2 sin(1000 t) + 2 cos(1000 t) (b) sin(1000 t) + 3 cos(1000 t)
(c) sin(1000 t) + cos(1000 t) (d) 3 sin(1000 t) + cos(1000 t)
50. Ans: (d)
Sol:

R R
C/3
C/3
C/3

R
C 1
R = 1000 Ω 3 3 F
at  1000 rad/sec
j j
X C  C  1
1000 # 3 # 10 6

X C  j3000

1000 Ω –j3000 Ω

= (900 – j300) Ω
51 GATE-19 EXAM PAPER


(900 - j300)Ω (900 - j300)Ω

ZT

(900 - j300)Ω

ZT = (900 – j 300) || (1800 – j600)
(1440000  j1080000)
ZT 
(2700  j900)
Z T  (600  j200) 

Finally

i(t)

2sin1000t V +
~
_
ZT = (600 - j200)Ω


2 sin 1000t  2 sin 1000t
So, i (t) 
(600  j200) 632.455+  18.434

So,

i(t) = 3.162 sin(1000t + 18.434o) mA

S(A + B) = sinA cosB + cosA sinB

So,

i(t) = 3.162 (sin1000t cos 18.434 + cos1000t sin18.434) mA

i(t) = (3 sin1000t + cos1000t) mA

End of Solution
52 Electronics & Communication Engineering

51. A CMOS inverter, designed to have a mid-point voltage V1 equal to half of Vdd, as shown in the figure, has the
following parameters:
Vdd = 3 V
n n C ox = 100nA/V2; Vtn = 0.7V for nMOS
n p C ox = 40nA/V2; Vtp = 0.9V for pMOS
The ratio of b W l bWl
L n to L p is equal to ____ (rounded off to 3 decimal places).

Vout

Vdd

Vdd
2

V1 = Vdd Vdd Vin


2

51. Ans: 0.225
Sol: n n C ox b L l ]1.5 - VTNg2 = n P C ox b L l ^1.5 - VTP h2
W W
N P

100 # b L l # 0.82 = 40 # b L l # 0.62



W W
N P

]W/LgN 9 # 4

]W/LgP = 16 10 = 0.225

End of Solution

52. The quantum efficiency (η) and responsivity (R) at a wavelenth λ (in µm) in a p-i-n photodetector are related by

m 1.24 # m
(a) R = h # 1.24 (b) R = h
h#m 1.24
(c) R = 1.24 (d) R = #
h m

52. Ans: (c)


Sol: hq hqm
R = hv = hc

hm
R = 1.24

End of Solution
53 GATE-19 EXAM PAPER

53. A voice signal m(t) is in the frequency range 5 kHz to 15 kHz. The signal is amplitude-modulated to generate an
AM signal ƒ(t) = A(1+m(t)) cos 2πƒct, where ƒc = 600 kHz. The AM signal ƒ(t) is to be digitized and archived.
This is done by first sampling ƒ(t) at 1.2 times the Nyquist frequency, and then quantizing each sample using
a 256-level quantizer. Finally, each quantized sample is binary coded using K bits, where K is the minimum
number of bits required for the encoding. The rate, in Megabits per second (rounded off to 2 decimal places), of

of the resulting stream of coded bits is ____ Mbps.


53. Ans:
As there is an ambiquity in the question, one of the question out of the following two cases may be possible

Sol: Spectrum of the voice signal m(t) is


M(f)

–15 k –5 k 0 5 k 15 k

The spectrum of modulated signal is

–615 –600 –585 585 595 600 605 615 f(kHz)


School of thought 1
The modulated signal is a bandpass signal
2f
so the minimum sampling rate is fs = NH

N = int b 30 l = 20.5 = 20
615
fH = 615kHz
2 # 615
fs = 20 kHz = 61.5kHz
sampling rate = 61.5×1.2k = 73.8kHz

L = 256 = 28

Minimum number of bits required to encode each sample is ‘8’.

so the bit rate = 73.8×103×8 = 0.5904 Mbps

school of thought 2
sampling rate = 2×615×103×1.2=1.476 MHz
so the bit rate = 1.476 × 8 = 11.808 Mbps
54 Electronics & Communication Engineering

54. A random variable X takes values -1 and +1 with probabilities 0.2 and 0.8, respectively. It is transmitted across a
channel output is Y = X + N. The noise N is independent of X, and is uniformly distributed over the interval [-2,2].
the receiver makes a decision

t = ) 1, if Y # i
-
X
+ 1, if Y > i
where the threshold θ∈[-1,1] is chosen so as to minimize the probability of error Pr 6X
t ! X@ . The minimum

probability of error, rounded off to 1 decimal place, is _____


54. Ans: 0.1
Sol: P(−1) = 0.2, P(1) = 0.8
the pdf of noise is

1/4

–2 0 2

the received signal Y = X + N


The pdf if ‘1’ is transmitted

1/4

–1 0 3

The pdf if ‘−1’ is transmitted

1/4

–3 0 1
The threshold voltage should lies between −1 and +1
Pe = 0.8 # 4 dz + 0.2 #V 4 dz
V 1 TH 3 1

-3 TH

= 0.8 # 14 dz + 0.2 # 14 dz
V TH 1

-1 V TH

Pe = 0.2 6VTH + 1@ + 0.05 61 - VTH@

VTH = −1 → Pe = 0.1
VTH = 0 → Pe = 0.25
VTH = 1 → Pe = 0.4
So the minimum probability of error is 0.1
GATE ESE PSU’s 2019-20
ECE ENGINEERING
GATE ECE 2003-2019 SOLVED

GATE ECE 2003-2019 SOLVED Detail Solution

CONTENT COVERED:
1.Theory Notes
2.Explanation
3.Derivation
4.Example
5.Shortcut & Formula Summary
6.Previous year Paper Q. Sol.
Noted-: Single Source Follow, Revise
Multiple Time Best key of Success
1
Page

http://www.orbitmentor.com [email protected]
55 GATE-19 EXAM PAPER

55. Consider a long-channel MOSFET with channel length 1 µm and width 10 µm. The device parameters are ac-
ceptor concentration NA = 5 × 1016 cm-3, electron mobility µn = 800 cm2/V-s, oxide capacitance/area Cox = 3.45 ×
10-7F/cm2, threshold voltage VT = 0.7 V. The drain saturation current (IDsat) for a gate voltage of 5 V is ____ mA
(rounded off to two decimal places). [ε0 = 8.854 × 10-14F/cm, εsi = 11.9]
55. Ans: 25.5
Sol: I D,sat = 2 n n C ox # L ]VGS - VTHg2
1 W

= 2 # 800 # 3.45 # 10 -7 # 1 # ]5 - 0.7g2


1 10

= 25.5 mA

End of Solution

You might also like